ΠΠ΄Ρ ΠΈΠ½Π΄ΡΠΊΡΠΈΠΈ Π² Π΄Π²ΠΈΠΆ ΠΏΡΠΎΠ²ΠΎΠ΄Π½ΠΈΠΊΠ΅. ΠΠ΄Ρ ΠΈΠ½Π΄ΡΠΊΡΠΈΠΈ Π² Π΄Π²ΠΈΠΆΡΡΠΈΡ ΡΡ ΠΏΡΠΎΠ²ΠΎΠ΄Π½ΠΈΠΊΠ°Ρ
Π Π°ΡΡΠΌΠΎΡΡΠΈΠΌ ΡΠ΅ΠΏΠ΅ΡΡ Π²ΡΠΎΡΠΎΠΉ ΡΠ»ΡΡΠ°ΠΉ Π²ΠΎΠ·Π½ΠΈΠΊΠ½ΠΎΠ²Π΅Π½ΠΈΡ ΠΈΠ½Π΄ΡΠΊΡΠΈΠΎΠ½Π½ΠΎΠ³ΠΎ ΡΠΎΠΊΠ°.
ΠΡΠΈ Π΄Π²ΠΈΠΆΠ΅Π½ΠΈΠΈ ΠΏΡΠΎΠ²ΠΎΠ΄Π½ΠΈΠΊΠ° Π΅Π³ΠΎ ΡΠ²ΠΎΠ±ΠΎΠ΄Π½ΡΠ΅ Π·Π°ΡΡΠ΄Ρ Π΄Π²ΠΈΠΆΡΡΡΡ Π²ΠΌΠ΅ΡΡΠ΅ Ρ Π½ΠΈΠΌ. ΠΠΎΡΡΠΎΠΌΡ Π½Π° Π·Π°ΡΡΠ΄Ρ ΡΠΎ ΡΡΠΎΡΠΎΠ½Ρ ΠΌΠ°Π³Π½ΠΈΡΠ½ΠΎΠ³ΠΎ ΠΏΠΎΠ»Ρ Π΄Π΅ΠΉΡΡΠ²ΡΠ΅Ρ ΡΠΈΠ»Π° ΠΠΎΡΠ΅Π½ΡΠ°. ΠΠ½Π°-ΡΠΎ ΠΈ Π²ΡΠ·ΡΠ²Π°Π΅Ρ ΠΏΠ΅ΡΠ΅ΠΌΠ΅ΡΠ΅Π½ΠΈΠ΅ Π·Π°ΡΡΠ΄ΠΎΠ² Π²Π½ΡΡΡΠΈ ΠΏΡΠΎΠ²ΠΎΠ΄Π½ΠΈΠΊΠ°. ΠΠΠ‘ ΠΈΠ½Π΄ΡΠΊΡΠΈΠΈ, ΡΠ»Π΅Π΄ΠΎΠ²Π°ΡΠ΅Π»ΡΠ½ΠΎ, ΠΈΠΌΠ΅Π΅Ρ ΠΌΠ°Π³Π½ΠΈΡΠ½ΠΎΠ΅ ΠΏΡΠΎΠΈΡΡ ΠΎΠΆΠ΄Π΅Π½ΠΈΠ΅.
ΠΠ° ΠΌΠ½ΠΎΠ³ΠΈΡ ΡΠ»Π΅ΠΊΡΡΠΎΡΡΠ°Π½ΡΠΈΡΡ Π·Π΅ΠΌΠ½ΠΎΠ³ΠΎ ΡΠ°ΡΠ° ΠΈΠΌΠ΅Π½Π½ΠΎ ΡΠΈΠ»Π° ΠΠΎΡΠ΅Π½ΡΠ° Π²ΡΠ·ΡΠ²Π°Π΅Ρ ΠΏΠ΅ΡΠ΅ΠΌΠ΅ΡΠ΅Π½ΠΈΠ΅ ΡΠ»Π΅ΠΊΡΡΠΎΠ½ΠΎΠ² Π² Π΄Π²ΠΈΠΆΡΡΠΈΡ ΡΡ ΠΏΡΠΎΠ²ΠΎΠ΄Π½ΠΈΠΊΠ°Ρ .
ΠΡΡΠΈΡΠ»ΠΈΠΌ ΠΠΠ‘ ΠΈΠ½Π΄ΡΠΊΡΠΈΠΈ, Π²ΠΎΠ·Π½ΠΈΠΊΠ°ΡΡΡΡ Π² ΠΏΡΠΎΠ²ΠΎΠ΄Π½ΠΈΠΊΠ΅, Π΄Π²ΠΈΠΆΡΡΠ΅ΠΌΡΡ Π² ΠΎΠ΄Π½ΠΎΡΠΎΠ΄Π½ΠΎΠΌ ΠΌΠ°Π³Π½ΠΈΡΠ½ΠΎΠΌ ΠΏΠΎΠ»Π΅ (ΡΠΈΡ. 2.10). ΠΡΡΡΡ ΡΡΠΎΡΠΎΠ½Π° ΠΊΠΎΠ½ΡΡΡΠ° MN Π΄Π»ΠΈΠ½ΠΎΠΉ I ΡΠΊΠΎΠ»ΡΠ·ΠΈΡ Ρ ΠΏΠΎΡΡΠΎΡΠ½Π½ΠΎΠΉ ΡΠΊΠΎΡΠΎΡΡΡΡ Π²Π΄ΠΎΠ»Ρ ΡΡΠΎΡΠΎΠ½ NC ΠΈ MD, ΠΎΡΡΠ°Π²Π°ΡΡΡ Π²ΡΠ΅ Π²ΡΠ΅ΠΌΡ ΠΏΠ°ΡΠ°Π»Π»Π΅Π»ΡΠ½ΠΎΠΉ ΡΡΠΎΡΠΎΠ½Π΅ CD. ΠΠ΅ΠΊΡΠΎΡ ΠΌΠ°Π³Π½ΠΈΡΠ½ΠΎΠΉ ΠΈΠ½Π΄ΡΠΊΡΠΈΠΈ ΠΎΠ΄Π½ΠΎΡΠΎΠ΄Π½ΠΎΠ³ΠΎ ΠΏΠΎΠ»Ρ ΠΏΠ΅ΡΠΏΠ΅Π½Π΄ΠΈΠΊΡΠ»ΡΡΠ΅Π½ ΠΏΡΠΎΠ²ΠΎΠ΄Π½ΠΈΠΊΡ ΠΈ ΡΠΎΡΡΠ°Π²Π»ΡΠ΅Ρ ΡΠ³ΠΎΠ» Ξ± Ρ Π½Π°ΠΏΡΠ°Π²Π»Π΅Π½ΠΈΠ΅ΠΌ Π΅Π³ΠΎ ΡΠΊΠΎΡΠΎΡΡΠΈ.
Π‘ΠΈΠ»Π°, Ρ ΠΊΠΎΡΠΎΡΠΎΠΉ ΠΌΠ°Π³Π½ΠΈΡΠ½ΠΎΠ΅ ΠΏΠΎΠ»Π΅ Π΄Π΅ΠΉΡΡΠ²ΡΠ΅Ρ Π½Π° Π΄Π²ΠΈΠΆΡΡΡΡΡΡ Π·Π°ΡΡΠΆΠ΅Π½Π½ΡΡ ΡΠ°ΡΡΠΈΡΡ, ΡΠ°Π²Π½Π° ΠΏΠΎ ΠΌΠΎΠ΄ΡΠ»Ρ
F Π» = | q |Ο B sin Ξ±. (2.5)
ΠΠ°ΠΏΡΠ°Π²Π»Π΅Π½Π° ΡΡΠ° ΡΠΈΠ»Π° Π²Π΄ΠΎΠ»Ρ ΠΏΡΠΎΠ²ΠΎΠ΄Π½ΠΈΠΊΠ° MN. Π Π°Π±ΠΎΡΠ° ΡΠΈΠ»Ρ ΠΠΎΡΠ΅Π½ΡΠ° 1 Π½Π° ΠΏΡΡΠΈ I ΠΏΠΎΠ»ΠΎΠΆΠΈΡΠ΅Π»ΡΠ½Π° ΠΈ ΡΠΎΡΡΠ°Π²Π»ΡΠ΅Ρ:
Π = F Π» l = | q | Ο Bl sin Ξ±.
1 ΠΡΠΎ Π½Π΅ΠΏΠΎΠ»Π½Π°Ρ ΡΠ°Π±ΠΎΡΠ° ΡΠΈΠ»Ρ ΠΠΎΡΠ΅Π½ΡΠ°. ΠΡΠΎΠΌΠ΅ ΡΠΈΠ»Ρ ΠΠΎΡΠ΅Π½ΡΠ° (ΡΠΌ. ΡΠΎΡΠΌΡΠ»Ρ (2.5)), ΠΈΠΌΠ΅Π΅ΡΡΡ ΡΠΎΡΡΠ°Π²Π»ΡΡΡΠ°Ρ ΡΠΈΠ»Ρ ΠΠΎΡΠ΅Π½ΡΠ°, Π½Π°ΠΏΡΠ°Π²Π»Π΅Π½Π½Π°Ρ ΠΏΡΠΎΡΠΈΠ² ΡΠΊΠΎΡΠΎΡΡΠΈ ΠΈ ΠΏΡΠΎΠ²ΠΎΠ΄Π½ΠΈΠΊΠ°. ΠΡΠ° ΡΠΎΡΡΠ°Π²Π»ΡΡΡΠ°Ρ ΡΠΎΡΠΌΠΎΠ·ΠΈΡ Π΄Π²ΠΈΠΆΠ΅Π½ΠΈΠ΅ ΠΏΡΠΎΠ²ΠΎΠ΄Π½ΠΈΠΊΠ° ΠΈ ΡΠΎΠ²Π΅ΡΡΠ°Π΅Ρ ΠΎΡΡΠΈΡΠ°ΡΠ΅Π»ΡΠ½ΡΡ ΡΠ°Π±ΠΎΡΡ. Π ΡΠ΅Π·ΡΠ»ΡΡΠ°ΡΠ΅ ΠΏΠΎΠ»Π½Π°Ρ ΡΠ°Π±ΠΎΡΠ° ΡΠΈΠ»Ρ ΠΠΎΡΠ΅Π½ΡΠ° ΠΎΠΊΠ°Π·ΡΠ²Π°Π΅ΡΡΡ ΡΠ°Π²Π½ΠΎΠΉ Π½ΡΠ»Ρ.
ΠΠ»Π΅ΠΊΡΡΠΎΠ΄Π²ΠΈΠΆΡΡΠ°Ρ ΡΠΈΠ»Π° ΠΈΠ½Π΄ΡΠΊΡΠΈΠΈ Π² ΠΏΡΠΎΠ²ΠΎΠ΄Π½ΠΈΠΊΠ΅ MN ΡΠ°Π²Π½Π°, ΠΏΠΎ ΠΎΠΏΡΠ΅Π΄Π΅Π»Π΅Π½ΠΈΡ, ΠΎΡΠ½ΠΎΡΠ΅Π½ΠΈΡ ΡΠ°Π±ΠΎΡΡ ΠΏΠΎ ΠΏΠ΅ΡΠ΅ΠΌΠ΅ΡΠ΅Π½ΠΈΡ Π·Π°ΡΡΠ΄Π° q ΠΊ ΡΡΠΎΠΌΡ Π·Π°ΡΡΠ΄Ρ:
ΠΡΠ° ΡΠΎΡΠΌΡΠ»Π° ΡΠΏΡΠ°Π²Π΅Π΄Π»ΠΈΠ²Π° Π΄Π»Ρ Π»ΡΠ±ΠΎΠ³ΠΎ ΠΏΡΠΎΠ²ΠΎΠ΄Π½ΠΈΠΊΠ° Π΄Π»ΠΈΠ½ΠΎΠΉ l, Π΄Π²ΠΈΠΆΡΡΠ΅Π³ΠΎΡΡ ΡΠΎ ΡΠΊΠΎΡΠΎΡΡΡΡ Π² ΠΎΠ΄Π½ΠΎΡΠΎΠ΄Π½ΠΎΠΌ ΠΌΠ°Π³Π½ΠΈΡΠ½ΠΎΠΌ ΠΏΠΎΠ»Π΅.
Π Π΄ΡΡΠ³ΠΈΡ ΠΏΡΠΎΠ²ΠΎΠ΄Π½ΠΈΠΊΠ°Ρ ΠΊΠΎΠ½ΡΡΡΠ° ΠΠΠ‘ ΡΠ°Π²Π½Π° Π½ΡΠ»Ρ, ΡΠ°ΠΊ ΠΊΠ°ΠΊ ΡΡΠΈ ΠΏΡΠΎΠ²ΠΎΠ΄Π½ΠΈΠΊΠΈ Π½Π΅ΠΏΠΎΠ΄Π²ΠΈΠΆΠ½Ρ. Π‘Π»Π΅Π΄ΠΎΠ²Π°ΡΠ΅Π»ΡΠ½ΠΎ, ΠΠΠ‘ Π²ΠΎ Π²ΡΠ΅ΠΌ ΠΊΠΎΠ½ΡΡΡΠ΅ MNCD ΡΠ°Π²Π½Π° ΠΈ ΠΎΡΡΠ°Π΅ΡΡΡ Π½Π΅ΠΈΠ·ΠΌΠ΅Π½Π½ΠΎΠΉ, Π΅ΡΠ»ΠΈ ΡΠΊΠΎΡΠΎΡΡΡ Π΄Π²ΠΈΠΆΠ΅Π½ΠΈΡ ΠΏΠΎΡΡΠΎΡΠ½Π½Π°. ΠΠ»Π΅ΠΊΡΡΠΈΡΠ΅ΡΠΊΠΈΠΉ ΡΠΎΠΊ ΠΏΡΠΈ ΡΡΠΎΠΌ Π±ΡΠ΄Π΅Ρ ΡΠ²Π΅Π»ΠΈΡΠΈΠ²Π°ΡΡΡΡ, ΡΠ°ΠΊ ΠΊΠ°ΠΊ ΠΏΡΠΈ ΡΠΌΠ΅ΡΠ΅Π½ΠΈΠΈ ΠΏΡΠΎΠ²ΠΎΠ΄Π½ΠΈΠΊΠ° MN Π²ΠΏΡΠ°Π²ΠΎ ΡΠΌΠ΅Π½ΡΡΠ°Π΅ΡΡΡ ΠΎΠ±ΡΠ΅Π΅ ΡΠΎΠΏΡΠΎΡΠΈΠ²Π»Π΅Π½ΠΈΠ΅ ΠΊΠΎΠ½ΡΡΡΠ°.
ΠΠΠ‘ ΠΈΠ½Π΄ΡΠΊΡΠΈΠΈ ΠΌΠΎΠΆΠ½ΠΎ Π²ΡΡΠΈΡΠ»ΠΈΡΡ ΡΠ°ΠΊΠΆΠ΅ ΠΈ Ρ ΠΏΠΎΠΌΠΎΡΡΡ Π·Π°ΠΊΠΎΠ½Π° ΡΠ»Π΅ΠΊΡΡΠΎΠΌΠ°Π³Π½ΠΈΡΠ½ΠΎΠΉ ΠΈΠ½Π΄ΡΠΊΡΠΈΠΈ (ΡΠΌ. ΡΠΎΡΠΌΡΠ»Ρ (2.4)). ΠΠ΅ΠΉΡΡΠ²ΠΈΡΠ΅Π»ΡΠ½ΠΎ, ΠΌΠ°Π³Π½ΠΈΡΠ½ΡΠΉ ΠΏΠΎΡΠΎΠΊ ΡΠ΅ΡΠ΅Π· ΠΊΠΎΠ½ΡΡΡ MNCD ΡΠ°Π²Π΅Π½:
Π€ = BS cos (90Β° — Ξ±) = BS sin Ξ±,
Π³Π΄Π΅ ΡΠ³ΠΎΠ» (90Β° — Ξ±) Π΅ΡΡΡ ΡΠ³ΠΎΠ» ΠΌΠ΅ΠΆΠ΄Ρ Π²Π΅ΠΊΡΠΎΡΠΎΠΌ ΠΈ Π½ΠΎΡΠΌΠ°Π»ΡΡ ΠΊ ΠΏΠΎΠ²Π΅ΡΡ Π½ΠΎΡΡΠΈ ΠΊΠΎΠ½ΡΡΡΠ° (ΡΠΈΡ. 2.11, Π²ΠΈΠ΄ ΡΠ±ΠΎΠΊΡ), a S — ΠΏΠ»ΠΎΡΠ°Π΄Ρ, ΠΎΠ³ΡΠ°Π½ΠΈΡΠ΅Π½Π½Π°Ρ ΠΊΠΎΠ½ΡΡΡΠΎΠΌ MNCD. ΠΡΠ»ΠΈ ΡΡΠΈΡΠ°ΡΡ, ΡΡΠΎ Π² Π½Π°ΡΠ°Π»ΡΠ½ΡΠΉ ΠΌΠΎΠΌΠ΅Π½Ρ Π²ΡΠ΅ΠΌΠ΅Π½ΠΈ (t = 0) ΠΏΡΠΎΠ²ΠΎΠ΄Π½ΠΈΠΊ MN Π½Π°Ρ ΠΎΠ΄ΠΈΡΡΡ Π½Π° ΡΠ°ΡΡΡΠΎΡΠ½ΠΈΠΈ NC ΠΎΡ ΠΏΡΠΎΠ²ΠΎΠ΄Π½ΠΈΠΊΠ° CD (ΡΠΌ. ΡΠΈΡ. 2.10), ΡΠΎ ΠΏΡΠΈ ΠΏΠ΅ΡΠ΅ΠΌΠ΅ΡΠ΅Π½ΠΈΠΈ ΠΏΡΠΎΠ²ΠΎΠ΄Π½ΠΈΠΊΠ° ΠΏΠ»ΠΎΡΠ°Π΄Ρ S ΠΈΠ·ΠΌΠ΅Π½ΡΠ΅ΡΡΡ ΡΠΎ Π²ΡΠ΅ΠΌΠ΅Π½Π΅ΠΌ ΡΠ»Π΅Π΄ΡΡΡΠΈΠΌ ΠΎΠ±ΡΠ°Π·ΠΎΠΌ:
S = l (NC — Ο t).
ΠΠ° Π²ΡΠ΅ΠΌΡ Ξt ΠΏΠ»ΠΎΡΠ°Π΄Ρ ΠΊΠΎΠ½ΡΡΡΠ° ΠΌΠ΅Π½ΡΠ΅ΡΡΡ Π½Π° ΞS = -lΟ Ξt. ΠΠ½Π°ΠΊ Β«-Β» ΡΠΊΠ°Π·ΡΠ²Π°Π΅Ρ Π½Π° ΡΠΎ, ΡΡΠΎ ΠΎΠ½Π° ΡΠΌΠ΅Π½ΡΡΠ°Π΅ΡΡΡ. ΠΠ·ΠΌΠ΅Π½Π΅Π½ΠΈΠ΅ ΠΌΠ°Π³Π½ΠΈΡΠ½ΠΎΠ³ΠΎ ΠΏΠΎΡΠΎΠΊΠ° Π·Π° ΡΡΠΎ Π²ΡΠ΅ΠΌΡ ΡΠ°Π²Π½ΠΎ:
ΠΡΠ»ΠΈ Π²Π΅ΡΡ ΠΊΠΎΠ½ΡΡΡ MNCD Π΄Π²ΠΈΠΆΠ΅ΡΡΡ Π² ΠΎΠ΄Π½ΠΎΡΠΎΠ΄Π½ΠΎΠΌ ΠΌΠ°Π³Π½ΠΈΡΠ½ΠΎΠΌ ΠΏΠΎΠ»Π΅, ΡΠΎΡ ΡΠ°Π½ΡΡ ΡΠ²ΠΎΡ ΠΎΡΠΈΠ΅Π½ΡΠ°ΡΠΈΡ ΠΏΠΎ ΠΎΡΠ½ΠΎΡΠ΅Π½ΠΈΡ ΠΊ Π²Π΅ΠΊΡΠΎΡΡ , ΡΠΎ ΠΠΠ‘ ΠΈΠ½Π΄ΡΠΊΡΠΈΠΈ Π² ΠΊΠΎΠ½ΡΡΡΠ΅ Π±ΡΠ΄Π΅Ρ ΡΠ°Π²Π½Π° Π½ΡΠ»Ρ, ΡΠ°ΠΊ ΠΊΠ°ΠΊ ΠΏΠΎΡΠΎΠΊ Π€ ΡΠ΅ΡΠ΅Π· ΠΏΠΎΠ²Π΅ΡΡ Π½ΠΎΡΡΡ, ΠΎΠ³ΡΠ°Π½ΠΈΡΠ΅Π½Π½ΡΡ ΠΊΠΎΠ½ΡΡΡΠΎΠΌ, Π½Π΅ ΠΌΠ΅Π½ΡΠ΅ΡΡΡ. ΠΠ±ΡΡΡΠ½ΠΈΡΡ ΡΡΠΎ ΠΌΠΎΠΆΠ½ΠΎ ΡΠ°ΠΊ.
ΠΠΠ‘ ΠΈΠ½Π΄ΡΠΊΡΠΈΠΈ Π²ΠΎΠ·Π½ΠΈΠΊΠ°Π΅Ρ ΡΠ°ΠΊΠΆΠ΅ ΠΏΡΠΈ ΠΏΠΎΠ²ΠΎΡΠΎΡΠ΅ ΡΠ°ΠΌΠΊΠΈ Π² ΠΌΠ°Π³Π½ΠΈΡΠ½ΠΎΠΌ ΠΏΠΎΠ»Π΅, Ρ. Π΅. ΠΏΡΠΈ ΠΈΠ·ΠΌΠ΅Π½Π΅Π½ΠΈΠΈ ΡΠΎ Π²ΡΠ΅ΠΌΠ΅Π½Π΅ΠΌ ΡΠ³Π»Π° ΠΎΡ (ΡΠΌ. Β§ 31).
ΠΠΠ‘ ΠΈΠ½Π΄ΡΠΊΡΠΈΠΈ Π² ΠΏΡΠΎΠ²ΠΎΠ΄Π½ΠΈΠΊΠ°Ρ , Π΄Π²ΠΈΠΆΡΡΠΈΡ ΡΡ Π² ΠΏΠΎΡΡΠΎΡΠ½Π½ΠΎΠΌ ΠΌΠ°Π³Π½ΠΈΡΠ½ΠΎΠΌ ΠΏΠΎΠ»Π΅, Π²ΠΎΠ·Π½ΠΈΠΊΠ°Π΅Ρ Π·Π° ΡΡΠ΅Ρ Π΄Π΅ΠΉΡΡΠ²ΠΈΡ Π½Π° Π·Π°ΡΡΠ΄Ρ ΠΏΡΠΎΠ²ΠΎΠ΄Π½ΠΈΠΊΠ° ΡΠΈΠ»Ρ ΠΠΎΡΠ΅Π½ΡΠ°.
ΠΠΎΠΏΡΠΎΡΡ ΠΊ ΠΏΠ°ΡΠ°Π³ΡΠ°ΡΡ
1. Π§Π΅ΠΌΡ ΡΠ°Π²Π½Π° ΡΠΈΠ»Π° ΠΠΎΡΠ΅Π½ΡΠ° ΠΈ ΠΊΠ°ΠΊ ΠΎΠ½Π° Π½Π°ΠΏΡΠ°Π²Π»Π΅Π½Π°?
2. ΠΡ ΡΠ΅Π³ΠΎ Π·Π°Π²ΠΈΡΠΈΡ ΠΠΠ‘ ΠΈΠ½Π΄ΡΠΊΡΠΈΠΈ, Π²ΠΎΠ·Π½ΠΈΠΊΠ°ΡΡΠ°Ρ Π² ΠΏΡΠΎΠ²ΠΎΠ΄Π½ΠΈΠΊΠ΅, ΠΊΠΎΡΠΎΡΡΠΉ Π΄Π²ΠΈΠΆΠ΅ΡΡΡ Π² ΠΏΠ΅ΡΠ΅ΠΌΠ΅Π½Π½ΠΎΠΌ Π²ΠΎ Π²ΡΠ΅ΠΌΠ΅Π½ΠΈ ΠΌΠ°Π³Π½ΠΈΡΠ½ΠΎΠΌ ΠΏΠΎΠ»Π΅?
««ЯвлСниС ΡΠ»Π΅ΠΊΡΡΠΎΠΌΠ°Π³Π½ΠΈΡΠ½ΠΎΠΉ ΠΈΠ½Π΄ΡΠΊΡΠΈΠΈΒ» ΡΠΈΠ·ΠΈΠΊΠ°Β» — ΠΠ»Π°ΡΡΠΈΠ½Π° ΠΏΡΠ°ΠΊΡΠΈΡΠ΅ΡΠΊΠΈ ΠΎΡΡΠ°Π½ΠΎΠ²ΠΈΡΡΡ. ΠΠΎΡΠΎΠΊΠΎΡΡΠ΅ΠΏΠ»Π΅Π½ΠΈΠ΅. ΠΠ½Π΅ΡΠ³ΠΈΡ ΠΌΠ°Π³Π½ΠΈΡΠ½ΠΎΠ³ΠΎ ΠΏΠΎΠ»Ρ. ΠΠ½Π΅ΡΠ³ΠΈΡ ΠΎΠ΄Π½ΠΎΡΠΎΠ΄Π½ΠΎΠ³ΠΎ ΠΌΠ°Π³Π½ΠΈΡΠ½ΠΎΠ³ΠΎ ΠΏΠΎΠ»Ρ. Π―Π²Π»Π΅Π½ΠΈΠ΅ ΡΠ°ΠΌΠΎΠΈΠ½Π΄ΡΠΊΡΠΈΠΈ ΠΈΠ³ΡΠ°Π΅Ρ Π²Π°ΠΆΠ½ΡΡ ΡΠΎΠ»Ρ Π² ΡΠ»Π΅ΠΊΡΡΠΎΡΠ΅Ρ Π½ΠΈΠΊΠ΅. ΠΠΠ‘ ΡΠ°ΠΌΠΎΠΈΠ½Π΄ΡΠΊΡΠΈΠΈ Π±ΡΠ΄Π΅Ρ ΠΏΠΎΠ΄Π΄Π΅ΡΠΆΠΈΠ²Π°ΡΡ ΡΠΎΠΊ Π² ΡΠ΅ΠΏΠΈ. ΠΡΡΠ°ΠΆΠ΅Π½ΠΈΡ Π΄Π»Ρ ΡΠΈΡΠΊΡΠ»ΡΡΠΈΠΈ ΡΠΏΡΠ°Π²Π΅Π΄Π»ΠΈΠ²Ρ Π²ΡΠ΅Π³Π΄Π°. ΠΠ°Π³ΡΠ΅Π²Π°Π½ΠΈΠ΅ ΠΏΡΠΎΠ²ΠΎΠ΄Π½ΠΈΠΊΠΎΠ². Π‘ΠΊΠ°ΡΠ΅ΠΊ Π²ΡΠ»Π΅Π΄ΡΡΠ²ΠΈΠ΅ Π±ΠΎΠ»ΡΡΠΎΠΉ Π²Π΅Π»ΠΈΡΠΈΠ½Ρ ΡΠΊΠΎΡΠΎΡΡΠΈ ΠΈΠ·ΠΌΠ΅Π½Π΅Π½ΠΈΡ ΡΠΎΠΊΠ°.
Β«ΠΠ»Π΅ΠΊΡΡΠΎΠΌΠ°Π³Π½ΠΈΡΠ½Π°Ρ ΠΈΠ½Π΄ΡΠΊΡΠΈΡΒ» — ΠΠ°ΠΉΠΊΠ» Π€Π°ΡΠ°Π΄Π΅ΠΉ. ΠΠ°ΡΠ΅ΡΠΈΠ°Π». ΠΠ΅Π»ΠΈΡΠΈΠ½Π° ΡΠΎΠΊΠ°. ΠΠ½Π΄ΡΠΊΡΠΈΠΎΠ½Π½ΡΠΉ ΡΠΎΠΊ. ΠΡΠ΅Π½ΠΊΠ°. ΠΡΡΠΎΡΠΈΡ. ΠΠ»Π΅ΠΊΡΡΠΎΠΌΠ°Π³Π½ΠΈΡΠ½Π°Ρ ΠΈΠ½Π΄ΡΠΊΡΠΈΡ ΠΈ ΠΏΡΠΈΠ±ΠΎΡ. ΠΠ΅Π½Π΅ΡΠ°ΡΠΎΡ ΠΏΠ΅ΡΠ΅ΠΌΠ΅Π½Π½ΠΎΠ³ΠΎ ΡΠΎΠΊΠ°. Π‘ΠΈΠ½ΠΊΠ²Π΅ΠΉΠ½. ΠΡΡΠΎΡΠΈΡΠ΅ΡΠΊΠ°Ρ ΡΠΏΡΠ°Π²ΠΊΠ°. Π£ΡΠΎΠ²Π΅Π½Ρ. ΠΠΏΡΡΡ Π€Π°ΡΠ°Π΄Π΅Ρ. Π’Π΅ΡΡ-Π»ΠΈΡΡ Ρ Π·Π°Π΄Π°Π½ΠΈΡΠΌΠΈ. Π―Π²Π»Π΅Π½ΠΈΠ΅. Π£Π½ΠΈΠΏΠΎΠ»ΡΡΠ½Π°Ρ ΠΈΠ½Π΄ΡΠΊΡΠΈΡ. ΠΡΡΡΠΈΠ΅. ΠΠ°Π³Π½ΠΈΡΠ½Π°Ρ ΡΡΡΠ΅Π»ΠΊΠ°. ΠΡΠΎΠ²ΠΎΠ΄Π½ΠΈΠΊ. ΠΠΈΠ΄Π΅ΠΎΡΡΠ°Π³ΠΌΠ΅Π½Ρ.
Β«ΠΠ·ΡΡΠ΅Π½ΠΈΠ΅ ΡΠ»Π΅ΠΊΡΡΠΎΠΌΠ°Π³Π½ΠΈΡΠ½ΠΎΠΉ ΠΈΠ½Π΄ΡΠΊΡΠΈΠΈΒ» — ΠΠΎΡΡΡΠ΅Ρ ΠΠ°ΠΉΠΊΠ»Π° Π€Π°ΡΠ°Π΄Π΅Ρ. Π‘ΠΈΠ»Π° ΠΈΠ½Π΄ΡΠΊΡΠΈΠΎΠ½Π½ΠΎΠ³ΠΎ ΡΠΎΠΊΠ°. ΠΠΎΠΏΡΠΎΡΡ. ΠΠΎΠΏΡΠΎΡΡ ΠΈ Π·Π°Π΄Π°Π½ΠΈΡ. Π£ΡΠ²Π΅ΡΠΆΠ΄Π΅Π½ΠΈΠ΅. ΠΠ»Π΅ΠΊΡΡΠΎΠΌΠ°Π³Π½ΠΈΡΠ½Π°Ρ ΠΈΠ½Π΄ΡΠΊΡΠΈΡ. Π―Π²Π»Π΅Π½ΠΈΠ΅ ΠΠΠ. ΠΠ»Π΅ΠΊΡΡΠΎΠΌΠ°Π³Π½ΠΈΡΠ½ΠΎΠ΅ ΠΏΠΎΠ»Π΅. ΠΠ½Π΅ΡΠ³ΠΈΡ ΠΌΠ°Π³Π½ΠΈΡΠ½ΠΎΠ³ΠΎ ΠΏΠΎΠ»Ρ ΡΠΎΠΊΠ°. ΠΠ°ΠΏΡΠ°Π²Π»Π΅Π½ΠΈΠ΅ Π»ΠΈΠ½ΠΈΠΉ Π½Π°ΠΏΡΡΠΆΠ΅Π½Π½ΠΎΡΡΠΈ. ΠΡΠ°Π²ΠΈΠ»ΠΎ ΠΠ΅Π½ΡΠ°. ΠΠ°Π³Π½ΠΈΡΠ½ΡΠΉ ΠΏΠΎΡΠΎΠΊ. ΠΠ°Π³Π½ΠΈΡΠ½ΡΠΉ ΠΏΠΎΡΠΎΠΊ ΡΠ΅ΡΠ΅Π· ΠΏΠΎΠ²Π΅ΡΡ Π½ΠΎΡΡΡ. ΠΠΠ‘ ΠΈΠ½Π΄ΡΠΊΡΠΈΠΈ Π² Π΄Π²ΠΈΠΆΡΡΠΈΡ ΡΡ ΠΏΡΠΎΠ²ΠΎΠ΄Π½ΠΈΠΊΠ°Ρ .
Β«Π‘Π°ΠΌΠΎΠΈΠ½Π΄ΡΠΊΡΠΈΡ ΠΈ ΠΈΠ½Π΄ΡΠΊΡΠΈΠ²Π½ΠΎΡΡΡΒ» — Π―Π²Π»Π΅Π½ΠΈΠ΅ Π²ΠΎΠ·Π½ΠΈΠΊΠ½ΠΎΠ²Π΅Π½ΠΈΡ ΠΠΠ‘. Π‘Π°ΠΌΠΎΠΈΠ½Π΄ΡΠΊΡΠΈΡ. ΠΡΠΎΡΠ²Π»Π΅Π½ΠΈΠ΅ ΡΠ²Π»Π΅Π½ΠΈΡ ΡΠ°ΠΌΠΎΠΈΠ½Π΄ΡΠΊΡΠΈΠΈ. ΠΡΠΎΠ²ΠΎΠ΄Π½ΠΈΠΊ. ΠΠ°Π³Π½ΠΈΡΠ½ΡΠΉ ΠΏΠΎΡΠΎΠΊ ΡΠ΅ΡΠ΅Π· ΠΊΠΎΠ½ΡΡΡ. ΠΠ½Π΅ΡΠ³ΠΈΡ ΠΌΠ°Π³Π½ΠΈΡΠ½ΠΎΠ³ΠΎ ΠΏΠΎΠ»Ρ. ΠΠ½Π΄ΡΠΊΡΠΈΠ²Π½ΠΎΡΡΡ ΠΊΠ°ΡΡΡΠΊΠΈ. ΠΠ°Π³Π½ΠΈΡΠ½ΡΠΉ ΠΏΠΎΡΠΎΠΊ.
Β«ΠΠ½Π΄ΡΠΊΡΠΈΡ ΠΏΠΎΠ»ΡΒ» — ΠΠΎΡΠΎΠΊ Π²Π΅ΠΊΡΠΎΡΠ° ΠΈΠ½Π΄ΡΠΊΡΠΈΠΈ. ΠΠΎΡΠΎΠΊ ΠΌΠ°Π³Π½ΠΈΡΠ½ΠΎΠΉ ΠΈΠ½Π΄ΡΠΊΡΠΈΠΈ. ΠΠΎΠ½ΡΡΡ Π²ΡΠΏΠΎΠ»Π½Π΅Π½ ΠΈΠ· Π΄ΠΈΡΠ»Π΅ΠΊΡΡΠΈΠΊΠ°. ΠΠΠ‘ ΠΈΠ½Π΄ΡΠΊΡΠΈΠΈ. Π’ΠΎΠΊ ΠΏΡΠ°ΠΊΡΠΈΡΠ΅ΡΠΊΠΈ ΡΠ°Π²Π½ΠΎΠΌΠ΅ΡΠ½ΠΎ ΡΠ°ΡΠΏΡΠ΅Π΄Π΅Π»Π΅Π½ ΠΏΠΎ ΠΎΠ±ΡΠ΅ΠΌΡ ΠΏΡΠΎΠ²ΠΎΠ΄ΠΎΠ². Π Π°Π·ΠΌΠ΅ΡΠ½ΠΎΡΡΡ ΠΠΠ‘ ΠΈΠ½Π΄ΡΠΊΡΠΈΠΈ. Π€Π°ΠΊΡ. Π¦ΠΈΡΠΊΡΠ»ΡΡΠΈΡ Π²Π΅ΠΊΡΠΎΡΠ°. Π’ΠΎΠΊΠΈ Π²ΡΡΠΎΠΊΠΎΠΉ ΡΠ°ΡΡΠΎΡΡ. Π’ΠΎΠΊΠΈ Π€ΡΠΊΠΎ. ΠΡΠΎΠ²ΠΎΠ΄Π½ΠΈΠΊ Π½Π΅ΠΏΠΎΠ΄Π²ΠΈΠΆΠ΅Π½. ΠΠ΅Π»ΠΈΡΠΈΠ½Π° Π.Π.Π‘. ΠΈΠ½Π΄ΡΠΊΡΠΈΠΈ. ΠΠ»ΠΎΡΠ½ΠΎΡΡΡ ΡΠΎΠΊΠ°.
Β«ΠΠ»Π΅ΠΊΡΡΠΎΠΌΠ°Π³Π½ΠΈΡΠ½Π°Ρ ΠΈΠ½Π΄ΡΠΊΡΠΈΡ Π€Π°ΡΠ°Π΄Π΅ΡΒ» — Π€ΠΈΠ·ΠΊΡΠ»ΡΡΠΌΠΈΠ½ΡΡΠΊΠ°. ΠΡΠΈΠ½ΡΠΈΠΏ Π΄Π΅ΠΉΡΡΠ²ΠΈΡ Π³Π΅Π½Π΅ΡΠ°ΡΠΎΡΠ°. ΠΡΠ΅ΠΌΡ Π΄Π²ΠΈΠΆΠ΅Π½ΠΈΡ ΠΌΠ°Π³Π½ΠΈΡΠ°. ΠΠ½Π΅ΡΠ½ΠΈΠΉ Π²ΠΈΠ΄ Π³Π΅Π½Π΅ΡΠ°ΡΠΎΡΠ°. ΠΠΏΡΡ. Π―Π²Π»Π΅Π½ΠΈΠ΅ ΠΠΠ. Π‘ΠΈΡΡΠ΅ΠΌΠ°ΡΠΈΠ·ΠΈΡΠΎΠ²Π°ΡΡ Π·Π½Π°Π½ΠΈΡ. ΠΡΠΊΡΡΡΠΎ Π€Π°ΡΠ°Π΄Π΅Π΅ΠΌ. Π―Π²Π»Π΅Π½ΠΈΠ΅ ΡΠ»Π΅ΠΊΡΡΠΎΠΌΠ°Π³Π½ΠΈΡΠ½ΠΎΠΉ ΠΈΠ½Π΄ΡΠΊΡΠΈΠΈ. ΡΠ΅ΡΠ΅Π½ΠΈΠ΅ Π·Π°Π΄Π°Ρ Π»ΠΈΠ½Π΅ΠΉΠ½ΠΎΠΉ ΡΡΡΡΠΊΡΡΡΡ. ΠΠ½Π΄ΡΠΊΡΠΈΠΎΠ½Π½ΡΠΉ ΡΠΎΠΊ. ΠΠΎΠΏΡΠΎΡΡ.
ΠΡΠ΅Π³ΠΎ Π² ΡΠ΅ΠΌΠ΅ 18 ΠΏΡΠ΅Π·Π΅Π½ΡΠ°ΡΠΈΠΉ
ΠΡΠΏΠΎΠ»ΡΠ·ΡΡ ΠΎΠΏΡΠ΅Π΄Π΅Π»Π΅Π½ΠΈΠ΅ ΠΌΠ°Π³Π½ΠΈΡΠ½ΠΎΠ³ΠΎ ΠΏΠΎΠ»Ρ (5Π»16) ΠΈ ΡΠ²ΠΎΠ΄Ρ ΠΌΠ°Π³Π½ΠΈΡΠ½ΡΡ ΡΠΈΠ»Ρ, Π΄Π΅ΠΉΡΡΠ²ΡΡΡΡΡ Π½Π° ΠΏΡΠΎΠ²ΠΎΠ΄Π½ΠΈΠΊ Ρ ΡΠΎΠΊΠΎΠΌ, ΠΊ ΡΠΈΠ»Π°ΠΌ, ΠΈΡΠΏΡΡΡΠ²Π°Π΅ΠΌΡΠΌ Π΄Π²ΠΈΠΆΡΡΠΈΠΌΠΈΡΡ Π² Π½Π΅ΠΌ Π·Π°ΡΡΠ΄Π°ΠΌΠΈ, ΠΌΡ ΠΏΠΎΠ»ΡΡΠΈΠ»ΠΈ Π²ΡΡΠ°ΠΆΠ΅Π½ΠΈΠ΅ Π΄Π»Ρ ΡΠΈΠ»Ρ ΠΠΎΡΠ΅Π½ΡΠ° (16Π»17). Π‘ΠΎΠ³Π»Π°ΡΠ½ΠΎ Π΄Π°Π½Π½ΠΎΠΌΡ Π½Π°ΠΌΠΈ Π² Π»Π΅ΠΊΡΠΈΠΈ 15 ΠΎΠΏΡΠ΅Π΄Π΅Π»Π΅Π½ΠΈΡ, ΡΠΈΠ»Π° ΡΡΠ° ΡΠ²Π»ΡΠ΅ΡΡΡ ΡΡΠΎΡΠΎΠ½Π½Π΅ΠΉ (ΠΈΠ±ΠΎ ΠΎΠ½Π° Π½Π΅ΠΊΡΠ»ΠΎΠ½ΠΎΠ²ΡΠΊΠ°Ρ) ΠΈ Π΄ΠΎΠ»ΠΆΠ½Π° Π²ΠΎΠ·Π½ΠΈΠΊΠ°ΡΡ Π½Π΅ ΡΠΎΠ»ΡΠΊΠΎ ΠΏΡΠΈ Π΄Π²ΠΈΠΆΠ΅Π½ΠΈΠΈ Π·Π°ΡΡΠ΄ΠΎΠ² Π²Π½ΡΡΡΠΈ ΠΏΡΠΎΠ²ΠΎΠ΄Π½ΠΈΠΊΠ° (Ρ.
ΠΡΡΡΡ ΠΏΡΡΠΌΠΎΠ»ΠΈΠ½Π΅ΠΉΠ½ΡΠΉ ΠΎΡΡΠ΅Π·ΠΎΠΊ ΡΠΈΠ»ΠΈΠ½Π΄ΡΠΈΡΠ΅ΡΠΊΠΎΠ³ΠΎ ΠΏΡΠΎΠ²ΠΎΠ΄Π½ΠΈΠΊΠ° l ΠΏΠ΅ΡΠ΅ΠΌΠ΅ΡΠ°Π΅ΡΡΡ Π² ΠΎΠ΄Π½ΠΎΡΠΎΠ΄Π½ΠΎΠΌ ΠΌΠ°Π³Π½ΠΈΡΠ½ΠΎΠΌ ΠΏΠΎΠ»Π΅ B ΠΈ ΠΏΡΡΡΡ Π΅Π³ΠΎ ΡΠΊΠΎΡΠΎΡΡΡ v ΠΏΠ΅ΡΠΏΠ΅Π½Π΄ΠΈΠΊΡΠ»ΡΡΠ½Π° B ΠΈ ΠΎΡΠΈ ΠΏΡΠΎΠ²ΠΎΠ΄Π½ΠΈΠΊΠ° (ΡΠΈΡ. 1). ΠΠ° ΠΏΠΎΠ»ΠΎΠΆΠΈΡΠ΅Π»ΡΠ½ΡΠ΅ Π·Π°ΡΡΠ΄Ρ
Π ΠΈΡ. 1. |
F Π» = qvB , (1)
Π° Π½Π°ΠΏΡΠ°Π²Π»Π΅Π½ΠΈΠ΅ ΠΏΠΎΠΊΠ°Π·Π°Π½ΠΎ Π½Π° ΡΠΈΡΡΠ½ΠΊΠ΅. ΠΠ° ΠΎΡΡΠΈΡΠ°ΡΠ΅Π»ΡΠ½ΡΠ΅ Π·Π°ΡΡΠ΄Ρ ΡΠΈΠ»Π° F Π» Π±ΡΠ΄Π΅Ρ Π΄Π΅ΠΉΡΡΠ²ΠΎΠ²Π°ΡΡ Π² ΠΏΡΠΎΡΠΈΠ²ΠΎΠΏΠΎΠ»ΠΎΠΆΠ½ΠΎΠΌ Π½Π°ΠΏΡΠ°Π²Π»Π΅Π½ΠΈΠΈ. ΠΠΎΠ·Π½ΠΈΠΊΠ°ΡΡΠ°Ρ Π½Π° ΡΡΠ°ΡΡΠΊΠ΅ l ΠΠΠ‘ ΠΏΠΎ ΠΎΠΏΡΠ΅Π΄Π΅Π»Π΅Π½ΠΈΡ
e 12 = A 12 = F Π» l = vBl (2)
ΠΈ Π½Π°ΠΏΡΠ°Π²Π»Π΅Π½Π° Π΄Π»Ρ Π·Π°ΡΡΠ΄ΠΎΠ² ΠΎΠ±ΠΎΠΈΡ Π·Π½Π°ΠΊΠΎΠ² Π²Π΄ΠΎΠ»Ρ ΠΈΠ·ΠΎΠ±ΡΠ°ΠΆΠ΅Π½Π½ΠΎΠΉ Π½Π° ΡΠΈΡ. 1 F Π» .
ΠΡΠ»ΠΈ ΠΏΡΠ΅Π΄ΡΡΠ°Π²ΠΈΡΡ ΡΠ΅Π±Π΅, ΡΡΠΎ ΠΎΡΡΠ΅Π·ΠΎΠΊ l ΠΏΡΠΎΠ²ΠΎΠ΄Π½ΠΈΠΊΠ° ΡΠ²Π»ΡΠ΅ΡΡΡ ΡΠ°ΡΡΡΡ Π·Π°ΠΌΠΊΠ½ΡΡΠΎΠΉ ΠΊΠ²Π°Π·ΠΈΠ»ΠΈΠ½Π΅ΠΉΠ½ΠΎΠΉ ΡΠ΅ΠΏΠΈ, ΠΊΠΎΠ½ΡΡΡ ΠΊΠΎΡΠΎΡΠΎΠΉ ΠΈΠ·ΠΎΠ±ΡΠ°ΠΆΠ΅Π½ Π½Π° ΡΠΈΡ. 1 ΠΏΡΠ½ΠΊΡΠΈΡΠΎΠΌ, ΡΠΎ ΠΏΠΎΠ»ΡΡΠ΅Π½Π½ΠΎΠΌΡ ΡΠ΅Π·ΡΠ»ΡΡΠ°ΡΡ ΠΌΠΎΠΆΠ΅Ρ Π±ΡΡΡ ΠΏΡΠΈΠ΄Π°Π½Π° ΡΠ»Π΅Π΄ΡΡΡΠ°Ρ ΡΠΎΡΠΌΠ°. ΠΠΎΡΠΊΠΎΠ»ΡΠΊΡ ,
vBl = = = = , (3)
Π³Π΄Π΅ DS = l Dx β ΡΠ²Π΅Π»ΠΈΡΠ΅Π½ΠΈΠ΅ ΠΏΠ»ΠΎΡΠ°Π΄ΠΈ ΠΊΠΎΠ½ΡΡΡΠ°, Π° DΠ€ = D(BS ) β ΠΏΠΎΡΠΎΠΊΠ° Π²Π΅ΠΊΡΠΎΡΠ° B ΡΠ΅ΡΠ΅Π· Π½Π΅Π³ΠΎ Π·Π° Π²ΡΠ΅ΠΌΡ Dt . Π’Π°ΠΊ ΠΊΠ°ΠΊ ΠΎΡΡΠ°Π»ΡΠ½ΡΠ΅ ΡΡΠ°ΡΡΠΊΠΈ Π Π½Π΅ΠΏΠΎΠ΄Π²ΠΈΠΆΠ½Ρ, Π² Π½ΠΈΡ Π½Π΅ ΠΏΠΎΡΠ²Π»ΡΡΡΡΡ ΡΡΠΎΡΠΎΠ½Π½ΠΈΠ΅ ΡΠΈΠ»Ρ ΠΈ, ΡΠ»Π΅Π΄ΠΎΠ²Π°ΡΠ΅Π»ΡΠ½ΠΎ, ΠΏΠΎΠ»Π½Π°Ρ ΠΠΠ‘ e, Π΄Π΅ΠΉΡΡΠ²ΡΡΡΠ°Ρ Π²Π΄ΠΎΠ»Ρ Π²ΡΠ΅Π³ΠΎ ΠΊΠΎΠ½ΡΡΡΠ°, ΡΠ°ΠΊΠΆΠ΅ ΠΎΠΏΡΠ΅Π΄Π΅Π»ΡΠ΅ΡΡΡ Π²ΡΡΠ°ΠΆΠ΅Π½ΠΈΠ΅ΠΌ (2). ΠΠ· ΡΠΈΡ. 1 Π²ΠΈΠ΄Π½ΠΎ, ΡΡΠΎ ΠΎΠ½Π° ΡΠΎΡΡΠ°Π²Π»ΡΠ΅Ρ Ρ Π½Π°ΠΏΡΠ°Π²Π»Π΅Π½ΠΈΠ΅ΠΌ B Π»Π΅Π²ΠΎΠ²ΠΈΠ½ΡΠΎΠ²ΡΡ ΡΠΈΡΡΠ΅ΠΌΡ. Π’Π°ΠΊΠΈΠΌ ΠΎΠ±ΡΠ°Π·ΠΎΠΌ, ΠΌΠΎΠΆΠ½ΠΎ Π½Π°ΠΏΠΈΡΠ°ΡΡ, ΡΡΠΎ
ΠΏΡΠΈΡΠ΅ΠΌ Π·Π½Π°ΠΊ ΠΌΠΈΠ½ΡΡ ΡΠΎΠΎΡΠ²Π΅ΡΡΡΠ²ΡΠ΅Ρ ΡΡΡΠ°Π½ΠΎΠ²Π»Π΅Π½Π½ΠΎΠΌΡ Π½Π°ΠΌΠΈ Π² ΠΏΡΠ΅Π΄ΡΠ΄ΡΡΠ΅ΠΉ Π»Π΅ΠΊΡΠΈΠΈ ΠΏΡΠ°Π²ΠΈΠ»Ρ, ΡΠ²ΡΠ·ΡΠ²Π°ΡΡΠ΅ΠΌΡ ΠΏΠΎΠ»ΠΎΠΆΠΈΡΠ΅Π»ΡΠ½ΠΎΠ΅ Π½Π°ΠΏΡΠ°Π²Π»Π΅Π½ΠΈΠ΅ ΠΎΠ±Ρ ΠΎΠ΄Π° ΠΊΠΎΠ½ΡΡΡΠ° ΠΈ ΠΏΠΎΠ»ΠΎΠΆΠΈΡΠ΅Π»ΡΠ½ΡΡ Π½ΠΎΡΠΌΠ°Π»Ρ ΠΊ Π½Π΅ΠΌΡ ΠΏΠΎΡΡΠ΅Π΄ΡΡΠ²ΠΎΠΌ
ΠΠΎΠΆΠ½ΠΎ ΠΏΠΎΠΊΠ°Π·Π°ΡΡ, ΡΡΠΎ ΡΠΎΠΎΡΠ½ΠΎΡΠ΅Π½ΠΈΠ΅ (4) ΡΠΏΡΠ°Π²Π΅Π΄Π»ΠΈΠ²ΠΎ Π² ΡΠ°ΠΌΠΎΠΌ ΠΎΠ±ΡΠ΅ΠΌ ΡΠ»ΡΡΠ°Π΅ ΠΏΡΠΎΠΈΠ·Π²ΠΎΠ»ΡΠ½ΠΎΠ³ΠΎ Π΄Π²ΠΈΠΆΠ΅Π½ΠΈΡ (Π²ΠΊΠ»ΡΡΠ°Ρ Π΄Π΅ΡΠΎΡΠΌΠ°ΡΠΈΡ) ΠΊΠΎΠ½ΡΡΡΠ° Π² ΡΡΠ°ΡΠΈΠΎΠ½Π°ΡΠ½ΠΎΠΌ ΠΌΠ°Π³Π½ΠΈΡΠ½ΠΎΠΌ ΠΏΠΎΠ»Π΅. ΠΠ½ΠΎ Π²ΡΡΠ°ΠΆΠ°Π΅Ρ ΡΠΎΠ±ΠΎΠΉ ΡΠ°ΠΊ Π½Π°Π·ΡΠ²Π°Π΅ΠΌΡΠΉ Π·Π°ΠΊΠΎΠ½ ΠΈΠ½Π΄ΡΠΊΡΠΈΠΈ ΡΠΎΠΊΠΎΠ² Π² Π΄Π²ΠΈΠΆΡΡΠΈΡ ΡΡ ΠΏΡΠΎΠ²ΠΎΠ΄Π½ΠΈΠΊΠ°Ρ : Π²ΠΎΠ·Π½ΠΈΠΊΠ°ΡΡΠ°Ρ Π² ΠΊΠΎΠ½ΡΡΡΠ΅ ΠΠΠ‘ ΠΈΠ½Π΄ΡΠΊΡΠΈΠΈ ΡΠ°Π²Π½Π° ΡΠΊΠΎΡΠΎΡΡΠΈ ΠΈΠ·ΠΌΠ΅Π½Π΅Π½ΠΈΡ ΠΌΠ°Π³Π½ΠΈΡΠ½ΠΎΠ³ΠΎ ΠΏΠΎΡΠΎΠΊΠ° ΡΠ΅ΡΠ΅Π· ΠΊΠΎΠ½ΡΡΡ ΠΈ ΡΠΎΡΡΠ°Π²Π»ΡΠ΅Ρ Ρ Π½ΠΈΠΌ (Ρ. Π΅. Ρ ΠΈΠ·ΠΌΠ΅Π½Π΅Π½ΠΈΠ΅ΠΌ) Π½Π΅ ΠΏΡΠ°Π²ΠΎ- (ΡΡΠΎ ΠΎΠ·Π½Π°ΡΠ°Π» Π±Ρ Π·Π½Π°ΠΊ Β«ΠΏΠ»ΡΡΒ» Π² (4)), Π° Π»Π΅Π²ΠΎΠ²ΠΈΠ½ΡΠΎΠ²ΡΡ ΡΠΈΡΡΠ΅ΠΌΡ .
Π ΠΈΡ. 2. |
ΠΠ°ΠΌΠ΅ΡΠ°Π½ΠΈΠ΅ 1. Π Π·Π°ΠΊΠΎΠ½Π΅ ΠΈΠ½Π΄ΡΠΊΡΠΈΠΈ (4) ΡΠ΅ΡΡ ΠΈΠ΄Π΅Ρ ΠΎ ΠΏΠΎΡΠΎΠΊΠ΅ Π²Π΅ΠΊΡΠΎΡΠ° B ΡΠ΅ΡΠ΅Π· Π·Π°ΠΌΠΊΠ½ΡΡΡΠΉ ΠΊΠΎΠ½ΡΡΡ Π, Ρ ΠΎΡΡ ΠΈΠΌΠ΅Π΅ΡΡΡ Π² Π²ΠΈΠ΄Ρ, ΠΊΠΎΠ½Π΅ΡΠ½ΠΎ, ΠΏΠΎΡΠΎΠΊ Π΅Π³ΠΎ ΡΠ΅ΡΠ΅Π· ΠΊΠ°ΠΊΡΡ-Π»ΠΈΠ±ΠΎ ΠΏΠΎΠ²Π΅ΡΡ Π½ΠΎΡΡΡ, ΠΎΠΏΠΈΡΠ°ΡΡΡΡΡΡ Π½Π° ΡΡΠΎΡ ΠΊΠΎΠ½ΡΡΡ (Π²Π΅Π΄Ρ ΠΈΠΌΠ΅Π½Π½ΠΎ ΡΠ΅ΡΠ΅Π· ΠΏΠΎΠ²Π΅ΡΡ Π½ΠΎΡΡΡ ΠΈ ΠΎΠΏΡΠ΅Π΄Π΅Π»ΡΠ΅ΡΡΡ ΠΏΠΎΡΠΎΠΊ Π»ΡΠ±ΠΎΠ³ΠΎ Π²Π΅ΠΊΡΠΎΡΠ°). ΠΠ΅ΡΡΡΠ΄Π½ΠΎ Π²ΠΈΠ΄Π΅ΡΡ, ΡΡΠΎ ΠΏΡΠΎΠΈΠ·Π²ΠΎΠ» Π² Π²ΡΠ±ΠΎΡΠ΅ ΡΡΠΎΠΉ ΠΏΠΎΠ²Π΅ΡΡ Π½ΠΎΡΡΠΈ Π½Π΅ ΡΠΊΠ°ΠΆΠ΅ΡΡΡ Π½Π° Π²Π΅Π»ΠΈΡΠΈΠ½Π΅ Π€. ΠΠ΅ΠΉΡΡΠ²ΠΈΡΠ΅Π»ΡΠ½ΠΎ, Π½Π°ΡΡΠ³ΠΈΠ²Π°Ρ Π½Π° ΠΊΠΎΠ½ΡΡΡ Π Π΄Π²Π΅ ΠΏΡΠΎΠΈΠ·Π²ΠΎΠ»ΡΠ½ΡΠ΅ ΠΏΠΎΠ²Π΅ΡΡ Π½ΠΎΡΡΠΈ S 1 ΠΈ S 2 , ΠΌΡ ΠΏΠΎΠ»ΡΡΠΈΠΌ Π·Π°ΠΌΠΊΠ½ΡΡΡΡ ΠΏΠΎΠ²Π΅ΡΡ Π½ΠΎΡΡΡ S S , ΠΏΠΎΡΠΎΠΊ Π²Π΅ΠΊΡΠΎΡΠ° B ΡΠ΅ΡΠ΅Π· ΠΊΠΎΡΠΎΡΡΡ ΡΠΎΠ³Π»Π°ΡΠ½ΠΎ ΡΡΠ°Π²Π½Π΅Π½ΠΈΡ (9Π»17) ΡΠ°Π²Π΅Π½ Π½ΡΠ»Ρ. ΠΡΠΎ Π·Π½Π°ΡΠΈΡ, ΡΡΠΎ ΠΏΠΎΡΠΎΠΊΠΈ ΡΠ΅ΡΠ΅Π· S 1 ΠΈ S 2 ΡΠ°Π²Π½Ρ ΠΈ ΠΏΡΠΎΡΠΈΠ²ΠΎΠΏΠΎΠ»ΠΎΠΆΠ½Ρ, ΠΏΡΠΈΡΠ΅ΠΌ ΠΏΠΎ ΡΠΌΡΡΠ»Ρ (9Π»17) Π½ΠΎΡΠΌΠ°Π»ΠΈ ΠΊ S 1 ΠΈ S 2 ΠΏΡΠΈ ΡΡΠΎΠΌ Π΄ΠΎΠ»ΠΆΠ½Ρ Π±ΡΡΡ Π½Π°ΠΏΡΠ°Π²Π»Π΅Π½Ρ Π½Π°ΡΡΠΆΡ, Ρ. Π΅. ΠΎΠ΄Π½Π° ΠΈΠ· Π½ΠΈΡ ΠΎΠ±ΡΠ°Π·ΡΡΡ Ρ Π½Π°ΠΏΡΠ°Π²Π»Π΅Π½ΠΈΠ΅ΠΌ ΠΎΠ±Ρ ΠΎΠ΄Π° Π ΠΏΡΠ°Π²ΠΎΠ²ΠΈΠ½ΡΠΎΠ²ΡΡ, Π° Π΄ΡΡΠ³Π°Ρ β Π»Π΅Π²ΠΎΠ²ΠΈΠ½ΡΠΎΠ²ΡΡ ΡΠΈΡΡΠ΅ΠΌΡ. ΠΠ΅Π½ΡΡ Π½Π°ΠΏΡΠ°Π²Π»Π΅Π½ΠΈΠ΅ ΠΏΠΎΡΠ»Π΅Π΄Π½Π΅ΠΉ Π½Π° ΠΏΡΠΎΡΠΈΠ²ΠΎΠΏΠΎΠ»ΠΎΠΆΠ½ΠΎΠ΅ (Π° Π²ΠΌΠ΅ΡΡΠ΅ Ρ Π½Π΅ΠΉ ΠΈ Π·Π½Π°ΠΊ ΡΠΎΠΎΡΠ²Π΅ΡΡΡΠ²ΡΡΡΠ΅Π³ΠΎ Π€), ΠΏΠΎΠ»ΡΡΠ°Π΅ΠΌ Π½Π΅Π·Π°Π²ΠΈΡΠΈΠΌΠΎΡΡΡ ΠΏΠΎΡΠΎΠΊΠ°, Π²Ρ ΠΎΠ΄ΡΡΠ΅Π³ΠΎ Π² (4), ΠΎΡ Π²ΡΠ±ΠΎΡΠ° ΠΏΠΎΠ²Π΅ΡΡ Π½ΠΎΡΡΠΈ S .
ΠΠ°ΠΌΠ΅ΡΠ°Π½ΠΈΠ΅ 2. ΠΡΠΈ Π²ΡΠ²ΠΎΠ΄Π΅ ΡΠΎΡΠΌΡΠ»Ρ (2) ΠΏΡΠ΅Π΄ΠΏΠΎΠ»Π°Π³Π°Π»ΠΎΡΡ, ΡΡΠΎ Π΄Π²ΠΈΠΆΡΡΠΈΠΉΡΡ Π² ΠΌΠ°Π³Π½ΠΈΡΠ½ΠΎΠΌ ΠΏΠΎΠ»Π΅ ΠΎΡΡΠ΅Π·ΠΎΠΊ ΠΏΡΠΎΠ²ΠΎΠ΄Π½ΠΈΠΊΠ° Π·Π°ΠΌΠΊΠ½ΡΡΠΎΠΉ ΡΠ΅ΠΏΠΈ Π½Π΅ ΠΎΠ±ΡΠ°Π·ΡΠ΅Ρ, Ρ. Π΅. Π² Π½Π΅ΠΌ ΡΠΎΠΊ Π½Π΅ ΡΠ΅ΡΠ΅Ρ, Ρ ΠΎΡΡ ΠΏΠΎΠ»ΡΡΠ΅Π½Π½ΡΠΉ Π² ΡΠ΅Π·ΡΠ»ΡΡΠ°ΡΠ΅ Π΅Π΅ ΠΎΠ±ΠΎΠ±ΡΠ΅Π½ΠΈΡ Π·Π°ΠΊΠΎΠ½ (4) ΠΎΡΠ½ΠΎΡΠΈΡΡΡ ΠΈΠΌΠ΅Π½Π½ΠΎ ΠΊ Π·Π°ΠΌΠΊΠ½ΡΡΠΎΠΌΡ ΠΏΡΠΎΠ²ΠΎΠ΄ΡΡΠ΅ΠΌΡ ΠΊΠΎΠ½ΡΡΡΡ. ΠΠΎΡΠΌΠΎΡΡΠΈΠΌ, ΠΊ ΠΊΠ°ΠΊΠΈΠΌ ΡΡΡΠ΅ΠΊΡΠ°ΠΌ ΠΏΡΠΈΠ²Π΅Π΄Π΅Ρ ΠΏΠΎΡΠ²Π»Π΅Π½ΠΈΠ΅ ΡΠΎΠΊΠ° Π² ΡΠ°ΡΡΠΌΠ°ΡΡΠΈΠ²Π°Π΅ΠΌΠΎΠΌ ΠΏΡΠΎΠ²ΠΎΠ΄Π½ΠΈΠΊΠ΅ (ΡΠΈΡ. 2). ΠΠΎΠ·Π½ΠΈΠΊΠ½ΠΎΠ²Π΅Π½ΠΈΠ΅ ΡΠΊΠΎΡΠΎΡΡΠΈ u ΡΠΏΠΎΡΡΠ΄ΠΎΡΠ΅Π½Π½ΠΎΠ³ΠΎ Π΄Π²ΠΈΠΆΠ΅Π½ΠΈΡ Π½ΠΎΡΠΈΡΠ΅Π»Π΅ΠΉ, Π½Π°ΠΏΡΠ°Π²Π»Π΅Π½Π½ΠΎΠΉ Π²Π΄ΠΎΠ»Ρ ΠΎΡΠΈ ΠΏΡΠΎΠ²ΠΎΠ΄Π½ΠΈΠΊΠ°, Π²ΡΠ·ΠΎΠ²Π΅Ρ ΠΏΠΎΠ²ΠΎΡΠΎΡ Π½Π° Π½Π΅ΠΊΠΎΡΠΎΡΡΠΉ ΡΠ³ΠΎΠ» a Π°Π±ΡΠΎΠ»ΡΡΠ½ΠΎΠΉ ΡΠΊΠΎΡΠΎΡΡΠΈ v Π°Π±Ρ Π·Π°ΡΡΠ΄ΠΎΠ² ΠΎΡΠ½ΠΎΡΠΈΡΠ΅Π»ΡΠ½ΠΎ Π½Π°ΠΏΡΠ°Π²Π»Π΅Π½ΠΈΡ Π΄Π²ΠΈΠΆΠ΅Π½ΠΈΡ ΠΏΡΠΎΠ²ΠΎΠ΄Π½ΠΈΠΊΠ° (Ρ. Π΅. v ). ΠΡΠΈ ΡΡΠΎΠΌ ΡΠΈΠ»Π° ΠΠΎΡΠ΅Π½ΡΠ° F Π» ΠΎΡΡΠ°Π²Π°ΡΡΡ Π²ΡΠ΅Π³Π΄Π° ΠΏΠ΅ΡΠΏΠ΅Π½Π΄ΠΈΠΊΡΠ»ΡΡΠ½ΠΎΠΉ v Π°Π±Ρ , ΡΠΎΠΆΠ΅ ΠΏΠΎΠ²Π΅ΡΠ½Π΅ΡΡΡ Π½Π° ΡΠ³ΠΎΠ» a ΠΎΡΠ½ΠΎΡΠΈΡΠ΅Π»ΡΠ½ΠΎ ΠΎΡΠΈ ΠΏΡΠΎΠ²ΠΎΠ΄Π½ΠΈΠΊΠ°. S , Π΄ΠΎΠ»ΠΆΠ½Π°, ΠΎΡΠ΅Π²ΠΈΠ΄Π½ΠΎ, ΡΠ°Π·Π²ΠΈΡΡ ΠΌΠΎΡΠ½ΠΎΡΡΡ
P ΠΌΠ΅Ρ = β PΒ’ ΠΌΠ΅Ρ = P ΡΡΡ ,
ΠΊΠΎΡΠΎΡΠ°Ρ ΠΈ Β«ΠΏΠ΅ΡΠ΅ΠΉΠ΄Π΅ΡΒ» Π² ΡΠ°Π±ΠΎΡΡ (Π² Π΅Π΄ΠΈΠ½ΠΈΡΡ Π²ΡΠ΅ΠΌΠ΅Π½ΠΈ) Π΄Π΅ΠΉΡΡΠ²ΡΡΡΠΈΡ Π²Π½ΡΡΡΠΈ Π½Π΅Π³ΠΎ ΡΡΠΎΡΠΎΠ½Π½ΠΈΡ ΡΠΈΠ» ΠΈΠ½Π΄ΡΠΊΡΠΈΠΈ.
ΠΠ½Π°Π»ΠΎΠ³ΠΈΡΠ½ΡΠ΅ ΡΠ²Π»Π΅Π½ΠΈΡ ΠΏΡΠΎΠΈΡΡ ΠΎΠ΄ΡΡ ΠΈ ΠΏΡΠΈ Π΄Π²ΠΈΠΆΠ΅Π½ΠΈΠΈ Π² ΠΌΠ°Π³Π½ΠΈΡΠ½ΠΎΠΌ ΠΏΠΎΠ»Π΅ ΠΏΡΠΎΠ²ΠΎΠ΄Π½ΠΈΠΊΠ°, ΠΊ ΠΊΠΎΠ½ΡΠ°ΠΌ ΠΊΠΎΡΠΎΡΠΎΠ³ΠΎ ΠΏΡΠΈΠ»ΠΎΠΆΠ΅Π½Π° ΡΠ°Π·Π½ΠΎΡΡΡ ΠΏΠΎΡΠ΅Π½ΡΠΈΠ°Π»ΠΎΠ². ΠΡΠ»ΠΈ ΠΏΡΠΎΠ²ΠΎΠ΄Π½ΠΈΠΊ Π½Π΅ΠΏΠΎΠ΄Π²ΠΈΠΆΠ΅Π½, ΡΠΎ ΡΠΎΠΊ Π½Π° ΡΡΠ°ΡΡΠΊΠ΅ 1 β 2 (ΡΠΈΡ. 3) ΡΠ΅ΡΠ΅Ρ ΡΠΎΠ»ΡΠΊΠΎ Π·Π° ΡΡΠ΅Ρ ΡΠ»Π΅ΠΊΡΡΠΈΡΠ΅ΡΠΊΠΈΡ ΡΠΈΠ». ΠΡΠ»ΠΈ ΠΆΠ΅ Π΅Π³ΠΎ Β«ΠΎΡΠΏΡΡΡΠΈΡΡΒ», ΡΠΎ ΠΏΠΎΠ΄ Π΄Π΅ΠΉΡΡΠ²ΠΈΠ΅ΠΌ ΠΌΠ°Π³Π½ΠΈΡΠ½ΠΎΠΉ ΡΠΈΠ»Ρ ΠΏΠΎΡΠ²ΠΈΡΡΡ ΡΠΊΠΎΡΠΎΡΡΡ v ΠΈ Π°Π±ΡΠΎΠ»ΡΡΠ½Π°Ρ ΡΠΊΠΎΡΠΎΡΡΡ Π½ΠΎΡΠΈΡΠ΅Π»Π΅ΠΉ v Π°Π±Ρ ΠΎΡΠΊΠ»ΠΎΠ½ΠΈΡΡΡ ΠΎΡ ΠΎΡΠΈ ΠΏΡΠΎΠ²ΠΎΠ΄Π½ΠΈΠΊΠ°. Π’ΠΎΡΡΠ°Ρ ΠΆΠ΅ ΠΏΠΎΠ²Π΅ΡΠ½Π΅ΡΡΡ ΠΈ ΡΠΈΠ»Π° F Π» ΠΠΎΡΠ΅Π½ΡΠ° ΠΈ Π²ΠΎΠ·Π½ΠΈΠΊΠ½Π΅Ρ Π΅Π΅ ΠΎΡΠ΅Π²Π°Ρ ΡΠΎΡΡΠ°Π²Π»ΡΡΡΠ°Ρ F || , Π½Π°ΠΏΡΠ°Π²Π»Π΅Π½Π½Π°Ρ Π½Π°Π²ΡΡΡΠ΅ΡΡ ΡΠΎΠΊΡ. ΠΠ½Π° ΠΏΠΎΠ²Π»Π΅ΡΠ΅Ρ Π·Π° ΡΠΎΠ±ΠΎΠΉ ΠΏΠΎΡΠ²Π»Π΅Π½ΠΈΠ΅ ΡΡΠΎΡΠΎΠ½Π½Π΅ΠΉ ΠΠΠ‘ e 21 Π΄Π»Ρ ΠΊΠΎΠΌΠΏΠ΅Π½ΡΠ°ΡΠΈΠΈ Π΄Π΅ΠΉΡΡΠ²ΠΈΡ ΠΊΠΎΡΠΎΡΠΎΠΉ (Ρ. Π΅. ΡΠΎΡ ΡΠ°Π½Π΅Π½ΠΈΡ Π½Π΅ΠΈΠ·ΠΌΠ΅Π½Π½ΡΠΌ ΡΠΎΠΊΠ°) ΠΈΡΡΠΎΡΠ½ΠΈΠΊΡ Π½Π΅ΠΎΠ±Ρ ΠΎΠ΄ΠΈΠΌΠΎ ΡΠ°Π·Π²ΠΈΡΡ Π΄ΠΎΠΏΠΎΠ»Π½ΠΈΡΠ΅Π»ΡΠ½ΡΡ ΠΌΠΎΡΠ½ΠΎΡΡΡ e 21 I . ΠΠΎΠ²ΡΠΎΡΡΡ ΠΏΡΠΈΠ²Π΅Π΄Π΅Π½Π½ΡΠ΅ Π²ΡΡΠ΅ ΡΠ°ΡΡΡΠΆΠ΄Π΅Π½ΠΈΡ, Π½Π΅ΡΡΡΠ΄Π½ΠΎ ΠΏΠΎΠΊΠ°Π·Π°ΡΡ, ΡΡΠΎ ΠΈΠΌΠ΅Π½Π½ΠΎ ΡΡΠ° ΠΌΠΎΡΠ½ΠΎΡΡΡ Β«Π²ΡΠ΄Π΅Π»ΠΈΡΡΡΒ» Π² Π²ΠΈΠ΄Π΅ ΡΠΎΠ²Π΅ΡΡΠ΅Π½Π½ΠΎΠΉ ΠΏΡΠΎΠ²ΠΎΠ΄Π½ΠΈΠΊΠΎΠΌ (Π² Π΅Π΄ΠΈΠ½ΠΈΡΡ Π²ΡΠ΅ΠΌΠ΅Π½ΠΈ) ΠΌΠ΅Ρ Π°Π½ΠΈΡΠ΅ΡΠΊΠΎΠΉ ΡΠ°Π±ΠΎΡΡ. v Π°Π±Ρ ). ΠΡΡΠΈΡΠ°ΡΠ΅Π»ΡΠ½Π°Ρ ΡΠ°ΡΡΡ Π΅Π΅, Π²ΡΠ·Π²Π°Π½Π½Π°Ρ F || , ΠΊΠΎΠΌΠΏΠ΅Π½ΡΠΈΡΡΠ΅ΡΡΡ ΡΠ°Π±ΠΎΡΠΎΠΉ ΠΈΡΡΠΎΡΠ½ΠΈΠΊΠ° ΡΠΎΠΊΠ°, ΠΏΠΎΠ»ΠΎΠΆΠΈΡΠ΅Π»ΡΠ½Π°Ρ ΠΆΠ΅ ΠΏΡΠ΅Π΄ΡΡΠ°Π²Π»ΡΠ΅Ρ ΡΠΎΠ±ΠΎΠΉ ΠΏΠΎΠ»Π΅Π·Π½ΡΡ ΡΠ°Π±ΠΎΡΡ ΠΏΡΠΎΠ²ΠΎΠ΄Π½ΠΈΠΊΠ°.
Π ΠΈΡ. 3. |
17.04.2020 Π³. ΠΠΠ‘ ΠΈΠ½Π΄ΡΠΊΡΠΈΠΈ Π² Π΄Π²ΠΈΠΆΡΡΠΈΡ ΡΡ ΠΏΡΠΎΠ²ΠΎΠ΄Π½ΠΈΠΊΠ°Ρ . Π‘Π°ΠΌΠΎΠΈΠ½Π΄ΡΠΊΡΠΈΡ ΠΏΡΠ΅Π·Π΅Π½ΡΠ°ΡΠΈΡ, Π΄ΠΎΠΊΠ»Π°Π΄, ΠΏΡΠΎΠ΅ΠΊΡ
17.04.2020 Π³.
ΠΠΠ‘ ΠΈΠ½Π΄ΡΠΊΡΠΈΠΈ Π² Π΄Π²ΠΈΠΆΡΡΠΈΡ
ΡΡ ΠΏΡΠΎΠ²ΠΎΠ΄Π½ΠΈΠΊΠ°Ρ
. Π‘Π°ΠΌΠΎΠΈΠ½Π΄ΡΠΊΡΠΈΡ.
ΠΠ»Π°Π½ ΡΡΠΎΠΊΠ°:
ΠΡΠΎΠ²Π΅ΡΡΡΠ΅ ΡΠ²ΠΎΡ Π³ΠΎΡΠΎΠ²Π½ΠΎΡΡΡ ΠΊ ΠΈΠ·ΡΡΠ΅Π½ΠΈΡ Π½ΠΎΠ²ΠΎΠ³ΠΎ ΠΌΠ°ΡΠ΅ΡΠΈΠ°Π»Π° β ΠΎΡΠ²Π΅ΡΡΡΠ΅ Π½Π° Π²ΠΎΠΏΡΠΎΡΡ (ΡΠ»Π°ΠΉΠ΄Ρ2-3) ΠΈ
ΠΠ·ΡΡΠΈΡΠ΅ Π½ΠΎΠ²ΡΠΉ ΠΌΠ°ΡΠ΅ΡΠΈΠ°Π» (ΡΠ»Π°ΠΉΠ΄Ρ 4-15) ΠΈ Π·Π°ΠΏΠΈΡΠΈΡΠ΅ Π² ΡΠ΅ΡΡΠ°Π΄Ρ ΡΠ°ΠΌΠΎΠ΅ ΠΎΡΠ½ΠΎΠ²Π½ΠΎΠ΅.
Π Π΅ΡΠΈΡΠ΅ Π·Π°Π΄Π°ΡΠΈ (ΡΠ»Π°ΠΉΠ΄Ρ 16-27), ΠΏΠΎΡΡΠ°Π²ΠΈΠ² ΠΏΡΠ΅Π·Π΅Π½ΡΠ°ΡΠΈΡ Π½Π° Π²ΠΎΡΠΏΡΠΎΠΈΠ·Π²Π΅Π΄Π΅Π½ΠΈΠ΅ (ΡΠ½Π°ΡΠ°Π»Π° ΠΏΠΎΠΏΡΠΎΠ±ΡΠΉΡΠ΅ ΡΠ΅ΡΠΈΡΡ ΡΠ°ΠΌΠΎΡΡΠΎΡΡΠ΅Π»ΡΠ½ΠΎ, Π·Π°ΡΠ΅ΠΌ ΡΠΌΠΎΡΡΠΈΡΠ΅ ΠΎΡΠ²Π΅Ρ)
ΠΡΠΏΠΎΠ»Π½ΠΈΡΠ΅ ΡΠ΅ΡΡ Β«+ΡΠ°ΠΌΠΎΠΈΠ½Π΄ΡΠΊΡΠΈΡΒ» (Π½Π° ΡΡΠ΅Π½Π΅ Π² Π³ΡΡΠΏΠΏΠ΅)
ΠΡΠΎΠ²Π΅ΡΡΡΠ΅ ΡΠ²ΠΎΡ Π³ΠΎΡΠΎΠ²Π½ΠΎΡΡΡ ΠΊ ΠΈΠ·ΡΡΠ΅Π½ΠΈΡ Π½ΠΎΠ²ΠΎΠ³ΠΎ ΠΌΠ°ΡΠ΅ΡΠΈΠ°Π»Π°
ΠΠ°ΠΊ ΠΏΠΎΠΊΠ°Π·Π°ΡΡ Π½Π° ΠΎΠΏΡΡΠ΅, ΡΡΠΎ Π²Π΅Π»ΠΈΡΠΈΠ½Π° ΠΈΠ½Π΄ΡΠΊΡΠΈΠΎΠ½Π½ΠΎΠ³ΠΎ ΡΠΎΠΊΠ° Π·Π°Π²ΠΈΡΠΈΡ ΠΎΡ ΡΠΊΠΎΡΠΎΡΡΠΈ ΠΈΠ·ΠΌΠ΅Π½Π΅Π½ΠΈΡ ΠΌΠ°Π³Π½ΠΈΡΠ½ΠΎΠ³ΠΎ ΠΏΠΎΡΠΎΠΊΠ°?
ΠΠ°ΠΊ ΠΏΠΎΠΊΠ°Π·Π°ΡΡ Π½Π° ΠΎΠΏΡΡΠ΅, ΡΡΠΎ Π½Π°ΠΏΡΠ°Π²Π»Π΅Π½ΠΈΠ΅ ΠΈΠ½Π΄ΡΠΊΡΠΈΠΎΠ½Π½ΠΎΠ³ΠΎ ΡΠΎΠΊΠ° Π·Π°Π²ΠΈΡΠΈΡ ΠΎΡ Π½Π°ΠΏΡΠ°Π²Π»Π΅Π½ΠΈΡ Π»ΠΈΠ½ΠΈΠΉ ΠΌΠ°Π³Π½ΠΈΡΠ½ΠΎΠΉ ΠΈΠ½Π΄ΡΠΊΡΠΈΠΈ Π²Π½Π΅ΡΠ½Π΅Π³ΠΎ ΠΌΠ°Π³Π½ΠΈΡΠ½ΠΎΠ³ΠΎ ΠΏΠΎΠ»Ρ?
ΠΠ°ΠΊ ΡΠΈΡΠ°Π΅ΡΡΡ Π·Π°ΠΊΠΎΠ½ ΡΠ»Π΅ΠΊΡΡΠΎΠΌΠ°Π³Π½ΠΈΡΠ½ΠΎΠΉ ΠΈΠ½Π΄ΡΠΊΡΠΈΠΈ?
ΠΠΎΡΠ΅ΠΌΡ Π² Π·Π°ΠΊΠΎΠ½Π΅ ΡΠ»Π΅ΠΊΡΡΠΎΠΌΠ°Π³Π½ΠΈΡΠ½ΠΎΠΉ ΠΈΠ½Π΄ΡΠΊΡΠΈΠΈ ΡΡΠΎΠΈΡ Π·Π½Π°ΠΊ Β«ΠΌΠΈΠ½ΡΡΒ»?
ΠΠΎΡΠ΅ΠΌΡ Π·Π°ΠΊΠΎΠ½ ΡΠ»Π΅ΠΊΡΡΠΎΠΌΠ°Π³Π½ΠΈΡΠ½ΠΎΠΉ ΠΈΠ½Π΄ΡΠΊΡΠΈΠΈ ΡΠΎΡΠΌΡΠ»ΠΈΡΡΠ΅ΡΡΡ Π΄Π»Ρ ΠΠΠ‘ ΠΈΠ½Π΄ΡΠΊΡΠΈΠΈ, Π° Π½Π΅ Π΄Π»Ρ ΡΠΈΠ»Ρ ΠΈΠ½Π΄ΡΠΊΡΠΈΠΎΠ½Π½ΠΎΠ³ΠΎ ΡΠΎΠΊΠ°?
ΠΡΠΎΠ²Π΅ΡΡΡΠ΅ ΡΠ²ΠΎΡ Π³ΠΎΡΠΎΠ²Π½ΠΎΡΡΡ ΠΊ ΠΈΠ·ΡΡΠ΅Π½ΠΈΡ Π½ΠΎΠ²ΠΎΠ³ΠΎ ΠΌΠ°ΡΠ΅ΡΠΈΠ°Π»Π°
ΠΠ°ΠΊ Π½Π°ΠΏΡΠ°Π²Π»Π΅Π½Ρ Π»ΠΈΠ½ΠΈΠΈ ΠΌΠ°Π³Π½ΠΈΡΠ½ΠΎΠΉ ΠΈΠ½Π΄ΡΠΊΡΠΈΠΈ Π² ΠΏΠΎΡΡΠΎΡΠ½Π½ΠΎΠΌ ΠΌΠ°Π³Π½ΠΈΡΠ΅?
ΠΠ°ΠΊ Π½Π°ΠΏΡΠ°Π²Π»Π΅Π½Ρ Π»ΠΈΠ½ΠΈΠΈ ΠΌΠ°Π³Π½ΠΈΡΠ½ΠΎΠΉ ΠΈΠ½Π΄ΡΠΊΡΠΈΠΈ ΠΎΠΊΠΎΠ»ΠΎ ΠΏΡΠΎΠ²ΠΎΠ΄Π½ΠΈΠΊΠ° Ρ ΡΠΎΠΊΠΎΠΌ?
ΠΠ°ΠΊ Π½Π°ΠΏΡΠ°Π²Π»Π΅Π½Ρ Π»ΠΈΠ½ΠΈΠΈ ΠΌΠ°Π³Π½ΠΈΡΠ½ΠΎΠΉ ΠΈΠ½Π΄ΡΠΊΡΠΈΠΈ ΠΎΠΊΠΎΠ»ΠΎ Π²ΠΈΡΠΊΠ° Ρ ΡΠΎΠΊΠΎΠΌ?
ΠΠ΅ΡΠ΅ΡΠΈΡΠ»ΠΈΡΡ ΡΠ»ΡΡΠ°ΠΈ Π²ΠΎΠ·Π½ΠΈΠΊΠ½ΠΎΠ²Π΅Π½ΠΈΡ ΠΈΠ½Π΄ΡΠΊΡΠΈΠΎΠ½Π½ΠΎΠ³ΠΎ ΡΠΎΠΊΠ°.
Π§ΡΠΎ Π½Π°Π·ΡΠ²Π°Π΅ΡΡΡ ΠΌΠ°Π³Π½ΠΈΡΠ½ΡΠΌ ΠΏΠΎΡΠΎΠΊΠΎΠΌ?
Π ΡΠ΅ΠΌ ΡΠΌΡΡΠ» ΠΏΡΠ°Π²ΠΈΠ»Π° ΠΠ΅Π½ΡΠ°?
ΠΠΠ‘ ΠΠΠΠ£ΠΠ¦ΠΠ Π ΠΠΠΠΠ£Π©ΠΠ₯Π‘Π― ΠΠ ΠΠΠΠΠΠΠΠΠ₯.
ΠΠΠ‘ ΠΈΠ½Π΄ΡΠΊΡΠΈΠΈ Π² Π΄Π²ΠΈΠΆΡΡΠΈΡ
ΡΡ ΠΏΡΠΎΠ²ΠΎΠ΄Π½ΠΈΠΊΠ°Ρ
.
Π‘ΠΈΠ»Π° ΠΠΎΡΠ΅Π½ΡΠ°:
Π Π°Π±ΠΎΡΠ° ΡΠΈΠ»Ρ (Π²Π΄ΠΎΠ»Ρ ΠΏΡΠΎΠ²ΠΎΠ΄Π½ΠΈΠΊΠ°):
ΠΠΎ ΠΎΠΏΡΠ΅Π΄Π΅Π»Π΅Π½ΠΈΡ: ΡΠ°Π±ΠΎΡΠ° ΡΠΈΠ»Ρ ΠΏΠΎ ΠΏΠ΅ΡΠ΅ΠΌΠ΅ΡΠ΅Π½ΠΈΡ Π·Π°ΡΡΠ΄Π° β Π΅ΡΡΡ ΠΠΠ‘:
1
ΠΡΠ²ΠΎΠ΄ ΡΠΎΡΠΌΡΠ»Ρ
Π‘ Π΄ΡΡΠ³ΠΎΠΉ ΡΡΠΎΡΠΎΠ½Ρ:
ΠΠ°Π³Π½ΠΈΡΠ½ΡΠΉ ΠΏΠΎΡΠΎΠΊ:
ΠΠ·ΠΌΠ΅Π½Π΅Π½ΠΈΠ΅ ΠΌΠ°Π³Π½ΠΈΡΠ½ΠΎΠ³ΠΎ ΠΏΠΎΡΠΎΠΊΠ° ΠΌΠΎΠΆΠ΅Ρ ΠΏΡΠΎΠΈΠ·ΠΎΠΉΡΠΈ ΡΠΎΠ»ΡΠΊΠΎ Π·Π° ΡΡΠ΅Ρ ΡΠΌΠ΅Π½ΡΡΠ΅Π½ΠΈΡ ΠΏΠ»ΠΎΡΠ°Π΄ΠΈ ΠΊΠΎΠ½ΡΡΡΠ°:
ΠΠ·ΠΌΠ΅Π½Π΅Π½ΠΈΠ΅ ΠΌΠ°Π³Π½ΠΈΡΠ½ΠΎΠ³ΠΎ ΠΏΠΎΡΠΎΠΊΠ°:
ΠΠΎ Π·Π°ΠΊΠΎΠ½Ρ ΠΠΠ:
2
ΠΡΠ²ΠΎΠ΄ ΡΠΎΡΠΌΡΠ»Ρ
ΠΡΠ²ΠΎΠ΄Ρ (ΠΊΠ°ΠΊΠΈΠ΅ Π·Π°ΠΊΠΎΠ½ΠΎΠΌΠ΅ΡΠ½ΠΎΡΡΠΈ Π·Π°ΠΌΠ΅ΡΠ΅Π½Ρ?):
ΠΠ²ΡΠΌΡ ΡΠ°Π·Π»ΠΈΡΠ½ΡΠΌΠΈ ΡΠΏΠΎΡΠΎΠ±Π°ΠΌΠΈ ΠΏΠΎΠ»ΡΡΠ΅Π½Π° ΠΎΠ΄Π½Π° ΠΈ ΡΠ° ΠΆΠ΅ ΡΠΎΡΠΌΡΠ»Π°:
ΠΠΎ ΠΎΠΏΡΠ΅Π΄Π΅Π»Π΅Π½ΠΈΡ ΡΠΈΠ»Ρ ΠΠΎΡΠ΅Π½ΡΠ°,
Π ΠΏΠΎ Π·Π°ΠΊΠΎΠ½Ρ ΡΠ»Π΅ΠΊΡΡΠΎΠΌΠ°Π³Π½ΠΈΡΠ½ΠΎΠΉ ΠΈΠ½Π΄ΡΠΊΡΠΈΠΈ.
ΠΠΠ‘ ΠΈΠ½Π΄ΡΠΊΡΠΈΠΈ Π² Π΄Π²ΠΈΠΆΡΡΠ΅ΠΌΡΡ ΠΏΡΠΎΠ²ΠΎΠ΄Π½ΠΈΠΊΠ΅ ΡΠΈΡΠ»Π΅Π½Π½ΠΎ ΡΠΎΠ²ΠΏΠ°Π΄Π°Π΅Ρ Ρ ΡΠΈΠ»ΠΎΠΉ ΠΠΌΠΏΠ΅ΡΠ°.
ΠΠ½Π°Π»ΠΎΠ³ΠΈΡΠ½ΡΠ΅ Π΄Π΅ΠΉΡΡΠ²ΠΈΡ Π±ΡΠ»ΠΈ ΠΏΡΠΎΠ²Π΅Π΄Π΅Π½Ρ ΡΠ°Π½Π΅Π΅ ΠΏΡΠΈ ΡΠ΅ΡΠ΅Π½ΠΈΠΈ Π·Π°Π΄Π°Ρ.
Π‘ΠΠΠΠΠΠΠ£ΠΠ¦ΠΠ―.
Π―Π²Π»Π΅Π½ΠΈΠ΅ ΡΠ°ΠΌΠΎΠΈΠ½Π΄ΡΠΊΡΠΈΠΈ:
ΠΡΠΈ Π·Π°ΠΌΡΠΊΠ°Π½ΠΈΠΈ ΡΠ΅ΠΏΠΈ ΠΎΠΏΡΠ΅Π΄Π΅Π»Π΅Π½Π½ΠΎΠ΅ Π·Π½Π°ΡΠ΅Π½ΠΈΠ΅ ΡΠΈΠ»Ρ ΡΠΎΠΊΠ° ΡΠΎΠ·Π΄Π°Π΅ΡΡΡ Π½Π΅ ΡΡΠ°Π·Ρ, Π° ΠΏΠΎΡΡΠ΅ΠΏΠ΅Π½Π½ΠΎ Ρ ΡΠ΅ΡΠ΅Π½ΠΈΠ΅ΠΌ Π²ΡΠ΅ΠΌΠ΅Π½ΠΈ.
ΠΡΠΈ ΡΠ°ΠΌΠΎΠΈΠ½Π΄ΡΠΊΡΠΈΠΈ ΠΏΡΠΎΠ²ΠΎΠ΄ΡΡΠΈΠΉ ΠΊΠΎΠ½ΡΡΡ ΠΈΠ³ΡΠ°Π΅Ρ Π΄Π²ΠΎΡΠΊΡΡ ΡΠΎΠ»Ρ: ΠΏΠΎ Π½Π΅ΠΌΡ ΠΏΡΠΎΡΠ΅ΠΊΠ°Π΅Ρ ΡΠΎΠΊ, Π²ΡΠ·ΡΠ²Π°ΡΡΠΈΠΉ ΠΈΠ½Π΄ΡΠΊΡΠΈΡ, ΠΈ Π² Π½Π΅ΠΌ ΠΆΠ΅ ΠΏΠΎΡΠ²Π»ΡΠ΅ΡΡΡ ΠΠΠ‘ ΠΈΠ½Π΄ΡΠΊΡΠΈΠΈ.
ΠΠΎ ΠΏΡΠ°Π²ΠΈΠ»Ρ ΠΠ΅Π½ΡΠ°: Π² ΠΌΠΎΠΌΠ΅Π½Ρ Π½Π°ΡΠ°ΡΡΠ°Π½ΠΈΡ ΡΠΎΠΊΠ° Π² ΠΏΡΠΎΠ²ΠΎΠ΄Π½ΠΈΠΊΠ΅, Π² Π½Π΅ΠΌ ΠΆΠ΅ ΡΠΎΠ·Π΄Π°Π΅ΡΡΡ ΠΈΠ½Π΄ΡΠΊΡΠΈΠΎΠ½Π½ΡΠΉ ΡΠΎΠΊ (Ρ.Π΅. ΡΠΎΠ·Π΄Π°Π΅ΡΡΡ Π²ΠΈΡ
ΡΠ΅Π²ΠΎΠ΅ ΡΠ»Π΅ΠΊΡΡΠΈΡΠ΅ΡΠΊΠΎΠ΅ ΠΏΠΎΠ»Π΅), ΠΏΡΠ΅ΠΏΡΡΡΡΠ²ΡΡΡΠΈΠΉ ΡΡΠΎΠΌΡ Π½Π°ΡΠ°ΡΡΠ°Π½ΠΈΡ ΡΠΎΠΊΠ°.
ΠΡΠΈ ΡΠ±ΡΠ²Π°Π½ΠΈΠΈ ΡΠ»Π΅ΠΊΡΡΠΈΡΠ΅ΡΠΊΠΎΠ³ΠΎ ΡΠΎΠΊΠ° Π²ΠΈΡ
ΡΠ΅Π²ΠΎΠ΅ ΠΏΠΎΠ»Π΅ ΡΡΠΎΠΌΡ ΠΏΡΠ΅ΠΏΡΡΡΡΠ²ΡΠ΅Ρ.
ΠΠ°Π±Π»ΡΠ΄Π΅Π½ΠΈΠ΅ ΡΠ²Π»Π΅Π½ΠΈΡ ΡΠ°ΠΌΠΎΠΈΠ½Π΄ΡΠΊΡΠΈΠΈ Π½Π° ΠΎΠΏΡΡΠ°Ρ
:
ΠΡΠΈ Π·Π°ΠΌΡΠΊΠ°Π½ΠΈΠΈ ΠΊΠ»ΡΡΠ° ΠΏΠ΅ΡΠ²Π°Ρ Π»Π°ΠΌΠΏΠ° Π²ΡΠΏΡΡ
ΠΈΠ²Π°Π΅Ρ ΠΏΡΠ°ΠΊΡΠΈΡΠ΅ΡΠΊΠΈ ΡΡΠ°Π·Ρ, Π° Π²ΡΠΎΡΠ°Ρ β Ρ Π·Π°ΠΌΠ΅ΡΠ½ΡΠΌ ΠΎΠΏΠΎΠ·Π΄Π°Π½ΠΈΠ΅ΠΌ. ΠΠΎΠ·Π½ΠΈΠΊΠ°ΡΡΠ°Ρ ΠΠΠ‘ ΡΠ°ΠΌΠΎΠΈΠ½Π΄ΡΠΊΡΠΈΠΈ Π² ΠΊΠ°ΡΡΡΠΊΠ΅ Β«ΡΠΎΡΠΌΠΎΠ·ΠΈΡΒ» Π½Π°ΡΠ°ΡΡΠ°Π½ΠΈΠ΅ ΡΠΎΠΊΠ° Π² Π²Π΅ΡΠ²ΠΈ.
ΠΡΠΈ ΡΠ°Π·ΠΌΡΠΊΠ°Π½ΠΈΠΈ ΠΊΠ»ΡΡΠ° Π² ΠΊΠ°ΡΡΡΠΊΠ΅ Π²ΠΎΠ·Π½ΠΈΠΊΠ°Π΅Ρ ΠΠΠ‘ ΡΠ°ΠΌΠΎΠΈΠ½Π΄ΡΠΊΡΠΈΠΈ, ΠΏΠΎΠ΄Π΄Π΅ΡΠΆΠΈΠ²Π°ΡΡΠ°Ρ ΠΏΠ΅ΡΠ²ΠΎΠ½Π°ΡΠ°Π»ΡΠ½ΡΠΉ ΡΠΎΠΊ, ΠΏΠΎΡΡΠΎΠΌΡ Π»Π°ΠΌΠΏΠ° Π³Π°ΡΠ½Π΅Ρ Π½Π΅ ΡΡΠ°Π·Ρ ΠΏΠΎΡΠ»Π΅ ΡΠ°Π·ΠΌΡΠΊΠ°Π½ΠΈΡ ΠΊΠ»ΡΡΠ°.
1-ΠΉ ΠΎΠΏΡΡ ΠΠ΅Π½ΡΠΈ
Π’ΠΎΠΊ Π·Π°ΠΌΡΠΊΠ°Π½ΠΈΡ.
Π ΠΊΠ°ΡΡΡΠΊΠ΅ Π½Π°ΡΠ°ΡΡΠ°Π΅Ρ ΠΌΠ°Π³Π½ΠΈΡΠ½ΡΠΉ ΠΏΠΎΡΠΎΠΊ ΠΈ ΡΠ²Π»Π΅Π½ΠΈΠ΅ ΡΠ°ΠΌΠΎΠΈΠ½Π΄ΡΠΊΡΠΈΠΈ Β«ΡΠΎΡΠΌΠΎΠ·ΠΈΡΒ» Π½Π°ΡΠ°ΡΡΠ°Π½ΠΈΠ΅ ΡΠΎΠΊΠ° Π² ΡΠ΅ΠΏΠΈ.
2-ΠΉ ΠΎΠΏΡΡ ΠΠ΅Π½ΡΠΈ
Π’ΠΎΠΊ ΡΠ°Π·ΠΌΡΠΊΠ°Π½ΠΈΡ.
ΠΠ°Π³Π½ΠΈΡΠ½ΡΠΉ ΠΏΠΎΡΠΎΠΊ ΡΠ±ΡΠ²Π°Π΅Ρ; ΡΠ²Π»Π΅Π½ΠΈΠ΅ ΡΠ°ΠΌΠΎΠΈΠ½Π΄ΡΠΊΡΠΈΠΈ Π½Π΅ΠΊΠΎΡΠΎΡΠΎΠ΅ Π²ΡΠ΅ΠΌΡ ΠΏΠΎΠ΄Π΄Π΅ΡΠΆΠΈΠ²Π°Π΅Ρ ΡΠΎΠΊ Π² ΡΠ΅ΠΏΠΈ.
ΠΠ½Π°Π»ΠΎΠ³ΠΈΡ ΠΌΠ΅ΠΆΠ΄Ρ ΡΠ°ΠΌΠΎΠΈΠ½Π΄ΡΠΊΡΠΈΠ΅ΠΉ ΠΈ ΠΈΠ½Π΅ΡΡΠΈΠ΅ΠΉ:
ΠΠ°ΠΆΠ΄Π°Ρ ΠΊΠ°ΡΡΡΠΊΠ° ΠΈΠ½Π΄ΠΈΠ²ΠΈΠ΄ΡΠ°Π»ΡΠ½Π°, ΠΈΠΌΠ΅Ρ ΡΠ°Π·Π½ΠΎΠ΅ ΡΠΈΡΠ»ΠΎ Π²ΠΈΡΠΊΠΎΠ² ΠΈ ΡΠ°Π·Π½ΡΠΉ Π΄ΠΈΠ°ΠΌΠ΅ΡΡ Π±ΠΎΠ±ΠΈΠ½Ρ.
Π§ΡΠΎ ΠΏΡΠΎΠΈΠ·ΠΎΠΉΠ΄Π΅Ρ, Π΅ΡΠ»ΠΈ Π² ΠΏΠ΅ΡΠ²ΠΎΠΌ ΡΠΊΡΠΏΠ΅ΡΠΈΠΌΠ΅Π½ΡΠ΅ Π·Π°ΠΌΠ΅Π½ΠΈΡΡ ΠΊΠ°ΡΡΡΠΊΡ Π½Π° Π΄ΡΡΠ³ΡΡ, Ρ Π±ΠΎΠ»ΡΡΠΈΠΌ ΡΠΈΡΠ»ΠΎΠΌ Π²ΠΈΡΠΊΠΎΠ²?
ΠΠ°ΠΊΠΎΠΉ ΠΈΠ· Π°Π²ΡΠΎΠΌΠΎΠ±ΠΈΠ»Π΅ΠΉ Π»Π΅Π³ΡΠ΅ Β«ΡΠ°ΡΡΠΎΠ»ΠΊΠ°ΡΡΒ»?
Π ΠΎΡΡΠ°Π½ΠΎΠ²ΠΈΡΡ?
ΠΠ°ΠΊ Π½Π°Π·ΡΠ²Π°Π΅ΡΡΡ ΡΠ²Π»Π΅Π½ΠΈΠ΅ ΡΡΡΠ΅ΠΌΠ»Π΅Π½ΠΈΡ ΡΠ΅Π»Π° ΡΠΎΡ
ΡΠ°Π½ΠΈΡΡ ΡΠ²ΠΎΡ ΡΠΊΠΎΡΠΎΡΡΡ Π² ΠΌΠ΅Ρ
Π°Π½ΠΈΠΊΠ΅?
Π‘Π°ΠΌΠΎΠΈΠ½Π΄ΡΠΊΡΠΈΡ Π°Π½Π°Π»ΠΎΠ³ΠΈΡΠ½Π° ΠΈΠ½Π΅ΡΡΠΈΠΈ: Π±ΠΎΠ»Π΅Π΅ Β«ΠΌΠΎΡΠ½Π°ΡΒ» ΠΊΠ°ΡΡΡΠΊΠ° ΡΠΈΠ»ΡΠ½Π΅Π΅ ΠΏΡΠ΅ΠΏΡΡΡΡΠ²ΡΠ΅Ρ Π½Π°ΡΠ°ΡΡΠ°Π½ΠΈΡ ΡΠΎΠΊΠ°; ΠΎΠ½Π° ΠΆΠ΅ ΠΏΠΎΡΠΎΠΌ, ΠΏΡΠΈ ΡΠ°Π·ΠΌΡΠΊΠ°Π½ΠΈΠΈ, Π΄ΠΎΠ»ΡΡΠ΅ Π΅Π³ΠΎ ΠΏΠΎΠ΄Π΄Π΅ΡΠΆΠΈΠ²Π°Π΅Ρ.
ΠΠ½Π΄ΡΠΊΡΠΈΠ²Π½ΠΎΡΡΡ.
ΠΡΠΎΡΠ»Π΅Π΄ΠΈΠΌ ΠΏΠΎΡΠ»Π΅Π΄ΠΎΠ²Π°ΡΠ΅Π»ΡΠ½ΠΎΡΡΡ: Π€~B~I.
ΠΠΎΠΆΠ½ΠΎ ΡΡΠ²Π΅ΡΠΆΠ΄Π°ΡΡ, ΡΡΠΎ ΠΌΠ°Π³Π½ΠΈΡΠ½ΡΠΉ ΠΏΠΎΡΠΎΠΊ ΠΏΡΠΎΠΏΠΎΡΡΠΈΠΎΠ½Π°Π»Π΅Π½ ΡΠΈΠ»Π΅ ΡΠΎΠΊΠ°: Π€=LI, Π³Π΄Π΅ L β ΠΊΠΎΡΡΡΠΈΡΠΈΠ΅Π½Ρ ΠΏΡΠΎΠΏΠΎΡΡΠΈΠΎΠ½Π°Π»ΡΠ½ΠΎΡΡΠΈ; ΠΎΠ½ ΠΆΠ΅ β ΠΈΠ½Π΄ΡΠΊΡΠΈΠ²Π½ΠΎΡΡΡ ΠΊΠΎΠ½ΡΡΡΠ°; ΠΎΠ½ ΠΆΠ΅ β ΠΊΠΎΡΡΡΠΈΡΠΈΠ΅Π½Ρ ΡΠ°ΠΌΠΎΠΈΠ½Π΄ΡΠΊΡΠΈΠΈ.
ΠΠ½Π΄ΡΠΊΡΠΈΠ²Π½ΠΎΡΡΡ β ΡΡΠΎ ΡΠΈΠ·ΠΈΡΠ΅ΡΠΊΠ°Ρ Π²Π΅Π»ΠΈΡΠΈΠ½Π°, ΡΠΈΡΠ»Π΅Π½Π½ΠΎ ΡΠ°Π²Π½Π°Ρ ΠΠΠ‘ ΡΠ°ΠΌΠΎΠΈΠ½Π΄ΡΠΊΡΠΈΠΈ, Π²ΠΎΠ·Π½ΠΈΠΊΠ°ΡΡΠ΅ΠΉ Π² ΠΊΠΎΠ½ΡΡΡΠ΅ ΠΏΡΠΈ ΠΈΠ·ΠΌΠ΅Π½Π΅Π½ΠΈΠΈ ΡΠΈΠ»Ρ ΡΠΎΠΊΠ° Π½Π° 1 Π Π·Π° 1 Ρ.
ΠΠ΄ΠΈΠ½ΠΈΡΡ ΠΈΠ·ΠΌΠ΅ΡΠ΅Π½ΠΈΡ ΠΈΠ½Π΄ΡΠΊΡΠΈΠ²Π½ΠΎΡΡΠΈ β 1 ΠΠ΅Π½ΡΠΈ:
ΠΠ½Π°Π»ΠΎΠ³ΠΈΡ ΠΌΠ΅ΠΆΠ΄Ρ ΠΌΠ΅Ρ
Π°Π½ΠΈΠΊΠΎΠΉ ΠΈ ΡΠ»Π΅ΠΊΡΡΠΎΠ΄ΠΈΠ½Π°ΠΌΠΈΠΊΠΎΠΉ (Π½Π°ΡΠ°Π»ΠΎ ΡΠ°Π±Π»ΠΈΡΡ; ΠΏΡΠΎΠ΄ΠΎΠ»ΠΆΠ΅Π½ΠΈΠ΅ Π½Π° ΡΠ»Π΅Π΄ΡΡΡΠ΅ΠΌ ΡΡΠΎΠΊΠ΅)
ΠΠ°Π΄Π°ΡΠ° 14:
ΠΠ°ΠΊΠ°Ρ ΠΠΠ‘ ΡΠ°ΠΌΠΎΠΈΠ½Π΄ΡΠΊΡΠΈΠΈ Π²ΠΎΠ·Π½ΠΈΠΊΠ°Π΅Ρ Π² ΠΊΠ°ΡΡΡΠΊΠ΅, Π΅ΡΠ»ΠΈ ΡΠΈΠ»Π° ΡΠΎΠΊΠ° Π² ΠΊΠ°ΡΡΡΠΊΠ΅ ΠΈΠ½Π΄ΡΠΊΡΠΈΠ²Π½ΠΎΡΡΡΡ 1 ΠΠ½ ΠΈΠ·ΠΌΠ΅Π½ΡΠ΅ΡΡΡ Ρ ΡΠ΅ΡΠ΅Π½ΠΈΠ΅ΠΌ Π²ΡΠ΅ΠΌΠ΅Π½ΠΈ, ΠΊΠ°ΠΊ ΠΏΠΎΠΊΠ°Π·Π°Π½ΠΎ Π½Π° Π³ΡΠ°ΡΠΈΠΊΠ΅.
ΠΡΠΈΠΌΠ΅Π½ΡΠ΅ΡΡΡ Π·Π°ΠΊΠΎΠ½ ΠΠΠ‘ ΡΠ°ΠΌΠΎΠΈΠ½Π΄ΡΠΊΡΠΈΠΈ.
ΠΠΎ Π³ΡΠ°ΡΠΈΠΊΡ ΠΎΠΏΡΠ΅Π΄Π΅Π»ΡΠ΅ΡΡΡ ΡΠΈΠ»Π° ΡΠΎΠΊΠ° ΠΈ Π²ΡΠ΅ΠΌΡ Π΅Π³ΠΎ ΠΏΡΠΎΡΠ΅ΠΊΠ°Π½ΠΈΡ
ΠΡΠ²Π΅Ρ:10 Π
ΠΠ°Π΄Π°ΡΠ° 15:
ΠΠ° ΡΠΈΡΡΠ½ΠΊΠ΅ ΠΏΡΠΈΠ²Π΅Π΄Π΅Π½ Π³ΡΠ°ΡΠΈΠΊ Π·Π°Π²ΠΈΡΠΈΠΌΠΎΡΡΠΈ ΡΠΈΠ»Ρ ΡΠΎΠΊΠ° Π² ΠΊΠ°ΡΡΡΠΊΠ΅ ΠΈΠ½Π΄ΡΠΊΡΠΈΠ²Π½ΠΎΡΡΠΈ ΠΎΡ Π²ΡΠ΅ΠΌΠ΅Π½ΠΈ. Π ΠΊΠ°ΠΊΠΎΠΌ ΠΏΡΠΎΠΌΠ΅ΠΆΡΡΠΊΠ΅ ΠΠΠ‘ ΡΠ°ΠΌΠΎΠΈΠ½Π΄ΡΠΊΡΠΈΠΈ ΠΏΡΠΈΠ½ΠΈΠΌΠ°Π΅Ρ Π½Π°ΠΈΠΌΠ΅Π½ΡΡΠ΅Π΅ Π·Π½Π°ΡΠ΅Π½ΠΈΠ΅?
Π§Π΅ΠΌ ΠΌΠ΅Π½ΡΡΠ΅ ΡΠΊΠΎΡΠΎΡΡΡ ΠΈΠ·ΠΌΠ΅Π½Π΅Π½ΠΈΡ ΡΠΎΠΊΠ°, ΡΠ΅ΠΌ ΠΌΠ΅Π½ΡΡΠ΅ ΠΠΠ‘ ΡΠ°ΠΌΠΎΠΈΠ½Π΄ΡΠΊΡΠΈΠΈ.
Π‘Π°ΠΌΠ°Ρ ΠΌΠ°Π»Π΅Π½ΡΠΊΠ°Ρ ΡΠΊΠΎΡΠΎΡΡΡ ΠΈΠ·ΠΌΠ΅Π½Π΅Π½ΠΈΡ ΡΠΎΠΊΠ° Π·Π° ΠΏΡΠΎΠΌΠ΅ΠΆΡΡΠΎΠΊ Π²ΡΠ΅ΠΌΠ΅Π½ΠΈ ΠΎΡ 1 Π΄ΠΎ 5 Ρ (Π½Π°ΠΈΠΌΠ΅Π½ΡΡΠΈΠΉ ΡΠ³ΠΎΠ» Π½Π°ΠΊΠ»ΠΎΠ½Π° ΡΡΠ°ΡΡΠΊΠ° Π³ΡΠ°ΡΠΈΠΊΠ°)
ΠΠ°Π΄Π°ΡΠ° 16:
1) 0-1 Ρ ΠΈ 2-3 Ρ
2) 1-2 Ρ ΠΈ 2-3 Ρ
3) 0-1 Ρ ΠΈ 3-4 Ρ
4) 2-3 Ρ ΠΈ 3-4-Ρ
ΠΡΠ²Π΅Ρ: 2), Ρ.ΠΊ. ΠΈΠΌΠ΅Π½Π½ΠΎ Π² ΡΡΠΎΠΌ ΠΏΡΠΎΠΌΠ΅ΠΆΡΡΠΊΠ΅ Π²ΡΠ΅ΠΌΠ΅Π½ΠΈ ΡΠΈΠ»Π° ΡΠΎΠΊΠ° ΠΈΠ·ΠΌΠ΅Π½ΡΠ΅ΡΡΡ ΠΌΠ΅Π΄Π»Π΅Π½Π½ΠΎ.
ΠΠ° ΡΠΈΡΡΠ½ΠΊΠ΅ ΠΏΠΎΠΊΠ°Π·Π°Π½ Π³ΡΠ°ΡΠΈΠΊ ΠΈΠ·ΠΌΠ΅Π½Π΅Π½ΠΈΡ ΡΠΈΠ»Ρ ΡΠΎΠΊΠ° Π² ΠΊΠ°ΡΡΡΠΊΠ΅ ΠΈΠ½Π΄ΡΠΊΡΠΈΠ²Π½ΠΎΡΡΠΈ Ρ ΡΠ΅ΡΠ΅Π½ΠΈΠ΅ΠΌ Π²ΡΠ΅ΠΌΠ΅Π½ΠΈ. Π ΠΊΠ°ΠΊΠΎΠΌ ΠΏΡΠΎΠΌΠ΅ΠΆΡΡΠΊΠ΅ Π²ΡΠ΅ΠΌΠ΅Π½ΠΈ ΠΌΠΎΠ΄ΡΠ»Ρ ΠΠΠ‘ ΡΠ°ΠΌΠΎΠΈΠ½Π΄ΡΠΊΡΠΈΠΈ ΠΏΡΠΈΠ½ΠΈΠΌΠ°Π΅Ρ Π½Π°ΠΈΠΌΠ΅Π½ΡΡΠΈΠ΅ Π·Π½Π°ΡΠ΅Π½ΠΈΡ?
ΠΠ°Π΄Π°ΡΠ° 17:
ΠΠ° ΡΠΈΡΡΠ½ΠΊΠ΅ ΠΏΡΠ΅Π΄ΡΡΠ°Π²Π»Π΅Π½ Π³ΡΠ°ΡΠΈΠΊ ΠΈΠ·ΠΌΠ΅Π½Π΅Π½ΠΈΡ ΡΠΈΠ»Ρ ΡΠΎΠΊΠ° Ρ ΡΠ΅ΡΠ΅Π½ΠΈΠ΅ΠΌ Π²ΡΠ΅ΠΌΠ΅Π½ΠΈ Π² ΠΊΠ°ΡΡΡΠΊΠ΅ ΠΈΠ½Π΄ΡΠΊΡΠΈΠ²Π½ΠΎΡΡΡΡ 6 ΠΌΠΠ½. Π§Π΅ΠΌΡ ΡΠ°Π²Π½Π° ΠΠΠ‘ ΡΠ°ΠΌΠΎΠΈΠ½Π΄ΡΠΊΡΠΈΠΈ?
ΠΡΠΏΠΎΠ»ΡΠ·ΡΠ΅ΡΡΡ Π·Π°ΠΊΠΎΠ½ ΠΠΠ‘ ΡΠ°ΠΌΠΎΠΈΠ½Π΄ΡΠΊΡΠΈΠΈ.
ΠΡΠ±ΠΈΡΠ°Π΅ΡΡΡ ΠΈΠ·ΠΌΠ΅Π½Π΅Π½ΠΈΠ΅ ΡΠΈΠ»Ρ ΡΠΎΠΊΠ° ΠΈ ΡΠΎΠΎΡΠ²Π΅ΡΡΡΠ²ΡΡΡΠΈΠΉ Π΅ΠΌΡ ΠΏΡΠΎΠΌΠ΅ΠΆΡΡΠΎΠΊ Π²ΡΠ΅ΠΌΠ΅Π½ΠΈ:
6β10-3 ΠΠ½ β 3 Π/2 Ρ
ΠΡΠ²Π΅Ρ: 9 ΠΌΠ
ΠΠ°Π΄Π°ΡΠΈ 18, 19:
Π ΠΏΡΠΎΠ²ΠΎΠ΄Π½ΠΈΠΊΠ΅ ΠΈΠ½Π΄ΡΠΊΡΠΈΠ²Π½ΠΎΡΡΡΡ 5 ΠΌΠΠ½ ΡΠΈΠ»Π° ΡΠΎΠΊΠ° Π² ΡΠ΅ΡΠ΅Π½ΠΈΠ΅ 0,2 Ρ ΡΠ°Π²Π½ΠΎΠΌΠ΅ΡΠ½ΠΎ Π²ΠΎΠ·ΡΠ°ΡΡΠ°Π΅Ρ Ρ 2 Π Π΄ΠΎ ΠΊΠ°ΠΊΠΎΠ³ΠΎ-ΡΠΎ ΠΊΠΎΠ½Π΅ΡΠ½ΠΎΠ³ΠΎ Π·Π½Π°ΡΠ΅Π½ΠΈΡ. ΠΡΠΈ ΡΡΠΎΠΌ Π² ΠΏΡΠΎΠ²ΠΎΠ΄Π½ΠΈΠΊΠ΅ Π²ΠΎΠ·Π½ΠΈΠΊΠ°Π΅Ρ ΠΠΠ‘ ΡΠ°ΠΌΠΎΠΈΠ½Π΄ΡΠΊΡΠΈΠΈ 0,2 Π. ΠΠΏΡΠ΅Π΄Π΅Π»ΠΈΡΠ΅ ΠΊΠΎΠ½Π΅ΡΠ½ΠΎΠ΅ Π·Π½Π°ΡΠ΅Π½ΠΈΠ΅ ΡΠΈΠ»Ρ ΡΠΎΠΊΠ° Π² ΠΏΡΠΎΠ²ΠΎΠ΄Π½ΠΈΠΊΠ΅.
ΠΡΠ²Π΅Ρ: 10 Π
ΠΠ½Π΄ΡΠΊΡΠΈΠ²Π½ΠΎΡΡΡ ΠΊΠ°ΡΡΡΠΊΠΈ ΡΠ²Π΅Π»ΠΈΡΠΈΠ»ΠΈ Π² 2 ΡΠ°Π·Π°, Π° ΡΠΊΠΎΡΠΎΡΡΡ ΠΈΠ·ΠΌΠ΅Π½Π΅Π½ΠΈΡ ΡΠΎΠΊΠ° ΡΠΌΠ΅Π½ΡΡΠΈΠ»ΠΈ Π² 4 ΡΠ°Π·Π°. ΠΠ°ΠΊ ΠΈΠ·ΠΌΠ΅Π½ΠΈΠ»Π°ΡΡ ΠΠΠ‘ ΡΠ°ΠΌΠΎΠΈΠ½Π΄ΡΠΊΡΠΈΠΈ?
ΠΡΠ²Π΅Ρ: ΠΠΠ‘ ΡΠ°ΠΌΠΎΠΈΠ½Π΄ΡΠΊΡΠΈΠΈ ΡΠΌΠ΅Π½ΡΡΠΈΠ»Π°ΡΡ Π² 2 ΡΠ°Π·Π°.
ΠΠ°Π΄Π°ΡΠ° 20
ΠΡΡΠΌΠΎΠ»ΠΈΠ½Π΅ΠΉΠ½ΡΠΉ ΠΏΡΠΎΠ²ΠΎΠ΄Π½ΠΈΠΊ Π΄Π²ΠΈΠΆΠ΅ΡΡΡ ΡΠΎ ΡΠΊΠΎΡΠΎΡΡΡΡ 25 ΠΌ/Ρ Π² ΠΏΠΎΠ»Π΅ Ρ ΠΈΠ½Π΄ΡΠΊΡΠΈΠ΅ΠΉ 0,0038 Π’Π» ΠΏΠ΅ΡΠΏΠ΅Π½Π΄ΠΈΠΊΡΠ»ΡΡΠ½ΠΎ ΡΠΈΠ»ΠΎΠ²ΡΠΌ Π»ΠΈΠ½ΠΈΡΠΌ. Π§Π΅ΠΌΡ ΡΠ°Π²Π½Π° Π΄Π»ΠΈΠ½Π° ΠΏΡΠΎΠ²ΠΎΠ΄Π½ΠΈΠΊΠ°, Π΅ΡΠ»ΠΈ Π½Π° Π΅Π³ΠΎ ΠΊΠΎΠ½ΡΠ°Ρ
ΠΈΠΌΠ΅Π΅ΡΡΡ Π½Π°ΠΏΡΡΠΆΠ΅Π½ΠΈΠ΅ 0,028 Π?
ΠΡΠ²Π΅Ρ: 29 ΡΠΌ
ΠΠ°Π΄Π°ΡΠ° 21
ΠΠΈΡΠΎΠΊ ΠΏΠ»ΠΎΡΠ°Π΄ΡΡ 100 ΡΠΌ2 Π½Π°Ρ
ΠΎΠ΄ΠΈΡΡΡ Π² ΠΌΠ°Π³Π½ΠΈΡΠ½ΠΎΠΌ ΠΏΠΎΠ»Π΅ Ρ ΠΈΠ½Π΄ΡΠΊΡΠΈΠ΅ΠΉ 1 Π’Π». ΠΠ»ΠΎΡΠΊΠΎΡΡΡ Π²ΠΈΡΠΊΠ° ΠΏΠ΅ΡΠΏΠ΅Π½Π΄ΠΈΠΊΡΠ»ΡΡΠ½Π° Π»ΠΈΠ½ΠΈΡΠΌ ΠΏΠΎΠ»Ρ. ΠΠΏΡΠ΅Π΄Π΅Π»ΠΈΡΠ΅ ΡΡΠ΅Π΄Π½Π΅Π΅ Π·Π½Π°ΡΠ΅Π½ΠΈΠ΅ ΠΠΠ‘ ΠΈΠ½Π΄ΡΠΊΡΠΈΠΈ ΠΏΡΠΈ Π²ΡΠΊΠ»ΡΡΠ΅Π½ΠΈΠΈ ΠΏΠΎΠ»Ρ Π·Π° 0,01 Ρ.
ΠΡΠ²Π΅Ρ: 1 Π
ΠΠ°Π΄Π°ΡΠ° 22
ΠΡΡΠΌΠΎΠ»ΠΈΠ½Π΅ΠΉΠ½ΡΠΉ ΠΏΡΠΎΠ²ΠΎΠ΄Π½ΠΈΠΊ Π΄Π»ΠΈΠ½ΠΎΠΉ 120 ΡΠΌ Π΄Π²ΠΈΠΆΠ΅ΡΡΡ Π² ΠΎΠ΄Π½ΠΎΡΠΎΠ΄Π½ΠΎΠΌ ΠΌΠ°Π³Π½ΠΈΡΠ½ΠΎΠΌ ΠΏΠΎΠ»Π΅ ΠΏΠΎΠ΄ ΡΠ³Π»ΠΎΠΌ 90Β° ΠΊ ΡΠΈΠ»ΠΎΠ²ΡΠΌ Π»ΠΈΠ½ΠΈΡΠΌ ΡΠΎ ΡΠΊΠΎΡΠΎΡΡΡΡ 15 ΠΌ/Ρ. ΠΠΏΡΠ΅Π΄Π΅Π»ΠΈΡΠ΅ ΠΈΠ½Π΄ΡΠΊΡΠΈΡ ΠΏΠΎΠ»Ρ, Π΅ΡΠ»ΠΈ Π² ΠΏΡΠΎΠ²ΠΎΠ΄Π½ΠΈΠΊΠ΅ ΡΠΎΠ·Π΄Π°Π΅ΡΡΡ ΠΠΠ‘ ΠΈΠ½Π΄ΡΠΊΡΠΈΠΈ 0,12 Π.
ΠΡΠ²Π΅Ρ: 6,67 ΠΌΠ’Π»
ΠΠ°Π΄Π°ΡΠ° 23
ΠΠ°ΠΉΠ΄ΠΈΡΠ΅ ΠΈΠ½Π΄ΡΠΊΡΠΈΠ²Π½ΠΎΡΡΡ ΠΏΡΠΎΠ²ΠΎΠ΄Π½ΠΈΠΊΠ°, Π² ΠΊΠΎΡΠΎΡΠΎΠΌ ΡΠ°Π²Π½ΠΎΠΌΠ΅ΡΠ½ΠΎΠ΅ ΠΈΠ·ΠΌΠ΅Π½Π΅Π½ΠΈΠ΅ ΡΠΈΠ»Ρ ΡΠΎΠΊΠ° Π½Π° 2 Π Π² ΡΠ΅ΡΠ΅Π½ΠΈΠ΅ 0,25 Ρ Π²ΠΎΠ·Π±ΡΠΆΠ΄Π°Π΅Ρ ΠΠΠ‘ ΡΠ°ΠΌΠΎΠΈΠ½Π΄ΡΠΊΡΠΈΠΈ 20 ΠΌΠ.
ΠΡΠ²Π΅Ρ: 2,5 ΠΌΠΠ½
ΠΠ°Π΄Π°ΡΠ° 24
Π‘Π°ΠΌΠΎΠ»Π΅Ρ Π»Π΅ΡΠΈΡ Π³ΠΎΡΠΈΠ·ΠΎΠ½ΡΠ°Π»ΡΠ½ΠΎ ΡΠΎ ΡΠΊΠΎΡΠΎΡΡΡΡ 900 ΠΊΠΌ/Ρ. ΠΠ°ΠΉΠ΄ΠΈΡΠ΅ ΡΠ°Π·Π½ΠΎΡΡΡ ΠΏΠΎΡΠ΅Π½ΡΠΈΠ°Π»ΠΎΠ², Π²ΠΎΠ·Π½ΠΈΠΊΠ°ΡΡΡΡ ΠΌΠ΅ΠΆΠ΄Ρ ΠΊΠΎΠ½ΡΠ°ΠΌΠΈ ΠΊΡΡΠ»ΡΠ΅Π² ΡΠ°ΠΌΠΎΠ»Π΅ΡΠ°, Π΅ΡΠ»ΠΈ Π²Π΅ΡΡΠΈΠΊΠ°Π»ΡΠ½Π°Ρ ΡΠΎΡΡΠ°Π²Π»ΡΡΡΠ°Ρ Π·Π΅ΠΌΠ½ΠΎΠ³ΠΎ ΠΌΠ°Π³Π½ΠΈΡΠ½ΠΎΠ³ΠΎ ΠΏΠΎΠ»Ρ ΡΠ°Π²Π½Π° 50 ΠΌΠΊΠ’Π» ΠΈ ΡΠ°Π·ΠΌΠ°Ρ
ΠΊΡΡΠ»ΡΠ΅Π² 12 ΠΌ.
ΠΠΎΠ΄ΡΠΊΠ°Π·ΠΊΠ°: ΠΈΡΠΏΠΎΠ»ΡΠ·ΡΠ΅ΡΡΡ ΡΠΎΡΠΌΡΠ»Π° ΠΠΠ‘ ΠΈΠ½Π΄ΡΠΊΡΠΈΠΈ Π² Π΄Π²ΠΈΠΆΡΡΠΈΡ
ΡΡ ΠΏΡΠΎΠ²ΠΎΠ΄Π½ΠΈΠΊΠ°Ρ
, Π³Π΄Π΅ ΡΠ°Π·ΠΌΠ°Ρ
ΠΊΡΡΠ»ΡΠ΅Π² β ΡΡΠΎ Π΄Π»ΠΈΠ½Π° ΠΏΡΠΎΠ²ΠΎΠ΄Π½ΠΈΠΊΠ°.
ΠΡΠ²Π΅Ρ: 0,15 Π
ΠΠ°Π΄Π°ΡΠ° 25
Π‘ΠΊΠΎΠ»ΡΠΊΠΎ Π²ΠΈΡΠΊΠΎΠ² Π΄ΠΎΠ»ΠΆΠ½Π° ΠΈΠΌΠ΅ΡΡ ΠΊΠ°ΡΡΡΠΊΠ°, ΡΡΠΎΠ±Ρ ΠΏΡΠΈ ΠΈΠ·ΠΌΠ΅Π½Π΅Π½ΠΈΠΈ ΠΌΠ°Π³Π½ΠΈΡΠ½ΠΎΠ³ΠΎ ΠΏΠΎΡΠΎΠΊΠ° Π²Π½ΡΡΡΠΈ Π½Π΅Π΅ ΠΎΡ 0,024 ΠΠ± Π΄ΠΎ 0,056 ΠΠ± Π·Π° ΠΏΡΠΎΠΌΠ΅ΠΆΡΡΠΎΠΊ Π²ΡΠ΅ΠΌΠ΅Π½ΠΈ 0,32 Ρ Π² ΠΊΠ°ΡΡΡΠΊΠ΅ Π²ΠΎΠ·Π½ΠΈΠΊΠ°Π»Π° ΡΡΠ΅Π΄Π½ΡΡ ΠΠΠ‘ ΠΈΠ½Π΄ΡΠΊΡΠΈΠΈ 10 Π?
ΠΡΠ²Π΅Ρ: 100 Π²ΠΈΡΠΊΠΎΠ²
Π£ΡΠΎΠΊ Π€ΠΈΠ·ΠΈΠΊΠΈ Π² 11 ΠΊΠ»Π°ΡΡΠ΅ ΠΏΠΎ ΡΠ΅ΠΌΠ΅ «ΠΠΠ‘ ΠΈΠ½Π΄ΡΠΊΡΠΈΠΈ Π² Π΄Π²ΠΈΠΆΡΡΠΈΡ ΡΡ ΠΏΡΠΎΠ²ΠΎΠ΄Π½ΠΈΠΊΠ°Ρ »
Π’Π΅ΠΌΠ° ΡΡΠΎΠΊΠ°: ΠΠΠ‘ ΠΈΠ½Π΄ΡΠΊΡΠΈΠΈ Π² ΠΏΡΠΎΠ²ΠΎΠ΄Π½ΠΈΠΊΠ°Ρ , Π΄Π²ΠΈΠΆΡΡΠΈΡ ΡΡ Π² ΠΌΠ°Π³Π½ΠΈΡΠ½ΠΎΠΌ ΠΏΠΎΠ»Π΅.
Π¦Π΅Π»Ρ ΡΡΠΎΠΊΠ°: ΠΡΡΡΠ½ΠΈΡΡ ΡΡΠ»ΠΎΠ²ΠΈΡ Π²ΠΎΠ·Π½ΠΈΠΊΠ½ΠΎΠ²Π΅Π½ΠΈΡ ΠΠΠ‘ Π² ΠΏΡΠΎΠ²ΠΎΠ΄Π½ΠΈΠΊΠ°Ρ , Π΄Π²ΠΈΠΆΡΡΠΈΡ ΡΡ Π² ΠΌΠ°Π³Π½ΠΈΡΠ½ΠΎΠΌ ΠΏΠΎΠ»Π΅.
ΠΠ°Π΄Π°ΡΠΈ:
ΠΠ±ΡΠ°Π·ΠΎΠ²Π°ΡΠ΅Π»ΡΠ½ΡΠ΅: ΠΡΡΡΠ½ΠΈΡΡ ΡΡΠ»ΠΎΠ²ΠΈΡ Π²ΠΎΠ·Π½ΠΈΠΊΠ½ΠΎΠ²Π΅Π½ΠΈΡ ΠΠΠ‘ Π² ΠΏΡΠΎΠ²ΠΎΠ΄Π½ΠΈΠΊΠ°Ρ , Π΄Π²ΠΈΠΆΡΡΠΈΡ ΡΡ Π² ΠΌΠ°Π³Π½ΠΈΡΠ½ΠΎΠΌ ΠΏΠΎΠ»Π΅.
ΠΠΎΡΠΏΠΈΡΠ°ΡΠ΅Π»ΡΠ½ΡΠ΅: Π€ΠΎΡΠΌΠΈΡΠΎΠ²Π°ΡΡ ΠΊΡΠ»ΡΡΡΡΡ ΠΎΠ±ΡΠ΅Π½ΠΈΡ (Π²ΡΡΠ»ΡΡΠΈΠ²Π°ΡΡ ΡΠΎΠ±Π΅ΡΠ΅Π΄Π½ΠΈΠΊΠ°, Π΄Π°ΡΡ Π°Π½Π°Π»ΠΈΠ· ΡΡΠ»ΡΡΠ°Π½Π½ΠΎΠΌΡ), ΡΠΌΠ΅Π½ΠΈΠ΅ ΠΊΠΎΠ»Π»Π΅ΠΊΡΠΈΠ²Π½ΠΎΠ³ΠΎ Π°Π½Π°Π»ΠΈΠ·Π°.
Π Π°Π·Π²ΠΈΠ²Π°ΡΡΠΈΠ΅: Π Π°Π·Π²ΠΈΠ²Π°ΡΡ ΡΠΈΠ·ΠΈΡΠ΅ΡΠΊΠΎΠ΅ ΠΌΡΡΠ»Π΅Π½ΠΈΠ΅, ΡΠ°ΡΡΠΈΡΡΡΡ ΠΏΠΎΠ½ΡΡΠΈΠΉΠ½ΡΠΉ Π°ΠΏΠΏΠ°ΡΠ°Ρ ΠΎΠ±ΡΡΠ°ΡΡΠΈΡ ΡΡ, ΡΠΎΡΠΌΠΈΡΠΎΠ²Π°ΡΡ ΡΠΌΠ΅Π½ΠΈΠ΅ Π°Π½Π°Π»ΠΈΠ·ΠΈΡΠΎΠ²Π°ΡΡ ΠΈΠ½ΡΠΎΡΠΌΠ°ΡΠΈΡ, Π΄Π΅Π»Π°ΡΡ Π²ΡΠ²ΠΎΠ΄ ΠΈΠ· Π½Π°Π±Π»ΡΠ΄Π΅Π½ΠΈΠΉ ΠΈ ΠΎΠΏΡΡΠΎΠ².
ΠΠ±ΠΎΡΡΠ΄ΠΎΠ²Π°Π½ΠΈΠ΅: ΠΠ, ΠΏΡΠΎΠ΅ΠΊΡΠΎΡ, ΠΈΠ½ΡΠ΅ΡΠ°ΠΊΡΠΈΠ²Π½Π°Ρ Π΄ΠΎΡΠΊΠ°, ΠΏΡΠ΅Π·Π΅Π½ΡΠ°ΡΠΈΡ, ΡΠ°Π³ΠΎΠ²ΡΠΉ Π΄Π²ΠΈΠ³Π°ΡΠ΅Π»Ρ, Π³Π°Π»ΡΠ²Π°Π½ΠΎΠΌΠ΅ΡΡ.
Π₯ΠΎΠ΄ ΡΡΠΎΠΊΠ°:
I. ΠΡΠ³Π°Π½ΠΈΠ·Π°ΡΠΈΠΎΠ½Π½ΡΠΉ ΠΌΠΎΠΌΠ΅Π½Ρ.
Π‘Π΅Π³ΠΎΠ΄Π½Ρ Π½Π° ΡΡΠΎΠΊΠ΅ ΠΌΡ ΠΏΡΠΎΠ΄ΠΎΠ»ΠΆΠΈΠΌ ΠΈΡΡΠ»Π΅Π΄ΠΎΠ²Π°ΡΡ ΠΏΡΠΈΡΠΈΠ½Ρ Π²ΠΎΠ·Π½ΠΈΠΊΠ½ΠΎΠ²Π΅Π½ΠΈΡ ΠΈΠ½Π΄ΡΠΊΡΠΈΠΎΠ½Π½ΠΎΠ³ΠΎ ΡΠΎΠΊΠ° Π·Π°ΠΌΠΊΠ½ΡΡΠΎΠ³ΠΎ ΠΏΡΠΎΠ²ΠΎΠ΄ΡΡΠ΅Π³ΠΎ ΠΊΠΎΠ½ΡΡΡΠ°. ΠΠ° ΠΏΡΠΎΡΠ»ΡΡ Π·Π°Π½ΡΡΠΈΡΡ Π½Π°ΠΌΠΈ Π±ΡΠ» ΡΡΡΠ°Π½ΠΎΠ²Π»Π΅Π½ ΡΠΎΡ ΡΠ°ΠΊΡ, ΡΡΠΎ ΠΈΠ½Π΄ΡΠΊΡΠΈΠΎΠ½Π½ΡΠΉ ΡΠΎΠΊ Π²ΠΎΠ·Π½ΠΈΠΊΠ°Π΅Ρ Π² ΡΠ»Π΅Π΄ΡΡΡΠΈΡ ΡΠ»ΡΡΠ°ΡΡ : ΠΊΠΎΠ½ΡΡΡ ΠΏΠΎΠΊΠΎΠΈΡΡΡ Π² ΠΏΠ΅ΡΠ΅ΠΌΠ΅Π½Π½ΠΎΠΌ ΠΌΠ°Π³Π½ΠΈΡΠ½ΠΎΠΌ ΠΏΠΎΠ»Π΅; ΠΏΠ΅ΡΠ΅ΠΌΠ΅ΡΠ°Π΅ΡΡΡ Π² ΠΏΠΎΡΡΠΎΡΠ½Π½ΠΎΠΌ ΠΌΠ°Π³Π½ΠΈΡΠ½ΠΎΠΌ ΠΏΠΎΠ»Π΅. ΠΠ΅ΡΠ²ΠΎΠ΅ ΡΡΠ»ΠΎΠ²ΠΈΠ΅ Π²ΠΎΠ·Π±ΡΠΆΠ΄Π΅Π½ΠΈΡ ΠΈΠ½Π΄ΡΠΊΡΠΈΠΎΠ½Π½ΠΎΠ³ΠΎ ΡΠΎΠΊΠ° ΠΌΡ ΡΠΆΠ΅ ΡΠ°Π·ΠΎΠ±ΡΠ°Π»ΠΈ, ΠΈ ΡΠ΅ΠΏΠ΅ΡΡ Π΄Π°Π²Π°ΠΉΡΠ΅ ΠΈΡΡΠ»Π΅Π΄ΡΠ΅ΠΌ Π²ΡΠΎΡΠΎΠ΅.
/Π‘Π»Π°ΠΉΠ΄_1/
Π’Π΅ΠΌΠ° ΡΠ΅Π³ΠΎΠ΄Π½ΡΡΠ½Π΅Π³ΠΎ Π½Π°ΡΠ΅Π³ΠΎ ΡΡΠΎΠΊΠ° «ΠΠΠ‘ ΠΈΠ½Π΄ΡΠΊΡΠΈΠΈ Π² ΠΏΡΠΎΠ²ΠΎΠ΄Π½ΠΈΠΊΠ°Ρ , Π΄Π²ΠΈΠΆΡΡΠΈΡ ΡΡ Π² ΠΌΠ°Π³Π½ΠΈΡΠ½ΠΎΠΌ ΠΏΠΎΠ»Π΅».
Π¦Π΅Π»Ρ ΡΡΠΎΠΊΠ°: Π²ΡΡΡΠ½ΠΈΡΡ ΡΡΠ»ΠΎΠ²ΠΈΡ Π²ΠΎΠ·Π±ΡΠΆΠ΄Π΅Π½ΠΈΠ΅ ΠΠΠ‘ Π² ΠΏΡΠΎΠ²ΠΎΠ΄Π½ΠΈΠΊΠ°Ρ , Π΄Π²ΠΈΠΆΡΡΠΈΡ ΡΡ Π² ΠΏΠΎΡΡΠΎΡΠ½Π½ΠΎΠΌ ΠΠ.
II. ΠΠΊΡΡΠ°Π»ΠΈΠ·Π°ΡΠΈΡ Π·Π½Π°Π½ΠΈΠΉ.
ΠΠ° ΡΠ΅ΠΊΡΡΠΈΠΉ ΠΌΠΎΠΌΠ΅Π½Ρ ΠΌΡ ΠΏΠΎΠ·Π½Π°ΠΊΠΎΠΌΠΈΠ»ΠΈΡΡ Ρ ΡΡΠ΄ΠΎΠΌ ΠΏΡΠ°Π²ΠΈΠ», ΠΊΠΎΡΠΎΡΡΠ΅ ΠΈΡΠΏΠΎΠ»ΡΠ·ΡΡΡΡΡ Π΄Π»Ρ ΠΎΠΏΡΠ΅Π΄Π΅Π»Π΅Π½ΠΈΡ ΡΠΈΠ»Ρ ΠΠΌΠΏΠ΅ΡΠ° ΠΈ ΠΠΎΡΠ΅Π½ΡΠ°, Π° ΡΠ°ΠΊΠΆΠ΅ Π½Π°ΠΏΡΠ°Π²Π»Π΅Π½ΠΈΡ Π²Π΅ΠΊΡΠΎΡΠ° ΠΠ. Π§ΡΠΎΠ±Ρ Π΄ΠΎΡΡΠΈΡΡ ΠΏΠΎΡΡΠ°Π²Π»Π΅Π½Π½ΠΎΠΉ ΡΠ΅Π»ΠΈ Π½Π°ΠΌ ΡΠ΅Π³ΠΎΠ΄Π½Ρ ΠΏΠΎΠ½Π°Π΄ΠΎΠ±ΠΈΡΡΡΡ ΡΠ°ΡΡΡ ΡΡΠΈΡ Π·Π½Π°Π½ΠΈΠΉ. ΠΠ°Π²Π°ΠΉΡΠ΅ ΠΈΡ ΠΏΠΎΠ²ΡΠΎΡΠΈΠΌ.
/Π‘Π»Π°ΠΉΠ΄_2/
Π ΡΠ΅ΠΌ ΠΎΠ½ΠΈ Π³ΠΎΠ²ΠΎΡΡΡ? (ΠΠ° ΡΠ»Π°ΠΉΠ΄Π°Ρ ΠΏΡΠ°Π²ΠΈΠ»Π° ΠΏΠΎΡΠ²Π»ΡΡΡΡΡ ΠΏΠΎΡΠ»Π΅ ΠΎΡΠ²Π΅ΡΠ° ΠΎΠ±ΡΡΠ°ΡΡΠ΅Π³ΠΎΡΡ (Π΄Π»Ρ ΠΊΠΎΡΡΠ΅ΠΊΡΠΈΡΠΎΠ²ΠΊΠΈ, Π΅ΡΠ»ΠΈ ΠΎΡΠ²Π΅Ρ ΡΠΎΠ΄Π΅ΡΠΆΠ°Π» ΠΎΡΠΈΠ±ΠΊΡ))
/Π‘Π»Π°ΠΉΠ΄_3/
ΠΡΠ°Π²ΠΈΠ»ΠΎ Π±ΡΡΠ°Π²ΡΠΈΠΊΠ°.
/Π‘Π»Π°ΠΉΠ΄_4/
ΠΡΠ°Π²ΠΈΠ»ΠΎ Π»Π΅Π²ΠΎΠΉ ΡΡΠΊΠΈ.
— ΠΠ»Ρ Π΄Π²ΠΈΠΆΡΡΠ΅Π³ΠΎΡΡ ΡΡΠ°ΡΡΠΊΠ° ΠΏΡΠΎΠ²ΠΎΠ΄Π½ΠΈΠΊΠ° Ρ ΡΠΎΠΊΠΎΠΌ.
/Π‘Π»Π°ΠΉΠ΄_5/
— ΠΠ»Ρ Π΄Π²ΠΈΠΆΡΡΠ΅ΠΉΡΡ ΡΠ°ΡΡΠΈΡΡ.
/Π‘Π»Π°ΠΉΠ΄_6/
ΠΡΠ°Π²ΠΈΠ»ΠΎ ΠΠ΅Π½ΡΠ°
ΠΡΠ΅ ΡΡΠΈ ΠΏΡΠ°Π²ΠΈΠ»Π° ΡΠ²ΡΠ·Π°Π½Ρ Ρ ΡΠ²Π»Π΅Π½ΠΈΠ΅ΠΌ ΠΠΠ.
/Π‘Π»Π°ΠΉΠ΄_7/
— ΠΠΎΠ³Π΄Π° ΠΈ ΠΊΠ΅ΠΌ ΠΎΠ½ΠΎ Π±ΡΠ»ΠΎ ΠΎΡΠΊΡΡΡΠΎ ΠΈ Π² ΡΠ΅ΠΌ ΠΎΠ½ΠΎ Π·Π°ΠΊΠ»ΡΡΠ°Π΅ΡΡΡ?
/Π‘Π»Π°ΠΉΠ΄_8/
— Π‘ΡΠΎΡΠΌΡΠ»ΠΈΡΡΠΉΡΠ΅ Π·Π°ΠΊΠΎΠ½ ΡΠ»Π΅ΠΊΡΡΠΎΠΌΠ°Π³Π½ΠΈΡΠ½ΠΎΠΉ ΠΈΠ½Π΄ΡΠΊΡΠΈΠΈ.
/Π‘Π»Π°ΠΉΠ΄_9/
— Π§ΡΠΎ ΡΠ²Π»ΡΠ΅ΡΡΡ ΠΏΡΠΈΡΠΈΠ½ΠΎΠΉ Π²ΠΎΠ·Π½ΠΈΠΊΠ½ΠΎΠ²Π΅Π½ΠΈΡ ΠΈΠ½Π΄ΡΠΊΡΠΈΠΎΠ½Π½ΠΎΠ³ΠΎ ΡΠΎΠΊΠ° Π² ΠΏΡΠΎΠ²ΠΎΠ΄Π½ΠΈΠΊΠ΅, ΠΏΠΎΠΊΠΎΡΡΠ΅ΠΌΡΡ Π² ΠΏΠ΅ΡΠ΅ΠΌΠ΅Π½Π½ΠΎΠΌ ΠΌΠ°Π³Π½ΠΈΡΠ½ΠΎΠΌ ΠΏΠΎΠ»Π΅?
(ΠΡΠ²Π΅Ρ)
— Π§Π΅ΠΌΡ ΡΠ°Π²Π½Π° ΡΠ°Π±ΠΎΡΠ° ΡΡΠΎΠ³ΠΎ ΡΠ»Π΅ΠΊΡΡΠΈΡΠ΅ΡΠΊΠΎΠ³ΠΎ ΠΏΠΎΠ»Ρ?
(ΠΡΠ²Π΅Ρ)
III. ΠΠ·ΡΡΠ΅Π½ΠΈΠ΅ Π½ΠΎΠ²ΠΎΠ³ΠΎ ΠΌΠ°ΡΠ΅ΡΠΈΠ°Π»Π°.
ΠΡΠ»ΠΈ ΠΏΡΠΎΠ²ΠΎΠ΄Π½ΠΈΠΊ Π΄Π²ΠΈΠΆΠ΅ΡΡΡ Π² ΠΏΠΎΡΡΠΎΡΠ½Π½ΠΎΠΌ ΠΌΠ°Π³Π½ΠΈΡΠ½ΠΎΠΌ ΠΏΠΎΠ»Π΅, ΡΠΎ ΠΠΠ‘ ΠΈΠ½Π΄ΡΠΊΡΠΈΠΈ Π² ΠΏΡΠΎΠ²ΠΎΠ΄Π½ΠΈΠΊΠ΅ ΠΎΠ±ΡΡΠ»ΠΎΠ²Π»Π΅Π½Π° Π½Π΅ Π²ΠΈΡ ΡΠ΅Π²ΡΠΌ ΡΠ»Π΅ΠΊΡΡΠΈΡΠ΅ΡΠΊΠΈΠΌ ΠΏΠΎΠ»Π΅ΠΌ, ΠΊΠΎΡΠΎΡΠΎΠ΅ Π² ΡΡΠΎΠΌ ΡΠ»ΡΡΠ°Π΅ Π½Π΅ ΠΌΠΎΠΆΠ΅Ρ Π²ΠΎΠ·Π½ΠΈΠΊΠ½ΡΡΡ, Π° Π΄ΡΡΠ³ΠΎΠΉ ΠΏΡΠΈΡΠΈΠ½ΠΎΠΉ.
ΠΠ°Π²Π°ΠΉΡΠ΅ Π²ΡΡΡΠ½ΠΈΠΌ, ΡΡΠΎ ΠΆΠ΅ ΡΠ²Π»ΡΠ΅ΡΡΡ ΠΏΡΠΈΡΠΈΠ½ΠΎΠΉ Π²ΠΎΠ·Π½ΠΈΠΊΠ½ΠΎΠ²Π΅Π½ΠΈΡ ΠΠΠ‘.
ΠΠ°ΠΊΠΈΠ΅ ΡΠ°ΡΡΠΈΡΡ ΡΠ²Π»ΡΡΡΡΡ Π½Π΅ΠΎΡΡΠ΅ΠΌΠ»Π΅ΠΌΠΎΠΉ ΡΠ°ΡΡΡΡ Π»ΡΠ±ΠΎΠ³ΠΎ ΠΏΡΠΎΠ²ΠΎΠ΄Π½ΠΈΠΊΠ°? (Π‘Π²ΠΎΠ±ΠΎΠ΄Π½ΡΠ΅ Π·Π°ΡΡΠΆΠ΅Π½Π½ΡΠ΅ ΡΠ°ΡΡΠΈΡΡ)
Π§ΡΠΎ ΠΏΡΠΎΠΈΡΡ ΠΎΠ΄ΠΈΡ Ρ Π½ΠΈΠΌΠΈ, Π΅ΡΠ»ΠΈ ΠΌΡ ΠΏΠ΅ΡΠ΅ΠΌΠ΅ΡΠ°Π΅ΠΌ ΠΏΡΠΎΠ²ΠΎΠ΄Π½ΠΈΠΊ Π² ΠΌΠ°Π³Π½ΠΈΡΠ½ΠΎΠΌ ΠΏΠΎΠ»Π΅? (ΠΡΠΈ Π΄Π²ΠΈΠΆΠ΅Π½ΠΈΠΈ ΠΏΡΠΎΠ²ΠΎΠ΄Π½ΠΈΠΊΠ° Π΅Π³ΠΎ ΡΠ²ΠΎΠ±ΠΎΠ΄Π½ΡΠ΅ Π·Π°ΡΡΠ΄Ρ Π΄Π²ΠΈΠΆΡΡΡΡ Π²ΠΌΠ΅ΡΡΠ΅ Ρ Π½ΠΈΠΌ. ΠΠΎΡΡΠΎΠΌΡ Π½Π° Π½ΠΈΡ ΡΠΎ ΡΡΠΎΡΠΎΠ½Ρ ΠΌΠ°Π³Π½ΠΈΡΠ½ΠΎΠ³ΠΎ ΠΏΠΎΠ»Ρ Π΄Π΅ΠΉΡΡΠ²ΡΠ΅Ρ ΡΠΈΠ»Π° ΠΠΎΡΠ΅Π½ΡΠ°. ΠΠ½Π°-ΡΠΎ ΠΈ Π²ΡΠ·ΡΠ²Π°Π΅Ρ ΠΏΠ΅ΡΠ΅ΠΌΠ΅ΡΠ΅Π½ΠΈΠ΅ Π·Π°ΡΡΠ΄ΠΎΠ² Π²Π½ΡΡΡΠΈ ΠΏΡΠΎΠ²ΠΎΠ΄Π½ΠΈΠΊΠ°. ΠΠΠ‘ ΠΈΠ½Π΄ΡΠΊΡΠΈΠΈ, ΡΠ»Π΅Π΄ΠΎΠ²Π°ΡΠ΅Π»ΡΠ½ΠΎ, ΠΈΠΌΠ΅Π΅Ρ ΠΌΠ°Π³Π½ΠΈΡΠ½ΠΎΠ΅ ΠΏΡΠΎΠΈΡΡ ΠΎΠΆΠ΄Π΅Π½ΠΈΠ΅).
/Π‘Π»Π°ΠΉΠ΄_10/
ΠΡΡΠΈΡΠ»ΠΈΠΌ ΠΠΠ‘ ΠΈΠ½Π΄ΡΠΊΡΠΈΠΈ, Π²ΠΎΠ·Π½ΠΈΠΊΠ°ΡΡΡΡ Π² ΠΏΡΠΎΠ²ΠΎΠ΄Π½ΠΈΠΊΠ΅, Π΄Π²ΠΈΠΆΡΡΠ΅ΠΌΡΡ Π² ΠΎΠ΄Π½ΠΎΡΠΎΠ΄Π½ΠΎΠΌ ΠΌΠ°Π³Π½ΠΈΡΠ½ΠΎΠΌ ΠΏΠΎΠ»Π΅. (ΠΠ±ΡΡΠ°ΡΡΠΈΠΉΡΡ Ρ ΠΏΠΎΠΌΠΎΡΡΡ ΡΡΠΈΡΠ΅Π»Ρ Π²ΡΠ²ΠΎΠ΄ΠΈΡ ΡΠΎΡΠΌΡΠ»Ρ)
(ΠΡΡΡΡ ΡΡΠΎΡΠΎΠ½Π° ΠΊΠΎΠ½ΡΡΡΠ° MN Π΄Π»ΠΈΠ½ΠΎΠΉ l ΡΠΊΠΎΠ»ΡΠ·ΠΈΡ Ρ ΠΏΠΎΡΡΠΎΡΠ½Π½ΠΎΠΉ ΡΠΊΠΎΡΠΎΡΡΡΡ Ο Π²Π΄ΠΎΠ»Ρ ΡΡΠΎΡΠΎΠ½ NC ΠΈ MD, ΠΎΡΡΠ°Π²Π°ΡΡΡ Π²ΡΠ΅ Π²ΡΠ΅ΠΌΡ ΠΏΠ°ΡΠ°Π»Π»Π΅Π»ΡΠ½ΠΎΠΉ ΡΡΠΎΡΠΎΠ½Π΅ CD. ΠΠ΅ΠΊΡΠΎΡ ΠΌΠ°Π³Π½ΠΈΡΠ½ΠΎΠΉ ΠΈΠ½Π΄ΡΠΊΡΠΈΠΈ ΠΎΠ΄Π½ΠΎΡΠΎΠ΄Π½ΠΎΠ³ΠΎ ΠΏΠΎΠ»Ρ ΠΏΠ΅ΡΠΏΠ΅Π½Π΄ΠΈΠΊΡΠ»ΡΡΠ΅Π½ ΠΏΡΠΎΠ²ΠΎΠ΄Π½ΠΈΠΊΡ ΠΈ ΡΠΎΡΡΠ°Π²Π»ΡΠ΅Ρ ΡΠ³ΠΎΠ» Ξ± Ρ Π½Π°ΠΏΡΠ°Π²Π»Π΅Π½ΠΈΠ΅ΠΌ Π΅Π³ΠΎ ΡΠΊΠΎΡΠΎΡΡΠΈ.
Π‘ΠΈΠ»Π°, Ρ ΠΊΠΎΡΠΎΡΠΎΠΉ ΠΌΠ°Π³Π½ΠΈΡΠ½ΠΎΠ΅ ΠΏΠΎΠ»Π΅ Π΄Π΅ΠΉΡΡΠ²ΡΠ΅Ρ Π½Π° Π΄Π²ΠΈΠΆΡΡΡΡΡΡ Π·Π°ΡΡΠΆΠ΅Π½Π½ΡΡ ΡΠ°ΡΡΠΈΡΡ, ΡΠ°Π²Π½Π° ΠΏΠΎ ΠΌΠΎΠ΄ΡΠ»Ρ .
ΠΠ°ΠΏΡΠ°Π²Π»Π΅Π½Π° ΠΎΠ½Π° Π²Π΄ΠΎΠ»Ρ ΠΏΡΠΎΠ²ΠΎΠ΄Π½ΠΈΠΊΠ° MN.
Π Π°Π±ΠΎΡΠ° Π΄Π°Π½Π½ΠΎΠΉ ΡΠΈΠ»Ρ Π½Π° ΠΏΡΡΠΈ l ΠΏΠΎΠ»ΠΎΠΆΠΈΡΠ΅Π»ΡΠ½Π° ΠΈ ΡΠ°Π²Π½Π° .
/Π‘Π»Π°ΠΉΠ΄_11/
ΠΠΠ‘ ΠΈΠ½Π΄ΡΠΊΡΠΈΠΈ Π² ΠΏΡΠΎΠ²ΠΎΠ΄Π½ΠΈΠΊΠ΅ MN ΡΠ°Π²Π½Π° ΠΏΠΎ ΠΎΠΏΡΠ΅Π΄Π΅Π»Π΅Π½ΠΈΡ ΠΎΡΠ½ΠΎΡΠ΅Π½ΠΈΡ ΡΠ°Π±ΠΎΡΡ ΠΏΠΎ ΠΏΠ΅ΡΠ΅ΠΌΠ΅ΡΠ΅Π½ΠΈΡ Π·Π°ΡΡΠ΄Π° q ΠΊ ΡΡΠΎΠΌΡ Π·Π°ΡΡΠ΄Ρ. .)
/Π‘Π»Π°ΠΉΠ΄_12/
Π‘ΡΡΠ΅ΡΡΠ²ΡΠ΅Ρ Π²ΡΠΎΡΠΎΠΉ ΡΠΏΠΎΡΠΎΠ± Π²ΡΠ²ΠΎΠ΄Π° ΡΠΎΡΠΌΡΠ»Ρ. ΠΡΠΎ ΡΠΏΠΎΡΠΎΠ± ΠΌΡ ΠΏΡΠΎΡΠΌΠΎΡΡΠΈΠΌ Π½Π° ΡΡΠ°Π³ΠΌΠ΅Π½ΡΠ΅ Π²ΠΈΠ΄Π΅ΠΎΡΡΠΎΠΊΠ°.
ΠΠ°ΠΊ Π²Ρ ΡΡΠΈΡΠ°Π΅ΡΠ΅, ΠΊΠ°ΠΊΠΎΠΉ ΡΠΏΠΎΡΠΎΠ± Π΄Π»Ρ Π²Π°Ρ ΡΠ΄ΠΎΠ±Π½Π΅Π΅? (ΠΎΡΠ²Π΅Ρ)
ΠΡ ΠΌΠΎΠΆΠ΅ΡΠ΅ ΠΏΠΎΠ»ΡΠ·ΠΎΠ²Π°ΡΡΡΡ Π»ΡΠ±ΡΠΌ ΡΠΏΠΎΡΠΎΠ±ΠΎΠΌ.
/Π‘Π»Π°ΠΉΠ΄_13/
ΠΠ»Ρ ΠΎΠΏΡΠ΅Π΄Π΅Π»Π΅Π½ΠΈΡ Π½Π°ΠΏΡΠ°Π²Π»Π΅Π½ΠΈΡ ΡΠ»Π΅ΠΊΡΡΠΈΡΠ΅ΡΠΊΠΎΠ³ΠΎ ΡΠΎΠΊΠ° Π² Π·Π°ΠΌΠΊΠ½ΡΡΠΎΠΌ ΠΏΡΠΎΠ²ΠΎΠ΄Π½ΠΈΠΊΠ΅ ΠΏΡΠΈΠΌΠ΅Π½ΡΠ΅ΡΡΡ ΠΏΡΠ°Π²ΠΈΠ»ΠΎ ΠΏΡΠ°Π²ΠΎΠΉ ΡΡΠΊΠΈ (ΠΎΠ±ΡΡΡΠ½Π΅Π½ΠΈΠ΅ ΠΏΡΠ°Π²ΠΈΠ»Π° ΠΏΡΠ°Π²ΠΎΠΉ ΡΡΠΊΠΈ).
Π ΡΠ΅ΠΉΡΠ°Ρ Π΄Π°Π²Π°ΠΉΡΠ΅ ΠΏΡΠΎΠ²Π΅Π΄Π΅ΠΌ Π½Π΅Π±ΠΎΠ»ΡΡΠΎΠΉ ΡΠΊΡΠΏΠ΅ΡΠΈΠΌΠ΅Π½Ρ Ρ ΡΠ°Π³ΠΎΠ²ΡΠΌ Π΄Π²ΠΈΠ³Π°ΡΠ΅Π»Π΅ΠΌ. (1. ΠΡΠ°ΡΠ΅Π½ΠΈΠ΅ ΡΠΎΡΠΎΡΠ° Ρ Π½Π΅Π·Π°ΠΌΠΊΠ½ΡΡΡΠΌΠΈ ΡΠ°Π·Π°ΠΌΠΈ. 2. ΠΡΠ°ΡΠ΅Π½ΠΈΠ΅ ΡΠΎΡΠΎΡΠ° Ρ ΠΎΠ΄Π½ΠΎΠΉ Π·Π°ΠΌΠΊΠ½ΡΡΠΎΠΉ ΡΠ°Π·ΠΎΠΉ. 3. ΠΡΠ°ΡΠ΅Π½ΠΈΠ΅ ΡΠΎΡΠΎΡΠ° Ρ Π΄Π²ΡΠΌΡ Π·Π°ΠΌΠΊΠ½ΡΡΡΠΌΠΈ ΡΠ°Π·Π°ΠΌΠΈ)
/Π‘Π»Π°ΠΉΠ΄_14/
ΠΠΎΠ·Π½ΠΈΠΊΠ°Π΅Ρ Π²ΠΎΠΏΡΠΎΡ: ΠΠ°ΠΊ Π²Ρ ΡΡΠΈΡΠ°Π΅ΡΠ΅, ΠΏΠΎΡΠ΅ΠΌΡ Π΄Π²ΠΈΠ³Π°ΡΠ΅Π»Ρ Π½Π°ΡΠ°Π» ΠΏΡΠ΅ΠΏΡΡΡΡΠ²ΠΎΠ²Π°ΡΡ Π²ΡΠ°ΡΠ΅Π½ΠΈΡ ΡΠΎΡΠΎΡΠ°?
(ΠΡΠΈΡΠΈΠ½ΠΎΠΉ ΡΠ²Π»ΡΠ΅ΡΡΡ Π²ΠΎΠ·Π½ΠΈΠΊΠ½ΠΎΠ²Π΅Π½ΠΈΠ΅ ΡΠΈΠ»Ρ ΠΠΌΠΏΠ΅ΡΠ°, Ρ.ΠΊ. ΡΠ΅ΡΠ΅Π· Π·Π°ΠΌΠΊΠ½ΡΡΡΠΉ ΠΏΡΠΎΠ²ΠΎΠ΄Π½ΠΈΠΊ Π½Π°ΡΠΈΠ½Π°Π΅Ρ ΡΠ΅ΡΡ ΡΠ»Π΅ΠΊΡΡΠΈΡΠ΅ΡΠΊΠΈΠΉ ΡΠΎΠΊ, Π° ΠΏΡΠΎΠ²ΠΎΠ΄Π½ΠΈΠΊ ΠΏΡΠΈ ΡΡΠΎΠΌ Π΄Π²ΠΈΠΆΠ΅ΡΡΡ Π² ΠΌΠ°Π³Π½ΠΈΡΠ½ΠΎΠΌ ΠΏΠΎΠ»Π΅).
IV. ΠΠ°ΠΊΡΠ΅ΠΏΠ»Π΅Π½ΠΈΠ΅ ΠΈΠ·ΡΡΠ΅Π½Π½ΠΎΠ³ΠΎ.
Π Π΅ΡΠ΅Π½ΠΈΠ΅ Π·Π°Π΄Π°Ρ.
/Π‘Π»Π°ΠΉΠ΄_15/
ΠΠ°ΠΉΡΠΈ ΠΠΠ‘ ΠΈΠ½Π΄ΡΠΊΡΠΈΠΈ Π² ΠΏΡΠΎΠ²ΠΎΠ΄Π½ΠΈΠΊΠ΅ Ρ Π΄Π»ΠΈΠ½ΠΎΠΉ Π°ΠΊΡΠΈΠ²Π½ΠΎΠΉ ΡΠ°ΡΡΠΈ 25 ΡΠΌ, ΠΏΠ΅ΡΠ΅ΠΌΠ΅ΡΠ°ΡΡΠ΅ΠΌΡΡ Π² ΠΎΠ΄Π½ΠΎΡΠΎΠ΄Π½ΠΎΠΌ ΠΌΠ°Π³Π½ΠΈΡΠ½ΠΎΠΌ ΠΏΠΎΠ»Π΅ ΠΈΠ½Π΄ΡΠΊΡΠΈΠ΅ΠΉ 8 ΠΌΠ’Π» ΡΠΎ ΡΠΊΠΎΡΠΎΡΡΡΡ 5 ΠΌ/Ρ ΠΏΠΎΠ΄ ΡΠ³Π»ΠΎΠΌ 30Β° ΠΊ Π²Π΅ΠΊΡΠΎΡΡ ΠΌΠ°Π³Π½ΠΈΡΠ½ΠΎΠΉ ΠΈΠ½Π΄ΡΠΊΡΠΈΠΈ. (0,005 Π)
ΠΠ°ΠΊΡΡ ΡΠΈΠ»Ρ Π½Π΅ΠΎΠ±Ρ ΠΎΠ΄ΠΈΠΌΠΎ ΠΏΡΠΈΠ»ΠΎΠΆΠΈΡΡ, ΡΡΠΎΠ±Ρ ΠΏΠ΅ΡΠ΅ΠΌΠ΅ΡΠ°ΡΡ Π·Π°ΠΌΠΊΠ½ΡΡΡΠΉ ΠΏΡΠΎΠ²ΠΎΠ΄Π½ΠΈΠΊ ΡΠΎΠΏΡΠΎΡΠΈΠ²Π»Π΅Π½ΠΈΠ΅ΠΌ 1 ΠΠΌ ΠΈ Π΄Π»ΠΈΠ½ΠΎΠΉ 2 ΠΌΠ΅ΡΡΠ° Π² ΠΌΠ°Π³Π½ΠΈΡΠ½ΠΎΠΌ ΠΏΠΎΠ»Π΅ ΠΈΠ½Π΄ΡΠΊΡΠΈΠ΅ΠΉ 0,5 Π’Π» ΡΠΎ ΡΠΊΠΎΡΠΎΡΡΡΡ 6Β ΠΌ/Ρ ΠΏΠ΅ΡΠΏΠ΅Π½Π΄ΠΈΠΊΡΠ»ΡΡΠ½ΠΎ Π²Π΅ΠΊΡΠΎΡΡ ΠΌΠ°Π³Π½ΠΈΡΠ½ΠΎΠΉ ΠΈΠ½Π΄ΡΠΊΡΠΈΠΈ? (6 Π)
V. ΠΡΠΎΠ³ ΡΡΠΎΠΊΠ°.
/Π‘Π»Π°ΠΉΠ΄_1/
Π ΡΠ΅ΠΉΡΠ°Ρ Π΄Π°Π²Π°ΠΉΡΠ΅ Π²ΡΠΏΠΎΠΌΠ½ΠΈΠΌ ΡΠ΅ΠΌΡ ΠΈ ΡΠ΅Π»Ρ Π½Π°ΡΠ΅Π³ΠΎ ΡΡΠΎΠΊΠ°. ΠΠ°ΠΊ Π²Ρ ΡΡΠΈΡΠ°Π΅ΡΠ΅, Π΄ΠΎΡΡΠΈΠ³Π»ΠΈ ΠΌΡ ΡΠ΅Π»ΠΈ?
ΠΡΠ°ΠΊ, ΡΡΠΎ ΠΆΠ΅ ΠΌΡ Π²ΡΡΡΠ½ΠΈΠ»ΠΈ ΠΎ Π²ΠΎΠ·Π½ΠΈΠΊΠ½ΠΎΠ²Π΅Π½ΠΈΠΈ ΠΠΠ‘ Π½Π° ΡΠ΅Π³ΠΎΠ΄Π½ΡΡΠ½Π΅ΠΌ ΡΡΠΎΠΊΠ΅? (ΠΠΠ‘ ΠΈΠ½Π΄ΡΠΊΡΠΈΠΈ Π² ΠΏΡΠΎΠ²ΠΎΠ΄Π½ΠΈΠΊΠ°Ρ , Π΄Π²ΠΈΠΆΡΡΠΈΡ ΡΡ Π² ΠΏΠΎΡΡΠΎΡΠ½Π½ΠΎΠΌ ΠΌΠ°Π³Π½ΠΈΡΠ½ΠΎΠΌ ΠΏΠΎΠ»Π΅, Π²ΠΎΠ·Π½ΠΈΠΊΠ°Π΅Ρ Π·Π° ΡΡΠ΅Ρ Π΄Π΅ΠΉΡΡΠ²ΠΈΡ Π½Π° ΡΠ²ΠΎΠ±ΠΎΠ΄Π½ΡΠ΅ Π·Π°ΡΡΠ΄Ρ ΠΏΡΠΎΠ²ΠΎΠ΄Π½ΠΈΠΊΠ° ΡΠΈΠ»Ρ ΠΠΎΡΠ΅Π½ΡΠ°).
Π§ΡΠΎ Π½Π΅ΠΎΠ±Ρ ΠΎΠ΄ΠΈΠΌΠΎ Π·Π½Π°ΡΡ Π΄Π»Ρ ΠΎΠΏΡΠ΅Π΄Π΅Π»Π΅Π½ΠΈΡ ΠΠΠ‘ Π² ΡΠ°ΠΊΠΎΠΌ ΠΏΡΠΎΠ²ΠΎΠ΄Π½ΠΈΠΊΠ΅? (ΠΠ»Ρ ΠΊΠΎΠ»ΠΈΡΠ΅ΡΡΠ²Π΅Π½Π½ΠΎΠ³ΠΎ ΠΎΠΏΡΠ΅Π΄Π΅Π»Π΅Π½ΠΈΡ ΠΠΠ‘ Π½Π°ΠΌ Π½Π΅ΠΎΠ±Ρ ΠΎΠ΄ΠΈΠΌΠΎ Π·Π½Π°ΡΡ ΠΈΠ½Π΄ΡΠΊΡΠΈΡ ΠΏΠΎΡΡΠΎΡΠ½Π½ΠΎΠ³ΠΎ ΠΌΠ°Π³Π½ΠΈΡΠ½ΠΎΠ³ΠΎ ΠΏΠΎΠ»Ρ, Π΄Π»ΠΈΠ½Ρ Π°ΠΊΡΠΈΠ²Π½ΠΎΠΉ ΡΠ°ΡΡΠΈ ΠΏΡΠΎΠ²ΠΎΠ΄Π½ΠΈΠΊΠ°, ΡΠΊΠΎΡΠΎΡΡΡ Π΄Π²ΠΈΠΆΠ΅Π½ΠΈΡ ΠΏΡΠΎΠ²ΠΎΠ΄Π½ΠΈΠΊΠ° Π² ΠΌΠ°Π³Π½ΠΈΡΠ½ΠΎΠΌ ΠΏΠΎΠ»Π΅ ΠΈ ΡΠ³ΠΎΠ» ΠΌΠ΅ΠΆΠ΄Ρ Π²Π΅ΠΊΡΠΎΡΠΎΠΌ ΠΌΠ°Π³Π½ΠΈΡΠ½ΠΎΠΉ ΠΈΠ½Π΄ΡΠΊΡΠΈΠΈ ΠΈ Π²Π΅ΠΊΡΠΎΡΠΎΠΌ ΡΠΊΠΎΡΠΎΡΡΠΈ Π΄Π²ΠΈΠΆΠ΅Π½ΠΈΡ).
VI. ΠΠΎΠΌΠ°ΡΠ½Π΅Π΅ Π·Π°Π΄Π°Π½ΠΈΠ΅.
Β§ 13, ΡΠΏΡ.2(5), β929, 930, Π΄ΠΎΠΏΠΎΠ»Π½ΠΈΡΠ΅Π»ΡΠ½Π°Ρ Π·Π°Π΄Π°ΡΠ° Π½Π° ΠΊΠ°ΡΡΠΎΡΠΊΠ΅.
ru π
1
ΠΠ΅ΡΠ²ΡΠΉ ΡΠ»Π°ΠΉΠ΄ ΠΏΡΠ΅Π·Π΅Π½ΡΠ°ΡΠΈΠΈ: 17.04.2020 Π³. ΠΠΠ‘ ΠΈΠ½Π΄ΡΠΊΡΠΈΠΈ Π² Π΄Π²ΠΈΠΆΡΡΠΈΡ ΡΡ ΠΏΡΠΎΠ²ΠΎΠ΄Π½ΠΈΠΊΠ°Ρ . Π‘Π°ΠΌΠΎΠΈΠ½Π΄ΡΠΊΡΠΈΡ
ΠΠ»Π°Π½ ΡΡΠΎΠΊΠ°: ΠΡΠΎΠ²Π΅ΡΡΡΠ΅ ΡΠ²ΠΎΡ Π³ΠΎΡΠΎΠ²Π½ΠΎΡΡΡ ΠΊ ΠΈΠ·ΡΡΠ΅Π½ΠΈΡ Π½ΠΎΠ²ΠΎΠ³ΠΎ ΠΌΠ°ΡΠ΅ΡΠΈΠ°Π»Π° β ΠΎΡΠ²Π΅ΡΡΡΠ΅ Π½Π° Π²ΠΎΠΏΡΠΎΡΡ (ΡΠ»Π°ΠΉΠ΄Ρ2-3) ΠΈ ΠΠ·ΡΡΠΈΡΠ΅ Π½ΠΎΠ²ΡΠΉ ΠΌΠ°ΡΠ΅ΡΠΈΠ°Π» (ΡΠ»Π°ΠΉΠ΄Ρ 4-15) ΠΈ Π·Π°ΠΏΠΈΡΠΈΡΠ΅ Π² ΡΠ΅ΡΡΠ°Π΄Ρ ΡΠ°ΠΌΠΎΠ΅ ΠΎΡΠ½ΠΎΠ²Π½ΠΎΠ΅. Π Π΅ΡΠΈΡΠ΅ Π·Π°Π΄Π°ΡΠΈ (ΡΠ»Π°ΠΉΠ΄Ρ 16-27), ΠΏΠΎΡΡΠ°Π²ΠΈΠ² ΠΏΡΠ΅Π·Π΅Π½ΡΠ°ΡΠΈΡ Π½Π° Π²ΠΎΡΠΏΡΠΎΠΈΠ·Π²Π΅Π΄Π΅Π½ΠΈΠ΅ (ΡΠ½Π°ΡΠ°Π»Π° ΠΏΠΎΠΏΡΠΎΠ±ΡΠΉΡΠ΅ ΡΠ΅ΡΠΈΡΡ ΡΠ°ΠΌΠΎΡΡΠΎΡΡΠ΅Π»ΡΠ½ΠΎ, Π·Π°ΡΠ΅ΠΌ ΡΠΌΠΎΡΡΠΈΡΠ΅ ΠΎΡΠ²Π΅Ρ) ΠΡΠΏΠΎΠ»Π½ΠΈΡΠ΅ ΡΠ΅ΡΡ Β«+ΡΠ°ΠΌΠΎΠΈΠ½Π΄ΡΠΊΡΠΈΡΒ» (Π½Π° ΡΡΠ΅Π½Π΅ Π² Π³ΡΡΠΏΠΏΠ΅)
ΠΠ·ΠΎΠ±ΡΠ°ΠΆΠ΅Π½ΠΈΠ΅ ΡΠ»Π°ΠΉΠ΄Π°
2
Π‘Π»Π°ΠΉΠ΄ 2: ΠΡΠΎΠ²Π΅ΡΡΡΠ΅ ΡΠ²ΠΎΡ Π³ΠΎΡΠΎΠ²Π½ΠΎΡΡΡ ΠΊ ΠΈΠ·ΡΡΠ΅Π½ΠΈΡ Π½ΠΎΠ²ΠΎΠ³ΠΎ ΠΌΠ°ΡΠ΅ΡΠΈΠ°Π»Π°
ΠΠ°ΠΊ ΠΏΠΎΠΊΠ°Π·Π°ΡΡ Π½Π° ΠΎΠΏΡΡΠ΅, ΡΡΠΎ Π²Π΅Π»ΠΈΡΠΈΠ½Π° ΠΈΠ½Π΄ΡΠΊΡΠΈΠΎΠ½Π½ΠΎΠ³ΠΎ ΡΠΎΠΊΠ° Π·Π°Π²ΠΈΡΠΈΡ ΠΎΡ ΡΠΊΠΎΡΠΎΡΡΠΈ ΠΈΠ·ΠΌΠ΅Π½Π΅Π½ΠΈΡ ΠΌΠ°Π³Π½ΠΈΡΠ½ΠΎΠ³ΠΎ ΠΏΠΎΡΠΎΠΊΠ°? ΠΠ°ΠΊ ΠΏΠΎΠΊΠ°Π·Π°ΡΡ Π½Π° ΠΎΠΏΡΡΠ΅, ΡΡΠΎ Π½Π°ΠΏΡΠ°Π²Π»Π΅Π½ΠΈΠ΅ ΠΈΠ½Π΄ΡΠΊΡΠΈΠΎΠ½Π½ΠΎΠ³ΠΎ ΡΠΎΠΊΠ° Π·Π°Π²ΠΈΡΠΈΡ ΠΎΡ Π½Π°ΠΏΡΠ°Π²Π»Π΅Π½ΠΈΡ Π»ΠΈΠ½ΠΈΠΉ ΠΌΠ°Π³Π½ΠΈΡΠ½ΠΎΠΉ ΠΈΠ½Π΄ΡΠΊΡΠΈΠΈ Π²Π½Π΅ΡΠ½Π΅Π³ΠΎ ΠΌΠ°Π³Π½ΠΈΡΠ½ΠΎΠ³ΠΎ ΠΏΠΎΠ»Ρ? ΠΠ°ΠΊ ΡΠΈΡΠ°Π΅ΡΡΡ Π·Π°ΠΊΠΎΠ½ ΡΠ»Π΅ΠΊΡΡΠΎΠΌΠ°Π³Π½ΠΈΡΠ½ΠΎΠΉ ΠΈΠ½Π΄ΡΠΊΡΠΈΠΈ? ΠΠΎΡΠ΅ΠΌΡ Π² Π·Π°ΠΊΠΎΠ½Π΅ ΡΠ»Π΅ΠΊΡΡΠΎΠΌΠ°Π³Π½ΠΈΡΠ½ΠΎΠΉ ΠΈΠ½Π΄ΡΠΊΡΠΈΠΈ ΡΡΠΎΠΈΡ Π·Π½Π°ΠΊ Β«ΠΌΠΈΠ½ΡΡΒ»? ΠΠΎΡΠ΅ΠΌΡ Π·Π°ΠΊΠΎΠ½ ΡΠ»Π΅ΠΊΡΡΠΎΠΌΠ°Π³Π½ΠΈΡΠ½ΠΎΠΉ ΠΈΠ½Π΄ΡΠΊΡΠΈΠΈ ΡΠΎΡΠΌΡΠ»ΠΈΡΡΠ΅ΡΡΡ Π΄Π»Ρ ΠΠΠ‘ ΠΈΠ½Π΄ΡΠΊΡΠΈΠΈ, Π° Π½Π΅ Π΄Π»Ρ ΡΠΈΠ»Ρ ΠΈΠ½Π΄ΡΠΊΡΠΈΠΎΠ½Π½ΠΎΠ³ΠΎ ΡΠΎΠΊΠ°?
ΠΠ·ΠΎΠ±ΡΠ°ΠΆΠ΅Π½ΠΈΠ΅ ΡΠ»Π°ΠΉΠ΄Π°
3
Π‘Π»Π°ΠΉΠ΄ 3: ΠΡΠΎΠ²Π΅ΡΡΡΠ΅ ΡΠ²ΠΎΡ Π³ΠΎΡΠΎΠ²Π½ΠΎΡΡΡ ΠΊ ΠΈΠ·ΡΡΠ΅Π½ΠΈΡ Π½ΠΎΠ²ΠΎΠ³ΠΎ ΠΌΠ°ΡΠ΅ΡΠΈΠ°Π»Π°
ΠΠ°ΠΊ Π½Π°ΠΏΡΠ°Π²Π»Π΅Π½Ρ Π»ΠΈΠ½ΠΈΠΈ ΠΌΠ°Π³Π½ΠΈΡΠ½ΠΎΠΉ ΠΈΠ½Π΄ΡΠΊΡΠΈΠΈ Π² ΠΏΠΎΡΡΠΎΡΠ½Π½ΠΎΠΌ ΠΌΠ°Π³Π½ΠΈΡΠ΅? ΠΠ°ΠΊ Π½Π°ΠΏΡΠ°Π²Π»Π΅Π½Ρ Π»ΠΈΠ½ΠΈΠΈ ΠΌΠ°Π³Π½ΠΈΡΠ½ΠΎΠΉ ΠΈΠ½Π΄ΡΠΊΡΠΈΠΈ ΠΎΠΊΠΎΠ»ΠΎ ΠΏΡΠΎΠ²ΠΎΠ΄Π½ΠΈΠΊΠ° Ρ ΡΠΎΠΊΠΎΠΌ? ΠΠ°ΠΊ Π½Π°ΠΏΡΠ°Π²Π»Π΅Π½Ρ Π»ΠΈΠ½ΠΈΠΈ ΠΌΠ°Π³Π½ΠΈΡΠ½ΠΎΠΉ ΠΈΠ½Π΄ΡΠΊΡΠΈΠΈ ΠΎΠΊΠΎΠ»ΠΎ Π²ΠΈΡΠΊΠ° Ρ ΡΠΎΠΊΠΎΠΌ? ΠΠ΅ΡΠ΅ΡΠΈΡΠ»ΠΈΡΡ ΡΠ»ΡΡΠ°ΠΈ Π²ΠΎΠ·Π½ΠΈΠΊΠ½ΠΎΠ²Π΅Π½ΠΈΡ ΠΈΠ½Π΄ΡΠΊΡΠΈΠΎΠ½Π½ΠΎΠ³ΠΎ ΡΠΎΠΊΠ°. Π§ΡΠΎ Π½Π°Π·ΡΠ²Π°Π΅ΡΡΡ ΠΌΠ°Π³Π½ΠΈΡΠ½ΡΠΌ ΠΏΠΎΡΠΎΠΊΠΎΠΌ? Π ΡΠ΅ΠΌ ΡΠΌΡΡΠ» ΠΏΡΠ°Π²ΠΈΠ»Π° ΠΠ΅Π½ΡΠ°?
ΠΠ·ΠΎΠ±ΡΠ°ΠΆΠ΅Π½ΠΈΠ΅ ΡΠ»Π°ΠΉΠ΄Π°
4
Π‘Π»Π°ΠΉΠ΄ 4: ΠΠΠ‘ ΠΠΠΠ£ΠΠ¦ΠΠ Π ΠΠΠΠΠ£Π©ΠΠ₯Π‘Π― ΠΠ ΠΠΠΠΠΠΠΠΠ₯
ΠΠ·ΠΎΠ±ΡΠ°ΠΆΠ΅Π½ΠΈΠ΅ ΡΠ»Π°ΠΉΠ΄Π°
5
Π‘Π»Π°ΠΉΠ΄ 5: ΠΠΠ‘ ΠΈΠ½Π΄ΡΠΊΡΠΈΠΈ Π² Π΄Π²ΠΈΠΆΡΡΠΈΡ ΡΡ ΠΏΡΠΎΠ²ΠΎΠ΄Π½ΠΈΠΊΠ°Ρ
Π‘ΠΈΠ»Π° ΠΠΎΡΠ΅Π½ΡΠ°: Π Π°Π±ΠΎΡΠ° ΡΠΈΠ»Ρ (Π²Π΄ΠΎΠ»Ρ ΠΏΡΠΎΠ²ΠΎΠ΄Π½ΠΈΠΊΠ°): ΠΠΎ ΠΎΠΏΡΠ΅Π΄Π΅Π»Π΅Π½ΠΈΡ: ΡΠ°Π±ΠΎΡΠ° ΡΠΈΠ»Ρ ΠΏΠΎ ΠΏΠ΅ΡΠ΅ΠΌΠ΅ΡΠ΅Π½ΠΈΡ Π·Π°ΡΡΠ΄Π° β Π΅ΡΡΡ ΠΠΠ‘: 1 ΠΡΠ²ΠΎΠ΄ ΡΠΎΡΠΌΡΠ»Ρ
ΠΠ·ΠΎΠ±ΡΠ°ΠΆΠ΅Π½ΠΈΠ΅ ΡΠ»Π°ΠΉΠ΄Π°
6
Π‘Π»Π°ΠΉΠ΄ 6: Π‘ Π΄ΡΡΠ³ΠΎΠΉ ΡΡΠΎΡΠΎΠ½Ρ:
ΠΠ°Π³Π½ΠΈΡΠ½ΡΠΉ ΠΏΠΎΡΠΎΠΊ: ΠΠ·ΠΌΠ΅Π½Π΅Π½ΠΈΠ΅ ΠΌΠ°Π³Π½ΠΈΡΠ½ΠΎΠ³ΠΎ ΠΏΠΎΡΠΎΠΊΠ° ΠΌΠΎΠΆΠ΅Ρ ΠΏΡΠΎΠΈΠ·ΠΎΠΉΡΠΈ ΡΠΎΠ»ΡΠΊΠΎ Π·Π° ΡΡΠ΅Ρ ΡΠΌΠ΅Π½ΡΡΠ΅Π½ΠΈΡ ΠΏΠ»ΠΎΡΠ°Π΄ΠΈ ΠΊΠΎΠ½ΡΡΡΠ°: ΠΠ·ΠΌΠ΅Π½Π΅Π½ΠΈΠ΅ ΠΌΠ°Π³Π½ΠΈΡΠ½ΠΎΠ³ΠΎ ΠΏΠΎΡΠΎΠΊΠ°: ΠΠΎ Π·Π°ΠΊΠΎΠ½Ρ ΠΠΠ: 2 ΠΡΠ²ΠΎΠ΄ ΡΠΎΡΠΌΡΠ»Ρ
ΠΠ·ΠΎΠ±ΡΠ°ΠΆΠ΅Π½ΠΈΠ΅ ΡΠ»Π°ΠΉΠ΄Π°
7
Π‘Π»Π°ΠΉΠ΄ 7: ΠΡΠ²ΠΎΠ΄Ρ (ΠΊΠ°ΠΊΠΈΠ΅ Π·Π°ΠΊΠΎΠ½ΠΎΠΌΠ΅ΡΠ½ΠΎΡΡΠΈ Π·Π°ΠΌΠ΅ΡΠ΅Π½Ρ?):
ΠΠ²ΡΠΌΡ ΡΠ°Π·Π»ΠΈΡΠ½ΡΠΌΠΈ ΡΠΏΠΎΡΠΎΠ±Π°ΠΌΠΈ ΠΏΠΎΠ»ΡΡΠ΅Π½Π° ΠΎΠ΄Π½Π° ΠΈ ΡΠ° ΠΆΠ΅ ΡΠΎΡΠΌΡΠ»Π°: ΠΠΎ ΠΎΠΏΡΠ΅Π΄Π΅Π»Π΅Π½ΠΈΡ ΡΠΈΠ»Ρ ΠΠΎΡΠ΅Π½ΡΠ°, Π ΠΏΠΎ Π·Π°ΠΊΠΎΠ½Ρ ΡΠ»Π΅ΠΊΡΡΠΎΠΌΠ°Π³Π½ΠΈΡΠ½ΠΎΠΉ ΠΈΠ½Π΄ΡΠΊΡΠΈΠΈ. ΠΠΠ‘ ΠΈΠ½Π΄ΡΠΊΡΠΈΠΈ Π² Π΄Π²ΠΈΠΆΡΡΠ΅ΠΌΡΡ ΠΏΡΠΎΠ²ΠΎΠ΄Π½ΠΈΠΊΠ΅ ΡΠΈΡΠ»Π΅Π½Π½ΠΎ ΡΠΎΠ²ΠΏΠ°Π΄Π°Π΅Ρ Ρ ΡΠΈΠ»ΠΎΠΉ ΠΠΌΠΏΠ΅ΡΠ°. ΠΠ½Π°Π»ΠΎΠ³ΠΈΡΠ½ΡΠ΅ Π΄Π΅ΠΉΡΡΠ²ΠΈΡ Π±ΡΠ»ΠΈ ΠΏΡΠΎΠ²Π΅Π΄Π΅Π½Ρ ΡΠ°Π½Π΅Π΅ ΠΏΡΠΈ ΡΠ΅ΡΠ΅Π½ΠΈΠΈ Π·Π°Π΄Π°Ρ.
ΠΠ·ΠΎΠ±ΡΠ°ΠΆΠ΅Π½ΠΈΠ΅ ΡΠ»Π°ΠΉΠ΄Π°
8
Π‘Π»Π°ΠΉΠ΄ 8: Π‘ΠΠΠΠΠΠΠ£ΠΠ¦ΠΠ―
ΠΠ·ΠΎΠ±ΡΠ°ΠΆΠ΅Π½ΠΈΠ΅ ΡΠ»Π°ΠΉΠ΄Π°
9
Π‘Π»Π°ΠΉΠ΄ 9: Π―Π²Π»Π΅Π½ΠΈΠ΅ ΡΠ°ΠΌΠΎΠΈΠ½Π΄ΡΠΊΡΠΈΠΈ:
ΠΡΠΈ Π·Π°ΠΌΡΠΊΠ°Π½ΠΈΠΈ ΡΠ΅ΠΏΠΈ ΠΎΠΏΡΠ΅Π΄Π΅Π»Π΅Π½Π½ΠΎΠ΅ Π·Π½Π°ΡΠ΅Π½ΠΈΠ΅ ΡΠΈΠ»Ρ ΡΠΎΠΊΠ° ΡΠΎΠ·Π΄Π°Π΅ΡΡΡ Π½Π΅ ΡΡΠ°Π·Ρ, Π° ΠΏΠΎΡΡΠ΅ΠΏΠ΅Π½Π½ΠΎ Ρ ΡΠ΅ΡΠ΅Π½ΠΈΠ΅ΠΌ Π²ΡΠ΅ΠΌΠ΅Π½ΠΈ. ΠΡΠΈ ΡΠ°ΠΌΠΎΠΈΠ½Π΄ΡΠΊΡΠΈΠΈ ΠΏΡΠΎΠ²ΠΎΠ΄ΡΡΠΈΠΉ ΠΊΠΎΠ½ΡΡΡ ΠΈΠ³ΡΠ°Π΅Ρ Π΄Π²ΠΎΡΠΊΡΡ ΡΠΎΠ»Ρ: ΠΏΠΎ Π½Π΅ΠΌΡ ΠΏΡΠΎΡΠ΅ΠΊΠ°Π΅Ρ ΡΠΎΠΊ, Π²ΡΠ·ΡΠ²Π°ΡΡΠΈΠΉ ΠΈΠ½Π΄ΡΠΊΡΠΈΡ, ΠΈ Π² Π½Π΅ΠΌ ΠΆΠ΅ ΠΏΠΎΡΠ²Π»ΡΠ΅ΡΡΡ ΠΠΠ‘ ΠΈΠ½Π΄ΡΠΊΡΠΈΠΈ. ΠΠΎ ΠΏΡΠ°Π²ΠΈΠ»Ρ ΠΠ΅Π½ΡΠ°: Π² ΠΌΠΎΠΌΠ΅Π½Ρ Π½Π°ΡΠ°ΡΡΠ°Π½ΠΈΡ ΡΠΎΠΊΠ° Π² ΠΏΡΠΎΠ²ΠΎΠ΄Π½ΠΈΠΊΠ΅, Π² Π½Π΅ΠΌ ΠΆΠ΅ ΡΠΎΠ·Π΄Π°Π΅ΡΡΡ ΠΈΠ½Π΄ΡΠΊΡΠΈΠΎΠ½Π½ΡΠΉ ΡΠΎΠΊ (Ρ.Π΅. ΡΠΎΠ·Π΄Π°Π΅ΡΡΡ Π²ΠΈΡ ΡΠ΅Π²ΠΎΠ΅ ΡΠ»Π΅ΠΊΡΡΠΈΡΠ΅ΡΠΊΠΎΠ΅ ΠΏΠΎΠ»Π΅), ΠΏΡΠ΅ΠΏΡΡΡΡΠ²ΡΡΡΠΈΠΉ ΡΡΠΎΠΌΡ Π½Π°ΡΠ°ΡΡΠ°Π½ΠΈΡ ΡΠΎΠΊΠ°. ΠΡΠΈ ΡΠ±ΡΠ²Π°Π½ΠΈΠΈ ΡΠ»Π΅ΠΊΡΡΠΈΡΠ΅ΡΠΊΠΎΠ³ΠΎ ΡΠΎΠΊΠ° Π²ΠΈΡ ΡΠ΅Π²ΠΎΠ΅ ΠΏΠΎΠ»Π΅ ΡΡΠΎΠΌΡ ΠΏΡΠ΅ΠΏΡΡΡΡΠ²ΡΠ΅Ρ.
ΠΠ·ΠΎΠ±ΡΠ°ΠΆΠ΅Π½ΠΈΠ΅ ΡΠ»Π°ΠΉΠ΄Π°
10
Π‘Π»Π°ΠΉΠ΄ 10: ΠΠ°Π±Π»ΡΠ΄Π΅Π½ΠΈΠ΅ ΡΠ²Π»Π΅Π½ΠΈΡ ΡΠ°ΠΌΠΎΠΈΠ½Π΄ΡΠΊΡΠΈΠΈ Π½Π° ΠΎΠΏΡΡΠ°Ρ :
ΠΡΠΈ Π·Π°ΠΌΡΠΊΠ°Π½ΠΈΠΈ ΠΊΠ»ΡΡΠ° ΠΏΠ΅ΡΠ²Π°Ρ Π»Π°ΠΌΠΏΠ° Π²ΡΠΏΡΡ ΠΈΠ²Π°Π΅Ρ ΠΏΡΠ°ΠΊΡΠΈΡΠ΅ΡΠΊΠΈ ΡΡΠ°Π·Ρ, Π° Π²ΡΠΎΡΠ°Ρ β Ρ Π·Π°ΠΌΠ΅ΡΠ½ΡΠΌ ΠΎΠΏΠΎΠ·Π΄Π°Π½ΠΈΠ΅ΠΌ. ΠΠΎΠ·Π½ΠΈΠΊΠ°ΡΡΠ°Ρ ΠΠΠ‘ ΡΠ°ΠΌΠΎΠΈΠ½Π΄ΡΠΊΡΠΈΠΈ Π² ΠΊΠ°ΡΡΡΠΊΠ΅ Β«ΡΠΎΡΠΌΠΎΠ·ΠΈΡΒ» Π½Π°ΡΠ°ΡΡΠ°Π½ΠΈΠ΅ ΡΠΎΠΊΠ° Π² Π²Π΅ΡΠ²ΠΈ. ΠΡΠΈ ΡΠ°Π·ΠΌΡΠΊΠ°Π½ΠΈΠΈ ΠΊΠ»ΡΡΠ° Π² ΠΊΠ°ΡΡΡΠΊΠ΅ Π²ΠΎΠ·Π½ΠΈΠΊΠ°Π΅Ρ ΠΠΠ‘ ΡΠ°ΠΌΠΎΠΈΠ½Π΄ΡΠΊΡΠΈΠΈ, ΠΏΠΎΠ΄Π΄Π΅ΡΠΆΠΈΠ²Π°ΡΡΠ°Ρ ΠΏΠ΅ΡΠ²ΠΎΠ½Π°ΡΠ°Π»ΡΠ½ΡΠΉ ΡΠΎΠΊ, ΠΏΠΎΡΡΠΎΠΌΡ Π»Π°ΠΌΠΏΠ° Π³Π°ΡΠ½Π΅Ρ Π½Π΅ ΡΡΠ°Π·Ρ ΠΏΠΎΡΠ»Π΅ ΡΠ°Π·ΠΌΡΠΊΠ°Π½ΠΈΡ ΠΊΠ»ΡΡΠ°.
ΠΠ·ΠΎΠ±ΡΠ°ΠΆΠ΅Π½ΠΈΠ΅ ΡΠ»Π°ΠΉΠ΄Π°
11
Π‘Π»Π°ΠΉΠ΄ 11: 1- ΠΉ ΠΎΠΏΡΡ ΠΠ΅Π½ΡΠΈ
Π’ΠΎΠΊ Π·Π°ΠΌΡΠΊΠ°Π½ΠΈΡ. Π ΠΊΠ°ΡΡΡΠΊΠ΅ Π½Π°ΡΠ°ΡΡΠ°Π΅Ρ ΠΌΠ°Π³Π½ΠΈΡΠ½ΡΠΉ ΠΏΠΎΡΠΎΠΊ ΠΈ ΡΠ²Π»Π΅Π½ΠΈΠ΅ ΡΠ°ΠΌΠΎΠΈΠ½Π΄ΡΠΊΡΠΈΠΈ Β«ΡΠΎΡΠΌΠΎΠ·ΠΈΡΒ» Π½Π°ΡΠ°ΡΡΠ°Π½ΠΈΠ΅ ΡΠΎΠΊΠ° Π² ΡΠ΅ΠΏΠΈ.
ΠΠ·ΠΎΠ±ΡΠ°ΠΆΠ΅Π½ΠΈΠ΅ ΡΠ»Π°ΠΉΠ΄Π°
12
Π‘Π»Π°ΠΉΠ΄ 12: 2-ΠΉ ΠΎΠΏΡΡ ΠΠ΅Π½ΡΠΈ
Π’ΠΎΠΊ ΡΠ°Π·ΠΌΡΠΊΠ°Π½ΠΈΡ. ΠΠ°Π³Π½ΠΈΡΠ½ΡΠΉ ΠΏΠΎΡΠΎΠΊ ΡΠ±ΡΠ²Π°Π΅Ρ; ΡΠ²Π»Π΅Π½ΠΈΠ΅ ΡΠ°ΠΌΠΎΠΈΠ½Π΄ΡΠΊΡΠΈΠΈ Π½Π΅ΠΊΠΎΡΠΎΡΠΎΠ΅ Π²ΡΠ΅ΠΌΡ ΠΏΠΎΠ΄Π΄Π΅ΡΠΆΠΈΠ²Π°Π΅Ρ ΡΠΎΠΊ Π² ΡΠ΅ΠΏΠΈ.
ΠΠ·ΠΎΠ±ΡΠ°ΠΆΠ΅Π½ΠΈΠ΅ ΡΠ»Π°ΠΉΠ΄Π°
13
Π‘Π»Π°ΠΉΠ΄ 13: ΠΠ½Π°Π»ΠΎΠ³ΠΈΡ ΠΌΠ΅ΠΆΠ΄Ρ ΡΠ°ΠΌΠΎΠΈΠ½Π΄ΡΠΊΡΠΈΠ΅ΠΉ ΠΈ ΠΈΠ½Π΅ΡΡΠΈΠ΅ΠΉ:
ΠΠ°ΠΆΠ΄Π°Ρ ΠΊΠ°ΡΡΡΠΊΠ° ΠΈΠ½Π΄ΠΈΠ²ΠΈΠ΄ΡΠ°Π»ΡΠ½Π°, ΠΈΠΌΠ΅Ρ ΡΠ°Π·Π½ΠΎΠ΅ Ρ ΠΈΡΠ»ΠΎ Π²ΠΈΡΠΊΠΎΠ² ΠΈ ΡΠ°Π·Π½ΡΠΉ Π΄ΠΈΠ°ΠΌΠ΅ΡΡ Π±ΠΎΠ±ΠΈΠ½Ρ. Π§ΡΠΎ ΠΏΡΠΎΠΈΠ·ΠΎΠΉΠ΄Π΅Ρ, Π΅ΡΠ»ΠΈ Π² ΠΏΠ΅ΡΠ²ΠΎΠΌ ΡΠΊΡΠΏΠ΅ΡΠΈΠΌΠ΅Π½ΡΠ΅ Π·Π°ΠΌΠ΅Π½ΠΈΡΡ ΠΊΠ°ΡΡΡΠΊΡ Π½Π° Π΄ΡΡΠ³ΡΡ, Ρ Π±ΠΎΠ»ΡΡΠΈΠΌ ΡΠΈΡΠ»ΠΎΠΌ Π²ΠΈΡΠΊΠΎΠ²? ΠΠ°ΠΊΠΎΠΉ ΠΈΠ· Π°Π²ΡΠΎΠΌΠΎΠ±ΠΈΠ»Π΅ΠΉ Π»Π΅Π³ΡΠ΅ Β«ΡΠ°ΡΡΠΎΠ»ΠΊΠ°ΡΡΒ»? Π ΠΎΡΡΠ°Π½ΠΎΠ²ΠΈΡΡ? ΠΠ°ΠΊ Π½Π°Π·ΡΠ²Π°Π΅ΡΡΡ ΡΠ²Π»Π΅Π½ΠΈΠ΅ ΡΡΡΠ΅ΠΌΠ»Π΅Π½ΠΈΡ ΡΠ΅Π»Π° ΡΠΎΡ ΡΠ°Π½ΠΈΡΡ ΡΠ²ΠΎΡ ΡΠΊΠΎΡΠΎΡΡΡ Π² ΠΌΠ΅Ρ Π°Π½ΠΈΠΊΠ΅? Π‘Π°ΠΌΠΎΠΈΠ½Π΄ΡΠΊΡΠΈΡ Π°Π½Π°Π»ΠΎΠ³ΠΈΡΠ½Π° ΠΈΠ½Π΅ΡΡΠΈΠΈ: Π±ΠΎΠ»Π΅Π΅ Β«ΠΌΠΎΡΠ½Π°ΡΒ» ΠΊΠ°ΡΡΡΠΊΠ° ΡΠΈΠ»ΡΠ½Π΅Π΅ ΠΏΡΠ΅ΠΏΡΡΡΡΠ²ΡΠ΅Ρ Π½Π°ΡΠ°ΡΡΠ°Π½ΠΈΡ ΡΠΎΠΊΠ°; ΠΎΠ½Π° ΠΆΠ΅ ΠΏΠΎΡΠΎΠΌ, ΠΏΡΠΈ ΡΠ°Π·ΠΌΡΠΊΠ°Π½ΠΈΠΈ, Π΄ΠΎΠ»ΡΡΠ΅ Π΅Π³ΠΎ ΠΏΠΎΠ΄Π΄Π΅ΡΠΆΠΈΠ²Π°Π΅Ρ.
ΠΠ·ΠΎΠ±ΡΠ°ΠΆΠ΅Π½ΠΈΠ΅ ΡΠ»Π°ΠΉΠ΄Π°
14
Π‘Π»Π°ΠΉΠ΄ 14: ΠΠ½Π΄ΡΠΊΡΠΈΠ²Π½ΠΎΡΡΡ
ΠΡΠΎΡΠ»Π΅Π΄ΠΈΠΌ ΠΏΠΎΡΠ»Π΅Π΄ΠΎΠ²Π°ΡΠ΅Π»ΡΠ½ΠΎΡΡΡ: Π€ ~B~I. ΠΠΎΠΆΠ½ΠΎ ΡΡΠ²Π΅ΡΠΆΠ΄Π°ΡΡ, ΡΡΠΎ ΠΌΠ°Π³Π½ΠΈΡΠ½ΡΠΉ ΠΏΠΎΡΠΎΠΊ ΠΏΡΠΎΠΏΠΎΡΡΠΈΠΎΠ½Π°Π»Π΅Π½ ΡΠΈΠ»Π΅ ΡΠΎΠΊΠ°: Π€= LI, Π³Π΄Π΅ L β ΠΊΠΎΡΡΡΠΈΡΠΈΠ΅Π½Ρ ΠΏΡΠΎΠΏΠΎΡΡΠΈΠΎΠ½Π°Π»ΡΠ½ΠΎΡΡΠΈ; ΠΎΠ½ ΠΆΠ΅ β ΠΈΠ½Π΄ΡΠΊΡΠΈΠ²Π½ΠΎΡΡΡ ΠΊΠΎΠ½ΡΡΡΠ°; ΠΎΠ½ ΠΆΠ΅ β ΠΊΠΎΡΡΡΠΈΡΠΈΠ΅Π½Ρ ΡΠ°ΠΌΠΎΠΈΠ½Π΄ΡΠΊΡΠΈΠΈ. ΠΠ½Π΄ΡΠΊΡΠΈΠ²Π½ΠΎΡΡΡ β ΡΡΠΎ ΡΠΈΠ·ΠΈΡΠ΅ΡΠΊΠ°Ρ Π²Π΅Π»ΠΈΡΠΈΠ½Π°, ΡΠΈΡΠ»Π΅Π½Π½ΠΎ ΡΠ°Π²Π½Π°Ρ ΠΠΠ‘ ΡΠ°ΠΌΠΎΠΈΠ½Π΄ΡΠΊΡΠΈΠΈ, Π²ΠΎΠ·Π½ΠΈΠΊΠ°ΡΡΠ΅ΠΉ Π² ΠΊΠΎΠ½ΡΡΡΠ΅ ΠΏΡΠΈ ΠΈΠ·ΠΌΠ΅Π½Π΅Π½ΠΈΠΈ ΡΠΈΠ»Ρ ΡΠΎΠΊΠ° Π½Π° 1 Π Π·Π° 1 Ρ. ΠΠ΄ΠΈΠ½ΠΈΡΡ ΠΈΠ·ΠΌΠ΅ΡΠ΅Π½ΠΈΡ ΠΈΠ½Π΄ΡΠΊΡΠΈΠ²Π½ΠΎΡΡΠΈ β 1 ΠΠ΅Π½ΡΠΈ:
ΠΠ·ΠΎΠ±ΡΠ°ΠΆΠ΅Π½ΠΈΠ΅ ΡΠ»Π°ΠΉΠ΄Π°
15
Π‘Π»Π°ΠΉΠ΄ 15: ΠΠ½Π°Π»ΠΎΠ³ΠΈΡ ΠΌΠ΅ΠΆΠ΄Ρ ΠΌΠ΅Ρ Π°Π½ΠΈΠΊΠΎΠΉ ΠΈ ΡΠ»Π΅ΠΊΡΡΠΎΠ΄ΠΈΠ½Π°ΠΌΠΈΠΊΠΎΠΉ (Π½Π°ΡΠ°Π»ΠΎ ΡΠ°Π±Π»ΠΈΡΡ; ΠΏΡΠΎΠ΄ΠΎΠ»ΠΆΠ΅Π½ΠΈΠ΅ Π½Π° ΡΠ»Π΅Π΄ΡΡΡΠ΅ΠΌ ΡΡΠΎΠΊΠ΅)
ΠΌΠ΅Ρ Π°Π½ΠΈΠΊΠ° ΡΠ»Π΅ΠΊΡΡΠΎΠ΄ΠΈΠ½Π°ΠΌΠΈΠΊΠ° ΠΠ½Π΅ΡΡΠΈΡ Π‘Π°ΠΌΠΎΠΈΠ½Π΄ΡΠΊΡΠΈΡ ΠΠ°ΡΡΠ° m ΠΠ½Π΄ΡΠΊΡΠΈΠ²Π½ΠΎΡΡΡ L Π‘ΠΊΠΎΡΠΎΡΡΡ V Π‘ΠΈΠ»Π° ΡΠΎΠΊΠ° I
ΠΠ·ΠΎΠ±ΡΠ°ΠΆΠ΅Π½ΠΈΠ΅ ΡΠ»Π°ΠΉΠ΄Π°
16
Π‘Π»Π°ΠΉΠ΄ 16
ΠΠ·ΠΎΠ±ΡΠ°ΠΆΠ΅Π½ΠΈΠ΅ ΡΠ»Π°ΠΉΠ΄Π°
17
Π‘Π»Π°ΠΉΠ΄ 17: ΠΠ°Π΄Π°ΡΠ° 14:
ΠΠ°ΠΊΠ°Ρ ΠΠΠ‘ ΡΠ°ΠΌΠΎΠΈΠ½Π΄ΡΠΊΡΠΈΠΈ Π²ΠΎΠ·Π½ΠΈΠΊΠ°Π΅Ρ Π² ΠΊΠ°ΡΡΡΠΊΠ΅, Π΅ΡΠ»ΠΈ ΡΠΈΠ»Π° ΡΠΎΠΊΠ° Π² ΠΊΠ°ΡΡΡΠΊΠ΅ ΠΈΠ½Π΄ΡΠΊΡΠΈΠ²Π½ΠΎΡΡΡΡ 1 ΠΠ½ ΠΈΠ·ΠΌΠ΅Π½ΡΠ΅ΡΡΡ Ρ ΡΠ΅ΡΠ΅Π½ΠΈΠ΅ΠΌ Π²ΡΠ΅ΠΌΠ΅Π½ΠΈ, ΠΊΠ°ΠΊ ΠΏΠΎΠΊΠ°Π·Π°Π½ΠΎ Π½Π° Π³ΡΠ°ΡΠΈΠΊΠ΅. ΠΡΠΈΠΌΠ΅Π½ΡΠ΅ΡΡΡ Π·Π°ΠΊΠΎΠ½ ΠΠΠ‘ ΡΠ°ΠΌΠΎΠΈΠ½Π΄ΡΠΊΡΠΈΠΈ. ΠΠΎ Π³ΡΠ°ΡΠΈΠΊΡ ΠΎΠΏΡΠ΅Π΄Π΅Π»ΡΠ΅ΡΡΡ ΡΠΈΠ»Π° ΡΠΎΠΊΠ° ΠΈ Π²ΡΠ΅ΠΌΡ Π΅Π³ΠΎ ΠΏΡΠΎΡΠ΅ΠΊΠ°Π½ΠΈΡ ΠΡΠ²Π΅Ρ:10 Π
ΠΠ·ΠΎΠ±ΡΠ°ΠΆΠ΅Π½ΠΈΠ΅ ΡΠ»Π°ΠΉΠ΄Π°
18
Π‘Π»Π°ΠΉΠ΄ 18: ΠΠ°Π΄Π°ΡΠ° 15:
ΠΠ° ΡΠΈΡΡΠ½ΠΊΠ΅ ΠΏΡΠΈΠ²Π΅Π΄Π΅Π½ Π³ΡΠ°ΡΠΈΠΊ Π·Π°Π²ΠΈΡΠΈΠΌΠΎΡΡΠΈ ΡΠΈΠ»Ρ ΡΠΎΠΊΠ° Π² ΠΊΠ°ΡΡΡΠΊΠ΅ ΠΈΠ½Π΄ΡΠΊΡΠΈΠ²Π½ΠΎΡΡΠΈ ΠΎΡ Π²ΡΠ΅ΠΌΠ΅Π½ΠΈ. Π ΠΊΠ°ΠΊΠΎΠΌ ΠΏΡΠΎΠΌΠ΅ΠΆΡΡΠΊΠ΅ ΠΠΠ‘ ΡΠ°ΠΌΠΎΠΈΠ½Π΄ΡΠΊΡΠΈΠΈ ΠΏΡΠΈΠ½ΠΈΠΌΠ°Π΅Ρ Π½Π°ΠΈΠΌΠ΅Π½ΡΡΠ΅Π΅ Π·Π½Π°ΡΠ΅Π½ΠΈΠ΅? Π§Π΅ΠΌ ΠΌΠ΅Π½ΡΡΠ΅ ΡΠΊΠΎΡΠΎΡΡΡ ΠΈΠ·ΠΌΠ΅Π½Π΅Π½ΠΈΡ ΡΠΎΠΊΠ°, ΡΠ΅ΠΌ ΠΌΠ΅Π½ΡΡΠ΅ ΠΠΠ‘ ΡΠ°ΠΌΠΎΠΈΠ½Π΄ΡΠΊΡΠΈΠΈ. Π‘Π°ΠΌΠ°Ρ ΠΌΠ°Π»Π΅Π½ΡΠΊΠ°Ρ ΡΠΊΠΎΡΠΎΡΡΡ ΠΈΠ·ΠΌΠ΅Π½Π΅Π½ΠΈΡ ΡΠΎΠΊΠ° Π·Π° ΠΏΡΠΎΠΌΠ΅ΠΆΡΡΠΎΠΊ Π²ΡΠ΅ΠΌΠ΅Π½ΠΈ ΠΎΡ 1 Π΄ΠΎ 5 Ρ (Π½Π°ΠΈΠΌΠ΅Π½ΡΡΠΈΠΉ ΡΠ³ΠΎΠ» Π½Π°ΠΊΠ»ΠΎΠ½Π° ΡΡΠ°ΡΡΠΊΠ° Π³ΡΠ°ΡΠΈΠΊΠ°)
ΠΠ·ΠΎΠ±ΡΠ°ΠΆΠ΅Π½ΠΈΠ΅ ΡΠ»Π°ΠΉΠ΄Π°
19
Π‘Π»Π°ΠΉΠ΄ 19: ΠΠ°Π΄Π°ΡΠ° 16:
1) 0-1 Ρ ΠΈ 2-3 Ρ 2) 1-2 Ρ ΠΈ 2-3 Ρ 3) 0-1 Ρ ΠΈ 3-4 Ρ 4) 2-3 Ρ ΠΈ 3-4-Ρ ΠΡΠ²Π΅Ρ: 2), Ρ.ΠΊ. ΠΈΠΌΠ΅Π½Π½ΠΎ Π² ΡΡΠΎΠΌ ΠΏΡΠΎΠΌΠ΅ΠΆΡΡΠΊΠ΅ Π²ΡΠ΅ΠΌΠ΅Π½ΠΈ ΡΠΈΠ»Π° ΡΠΎΠΊΠ° ΠΈΠ·ΠΌΠ΅Π½ΡΠ΅ΡΡΡ ΠΌΠ΅Π΄Π»Π΅Π½Π½ΠΎ. ΠΠ° ΡΠΈΡΡΠ½ΠΊΠ΅ ΠΏΠΎΠΊΠ°Π·Π°Π½ Π³ΡΠ°ΡΠΈΠΊ ΠΈΠ·ΠΌΠ΅Π½Π΅Π½ΠΈΡ ΡΠΈΠ»Ρ ΡΠΎΠΊΠ° Π² ΠΊΠ°ΡΡΡΠΊΠ΅ ΠΈΠ½Π΄ΡΠΊΡΠΈΠ²Π½ΠΎΡΡΠΈ Ρ ΡΠ΅ΡΠ΅Π½ΠΈΠ΅ΠΌ Π²ΡΠ΅ΠΌΠ΅Π½ΠΈ. Π ΠΊΠ°ΠΊΠΎΠΌ ΠΏΡΠΎΠΌΠ΅ΠΆΡΡΠΊΠ΅ Π²ΡΠ΅ΠΌΠ΅Π½ΠΈ ΠΌΠΎΠ΄ΡΠ»Ρ ΠΠΠ‘ ΡΠ°ΠΌΠΎΠΈΠ½Π΄ΡΠΊΡΠΈΠΈ ΠΏΡΠΈΠ½ΠΈΠΌΠ°Π΅Ρ Π½Π°ΠΈΠΌΠ΅Π½ΡΡΠΈΠ΅ Π·Π½Π°ΡΠ΅Π½ΠΈΡ?
ΠΠ·ΠΎΠ±ΡΠ°ΠΆΠ΅Π½ΠΈΠ΅ ΡΠ»Π°ΠΉΠ΄Π°
20
Π‘Π»Π°ΠΉΠ΄ 20: ΠΠ°Π΄Π°ΡΠ° 17:
ΠΠ° ΡΠΈΡΡΠ½ΠΊΠ΅ ΠΏΡΠ΅Π΄ΡΡΠ°Π²Π»Π΅Π½ Π³ΡΠ°ΡΠΈΠΊ ΠΈΠ·ΠΌΠ΅Π½Π΅Π½ΠΈΡ ΡΠΈΠ»Ρ ΡΠΎΠΊΠ° Ρ ΡΠ΅ΡΠ΅Π½ΠΈΠ΅ΠΌ Π²ΡΠ΅ΠΌΠ΅Π½ΠΈ Π² ΠΊΠ°ΡΡΡΠΊΠ΅ ΠΈΠ½Π΄ΡΠΊΡΠΈΠ²Π½ΠΎΡΡΡΡ 6 ΠΌΠΠ½. Π§Π΅ΠΌΡ ΡΠ°Π²Π½Π° ΠΠΠ‘ ΡΠ°ΠΌΠΎΠΈΠ½Π΄ΡΠΊΡΠΈΠΈ? ΠΡΠΏΠΎΠ»ΡΠ·ΡΠ΅ΡΡΡ Π·Π°ΠΊΠΎΠ½ ΠΠΠ‘ ΡΠ°ΠΌΠΎΠΈΠ½Π΄ΡΠΊΡΠΈΠΈ. ΠΡΠ±ΠΈΡΠ°Π΅ΡΡΡ ΠΈΠ·ΠΌΠ΅Π½Π΅Π½ΠΈΠ΅ ΡΠΈΠ»Ρ ΡΠΎΠΊΠ° ΠΈ ΡΠΎΠΎΡΠ²Π΅ΡΡΡΠ²ΡΡΡΠΈΠΉ Π΅ΠΌΡ ΠΏΡΠΎΠΌΠ΅ΠΆΡΡΠΎΠΊ Π²ΡΠ΅ΠΌΠ΅Π½ΠΈ: 6β10-3 ΠΠ½ β 3 Π/2 Ρ ΠΡΠ²Π΅Ρ: 9 ΠΌΠ
ΠΠ·ΠΎΠ±ΡΠ°ΠΆΠ΅Π½ΠΈΠ΅ ΡΠ»Π°ΠΉΠ΄Π°
21
Π‘Π»Π°ΠΉΠ΄ 21: ΠΠ°Π΄Π°ΡΠΈ 18, 19:
Π ΠΏΡΠΎΠ²ΠΎΠ΄Π½ΠΈΠΊΠ΅ ΠΈΠ½Π΄ΡΠΊΡΠΈΠ²Π½ΠΎΡΡΡΡ 5 ΠΌΠΠ½ ΡΠΈΠ»Π° ΡΠΎΠΊΠ° Π² ΡΠ΅ΡΠ΅Π½ΠΈΠ΅ 0,2 Ρ ΡΠ°Π²Π½ΠΎΠΌΠ΅ΡΠ½ΠΎ Π²ΠΎΠ·ΡΠ°ΡΡΠ°Π΅Ρ Ρ 2 Π Π΄ΠΎ ΠΊΠ°ΠΊΠΎΠ³ΠΎ-ΡΠΎ ΠΊΠΎΠ½Π΅ΡΠ½ΠΎΠ³ΠΎ Π·Π½Π°ΡΠ΅Π½ΠΈΡ. ΠΡΠΈ ΡΡΠΎΠΌ Π² ΠΏΡΠΎΠ²ΠΎΠ΄Π½ΠΈΠΊΠ΅ Π²ΠΎΠ·Π½ΠΈΠΊΠ°Π΅Ρ ΠΠΠ‘ ΡΠ°ΠΌΠΎΠΈΠ½Π΄ΡΠΊΡΠΈΠΈ 0,2 Π. ΠΠΏΡΠ΅Π΄Π΅Π»ΠΈΡΠ΅ ΠΊΠΎΠ½Π΅ΡΠ½ΠΎΠ΅ Π·Π½Π°ΡΠ΅Π½ΠΈΠ΅ ΡΠΈΠ»Ρ ΡΠΎΠΊΠ° Π² ΠΏΡΠΎΠ²ΠΎΠ΄Π½ΠΈΠΊΠ΅. ΠΡΠ²Π΅Ρ: 10 Π ΠΠ½Π΄ΡΠΊΡΠΈΠ²Π½ΠΎΡΡΡ ΠΊΠ°ΡΡΡΠΊΠΈ ΡΠ²Π΅Π»ΠΈΡΠΈΠ»ΠΈ Π² 2 ΡΠ°Π·Π°, Π° ΡΠΊΠΎΡΠΎΡΡΡ ΠΈΠ·ΠΌΠ΅Π½Π΅Π½ΠΈΡ ΡΠΎΠΊΠ° ΡΠΌΠ΅Π½ΡΡΠΈΠ»ΠΈ Π² 4 ΡΠ°Π·Π°. ΠΠ°ΠΊ ΠΈΠ·ΠΌΠ΅Π½ΠΈΠ»Π°ΡΡ ΠΠΠ‘ ΡΠ°ΠΌΠΎΠΈΠ½Π΄ΡΠΊΡΠΈΠΈ? ΠΡΠ²Π΅Ρ: ΠΠΠ‘ ΡΠ°ΠΌΠΎΠΈΠ½Π΄ΡΠΊΡΠΈΠΈ ΡΠΌΠ΅Π½ΡΡΠΈΠ»Π°ΡΡ Π² 2 ΡΠ°Π·Π°.
ΠΠ·ΠΎΠ±ΡΠ°ΠΆΠ΅Π½ΠΈΠ΅ ΡΠ»Π°ΠΉΠ΄Π°
22
Π‘Π»Π°ΠΉΠ΄ 22: ΠΠ°Π΄Π°ΡΠ° 20
ΠΡΡΠΌΠΎΠ»ΠΈΠ½Π΅ΠΉΠ½ΡΠΉ ΠΏΡΠΎΠ²ΠΎΠ΄Π½ΠΈΠΊ Π΄Π²ΠΈΠΆΠ΅ΡΡΡ ΡΠΎ ΡΠΊΠΎΡΠΎΡΡΡΡ 25 ΠΌ/Ρ Π² ΠΏΠΎΠ»Π΅ Ρ ΠΈΠ½Π΄ΡΠΊΡΠΈΠ΅ΠΉ 0,0038 Π’Π» ΠΏΠ΅ΡΠΏΠ΅Π½Π΄ΠΈΠΊΡΠ»ΡΡΠ½ΠΎ ΡΠΈΠ»ΠΎΠ²ΡΠΌ Π»ΠΈΠ½ΠΈΡΠΌ. Π§Π΅ΠΌΡ ΡΠ°Π²Π½Π° Π΄Π»ΠΈΠ½Π° ΠΏΡΠΎΠ²ΠΎΠ΄Π½ΠΈΠΊΠ°, Π΅ΡΠ»ΠΈ Π½Π° Π΅Π³ΠΎ ΠΊΠΎΠ½ΡΠ°Ρ ΠΈΠΌΠ΅Π΅ΡΡΡ Π½Π°ΠΏΡΡΠΆΠ΅Π½ΠΈΠ΅ 0,028 Π? ΠΡΠ²Π΅Ρ: 29 ΡΠΌ
ΠΠ·ΠΎΠ±ΡΠ°ΠΆΠ΅Π½ΠΈΠ΅ ΡΠ»Π°ΠΉΠ΄Π°
23
Π‘Π»Π°ΠΉΠ΄ 23: ΠΠ°Π΄Π°ΡΠ° 21
ΠΠΈΡΠΎΠΊ ΠΏΠ»ΠΎΡΠ°Π΄ΡΡ 100 ΡΠΌ 2 Π½Π°Ρ ΠΎΠ΄ΠΈΡΡΡ Π² ΠΌΠ°Π³Π½ΠΈΡΠ½ΠΎΠΌ ΠΏΠΎΠ»Π΅ Ρ ΠΈΠ½Π΄ΡΠΊΡΠΈΠ΅ΠΉ 1 Π’Π». ΠΠ»ΠΎΡΠΊΠΎΡΡΡ Π²ΠΈΡΠΊΠ° ΠΏΠ΅ΡΠΏΠ΅Π½Π΄ΠΈΠΊΡΠ»ΡΡΠ½Π° Π»ΠΈΠ½ΠΈΡΠΌ ΠΏΠΎΠ»Ρ. ΠΠΏΡΠ΅Π΄Π΅Π»ΠΈΡΠ΅ ΡΡΠ΅Π΄Π½Π΅Π΅ Π·Π½Π°ΡΠ΅Π½ΠΈΠ΅ ΠΠΠ‘ ΠΈΠ½Π΄ΡΠΊΡΠΈΠΈ ΠΏΡΠΈ Π²ΡΠΊΠ»ΡΡΠ΅Π½ΠΈΠΈ ΠΏΠΎΠ»Ρ Π·Π° 0,01 Ρ. ΠΡΠ²Π΅Ρ: 1 Π
ΠΠ·ΠΎΠ±ΡΠ°ΠΆΠ΅Π½ΠΈΠ΅ ΡΠ»Π°ΠΉΠ΄Π°
24
Π‘Π»Π°ΠΉΠ΄ 24: ΠΠ°Π΄Π°ΡΠ° 22
ΠΡΡΠΌΠΎΠ»ΠΈΠ½Π΅ΠΉΠ½ΡΠΉ ΠΏΡΠΎΠ²ΠΎΠ΄Π½ΠΈΠΊ Π΄Π»ΠΈΠ½ΠΎΠΉ 120 ΡΠΌ Π΄Π²ΠΈΠΆΠ΅ΡΡΡ Π² ΠΎΠ΄Π½ΠΎΡΠΎΠ΄Π½ΠΎΠΌ ΠΌΠ°Π³Π½ΠΈΡΠ½ΠΎΠΌ ΠΏΠΎΠ»Π΅ ΠΏΠΎΠ΄ ΡΠ³Π»ΠΎΠΌ 90Β° ΠΊ ΡΠΈΠ»ΠΎΠ²ΡΠΌ Π»ΠΈΠ½ΠΈΡΠΌ ΡΠΎ ΡΠΊΠΎΡΠΎΡΡΡΡ 15 ΠΌ/Ρ. ΠΠΏΡΠ΅Π΄Π΅Π»ΠΈΡΠ΅ ΠΈΠ½Π΄ΡΠΊΡΠΈΡ ΠΏΠΎΠ»Ρ, Π΅ΡΠ»ΠΈ Π² ΠΏΡΠΎΠ²ΠΎΠ΄Π½ΠΈΠΊΠ΅ ΡΠΎΠ·Π΄Π°Π΅ΡΡΡ ΠΠΠ‘ ΠΈΠ½Π΄ΡΠΊΡΠΈΠΈ 0,12 Π. ΠΡΠ²Π΅Ρ: 6,67 ΠΌΠ’Π»
ΠΠ·ΠΎΠ±ΡΠ°ΠΆΠ΅Π½ΠΈΠ΅ ΡΠ»Π°ΠΉΠ΄Π°
25
Π‘Π»Π°ΠΉΠ΄ 25: ΠΠ°Π΄Π°ΡΠ° 23
ΠΠ°ΠΉΠ΄ΠΈΡΠ΅ ΠΈΠ½Π΄ΡΠΊΡΠΈΠ²Π½ΠΎΡΡΡ ΠΏΡΠΎΠ²ΠΎΠ΄Π½ΠΈΠΊΠ°, Π² ΠΊΠΎΡΠΎΡΠΎΠΌ ΡΠ°Π²Π½ΠΎΠΌΠ΅ΡΠ½ΠΎΠ΅ ΠΈΠ·ΠΌΠ΅Π½Π΅Π½ΠΈΠ΅ ΡΠΈΠ»Ρ ΡΠΎΠΊΠ° Π½Π° 2 Π Π² ΡΠ΅ΡΠ΅Π½ΠΈΠ΅ 0,25 Ρ Π²ΠΎΠ·Π±ΡΠΆΠ΄Π°Π΅Ρ ΠΠΠ‘ ΡΠ°ΠΌΠΎΠΈΠ½Π΄ΡΠΊΡΠΈΠΈ 20 ΠΌΠ. ΠΡΠ²Π΅Ρ: 2,5 ΠΌΠΠ½
ΠΠ·ΠΎΠ±ΡΠ°ΠΆΠ΅Π½ΠΈΠ΅ ΡΠ»Π°ΠΉΠ΄Π°
26
Π‘Π»Π°ΠΉΠ΄ 26: ΠΠ°Π΄Π°ΡΠ° 24
Π‘Π°ΠΌΠΎΠ»Π΅Ρ Π»Π΅ΡΠΈΡ Π³ΠΎΡΠΈΠ·ΠΎΠ½ΡΠ°Π»ΡΠ½ΠΎ ΡΠΎ ΡΠΊΠΎΡΠΎΡΡΡΡ 900 ΠΊΠΌ/Ρ. ΠΠ°ΠΉΠ΄ΠΈΡΠ΅ ΡΠ°Π·Π½ΠΎΡΡΡ ΠΏΠΎΡΠ΅Π½ΡΠΈΠ°Π»ΠΎΠ², Π²ΠΎΠ·Π½ΠΈΠΊΠ°ΡΡΡΡ ΠΌΠ΅ΠΆΠ΄Ρ ΠΊΠΎΠ½ΡΠ°ΠΌΠΈ ΠΊΡΡΠ»ΡΠ΅Π² ΡΠ°ΠΌΠΎΠ»Π΅ΡΠ°, Π΅ΡΠ»ΠΈ Π²Π΅ΡΡΠΈΠΊΠ°Π»ΡΠ½Π°Ρ ΡΠΎΡΡΠ°Π²Π»ΡΡΡΠ°Ρ Π·Π΅ΠΌΠ½ΠΎΠ³ΠΎ ΠΌΠ°Π³Π½ΠΈΡΠ½ΠΎΠ³ΠΎ ΠΏΠΎΠ»Ρ ΡΠ°Π²Π½Π° 50 ΠΌΠΊΠ’Π» ΠΈ ΡΠ°Π·ΠΌΠ°Ρ ΠΊΡΡΠ»ΡΠ΅Π² 12 ΠΌ. ΠΠΎΠ΄ΡΠΊΠ°Π·ΠΊΠ°: ΠΈΡΠΏΠΎΠ»ΡΠ·ΡΠ΅ΡΡΡ ΡΠΎΡΠΌΡΠ»Π° ΠΠΠ‘ ΠΈΠ½Π΄ΡΠΊΡΠΈΠΈ Π² Π΄Π²ΠΈΠΆΡΡΠΈΡ ΡΡ ΠΏΡΠΎΠ²ΠΎΠ΄Π½ΠΈΠΊΠ°Ρ , Π³Π΄Π΅ ΡΠ°Π·ΠΌΠ°Ρ ΠΊΡΡΠ»ΡΠ΅Π² β ΡΡΠΎ Π΄Π»ΠΈΠ½Π° ΠΏΡΠΎΠ²ΠΎΠ΄Π½ΠΈΠΊΠ°. ΠΡΠ²Π΅Ρ: 0,15 Π
ΠΠ·ΠΎΠ±ΡΠ°ΠΆΠ΅Π½ΠΈΠ΅ ΡΠ»Π°ΠΉΠ΄Π°
27
Π‘Π»Π°ΠΉΠ΄ 27: ΠΠ°Π΄Π°ΡΠ° 25
Π‘ΠΊΠΎΠ»ΡΠΊΠΎ Π²ΠΈΡΠΊΠΎΠ² Π΄ΠΎΠ»ΠΆΠ½Π° ΠΈΠΌΠ΅ΡΡ ΠΊΠ°ΡΡΡΠΊΠ°, ΡΡΠΎΠ±Ρ ΠΏΡΠΈ ΠΈΠ·ΠΌΠ΅Π½Π΅Π½ΠΈΠΈ ΠΌΠ°Π³Π½ΠΈΡΠ½ΠΎΠ³ΠΎ ΠΏΠΎΡΠΎΠΊΠ° Π²Π½ΡΡΡΠΈ Π½Π΅Π΅ ΠΎΡ 0,024 ΠΠ± Π΄ΠΎ 0,056 ΠΠ± Π·Π° ΠΏΡΠΎΠΌΠ΅ΠΆΡΡΠΎΠΊ Π²ΡΠ΅ΠΌΠ΅Π½ΠΈ 0,32 Ρ Π² ΠΊΠ°ΡΡΡΠΊΠ΅ Π²ΠΎΠ·Π½ΠΈΠΊΠ°Π»Π° ΡΡΠ΅Π΄Π½ΡΡ ΠΠΠ‘ ΠΈΠ½Π΄ΡΠΊΡΠΈΠΈ 10 Π? ΠΡΠ²Π΅Ρ: 100 Π²ΠΈΡΠΊΠΎΠ²
ΠΠ·ΠΎΠ±ΡΠ°ΠΆΠ΅Π½ΠΈΠ΅ ΡΠ»Π°ΠΉΠ΄Π°
28
ΠΠΎΡΠ»Π΅Π΄Π½ΠΈΠΉ ΡΠ»Π°ΠΉΠ΄ ΠΏΡΠ΅Π·Π΅Π½ΡΠ°ΡΠΈΠΈ: 17.04.2020 Π³. ΠΠΠ‘ ΠΈΠ½Π΄ΡΠΊΡΠΈΠΈ Π² Π΄Π²ΠΈΠΆΡΡΠΈΡ ΡΡ ΠΏΡΠΎΠ²ΠΎΠ΄Π½ΠΈΠΊΠ°Ρ . Π‘Π°ΠΌΠΎΠΈΠ½Π΄ΡΠΊΡΠΈΡ
ΠΠ·ΠΎΠ±ΡΠ°ΠΆΠ΅Π½ΠΈΠ΅ ΡΠ»Π°ΠΉΠ΄Π°
ΠΠ°ΠΊΠΎΠ½ ΡΠ»Π΅ΠΊΡΡΠΎΠΌΠ°Π³Π½ΠΈΡΠ½ΠΎΠΉ ΠΈΠ½Π΄ΡΠΊΡΠΈΠΈ — Chip Stock
ΠΠ»Π΅ΠΊΡΡΠΎΠΌΠ°Π³Π½ΠΈΡΠ½Π°Ρ ΠΈΠ½Π΄ΡΠΊΡΠΈΡ β FIZI4KA
ΠΠΠ 2018 ΠΏΠΎ ΡΠΈΠ·ΠΈΠΊΠ΅ βΊ
ΠΠ»Π΅ΠΊΡΡΠΎΠΌΠ°Π³Π½ΠΈΡΠ½Π°Ρ ΠΈΠ½Π΄ΡΠΊΡΠΈΡ β ΡΠ²Π»Π΅Π½ΠΈΠ΅ Π²ΠΎΠ·Π½ΠΈΠΊΠ½ΠΎΠ²Π΅Π½ΠΈΡ ΡΠΎΠΊΠ° Π² Π·Π°ΠΌΠΊΠ½ΡΡΠΎΠΌ ΠΏΡΠΎΠ²ΠΎΠ΄ΡΡΠ΅ΠΌ ΠΊΠΎΠ½ΡΡΡΠ΅ ΠΏΡΠΈ ΠΈΠ·ΠΌΠ΅Π½Π΅Π½ΠΈΠΈ ΠΌΠ°Π³Π½ΠΈΡΠ½ΠΎΠ³ΠΎ ΠΏΠΎΡΠΎΠΊΠ°, ΠΏΡΠΎΠ½ΠΈΠ·ΡΠ²Π°ΡΡΠ΅Π³ΠΎ Π΅Π³ΠΎ.
Π―Π²Π»Π΅Π½ΠΈΠ΅ ΡΠ»Π΅ΠΊΡΡΠΎΠΌΠ°Π³Π½ΠΈΡΠ½ΠΎΠΉ ΠΈΠ½Π΄ΡΠΊΡΠΈΠΈ Π±ΡΠ»ΠΎ ΠΎΡΠΊΡΡΡΠΎ Π. Π€Π°ΡΠ°Π΄Π΅Π΅ΠΌ.
ΠΠΏΡΡΡ Π€Π°ΡΠ°Π΄Π΅Ρ
- ΠΠ° ΠΎΠ΄Π½Ρ Π½Π΅ΠΏΡΠΎΠ²ΠΎΠ΄ΡΡΡΡ ΠΎΡΠ½ΠΎΠ²Ρ Π±ΡΠ»ΠΈ Π½Π°ΠΌΠΎΡΠ°Π½Ρ Π΄Π²Π΅ ΠΊΠ°ΡΡΡΠΊΠΈ: Π²ΠΈΡΠΊΠΈ ΠΏΠ΅ΡΠ²ΠΎΠΉ ΠΊΠ°ΡΡΡΠΊΠΈ Π±ΡΠ»ΠΈ ΡΠ°ΡΠΏΠΎΠ»ΠΎΠΆΠ΅Π½Ρ ΠΌΠ΅ΠΆΠ΄Ρ Π²ΠΈΡΠΊΠ°ΠΌΠΈ Π²ΡΠΎΡΠΎΠΉ. ΠΠΈΡΠΊΠΈ ΠΎΠ΄Π½ΠΎΠΉ ΠΊΠ°ΡΡΡΠΊΠΈ Π±ΡΠ»ΠΈ Π·Π°ΠΌΠΊΠ½ΡΡΡ Π½Π° Π³Π°Π»ΡΠ²Π°Π½ΠΎΠΌΠ΅ΡΡ, Π° Π²ΡΠΎΡΠΎΠΉ β ΠΏΠΎΠ΄ΠΊΠ»ΡΡΠ΅Π½Ρ ΠΊ ΠΈΡΡΠΎΡΠ½ΠΈΠΊΡ ΡΠΎΠΊΠ°. ΠΡΠΈ Π·Π°ΠΌΡΠΊΠ°Π½ΠΈΠΈ ΠΊΠ»ΡΡΠ° ΠΈ ΠΏΡΠΎΡΠ΅ΠΊΠ°Π½ΠΈΠΈ ΡΠΎΠΊΠ° ΠΏΠΎ Π²ΡΠΎΡΠΎΠΉ ΠΊΠ°ΡΡΡΠΊΠ΅ Π² ΠΏΠ΅ΡΠ²ΠΎΠΉ Π²ΠΎΠ·Π½ΠΈΠΊΠ°Π» ΠΈΠΌΠΏΡΠ»ΡΡ ΡΠΎΠΊΠ°. ΠΡΠΈ ΡΠ°Π·ΠΌΡΠΊΠ°Π½ΠΈΠΈ ΠΊΠ»ΡΡΠ° ΡΠ°ΠΊΠΆΠ΅ Π½Π°Π±Π»ΡΠ΄Π°Π»ΡΡ ΠΈΠΌΠΏΡΠ»ΡΡ ΡΠΎΠΊΠ°, Π½ΠΎ ΡΠΎΠΊ ΡΠ΅ΡΠ΅Π· Π³Π°Π»ΡΠ²Π°Π½ΠΎΠΌΠ΅ΡΡ ΡΠ΅ΠΊ Π² ΠΏΡΠΎΡΠΈΠ²ΠΎΠΏΠΎΠ»ΠΎΠΆΠ½ΠΎΠΌ Π½Π°ΠΏΡΠ°Π²Π»Π΅Π½ΠΈΠΈ.
- ΠΠ΅ΡΠ²Π°Ρ ΠΊΠ°ΡΡΡΠΊΠ° Π±ΡΠ»Π° ΠΏΠΎΠ΄ΠΊΠ»ΡΡΠ΅Π½Π° ΠΊ ΠΈΡΡΠΎΡΠ½ΠΈΠΊΡ ΡΠΎΠΊΠ°, Π²ΡΠΎΡΠ°Ρ, ΠΏΠΎΠ΄ΠΊΠ»ΡΡΠ΅Π½Π½Π°Ρ ΠΊ Π³Π°Π»ΡΠ²Π°Π½ΠΎΠΌΠ΅ΡΡΡ, ΠΏΠ΅ΡΠ΅ΠΌΠ΅ΡΠ°Π»Π°ΡΡ ΠΎΡΠ½ΠΎΡΠΈΡΠ΅Π»ΡΠ½ΠΎ Π½Π΅Π΅. ΠΡΠΈ ΠΏΡΠΈΠ±Π»ΠΈΠΆΠ΅Π½ΠΈΠΈ ΠΈΠ»ΠΈ ΡΠ΄Π°Π»Π΅Π½ΠΈΠΈ ΠΊΠ°ΡΡΡΠΊΠΈ ΡΠΈΠΊΡΠΈΡΠΎΠ²Π°Π»ΡΡ ΡΠΎΠΊ.
- ΠΠ°ΡΡΡΠΊΠ° Π·Π°ΠΌΠΊΠ½ΡΡΠ° Π½Π° Π³Π°Π»ΡΠ²Π°Π½ΠΎΠΌΠ΅ΡΡ, Π° ΠΌΠ°Π³Π½ΠΈΡ Π΄Π²ΠΈΠΆΠ΅ΡΡΡ β Π²Π΄Π²ΠΈΠ³Π°Π΅ΡΡΡ (Π²ΡΠ΄Π²ΠΈΠ³Π°Π΅ΡΡΡ) β ΠΎΡΠ½ΠΎΡΠΈΡΠ΅Π»ΡΠ½ΠΎ ΠΊΠ°ΡΡΡΠΊΠΈ.
ΠΠΏΡΡΡ ΠΏΠΎΠΊΠ°Π·Π°Π»ΠΈ, ΡΡΠΎ ΠΈΠ½Π΄ΡΠΊΡΠΈΠΎΠ½Π½ΡΠΉ ΡΠΎΠΊ Π²ΠΎΠ·Π½ΠΈΠΊΠ°Π΅Ρ ΡΠΎΠ»ΡΠΊΠΎ ΠΏΡΠΈ ΠΈΠ·ΠΌΠ΅Π½Π΅Π½ΠΈΠΈ Π»ΠΈΠ½ΠΈΠΉ ΠΌΠ°Π³Π½ΠΈΡΠ½ΠΎΠΉ ΠΈΠ½Π΄ΡΠΊΡΠΈΠΈ. ΠΠ°ΠΏΡΠ°Π²Π»Π΅Π½ΠΈΠ΅ ΡΠΎΠΊΠ° Π±ΡΠ΄Π΅Ρ ΡΠ°Π·Π»ΠΈΡΠ½ΠΎ ΠΏΡΠΈ ΡΠ²Π΅Π»ΠΈΡΠ΅Π½ΠΈΠΈ ΡΠΈΡΠ»Π° Π»ΠΈΠ½ΠΈΠΉ ΠΈ ΠΏΡΠΈ ΠΈΡ ΡΠΌΠ΅Π½ΡΡΠ΅Π½ΠΈΠΈ.
Π‘ΠΈΠ»Π° ΠΈΠ½Π΄ΡΠΊΡΠΈΠΎΠ½Π½ΠΎΠ³ΠΎ ΡΠΎΠΊΠ° Π·Π°Π²ΠΈΡΠΈΡ ΠΎΡ ΡΠΊΠΎΡΠΎΡΡΠΈ ΠΈΠ·ΠΌΠ΅Π½Π΅Π½ΠΈΡ ΠΌΠ°Π³Π½ΠΈΡΠ½ΠΎΠ³ΠΎ ΠΏΠΎΡΠΎΠΊΠ°. ΠΠΎΠΆΠ΅Ρ ΠΈΠ·ΠΌΠ΅Π½ΡΡΡΡΡ ΡΠ°ΠΌΠΎ ΠΏΠΎΠ»Π΅, ΠΈΠ»ΠΈ ΠΊΠΎΠ½ΡΡΡ ΠΌΠΎΠΆΠ΅Ρ ΠΏΠ΅ΡΠ΅ΠΌΠ΅ΡΠ°ΡΡΡΡ Π² Π½Π΅ΠΎΠ΄Π½ΠΎΡΠΎΠ΄Π½ΠΎΠΌ ΠΌΠ°Π³Π½ΠΈΡΠ½ΠΎΠΌ ΠΏΠΎΠ»Π΅.
ΠΠ±ΡΡΡΠ½Π΅Π½ΠΈΡ Π²ΠΎΠ·Π½ΠΈΠΊΠ½ΠΎΠ²Π΅Π½ΠΈΡ ΠΈΠ½Π΄ΡΠΊΡΠΈΠΎΠ½Π½ΠΎΠ³ΠΎ ΡΠΎΠΊΠ°
Π’ΠΎΠΊ Π² ΡΠ΅ΠΏΠΈ ΠΌΠΎΠΆΠ΅Ρ ΡΡΡΠ΅ΡΡΠ²ΠΎΠ²Π°ΡΡ, ΠΊΠΎΠ³Π΄Π° Π½Π° ΡΠ²ΠΎΠ±ΠΎΠ΄Π½ΡΠ΅ Π·Π°ΡΡΠ΄Ρ Π΄Π΅ΠΉΡΡΠ²ΡΡΡ ΡΡΠΎΡΠΎΠ½Π½ΠΈΠ΅ ΡΠΈΠ»Ρ. Π Π°Π±ΠΎΡΠ° ΡΡΠΈΡ ΡΠΈΠ» ΠΏΠΎ ΠΏΠ΅ΡΠ΅ΠΌΠ΅ΡΠ΅Π½ΠΈΡ Π΅Π΄ΠΈΠ½ΠΈΡΠ½ΠΎΠ³ΠΎ ΠΏΠΎΠ»ΠΎΠΆΠΈΡΠ΅Π»ΡΠ½ΠΎΠ³ΠΎ Π·Π°ΡΡΠ΄Π° Π²Π΄ΠΎΠ»Ρ Π·Π°ΠΌΠΊΠ½ΡΡΠΎΠ³ΠΎ ΠΊΠΎΠ½ΡΡΡΠ° ΡΠ°Π²Π½Π° ΠΠΠ‘. ΠΠ½Π°ΡΠΈΡ, ΠΏΡΠΈ ΠΈΠ·ΠΌΠ΅Π½Π΅Π½ΠΈΠΈ ΡΠΈΡΠ»Π° ΠΌΠ°Π³Π½ΠΈΡΠ½ΡΡ Π»ΠΈΠ½ΠΈΠΉ ΡΠ΅ΡΠ΅Π· ΠΏΠΎΠ²Π΅ΡΡ Π½ΠΎΡΡΡ, ΠΎΠ³ΡΠ°Π½ΠΈΡΠ΅Π½Π½ΡΡ ΠΊΠΎΠ½ΡΡΡΠΎΠΌ, Π² Π½Π΅ΠΌ ΠΏΠΎΡΠ²Π»ΡΠ΅ΡΡΡ ΠΠΠ‘, ΠΊΠΎΡΠΎΡΡΡ Π½Π°Π·ΡΠ²Π°ΡΡ ΠΠΠ‘ ΠΈΠ½Π΄ΡΠΊΡΠΈΠΈ.
ΠΠ±ΡΠ°ΡΠΈΡΠ΅ Π²Π½ΠΈΠΌΠ°Π½ΠΈΠ΅
ΠΠ»Π΅ΠΊΡΡΠΎΠ½Ρ Π² Π½Π΅ΠΏΠΎΠ΄Π²ΠΈΠΆΠ½ΠΎΠΌ ΠΏΡΠΎΠ²ΠΎΠ΄Π½ΠΈΠΊΠ΅ ΠΌΠΎΠ³ΡΡ ΠΏΡΠΈΠ²ΠΎΠ΄ΠΈΡΡΡΡ Π² Π΄Π²ΠΈΠΆΠ΅Π½ΠΈΠ΅ ΡΠΎΠ»ΡΠΊΠΎ ΡΠ»Π΅ΠΊΡΡΠΈΡΠ΅ΡΠΊΠΈΠΌ ΠΏΠΎΠ»Π΅ΠΌ. ΠΡΠΎ ΡΠ»Π΅ΠΊΡΡΠΈΡΠ΅ΡΠΊΠΎΠ΅ ΠΏΠΎΠ»Π΅ ΠΏΠΎΡΠΎΠΆΠ΄Π°Π΅ΡΡΡ ΠΈΠ·ΠΌΠ΅Π½ΡΡΡΠΈΠΌΡΡ Π²ΠΎ Π²ΡΠ΅ΠΌΠ΅Π½ΠΈ ΠΌΠ°Π³Π½ΠΈΡΠ½ΡΠΌ ΠΏΠΎΠ»Π΅ΠΌ. ΠΠ³ΠΎ Π½Π°Π·ΡΠ²Π°ΡΡ Π²ΠΈΡ ΡΠ΅Π²ΡΠΌ ΡΠ»Π΅ΠΊΡΡΠΈΡΠ΅ΡΠΊΠΈΠΌ ΠΏΠΎΠ»Π΅ΠΌ. ΠΡΠ΅Π΄ΡΡΠ°Π²Π»Π΅Π½ΠΈΠ΅ ΠΎ Π²ΠΈΡ ΡΠ΅Π²ΠΎΠΌ ΡΠ»Π΅ΠΊΡΡΠΈΡΠ΅ΡΠΊΠΎΠΌ ΠΏΠΎΠ»Π΅ Π±ΡΠ»ΠΎ Π²Π²Π΅Π΄Π΅Π½ΠΎ Π² ΡΠΈΠ·ΠΈΠΊΡ Π²Π΅Π»ΠΈΠΊΠΈΠΌ Π°Π½Π³Π»ΠΈΠΉΡΠΊΠΈΠΌ ΡΠΈΠ·ΠΈΠΊΠΎΠΌ ΠΠΆ. ΠΠ°ΠΊΡΠ²Π΅Π»Π»ΠΎΠΌ Π² 1861 Π³ΠΎΠ΄Ρ.
Π‘Π²ΠΎΠΉΡΡΠ²Π° Π²ΠΈΡ ΡΠ΅Π²ΠΎΠ³ΠΎ ΡΠ»Π΅ΠΊΡΡΠΈΡΠ΅ΡΠΊΠΎΠ³ΠΎ ΠΏΠΎΠ»Ρ:
- ΠΈΡΡΠΎΡΠ½ΠΈΠΊ β ΠΏΠ΅ΡΠ΅ΠΌΠ΅Π½Π½ΠΎΠ΅ ΠΌΠ°Π³Π½ΠΈΡΠ½ΠΎΠ΅ ΠΏΠΎΠ»Π΅;
- ΠΎΠ±Π½Π°ΡΡΠΆΠΈΠ²Π°Π΅ΡΡΡ ΠΏΠΎ Π΄Π΅ΠΉΡΡΠ²ΠΈΡ Π½Π° Π·Π°ΡΡΠ΄;
- Π½Π΅ ΡΠ²Π»ΡΠ΅ΡΡΡ ΠΏΠΎΡΠ΅Π½ΡΠΈΠ°Π»ΡΠ½ΡΠΌ;
- Π»ΠΈΠ½ΠΈΠΈ ΠΏΠΎΠ»Ρ Π·Π°ΠΌΠΊΠ½ΡΡΡΠ΅.
Π Π°Π±ΠΎΡΠ° ΡΡΠΎΠ³ΠΎ ΠΏΠΎΠ»Ρ ΠΏΡΠΈ ΠΏΠ΅ΡΠ΅ΠΌΠ΅ΡΠ΅Π½ΠΈΠΈ Π΅Π΄ΠΈΠ½ΠΈΡΠ½ΠΎΠ³ΠΎ ΠΏΠΎΠ»ΠΎΠΆΠΈΡΠ΅Π»ΡΠ½ΠΎΠ³ΠΎ Π·Π°ΡΡΠ΄Π° ΠΏΠΎ Π·Π°ΠΌΠΊΠ½ΡΡΠΎΠΌΡ ΠΊΠΎΠ½ΡΡΡΡ ΡΠ°Π²Π½Π° ΠΠΠ‘ ΠΈΠ½Π΄ΡΠΊΡΠΈΠΈ Π² Π½Π΅ΠΏΠΎΠ΄Π²ΠΈΠΆΠ½ΠΎΠΌ ΠΏΡΠΎΠ²ΠΎΠ΄Π½ΠΈΠΊΠ΅.
ΠΠ°Π³Π½ΠΈΡΠ½ΡΠΉ ΠΏΠΎΡΠΎΠΊ
ΠΠ°Π³Π½ΠΈΡΠ½ΡΠΌ ΠΏΠΎΡΠΎΠΊΠΎΠΌ ΡΠ΅ΡΠ΅Π· ΠΏΠ»ΠΎΡΠ°Π΄Ρ β( S )β ΠΊΠΎΠ½ΡΡΡΠ° Π½Π°Π·ΡΠ²Π°ΡΡ ΡΠΊΠ°Π»ΡΡΠ½ΡΡ ΡΠΈΠ·ΠΈΡΠ΅ΡΠΊΡΡ Π²Π΅Π»ΠΈΡΠΈΠ½Ρ, ΡΠ°Π²Π½ΡΡ ΠΏΡΠΎΠΈΠ·Π²Π΅Π΄Π΅Π½ΠΈΡ ΠΌΠΎΠ΄ΡΠ»Ρ Π²Π΅ΠΊΡΠΎΡΠ° ΠΌΠ°Π³Π½ΠΈΡΠ½ΠΎΠΉ ΠΈΠ½Π΄ΡΠΊΡΠΈΠΈ β( B )β, ΠΏΠ»ΠΎΡΠ°Π΄ΠΈ ΠΏΠΎΠ²Π΅ΡΡ Π½ΠΎΡΡΠΈ β( S )β, ΠΏΡΠΎΠ½ΠΈΠ·ΡΠ²Π°Π΅ΠΌΠΎΠΉ Π΄Π°Π½Π½ΡΠΌ ΠΏΠΎΡΠΎΠΊΠΎΠΌ, ΠΈ ΠΊΠΎΡΠΈΠ½ΡΡΠ° ΡΠ³Π»Π° β( alpha )β ΠΌΠ΅ΠΆΠ΄Ρ Π½Π°ΠΏΡΠ°Π²Π»Π΅Π½ΠΈΠ΅ΠΌ Π²Π΅ΠΊΡΠΎΡΠ° ΠΌΠ°Π³Π½ΠΈΡΠ½ΠΎΠΉ ΠΈΠ½Π΄ΡΠΊΡΠΈΠΈ ΠΈ Π²Π΅ΠΊΡΠΎΡΠ° Π½ΠΎΡΠΌΠ°Π»ΠΈ (ΠΏΠ΅ΡΠΏΠ΅Π½Π΄ΠΈΠΊΡΠ»ΡΡΠ° ΠΊ ΠΏΠ»ΠΎΡΠΊΠΎΡΡΠΈ Π΄Π°Π½Π½ΠΎΠΉ ΠΏΠΎΠ²Π΅ΡΡ Π½ΠΎΡΡΠΈ):
ΠΠ±ΠΎΠ·Π½Π°ΡΠ΅Π½ΠΈΠ΅ β β( Phi )β, Π΅Π΄ΠΈΠ½ΠΈΡΠ° ΠΈΠ·ΠΌΠ΅ΡΠ΅Π½ΠΈΡ Π² Π‘Π β Π²Π΅Π±Π΅Ρ (ΠΠ±).
ΠΠ°Π³Π½ΠΈΡΠ½ΡΠΉ ΠΏΠΎΡΠΎΠΊ Π² 1 Π²Π΅Π±Π΅Ρ ΡΠΎΠ·Π΄Π°Π΅ΡΡΡ ΠΎΠ΄Π½ΠΎΡΠΎΠ΄Π½ΡΠΌ ΠΌΠ°Π³Π½ΠΈΡΠ½ΡΠΌ ΠΏΠΎΠ»Π΅ΠΌ Ρ ΠΈΠ½Π΄ΡΠΊΡΠΈΠ΅ΠΉ 1 Π’Π» ΡΠ΅ΡΠ΅Π· ΠΏΠΎΠ²Π΅ΡΡ Π½ΠΎΡΡΡ ΠΏΠ»ΠΎΡΠ°Π΄ΡΡ 1 ΠΌ2, ΡΠ°ΡΠΏΠΎΠ»ΠΎΠΆΠ΅Π½Π½ΡΡ ΠΏΠ΅ΡΠΏΠ΅Π½Π΄ΠΈΠΊΡΠ»ΡΡΠ½ΠΎ Π²Π΅ΠΊΡΠΎΡΡ ΠΌΠ°Π³Π½ΠΈΡΠ½ΠΎΠΉ ΠΈΠ½Π΄ΡΠΊΡΠΈΠΈ:
ΠΠ°Π³Π½ΠΈΡΠ½ΡΠΉ ΠΏΠΎΡΠΎΠΊ ΠΌΠΎΠΆΠ½ΠΎ Π½Π°Π³Π»ΡΠ΄Π½ΠΎ ΠΏΡΠ΅Π΄ΡΡΠ°Π²ΠΈΡΡ ΠΊΠ°ΠΊ Π²Π΅Π»ΠΈΡΠΈΠ½Ρ, ΠΏΡΠΎΠΏΠΎΡΡΠΈΠΎΠ½Π°Π»ΡΠ½ΡΡ ΡΠΈΡΠ»Ρ ΠΌΠ°Π³Π½ΠΈΡΠ½ΡΡ Π»ΠΈΠ½ΠΈΠΉ, ΠΏΡΠΎΡ ΠΎΠ΄ΡΡΠΈΡ ΡΠ΅ΡΠ΅Π· Π΄Π°Π½Π½ΡΡ ΠΏΠ»ΠΎΡΠ°Π΄Ρ.
Π Π·Π°Π²ΠΈΡΠΈΠΌΠΎΡΡΠΈ ΠΎΡ ΡΠ³Π»Π° β( alpha )β ΠΌΠ°Π³Π½ΠΈΡΠ½ΡΠΉ ΠΏΠΎΡΠΎΠΊ ΠΌΠΎΠΆΠ΅Ρ Π±ΡΡΡ ΠΏΠΎΠ»ΠΎΠΆΠΈΡΠ΅Π»ΡΠ½ΡΠΌ (( alpha ) 90Β°). ΠΡΠ»ΠΈ ( alpha ) = 90Β°, ΡΠΎ ΠΌΠ°Π³Π½ΠΈΡΠ½ΡΠΉ ΠΏΠΎΡΠΎΠΊ ΡΠ°Π²Π΅Π½ 0.
ΠΠ·ΠΌΠ΅Π½ΠΈΡΡ ΠΌΠ°Π³Π½ΠΈΡΠ½ΡΠΉ ΠΏΠΎΡΠΎΠΊ ΠΌΠΎΠΆΠ½ΠΎ ΠΌΠ΅Π½ΡΡ ΠΏΠ»ΠΎΡΠ°Π΄Ρ ΠΊΠΎΠ½ΡΡΡΠ°, ΠΌΠΎΠ΄ΡΠ»Ρ ΠΈΠ½Π΄ΡΠΊΡΠΈΠΈ ΠΏΠΎΠ»Ρ ΠΈΠ»ΠΈ ΡΠ°ΡΠΏΠΎΠ»ΠΎΠΆΠ΅Π½ΠΈΠ΅ ΠΊΠΎΠ½ΡΡΡΠ° Π² ΠΌΠ°Π³Π½ΠΈΡΠ½ΠΎΠΌ ΠΏΠΎΠ»Π΅ (ΠΏΠΎΠ²ΠΎΡΠ°ΡΠΈΠ²Π°Ρ Π΅Π³ΠΎ).
Π ΡΠ»ΡΡΠ°Π΅ Π½Π΅ΠΎΠ΄Π½ΠΎΡΠΎΠ΄Π½ΠΎΠ³ΠΎ ΠΌΠ°Π³Π½ΠΈΡΠ½ΠΎΠ³ΠΎ ΠΏΠΎΠ»Ρ ΠΈ Π½Π΅ΠΏΠ»ΠΎΡΠΊΠΎΠ³ΠΎ ΠΊΠΎΠ½ΡΡΡΠ° ΠΌΠ°Π³Π½ΠΈΡΠ½ΡΠΉ ΠΏΠΎΡΠΎΠΊ Π½Π°Ρ ΠΎΠ΄ΡΡ ΠΊΠ°ΠΊ ΡΡΠΌΠΌΡ ΠΌΠ°Π³Π½ΠΈΡΠ½ΡΡ ΠΏΠΎΡΠΎΠΊΠΎΠ², ΠΏΡΠΎΠ½ΠΈΠ·ΡΠ²Π°ΡΡΠΈΡ ΠΏΠ»ΠΎΡΠ°Π΄Ρ ΠΊΠ°ΠΆΠ΄ΠΎΠ³ΠΎ ΠΈΠ· ΡΡΠ°ΡΡΠΊΠΎΠ², Π½Π° ΠΊΠΎΡΠΎΡΡΠ΅ ΠΌΠΎΠΆΠ½ΠΎ ΡΠ°Π·Π±ΠΈΡΡ Π΄Π°Π½Π½ΡΡ ΠΏΠΎΠ²Π΅ΡΡ Π½ΠΎΡΡΡ.
ΠΠ°ΠΊΠΎΠ½ ΡΠ»Π΅ΠΊΡΡΠΎΠΌΠ°Π³Π½ΠΈΡΠ½ΠΎΠΉ ΠΈΠ½Π΄ΡΠΊΡΠΈΠΈ Π€Π°ΡΠ°Π΄Π΅Ρ
ΠΠ°ΠΊΠΎΠ½ ΡΠ»Π΅ΠΊΡΡΠΎΠΌΠ°Π³Π½ΠΈΡΠ½ΠΎΠΉ ΠΈΠ½Π΄ΡΠΊΡΠΈΠΈ (Π·Π°ΠΊΠΎΠ½ Π€Π°ΡΠ°Π΄Π΅Ρ):
ΠΠΠ‘ ΠΈΠ½Π΄ΡΠΊΡΠΈΠΈ Π² Π·Π°ΠΌΠΊΠ½ΡΡΠΎΠΌ ΠΊΠΎΠ½ΡΡΡΠ΅ ΡΠ°Π²Π½Π° ΠΈ ΠΏΡΠΎΡΠΈΠ²ΠΎΠΏΠΎΠ»ΠΎΠΆΠ½Π° ΠΏΠΎ Π·Π½Π°ΠΊΡ ΡΠΊΠΎΡΠΎΡΡΠΈ ΠΈΠ·ΠΌΠ΅Π½Π΅Π½ΠΈΡ ΠΌΠ°Π³Π½ΠΈΡΠ½ΠΎΠ³ΠΎ ΠΏΠΎΡΠΎΠΊΠ° ΡΠ΅ΡΠ΅Π· ΠΏΠΎΠ²Π΅ΡΡ Π½ΠΎΡΡΡ, ΠΎΠ³ΡΠ°Π½ΠΈΡΠ΅Π½Π½ΡΡ ΠΊΠΎΠ½ΡΡΡΠΎΠΌ:
ΠΠ½Π°ΠΊ Β«βΒ» Π² ΡΠΎΡΠΌΡΠ»Π΅ ΠΏΠΎΠ·Π²ΠΎΠ»ΡΠ΅Ρ ΡΡΠ΅ΡΡΡ Π½Π°ΠΏΡΠ°Π²Π»Π΅Π½ΠΈΠ΅ ΠΈΠ½Π΄ΡΠΊΡΠΈΠΎΠ½Π½ΠΎΠ³ΠΎ ΡΠΎΠΊΠ°. ΠΠ½Π΄ΡΠΊΡΠΈΠΎΠ½Π½ΡΠΉ ΡΠΎΠΊ Π² Π·Π°ΠΌΠΊΠ½ΡΡΠΎΠΌ ΠΊΠΎΠ½ΡΡΡΠ΅ ΠΈΠΌΠ΅Π΅Ρ Π²ΡΠ΅Π³Π΄Π° ΡΠ°ΠΊΠΎΠ΅ Π½Π°ΠΏΡΠ°Π²Π»Π΅Π½ΠΈΠ΅, ΡΡΠΎΠ±Ρ ΠΌΠ°Π³Π½ΠΈΡΠ½ΡΠΉ ΠΏΠΎΡΠΎΠΊ ΠΏΠΎΠ»Ρ, ΡΠΎΠ·Π΄Π°Π½Π½ΠΎΠ³ΠΎ ΡΡΠΈΠΌ ΡΠΎΠΊΠΎΠΌ ΡΠΊΠ²ΠΎΠ·Ρ ΠΏΠΎΠ²Π΅ΡΡ Π½ΠΎΡΡΡ, ΠΎΠ³ΡΠ°Π½ΠΈΡΠ΅Π½Π½ΡΡ ΠΊΠΎΠ½ΡΡΡΠΎΠΌ, ΡΠΌΠ΅Π½ΡΡΠ°Π» Π±Ρ ΡΠ΅ ΠΈΠ·ΠΌΠ΅Π½Π΅Π½ΠΈΡ ΠΏΠΎΠ»Ρ, ΠΊΠΎΡΠΎΡΡΠ΅ Π²ΡΠ·Π²Π°Π»ΠΈ ΠΏΠΎΡΠ²Π»Π΅Π½ΠΈΠ΅ ΠΈΠ½Π΄ΡΠΊΡΠΈΠΎΠ½Π½ΠΎΠ³ΠΎ ΡΠΎΠΊΠ°.
ΠΡΠ»ΠΈ ΠΊΠΎΠ½ΡΡΡ ΡΠΎΡΡΠΎΠΈΡ ΠΈΠ· β( N )β Π²ΠΈΡΠΊΠΎΠ², ΡΠΎ ΠΠΠ‘ ΠΈΠ½Π΄ΡΠΊΡΠΈΠΈ:
Π‘ΠΈΠ»Π° ΠΈΠ½Π΄ΡΠΊΡΠΈΠΎΠ½Π½ΠΎΠ³ΠΎ ΡΠΎΠΊΠ° Π² Π·Π°ΠΌΠΊΠ½ΡΡΠΎΠΌ ΠΏΡΠΎΠ²ΠΎΠ΄ΡΡΠ΅ΠΌ ΠΊΠΎΠ½ΡΡΡΠ΅ Ρ ΡΠΎΠΏΡΠΎΡΠΈΠ²Π»Π΅Π½ΠΈΠ΅ΠΌ β( R )β:
ΠΡΠΈ Π΄Π²ΠΈΠΆΠ΅Π½ΠΈΠΈ ΠΏΡΠΎΠ²ΠΎΠ΄Π½ΠΈΠΊΠ° Π΄Π»ΠΈΠ½ΠΎΠΉ β( l )β ΡΠΎ ΡΠΊΠΎΡΠΎΡΡΡΡ β( v )β Π² ΠΏΠΎΡΡΠΎΡΠ½Π½ΠΎΠΌ ΠΎΠ΄Π½ΠΎΡΠΎΠ΄Π½ΠΎΠΌ ΠΌΠ°Π³Π½ΠΈΡΠ½ΠΎΠΌ ΠΏΠΎΠ»Π΅ Ρ ΠΈΠ½Π΄ΡΠΊΡΠΈΠ΅ΠΉΒ β( vec{B} )β ΠΠΠ‘ ΡΠ»Π΅ΠΊΡΡΠΎΠΌΠ°Π³Π½ΠΈΡΠ½ΠΎΠΉ ΠΈΠ½Π΄ΡΠΊΡΠΈΠΈ ΡΠ°Π²Π½Π°:
Π³Π΄Π΅ β( alpha )β β ΡΠ³ΠΎΠ» ΠΌΠ΅ΠΆΠ΄Ρ Π²Π΅ΠΊΡΠΎΡΠ°ΠΌΠΈΒ β( vec{B} )β ΠΈ ( vec{v} ).
ΠΠ°ΠΆΠ½ΠΎ
ΠΠΎΠ·Π½ΠΈΠΊΠ½ΠΎΠ²Π΅Π½ΠΈΠ΅ ΠΠΠ‘ ΠΈΠ½Π΄ΡΠΊΡΠΈΠΈ Π² Π΄Π²ΠΈΠΆΡΡΠ΅ΠΌΡΡ Π² ΠΌΠ°Π³Π½ΠΈΡΠ½ΠΎΠΌ ΠΏΠΎΠ»Π΅ ΠΏΡΠΎΠ²ΠΎΠ΄Π½ΠΈΠΊΠ΅ ΠΎΠ±ΡΡΡΠ½ΡΠ΅ΡΡΡ Π΄Π΅ΠΉΡΡΠ²ΠΈΠ΅ΠΌ ΡΠΈΠ»Ρ ΠΠΎΡΠ΅Π½ΡΠ° Π½Π° ΡΠ²ΠΎΠ±ΠΎΠ΄Π½ΡΠ΅ Π·Π°ΡΡΠ΄Ρ Π² Π΄Π²ΠΈΠΆΡΡΠΈΡ ΡΡ ΠΏΡΠΎΠ²ΠΎΠ΄Π½ΠΈΠΊΠ°Ρ . Π‘ΠΈΠ»Π° ΠΠΎΡΠ΅Π½ΡΠ° ΠΈΠ³ΡΠ°Π΅Ρ Π² ΡΡΠΎΠΌ ΡΠ»ΡΡΠ°Π΅ ΡΠΎΠ»Ρ ΡΡΠΎΡΠΎΠ½Π½Π΅ΠΉ ΡΠΈΠ»Ρ.
ΠΠ²ΠΈΠΆΡΡΠΈΠΉΡΡ Π² ΠΌΠ°Π³Π½ΠΈΡΠ½ΠΎΠΌ ΠΏΠΎΠ»Π΅ ΠΏΡΠΎΠ²ΠΎΠ΄Π½ΠΈΠΊ, ΠΏΠΎ ΠΊΠΎΡΠΎΡΠΎΠΌΡ ΠΏΡΠΎΡΠ΅ΠΊΠ°Π΅Ρ ΠΈΠ½Π΄ΡΠΊΡΠΈΠΎΠ½Π½ΡΠΉ ΡΠΎΠΊ, ΠΈΡΠΏΡΡΡΠ²Π°Π΅Ρ ΠΌΠ°Π³Π½ΠΈΡΠ½ΠΎΠ΅ ΡΠΎΡΠΌΠΎΠΆΠ΅Π½ΠΈΠ΅. ΠΠΎΠ»Π½Π°Ρ ΡΠ°Π±ΠΎΡΠ° ΡΠΈΠ»Ρ ΠΠΎΡΠ΅Π½ΡΠ° ΡΠ°Π²Π½Π° Π½ΡΠ»Ρ.
ΠΠΎΠ»ΠΈΡΠ΅ΡΡΠ²ΠΎ ΡΠ΅ΠΏΠ»ΠΎΡΡ Π² ΠΊΠΎΠ½ΡΡΡΠ΅ Π²ΡΠ΄Π΅Π»ΡΠ΅ΡΡΡ Π»ΠΈΠ±ΠΎ Π·Π° ΡΡΠ΅Ρ ΡΠ°Π±ΠΎΡΡ Π²Π½Π΅ΡΠ½Π΅ΠΉ ΡΠΈΠ»Ρ, ΠΊΠΎΡΠΎΡΠ°Ρ ΠΏΠΎΠ΄Π΄Π΅ΡΠΆΠΈΠ²Π°Π΅Ρ ΡΠΊΠΎΡΠΎΡΡΡ ΠΏΡΠΎΠ²ΠΎΠ΄Π½ΠΈΠΊΠ° Π½Π΅ΠΈΠ·ΠΌΠ΅Π½Π½ΠΎΠΉ, Π»ΠΈΠ±ΠΎ Π·Π° ΡΡΠ΅Ρ ΡΠΌΠ΅Π½ΡΡΠ΅Π½ΠΈΡ ΠΊΠΈΠ½Π΅ΡΠΈΡΠ΅ΡΠΊΠΎΠΉ ΡΠ½Π΅ΡΠ³ΠΈΠΈ ΠΏΡΠΎΠ²ΠΎΠ΄Π½ΠΈΠΊΠ°.
ΠΠ°ΠΆΠ½ΠΎ!
ΠΠ·ΠΌΠ΅Π½Π΅Π½ΠΈΠ΅ ΠΌΠ°Π³Π½ΠΈΡΠ½ΠΎΠ³ΠΎ ΠΏΠΎΡΠΎΠΊΠ°, ΠΏΡΠΎΠ½ΠΈΠ·ΡΠ²Π°ΡΡΠ΅Π³ΠΎ Π·Π°ΠΌΠΊΠ½ΡΡΡΠΉ ΠΊΠΎΠ½ΡΡΡ, ΠΌΠΎΠΆΠ΅Ρ ΠΏΡΠΎΠΈΡΡ
ΠΎΠ΄ΠΈΡΡ ΠΏΠΎ Π΄Π²ΡΠΌ ΠΏΡΠΈΡΠΈΠ½Π°ΠΌ:
- ΠΌΠ°Π³Π½ΠΈΡΠ½ΡΠΉ ΠΏΠΎΡΠΎΠΊ ΠΈΠ·ΠΌΠ΅Π½ΡΠ΅ΡΡΡ Π²ΡΠ»Π΅Π΄ΡΡΠ²ΠΈΠ΅ ΠΏΠ΅ΡΠ΅ΠΌΠ΅ΡΠ΅Π½ΠΈΡ ΠΊΠΎΠ½ΡΡΡΠ° ΠΈΠ»ΠΈ Π΅Π³ΠΎ ΡΠ°ΡΡΠ΅ΠΉ Π² ΠΏΠΎΡΡΠΎΡΠ½Π½ΠΎΠΌ Π²ΠΎ Π²ΡΠ΅ΠΌΠ΅Π½ΠΈ ΠΌΠ°Π³Π½ΠΈΡΠ½ΠΎΠΌ ΠΏΠΎΠ»Π΅. ΠΡΠΎ ΡΠ»ΡΡΠ°ΠΉ, ΠΊΠΎΠ³Π΄Π° ΠΏΡΠΎΠ²ΠΎΠ΄Π½ΠΈΠΊΠΈ, Π° Π²ΠΌΠ΅ΡΡΠ΅ Ρ Π½ΠΈΠΌΠΈ ΠΈ ΡΠ²ΠΎΠ±ΠΎΠ΄Π½ΡΠ΅ Π½ΠΎΡΠΈΡΠ΅Π»ΠΈ Π·Π°ΡΡΠ΄Π°, Π΄Π²ΠΈΠΆΡΡΡΡ Π² ΠΌΠ°Π³Π½ΠΈΡΠ½ΠΎΠΌ ΠΏΠΎΠ»Π΅;
- Π²ΡΠΎΡΠ°Ρ ΠΏΡΠΈΡΠΈΠ½Π° ΠΈΠ·ΠΌΠ΅Π½Π΅Π½ΠΈΡ ΠΌΠ°Π³Π½ΠΈΡΠ½ΠΎΠ³ΠΎ ΠΏΠΎΡΠΎΠΊΠ°, ΠΏΡΠΎΠ½ΠΈΠ·ΡΠ²Π°ΡΡΠ΅Π³ΠΎ ΠΊΠΎΠ½ΡΡΡ, β ΠΈΠ·ΠΌΠ΅Π½Π΅Π½ΠΈΠ΅ Π²ΠΎ Π²ΡΠ΅ΠΌΠ΅Π½ΠΈ ΠΌΠ°Π³Π½ΠΈΡΠ½ΠΎΠ³ΠΎ ΠΏΠΎΠ»Ρ ΠΏΡΠΈ Π½Π΅ΠΏΠΎΠ΄Π²ΠΈΠΆΠ½ΠΎΠΌ ΠΊΠΎΠ½ΡΡΡΠ΅. Π ΡΡΠΎΠΌ ΡΠ»ΡΡΠ°Π΅ Π²ΠΎΠ·Π½ΠΈΠΊΠ½ΠΎΠ²Π΅Π½ΠΈΠ΅ ΠΠΠ‘ ΠΈΠ½Π΄ΡΠΊΡΠΈΠΈ ΡΠΆΠ΅ Π½Π΅Π»ΡΠ·Ρ ΠΎΠ±ΡΡΡΠ½ΠΈΡΡ Π΄Π΅ΠΉΡΡΠ²ΠΈΠ΅ΠΌ ΡΠΈΠ»Ρ ΠΠΎΡΠ΅Π½ΡΠ°. Π―Π²Π»Π΅Π½ΠΈΠ΅ ΡΠ»Π΅ΠΊΡΡΠΎΠΌΠ°Π³Π½ΠΈΡΠ½ΠΎΠΉ ΠΈΠ½Π΄ΡΠΊΡΠΈΠΈ Π² Π½Π΅ΠΏΠΎΠ΄Π²ΠΈΠΆΠ½ΡΡ ΠΏΡΠΎΠ²ΠΎΠ΄Π½ΠΈΠΊΠ°Ρ , Π²ΠΎΠ·Π½ΠΈΠΊΠ°ΡΡΠ΅Π΅ ΠΏΡΠΈ ΠΈΠ·ΠΌΠ΅Π½Π΅Π½ΠΈΠΈ ΠΎΠΊΡΡΠΆΠ°ΡΡΠ΅Π³ΠΎ ΠΌΠ°Π³Π½ΠΈΡΠ½ΠΎΠ³ΠΎ ΠΏΠΎΠ»Ρ, ΡΠ°ΠΊΠΆΠ΅ ΠΎΠΏΠΈΡΡΠ²Π°Π΅ΡΡΡ ΡΠΎΡΠΌΡΠ»ΠΎΠΉ Π€Π°ΡΠ°Π΄Π΅Ρ.
Π’Π°ΠΊΠΈΠΌ ΠΎΠ±ΡΠ°Π·ΠΎΠΌ, ΡΠ²Π»Π΅Π½ΠΈΡ ΠΈΠ½Π΄ΡΠΊΡΠΈΠΈ Π² Π΄Π²ΠΈΠΆΡΡΠΈΡ ΡΡ ΠΈ Π½Π΅ΠΏΠΎΠ΄Π²ΠΈΠΆΠ½ΡΡ ΠΏΡΠΎΠ²ΠΎΠ΄Π½ΠΈΠΊΠ°Ρ ΠΏΡΠΎΡΠ΅ΠΊΠ°ΡΡ ΠΎΠ΄ΠΈΠ½Π°ΠΊΠΎΠ²ΠΎ, Π½ΠΎ ΡΠΈΠ·ΠΈΡΠ΅ΡΠΊΠ°Ρ ΠΏΡΠΈΡΠΈΠ½Π° Π²ΠΎΠ·Π½ΠΈΠΊΠ½ΠΎΠ²Π΅Π½ΠΈΡ ΠΈΠ½Π΄ΡΠΊΡΠΈΠΎΠ½Π½ΠΎΠ³ΠΎ ΡΠΎΠΊΠ° ΠΎΠΊΠ°Π·ΡΠ²Π°Π΅ΡΡΡ Π² ΡΡΠΈΡ Π΄Π²ΡΡ ΡΠ»ΡΡΠ°ΡΡ ΡΠ°Π·Π»ΠΈΡΠ½ΠΎΠΉ:
- Π² ΡΠ»ΡΡΠ°Π΅ Π΄Π²ΠΈΠΆΡΡΠΈΡ ΡΡ ΠΏΡΠΎΠ²ΠΎΠ΄Π½ΠΈΠΊΠΎΠ² ΠΠΠ‘ ΠΈΠ½Π΄ΡΠΊΡΠΈΠΈ ΠΎΠ±ΡΡΠ»ΠΎΠ²Π»Π΅Π½Π° ΡΠΈΠ»ΠΎΠΉ ΠΠΎΡΠ΅Π½ΡΠ°;
- Π² ΡΠ»ΡΡΠ°Π΅ Π½Π΅ΠΏΠΎΠ΄Π²ΠΈΠΆΠ½ΡΡ ΠΏΡΠΎΠ²ΠΎΠ΄Π½ΠΈΠΊΠΎΠ² ΠΠΠ‘ ΠΈΠ½Π΄ΡΠΊΡΠΈΠΈ ΡΠ²Π»ΡΠ΅ΡΡΡ ΡΠ»Π΅Π΄ΡΡΠ²ΠΈΠ΅ΠΌ Π΄Π΅ΠΉΡΡΠ²ΠΈΡ Π½Π° ΡΠ²ΠΎΠ±ΠΎΠ΄Π½ΡΠ΅ Π·Π°ΡΡΠ΄Ρ Π²ΠΈΡ ΡΠ΅Π²ΠΎΠ³ΠΎ ΡΠ»Π΅ΠΊΡΡΠΈΡΠ΅ΡΠΊΠΎΠ³ΠΎ ΠΏΠΎΠ»Ρ, Π²ΠΎΠ·Π½ΠΈΠΊΠ°ΡΡΠ΅Π³ΠΎ ΠΏΡΠΈ ΠΈΠ·ΠΌΠ΅Π½Π΅Π½ΠΈΠΈ ΠΌΠ°Π³Π½ΠΈΡΠ½ΠΎΠ³ΠΎ ΠΏΠΎΠ»Ρ.
ΠΠ°ΠΏΡΠ°Π²Π»Π΅Π½ΠΈΠ΅ ΠΈΠ½Π΄ΡΠΊΡΠΈΠΎΠ½Π½ΠΎΠ³ΠΎ ΡΠΎΠΊΠ° ΠΎΠΏΡΠ΅Π΄Π΅Π»ΡΠ΅ΡΡΡ ΠΏΠΎ ΠΏΡΠ°Π²ΠΈΠ»Ρ ΠΠ΅Π½ΡΠ°: ΠΈΠ½Π΄ΡΠΊΡΠΈΠΎΠ½Π½ΡΠΉ ΡΠΎΠΊ, Π²ΠΎΠ·Π±ΡΠΆΠ΄Π°Π΅ΠΌΡΠΉ Π² Π·Π°ΠΌΠΊΠ½ΡΡΠΎΠΌ ΠΊΠΎΠ½ΡΡΡΠ΅ ΠΏΡΠΈ ΠΈΠ·ΠΌΠ΅Π½Π΅Π½ΠΈΠΈ ΠΌΠ°Π³Π½ΠΈΡΠ½ΠΎΠ³ΠΎ ΠΏΠΎΡΠΎΠΊΠ°, Π²ΡΠ΅Π³Π΄Π° Π½Π°ΠΏΡΠ°Π²Π»Π΅Π½ ΡΠ°ΠΊ, ΡΡΠΎ ΡΠΎΠ·Π΄Π°Π²Π°Π΅ΠΌΠΎΠ΅ ΠΈΠΌ ΠΌΠ°Π³Π½ΠΈΡΠ½ΠΎΠ΅ ΠΏΠΎΠ»Π΅ ΠΏΡΠ΅ΠΏΡΡΡΡΠ²ΡΠ΅Ρ ΠΈΠ·ΠΌΠ΅Π½Π΅Π½ΠΈΡ ΠΌΠ°Π³Π½ΠΈΡΠ½ΠΎΠ³ΠΎ ΠΏΠΎΡΠΎΠΊΠ°, Π²ΡΠ·ΡΠ²Π°ΡΡΠ΅Π³ΠΎ ΠΈΠ½Π΄ΡΠΊΡΠΈΠΎΠ½Π½ΡΠΉ ΡΠΎΠΊ.
ΠΠ»Π³ΠΎΡΠΈΡΠΌ ΡΠ΅ΡΠ΅Π½ΠΈΡ Π·Π°Π΄Π°Ρ Ρ ΠΈΡΠΏΠΎΠ»ΡΠ·ΠΎΠ²Π°Π½ΠΈΠ΅ΠΌ ΠΏΡΠ°Π²ΠΈΠ»Π° ΠΠ΅Π½ΡΠ°:
- ΠΎΠΏΡΠ΅Π΄Π΅Π»ΠΈΡΡ Π½Π°ΠΏΡΠ°Π²Π»Π΅Π½ΠΈΠ΅ Π»ΠΈΠ½ΠΈΠΉ ΠΌΠ°Π³Π½ΠΈΡΠ½ΠΎΠΉ ΠΈΠ½Π΄ΡΠΊΡΠΈΠΈ Π²Π½Π΅ΡΠ½Π΅Π³ΠΎ ΠΌΠ°Π³Π½ΠΈΡΠ½ΠΎΠ³ΠΎ ΠΏΠΎΠ»Ρ;
- Π²ΡΡΡΠ½ΠΈΡΡ, ΠΊΠ°ΠΊ ΠΈΠ·ΠΌΠ΅Π½ΡΠ΅ΡΡΡ ΠΌΠ°Π³Π½ΠΈΡΠ½ΡΠΉ ΠΏΠΎΡΠΎΠΊ;
- ΠΎΠΏΡΠ΅Π΄Π΅Π»ΠΈΡΡ Π½Π°ΠΏΡΠ°Π²Π»Π΅Π½ΠΈΠ΅ Π»ΠΈΠ½ΠΈΠΉ ΠΌΠ°Π³Π½ΠΈΡΠ½ΠΎΠΉ ΠΈΠ½Π΄ΡΠΊΡΠΈΠΈ ΠΌΠ°Π³Π½ΠΈΡΠ½ΠΎΠ³ΠΎ ΠΏΠΎΠ»Ρ ΠΈΠ½Π΄ΡΠΊΡΠΈΠΎΠ½Π½ΠΎΠ³ΠΎ ΡΠΎΠΊΠ°: Π΅ΡΠ»ΠΈ ΠΌΠ°Π³Π½ΠΈΡΠ½ΡΠΉ ΠΏΠΎΡΠΎΠΊ ΡΠΌΠ΅Π½ΡΡΠ°Π΅ΡΡΡ, ΡΠΎ ΠΎΠ½ΠΈ ΡΠΎΠ½Π°ΠΏΡΠ°Π²Π»Π΅Π½Ρ Ρ Π»ΠΈΠ½ΠΈΡΠΌΠΈ Π²Π½Π΅ΡΠ½Π΅Π³ΠΎ ΠΌΠ°Π³Π½ΠΈΡΠ½ΠΎΠ³ΠΎ ΠΏΠΎΠ»Ρ; Π΅ΡΠ»ΠΈ ΠΌΠ°Π³Π½ΠΈΡΠ½ΡΠΉ ΠΏΠΎΡΠΎΠΊ ΡΠ²Π΅Π»ΠΈΡΠΈΠ²Π°Π΅ΡΡΡ, β ΠΏΡΠΎΡΠΈΠ²ΠΎΠΏΠΎΠ»ΠΎΠΆΠ½ΠΎ Π½Π°ΠΏΡΠ°Π²Π»Π΅Π½ΠΈΡ Π»ΠΈΠ½ΠΈΠΉ ΠΌΠ°Π³Π½ΠΈΡΠ½ΠΎΠΉ ΠΈΠ½Π΄ΡΠΊΡΠΈΠΈ Π²Π½Π΅ΡΠ½Π΅Π³ΠΎ ΠΏΠΎΠ»Ρ;
- ΠΏΠΎ ΠΏΡΠ°Π²ΠΈΠ»Ρ Π±ΡΡΠ°Π²ΡΠΈΠΊΠ°, Π·Π½Π°Ρ Π½Π°ΠΏΡΠ°Π²Π»Π΅Π½ΠΈΠ΅ Π»ΠΈΠ½ΠΈΠΉ ΠΈΠ½Π΄ΡΠΊΡΠΈΠΈ ΠΌΠ°Π³Π½ΠΈΡΠ½ΠΎΠ³ΠΎ ΠΏΠΎΠ»Ρ ΠΈΠ½Π΄ΡΠΊΡΠΈΠΎΠ½Π½ΠΎΠ³ΠΎ ΡΠΎΠΊΠ°, ΠΎΠΏΡΠ΅Π΄Π΅Π»ΠΈΡΡ Π½Π°ΠΏΡΠ°Π²Π»Π΅Π½ΠΈΠ΅ ΠΈΠ½Π΄ΡΠΊΡΠΈΠΎΠ½Π½ΠΎΠ³ΠΎ ΡΠΎΠΊΠ°.
Π‘ΠΎΠ²Π΅Ρ
ΠΡΠ°Π²ΠΈΠ»ΠΎ ΠΠ΅Π½ΡΠ° ΠΈΠΌΠ΅Π΅Ρ Π³Π»ΡΠ±ΠΎΠΊΠΈΠΉ ΡΠΈΠ·ΠΈΡΠ΅ΡΠΊΠΈΠΉ ΡΠΌΡΡΠ» β ΠΎΠ½ΠΎ Π²ΡΡΠ°ΠΆΠ°Π΅Ρ Π·Π°ΠΊΠΎΠ½ ΡΠΎΡ ΡΠ°Π½Π΅Π½ΠΈΡ ΡΠ½Π΅ΡΠ³ΠΈΠΈ.
Π‘Π°ΠΌΠΎΠΈΠ½Π΄ΡΠΊΡΠΈΡ
Π‘Π°ΠΌΠΎΠΈΠ½Π΄ΡΠΊΡΠΈΡ β ΡΡΠΎ ΡΠ²Π»Π΅Π½ΠΈΠ΅ Π²ΠΎΠ·Π½ΠΈΠΊΠ½ΠΎΠ²Π΅Π½ΠΈΡ ΠΠΠ‘ ΠΈΠ½Π΄ΡΠΊΡΠΈΠΈ Π² ΠΏΡΠΎΠ²ΠΎΠ΄Π½ΠΈΠΊΠ΅ Π² ΡΠ΅Π·ΡΠ»ΡΡΠ°ΡΠ΅ ΠΈΠ·ΠΌΠ΅Π½Π΅Π½ΠΈΡ ΡΠΎΠΊΠ° Π² Π½Π΅ΠΌ.
ΠΡΠΈ ΠΈΠ·ΠΌΠ΅Π½Π΅Π½ΠΈΠΈ ΡΠΈΠ»Ρ ΡΠΎΠΊΠ° Π² ΠΊΠ°ΡΡΡΠΊΠ΅ ΠΏΡΠΎΠΈΡΡ ΠΎΠ΄ΠΈΡ ΠΈΠ·ΠΌΠ΅Π½Π΅Π½ΠΈΠ΅ ΠΌΠ°Π³Π½ΠΈΡΠ½ΠΎΠ³ΠΎ ΠΏΠΎΡΠΎΠΊΠ°, ΡΠΎΠ·Π΄Π°Π²Π°Π΅ΠΌΠΎΠ³ΠΎ ΡΡΠΈΠΌ ΡΠΎΠΊΠΎΠΌ. ΠΠ·ΠΌΠ΅Π½Π΅Π½ΠΈΠ΅ ΠΌΠ°Π³Π½ΠΈΡΠ½ΠΎΠ³ΠΎ ΠΏΠΎΡΠΎΠΊΠ°, ΠΏΡΠΎΠ½ΠΈΠ·ΡΠ²Π°ΡΡΠ΅Π³ΠΎ ΠΊΠ°ΡΡΡΠΊΡ, Π΄ΠΎΠ»ΠΆΠ½ΠΎ Π²ΡΠ·ΡΠ²Π°ΡΡ ΠΏΠΎΡΠ²Π»Π΅Π½ΠΈΠ΅ ΠΠΠ‘ ΠΈΠ½Π΄ΡΠΊΡΠΈΠΈ Π² ΠΊΠ°ΡΡΡΠΊΠ΅.
Π ΡΠΎΠΎΡΠ²Π΅ΡΡΡΠ²ΠΈΠΈ Ρ ΠΏΡΠ°Π²ΠΈΠ»ΠΎΠΌ ΠΠ΅Π½ΡΠ° ΠΠΠ‘ ΡΠ°ΠΌΠΎΠΈΠ½Π΄ΡΠΊΡΠΈΠΈ ΠΏΡΠ΅ΠΏΡΡΡΡΠ²ΡΠ΅Ρ Π½Π°ΡΠ°ΡΡΠ°Π½ΠΈΡ ΡΠΈΠ»Ρ ΡΠΎΠΊΠ° ΠΏΡΠΈ Π²ΠΊΠ»ΡΡΠ΅Π½ΠΈΠΈ ΠΈ ΡΠ±ΡΠ²Π°Π½ΠΈΡ ΡΠΈΠ»Ρ ΡΠΎΠΊΠ° ΠΏΡΠΈ Π²ΡΠΊΠ»ΡΡΠ΅Π½ΠΈΠΈ ΡΠ΅ΠΏΠΈ.
ΠΡΠΎ ΠΏΡΠΈΠ²ΠΎΠ΄ΠΈΡ ΠΊ ΡΠΎΠΌΡ, ΡΡΠΎ ΠΏΡΠΈ Π·Π°ΠΌΡΠΊΠ°Π½ΠΈΠΈ ΡΠ΅ΠΏΠΈ, Π² ΠΊΠΎΡΠΎΡΠΎΠΉ Π΅ΡΡΡ ΠΈΡΡΠΎΡΠ½ΠΈΠΊ ΡΠΎΠΊΠ° Ρ ΠΏΠΎΡΡΠΎΡΠ½Π½ΠΎΠΉ ΠΠΠ‘, ΡΠΈΠ»Π° ΡΠΎΠΊΠ° ΡΡΡΠ°Π½Π°Π²Π»ΠΈΠ²Π°Π΅ΡΡΡ ΡΠ΅ΡΠ΅Π· Π½Π΅ΠΊΠΎΡΠΎΡΠΎΠ΅ Π²ΡΠ΅ΠΌΡ.
ΠΡΠΈ ΠΎΡΠΊΠ»ΡΡΠ΅Π½ΠΈΠΈ ΠΈΡΡΠΎΡΠ½ΠΈΠΊΠ° ΡΠΎΠΊ ΡΠ°ΠΊΠΆΠ΅ Π½Π΅ ΠΏΡΠ΅ΠΊΡΠ°ΡΠ°Π΅ΡΡΡ ΠΌΠ³Π½ΠΎΠ²Π΅Π½Π½ΠΎ. ΠΠΎΠ·Π½ΠΈΠΊΠ°ΡΡΠ°Ρ ΠΏΡΠΈ ΡΡΠΎΠΌ ΠΠΠ‘ ΡΠ°ΠΌΠΎΠΈΠ½Π΄ΡΠΊΡΠΈΠΈ ΠΌΠΎΠΆΠ΅Ρ ΠΏΡΠ΅Π²ΡΡΠ°ΡΡ ΠΠΠ‘ ΠΈΡΡΠΎΡΠ½ΠΈΠΊΠ°.
Π―Π²Π»Π΅Π½ΠΈΠ΅ ΡΠ°ΠΌΠΎΠΈΠ½Π΄ΡΠΊΡΠΈΠΈ ΠΌΠΎΠΆΠ½ΠΎ Π½Π°Π±Π»ΡΠ΄Π°ΡΡ, ΡΠΎΠ±ΡΠ°Π² ΡΠ»Π΅ΠΊΡΡΠΈΡΠ΅ΡΠΊΡΡ ΡΠ΅ΠΏΡ ΠΈΠ· ΠΊΠ°ΡΡΡΠΊΠΈ Ρ Π±ΠΎΠ»ΡΡΠΎΠΉ ΠΈΠ½Π΄ΡΠΊΡΠΈΠ²Π½ΠΎΡΡΡΡ, ΡΠ΅Π·ΠΈΡΡΠΎΡΠ°, Π΄Π²ΡΡ ΠΎΠ΄ΠΈΠ½Π°ΠΊΠΎΠ²ΡΡ Π»Π°ΠΌΠΏ Π½Π°ΠΊΠ°Π»ΠΈΠ²Π°Π½ΠΈΡ ΠΈ ΠΈΡΡΠΎΡΠ½ΠΈΠΊΠ° ΡΠΎΠΊΠ°. Π Π΅Π·ΠΈΡΡΠΎΡ Π΄ΠΎΠ»ΠΆΠ΅Π½ ΠΈΠΌΠ΅ΡΡ ΡΠ°ΠΊΠΎΠ΅ ΠΆΠ΅ ΡΠ»Π΅ΠΊΡΡΠΈΡΠ΅ΡΠΊΠΎΠ΅ ΡΠΎΠΏΡΠΎΡΠΈΠ²Π»Π΅Π½ΠΈΠ΅, ΠΊΠ°ΠΊ ΠΈ ΠΏΡΠΎΠ²ΠΎΠ΄ ΠΊΠ°ΡΡΡΠΊΠΈ.
ΠΠΏΡΡ ΠΏΠΎΠΊΠ°Π·ΡΠ²Π°Π΅Ρ, ΡΡΠΎ ΠΏΡΠΈ Π·Π°ΠΌΡΠΊΠ°Π½ΠΈΠΈ ΡΠ΅ΠΏΠΈ ΡΠ»Π΅ΠΊΡΡΠΈΡΠ΅ΡΠΊΠ°Ρ Π»Π°ΠΌΠΏΠ°, Π²ΠΊΠ»ΡΡΠ΅Π½Π½Π°Ρ ΠΏΠΎΡΠ»Π΅Π΄ΠΎΠ²Π°ΡΠ΅Π»ΡΠ½ΠΎ Ρ ΠΊΠ°ΡΡΡΠΊΠΎΠΉ, Π·Π°Π³ΠΎΡΠ°Π΅ΡΡΡ Π½Π΅ΡΠΊΠΎΠ»ΡΠΊΠΎ ΠΏΠΎΠ·ΠΆΠ΅, ΡΠ΅ΠΌ Π»Π°ΠΌΠΏΠ°, Π²ΠΊΠ»ΡΡΠ΅Π½Π½Π°Ρ ΠΏΠΎΡΠ»Π΅Π΄ΠΎΠ²Π°ΡΠ΅Π»ΡΠ½ΠΎ Ρ ΡΠ΅Π·ΠΈΡΡΠΎΡΠΎΠΌ. ΠΠ°ΡΠ°ΡΡΠ°Π½ΠΈΡ ΡΠΎΠΊΠ° Π² ΡΠ΅ΠΏΠΈ ΠΊΠ°ΡΡΡΠΊΠΈ ΠΏΡΠΈ Π·Π°ΠΌΡΠΊΠ°Π½ΠΈΠΈ ΠΏΡΠ΅ΠΏΡΡΡΡΠ²ΡΠ΅Ρ ΠΠΠ‘ ΡΠ°ΠΌΠΎΠΈΠ½Π΄ΡΠΊΡΠΈΠΈ, Π²ΠΎΠ·Π½ΠΈΠΊΠ°ΡΡΠ°Ρ ΠΏΡΠΈ Π²ΠΎΠ·ΡΠ°ΡΡΠ°Π½ΠΈΠΈ ΠΌΠ°Π³Π½ΠΈΡΠ½ΠΎΠ³ΠΎ ΠΏΠΎΡΠΎΠΊΠ° Π² ΠΊΠ°ΡΡΡΠΊΠ΅.
ΠΡΠΈ ΠΎΡΠΊΠ»ΡΡΠ΅Π½ΠΈΠΈ ΠΈΡΡΠΎΡΠ½ΠΈΠΊΠ° ΡΠΎΠΊΠ° Π²ΡΠΏΡΡ ΠΈΠ²Π°ΡΡ ΠΎΠ±Π΅ Π»Π°ΠΌΠΏΡ. Π ΡΡΠΎΠΌ ΡΠ»ΡΡΠ°Π΅ ΡΠΎΠΊ Π² ΡΠ΅ΠΏΠΈ ΠΏΠΎΠ΄Π΄Π΅ΡΠΆΠΈΠ²Π°Π΅ΡΡΡ ΠΠΠ‘ ΡΠ°ΠΌΠΎΠΈΠ½Π΄ΡΠΊΡΠΈΠΈ, Π²ΠΎΠ·Π½ΠΈΠΊΠ°ΡΡΠ΅ΠΉ ΠΏΡΠΈ ΡΠ±ΡΠ²Π°Π½ΠΈΠΈ ΠΌΠ°Π³Π½ΠΈΡΠ½ΠΎΠ³ΠΎ ΠΏΠΎΡΠΎΠΊΠ° Π² ΠΊΠ°ΡΡΡΠΊΠ΅.
ΠΠΠ‘ ΡΠ°ΠΌΠΎΠΈΠ½Π΄ΡΠΊΡΠΈΠΈ β( varepsilon_{is} )β, Π²ΠΎΠ·Π½ΠΈΠΊΠ°ΡΡΠ°Ρ Π² ΠΊΠ°ΡΡΡΠΊΠ΅ Ρ ΠΈΠ½Π΄ΡΠΊΡΠΈΠ²Π½ΠΎΡΡΡΡ β( L )β, ΠΏΠΎ Π·Π°ΠΊΠΎΠ½Ρ ΡΠ»Π΅ΠΊΡΡΠΎΠΌΠ°Π³Π½ΠΈΡΠ½ΠΎΠΉ ΠΈΠ½Π΄ΡΠΊΡΠΈΠΈ ΡΠ°Π²Π½Π°:
ΠΠΠ‘ ΡΠ°ΠΌΠΎΠΈΠ½Π΄ΡΠΊΡΠΈΠΈ ΠΏΡΡΠΌΠΎ ΠΏΡΠΎΠΏΠΎΡΡΠΈΠΎΠ½Π°Π»ΡΠ½Π° ΠΈΠ½Π΄ΡΠΊΡΠΈΠ²Π½ΠΎΡΡΠΈ ΠΊΠ°ΡΡΡΠΊΠΈ ΠΈ ΡΠΊΠΎΡΠΎΡΡΠΈ ΠΈΠ·ΠΌΠ΅Π½Π΅Π½ΠΈΡ ΡΠΈΠ»Ρ ΡΠΎΠΊΠ° Π² ΠΊΠ°ΡΡΡΠΊΠ΅.
ΠΠ½Π΄ΡΠΊΡΠΈΠ²Π½ΠΎΡΡΡ
ΠΠ»Π΅ΠΊΡΡΠΈΡΠ΅ΡΠΊΠΈΠΉ ΡΠΎΠΊ, ΠΏΡΠΎΡ ΠΎΠ΄ΡΡΠΈΠΉ ΠΏΠΎ ΠΏΡΠΎΠ²ΠΎΠ΄Π½ΠΈΠΊΡ, ΡΠΎΠ·Π΄Π°Π΅Ρ Π²ΠΎΠΊΡΡΠ³ Π½Π΅Π³ΠΎ ΠΌΠ°Π³Π½ΠΈΡΠ½ΠΎΠ΅ ΠΏΠΎΠ»Π΅. ΠΠ°Π³Π½ΠΈΡΠ½ΡΠΉ ΠΏΠΎΡΠΎΠΊ β( Phi )β ΡΠ΅ΡΠ΅Π· ΠΊΠΎΠ½ΡΡΡ ΠΈΠ· ΡΡΠΎΠ³ΠΎ ΠΏΡΠΎΠ²ΠΎΠ΄Π½ΠΈΠΊΠ° ΠΏΡΠΎΠΏΠΎΡΡΠΈΠΎΠ½Π°Π»Π΅Π½ ΠΌΠΎΠ΄ΡΠ»Ρ ΠΈΠ½Π΄ΡΠΊΡΠΈΠΈΒ β( vec{B} )β ΠΌΠ°Π³Π½ΠΈΡΠ½ΠΎΠ³ΠΎ ΠΏΠΎΠ»Ρ Π²Π½ΡΡΡΠΈ ΠΊΠΎΠ½ΡΡΡΠ°, Π° ΠΈΠ½Π΄ΡΠΊΡΠΈΡ ΠΌΠ°Π³Π½ΠΈΡΠ½ΠΎΠ³ΠΎ ΠΏΠΎΠ»Ρ, Π² ΡΠ²ΠΎΡ ΠΎΡΠ΅ΡΠ΅Π΄Ρ, ΠΏΡΠΎΠΏΠΎΡΡΠΈΠΎΠ½Π°Π»ΡΠ½Π° ΡΠΈΠ»Π΅ ΡΠΎΠΊΠ° Π² ΠΏΡΠΎΠ²ΠΎΠ΄Π½ΠΈΠΊΠ΅.
Π‘Π»Π΅Π΄ΠΎΠ²Π°ΡΠ΅Π»ΡΠ½ΠΎ, ΠΌΠ°Π³Π½ΠΈΡΠ½ΡΠΉ ΠΏΠΎΡΠΎΠΊ ΡΠ΅ΡΠ΅Π· ΠΊΠΎΠ½ΡΡΡ ΠΏΡΡΠΌΠΎ ΠΏΡΠΎΠΏΠΎΡΡΠΈΠΎΠ½Π°Π»Π΅Π½ ΡΠΈΠ»Π΅ ΡΠΎΠΊΠ° Π² ΠΊΠΎΠ½ΡΡΡΠ΅:
ΠΠ½Π΄ΡΠΊΡΠΈΠ²Π½ΠΎΡΡΡ β ΠΊΠΎΡΡΡΠΈΡΠΈΠ΅Π½Ρ ΠΏΡΠΎΠΏΠΎΡΡΠΈΠΎΠ½Π°Π»ΡΠ½ΠΎΡΡΠΈ β( L )β ΠΌΠ΅ΠΆΠ΄Ρ ΡΠΈΠ»ΠΎΠΉ ΡΠΎΠΊΠ° β( I )β Π² ΠΊΠΎΠ½ΡΡΡΠ΅ ΠΈ ΠΌΠ°Π³Π½ΠΈΡΠ½ΡΠΌ ΠΏΠΎΡΠΎΠΊΠΎΠΌ β( Phi )β, ΡΠΎΠ·Π΄Π°Π²Π°Π΅ΠΌΡΠΌ ΡΡΠΈΠΌ ΡΠΎΠΊΠΎΠΌ:
ΠΠ½Π΄ΡΠΊΡΠΈΠ²Π½ΠΎΡΡΡ Π·Π°Π²ΠΈΡΠΈΡ ΠΎΡ ΡΠ°Π·ΠΌΠ΅ΡΠΎΠ² ΠΈ ΡΠΎΡΠΌΡ ΠΏΡΠΎΠ²ΠΎΠ΄Π½ΠΈΠΊΠ°, ΠΎΡ ΠΌΠ°Π³Π½ΠΈΡΠ½ΡΡ ΡΠ²ΠΎΠΉΡΡΠ² ΡΡΠ΅Π΄Ρ, Π² ΠΊΠΎΡΠΎΡΠΎΠΉ Π½Π°Ρ ΠΎΠ΄ΠΈΡΡΡ ΠΏΡΠΎΠ²ΠΎΠ΄Π½ΠΈΠΊ.
Π‘ΠΎΠ²Π΅Ρ
ΠΠ΄ΠΈΠ½ΠΈΡΠ° ΠΈΠ½Π΄ΡΠΊΡΠΈΠ²Π½ΠΎΡΡΠΈ Π² Π‘Π β Π³Π΅Π½ΡΠΈ (ΠΠ½). ΠΠ½Π΄ΡΠΊΡΠΈΠ²Π½ΠΎΡΡΡ ΠΊΠΎΠ½ΡΡΡΠ° ΡΠ°Π²Π½Π° 1 Π³Π΅Π½ΡΠΈ, Π΅ΡΠ»ΠΈ ΠΏΡΠΈ ΡΠΈΠ»Π΅ ΠΏΠΎΡΡΠΎΡΠ½Π½ΠΎΠ³ΠΎ ΡΠΎΠΊΠ° 1 Π°ΠΌΠΏΠ΅Ρ ΠΌΠ°Π³Π½ΠΈΡΠ½ΡΠΉ ΠΏΠΎΡΠΎΠΊ ΡΠ΅ΡΠ΅Π· ΠΊΠΎΠ½ΡΡΡ ΡΠ°Π²Π΅Π½ 1 Π²Π΅Π±Π΅Ρ:
ΠΠΎΠΆΠ½ΠΎ Π΄Π°ΡΡ Π²ΡΠΎΡΠΎΠ΅ ΠΎΠΏΡΠ΅Π΄Π΅Π»Π΅Π½ΠΈΠ΅ Π΅Π΄ΠΈΠ½ΠΈΡΡ ΠΈΠ½Π΄ΡΠΊΡΠΈΠ²Π½ΠΎΡΡΠΈ: ΡΠ»Π΅ΠΌΠ΅Π½Ρ ΡΠ»Π΅ΠΊΡΡΠΈΡΠ΅ΡΠΊΠΎΠΉ ΡΠ΅ΠΏΠΈ ΠΎΠ±Π»Π°Π΄Π°Π΅Ρ ΠΈΠ½Π΄ΡΠΊΡΠΈΠ²Π½ΠΎΡΡΡΡ Π² 1 ΠΠ½, Π΅ΡΠ»ΠΈ ΠΏΡΠΈ ΡΠ°Π²Π½ΠΎΠΌΠ΅ΡΠ½ΠΎΠΌ ΠΈΠ·ΠΌΠ΅Π½Π΅Π½ΠΈΠΈ ΡΠΈΠ»Ρ ΡΠΎΠΊΠ° Π² ΡΠ΅ΠΏΠΈ Π½Π° 1 Π°ΠΌΠΏΠ΅Ρ Π·Π° 1 Ρ Π² Π½Π΅ΠΌ Π²ΠΎΠ·Π½ΠΈΠΊΠ°Π΅Ρ ΠΠΠ‘ ΡΠ°ΠΌΠΎΠΈΠ½Π΄ΡΠΊΡΠΈΠΈ 1 Π²ΠΎΠ»ΡΡ.
ΠΠ½Π΅ΡΠ³ΠΈΡ ΠΌΠ°Π³Π½ΠΈΡΠ½ΠΎΠ³ΠΎ ΠΏΠΎΠ»Ρ
ΠΡΠΈ ΠΎΡΠΊΠ»ΡΡΠ΅Π½ΠΈΠΈ ΠΊΠ°ΡΡΡΠΊΠΈ ΠΈΠ½Π΄ΡΠΊΡΠΈΠ²Π½ΠΎΡΡΠΈ ΠΎΡ ΠΈΡΡΠΎΡΠ½ΠΈΠΊΠ° ΡΠΎΠΊΠ° Π»Π°ΠΌΠΏΠ° Π½Π°ΠΊΠ°Π»ΠΈΠ²Π°Π½ΠΈΡ, Π²ΠΊΠ»ΡΡΠ΅Π½Π½Π°Ρ ΠΏΠ°ΡΠ°Π»Π»Π΅Π»ΡΠ½ΠΎ ΠΊΠ°ΡΡΡΠΊΠ΅, Π΄Π°Π΅Ρ ΠΊΡΠ°ΡΠΊΠΎΠ²ΡΠ΅ΠΌΠ΅Π½Π½ΡΡ Π²ΡΠΏΡΡΠΊΡ. Π’ΠΎΠΊ Π² ΡΠ΅ΠΏΠΈ Π²ΠΎΠ·Π½ΠΈΠΊΠ°Π΅Ρ ΠΏΠΎΠ΄ Π΄Π΅ΠΉΡΡΠ²ΠΈΠ΅ΠΌ ΠΠΠ‘ ΡΠ°ΠΌΠΎΠΈΠ½Π΄ΡΠΊΡΠΈΠΈ.
ΠΡΡΠΎΡΠ½ΠΈΠΊΠΎΠΌ ΡΠ½Π΅ΡΠ³ΠΈΠΈ, Π²ΡΠ΄Π΅Π»ΡΡΡΠ΅ΠΉΡΡ ΠΏΡΠΈ ΡΡΠΎΠΌ Π² ΡΠ»Π΅ΠΊΡΡΠΈΡΠ΅ΡΠΊΠΎΠΉ ΡΠ΅ΠΏΠΈ, ΡΠ²Π»ΡΠ΅ΡΡΡ ΠΌΠ°Π³Π½ΠΈΡΠ½ΠΎΠ΅ ΠΏΠΎΠ»Π΅ ΠΊΠ°ΡΡΡΠΊΠΈ.
ΠΠ»Ρ ΡΠΎΠ·Π΄Π°Π½ΠΈΡ ΡΠΎΠΊΠ° Π² ΠΊΠΎΠ½ΡΡΡΠ΅ Ρ ΠΈΠ½Π΄ΡΠΊΡΠΈΠ²Π½ΠΎΡΡΡΡ Π½Π΅ΠΎΠ±Ρ ΠΎΠ΄ΠΈΠΌΠΎ ΡΠΎΠ²Π΅ΡΡΠΈΡΡ ΡΠ°Π±ΠΎΡΡ Π½Π° ΠΏΡΠ΅ΠΎΠ΄ΠΎΠ»Π΅Π½ΠΈΠ΅ ΠΠΠ‘ ΡΠ°ΠΌΠΎΠΈΠ½Π΄ΡΠΊΡΠΈΠΈ. ΠΠ½Π΅ΡΠ³ΠΈΡ ΠΌΠ°Π³Π½ΠΈΡΠ½ΠΎΠ³ΠΎ ΠΏΠΎΠ»Ρ ΡΠΎΠΊΠ° Π²ΡΡΠΈΡΠ»ΡΠ΅ΡΡΡ ΠΏΠΎ ΡΠΎΡΠΌΡΠ»Π΅:
ΠΡΠ½ΠΎΠ²Π½ΡΠ΅ ΡΠΎΡΠΌΡΠ»Ρ ΡΠ°Π·Π΄Π΅Π»Π° Β«ΠΠ»Π΅ΠΊΡΡΠΎΠΌΠ°Π³Π½ΠΈΡΠ½Π°Ρ ΠΈΠ½Π΄ΡΠΊΡΠΈΡΒ»
ΠΠ»Π³ΠΎΡΠΈΡΠΌ ΡΠ΅ΡΠ΅Π½ΠΈΡ Π·Π°Π΄Π°Ρ ΠΏΠΎ ΡΠ΅ΠΌΠ΅ Β«ΠΠ»Π΅ΠΊΡΡΠΎΠΌΠ°Π³Π½ΠΈΡΠ½Π°Ρ ΠΈΠ½Π΄ΡΠΊΡΠΈΡΒ»:
1. ΠΠ½ΠΈΠΌΠ°ΡΠ΅Π»ΡΠ½ΠΎ ΠΏΡΠΎΡΠΈΡΠ°ΡΡ ΡΡΠ»ΠΎΠ²ΠΈΠ΅ Π·Π°Π΄Π°ΡΠΈ. Π£ΡΡΠ°Π½ΠΎΠ²ΠΈΡΡ ΠΏΡΠΈΡΠΈΠ½Ρ ΠΈΠ·ΠΌΠ΅Π½Π΅Π½ΠΈΡ ΠΌΠ°Π³Π½ΠΈΡΠ½ΠΎΠ³ΠΎ ΠΏΠΎΡΠΎΠΊΠ°, ΠΏΡΠΎΠ½ΠΈΠ·ΡΠ²Π°ΡΡΠ΅Π³ΠΎ ΠΊΠΎΠ½ΡΡΡ.
2. ΠΠ°ΠΏΠΈΡΠ°ΡΡ ΡΠΎΡΠΌΡΠ»Ρ:
- Π·Π°ΠΊΠΎΠ½Π° ΡΠ»Π΅ΠΊΡΡΠΎΠΌΠ°Π³Π½ΠΈΡΠ½ΠΎΠΉ ΠΈΠ½Π΄ΡΠΊΡΠΈΠΈ;
- ΠΠΠ‘ ΠΈΠ½Π΄ΡΠΊΡΠΈΠΈ Π² Π΄Π²ΠΈΠΆΡΡΠ΅ΠΌΡΡ ΠΏΡΠΎΠ²ΠΎΠ΄Π½ΠΈΠΊΠ΅, Π΅ΡΠ»ΠΈ Π² Π·Π°Π΄Π°ΡΠ΅ ΡΠ°ΡΡΠΌΠ°ΡΡΠΈΠ²Π°Π΅ΡΡΡ ΠΏΠΎΡΡΡΠΏΠ°ΡΠ΅Π»ΡΠ½ΠΎ Π΄Π²ΠΈΠΆΡΡΠΈΠΉΡΡ ΠΏΡΠΎΠ²ΠΎΠ΄Π½ΠΈΠΊ; Π΅ΡΠ»ΠΈ Π² Π·Π°Π΄Π°ΡΠ΅ ΡΠ°ΡΡΠΌΠ°ΡΡΠΈΠ²Π°Π΅ΡΡΡ ΡΠ»Π΅ΠΊΡΡΠΈΡΠ΅ΡΠΊΠ°Ρ ΡΠ΅ΠΏΡ, ΡΠΎΠ΄Π΅ΡΠΆΠ°ΡΠ°Ρ ΠΈΡΡΠΎΡΠ½ΠΈΠΊ ΡΠΎΠΊΠ°, ΠΈ Π²ΠΎΠ·Π½ΠΈΠΊΠ°ΡΡΠ°Ρ Π½Π° ΠΎΠ΄Π½ΠΎΠΌ ΠΈΠ· ΡΡΠ°ΡΡΠΊΠΎΠ² ΠΠΠ‘ ΠΈΠ½Π΄ΡΠΊΡΠΈΠΈ, Π²ΡΠ·Π²Π°Π½Π½Π°Ρ Π΄Π²ΠΈΠΆΠ΅Π½ΠΈΠ΅ΠΌ ΠΏΡΠΎΠ²ΠΎΠ΄Π½ΠΈΠΊΠ° Π² ΠΌΠ°Π³Π½ΠΈΡΠ½ΠΎΠΌ ΠΏΠΎΠ»Π΅, ΡΠΎ ΡΠ½Π°ΡΠ°Π»Π° Π½ΡΠΆΠ½ΠΎ ΠΎΠΏΡΠ΅Π΄Π΅Π»ΠΈΡΡ Π²Π΅Π»ΠΈΡΠΈΠ½Ρ ΠΈ Π½Π°ΠΏΡΠ°Π²Π»Π΅Π½ΠΈΠ΅ ΠΠΠ‘ ΠΈΠ½Π΄ΡΠΊΡΠΈΠΈ. ΠΠΎΡΠ»Π΅ ΡΡΠΎΠ³ΠΎ Π·Π°Π΄Π°ΡΠ° ΡΠ΅ΡΠ°Π΅ΡΡΡ ΠΏΠΎ Π°Π½Π°Π»ΠΎΠ³ΠΈΠΈ Ρ Π·Π°Π΄Π°ΡΠ°ΠΌΠΈ Π½Π° ΡΠ°ΡΡΠ΅Ρ ΡΠ΅ΠΏΠΈ ΠΏΠΎΡΡΠΎΡΠ½Π½ΠΎΠ³ΠΎ ΡΠΎΠΊΠ° Ρ Π½Π΅ΡΠΊΠΎΠ»ΡΠΊΠΈΠΌΠΈ ΠΈΡΡΠΎΡΠ½ΠΈΠΊΠ°ΠΌΠΈ.
3. ΠΠ°ΠΏΠΈΡΠ°ΡΡ Π²ΡΡΠ°ΠΆΠ΅Π½ΠΈΠ΅ Π΄Π»Ρ ΠΈΠ·ΠΌΠ΅Π½Π΅Π½ΠΈΡ ΠΌΠ°Π³Π½ΠΈΡΠ½ΠΎΠ³ΠΎ ΠΏΠΎΡΠΎΠΊΠ° ΠΈ ΠΏΠΎΠ΄ΡΡΠ°Π²ΠΈΡΡ Π² ΡΠΎΡΠΌΡΠ»Ρ Π·Π°ΠΊΠΎΠ½Π° ΡΠ»Π΅ΠΊΡΡΠΎΠΌΠ°Π³Π½ΠΈΡΠ½ΠΎΠΉ ΠΈΠ½Π΄ΡΠΊΡΠΈΠΈ.
4. ΠΠ°ΠΏΠΈΡΠ°ΡΡ ΠΌΠ°ΡΠ΅ΠΌΠ°ΡΠΈΡΠ΅ΡΠΊΠΈ Π²ΡΠ΅ Π΄ΠΎΠΏΠΎΠ»Π½ΠΈΡΠ΅Π»ΡΠ½ΡΠ΅ ΡΡΠ»ΠΎΠ²ΠΈΡ (ΡΠ°ΡΠ΅ Π²ΡΠ΅Π³ΠΎ ΡΡΠΎ ΡΠΎΡΠΌΡΠ»Ρ Π·Π°ΠΊΠΎΠ½Π° ΠΠΌΠ° Π΄Π»Ρ ΠΏΠΎΠ»Π½ΠΎΠΉ ΡΠ΅ΠΏΠΈ, ΡΠΈΠ»Ρ ΠΠΌΠΏΠ΅ΡΠ° ΠΈΠ»ΠΈ ΡΠΈΠ»Ρ ΠΠΎΡΠ΅Π½ΡΠ°, ΡΠΎΡΠΌΡΠ»Ρ ΠΊΠΈΠ½Π΅ΠΌΠ°ΡΠΈΠΊΠΈ ΠΈ Π΄ΠΈΠ½Π°ΠΌΠΈΠΊΠΈ).
5. Π Π΅ΡΠΈΡΡ ΠΏΠΎΠ»ΡΡΠ΅Π½Π½ΡΡ ΡΠΈΡΡΠ΅ΠΌΡ ΡΡΠ°Π²Π½Π΅Π½ΠΈΠΉ ΠΎΡΠ½ΠΎΡΠΈΡΠ΅Π»ΡΠ½ΠΎ ΠΈΡΠΊΠΎΠΌΠΎΠΉ Π²Π΅Π»ΠΈΡΠΈΠ½Ρ.
6. Π Π΅ΡΠ΅Π½ΠΈΠ΅ ΠΏΡΠΎΠ²Π΅ΡΠΈΡΡ.
ΠΡΡΠΎΡΠ½ΠΈΠΊ: https://fizi4ka.ru/egje-2018-po-fizike/jelektromagnitnaja-indukcija.html
ΠΠ°ΠΊΠΎΠ½ ΡΠ»Π΅ΠΊΡΡΠΎΠΌΠ°Π³Π½ΠΈΡΠ½ΠΎΠΉ ΠΈΠ½Π΄ΡΠΊΡΠΈΠΈ. ΠΡΠ°Π²ΠΈΠ»ΠΎ ΠΠ΅Π½ΡΠ°
Π 1831 Π³ΠΎΠ΄Ρ Π°Π½Π³Π»ΠΈΠΉΡΠΊΠΈΠΉ ΡΡΠ΅Π½ΡΠΉ ΡΠΈΠ·ΠΈΠΊ Π² ΡΠ²ΠΎΠΈΡ Β ΠΎΠΏΡΡΠ°Ρ Β Π.Π€Π°ΡΠ°Π΄Π΅ΠΉ ΠΎΡΠΊΡΡΠ» ΡΠ²Π»Π΅Π½ΠΈΠ΅Β ΡΠ»Π΅ΠΊΡΡΠΎΠΌΠ°Π³Π½ΠΈΡΠ½ΠΎΠΉ ΠΈΠ½Π΄ΡΠΊΡΠΈΠΈ. ΠΠ°ΡΠ΅ΠΌ ΠΈΠ·ΡΡΠ΅Π½ΠΈΠ΅ΠΌ ΡΡΠΎΠ³ΠΎ ΡΠ²Π»Π΅Π½ΠΈΡ Π·Π°Π½ΠΈΠΌΠ°Π»ΠΈΡΡ ΡΡΡΡΠΊΠΈΠ΅ ΡΡΠ΅Π½ΡΠΉ Π.Π₯. ΠΠ΅Π½Ρ ΠΈ Π.Π‘.Π―ΠΊΠΎΠ±ΠΈ.
Π Π½Π°ΡΡΠΎΡΡΠ΅Π΅ Π²ΡΠ΅ΠΌΡ, Π² ΠΎΡΠ½ΠΎΠ²Π΅ ΠΌΠ½ΠΎΠ³ΠΈΡ ΡΡΡΡΠΎΠΉΡΡΠ² Π»Π΅ΠΆΠΈΡ ΡΠ²Π»Π΅Π½ΠΈΠ΅ ΡΠ»Π΅ΠΊΡΡΠΎΠΌΠ°Π³Π½ΠΈΡΠ½ΠΎΠΉ ΠΈΠ½Π΄ΡΠΊΡΠΈΠΈ, Π½Π°ΠΏΡΠΈΠΌΠ΅Ρ Π² Π΄Π²ΠΈΠ³Π°ΡΠ΅Π»Π΅ ΠΈΠ»ΠΈΒ Π³Π΅Π½Π΅ΡΠ°ΡΠΎΡΠ΅ ΡΠ»Π΅ΠΊΡΡΠΈΡΠ΅ΡΠΊΠΎΠ³ΠΎ ΡΠΎΠΊΠ°Β ΡΠΎΠΊΠ°, Π² ΡΡΠ°Π½ΡΡΠΎΡΠΌΠ°ΡΠΎΡΠ°Ρ , ΡΠ°Π΄ΠΈΠΎΠΏΡΠΈΠ΅ΠΌΠ½ΠΈΠΊΠ°Ρ , ΠΈ ΠΌΠ½ΠΎΠ³ΠΈΡ Π΄ΡΡΠ³ΠΈΡ ΡΡΡΡΠΎΠΉΡΡΠ²Π°Ρ .
ΠΠ»Π΅ΠΊΡΡΠΎΠΌΠ°Π³Π½ΠΈΡΠ½Π°Ρ ΠΈΠ½Π΄ΡΠΊΡΠΈΡΒ βΒ ΡΡΠΎ ΡΠ²Π»Π΅Π½ΠΈΠ΅ Π²ΠΎΠ·Π½ΠΈΠΊΠ½ΠΎΠ²Π΅Π½ΠΈΡ ΡΠΎΠΊΠ° Π² Π·Π°ΠΌΠΊΠ½ΡΡΠΎΠΌ ΠΏΡΠΎΠ²ΠΎΠ΄Π½ΠΈΠΊΠ΅, ΠΏΡΠΈ ΠΏΡΠΎΡ ΠΎΠΆΠ΄Π΅Π½ΠΈΠΈ ΡΠ΅ΡΠ΅Π· Π½Π΅Π³ΠΎ ΠΌΠ°Π³Π½ΠΈΡΠ½ΠΎΠ³ΠΎ ΠΏΠΎΡΠΎΠΊΠ°. Π’ΠΎ Π΅ΡΡΡ, Π±Π»Π°Π³ΠΎΠ΄Π°ΡΡ ΡΡΠΎΠΌΡ ΡΠ²Π»Π΅Π½ΠΈΡ ΠΌΡ ΠΌΠΎΠΆΠ΅ΠΌ ΠΏΡΠ΅ΠΎΠ±ΡΠ°Π·ΠΎΠ²ΡΠ²Π°ΡΡ ΠΌΠ΅Ρ Π°Π½ΠΈΡΠ΅ΡΠΊΡΡ ΡΠ½Π΅ΡΠ³ΠΈΡ Π² ΡΠ»Π΅ΠΊΡΡΠΈΡΠ΅ΡΠΊΡΡ β ΠΈ ΡΡΠΎ Π·Π°ΠΌΠ΅ΡΠ°ΡΠ΅Π»ΡΠ½ΠΎ. ΠΠ΅Π΄Ρ Π΄ΠΎ ΠΎΡΠΊΡΡΡΠΈΡ ΡΡΠΎΠ³ΠΎ ΡΠ²Π»Π΅Π½ΠΈΡ Π»ΡΠ΄ΠΈ Π½Π΅ Π·Π½Π°Π»ΠΈ ΠΎ ΠΌΠ΅ΡΠΎΠ΄Π°Ρ ΠΏΠΎΠ»ΡΡΠ΅Π½ΠΈΡΒ ΡΠ»Π΅ΠΊΡΡΠΈΡΠ΅ΡΠΊΠΎΠ³ΠΎ ΡΠΎΠΊΠ°, ΠΊΡΠΎΠΌΠ΅ Π³Π°Π»ΡΠ²Π°Π½ΠΈΠΊΠΈ.
ΠΠΎΠ³Π΄Π° ΠΏΡΠΎΠ²ΠΎΠ΄Π½ΠΈΠΊ ΠΎΠΊΠ°Π·ΡΠ²Π°Π΅ΡΡΡ ΠΏΠΎΠ΄ Π΄Π΅ΠΉΡΡΠ²ΠΈΠ΅ΠΌ ΠΌΠ°Π³Π½ΠΈΡΠ½ΠΎΠ³ΠΎ ΠΏΠΎΠ»Ρ, Π² Π½Π΅ΠΌ Π²ΠΎΠ·Π½ΠΈΠΊΠ°Π΅Ρ ΠΠΠ‘, ΠΊΠΎΡΠΎΡΡΡ ΠΊΠΎΠ»ΠΈΡΠ΅ΡΡΠ²Π΅Π½Π½ΠΎ ΠΌΠΎΠΆΠ½ΠΎ Π²ΡΡΠ°Π·ΠΈΡΡ ΡΠ΅ΡΠ΅Π· Π·Π°ΠΊΠΎΠ½ ΡΠ»Π΅ΠΊΡΡΠΎΠΌΠ°Π³Π½ΠΈΡΠ½ΠΎΠΉ ΠΈΠ½Π΄ΡΠΊΡΠΈΠΈ.
ΠΠ°ΠΊΠΎΠ½ ΡΠ»Π΅ΠΊΡΡΠΎΠΌΠ°Π³Π½ΠΈΡΠ½ΠΎΠΉ ΠΈΠ½Π΄ΡΠΊΡΠΈΠΈ
ΠΠ»Π΅ΠΊΡΡΠΎΠ΄Π²ΠΈΠΆΡΡΠ°Ρ ΡΠΈΠ»Π°, ΠΈΠ½Π΄ΡΡΠΈΡΡΠ΅ΠΌΠ°Ρ Π² ΠΏΡΠΎΠ²ΠΎΠ΄ΡΡΠ΅ΠΌ ΠΊΠΎΠ½ΡΡΡΠ΅, ΡΠ°Π²Π½Π° ΡΠΊΠΎΡΠΎΡΡΠΈ ΠΈΠ·ΠΌΠ΅Π½Π΅Π½ΠΈΡ ΠΌΠ°Π³Π½ΠΈΡΠ½ΠΎΠ³ΠΎ ΠΏΠΎΡΠΎΠΊΠ°, ΡΡΠ΅ΠΏΠ»ΡΡΡΠ΅Π³ΠΎΡΡ Ρ ΡΡΠΈΠΌ ΠΊΠΎΠ½ΡΡΡΠΎΠΌ.Β
Π ΠΊΠ°ΡΡΡΠΊΠ΅, ΠΊΠΎΡΠΎΡΠ°Ρ ΠΈΠΌΠ΅Π΅Ρ Π½Π΅ΡΠΊΠΎΠ»ΡΠΊΠΎ Π²ΠΈΡΠΊΠΎΠ², ΠΎΠ±ΡΠ°Ρ ΠΠΠ‘ Π·Π°Π²ΠΈΡΠΈΡ ΠΎΡ ΠΊΠΎΠ»ΠΈΡΠ΅ΡΡΠ²Π° Π²ΠΈΡΠΊΠΎΠ² n:Β
ΠΠΎ Π² ΠΎΠ±ΡΠ΅ΠΌ ΡΠ»ΡΡΠ°Π΅, ΠΏΡΠΈΠΌΠ΅Π½ΡΡΡ ΡΠΎΡΠΌΡΠ»Ρ ΠΠΠ‘ Ρ ΠΎΠ±ΡΠΈΠΌ ΠΏΠΎΡΠΎΠΊΠΎΡΡΠ΅ΠΏΠ»Π΅Π½ΠΈΠ΅ΠΌ:Β
ΠΠΠ‘ Π²ΠΎΠ·Π±ΡΠΆΠ΄Π°Π΅ΠΌΠ°Ρ Π² ΠΊΠΎΠ½ΡΡΡΠ΅, ΡΠΎΠ·Π΄Π°Π΅Ρ ΡΠΎΠΊ. ΠΠ°ΠΈΠ±ΠΎΠ»Π΅Π΅ ΠΏΡΠΎΡΡΡΠΌ ΠΏΡΠΈΠΌΠ΅ΡΠΎΠΌ ΠΏΠΎΡΠ²Π»Π΅Π½ΠΈΡ ΡΠΎΠΊΠ° Π² Β ΠΏΡΠΎΠ²ΠΎΠ΄Π½ΠΈΠΊΠ΅ ΡΠ²Π»ΡΠ΅ΡΡΡ ΠΊΠ°ΡΡΡΠΊΠ°, ΡΠ΅ΡΠ΅Π· ΠΊΠΎΡΠΎΡΡΡ ΠΏΡΠΎΡ ΠΎΠ΄ΠΈΡΒ ΠΏΠΎΡΡΠΎΡΠ½Π½ΡΠΉ ΠΌΠ°Π³Π½ΠΈΡ. ΠΠ°ΠΏΡΠ°Π²Π»Π΅Π½ΠΈΠ΅ ΠΈΠ½Π΄ΡΡΠΈΡΡΠ΅ΠΌΠΎΠ³ΠΎ ΡΠΎΠΊΠ° ΠΌΠΎΠΆΠ½ΠΎ ΠΎΠΏΡΠ΅Π΄Π΅Π»ΠΈΡΡ Ρ ΠΏΠΎΠΌΠΎΡΡΡ ΠΏΡΠ°Π²ΠΈΠ»Π° ΠΠ΅Π½ΡΠ°.
ΠΡΠ°Π²ΠΈΠ»ΠΎ ΠΠ΅Π½ΡΠ°
Π’ΠΎΠΊ, ΠΈΠ½Π΄ΡΡΠΈΡΡΠ΅ΠΌΡΠΉ ΠΏΡΠΈ ΠΈΠ·ΠΌΠ΅Π½Π΅Π½ΠΈΠΈ ΠΌΠ°Π³Π½ΠΈΡΠ½ΠΎΠ³ΠΎ ΠΏΠΎΠ»Ρ ΠΏΡΠΎΡ ΠΎΠ΄ΡΡΠ΅Π³ΠΎ ΡΠ΅ΡΠ΅Π· ΠΊΠΎΠ½ΡΡΡ, ΡΠ²ΠΎΠΈΠΌ ΠΌΠ°Π³Π½ΠΈΡΠ½ΡΠΌ ΠΏΠΎΠ»Π΅ΠΌ ΠΏΡΠ΅ΠΏΡΡΡΡΠ²ΡΠ΅Ρ ΡΡΠΎΠΌΡ ΠΈΠ·ΠΌΠ΅Π½Π΅Π½ΠΈΡ.
Π ΡΠΎΠΌ ΡΠ»ΡΡΠ°Π΅, ΠΊΠΎΠ³Π΄Π° ΠΌΡ Π²Π²ΠΎΠ΄ΠΈΠΌ ΠΌΠ°Π³Π½ΠΈΡ Π² ΠΊΠ°ΡΡΡΠΊΡ, ΠΌΠ°Π³Π½ΠΈΡΠ½ΡΠΉ ΠΏΠΎΡΠΎΠΊ Π² ΠΊΠΎΠ½ΡΡΡΠ΅ ΡΠ²Π΅Π»ΠΈΡΠΈΠ²Π°Π΅ΡΡΡ, Π° Π·Π½Π°ΡΠΈΡ ΠΌΠ°Π³Π½ΠΈΡΠ½ΠΎΠ΅ ΠΏΠΎΠ»Π΅, ΡΠΎΠ·Π΄Π°Π²Π°Π΅ΠΌΠΎΠ΅ ΠΈΠ½Π΄ΡΡΠΈΡΡΠ΅ΠΌΡΠΌ ΡΠΎΠΊΠΎΠΌ, ΠΏΠΎ ΠΏΡΠ°Π²ΠΈΠ»Ρ ΠΠ΅Π½ΡΠ°, Π½Π°ΠΏΡΠ°Π²Π»Π΅Π½ΠΎ ΠΏΡΠΎΡΠΈΠ² ΡΠ²Π΅Π»ΠΈΡΠ΅Π½ΠΈΡ ΠΏΠΎΠ»Ρ ΠΌΠ°Π³Π½ΠΈΡΠ°.
Π§ΡΠΎΠ±Ρ ΠΎΠΏΡΠ΅Π΄Π΅Π»ΠΈΡΡ Π½Π°ΠΏΡΠ°Π²Π»Π΅Π½ΠΈΠ΅ ΡΠΎΠΊΠ°, Π½ΡΠΆΠ½ΠΎ ΠΏΠΎΡΠΌΠΎΡΡΠ΅ΡΡ Π½Π° ΠΌΠ°Π³Π½ΠΈΡ ΡΠΎ ΡΡΠΎΡΠΎΠ½Ρ ΡΠ΅Π²Π΅ΡΠ½ΠΎΠ³ΠΎ ΠΏΠΎΠ»ΡΡΠ°. Π‘ ΡΡΠΎΠΉ ΠΏΠΎΠ·ΠΈΡΠΈΠΈ ΠΌΡ Π±ΡΠ΄Π΅ΠΌ Π²ΠΊΡΡΡΠΈΠ²Π°ΡΡ Π±ΡΡΠ°Π²ΡΠΈΠΊ ΠΏΠΎ Π½Π°ΠΏΡΠ°Π²Π»Π΅Π½ΠΈΡ ΠΌΠ°Π³Π½ΠΈΡΠ½ΠΎΠ³ΠΎ ΠΏΠΎΠ»Ρ ΡΠΎΠΊΠ°, ΡΠΎ Π΅ΡΡΡ Π½Π°Π²ΡΡΡΠ΅ΡΡ ΡΠ΅Π²Π΅ΡΠ½ΠΎΠΌΡ ΠΏΠΎΠ»ΡΡΡ.
Π’ΠΎΠΊ Π±ΡΠ΄Π΅Ρ Π΄Π²ΠΈΠ³Π°ΡΡΡΡ ΠΏΠΎ Π½Π°ΠΏΡΠ°Π²Π»Π΅Π½ΠΈΡ Π²ΡΠ°ΡΠ΅Π½ΠΈΡ Π±ΡΡΠ°Π²ΡΠΈΠΊΠ°, ΡΠΎ Π΅ΡΡΡ ΠΏΠΎ ΡΠ°ΡΠΎΠ²ΠΎΠΉ ΡΡΡΠ΅Π»ΠΊΠ΅.
Π ΡΠΎΠΌ ΡΠ»ΡΡΠ°Π΅, ΠΊΠΎΠ³Π΄Π° ΠΌΡ Π²ΡΠ²ΠΎΠ΄ΠΈΠΌ ΠΌΠ°Π³Π½ΠΈΡ ΠΈΠ· ΠΊΠ°ΡΡΡΠΊΠΈ, ΠΌΠ°Π³Π½ΠΈΡΠ½ΡΠΉ ΠΏΠΎΡΠΎΠΊ Π² ΠΊΠΎΠ½ΡΡΡΠ΅ ΡΠΌΠ΅Π½ΡΡΠ°Π΅ΡΡΡ, Π° Π·Π½Π°ΡΠΈΡ ΠΌΠ°Π³Π½ΠΈΡΠ½ΠΎΠ΅ ΠΏΠΎΠ»Π΅, ΡΠΎΠ·Π΄Π°Π²Π°Π΅ΠΌΠΎΠ΅ ΠΈΠ½Π΄ΡΡΠΈΡΡΠ΅ΠΌΡΠΌ ΡΠΎΠΊΠΎΠΌ, Π½Π°ΠΏΡΠ°Π²Π»Π΅Π½ΠΎ ΠΏΡΠΎΡΠΈΠ² ΡΠΌΠ΅Π½ΡΡΠ΅Π½ΠΈΡ ΠΏΠΎΠ»Ρ ΠΌΠ°Π³Π½ΠΈΡΠ°. Π§ΡΠΎΠ±Ρ ΠΎΠΏΡΠ΅Π΄Π΅Π»ΠΈΡΡ Π½Π°ΠΏΡΠ°Π²Π»Π΅Π½ΠΈΠ΅ ΡΠΎΠΊΠ°, Π½ΡΠΆΠ½ΠΎ Π²ΡΠΊΡΡΡΠΈΠ²Π°ΡΡ Π±ΡΡΠ°Π²ΡΠΈΠΊ, Π½Π°ΠΏΡΠ°Π²Π»Π΅Π½ΠΈΠ΅ Π²ΡΠ°ΡΠ΅Π½ΠΈΡ Π±ΡΡΠ°Π²ΡΠΈΠΊΠ° ΡΠΊΠ°ΠΆΠ΅Ρ Π½Π°ΠΏΡΠ°Π²Π»Π΅Π½ΠΈΠ΅ ΡΠΎΠΊΠ° Π² ΠΏΡΠΎΠ²ΠΎΠ΄Π½ΠΈΠΊΠ΅ β ΠΏΡΠΎΡΠΈΠ² ΡΠ°ΡΠΎΠ²ΠΎΠΉ ΡΡΡΠ΅Π»ΠΊΠΈ.
Π Π΅ΠΊΠΎΠΌΠ΅Π½Π΄ΡΠ΅ΠΌ ΠΊ ΠΏΡΠΎΡΡΠ΅Π½ΠΈΡ βΒ Π·Π°ΠΊΠΎΠ½ ΠΠΌΠΏΠ΅ΡΠ°Β
1 1 1 1 1 1 1 1 1 1 4.15 (52 ΠΠΎΠ»ΠΎΡΠ°)
ΠΡΡΠΎΡΠ½ΠΈΠΊ: https://electroandi. ru/elektrichestvo-i-magnetizm/zakon-elektromagnitnoj-induktsii-pravilo-lentsa.html
ΠΠ°ΠΊΠΎΠ½ ΡΠ»Π΅ΠΊΡΡΠΎΠΌΠ°Π³Π½ΠΈΡΠ½ΠΎΠΉ ΠΈΠ½Π΄ΡΠΊΡΠΈΠΈ Π€Π°ΡΠ°Π΄Π΅Ρ Π΄Π»Ρ Π½Π°ΡΠΈΠ½Π°ΡΡΠΈΡ
Π§ΡΠΎ ΠΌΠΎΠΆΠ΅Ρ Π±ΡΡΡ Π»ΡΡΡΠ΅, ΡΠ΅ΠΌ Π²Π΅ΡΠ΅ΡΠΎΠΌ ΠΏΠΎΠ½Π΅Π΄Π΅Π»ΡΠ½ΠΈΠΊΠ° ΠΏΠΎΡΠΈΡΠ°ΡΡ ΠΏΡΠΎ ΠΎΡΠ½ΠΎΠ²Ρ ΡΠ»Π΅ΠΊΡΡΠΎΠ΄ΠΈΠ½Π°ΠΌΠΈΠΊΠΈ. ΠΡΠ°Π²ΠΈΠ»ΡΠ½ΠΎ, ΠΌΠΎΠΆΠ½ΠΎ Π½Π°ΠΉΡΠΈ ΠΌΠ½ΠΎΠΆΠ΅ΡΡΠ²ΠΎ Π²Π΅ΡΠ΅ΠΉ, ΠΊΠΎΡΠΎΡΡΠ΅ Π±ΡΠ΄ΡΡ Π»ΡΡΡΠ΅. Π’Π΅ΠΌ Π½Π΅ ΠΌΠ΅Π½Π΅Π΅, ΠΌΡ Π²ΡΠ΅ ΡΠ°Π²Π½ΠΎ ΠΏΡΠ΅Π΄Π»Π°Π³Π°Π΅ΠΌ ΠΠ°ΠΌ ΠΏΡΠΎΡΠ΅ΡΡΡ ΡΡΡ ΡΡΠ°ΡΡΡ.
ΠΡΠ΅ΠΌΠ΅Π½ΠΈ Π·Π°Π½ΠΈΠΌΠ°Π΅Ρ Π½Π΅ ΠΌΠ½ΠΎΠ³ΠΎ, Π° ΠΏΠΎΠ»Π΅Π·Π½Π°Ρ ΠΈΠ½ΡΠΎΡΠΌΠ°ΡΠΈΡ ΠΎΡΡΠ°Π½Π΅ΡΡΡ Π² ΠΏΠΎΠ΄ΡΠΎΠ·Π½Π°Π½ΠΈΠΈ. ΠΠ°ΠΏΡΠΈΠΌΠ΅Ρ, Π½Π° ΡΠΊΠ·Π°ΠΌΠ΅Π½Π΅, Π² ΡΡΠ»ΠΎΠ²ΠΈΡΡ ΡΡΡΠ΅ΡΡΠ°, ΠΌΠΎΠΆΠ½ΠΎ Π±ΡΠ΄Π΅ΡΒ ΡΡΠΏΠ΅ΡΠ½ΠΎ ΠΈΠ·Π²Π»Π΅ΡΡ ΠΈΠ· Π½Π΅Π΄Ρ ΠΏΠ°ΠΌΡΡΠΈ Π·Π°ΠΊΠΎΠ½ Π€Π°ΡΠ°Π΄Π΅Ρ.
Π’Π°ΠΊ ΠΊΠ°ΠΊ Π·Π°ΠΊΠΎΠ½ΠΎΠ² Π€Π°ΡΠ°Π΄Π΅Ρ Π½Π΅ΡΠΊΠΎΠ»ΡΠΊΠΎ, ΡΡΠΎΡΠ½ΠΈΠΌ, ΡΡΠΎ Π·Π΄Π΅ΡΡ ΠΌΡ Π³ΠΎΠ²ΠΎΡΠΈΠΌ ΠΎ Π·Π°ΠΊΠΎΠ½Π΅ ΠΈΠ½Π΄ΡΠΊΡΠΈΠΈ Π€Π°ΡΠ°Π΄Π΅Ρ.
ΠΠ»Π΅ΠΊΡΡΠΎΠ΄ΠΈΠ½Π°ΠΌΠΈΠΊΠ° β ΡΠ°Π·Π΄Π΅Π» ΡΠΈΠ·ΠΈΠΊΠΈ, ΠΈΠ·ΡΡΠ°ΡΡΠΈΠΉ ΡΠ»Π΅ΠΊΡΡΠΎΠΌΠ°Π³Π½ΠΈΡΠ½ΠΎΠ΅ ΠΏΠΎΠ»Π΅ Π²ΠΎ Π²ΡΠ΅Ρ Π΅Π³ΠΎ ΠΏΡΠΎΡΠ²Π»Π΅Π½ΠΈΡΡ .
ΠΠ±ΡΠ°ΡΠΈΡΠ΅ Π²Π½ΠΈΠΌΠ°Π½ΠΈΠ΅
ΠΡΠΎ ΠΈ Π²Π·Π°ΠΈΠΌΠΎΠ΄Π΅ΠΉΡΡΠ²ΠΈΠ΅ ΡΠ»Π΅ΠΊΡΡΠΈΡΠ΅ΡΠΊΠΎΠ³ΠΎ ΠΈ ΠΌΠ°Π³Π½ΠΈΡΠ½ΠΎΠ³ΠΎ ΠΏΠΎΠ»Π΅ΠΉ, ΡΠ»Π΅ΠΊΡΡΠΈΡΠ΅ΡΠΊΠΈΠΉ ΡΠΎΠΊ, ΡΠ»Π΅ΠΊΡΡΠΎ-ΠΌΠ°Π³Π½ΠΈΡΠ½ΠΎΠ΅ ΠΈΠ·Π»ΡΡΠ΅Π½ΠΈΠ΅, Π²Π»ΠΈΡΠ½ΠΈΠ΅ ΠΏΠΎΠ»Ρ Π½Π° Π·Π°ΡΡΠΆΠ΅Π½Π½ΡΠ΅ ΡΠ΅Π»Π°.
ΠΠ΄Π΅ΡΡ ΠΌΡ Π½Π΅ ΡΡΠ°Π²ΠΈΠΌ ΡΠ΅Π»ΡΡ ΡΠ°ΡΡΠΌΠΎΡΡΠ΅ΡΡ Π²ΡΡ ΡΠ»Π΅ΠΊΡΡΠΎΠ΄ΠΈΠ½Π°ΠΌΠΈΠΊΡ. Π£ΠΏΠ°ΡΠΈ ΠΠΎΠΆΠ΅! Π Π°ΡΡΠΌΠΎΡΡΠΈΠΌ Π»ΡΡΡΠ΅ ΠΎΠ΄ΠΈΠ½ ΠΈΠ· ΠΎΡΠ½ΠΎΠ²Π½ΡΡ Π΅Π΅ Π·Π°ΠΊΠΎΠ½ΠΎΠ², ΠΊΠΎΡΠΎΡΡΠΉ Π½Π°Π·ΡΠ²Π°Π΅ΡΡΡ Π·Π°ΠΊΠΎΠ½ΠΎΠΌ ΡΠ»Π΅ΠΊΡΡΠΎΠΌΠ°Π³Π½ΠΈΡΠ½ΠΎΠΉ ΠΈΠ½Π΄ΡΠΊΡΠΈΠΈ Π€Π°ΡΠ°Π΄Π΅Ρ.
ΠΠ°ΠΉΠΊΠ» Π€Π°ΡΠ°Π΄Π΅ΠΉ (1791-1867)
ΠΡΡΠΎΡΠΈΡ ΠΈ ΠΎΠΏΡΠ΅Π΄Π΅Π»Π΅Π½ΠΈΠ΅
Π€Π°ΡΠ°Π΄Π΅ΠΉ, ΠΏΠ°ΡΠ°Π»Π»Π΅Π»ΡΠ½ΠΎ Ρ ΠΠ΅Π½ΡΠΈ, ΠΎΡΠΊΡΡΠ» ΡΠ²Π»Π΅Π½ΠΈΠ΅ ΡΠ»Π΅ΠΊΡΡΠΎΠΌΠ°Π³Π½ΠΈΡΠ½ΠΎΠΉ ΠΈΠ½Π΄ΡΠΊΡΠΈΠΈ Π² 1831 Π³ΠΎΠ΄Ρ. ΠΡΠ°Π²Π΄Π°, ΡΡΠΏΠ΅Π» ΠΎΠΏΡΠ±Π»ΠΈΠΊΠΎΠ²Π°ΡΡ ΡΠ΅Π·ΡΠ»ΡΡΠ°ΡΡ ΡΠ°Π½ΡΡΠ΅. ΠΠ°ΠΊΠΎΠ½ Π€Π°ΡΠ°Π΄Π΅Ρ ΠΏΠΎΠ²ΡΠ΅ΠΌΠ΅ΡΡΠ½ΠΎ ΠΈΡΠΏΠΎΠ»ΡΠ·ΡΠ΅ΡΡΡ Π² ΡΠ΅Ρ Π½ΠΈΠΊΠ΅, Π² ΡΠ»Π΅ΠΊΡΡΠΎΠ΄Π²ΠΈΠ³Π°ΡΠ΅Π»ΡΡ , ΡΡΠ°Π½ΡΡΠΎΡΠΌΠ°ΡΠΎΡΠ°Ρ , Π³Π΅Π½Π΅ΡΠ°ΡΠΎΡΠ°Ρ ΠΈ Π΄ΡΠΎΡΡΠ΅Π»ΡΡ . Π ΡΠ΅ΠΌ ΡΡΡΡ Π·Π°ΠΊΠΎΠ½Π° Π€Π°ΡΠ°Π΄Π΅Ρ Π΄Π»Ρ ΡΠ»Π΅ΠΊΡΡΠΎΠΌΠ°Π³Π½ΠΈΡΠ½ΠΎΠΉ ΠΈΠ½Π΄ΡΠΊΡΠΈΠΈ, Π΅ΡΠ»ΠΈ Π³ΠΎΠ²ΠΎΡΠΈΡΡ ΠΏΡΠΎΡΡΠΎ? Π Π²ΠΎΡ Π² ΡΠ΅ΠΌ!
ΠΡΠΈ ΠΈΠ·ΠΌΠ΅Π½Π΅Π½ΠΈΠΈ ΠΌΠ°Π³Π½ΠΈΡΠ½ΠΎΠ³ΠΎ ΠΏΠΎΡΠΎΠΊΠ° ΡΠ΅ΡΠ΅Π· Π·Π°ΠΌΠΊΠ½ΡΡΡΠΉ ΠΏΡΠΎΠ²ΠΎΠ΄ΡΡΠΈΠΉ ΠΊΠΎΠ½ΡΡΡ, Π² ΠΊΠΎΠ½ΡΡΡΠ΅ Π²ΠΎΠ·Π½ΠΈΠΊΠ°Π΅Ρ ΡΠ»Π΅ΠΊΡΡΠΈΡΠ΅ΡΠΊΠΈΠΉ ΡΠΎΠΊ. Π’ΠΎ Π΅ΡΡΡ, Π΅ΡΠ»ΠΈ ΠΌΡ ΡΠΊΡΡΡΠΈΠΌ ΠΈΠ· ΠΏΡΠΎΠ²ΠΎΠ»ΠΎΠΊΠΈ ΡΠ°ΠΌΠΊΡ ΠΈ ΠΏΠΎΠΌΠ΅ΡΡΠΈΠΌ Π΅Π΅ Π² ΠΈΠ·ΠΌΠ΅Π½ΡΡΡΠ΅Π΅ΡΡ ΠΌΠ°Π³Π½ΠΈΡΠ½ΠΎΠ΅ ΠΏΠΎΠ»Π΅ (Π²ΠΎΠ·ΡΠΌΠ΅ΠΌ ΠΌΠ°Π³Π½ΠΈΡ, ΠΈ Π±ΡΠ΄Π΅ΠΌ ΠΊΡΡΡΠΈΡΡ Π΅Π³ΠΎ Π²ΠΎΠΊΡΡΠ³ ΡΠ°ΠΌΠΊΠΈ), ΠΏΠΎ ΡΠ°ΠΌΠΊΠ΅ ΠΏΠΎΡΠ΅ΡΠ΅Ρ ΡΠΎΠΊ!
Π Π°ΠΌΠΊΠ° Π² ΠΏΠΎΠ»Π΅
ΠΡΠΎΡ ΡΠΎΠΊ Π€Π°ΡΠ°Π΄Π΅ΠΉ Π½Π°Π·Π²Π°Π» ΠΈΠ½Π΄ΡΠΊΡΠΈΠΎΠ½Π½ΡΠΌ, Π° ΡΠ°ΠΌΠΎ ΡΠ²Π»Π΅Π½ΠΈΠ΅ ΠΎΠΊΡΠ΅ΡΡΠΈΠ» ΡΠ»Π΅ΠΊΡΡΠΎΠΌΠ°Π³Π½ΠΈΡΠ½ΠΎΠΉ ΠΈΠ½Π΄ΡΠΊΡΠΈΠ΅ΠΉ.
ΠΠ»Π΅ΠΊΡΡΠΎΠΌΠ°Π³Π½ΠΈΡΠ½Π°Ρ ΠΈΠ½Π΄ΡΠΊΡΠΈΡ β Π²ΠΎΠ·Π½ΠΈΠΊΠ½ΠΎΠ²Π΅Π½ΠΈΠ΅ Π² Π·Π°ΠΌΠΊΠ½ΡΡΠΎΠΌ ΠΊΠΎΠ½ΡΡΡΠ΅ ΡΠ»Π΅ΠΊΡΡΠΈΡΠ΅ΡΠΊΠΎΠ³ΠΎ ΡΠΎΠΊΠ° ΠΏΡΠΈ ΠΈΠ·ΠΌΠ΅Π½Π΅Π½ΠΈΠΈ ΠΌΠ°Π³Π½ΠΈΡΠ½ΠΎΠ³ΠΎ ΠΏΠΎΡΠΎΠΊΠ°, ΠΏΡΠΎΡ ΠΎΠ΄ΡΡΠ΅Π³ΠΎ ΡΠ΅ΡΠ΅Π· ΠΊΠΎΠ½ΡΡΡ.
Π€ΠΎΡΠΌΡΠ»ΠΈΡΠΎΠ²ΠΊΠ° ΠΎΡΠ½ΠΎΠ²Π½ΠΎΠ³ΠΎ Π·Π°ΠΊΠΎΠ½Π° ΡΠ»Π΅ΠΊΡΡΠΎΠ΄ΠΈΠ½Π°ΠΌΠΈΠΊΠΈ β Π·Π°ΠΊΠΎΠ½Π° ΡΠ»Π΅ΠΊΡΡΠΎΠΌΠ°Π³Π½ΠΈΡΠ½ΠΎΠΉ ΠΈΠ½Π΄ΡΠΊΡΠΈΠΈ Π€Π°ΡΠ°Π΄Π΅Ρ, Π²ΡΠ³Π»ΡΠ΄ΠΈΡ ΠΈ Π·Π²ΡΡΠΈΡ ΡΠ»Π΅Π΄ΡΡΡΠΈΠΌ ΠΎΠ±ΡΠ°Π·ΠΎΠΌ:
ΠΠΠ‘, Π²ΠΎΠ·Π½ΠΈΠΊΠ°ΡΡΠ°Ρ Π² ΠΊΠΎΠ½ΡΡΡΠ΅, ΠΏΡΠΎΠΏΠΎΡΡΠΈΠΎΠ½Π°Π»ΡΠ½Π° ΡΠΊΠΎΡΠΎΡΡΠΈ ΠΈΠ·ΠΌΠ΅Π½Π΅Π½ΠΈΡ ΠΌΠ°Π³Π½ΠΈΡΠ½ΠΎΠ³ΠΎ ΠΏΠΎΡΠΎΠΊΠ° Π€ ΡΠ΅ΡΠ΅Π· ΠΊΠΎΠ½ΡΡΡ.
Π ΠΎΡΠΊΡΠ΄Π° Π² ΡΠΎΡΠΌΡΠ»Π΅ ΠΌΠΈΠ½ΡΡ, ΡΠΏΡΠΎΡΠΈΡΠ΅ ΠΡ. ΠΠ»Ρ ΠΎΠ±ΡΡΡΠ½Π΅Π½ΠΈΡ Π·Π½Π°ΠΊΠ° ΠΌΠΈΠ½ΡΡ Π² ΡΡΠΎΠΉ ΡΠΎΡΠΌΡΠ»Π΅ Π΅ΡΡΡ ΡΠΏΠ΅ΡΠΈΠ°Π»ΡΠ½ΠΎΠ΅ ΠΏΡΠ°Π²ΠΈΠ»ΠΎ ΠΠ΅Π½ΡΠ°. ΠΠ½ΠΎ Π³Π»Π°ΡΠΈΡ, ΡΡΠΎ Π·Π½Π°ΠΊ ΠΌΠΈΠ½ΡΡ, Π² Π΄Π°Π½Π½ΠΎΠΌ ΡΠ»ΡΡΠ°Π΅, ΡΠΊΠ°Π·ΡΠ²Π°Π΅Ρ Π½Π° ΡΠΎ, ΠΊΠ°ΠΊ Π½Π°ΠΏΡΠ°Π²Π»Π΅Π½Π° Π²ΠΎΠ·Π½ΠΈΠΊΠ°ΡΡΠ°Ρ ΠΠΠ‘. ΠΠ΅Π»ΠΎ Π² ΡΠΎΠΌ, ΡΡΠΎ ΡΠΎΠ·Π΄Π°Π²Π°Π΅ΠΌΠΎΠ΅ ΠΈΠ½Π΄ΡΠΊΡΠΈΠΎΠ½Π½ΡΠΌ ΡΠΎΠΊΠΎΠΌ ΠΌΠ°Π³Π½ΠΈΡΠ½ΠΎΠ΅ ΠΏΠΎΠ»Π΅ Π½Π°ΠΏΡΠ°Π²Π»Π΅Π½ΠΎ ΡΠ°ΠΊ, ΡΡΠΎ ΠΏΡΠ΅ΠΏΡΡΡΡΠ²ΡΠ΅Ρ ΠΈΠ·ΠΌΠ΅Π½Π΅Π½ΠΈΡ ΠΌΠ°Π³Π½ΠΈΡΠ½ΠΎΠ³ΠΎ ΠΏΠΎΡΠΎΠΊΠ°, ΠΊΠΎΡΠΎΡΡΠΉ Π²ΡΠ·Π²Π°Π» ΠΈΠ½Π΄ΡΠΊΡΠΈΠΎΠ½Π½ΡΠΉ ΡΠΎΠΊ.
ΠΠ»Ρ ΠΎΠΏΡΠ΅Π΄Π΅Π»Π΅Π½ΠΈΡ Π½Π°ΠΏΡΠ°Π²Π»Π΅Π½ΠΈΡ ΠΈΠ½Π΄ΡΠΊΡΠΈΠΎΠ½Π½ΠΎΠ³ΠΎ ΡΠΎΠΊΠ° ΠΏΡΠΈΠΌΠ΅Π½ΡΠ΅ΡΡΡ Π·Π½Π°ΠΌΠ΅Π½ΠΈΡΠΎΠ΅ ΠΏΡΠ°Π²ΠΈΠ»ΠΎ Π±ΡΡΠ°Π²ΡΠΈΠΊΠ°, ΠΈΠ»ΠΈ ΠΏΡΠ°Π²ΠΈΠ»ΠΎ ΠΏΡΠ°Π²ΠΎΠΉ ΡΡΠΊΠΈ, ΠΎΠ½ΠΎ ΠΆΠ΅ ΠΏΡΠ°Π²ΠΈΠ»ΠΎ ΠΏΡΠ°Π²ΠΎΠ³ΠΎ Π²ΠΈΠ½ΡΠ°.
ΠΡΠ»ΠΈ Π»Π°Π΄ΠΎΠ½Ρ ΠΏΡΠ°Π²ΠΎΠΉ ΡΡΠΊΠΈ ΡΠ°ΡΠΏΠΎΠ»ΠΎΠΆΠΈΡΡ ΡΠ°ΠΊ, ΡΡΠΎΠ±Ρ Π² Π½Π΅Ρ Π²Ρ ΠΎΠ΄ΠΈΠ»ΠΈ ΡΠΈΠ»ΠΎΠ²ΡΠ΅ Π»ΠΈΠ½ΠΈΠΈ ΠΌΠ°Π³Π½ΠΈΡΠ½ΠΎΠ³ΠΎ ΠΏΠΎΠ»Ρ, Π° ΠΎΡΠΎΠ³Π½ΡΡΡΠΉ Π±ΠΎΠ»ΡΡΠΎΠΉ ΠΏΠ°Π»Π΅Ρ Π½Π°ΠΏΡΠ°Π²ΠΈΡΡ ΠΏΠΎ Π΄Π²ΠΈΠΆΠ΅Π½ΠΈΡ ΠΏΡΠΎΠ²ΠΎΠ΄Π½ΠΈΠΊΠ°, ΡΠΎ ΡΠ΅ΡΡΡΠ΅ Π²ΡΡΡΠ½ΡΡΡΡ ΠΏΠ°Π»ΡΡΠ° ΡΠΊΠ°ΠΆΡΡ Π½Π°ΠΏΡΠ°Π²Π»Π΅Π½ΠΈΠ΅ ΠΈΠ½Π΄ΡΠΊΡΠΈΠΎΠ½Π½ΠΎΠ³ΠΎ ΡΠΎΠΊΠ°. ΠΡΡΠΌΠΎ Ρ Π½Π°Ρ Π½Π° ΡΠ°ΠΉΡΠ΅, Π²Ρ ΡΠ°ΠΊΠΆΠ΅ ΠΌΠΎΠΆΠ΅ΡΠ΅ ΠΊΡΠΏΠΈΡΡ Π΄ΠΈΠΏΠ»ΠΎΠΌ ΠΏΠΎ ΠΠΠ‘.
ΠΡΠ°Π²ΠΈΠ»ΠΎ ΠΏΡΠ°Π²ΠΎΠΉ ΡΡΠΊΠΈ
ΠΡΠΈΠΌΠ΅ΡΡ ΡΠ΅ΡΠ΅Π½ΠΈΡ Π·Π°Π΄Π°Ρ
ΠΠΎΡ Π²ΡΠΎΠ΄Π΅ Π±Ρ ΠΈ Π²ΡΠ΅. ΠΠ½Π°ΡΠ΅Π½ΠΈΠ΅ Π·Π°ΠΊΠΎΠ½Π° Π€Π°ΡΠ°Π΄Π΅Ρ ΡΡΠ½Π΄Π°ΠΌΠ΅Π½ΡΠ°Π»ΡΠ½ΠΎ, Π²Π΅Π΄Ρ Π½Π° ΠΈΡΠΏΠΎΠ»ΡΠ·ΠΎΠ²Π°Π½ΠΈΠΈ Π΄Π°Π½Π½ΠΎΠ³ΠΎ Π·Π°ΠΊΠΎΠ½Π° ΠΏΠΎΡΡΡΠΎΠ΅Π½Π° ΠΎΡΠ½ΠΎΠ²Π° ΠΏΠΎΡΡΠΈ Π²ΡΠ΅ΠΉ ΡΠ»Π΅ΠΊΡΡΠΈΡΠ΅ΡΠΊΠΎΠΉ ΠΏΡΠΎΠΌΡΡΠ»Π΅Π½Π½ΠΎΡΡΠΈ. Π§ΡΠΎΠ±Ρ ΠΏΠΎΠ½ΠΈΠΌΠ°Π½ΠΈΠ΅ ΠΏΡΠΈΡΠ»ΠΎ Π±ΡΡΡΡΠ΅Π΅, ΡΠ°ΡΡΠΌΠΎΡΡΠΈΠΌ ΠΏΡΠΈΠΌΠ΅Ρ ΡΠ΅ΡΠ΅Π½ΠΈΡ Π·Π°Π΄Π°ΡΠΈ Π½Π° Π·Π°ΠΊΠΎΠ½ Π€Π°ΡΠ°Π΄Π΅Ρ.
Π ΠΏΠΎΠΌΠ½ΠΈΡΠ΅, Π΄ΡΡΠ·ΡΡ! ΠΡΠ»ΠΈ Π·Π°Π΄Π°ΡΠ° Π·Π°ΡΠ΅Π»Π°, ΠΊΠ°ΠΊ ΠΊΠΎΡΡΡ Π² Π³ΠΎΡΠ»Π΅, ΠΈ Π½Π΅Ρ Π±ΠΎΠ»ΡΡΠ΅ ΡΠΈΠ» Π΅Π΅ ΡΠ΅ΡΠΏΠ΅ΡΡ β ΠΎΠ±ΡΠ°ΡΠΈΡΠ΅ΡΡ ΠΊ Π½Π°ΡΠΈΠΌ Π°Π²ΡΠΎΡΠ°ΠΌ! Π’Π΅ΠΏΠ΅ΡΡ Π²Ρ Π·Π½Π°Π΅ΡΠ΅ Π³Π΄Π΅ Π·Π°ΠΊΠ°Π·Π°ΡΡ ΠΊΡΡΡΠΎΠ²ΡΡ ΡΠ°Π±ΠΎΡΡ. ΠΡ Π±ΡΡΡΡΠΎ ΠΏΡΠ΅Π΄ΠΎΡΡΠ°Π²ΠΈΠΌ ΠΏΠΎΠ΄ΡΠΎΠ±Π½ΠΎΠ΅ ΡΠ΅ΡΠ΅Π½ΠΈΠ΅ ΠΈ ΡΠ°Π·ΡΡΡΠ½ΠΈΠΌ Π²ΡΠ΅ Π²ΠΎΠΏΡΠΎΡΡ!
ΠΡΡΠΎΡΠ½ΠΈΠΊ: https://Zaochnik.ru/blog/zakon-elektromagnitnoj-indukcii-faradeya-dlya-nachinayushhix/
ΠΠ°ΠΊΠΎΠ½ ΡΠ»Π΅ΠΊΡΡΠΎΠΌΠ°Π³Π½ΠΈΡΠ½ΠΎΠΉ ΠΈΠ½Π΄ΡΠΊΡΠΈΠΈ Π€Π°ΡΠ°Π΄Π΅Ρ
Π 1831 Π³ΠΎΠ΄Ρ ΠΌΠΈΡ Π²ΠΏΠ΅ΡΠ²ΡΠ΅ ΡΠ·Π½Π°Π» ΠΎ ΠΏΠΎΠ½ΡΡΠΈΠΈ ΡΠ»Π΅ΠΊΡΡΠΎΠΌΠ°Π³Π½ΠΈΡΠ½ΠΎΠΉ ΠΈΠ½Π΄ΡΠΊΡΠΈΠΈ. ΠΠΌΠ΅Π½Π½ΠΎ ΡΠΎΠ³Π΄Π° ΠΠ°ΠΉΠΊΠ» Π€Π°ΡΠ°Π΄Π΅ΠΉ ΠΎΠ±Π½Π°ΡΡΠΆΠΈΠ» ΡΡΠΎ ΡΠ²Π»Π΅Π½ΠΈΠ΅, ΡΡΠ°Π²ΡΠ΅Π΅ Π² ΠΈΡΠΎΠ³Π΅ Π²Π°ΠΆΠ½Π΅ΠΉΡΠΈΠΌ ΠΎΡΠΊΡΡΡΠΈΠ΅ΠΌ Π² ΡΠ»Π΅ΠΊΡΡΠΎΠ΄ΠΈΠ½Π°ΠΌΠΈΠΊΠ΅.
ΠΠΎ ΡΠ΅ΡΠ΅Π΄ΠΈΠ½Ρ XIX Π²Π΅ΠΊΠ° ΡΡΠΈΡΠ°Π»ΠΎΡΡ, ΡΡΠΎ ΡΠ»Π΅ΠΊΡΡΠΈΡΠ΅ΡΠΊΠΎΠ΅ ΠΈ ΠΌΠ°Π³Π½ΠΈΡΠ½ΠΎΠ΅ ΠΏΠΎΠ»Π΅ Π½Π΅ ΠΈΠΌΠ΅ΡΡ Π½ΠΈΠΊΠ°ΠΊΠΎΠΉ ΡΠ²ΡΠ·ΠΈ, ΠΈ ΠΏΡΠΈΡΠΎΠ΄Π° ΠΈΡ ΡΡΡΠ΅ΡΡΠ²ΠΎΠ²Π°Π½ΠΈΡ ΡΠ°Π·Π»ΠΈΡΠ½Π°. ΠΠΎ Π. Π€Π°ΡΠ°Π΄Π΅ΠΉ Π±ΡΠ» ΡΠ²Π΅ΡΠ΅Π½ Π² Π΅Π΄ΠΈΠ½ΠΎΠΉ ΠΏΡΠΈΡΠΎΠ΄Π΅ ΡΡΠΈΡ ΠΏΠΎΠ»Π΅ΠΉ ΠΈ ΠΈΡ ΡΠ²ΠΎΠΉΡΡΠ².
Π―Π²Π»Π΅Π½ΠΈΠ΅ ΡΠ»Π΅ΠΊΡΡΠΎΠΌΠ°Π³Π½ΠΈΡΠ½ΠΎΠΉ ΠΈΠ½Π΄ΡΠΊΡΠΈΠΈ, ΠΎΠ±Π½Π°ΡΡΠΆΠ΅Π½Π½ΠΎΠ΅ ΠΈΠΌ, Π²ΠΏΠΎΡΠ»Π΅Π΄ΡΡΠ²ΠΈΠΈ ΡΡΠ°Π»ΠΎ ΡΡΠ½Π΄Π°ΠΌΠ΅Π½ΡΠΎΠΌ Π΄Π»Ρ ΡΡΡΡΠΎΠΉΡΡΠ²Π° Π³Π΅Π½Π΅ΡΠ°ΡΠΎΡΠΎΠ² Π²ΡΠ΅Ρ ΡΠ»Π΅ΠΊΡΡΠΎΡΡΠ°Π½ΡΠΈΠΉ. ΠΠ»Π°Π³ΠΎΠ΄Π°ΡΡ ΡΡΠΎΠΌΡ ΠΎΡΠΊΡΡΡΠΈΡ Π·Π½Π°Π½ΠΈΡ ΡΠ΅Π»ΠΎΠ²Π΅ΡΠ΅ΡΡΠ²Π° ΠΎ ΡΠ»Π΅ΠΊΡΡΠΎΠΌΠ°Π³Π½Π΅ΡΠΈΠ·ΠΌΠ΅ ΡΠ°Π³Π½ΡΠ»ΠΈ Π΄Π°Π»Π΅ΠΊΠΎ Π²ΠΏΠ΅ΡΠ΅Π΄.
Π€Π°ΡΠ°Π΄Π΅ΠΉ ΠΏΡΠΎΠ΄Π΅Π»Π°Π» ΡΠ»Π΅Π΄ΡΡΡΠΈΠΉ ΠΎΠΏΡΡ: ΠΎΠ½ Π·Π°ΠΌΡΠΊΠ°Π» ΡΠ΅ΠΏΡ Π² ΠΊΠ°ΡΡΡΠΊΠ΅ I ΠΈ Π²ΠΎΠΊΡΡΠ³ Π½Π΅Π΅ Π²ΠΎΠ·ΡΠ°ΡΡΠ°Π»ΠΎ ΠΌΠ°Π³Π½ΠΈΡΠ½ΠΎΠ΅ ΠΏΠΎΠ»Π΅. ΠΠ°Π»Π΅Π΅ Π»ΠΈΠ½ΠΈΠΈ ΠΈΠ½Π΄ΡΠΊΡΠΈΠΈ Π΄Π°Π½Π½ΠΎΠ³ΠΎ ΠΌΠ°Π³Π½ΠΈΡΠ½ΠΎΠ³ΠΎ ΠΏΠΎΠ»Ρ ΠΏΠ΅ΡΠ΅ΡΠ΅ΠΊΠ°Π»ΠΈ ΠΊΠ°ΡΡΡΠΊΡ II, Π² ΠΊΠΎΡΠΎΡΠΎΠΉ Π²ΠΎΠ·Π½ΠΈΠΊΠ°Π» ΠΈΠ½Π΄ΡΠΊΡΠΈΠΎΠ½Π½ΡΠΉ ΡΠΎΠΊ.
Π ΠΈΡ. 1. Π‘Ρ Π΅ΠΌΠ° ΠΎΠΏΡΡΠ° Π€Π°ΡΠ°Π΄Π΅Ρ
ΠΠ° ΡΠ°ΠΌΠΎΠΌ Π΄Π΅Π»Π΅, ΠΎΠ΄Π½ΠΎΠ²ΡΠ΅ΠΌΠ΅Π½Π½ΠΎ Ρ Π€Π°ΡΠ°Π΄Π΅Π΅ΠΌ, Π½ΠΎ Π½Π΅Π·Π°Π²ΠΈΡΠΈΠΌΠΎ ΠΎΡ Π½Π΅Π³ΠΎ, Π΄ΡΡΠ³ΠΎΠΉ ΡΡΠ΅Π½ΡΠΉ ΠΠΆΠΎΠ·Π΅Ρ ΠΠ΅Π½ΡΠΈ ΠΎΠ±Π½Π°ΡΡΠΆΠΈΠ» ΡΡΠΎ ΡΠ²Π»Π΅Π½ΠΈΠ΅. ΠΠ΄Π½Π°ΠΊΠΎ Π€Π°ΡΠ°Π΄Π΅ΠΉ ΠΎΠΏΡΠ±Π»ΠΈΠΊΠΎΠ²Π°Π» ΡΠ²ΠΎΠΈ ΠΈΡΡΠ»Π΅Π΄ΠΎΠ²Π°Π½ΠΈΡ ΡΠ°Π½ΡΡΠ΅. Π’Π°ΠΊΠΈΠΌ ΠΎΠ±ΡΠ°Π·ΠΎΠΌ, Π°Π²ΡΠΎΡΠΎΠΌ Π·Π°ΠΊΠΎΠ½Π° ΡΠ»Π΅ΠΊΡΡΠΎΠΌΠ°Π³Π½ΠΈΡΠ½ΠΎΠΉ ΠΈΠ½Π΄ΡΠΊΡΠΈΠΈ ΡΡΠ°Π» ΠΠ°ΠΉΠΊΠ» Π€Π°ΡΠ°Π΄Π΅ΠΉ.
ΠΠ°ΠΆΠ½ΠΎ
Π‘ΠΊΠΎΠ»ΡΠΊΠΎ Π±Ρ ΡΠΊΡΠΏΠ΅ΡΠΈΠΌΠ΅Π½ΡΠΎΠ² Π½Π΅ ΠΏΡΠΎΠ²ΠΎΠ΄ΠΈΠ» Π€Π°ΡΠ°Π΄Π΅ΠΉ, Π½Π΅ΠΈΠ·ΠΌΠ΅Π½Π½ΡΠΌ ΠΎΡΡΠ°Π²Π°Π»ΠΎΡΡ ΠΎΠ΄Π½ΠΎ ΡΡΠ»ΠΎΠ²ΠΈΠ΅: Π΄Π»Ρ ΠΎΠ±ΡΠ°Π·ΠΎΠ²Π°Π½ΠΈΡ ΠΈΠ½Π΄ΡΠΊΡΠΈΠΎΠ½Π½ΠΎΠ³ΠΎ ΡΠΎΠΊΠ° Π²Π°ΠΆΠ½ΡΠΌ ΡΠ²Π»ΡΠ΅ΡΡΡ ΠΈΠ·ΠΌΠ΅Π½Π΅Π½ΠΈΠ΅ ΠΌΠ°Π³Π½ΠΈΡΠ½ΠΎΠ³ΠΎ ΠΏΠΎΡΠΎΠΊΠ°, ΠΏΡΠΎΠ½ΠΈΠ·ΡΠ²Π°ΡΡΠ΅Π³ΠΎ Π·Π°ΠΌΠΊΠ½ΡΡΡΠΉ ΠΏΡΠΎΠ²ΠΎΠ΄ΡΡΠΈΠΉ ΠΊΠΎΠ½ΡΡΡ (ΠΊΠ°ΡΡΡΠΊΡ).
Π―Π²Π»Π΅Π½ΠΈΠ΅ ΡΠ»Π΅ΠΊΡΡΠΎΠΌΠ°Π³Π½ΠΈΡΠ½ΠΎΠΉ ΠΈΠ½Π΄ΡΠΊΡΠΈΠΈ ΠΎΠΏΡΠ΅Π΄Π΅Π»ΡΠ΅ΡΡΡ Π²ΠΎΠ·Π½ΠΈΠΊΠ½ΠΎΠ²Π΅Π½ΠΈΠ΅ΠΌ ΡΠ»Π΅ΠΊΡΡΠΈΡΠ΅ΡΠΊΠΎΠ³ΠΎ ΡΠΎΠΊΠ° Π² Π·Π°ΠΌΠΊΠ½ΡΡΠΎΠΌ ΡΠ»Π΅ΠΊΡΡΠΎΠΏΡΠΎΠ²ΠΎΠ΄ΡΡΠ΅ΠΌ ΠΊΠΎΠ½ΡΡΡΠ΅ ΠΏΡΠΈ ΠΈΠ·ΠΌΠ΅Π½Π΅Π½ΠΈΠΈ ΠΌΠ°Π³Π½ΠΈΡΠ½ΠΎΠ³ΠΎ ΠΏΠΎΡΠΎΠΊΠ° ΡΠ΅ΡΠ΅Π· ΠΏΠ»ΠΎΡΠ°Π΄Ρ ΡΡΠΎΠ³ΠΎ ΠΊΠΎΠ½ΡΡΡΠ°.
ΠΡΠ½ΠΎΠ²Π½ΠΎΠΉ Π·Π°ΠΊΠΎΠ½ Π€Π°ΡΠ°Π΄Π΅Ρ Π·Π°ΠΊΠ»ΡΡΠ°Π΅ΡΡΡ Π² ΡΠΎΠΌ, ΡΡΠΎ ΡΠ»Π΅ΠΊΡΡΠΎΠ΄Π²ΠΈΠΆΡΡΠ°Ρ ΡΠΈΠ»Π° (ΠΠΠ‘) ΠΏΡΡΠΌΠΎ ΠΏΡΠΎΠΏΠΎΡΡΠΈΠΎΠ½Π°Π»ΡΠ½Π° ΡΠΊΠΎΡΠΎΡΡΠΈ ΠΈΠ·ΠΌΠ΅Π½Π΅Π½ΠΈΡ ΠΌΠ°Π³Π½ΠΈΡΠ½ΠΎΠ³ΠΎ ΠΏΠΎΡΠΎΠΊΠ°.
Π€ΠΎΡΠΌΡΠ»Π° Π·Π°ΠΊΠΎΠ½Π° ΡΠ»Π΅ΠΊΡΡΠΎΠΌΠ°Π³Π½ΠΈΡΠ½ΠΎΠΉ ΠΈΠ½Π΄ΡΠΊΡΠΈΠΈ Π€Π°ΡΠ°Π΄Π΅Ρ Π²ΡΠ³Π»ΡΠ΄ΠΈΡ ΡΠ»Π΅Π΄ΡΡΡΠΈΠΌ ΠΎΠ±ΡΠ°Π·ΠΎΠΌ:
Π ΠΈΡ. 2. Π€ΠΎΡΠΌΡΠ»Π° Π·Π°ΠΊΠΎΠ½Π° ΡΠ»Π΅ΠΊΡΡΠΎΠΌΠ°Π³Π½ΠΈΡΠ½ΠΎΠΉ ΠΈΠ½Π΄ΡΠΊΡΠΈΠΈ
Π Π΅ΡΠ»ΠΈ ΡΠ°ΠΌΠ° ΡΠΎΡΠΌΡΠ»Π°, ΠΈΡΡ ΠΎΠ΄Ρ ΠΈΠ· Π²ΡΡΠ΅ΡΠΊΠ°Π·Π°Π½Π½ΡΡ ΠΎΠ±ΡΡΡΠ½Π΅Π½ΠΈΠΉ Π½Π΅ ΠΏΠΎΡΠΎΠΆΠ΄Π°Π΅Ρ Π²ΠΎΠΏΡΠΎΡΠΎΠ², ΡΠΎ Π·Π½Π°ΠΊ Β«-Β» ΠΌΠΎΠΆΠ΅Ρ Π²ΡΠ·Π²Π°ΡΡ ΡΠΎΠΌΠ½Π΅Π½ΠΈΡ. ΠΠΊΠ°Π·ΡΠ²Π°Π΅ΡΡΡ ΡΡΡΠ΅ΡΡΠ²ΡΠ΅Ρ ΠΏΡΠ°Π²ΠΈΠ»ΠΎ ΠΠ΅Π½ΡΠ° β ΡΡΡΡΠΊΠΎΠ³ΠΎ ΡΡΠ΅Π½ΠΎΠ³ΠΎ, ΠΊΠΎΡΠΎΡΡΠΉ ΠΏΡΠΎΠ²ΠΎΠ΄ΠΈΠ» ΡΠ²ΠΎΠΈ ΠΈΡΡΠ»Π΅Π΄ΠΎΠ²Π°Π½ΠΈΡ, ΠΎΡΠ½ΠΎΠ²ΡΠ²Π°ΡΡΡ Π½Π° ΠΏΠΎΡΡΡΠ»Π°ΡΠ°Ρ Π€Π°ΡΠ°Π΄Π΅Ρ.
ΠΠΎ ΠΠ΅Π½ΡΡ Π·Π½Π°ΠΊ Β«-Β» ΡΠΊΠ°Π·ΡΠ²Π°Π΅Ρ Π½Π° Π½Π°ΠΏΡΠ°Π²Π»Π΅Π½ΠΈΠ΅ Π²ΠΎΠ·Π½ΠΈΠΊΠ°ΡΡΠ΅ΠΉ ΠΠΠ‘, Ρ.Π΅.
ΠΈΠ½Π΄ΡΠΊΡΠΈΠΎΠ½Π½ΡΠΉ ΡΠΎΠΊ Π½Π°ΠΏΡΠ°Π²Π»Π΅Π½ ΡΠ°ΠΊ, ΡΡΠΎ ΠΌΠ°Π³Π½ΠΈΡΠ½ΡΠΉ ΠΏΠΎΡΠΎΠΊ, ΠΊΠΎΡΠΎΡΡΠΉ ΠΎΠ½ ΡΠΎΠ·Π΄Π°Π΅Ρ, ΡΠ΅ΡΠ΅Π· ΠΏΠ»ΠΎΡΠ°Π΄Ρ, ΠΎΠ³ΡΠ°Π½ΠΈΡΠ΅Π½Π½ΡΡ ΠΊΠΎΠ½ΡΡΡΠΎΠΌ, ΡΡΡΠ΅ΠΌΠΈΡΡΡ ΠΏΡΠ΅ΠΏΡΡΡΡΠ²ΠΎΠ²Π°ΡΡ ΡΠΎΠΌΡ ΠΈΠ·ΠΌΠ΅Π½Π΅Π½ΠΈΡ ΠΏΠΎΡΠΎΠΊΠ°, ΠΊΠΎΡΠΎΡΠΎΠ΅ Π²ΡΠ·ΡΠ²Π°Π΅Ρ Π΄Π°Π½Π½ΡΠΉ ΡΠΎΠΊ.
Π 1873 ΠΠΆ.Π.ΠΠ°ΠΊΡΠ²Π΅Π»Π» ΠΏΠΎ-Π½ΠΎΠ²ΠΎΠΌΡ ΠΈΠ·Π»ΠΎΠΆΠΈΠ» ΡΠ΅ΠΎΡΠΈΡ ΡΠ»Π΅ΠΊΡΡΠΎΠΌΠ°Π³Π½ΠΈΡΠ½ΠΎΠ³ΠΎ ΠΏΠΎΠ»Ρ. Π£ΡΠ°Π²Π½Π΅Π½ΠΈΡ, ΠΊΠΎΡΠΎΡΡΠ΅ ΠΎΠ½ Π²ΡΠ²Π΅Π», Π»Π΅Π³Π»ΠΈ Π² ΠΎΡΠ½ΠΎΠ²Ρ ΡΠΎΠ²ΡΠ΅ΠΌΠ΅Π½Π½ΠΎΠΉ ΡΠ°Π΄ΠΈΠΎΡΠ΅Ρ Π½ΠΈΠΊΠΈ ΠΈ ΡΠ»Π΅ΠΊΡΡΠΎΡΠ΅Ρ Π½ΠΈΠΊΠΈ. ΠΠ½ΠΈ Π²ΡΡΠ°ΠΆΠ°ΡΡΡΡ ΡΠ»Π΅Π΄ΡΡΡΠΈΠΌ ΠΎΠ±ΡΠ°Π·ΠΎΠΌ:
- Edl = -dΠ€/dt β ΡΡΠ°Π²Π½Π΅Π½ΠΈΠ΅ ΡΠ»Π΅ΠΊΡΡΠΎΠ΄Π²ΠΈΠΆΡΡΠ΅ΠΉ ΡΠΈΠ»Ρ
- Hdl = -dN/dt β ΡΡΠ°Π²Π½Π΅Π½ΠΈΠ΅ ΠΌΠ°Π³Π½ΠΈΡΠΎΠ΄Π²ΠΈΠΆΡΡΠ΅ΠΉ ΡΠΈΠ»Ρ.
ΠΠ΄Π΅ E β Π½Π°ΠΏΡΡΠΆΠ΅Π½Π½ΠΎΡΡΡ ΡΠ»Π΅ΠΊΡΡΠΈΡΠ΅ΡΠΊΠΎΠ³ΠΎ ΠΏΠΎΠ»Ρ Π½Π° ΡΡΠ°ΡΡΠΊΠ΅ dl; H β Π½Π°ΠΏΡΡΠΆΠ΅Π½Π½ΠΎΡΡΡ ΠΌΠ°Π³Π½ΠΈΡΠ½ΠΎΠ³ΠΎ ΠΏΠΎΠ»Ρ Π½Π° ΡΡΠ°ΡΡΠΊΠ΅ dl; N β ΠΏΠΎΡΠΎΠΊ ΡΠ»Π΅ΠΊΡΡΠΈΡΠ΅ΡΠΊΠΎΠΉ ΠΈΠ½Π΄ΡΠΊΡΠΈΠΈ, t β Π²ΡΠ΅ΠΌΡ.
Π‘ΠΈΠΌΠΌΠ΅ΡΡΠΈΡΠ½ΡΠΉ Ρ Π°ΡΠ°ΠΊΡΠ΅Ρ Π΄Π°Π½Π½ΡΡ ΡΡΠ°Π²Π½Π΅Π½ΠΈΠΉ ΡΡΡΠ°Π½Π°Π²Π»ΠΈΠ²Π°Π΅Ρ ΡΠ²ΡΠ·Ρ ΡΠ»Π΅ΠΊΡΡΠΈΡΠ΅ΡΠΊΠΈΡ ΠΈ ΠΌΠ°Π³Π½ΠΈΡΠ½ΡΡ ΡΠ²Π»Π΅Π½ΠΈΠΉ, Π° ΡΠ°ΠΊΠΆΠ΅ ΠΌΠ°Π³Π½ΠΈΡΠ½ΡΡ Ρ ΡΠ»Π΅ΠΊΡΡΠΈΡΠ΅ΡΠΊΠΈΠΌΠΈ. ΡΠΈΠ·ΠΈΡΠ΅ΡΠΊΠΈΠΉ ΡΠΌΡΡΠ», ΠΊΠΎΡΠΎΡΡΠΌ ΠΎΠΏΡΠ΅Π΄Π΅Π»ΡΡΡΡΡ ΡΡΠΈ ΡΡΠ°Π²Π½Π΅Π½ΠΈΡ, ΠΌΠΎΠΆΠ½ΠΎ Π²ΡΡΠ°Π·ΠΈΡΡ ΡΠ»Π΅Π΄ΡΡΡΠΈΠΌΠΈ ΠΏΠΎΠ»ΠΎΠΆΠ΅Π½ΠΈΡΠΌΠΈ:
- Π΅ΡΠ»ΠΈ ΡΠ»Π΅ΠΊΡΡΠΈΡΠ΅ΡΠΊΠΎΠ΅ ΠΏΠΎΠ»Π΅ ΠΈΠ·ΠΌΠ΅Π½ΡΠ΅ΡΡΡ, ΡΠΎ ΡΡΠΎ ΠΈΠ·ΠΌΠ΅Π½Π΅Π½ΠΈΠ΅ Π²ΡΠ΅Π³Π΄Π° ΡΠΎΠΏΡΠΎΠ²ΠΎΠΆΠ΄Π°Π΅ΡΡΡ ΠΌΠ°Π³Π½ΠΈΡΠ½ΡΠΌ ΠΏΠΎΠ»Π΅ΠΌ.
- Π΅ΡΠ»ΠΈ ΠΌΠ°Π³Π½ΠΈΡΠ½ΠΎΠ΅ ΠΏΠΎΠ»Π΅ ΠΈΠ·ΠΌΠ΅Π½ΡΠ΅ΡΡΡ, ΡΠΎ ΡΡΠΎ ΠΈΠ·ΠΌΠ΅Π½Π΅Π½ΠΈΠ΅ Π²ΡΠ΅Π³Π΄Π° ΡΠΎΠΏΡΠΎΠ²ΠΎΠΆΠ΄Π°Π΅ΡΡΡ ΡΠ»Π΅ΠΊΡΡΠΈΡΠ΅ΡΠΊΠΈΠΌ ΠΏΠΎΠ»Π΅ΠΌ.
Π ΠΈΡ. 3. ΠΠΎΠ·Π½ΠΈΠΊΠ½ΠΎΠ²Π΅Π½ΠΈΠ΅ Π²ΠΈΡ ΡΠ΅Π²ΠΎΠ³ΠΎ ΠΌΠ°Π³Π½ΠΈΡΠ½ΠΎΠ³ΠΎ ΠΏΠΎΠ»Ρ
Π’Π°ΠΊΠΆΠ΅ ΠΠ°ΠΊΡΠ²Π΅Π»Π» ΡΡΡΠ°Π½ΠΎΠ²ΠΈΠ», ΡΡΠΎ ΡΠ°ΡΠΏΡΠΎΡΡΡΠ°Π½Π΅Π½ΠΈΠ΅ ΡΠ»Π΅ΠΊΡΡΠΎΠΌΠ°Π³Π½ΠΈΡΠ½ΠΎΠ³ΠΎ ΠΏΠΎΠ»Ρ ΡΠ°Π²Π½Π° ΡΠΊΠΎΡΠΎΡΡΠΈ ΡΠ°ΡΠΏΡΠΎΡΡΡΠ°Π½Π΅Π½ΠΈΡ ΡΠ²Π΅ΡΠ°.
Π£ΡΠ΅Π½ΠΈΠΊΠ°ΠΌ 11 ΠΊΠ»Π°ΡΡΠ° Π½Π΅ΠΎΠ±Ρ ΠΎΠ΄ΠΈΠΌΠΎ Π·Π½Π°ΡΡ, ΡΡΠΎ ΡΠ»Π΅ΠΊΡΡΠΎΠΌΠ°Π³Π½ΠΈΡΠ½ΡΡ ΠΈΠ½Π΄ΡΠΊΡΠΈΡ Π²ΠΏΠ΅ΡΠ²ΡΠ΅ ΠΊΠ°ΠΊ ΡΠ²Π»Π΅Π½ΠΈΠ΅ ΠΎΠ±Π½Π°ΡΡΠΆΠΈΠ» ΠΠ°ΠΉΠΊΠ» Π€Π°ΡΠ°Π΄Π΅ΠΉ. ΠΠ½ Π΄ΠΎΠΊΠ°Π·Π°Π», ΡΡΠΎ ΡΠ»Π΅ΠΊΡΡΠΈΡΠ΅ΡΠΊΠΎΠ΅ ΠΈ ΠΌΠ°Π³Π½ΠΈΡΠ½ΠΎΠ΅ ΠΏΠΎΠ»Π΅ ΠΈΠΌΠ΅ΡΡ ΠΎΠ±ΡΡΡ ΠΏΡΠΈΡΠΎΠ΄Ρ. Π‘Π°ΠΌΠΎΡΡΠΎΡΡΠ΅Π»ΡΠ½ΡΠ΅ ΠΈΡΡΠ»Π΅Π΄ΠΎΠ²Π°Π½ΠΈΡ Π½Π° ΠΎΡΠ½ΠΎΠ²Π΅ ΠΎΠΏΡΡΠΎΠ² Π€Π°ΡΠ°Π΄Π΅Ρ ΡΠ°ΠΊΠΆΠ΅ ΠΏΡΠΎΠ²ΠΎΠ΄ΠΈΠ»ΠΈ ΡΠ°ΠΊΠΈΠ΅ Π²Π΅Π»ΠΈΠΊΠΈΠ΅ Π΄Π΅ΡΡΠ΅Π»ΠΈ ΠΊΠ°ΠΊ ΠΠ΅Π½Ρ ΠΈ ΠΠ°ΠΊΡΠ²Π΅Π»Π», ΠΊΠΎΡΠΎΡΡΠ΅ ΡΠ°ΡΡΠΈΡΠΈΠ»ΠΈ Π½Π°ΡΠΈ ΠΏΠΎΠ·Π½Π°Π½ΠΈΡ Π² ΠΎΠ±Π»Π°ΡΡΠΈ ΡΠ»Π΅ΠΊΡΡΠΎΠΌΠ°Π³Π½ΠΈΡΠ½ΠΎΠ³ΠΎ ΠΏΠΎΠ»Ρ.
Π‘ΡΠ΅Π΄Π½ΡΡ ΠΎΡΠ΅Π½ΠΊΠ°: 4.2. ΠΡΠ΅Π³ΠΎ ΠΏΠΎΠ»ΡΡΠ΅Π½ΠΎ ΠΎΡΠ΅Π½ΠΎΠΊ: 134.
ΠΡΠ΄Ρ Π² ΡΠΈΡΠ»Π΅ ΠΏΠ΅ΡΠ²ΡΡ Π½Π° Π΄ΠΎΡΠΊΠ΅ ΠΏΠΎΡΠ΅ΡΠ°
ΠΡΡΠΎΡΠ½ΠΈΠΊ: https://obrazovaka.ru/fizika/zakon-elektromagnitnoy-indukcii-faradeya-formula.html
ΠΠ°ΠΊΠΎΠ½ ΡΠ»Π΅ΠΊΡΡΠΎΠΌΠ°Π³Π½ΠΈΡΠ½ΠΎΠΉ ΠΈΠ½Π΄ΡΠΊΡΠΈΠΈ (Π·Π°ΠΊΠΎΠ½ Π€Π°ΡΠ°Π΄Π΅Ρ)
Π€Π΅Π΄ΡΠ½ Π.Π. ΠΠΎΠ½ΡΠΏΠ΅ΠΊΡ Π»Π΅ΠΊΡΠΈΠΉ ΠΏΠΎ ΡΠΈΠ·ΠΈΠΊΠ΅ ΠΠ»Π΅ΠΊΡΡΠΎΠΌΠ°Π³Π½Π΅ΡΠΈΠ·ΠΈ
ΠΠ΅ΠΊΡΠΈΡ 26.
ΠΠ»Π΅ΠΊΡΡΠΎΠΌΠ°Π³Π½ΠΈΡΠ½Π°Ρ
ΠΈΠ½Π΄ΡΠΊΡΠΈΡ. ΠΡΠΊΡΡΡΠΈΠ΅ Π€Π°ΡΠ°Π΄Π΅Ρ.
Π 1831 Π³. Π. Π€Π°ΡΠ°Π΄Π΅Π΅ΠΌ Π±ΡΠ»ΠΎ ΡΠ΄Π΅Π»Π°Π½ΠΎ ΠΎΠ΄Π½ΠΎ ΠΈΠ·
Π²Π°ΠΆΠ½Π΅ΠΉΡΠΈΡ
ΡΡΠ½Π΄Π°ΠΌΠ΅Π½ΡΠ°Π»ΡΠ½ΡΡ
ΠΎΡΠΊΡΡΡΠΈΠΉ Π²
ΡΠ»Π΅ΠΊΡΡΠΎΠ΄ΠΈΠ½Π°ΠΌΠΈΠΊΠ΅ β ΠΎΠ±Π½Π°ΡΡΠΆΠ΅Π½ΠΎ ΡΠ²Π»Π΅Π½ΠΈΠ΅ΡΠ»Π΅ΠΊΡΡΠΎΠΌΠ°Π³Π½ΠΈΡΠ½ΠΎΠΉ
ΠΈΠ½Π΄ΡΠΊΡΠΈΠΈ.
Π Π·Π°ΠΌΠΊΠ½ΡΡΠΎΠΌ ΠΏΡΠΎΠ²ΠΎΠ΄ΡΡΠ΅ΠΌ
ΠΊΠΎΠ½ΡΡΡΠ΅ ΠΏΡΠΈ ΠΈΠ·ΠΌΠ΅Π½Π΅Π½ΠΈΠΈ ΠΌΠ°Π³Π½ΠΈΡΠ½ΠΎΠ³ΠΎ ΠΏΠΎΡΠΎΠΊΠ°
(ΠΏΠΎΡΠΎΠΊΠ° Π²Π΅ΠΊΡΠΎΡΠ° ),
ΠΎΡ
Π²Π°ΡΡΠ²Π°Π΅ΠΌΠΎΠ³ΠΎ ΡΡΠΈΠΌ ΠΊΠΎΠ½ΡΡΡΠΎΠΌ, Π²ΠΎΠ·Π½ΠΈΠΊΠ°Π΅Ρ
ΡΠ»Π΅ΠΊΡΡΠΈΡΠ΅ΡΠΊΠΈΠΉ ΡΠΎΠΊ.
ΠΡΠΎΡ ΡΠΎΠΊ ΠΏΠΎΠ»ΡΡΠΈΠ» Π½Π°Π·Π²Π°Π½ΠΈΠ΅ ΠΈΠ½Π΄ΡΠΊΡΠΈΠΎΠ½Π½ΠΎΠ³ΠΎ.
ΠΠΎΡΠ²Π»Π΅Π½ΠΈΠ΅ ΠΈΠ½Π΄ΡΠΊΡΠΈΠΎΠ½Π½ΠΎΠ³ΠΎ ΡΠΎΠΊΠ° ΠΎΠ·Π½Π°ΡΠ°Π΅Ρ,
ΡΡΠΎ ΠΏΡΠΈ ΠΈΠ·ΠΌΠ΅Π½Π΅Π½ΠΈΠΈ ΠΌΠ°Π³Π½ΠΈΡΠ½ΠΎΠ³ΠΎ
ΠΏΠΎΡΠΎΠΊΠ° Π² ΠΊΠΎΠ½ΡΡΡΠ΅ Π²ΠΎΠ·Π½ΠΈΠΊΠ°Π΅Ρ Ρ.Π΄.Ρ. ΠΈΠ½Π΄ΡΠΊΡΠΈΠΈ (ΡΠ°Π±ΠΎΡΠ° ΠΏΠΎ ΠΏΠ΅ΡΠ΅Π½Π΅ΡΠ΅Π½ΠΈΡ Π΅Π΄ΠΈΠ½ΠΈΡΠ½ΠΎΠ³ΠΎ Π·Π°ΡΡΠ΄Π° ΠΏΠΎ Π·Π°ΠΌΠΊΠ½ΡΡΠΎΠΌΡ ΠΊΠΎΠ½ΡΡΡΡ). ΠΡΠΌΠ΅ΡΠΈΠΌ, ΡΡΠΎ Π·Π½Π°ΡΠ΅Π½ΠΈΠ΅ΡΠΎΠ²Π΅ΡΡΠ΅Π½Π½ΠΎ Π½Π΅ Π·Π°Π²ΠΈΡΠΈΡ ΠΎΡ ΡΠΎΠ³ΠΎ, ΠΊΠ°ΠΊΠΈΠΌ ΠΎΠ±ΡΠ°Π·ΠΎΠΌ ΠΎΡΡΡΠ΅ΡΡΠ²Π»ΡΠ΅ΡΡΡ ΠΈΠ·ΠΌΠ΅Π½Π΅Π½ΠΈΠ΅ ΠΌΠ°Π³Π½ΠΈΡΠ½ΠΎΠ³ΠΎ ΠΏΠΎΡΠΎΠΊΠ°, ΠΈ ΠΎΠΏΡΠ΅Π΄Π΅Π»ΡΠ΅ΡΡΡ Π»ΠΈΡΡ ΡΠΊΠΎΡΠΎΡΡΡΡ Π΅Π³ΠΎ ΠΈΠ·ΠΌΠ΅Π½Π΅Π½ΠΈΡ, Ρ.Π΅. Π²Π΅Π»ΠΈΡΠΈΠ½ΠΎΠΉ. ΠΠ·ΠΌΠ΅Π½Π΅Π½ΠΈΠ΅ Π·Π½Π°ΠΊΠ° ΠΏΡΠΎΠΈΠ·Π²ΠΎΠ΄Π½ΠΎΠΉΠΏΡΠΈΠ²ΠΎΠ΄ΠΈΡ ΠΊ ΠΈΠ·ΠΌΠ΅Π½Π΅Π½ΠΈΡ Π·Π½Π°ΠΊΠ°Ρ.Π΄.Ρ. ΠΈΠ½Π΄ΡΠΊΡΠΈΠΈ . |
Π ΠΈΡΡΠ½ΠΎΠΊ 26.1. |
Π€Π°ΡΠ°Π΄Π΅ΠΉ ΠΎΠ±Π½Π°ΡΡΠΆΠΈΠ», ΡΡΠΎ ΠΈΠ½Π΄ΡΠΊΡΠΈΠΎΠ½Π½ΡΠΉ
ΡΠΎΠΊ ΠΌΠΎΠΆΠ½ΠΎ Π²ΡΠ·Π²Π°ΡΡ Π΄Π²ΡΠΌΡ ΡΠ°Π·Π»ΠΈΡΠ½ΡΠΌΠΈ
ΡΠΏΠΎΡΠΎΠ±Π°ΠΌΠΈ, ΠΊΠΎΡΠΎΡΡΠ΅ ΡΠ΄ΠΎΠ±Π½ΠΎ ΠΎΠ±ΡΡΡΠ½ΠΈΡΡ Ρ
ΠΏΠΎΠΌΠΎΡΡΡ ΡΠΈΡΡΠ½ΠΊΠ°.
1-ΠΉ ΡΠΏΠΎΡΠΎΠ±: ΠΏΠ΅ΡΠ΅ΠΌΠ΅ΡΠ΅Π½ΠΈΠ΅ ΡΠ°ΠΌΠΊΠΈ Π² ΠΌΠ°Π³Π½ΠΈΡΠ½ΠΎΠΌ ΠΏΠΎΠ»Π΅ Π½Π΅ΠΏΠΎΠ΄Π²ΠΈΠΆΠ½ΠΎΠΉ ΠΊΠ°ΡΡΡΠΊΠΈ(ΡΠΌ.
ΡΠΈΡ.26.1).
2-ΠΉ ΡΠΏΠΎΡΠΎΠ±: ΠΈΠ·ΠΌΠ΅Π½Π΅Π½ΠΈΠ΅ ΠΌΠ°Π³Π½ΠΈΡΠ½ΠΎΠ³ΠΎ ΠΏΠΎΠ»Ρ
,
ΡΠΎΠ·Π΄Π°Π²Π°Π΅ΠΌΠΎΠ³ΠΎ ΠΊΠ°ΡΡΡΠΊΠΎΠΉ,
Π·Π° ΡΡΠ΅Ρ Π΅Π΅ Π΄Π²ΠΈΠΆΠ΅Π½ΠΈΡ ΠΈΠ»ΠΈ Π²ΡΠ»Π΅Π΄ΡΡΠ²ΠΈΠ΅
ΠΈΠ·ΠΌΠ΅Π½Π΅Π½ΠΈΡ ΡΠΈΠ»Ρ ΡΠΎΠΊΠ°Π² Π½Π΅ΠΉ (ΠΈΠ»ΠΈ ΡΠΎΠ³ΠΎ ΠΈ Π΄ΡΡΠ³ΠΎΠ³ΠΎ Π²ΠΌΠ΅ΡΡΠ΅). Π Π°ΠΌΠΊΠ°ΠΏΡΠΈ ΡΡΠΎΠΌ Π½Π΅ΠΏΠΎΠ΄Π²ΠΈΠΆΠ½Π°.
Π ΠΎΠ±ΠΎΠΈΡ
ΡΡΠΈΡ
ΡΠ»ΡΡΠ°ΡΡ
Π³Π°Π»ΡΠ²Π°Π½ΠΎΠΌΠ΅ΡΡ Π±ΡΠ΄Π΅Ρ ΠΏΠΎΠΊΠ°Π·ΡΠ²Π°ΡΡ Π½Π°Π»ΠΈΡΠΈΠ΅ ΠΈΠ½Π΄ΡΠΊΡΠΈΠΎΠ½Π½ΠΎΠ³ΠΎ
ΡΠΎΠΊΠ° Π² ΡΠ°ΠΌΠΊΠ΅.
Π‘ΠΎΠ²Π΅Ρ
ΠΠ°ΠΏΡΠ°Π²Π»Π΅Π½ΠΈΠ΅ ΠΈΠ½Π΄ΡΠΊΡΠΈΠΎΠ½Π½ΠΎΠ³ΠΎ ΡΠΎΠΊΠ° ΠΈ,
ΡΠΎΠΎΡΠ²Π΅ΡΡΡΠ²Π΅Π½Π½ΠΎ, Π·Π½Π°ΠΊ Ρ.Π΄.Ρ. ΠΈΠ½Π΄ΡΠΊΡΠΈΠΈ ΠΎΠΏΡΠ΅Π΄Π΅Π»ΡΡΡΡΡ ΠΏΡΠ°Π²ΠΈΠ»ΠΎΠΌ ΠΠ΅Π½ΡΠ°.
ΠΡΠ°Π²ΠΈΠ»ΠΎ ΠΠ΅Π½ΡΠ°.
ΠΠ½Π΄ΡΠΊΡΠΈΠΎΠ½Π½ΡΠΉ
ΡΠΎΠΊ Π²ΡΠ΅Π³Π΄Π° Π½Π°ΠΏΡΠ°Π²Π»Π΅Π½ ΡΠ°ΠΊ, ΡΡΠΎΠ±Ρ
ΠΏΡΠΎΡΠΈΠ²ΠΎΠ΄Π΅ΠΉΡΡΠ²ΠΎΠ²Π°ΡΡ ΠΏΡΠΈΡΠΈΠ½Π΅, Π΅Π³ΠΎ
Π²ΡΠ·ΡΠ²Π°ΡΡΠ΅ΠΉ.
ΠΡΠ°Π²ΠΈΠ»ΠΎ ΠΠ΅Π½ΡΠ° Π²ΡΡΠ°ΠΆΠ°Π΅Ρ Π²Π°ΠΆΠ½ΠΎΠ΅
ΡΠΈΠ·ΠΈΡΠ΅ΡΠΊΠΎΠ΅ ΡΠ²ΠΎΠΉΡΡΠ²ΠΎ β ΡΡΡΠ΅ΠΌΠ»Π΅Π½ΠΈΠ΅
ΡΠΈΡΡΠ΅ΠΌΡ ΠΏΡΠΎΡΠΈΠ²ΠΎΠ΄Π΅ΠΉΡΡΠ²ΠΎΠ²Π°ΡΡ ΠΈΠ·ΠΌΠ΅Π½Π΅Π½ΠΈΡ
Π΅Π΅ ΡΠΎΡΡΠΎΡΠ½ΠΈΡ. ΠΡΠΎ ΡΠ²ΠΎΠΉΡΡΠ²ΠΎ Π½Π°Π·ΡΠ²Π°ΡΡ
ΡΠ»Π΅ΠΊΡΡΠΎΠΌΠ°Π³Π½ΠΈΡΠ½ΠΎΠΉ
ΠΈΠ½Π΅ΡΡΠΈΠ΅ΠΉ.
ΠΠ°ΠΊΠΎΠ²Π° Π±Ρ Π½ΠΈ Π±ΡΠ»Π° ΠΏΡΠΈΡΠΈΠ½Π°
ΠΈΠ·ΠΌΠ΅Π½Π΅Π½ΠΈΡ ΠΌΠ°Π³Π½ΠΈΡΠ½ΠΎΠ³ΠΎ ΠΏΠΎΡΠΎΠΊΠ°, ΠΎΡ
Π²Π°ΡΡΠ²Π°Π΅ΠΌΠΎΠ³ΠΎ
Π·Π°ΠΌΠΊΠ½ΡΡΡΠΌ ΠΏΡΠΎΠ²ΠΎΠ΄ΡΡΠΈΠΌ ΠΊΠΎΠ½ΡΡΡΠΎΠΌ, Π²ΠΎΠ·Π½ΠΈΠΊΠ°ΡΡΠ°Ρ
Π² ΠΊΠΎΠ½ΡΡΡΠ΅ Ρ.Π΄.Ρ. ΠΈΠ½Π΄ΡΠΊΡΠΈΠΈ ΠΎΠΏΡΠ΅Π΄Π΅Π»ΡΠ΅ΡΡΡ
ΡΠΎΡΠΌΡΠ»ΠΎΠΉ
(26.1) |
ΠΡΠΈΡΠΎΠ΄Π° ΡΠ»Π΅ΠΊΡΡΠΎΠΌΠ°Π³Π½ΠΈΡΠ½ΠΎΠΉ ΠΈΠ½Π΄ΡΠΊΡΠΈΠΈ.
Π‘ ΡΠ΅Π»ΡΡ Π²ΡΡΡΠ½Π΅Π½ΠΈΡ ΡΠΈΠ·ΠΈΡΠ΅ΡΠΊΠΈΡ
ΠΏΡΠΈΡΠΈΠ½,
ΠΊΠΎΡΠΎΡΡΠ΅ ΠΏΡΠΈΠ²ΠΎΠ΄ΡΡ ΠΊ Π²ΠΎΠ·Π½ΠΈΠΊΠ½ΠΎΠ²Π΅Π½ΠΈΡ Ρ.Π΄.Ρ.
ΠΈΠ½Π΄ΡΠΊΡΠΈΠΈ, ΠΏΠΎΡΠ»Π΅Π΄ΠΎΠ²Π°ΡΠ΅Π»ΡΠ½ΠΎ ΡΠ°ΡΡΠΌΠΎΡΡΠΈΠΌ
Π΄Π²Π° ΡΠ»ΡΡΠ°Ρ.
1. ΠΠΎΠ½ΡΡΡ Π΄Π²ΠΈΠΆΠ΅ΡΡΡ Π² ΠΏΠΎΡΡΠΎΡΠ½Π½ΠΎΠΌ ΠΌΠ°Π³Π½ΠΈΡΠ½ΠΎΠΌ ΠΏΠΎΠ»Π΅
ΠΡΡΡΡ ΠΊΠΎΠ½ΡΡΡ Ρ ΠΏΠΎΠ΄Π²ΠΈΠΆΠ½ΠΎΠΉ ΠΏΠ΅ΡΠ΅ΠΌΡΡΠΊΠΎΠΉ Π΄Π»ΠΈΠ½ΠΎΠΉ Π½Π°Ρ ΠΎΠ΄ΠΈΡΡΡ Π² ΠΌΠ°Π³Π½ΠΈΡΠ½ΠΎΠΌ ΠΏΠΎΠ»Π΅, ΠΏΠ΅ΡΠΏΠ΅Π½Π΄ΠΈΠΊΡΠ»ΡΡΠ½ΠΎΠΌ ΠΏΠ»ΠΎΡΠΊΠΎΡΡΠΈ ΠΊΠΎΠ½ΡΡΡΠ° (ΡΠΌ.Π ΠΈΡΡΠ½ΠΎΠΊ 26.2). ΠΡΠ»ΠΈ Π΄Π²ΠΈΠ³Π°ΡΡ ΠΏΠ΅ΡΠ΅ΠΌΡΡΠΊΡ ΡΠΎ ΡΠΊΠΎΡΠΎΡΡΡΡΠ²ΠΏΡΠ°Π²ΠΎ, ΡΠΎ Ρ ΡΠ°ΠΊΠΎΠΉ ΠΆΠ΅ ΡΠΊΠΎΡΠΎΡΡΡΡ Π½Π°ΡΠ½ΡΡ Π΄Π²ΠΈΠ³Π°ΡΡΡΡ ΠΈ Π½ΠΎΡΠΈΡΠ΅Π»ΠΈ ΡΠΎΠΊΠ° Π² ΠΏΠ΅ΡΠ΅ΠΌΡΡΠΊΠ΅ β ΡΠ»Π΅ΠΊΡΡΠΎΠ½Ρ. Π ΡΠ΅Π·ΡΠ»ΡΡΠ°ΡΠ΅ Π½Π° ΠΊΠ°ΠΆΠ΄ΡΠΉ ΡΠ»Π΅ΠΊΡΡΠΎΠ½ Π½Π°ΡΠΈΠ½Π°Π΅Ρ |
Π ΠΈΡΡΠ½ΠΎΠΊ 26.2 |
Π΄Π΅ΠΉΡΡΠ²ΠΎΠ²Π°ΡΡ ΡΠΈΠ»Π°
Π²ΡΠ·ΡΠ²Π°ΡΡΠ°Ρ ΠΏΠ΅ΡΠ΅ΠΌΠ΅ΡΠ΅Π½ΠΈΠ΅ ΡΠ»Π΅ΠΊΡΡΠΎΠ½ΠΎΠ² ΠΏΠΎ
ΠΏΠ΅ΡΠ΅ΠΌΡΡΠΊΠ΅ Π²Π½ΠΈΠ·, Ρ.Π΅. ΠΏΠΎΡΠ΅ΡΠ΅Ρ ΡΠΎΠΊ,
Π½Π°ΠΏΡΠ°Π²Π»Π΅Π½Π½ΡΠΉ Π²Π²Π΅ΡΡ
.
ΠΠ΅ΡΠ΅ΡΠ°ΡΠΏΡΠ΅Π΄Π΅Π»ΠΈΠ²ΡΠΈΠ΅ΡΡ Π·Π°ΡΡΠ΄Ρ ΡΠΎΠ·Π΄Π°Π΄ΡΡ
ΡΠ»Π΅ΠΊΡΡΠΈΡΠ΅ΡΠΊΠΎΠ΅ ΠΏΠΎΠ»Π΅, ΠΊΠΎΡΠΎΡΠΎΠ΅ Π²ΠΎΠ·Π±ΡΠ΄ΠΈΡ
ΡΠΎΠΊ ΠΈ Π² ΠΎΡΡΠ°Π»ΡΠ½ΡΡ
ΡΡΠ°ΡΡΠΊΠ°Ρ
ΠΊΠΎΠ½ΡΡΡΠ°.
ΠΡΠΎ ΠΈ Π΅ΡΡΡ ΠΈΠ½Π΄ΡΠΊΡΠΈΠΎΠ½Π½ΡΠΉ ΡΠΎΠΊ.
ΠΠ°Π³Π½ΠΈΡΠ½Π°Ρ ΡΠΈΠ»Π° ΠΈΠ³ΡΠ°Π΅Ρ ΡΠΎΠ»Ρ ΡΡΠΎΡΠΎΠ½Π½Π΅ΠΉ ΡΠΈΠ»Ρ. ΠΠΉ ΠΌΠΎΠΆΠ½ΠΎ
ΡΠΎΠΏΠΎΡΡΠ°Π²ΠΈΡΡ ΡΠΊΠ²ΠΈΠ²Π°Π»Π΅Π½ΡΠ½ΠΎΠ΅ ΠΏΠΎΠ»Π΅ ΡΡΠΎΡΠΎΠ½Π½ΠΈΡ
ΡΠΈΠ»
. | (26.2) |
ΠΠ»Π΅ΠΊΡΡΠΎΠ΄Π²ΠΈΠΆΡΡΠ°Ρ ΡΠΈΠ»Π°,
ΡΠΎΠ·Π΄Π°Π²Π°Π΅ΠΌΠ°Ρ ΡΡΠΈΠΌ ΠΏΠΎΠ»Π΅ΠΌ, Π½Π°Π·ΡΠ²Π°Π΅ΡΡΡ
ΡΠ»Π΅ΠΊΡΡΠΎΠ΄Π²ΠΈΠΆΡΡΠ΅ΠΉ
ΡΠΈΠ»ΠΎΠΉ ΠΈΠ½Π΄ΡΠΊΡΠΈΠΈ
.
Π Π½Π°ΡΠ΅ΠΌ ΡΠ»ΡΡΠ°Π΅
. | (26.3) |
ΠΠ΄Π΅ΡΡ Π·Π½Π°ΠΊ Β«ΠΌΠΈΠ½ΡΡΒ» ΠΏΠΎΡΡΠ°Π²Π»Π΅Π½
ΠΏΠΎΡΠΎΠΌΡ, ΡΡΠΎ ΡΡΠΎΡΠΎΠ½Π½Π΅Π΅ ΠΏΠΎΠ»Π΅ Π½Π°ΠΏΡΠ°Π²Π»Π΅Π½ΠΎ ΠΏΡΠΎΡΠΈΠ² ΠΏΠΎΠ»ΠΎΠΆΠΈΡΠ΅Π»ΡΠ½ΠΎΠ³ΠΎ ΠΎΠ±Ρ
ΠΎΠ΄Π°
ΠΊΠΎΠ½ΡΡΡΠ°, ΠΎΠΏΡΠ΅Π΄Π΅Π»ΡΠ΅ΠΌΠΎΠ³ΠΎ ΠΏΡΠ°Π²ΠΈΠ»ΠΎΠΌ ΠΏΡΠ°Π²ΠΎΠ³ΠΎ
Π²ΠΈΠ½ΡΠ°. ΠΡΠΎΠΈΠ·Π²Π΅Π΄Π΅Π½ΠΈΠ΅Π΅ΡΡΡ ΡΠΊΠΎΡΠΎΡΡΡ ΠΏΡΠΈΡΠ°ΡΠ΅Π½ΠΈΡ ΠΏΠ»ΠΎΡΠ°Π΄ΠΈ ΠΊΠΎΠ½ΡΡΡΠ°
(ΠΏΡΠΈΡΠ°ΡΠ΅Π½ΠΈΠ΅ ΠΏΠ»ΠΎΡΠ°Π΄ΠΈ Π² Π΅Π΄ΠΈΠ½ΠΈΡΡ Π²ΡΠ΅ΠΌΠ΅Π½ΠΈ),
ΠΏΠΎΡΡΠΎΠΌΡ
Π³Π΄Π΅
β ΠΏΡΠΈΡΠ°ΡΠ΅Π½ΠΈΠ΅ ΠΌΠ°Π³Π½ΠΈΡΠ½ΠΎΠ³ΠΎ ΠΏΠΎΡΠΎΠΊΠ° ΡΠΊΠ²ΠΎΠ·Ρ
ΠΊΠΎΠ½ΡΡΡ.
Π’ΠΎΠ³Π΄Π°,
. | (26.4) |
ΠΠΎΠ»ΡΡΠ΅Π½Π½ΡΠΉ ΡΠ΅Π·ΡΠ»ΡΡΠ°Ρ ΠΌΠΎΠΆΠ½ΠΎ ΠΎΠ±ΠΎΠ±ΡΠΈΡΡ
Π½Π° ΡΠ»ΡΡΠ°ΠΉ ΠΏΡΠΎΠΈΠ·Π²ΠΎΠ»ΡΠ½ΠΎΠΉ ΠΎΡΠΈΠ΅Π½ΡΠ°ΡΠΈΠΈ
Π²Π΅ΠΊΡΠΎΡΠ° ΠΈΠ½Π΄ΡΠΊΡΠΈΠΈ ΠΌΠ°Π³Π½ΠΈΡΠ½ΠΎΠ³ΠΎ ΠΏΠΎΠ»Ρ ΠΎΡΠ½ΠΎΡΠΈΡΠ΅Π»ΡΠ½ΠΎ ΠΏΠ»ΠΎΡΠΊΠΎΡΡΠΈ ΠΊΠΎΠ½ΡΡΡΠ° ΠΈ Π½Π°
Π»ΡΠ±ΠΎΠΉ ΠΊΠΎΠ½ΡΡΡ, Π΄Π²ΠΈΠΆΡΡΠΈΠΉΡΡ (ΠΈ/ΠΈΠ»ΠΈ
Π΄Π΅ΡΠΎΡΠΌΠΈΡΡΠ΅ΠΌΡΠΉ) ΠΏΡΠΎΠΈΠ·Π²ΠΎΠ»ΡΠ½ΡΠΌ ΠΎΠ±ΡΠ°Π·ΠΎΠΌ Π²
ΠΏΠΎΡΡΠΎΡΠ½Π½ΠΎΠΌ Π½Π΅ΠΎΠ΄Π½ΠΎΡΠΎΠ΄Π½ΠΎΠΌ Π²Π½Π΅ΡΠ½Π΅ΠΌ
ΠΌΠ°Π³Π½ΠΈΡΠ½ΠΎΠΌ ΠΏΠΎΠ»Π΅.
ΠΡΠ°ΠΊ, Π²ΠΎΠ·Π±ΡΠΆΠ΄Π΅Π½ΠΈΠ΅ Ρ.Π΄.Ρ. ΠΈΠ½Π΄ΡΠΊΡΠΈΠΈ ΠΏΡΠΈ
Π΄Π²ΠΈΠΆΠ΅Π½ΠΈΠΈ ΠΊΠΎΠ½ΡΡΡΠ° Π² ΠΏΠΎΡΡΠΎΡΠ½Π½ΠΎΠΌ ΠΌΠ°Π³Π½ΠΈΡΠ½ΠΎΠΌ
ΠΏΠΎΠ»Π΅ ΠΎΠ±ΡΡΡΠ½ΡΠ΅ΡΡΡ Π΄Π΅ΠΉΡΡΠ²ΠΈΠ΅ΠΌ ΠΌΠ°Π³Π½ΠΈΡΠ½ΠΎΠΉ
ΡΠΎΡΡΠ°Π²Π»ΡΡΡΠ΅ΠΉ ΡΠΈΠ»Ρ ΠΠΎΡΠ΅Π½ΡΠ°, ΠΏΡΠΎΠΏΠΎΡΡΠΈΠΎΠ½Π°Π»ΡΠ½ΠΎΠΉ ,
ΠΊΠΎΡΠΎΡΠ°Ρ Π²ΠΎΠ·Π½ΠΈΠΊΠ°Π΅Ρ ΠΏΡΠΈ ΠΏΠ΅ΡΠ΅ΠΌΠ΅ΡΠ΅Π½ΠΈΠΈ
ΠΏΡΠΎΠ²ΠΎΠ΄Π½ΠΈΠΊΠ°.
2.
ΠΠΎΠ½ΡΡΡ ΠΏΠΎΠΊΠΎΠΈΡΡΡ Π² ΠΏΠ΅ΡΠ΅ΠΌΠ΅Π½Π½ΠΎΠΌ ΠΌΠ°Π³Π½ΠΈΡΠ½ΠΎΠΌ ΠΏΠΎΠ»Π΅ ΠΠ°Π±Π»ΡΠ΄Π°Π΅ΠΌΠΎΠ΅ Π½Π° ΠΎΠΏΡΡΠ΅ Π²ΠΎΠ·Π½ΠΈΠΊΠ½ΠΎΠ²Π΅Π½ΠΈΠ΅
ΠΈΠ½Π΄ΡΠΊΡΠΈΠΎΠ½Π½ΠΎΠ³ΠΎ ΡΠΎΠΊΠ° ΡΠ²ΠΈΠ΄Π΅ΡΠ΅Π»ΡΡΡΠ²ΡΠ΅Ρ ΠΎ
ΡΠΎΠΌ, ΡΡΠΎ ΠΈ Π² ΡΡΠΎΠΌ ΡΠ»ΡΡΠ°Π΅ Π² ΠΊΠΎΠ½ΡΡΡΠ΅
ΠΏΠΎΡΠ²Π»ΡΡΡΡΡ ΡΡΠΎΡΠΎΠ½Π½ΠΈΠ΅ ΡΠΈΠ»Ρ, ΠΊΠΎΡΠΎΡΡΠ΅
ΡΠ΅ΠΏΠ΅ΡΡ ΡΠ²ΡΠ·Π°Π½Ρ Ρ ΠΈΠ·ΠΌΠ΅Π½ΡΡΡΠΈΠΌΡΡ Π²ΠΎ Π²ΡΠ΅ΠΌΠ΅Π½ΠΈ
ΠΌΠ°Π³Π½ΠΈΡΠ½ΡΠΌ ΠΏΠΎΠ»Π΅ΠΌ. ΠΠ°ΠΊΠΎΠ²Π° ΠΆΠ΅ ΠΈΡ
ΠΏΡΠΈΡΠΎΠ΄Π°?
ΠΡΠ²Π΅Ρ Π½Π° ΡΡΠΎΡ ΠΏΡΠΈΠ½ΡΠΈΠΏΠΈΠ°Π»ΡΠ½ΡΠΉ Π²ΠΎΠΏΡΠΎΡ
Π±ΡΠ» Π΄Π°Π½ ΠΠ°ΠΊΡΠ²Π΅Π»Π»ΠΎΠΌ.
ΠΠΎΡΠΊΠΎΠ»ΡΠΊΡ ΠΏΡΠΎΠ²ΠΎΠ΄Π½ΠΈΠΊ ΠΏΠΎΠΊΠΎΠΈΡΡΡ, ΡΠΎ ΡΠΊΠΎΡΠΎΡΡΡ
ΡΠΏΠΎΡΡΠ΄ΠΎΡΠ΅Π½Π½ΠΎΠ³ΠΎ Π΄Π²ΠΈΠΆΠ΅Π½ΠΈΡ ΡΠ»Π΅ΠΊΡΡΠΈΡΠ΅ΡΠΊΠΈΡ
Π·Π°ΡΡΠ΄ΠΎΠ² ΠΈ, ΡΠ»Π΅Π΄ΠΎΠ²Π°ΡΠ΅Π»ΡΠ½ΠΎ, ΠΌΠ°Π³Π½ΠΈΡΠ½Π°Ρ ΡΠΈΠ»Π°,
ΠΏΡΠΎΠΏΠΎΡΡΠΈΠΎΠ½Π°Π»ΡΠ½Π°Ρ,
ΡΠ°ΠΊΠΆΠ΅ ΡΠ°Π²Π½Π° Π½ΡΠ»Ρ ΠΈ ΡΠΆΠ΅ Π½Π΅ ΠΌΠΎΠΆΠ΅Ρ ΠΏΡΠΈΠ²Π΅ΡΡΠΈ
Π·Π°ΡΡΠ΄Ρ Π² Π΄Π²ΠΈΠΆΠ΅Π½ΠΈΠ΅.
ΠΠ΄Π½Π°ΠΊΠΎ ΠΊΡΠΎΠΌΠ΅ ΠΌΠ°Π³Π½ΠΈΡΠ½ΠΎΠΉ
ΡΠΈΠ»Ρ Π½Π° ΡΠ»Π΅ΠΊΡΡΠΈΡΠ΅ΡΠΊΠΈΠΉ Π·Π°ΡΡΠ΄ ΠΌΠΎΠΆΠ΅Ρ
Π΄Π΅ΠΉΡΡΠ²ΠΎΠ²Π°ΡΡ ΡΠΎΠ»ΡΠΊΠΎ ΡΠΈΠ»Π° ΡΠΎ ΡΡΠΎΡΠΎΠ½Ρ
ΡΠ»Π΅ΠΊΡΡΠΈΡΠ΅ΡΠΊΠΎΠ³ΠΎ ΠΏΠΎΠ»Ρ, ΡΠ°Π²Π½Π°Ρ.
ΠΠΎΡΡΠΎΠΌΡ ΠΎΡΡΠ°Π΅ΡΡΡ Π·Π°ΠΊΠ»ΡΡΠΈΡΡ, ΡΡΠΎΠΈΠ½Π΄ΡΠΊΡΠΈΠΎΠ½Π½ΡΠΉ ΡΠΎΠΊ ΠΎΠ±ΡΡΠ»ΠΎΠ²Π»Π΅Π½
ΡΠ»Π΅ΠΊΡΡΠΈΡΠ΅ΡΠΊΠΈΠΌ ΠΏΠΎΠ»Π΅ΠΌ ,
Π²ΠΎΠ·Π½ΠΈΠΊΠ°ΡΡΠΈΠΌ ΠΏΡΠΈ ΠΈΠ·ΠΌΠ΅Π½Π΅Π½ΠΈΠΈ Π²ΠΎ Π²ΡΠ΅ΠΌΠ΅Π½ΠΈ
Π²Π½Π΅ΡΠ½Π΅Π³ΠΎ ΠΌΠ°Π³Π½ΠΈΡΠ½ΠΎΠ³ΠΎ ΠΏΠΎΠ»Ρ.
ΠΠΌΠ΅Π½Π½ΠΎ
ΡΡΠΎ ΡΠ»Π΅ΠΊΡΡΠΈΡΠ΅ΡΠΊΠΎΠ΅ ΠΏΠΎΠ»Π΅ ΠΈ ΠΎΡΠ²Π΅ΡΡΡΠ²Π΅Π½Π½ΠΎ
Π·Π° ΠΏΠΎΡΠ²Π»Π΅Π½ΠΈΠ΅ Ρ.Π΄.Ρ. ΠΈΠ½Π΄ΡΠΊΡΠΈΠΈ Π² Π½Π΅ΠΏΠΎΠ΄Π²ΠΈΠΆΠ½ΠΎΠΌ
ΠΊΠΎΠ½ΡΡΡΠ΅. Π‘ΠΎΠ³Π»Π°ΡΠ½ΠΎ ΠΠ°ΠΊΡΠ²Π΅Π»Π»Ρ,ΠΈΠ·ΠΌΠ΅Π½ΡΡΡΠ΅Π΅ΡΡ
Π²ΠΎ Π²ΡΠ΅ΠΌΠ΅Π½ΠΈ ΠΌΠ°Π³Π½ΠΈΡΠ½ΠΎΠ΅ ΠΏΠΎΠ»Π΅ ΠΏΠΎΡΠΎΠΆΠ΄Π°Π΅Ρ Π²
ΠΎΠΊΡΡΠΆΠ°ΡΡΠ΅ΠΌ ΠΏΡΠΎΡΡΡΠ°Π½ΡΡΠ²Π΅ ΡΠ»Π΅ΠΊΡΡΠΈΡΠ΅ΡΠΊΠΎΠ΅
ΠΏΠΎΠ»Π΅.
ΠΠΎΠ·Π½ΠΈΠΊΠ½ΠΎΠ²Π΅Π½ΠΈΠ΅ ΡΠ»Π΅ΠΊΡΡΠΈΡΠ΅ΡΠΊΠΎΠ³ΠΎ
ΠΏΠΎΠ»Ρ Π½Π΅ ΡΠ²ΡΠ·Π°Π½ΠΎ Ρ Π½Π°Π»ΠΈΡΠΈΠ΅ΠΌ ΠΏΡΠΎΠ²ΠΎΠ΄ΡΡΠ΅Π³ΠΎ
ΠΊΠΎΠ½ΡΡΡΠ°, ΠΊΠΎΡΠΎΡΡΠΉ Π»ΠΈΡΡ ΠΏΠΎΠ·Π²ΠΎΠ»ΡΠ΅Ρ ΠΎΠ±Π½Π°ΡΡΠΆΠΈΡΡ
ΠΏΠΎ Π²ΠΎΠ·Π½ΠΈΠΊΠ½ΠΎΠ²Π΅Π½ΠΈΡ Π² Π½Π΅ΠΌ ΠΈΠ½Π΄ΡΠΊΡΠΈΠΎΠ½Π½ΠΎΠ³ΠΎ
ΡΠΎΠΊΠ° ΡΡΡΠ΅ΡΡΠ²ΠΎΠ²Π°Π½ΠΈΠ΅ ΡΡΠΎΠ³ΠΎ ΠΏΠΎΠ»Ρ.
Π€ΠΎΡΠΌΡΠ»ΠΈΡΠΎΠ²ΠΊΠ° Π·Π°ΠΊΠΎΠ½Π°
ΡΠ»Π΅ΠΊΡΡΠΎΠΌΠ°Π³Π½ΠΈΡΠ½ΠΎΠΉ ΠΈΠ½Π΄ΡΠΊΡΠΈΠΈ,
Π΄Π°Π½Π½Π°Ρ ΠΠ°ΠΊΡΠ²Π΅Π»Π»ΠΎΠΌ, ΠΏΡΠΈΠ½Π°Π΄Π»Π΅ΠΆΠΈΡ ΠΊ ΡΠΈΡΠ»Ρ
Π½Π°ΠΈΠ±ΠΎΠ»Π΅Π΅ Π²Π°ΠΆΠ½ΡΡ
ΠΎΠ±ΠΎΠ±ΡΠ΅Π½ΠΈΠΉ ΡΠ»Π΅ΠΊΡΡΠΎΠ΄ΠΈΠ½Π°ΠΌΠΈΠΊΠΈ.
ΠΡΡΠΊΠΎΠ΅ ΠΈΠ·ΠΌΠ΅Π½Π΅Π½ΠΈΠ΅
ΠΌΠ°Π³Π½ΠΈΡΠ½ΠΎΠ³ΠΎ ΠΏΠΎΠ»Ρ Π²ΠΎ Π²ΡΠ΅ΠΌΠ΅Π½ΠΈ Π²ΠΎΠ·Π±ΡΠΆΠ΄Π°Π΅Ρ
Π² ΠΎΠΊΡΡΠΆΠ°ΡΡΠ΅ΠΌ ΠΏΡΠΎΡΡΡΠ°Π½ΡΡΠ²Π΅ ΡΠ»Π΅ΠΊΡΡΠΈΡΠ΅ΡΠΊΠΎΠ΅
ΠΏΠΎΠ»Π΅.
ΠΠ°ΡΠ΅ΠΌΠ°ΡΠΈΡΠ΅ΡΠΊΠ°Ρ ΡΠΎΡΠΌΡΠ»ΠΈΡΠΎΠ²ΠΊΠ° Π·Π°ΠΊΠΎΠ½Π°
ΡΠ»Π΅ΠΊΡΡΠΎΠΌΠ°Π³Π½ΠΈΡΠ½ΠΎΠΉ ΠΈΠ½Π΄ΡΠΊΡΠΈΠΈ Π² ΠΏΠΎΠ½ΠΈΠΌΠ°Π½ΠΈΠΈ
ΠΠ°ΠΊΡΠ²Π΅Π»Π»Π° ΠΈΠΌΠ΅Π΅Ρ Π²ΠΈΠ΄:
Π¦ΠΈΡΠΊΡΠ»ΡΡΠΈΡ Π²Π΅ΠΊΡΠΎΡΠ°
Π½Π°ΠΏΡΡΠΆΠ΅Π½Π½ΠΎΡΡΠΈ ΡΡΠΎΠ³ΠΎ ΠΏΠΎΠ»Ρ ΠΏΠΎ Π»ΡΠ±ΠΎΠΌΡ Π½Π΅ΠΏΠΎΠ΄Π²ΠΈΠΆΠ½ΠΎΠΌΡ
Π·Π°ΠΌΠΊΠ½ΡΡΠΎΠΌΡ ΠΊΠΎΠ½ΡΡΡΡΠΎΠΏΡΠ΅Π΄Π΅Π»ΡΠ΅ΡΡΡ Π²ΡΡΠ°ΠΆΠ΅Π½ΠΈΠ΅ΠΌ
, | (26.5) |
Π³Π΄Π΅ β ΠΌΠ°Π³Π½ΠΈΡΠ½ΡΠΉ ΠΏΠΎΡΠΎΠΊ, ΠΏΡΠΎΠ½ΠΈΠ·ΡΠ²Π°ΡΡΠΈΠΉ ΠΊΠΎΠ½ΡΡΡ.
ΠΡΠΏΠΎΠ»ΡΠ·ΡΠ΅ΠΌΡΠΉ Π΄Π»Ρ ΠΎΠ±ΠΎΠ·Π½Π°ΡΠ΅Π½ΠΈΡ ΡΠΊΠΎΡΠΎΡΡΠΈ
ΠΈΠ·ΠΌΠ΅Π½Π΅Π½ΠΈΡ ΠΌΠ°Π³Π½ΠΈΡΠ½ΠΎΠ³ΠΎ ΠΏΠΎΡΠΎΠΊΠ° Π·Π½Π°ΠΊ ΡΠ°ΡΡΠ½ΠΎΠΉ
ΠΏΡΠΎΠΈΠ·Π²ΠΎΠ΄Π½ΠΎΠΉ ΡΠΊΠ°Π·ΡΠ²Π°Π΅Ρ Π½Π° ΡΠΎ, ΡΡΠΎ ΠΊΠΎΠ½ΡΡΡ
ΡΠ²Π»ΡΠ΅ΡΡΡ Π½Π΅ΠΏΠΎΠ΄Π²ΠΈΠΆΠ½ΡΠΌ.
ΠΠΎΡΠΎΠΊ Π²Π΅ΠΊΡΠΎΡΠ° ΡΠ΅ΡΠ΅Π· ΠΏΠΎΠ²Π΅ΡΡ
Π½ΠΎΡΡΡ, ΠΎΠ³ΡΠ°Π½ΠΈΡΠ΅Π½Π½ΡΡ ΠΊΠΎΠ½ΡΡΡΠΎΠΌ,
ΡΠ°Π²Π΅Π½,
ΠΏΠΎΡΡΠΎΠΌΡ Π²ΡΡΠ°ΠΆΠ΅Π½ΠΈΠ΅ Π·Π°ΠΊΠΎΠ½Π° ΡΠ»Π΅ΠΊΡΡΠΎΠΌΠ°Π³Π½ΠΈΡΠ½ΠΎΠΉ
ΠΈΠ½Π΄ΡΠΊΡΠΈΠΈ ΠΌΠΎΠΆΠ½ΠΎ ΠΏΠ΅ΡΠ΅ΠΏΠΈΡΠ°ΡΡ ΡΠ»Π΅Π΄ΡΡΡΠΈΠΌ
ΠΎΠ±ΡΠ°Π·ΠΎΠΌ:
. | (26.6) |
ΠΠΎΡΠΏΠΎΠ»ΡΠ·ΠΎΠ²Π°Π²ΡΠΈΡΡ ΡΠ΅ΠΎΡΠ΅ΠΌΠΎΠΉ Π‘ΡΠΎΠΊΡΠ° ΠΌΠΎΠΆΠ½ΠΎ
ΠΏΠΎΠ»ΡΡΠΈΡΡ Π·Π°ΠΊΠΎΠ½ ΡΠ»Π΅ΠΊΡΡΠΎΠΌΠ°Π³Π½ΠΈΡΠ½ΠΎΠΉ ΠΈΠ½Π΄ΡΠΊΡΠΈΠΈ
Π² Π΄ΠΈΡΡΠ΅ΡΠ΅Π½ΡΠΈΠ°Π»ΡΠ½ΠΎΠΉ ΡΠΎΡΠΌΠ΅:
. | (26.7) |
ΠΡΠΎ ΠΎΠ΄Π½ΠΎ ΠΈΠ· ΡΡΠ°Π²Π½Π΅Π½ΠΈΠΉ ΡΠΈΡΡΠ΅ΠΌΡ ΡΡΠ°Π²Π½Π΅Π½ΠΈΠΉ
ΠΠ°ΠΊΡΠ²Π΅Π»Π»Π°.
Π’ΠΎΡ ΡΠ°ΠΊΡ, ΡΡΠΎ ΡΠΈΡΠΊΡΠ»ΡΡΠΈΡ ΡΠ»Π΅ΠΊΡΡΠΈΡΠ΅ΡΠΊΠΎΠ³ΠΎ
ΠΏΠΎΠ»Ρ, Π²ΠΎΠ·Π±ΡΠΆΠ΄Π°Π΅ΠΌΠΎΠ³ΠΎ ΠΏΠ΅ΡΠ΅ΠΌΠ΅Π½Π½ΡΠΌ Π²ΠΎ
Π²ΡΠ΅ΠΌΠ΅Π½ΠΈ ΠΌΠ°Π³Π½ΠΈΡΠ½ΡΠΌ ΠΏΠΎΠ»Π΅ΠΌ, ΠΎΡΠ»ΠΈΡΠ½Π° ΠΎΡ
Π½ΡΠ»Ρ, ΠΎΠ·Π½Π°ΡΠ°Π΅Ρ, ΡΡΠΎ ΡΠ°ΡΡΠΌΠ°ΡΡΠΈΠ²Π°Π΅ΠΌΠΎΠ΅
ΡΠ»Π΅ΠΊΡΡΠΈΡΠ΅ΡΠΊΠΎΠ΅ ΠΏΠΎΠ»Π΅ Π½Π΅
ΠΏΠΎΡΠ΅Π½ΡΠΈΠ°Π»ΡΠ½ΠΎΠ΅.ΠΠ½ΠΎ, ΠΊΠ°ΠΊ ΠΈ ΠΌΠ°Π³Π½ΠΈΡΠ½ΠΎΠ΅
ΠΏΠΎΠ»Π΅, ΡΠ²Π»ΡΠ΅ΡΡΡΠ²ΠΈΡ
ΡΠ΅Π²ΡΠΌ.
Π ΠΎΠ±ΡΠ΅ΠΌ ΡΠ»ΡΡΠ°Π΅ ΡΠ»Π΅ΠΊΡΡΠΈΡΠ΅ΡΠΊΠΎΠ΅ ΠΏΠΎΠ»Π΅ ΠΌΠΎΠΆΠ΅Ρ Π±ΡΡΡ ΠΏΡΠ΅Π΄ΡΡΠ°Π²Π»Π΅Π½ΠΎ Π²Π΅ΠΊΡΠΎΡΠ½ΠΎΠΉ
ΡΡΠΌΠΌΠΎΠΉ ΠΏΠΎΡΠ΅Π½ΡΠΈΠ°Π»ΡΠ½ΠΎΠ³ΠΎ (ΠΏΠΎΠ»Ρ ΡΡΠ°ΡΠΈΡΠ΅ΡΠΊΠΈΡ
ΡΠ»Π΅ΠΊΡΡΠΈΡΠ΅ΡΠΊΠΈΡ
Π·Π°ΡΡΠ΄ΠΎΠ², ΡΠΈΡΠΊΡΠ»ΡΡΠΈΡ
ΠΊΠΎΡΠΎΡΠΎΠ³ΠΎ ΡΠ°Π²Π½Π° Π½ΡΠ»Ρ) ΠΈ Π²ΠΈΡ
ΡΠ΅Π²ΠΎΠ³ΠΎ
(ΠΎΠ±ΡΡΠ»ΠΎΠ²Π»Π΅Π½Π½ΠΎΠ³ΠΎ ΠΈΠ·ΠΌΠ΅Π½ΡΡΡΠΈΠΌΡΡ Π²ΠΎ Π²ΡΠ΅ΠΌΠ΅Π½ΠΈ
ΠΌΠ°Π³Π½ΠΈΡΠ½ΡΠΌ ΠΏΠΎΠ»Π΅ΠΌ) ΡΠ»Π΅ΠΊΡΡΠΈΡΠ΅ΡΠΊΠΈΡ
ΠΏΠΎΠ»Π΅ΠΉ.
Π ΠΎΡΠ½ΠΎΠ²Π΅ ΡΠ°ΡΡΠΌΠΎΡΡΠ΅Π½Π½ΡΡ
Π½Π°ΠΌΠΈ ΡΠ²Π»Π΅Π½ΠΈΠΉ,
ΠΎΠ±ΡΡΡΠ½ΡΡΡΠΈΡ
Π·Π°ΠΊΠΎΠ½ ΡΠ»Π΅ΠΊΡΡΠΎΠΌΠ°Π³Π½ΠΈΡΠ½ΠΎΠΉ
ΠΈΠ½Π΄ΡΠΊΡΠΈΠΈ, Π½Π΅ ΠΏΡΠΎΡΠΌΠ°ΡΡΠΈΠ²Π°Π΅ΡΡΡ ΠΎΠ±ΡΠ΅Π³ΠΎ
ΠΏΡΠΈΠ½ΡΠΈΠΏΠ°, ΠΏΠΎΠ·Π²ΠΎΠ»ΡΡΡΠ΅Π³ΠΎ ΡΡΡΠ°Π½ΠΎΠ²ΠΈΡΡ
ΠΎΠ±ΡΠ½ΠΎΡΡΡ ΠΈΡ
ΡΠΈΠ·ΠΈΡΠ΅ΡΠΊΠΎΠΉ ΠΏΡΠΈΡΠΎΠ΄Ρ.
ΠΠ±ΡΠ°ΡΠΈΡΠ΅ Π²Π½ΠΈΠΌΠ°Π½ΠΈΠ΅
ΠΠΎΡΡΠΎΠΌΡ
ΡΡΠΈ ΡΠ²Π»Π΅Π½ΠΈΡ ΡΠ»Π΅Π΄ΡΠ΅Ρ ΡΠ°ΡΡΠΌΠ°ΡΡΠΈΠ²Π°ΡΡ ΠΊΠ°ΠΊ
Π½Π΅Π·Π°Π²ΠΈΡΠΈΠΌΡΠ΅, Π° Π·Π°ΠΊΠΎΠ½ ΡΠ»Π΅ΠΊΡΡΠΎΠΌΠ°Π³Π½ΠΈΡΠ½ΠΎΠΉ
ΠΈΠ½Π΄ΡΠΊΡΠΈΠΈ β ΠΊΠ°ΠΊ ΡΠ΅Π·ΡΠ»ΡΡΠ°Ρ ΠΈΡ
ΡΠΎΠ²ΠΌΠ΅ΡΡΠ½ΠΎΠ³ΠΎ
Π΄Π΅ΠΉΡΡΠ²ΠΈΡ. Π’Π΅ΠΌ Π±ΠΎΠ»Π΅Π΅ ΡΠ΄ΠΈΠ²ΠΈΡΠ΅Π»ΡΠ½ΡΠΌ
ΠΎΠΊΠ°Π·ΡΠ²Π°Π΅ΡΡΡ ΡΠΎΡ ΡΠ°ΠΊΡ, ΡΡΠΎ Ρ.Π΄.Ρ. ΠΈΠ½Π΄ΡΠΊΡΠΈΠΈ Π² ΠΊΠΎΠ½ΡΡΡΠ΅ Π²ΡΠ΅Π³Π΄Π° ΡΠ°Π²Π½Π° ΡΠΊΠΎΡΠΎΡΡΠΈ ΠΈΠ·ΠΌΠ΅Π½Π΅Π½ΠΈΡ
ΠΌΠ°Π³Π½ΠΈΡΠ½ΠΎΠ³ΠΎ ΠΏΠΎΡΠΎΠΊΠ° ΡΠΊΠ²ΠΎΠ·Ρ ΠΊΠΎΠ½ΡΡΡ.
Π ΡΠ΅Ρ
ΡΠ»ΡΡΠ°ΡΡ
, ΠΊΠΎΠ³Π΄Π° ΠΌΠ΅Π½ΡΠ΅ΡΡΡ ΠΈ ΠΏΠΎΠ»Π΅ ΠΈ ΡΠ°ΡΠΏΠΎΠ»ΠΎΠΆΠ΅Π½ΠΈΠ΅ ΠΈΠ»ΠΈ ΠΊΠΎΠ½ΡΠΈΠ³ΡΡΠ°ΡΠΈΡ ΠΊΠΎΠ½ΡΡΡΠ°
Π² ΠΌΠ°Π³Π½ΠΈΡΠ½ΠΎΠΌ ΠΏΠΎΠ»Π΅, Ρ.Π΄.Ρ. ΠΈΠ½Π΄ΡΠΊΡΠΈΠΈ ΡΠ»Π΅Π΄ΡΠ΅Ρ
ΡΠ°ΡΡΡΠΈΡΡΠ²Π°ΡΡ ΠΏΠΎ ΡΠΎΡΠΌΡΠ»Π΅
, | (26.8) |
Π° Π·Π°ΠΊΠΎΠ½ ΡΠ»Π΅ΠΊΡΡΠΎΠΌΠ°Π³Π½ΠΈΡΠ½ΠΎΠΉ ΠΈΠ½Π΄ΡΠΊΡΠΈΠΈ ΠΌΠΎΠΆΠ½ΠΎ
ΠΏΡΠ΅Π΄ΡΡΠ°Π²ΠΈΡΡ Π² Π²ΠΈΠ΄Π΅
. | (26.9) |
ΠΡΡΠ°ΠΆΠ΅Π½ΠΈΠ΅, ΡΡΠΎΡΡΠ΅Π΅ Π² ΠΏΡΠ°Π²ΠΎΠΉ ΡΠ°ΡΡΠΈ ΡΡΠΎΠ³ΠΎ
ΡΠ°Π²Π΅Π½ΡΡΠ²Π°, ΠΏΡΠ΅Π΄ΡΡΠ°Π²Π»ΡΠ΅Ρ ΡΠΎΠ±ΠΎΠΉ ΠΏΠΎΠ»Π½ΡΡ
ΠΏΡΠΎΠΈΠ·Π²ΠΎΠ΄Π½ΡΡ ΠΌΠ°Π³Π½ΠΈΡΠ½ΠΎΠ³ΠΎ ΠΏΠΎΡΠΎΠΊΠ° ΠΏΠΎ
Π²ΡΠ΅ΠΌΠ΅Π½ΠΈ: ΠΏΠ΅ΡΠ²ΠΎΠ΅ ΡΠ»Π°Π³Π°Π΅ΠΌΠΎΠ΅ ΡΠ²ΡΠ·Π°Π½ΠΎ Ρ
ΠΈΠ·ΠΌΠ΅Π½Π΅Π½ΠΈΠ΅ΠΌ ΠΌΠ°Π³Π½ΠΈΡΠ½ΠΎΠ³ΠΎ ΠΏΠΎΠ»Ρ Π²ΠΎ Π²ΡΠ΅ΠΌΠ΅Π½ΠΈ,
Π²ΡΠΎΡΠΎΠ΅ β Ρ Π΄Π²ΠΈΠΆΠ΅Π½ΠΈΠ΅ΠΌ ΠΊΠΎΠ½ΡΡΡΠ°.
ΠΠΎΠΆΠ½ΠΎ ΡΠΊΠ°Π·Π°ΡΡ, ΡΡΠΎ Π²ΠΎ Π²ΡΠ΅Ρ
ΡΠ»ΡΡΠ°ΡΡ
ΠΈΠ½Π΄ΡΠΊΡΠΈΠΎΠ½Π½ΡΠΉ ΡΠΎΠΊ Π²ΡΠ·ΡΠ²Π°Π΅ΡΡΡ ΠΏΠΎΠ»Π½ΠΎΠΉ
ΡΠΈΠ»ΠΎΠΉ ΠΠΎΡΠ΅Π½ΡΠ°
ΠΠ°ΠΊΠ°Ρ ΡΠ°ΡΡΡ ΠΈΠ½Π΄ΡΠΊΡΠΈΠΎΠ½Π½ΠΎΠ³ΠΎ ΡΠΎΠΊΠ° Π²ΡΠ·ΡΠ²Π°Π΅ΡΡΡ
ΡΠ»Π΅ΠΊΡΡΠΈΡΠ΅ΡΠΊΠΎΠΉ, Π° ΠΊΠ°ΠΊΠ°Ρ ΠΌΠ°Π³Π½ΠΈΡΠ½ΠΎΠΉ
ΡΠΎΡΡΠ°Π²Π»ΡΡΡΠ΅ΠΉ ΡΠΈΠ»Ρ ΠΠΎΡΠ΅Π½ΡΠ° β Π·Π°Π²ΠΈΡΠΈΡ ΠΎΡ
Π²ΡΠ±ΠΎΡΠ° ΡΠΈΡΡΠ΅ΠΌΡ ΠΎΡΡΡΠ΅ΡΠ°.
Π ΡΠ°Π±ΠΎΡΠ΅ ΡΠΈΠ» ΠΠΎΡΠ΅Π½ΡΠ° ΠΈ ΠΠΌΠΏΠ΅ΡΠ°.
ΠΠ· ΡΠ°ΠΌΠΎΠ³ΠΎ ΠΎΠΏΡΠ΅Π΄Π΅Π»Π΅Π½ΠΈΡ ΡΠ°Π±ΠΎΡΡ ΡΠ»Π΅Π΄ΡΠ΅Ρ,
ΡΡΠΎ ΡΠΈΠ»Π°, Π΄Π΅ΠΉΡΡΠ²ΡΡΡΠ°Ρ Π² ΠΌΠ°Π³Π½ΠΈΡΠ½ΠΎΠΌ ΠΏΠΎΠ»Π΅
Π½Π° ΡΠ»Π΅ΠΊΡΡΠΈΡΠ΅ΡΠΊΠΈΠΉ Π·Π°ΡΡΠ΄ ΠΈ ΠΏΠ΅ΡΠΏΠ΅Π½Π΄ΠΈΠΊΡΠ»ΡΡΠ½Π°Ρ
Π΅Π³ΠΎ ΡΠΊΠΎΡΠΎΡΡΠΈ, Π½Π΅ ΠΌΠΎΠΆΠ΅Ρ ΡΠΎΠ²Π΅ΡΡΠ°ΡΡ ΡΠ°Π±ΠΎΡΡ.
ΠΠ΄Π½Π°ΠΊΠΎ ΠΏΡΠΈ Π΄Π²ΠΈΠΆΠ΅Π½ΠΈΠΈ ΠΏΡΠΎΠ²ΠΎΠ΄Π½ΠΈΠΊΠ° Ρ ΡΠΎΠΊΠΎΠΌ,
ΡΠ²Π»Π΅ΠΊΠ°ΡΡΠ΅Π³ΠΎ Π·Π° ΡΠΎΠ±ΠΎΠΉ Π·Π°ΡΡΠ΄Ρ, ΡΠΈΠ»Π° ΠΠΌΠΏΠ΅ΡΠ°
Π²ΡΠ΅ ΠΆΠ΅ ΡΠ°Π±ΠΎΡΡ ΡΠΎΠ²Π΅ΡΡΠ°Π΅Ρ. ΠΠ°Π³Π»ΡΠ΄Π½ΡΠΌ
ΠΏΠΎΠ΄ΡΠ²Π΅ΡΠΆΠ΄Π΅Π½ΠΈΠ΅ΠΌ ΡΡΠΎΠ³ΠΎ ΡΠ»ΡΠΆΠ°Ρ ΡΠ»Π΅ΠΊΡΡΠΎΠΌΠΎΡΠΎΡΡ.
ΠΡΠΎ ΠΏΡΠΎΡΠΈΠ²ΠΎΡΠ΅ΡΠΈΠ΅ ΠΈΡΡΠ΅Π·Π°Π΅Ρ, Π΅ΡΠ»ΠΈ ΠΏΡΠΈΠ½ΡΡΡ
Π²ΠΎ Π²Π½ΠΈΠΌΠ°Π½ΠΈΠ΅, ΡΡΠΎ Π΄Π²ΠΈΠΆΠ΅Π½ΠΈΠ΅ ΠΏΡΠΎΠ²ΠΎΠ΄Π½ΠΈΠΊΠ° Π²
ΠΌΠ°Π³Π½ΠΈΡΠ½ΠΎΠΌ ΠΏΠΎΠ»Π΅ Π½Π΅ΠΈΠ·Π±Π΅ΠΆΠ½ΠΎ ΡΠΎΠΏΡΠΎΠ²ΠΎΠΆΠ΄Π°Π΅ΡΡΡ
ΡΠ²Π»Π΅Π½ΠΈΠ΅ΠΌ ΡΠ»Π΅ΠΊΡΡΠΎΠΌΠ°Π³Π½ΠΈΡΠ½ΠΎΠΉ ΠΈΠ½Π΄ΡΠΊΡΠΈΠΈ.
ΠΠΎΡΡΠΎΠΌΡ Π½Π°ΡΡΠ΄Ρ Ρ ΡΠΈΠ»ΠΎΠΉ ΠΠΌΠΏΠ΅ΡΠ° ΡΠ°Π±ΠΎΡΡ
Π½Π°Π΄ ΡΠ»Π΅ΠΊΡΡΠΈΡΠ΅ΡΠΊΠΈΠΌΠΈ Π·Π°ΡΡΠ΄Π°ΠΌΠΈ ΡΠΎΠ²Π΅ΡΡΠ°Π΅Ρ
ΠΈ Π²ΠΎΠ·Π½ΠΈΠΊΠ°ΡΡΠ°Ρ Π² ΠΏΡΠΎΠ²ΠΎΠ΄Π½ΠΈΠΊΠ΅ ΡΠ»Π΅ΠΊΡΡΠΎΠ΄Π²ΠΈΠΆΡΡΠ°Ρ
ΡΠΈΠ»Π° ΠΈΠ½Π΄ΡΠΊΡΠΈΠΈ. Π’.ΠΎ.
, ΠΏΠΎΠ»Π½Π°Ρ ΡΠ°Π±ΠΎΡΠ° ΡΠΈΠ»
ΠΌΠ°Π³Π½ΠΈΡΠ½ΠΎΠ³ΠΎ ΠΏΠΎΠ»Ρ ΡΠΊΠ»Π°Π΄ΡΠ²Π°Π΅ΡΡΡ ΠΈΠ·
ΠΌΠ΅Ρ
Π°Π½ΠΈΡΠ΅ΡΠΊΠΎΠΉ ΡΠ°Π±ΠΎΡΡ, ΠΎΠ±ΡΡΠ»ΠΎΠ²Π»Π΅Π½Π½ΠΎΠΉ
ΡΠΈΠ»ΠΎΠΉ ΠΠΌΠΏΠ΅ΡΠ°, ΠΈ ΡΠ°Π±ΠΎΡΡ Ρ. Π΄.Ρ., ΠΈΠ½Π΄ΡΡΠΈΡΡΠ΅ΠΌΠΎΠΉ
ΠΏΡΠΈ Π΄Π²ΠΈΠΆΠ΅Π½ΠΈΠΈ ΠΏΡΠΎΠ²ΠΎΠ΄Π½ΠΈΠΊΠ°. ΠΠ±Π΅ ΡΠ°Π±ΠΎΡΡ
ΡΠ°Π²Π½Ρ ΠΏΠΎ ΠΌΠΎΠ΄ΡΠ»Ρ ΠΈ ΠΏΡΠΎΡΠΈΠ²ΠΎΠΏΠΎΠ»ΠΎΠΆΠ½Ρ ΠΏΠΎ
Π·Π½Π°ΠΊΡ, ΠΏΠΎΡΡΠΎΠΌΡ ΠΈΡ
ΡΡΠΌΠΌΠ° ΡΠ°Π²Π½Π° Π½ΡΠ»Ρ.
ΠΠ΅ΠΉΡΡΠ²ΠΈΡΠ΅Π»ΡΠ½ΠΎ, ΡΠ°Π±ΠΎΡΠ° Π°ΠΌΠΏΠ΅ΡΠΎΠ²ΠΎΠΉ ΡΠΈΠ»Ρ
ΠΏΡΠΈ ΡΠ»Π΅ΠΌΠ΅Π½ΡΠ°ΡΠ½ΠΎΠΌ ΠΏΠ΅ΡΠ΅ΠΌΠ΅ΡΠ΅Π½ΠΈΠΈ ΠΏΡΠΎΠ²ΠΎΠ΄Π½ΠΈΠΊΠ°
Ρ ΡΠΎΠΊΠΎΠΌ Π² ΠΌΠ°Π³Π½ΠΈΡΠ½ΠΎΠΌ ΠΏΠΎΠ»Π΅ ΡΠ°Π²Π½Π° ,
Π·Π° ΡΡΠΎ ΠΆΠ΅ Π²ΡΠ΅ΠΌΡ Ρ.Π΄.Ρ. ΠΈΠ½Π΄ΡΠΊΡΠΈΠΈ ΡΠΎΠ²Π΅ΡΡΠ°Π΅Ρ
ΡΠ°Π±ΠΎΡΡ
, | (26.10) |
ΡΠΎΠ³Π΄Π° ΠΏΠΎΠ»Π½Π°Ρ ΡΠ°Π±ΠΎΡΠ° .
Π‘ΠΈΠ»Ρ ΠΠΌΠΏΠ΅ΡΠ° ΡΠΎΠ²Π΅ΡΡΠ°ΡΡ ΡΠ°Π±ΠΎΡΡ Π½Π΅ Π·Π° ΡΡΠ΅Ρ
ΡΠ½Π΅ΡΠ³ΠΈΠΈ Π²Π½Π΅ΡΠ½Π΅Π³ΠΎ ΠΌΠ°Π³Π½ΠΈΡΠ½ΠΎΠ³ΠΎ ΠΏΠΎΠ»Ρ,
ΠΊΠΎΡΠΎΡΠΎΠ΅ ΠΌΠΎΠΆΠ΅Ρ ΠΎΡΡΠ°Π²Π°ΡΡΡΡ ΠΏΠΎΡΡΠΎΡΠ½Π½ΡΠΌ,
Π° Π·Π° ΡΡΠ΅Ρ ΠΈΡΡΠΎΡΠ½ΠΈΠΊΠ° Ρ.Π΄.Ρ., ΠΏΠΎΠ΄Π΄Π΅ΡΠΆΠΈΠ²Π°ΡΡΠ΅Π³ΠΎ
ΡΠΎΠΊ Π² ΠΊΠΎΠ½ΡΡΡΠ΅.
ΠΡΡΠΎΡΠ½ΠΈΠΊ: https://StudFiles.net/preview/5735864/
Π―Π²Π»Π΅Π½ΠΈΠ΅ ΡΠ»Π΅ΠΊΡΡΠΎΠΌΠ°Π³Π½ΠΈΡΠ½ΠΎΠΉ ΠΈΠ½Π΄ΡΠΊΡΠΈΠΈ. ΠΠ°Π³Π½ΠΈΡΠ½ΡΠΉ ΠΏΠΎΡΠΎΠΊ. ΠΠ°ΠΊΠΎΠ½ ΡΠ»Π΅ΠΊΡΡΠΎΠΌΠ°Π³Π½ΠΈΡΠ½ΠΎΠΉ ΠΈΠ½Π΄ΡΠΊΡΠΈΠΈ. ΠΡΠ°Π²ΠΈΠ»ΠΎ ΠΠ΅Π½ΡΠ°
- Π―Π²Π»Π΅Π½ΠΈΠ΅ ΡΠ»Π΅ΠΊΡΡΠΎΠΌΠ°Π³Π½ΠΈΡΠ½ΠΎΠΉ ΠΈΠ½Π΄ΡΠΊΡΠΈΠΈ
ΠΠ»Π΅ΠΊΡΡΠΈΡΠ΅ΡΠΊΠΈΠ΅ ΠΈ ΠΌΠ°Π³Π½ΠΈΡΠ½ΡΠ΅ ΠΏΠΎΠ»Ρ ΠΏΠΎΡΠΎΠΆΠ΄Π°ΡΡΡΡ ΠΎΠ΄Π½ΠΈΠΌΠΈ ΠΈ ΡΠ΅ΠΌΠΈ ΠΆΠ΅ ΠΈΡΡΠΎΡΠ½ΠΈΠΊΠ°ΠΌΠΈ β ΡΠ»Π΅ΠΊΡΡΠΈΡΠ΅ΡΠΊΠΈΠΌΠΈ Π·Π°ΡΡΠ΄Π°ΠΌΠΈ, ΠΏΠΎΡΡΠΎΠΌΡ ΠΌΠΎΠΆΠ½ΠΎ ΠΏΡΠ΅Π΄ΠΏΠΎΠ»ΠΎΠΆΠΈΡΡ, ΡΡΠΎ ΠΌΠ΅ΠΆΠ΄Ρ ΡΡΠΈΠΌΠΈ ΠΏΠΎΠ»ΡΠΌΠΈ ΡΡΡΠ΅ΡΡΠ²ΡΠ΅Ρ ΠΎΠΏΡΠ΅Π΄Π΅Π»Π΅Π½Π½Π°Ρ ΡΠ²ΡΠ·Ρ. ΠΡΠΎ ΠΏΡΠ΅Π΄ΠΏΠΎΠ»ΠΎΠΆΠ΅Π½ΠΈΠ΅ Π½Π°ΡΠ»ΠΎ ΡΠΊΡΠΏΠ΅ΡΠΈΠΌΠ΅Π½ΡΠ°Π»ΡΠ½ΠΎΠ΅ ΠΏΠΎΠ΄ΡΠ²Π΅ΡΠΆΠ΄Π΅Π½ΠΈΠ΅ Π² 1831 Π³. Π² ΠΎΠΏΡΡΠ°Ρ Π²ΡΠ΄Π°ΡΡΠ΅Π³ΠΎΡΡ Π°Π½Π³Π»ΠΈΠΉΡΠΊΠΎΠ³ΠΎ ΡΠΈΠ·ΠΈΠΊΠ° Π.Π€Π°ΡΠ°Π΄Π΅Ρ. ΠΠ½ ΠΎΡΠΊΡΡΠ» ΡΠ²Π»Π΅Π½ΠΈΠ΅ ΡΠ»Π΅ΠΊΡΡΠΎΠΌΠ°Π³Π½ΠΈΡΠ½ΠΎΠΉ ΠΈΠ½Π΄ΡΠΊΡΠΈΠΈ.
Π―Π²Π»Π΅Π½ΠΈΠ΅ ΡΠ»Π΅ΠΊΡΡΠΎΠΌΠ°Π³Π½ΠΈΡΠ½ΠΎΠΉ ΠΈΠ½Π΄ΡΠΊΡΠΈΠΈ Π»Π΅ΠΆΠΈΡ Π² ΠΎΡΠ½ΠΎΠ²Π΅ ΡΠ°Π±ΠΎΡΡ ΠΈΠ½Π΄ΡΠΊΡΠΈΠΎΠ½Π½ΡΡ Π³Π΅Π½Π΅ΡΠ°ΡΠΎΡΠΎΠ² ΡΠ»Π΅ΠΊΡΡΠΈΡΠ΅ΡΠΊΠΎΠ³ΠΎ ΡΠΎΠΊΠ°, Π½Π° ΠΊΠΎΡΠΎΡΡΠ΅ ΠΏΡΠΈΡ ΠΎΠ΄ΠΈΡΡΡ Π²ΡΡ Π²ΡΡΠ°Π±Π°ΡΡΠ²Π°Π΅ΠΌΠ°Ρ Π² ΠΌΠΈΡΠ΅ ΡΠ»Π΅ΠΊΡΡΠΎΡΠ½Π΅ΡΠ³ΠΈΡ.
ΠΠ°ΠΌΠΊΠ½ΡΡΡΠΉ ΠΊΠΎΠ½ΡΡΡ, ΠΏΠΎΠΌΠ΅ΡΠ΅Π½Π½ΡΠΉ Π² ΠΎΠ΄Π½ΠΎΡΠΎΠ΄Π½ΠΎΠ΅ ΠΌΠ°Π³Π½ΠΈΡΠ½ΠΎΠ΅ ΠΏΠΎΠ»Π΅
ΠΠΎΠ»ΠΈΡΠ΅ΡΡΠ²Π΅Π½Π½ΠΎΠΉ Ρ Π°ΡΠ°ΠΊΡΠ΅ΡΠΈΡΡΠΈΠΊΠΎΠΉ ΠΏΡΠΎΡΠ΅ΡΡΠ° ΠΈΠ·ΠΌΠ΅Π½Π΅Π½ΠΈΡ ΠΌΠ°Π³Π½ΠΈΡΠ½ΠΎΠ³ΠΎ ΠΏΠΎΠ»Ρ ΡΠ΅ΡΠ΅Π· Π·Π°ΠΌΠΊΠ½ΡΡΡΠΉ ΠΊΠΎΠ½ΡΡΡ ΡΠ²Π»ΡΠ΅ΡΡΡ ΡΠΈΠ·ΠΈΡΠ΅ΡΠΊΠ°Ρ Π²Π΅Π»ΠΈΡΠΈΠ½Π° Π½Π°Π·ΡΠ²Π°Π΅ΠΌΠ°Ρ ΠΌΠ°Π³Π½ΠΈΡΠ½ΡΠΌ ΠΏΠΎΡΠΎΠΊΠΎΠΌ.
ΠΠ°Π³Π½ΠΈΡΠ½ΡΠΌ ΠΏΠΎΡΠΎΠΊΠΎΠΌ (Π€) ΡΠ΅ΡΠ΅Π· Π·Π°ΠΌΠΊΠ½ΡΡΡΠΉ ΠΊΠΎΠ½ΡΡΡ ΠΏΠ»ΠΎΡΠ°Π΄ΡΡ (S) Π½Π°Π·ΡΠ²Π°ΡΡ ΡΠΈΠ·ΠΈΡΠ΅ΡΠΊΡΡ Π²Π΅Π»ΠΈΡΠΈΠ½Ρ, ΡΠ°Π²Π½ΡΡ ΠΏΡΠΎΠΈΠ·Π²Π΅Π΄Π΅Π½ΠΈΡ ΠΌΠΎΠ΄ΡΠ»Ρ Π²Π΅ΠΊΡΠΎΡΠ° ΠΌΠ°Π³Π½ΠΈΡΠ½ΠΎΠΉ ΠΈΠ½Π΄ΡΠΊΡΠΈΠΈ (Π) Π½Π° ΠΏΠ»ΠΎΡΠ°Π΄Ρ ΠΊΠΎΠ½ΡΡΡΠ° (S) ΠΈ Π½Π° ΠΊΠΎΡΠΈΠ½ΡΡ ΡΠ³Π»Π° Β ΠΌΠ΅ΠΆΠ΄Ρ Π²Π΅ΠΊΡΠΎΡΠΎΠΌ Π ΠΈ Π½ΠΎΡΠΌΠ°Π»ΡΡ ΠΊ ΠΏΠΎΠ²Π΅ΡΡ Π½ΠΎΡΡΠΈ:Β Ξ¦ = BS cos Ξ±. Β ΠΠ΄ΠΈΠ½ΠΈΡΠ° ΠΌΠ°Π³Π½ΠΈΡΠ½ΠΎΠ³ΠΎ ΠΏΠΎΡΠΎΠΊΠ° Π€ β Π²Π΅Π±Π΅Ρ (ΠΠ±): 1Β ΠΠ±Β =Β 1Β Π’Π»Β Β·Β 1Β ΠΌ2.
ΠΠ°ΠΆΠ½ΠΎ
ΠΡΠ»ΠΈ Π²Π΅ΠΊΡΠΎΡ ΠΌΠ°Π³Π½ΠΈΡΠ½ΠΎΠΉ ΠΈΠ½Π΄ΡΠΊΡΠΈΠΈΒ ΠΏΠ΅ΡΠΏΠ΅Π½Π΄ΠΈΠΊΡΠ»ΡΡΠ΅Π½Β ΠΏΠ»ΠΎΡΠ°Π΄ΠΈ ΠΊΠΎΠ½ΡΡΡΠ°, ΡΠΎ ΠΌΠ°Π³Π½ΠΈΡΠ½ΡΠΉ ΠΏΠΎΡΠΎΠΊ ΠΌΠ°ΠΊΡΠΈΠΌΠ°Π»ΡΠ½ΡΠΉ.
ΠΡΠ»ΠΈ Π²Π΅ΠΊΡΠΎΡ ΠΌΠ°Π³Π½ΠΈΡΠ½ΠΎΠΉ ΠΈΠ½Π΄ΡΠΊΡΠΈΠΈΒ ΠΏΠ°ΡΠ°Π»Π»Π΅Π»Π΅Π½Β ΠΏΠ»ΠΎΡΠ°Π΄ΠΈ ΠΊΠΎΠ½ΡΡΡΠ°, ΡΠΎ ΠΌΠ°Π³Π½ΠΈΡΠ½ΡΠΉ ΠΏΠΎΡΠΎΠΊΒ ΡΠ°Π²Π΅Π½ Π½ΡΠ»Ρ.
- ΠΠ°ΠΊΠΎΠ½ ΡΠ»Π΅ΠΊΡΡΠΎΠΌΠ°Π³Π½ΠΈΡΠ½ΠΎΠΉ ΠΈΠ½Π΄ΡΠΊΡΠΈΠΈ
ΠΠΏΡΡΠ½ΡΠΌ ΠΏΡΡΠ΅ΠΌ Π±ΡΠ» ΡΡΡΠ°Π½ΠΎΠ²Π»Π΅Π½ Π·Π°ΠΊΠΎΠ½ ΡΠ»Π΅ΠΊΡΡΠΎΠΌΠ°Π³Π½ΠΈΡΠ½ΠΎΠΉ ΠΈΠ½Π΄ΡΠΊΡΠΈΠΈ: ΠΠΠ‘ ΠΈΠ½Π΄ΡΠΊΡΠΈΠΈ Π² Π·Π°ΠΌΠΊΠ½ΡΡΠΎΠΌ ΠΊΠΎΠ½ΡΡΡΠ΅ ΡΠ°Π²Π½Π° ΠΏΠΎ ΠΌΠΎΠ΄ΡΠ»Ρ ΡΠΊΠΎΡΠΎΡΡΠΈ ΠΈΠ·ΠΌΠ΅Π½Π΅Π½ΠΈΡ ΠΌΠ°Π³Π½ΠΈΡΠ½ΠΎΠ³ΠΎ ΠΏΠΎΡΠΎΠΊΠ° ΡΠ΅ΡΠ΅Π· ΠΏΠΎΠ²Π΅ΡΡ Π½ΠΎΡΡΡ, ΠΎΠ³ΡΠ°Π½ΠΈΡΠ΅Π½Π½ΡΡ ΠΊΠΎΠ½ΡΡΡΠΎΠΌ: ΠΡΠ° ΡΠΎΡΠΌΡΠ»Π° Π½ΠΎΡΠΈΡ Π½Π°Π·Π²Π°Π½ΠΈΠ΅Β Π·Π°ΠΊΠΎΠ½Π° Π€Π°ΡΠ°Π΄Π΅Ρ.
ΠΠ»Π°ΡΡΠΈΡΠ΅ΡΠΊΠΎΠΉ Π΄Π΅ΠΌΠΎΠ½ΡΡΡΠ°ΡΠΈΠ΅ΠΉ ΠΎΡΠ½ΠΎΠ²Π½ΠΎΠ³ΠΎ Π·Π°ΠΊΠΎΠ½Π° ΡΠ»Π΅ΠΊΡΡΠΎΠΌΠ°Π³Π½ΠΈΡΠ½ΠΎΠΉ ΠΈΠ½Π΄ΡΠΊΡΠΈΠΈ ΡΠ²Π»ΡΠ΅ΡΡΡ ΠΏΠ΅ΡΠ²ΡΠΉ ΠΎΠΏΡΡ Π€Π°ΡΠ°Π΄Π΅Ρ. Π Π½Π΅ΠΌ, ΡΠ΅ΠΌ Π±ΡΡΡΡΠ΅Π΅ ΠΏΠ΅ΡΠ΅ΠΌΠ΅ΡΠ°ΡΡ ΠΌΠ°Π³Π½ΠΈΡ ΡΠ΅ΡΠ΅Π· Π²ΠΈΡΠΊΠΈ ΠΊΠ°ΡΡΡΠΊΠΈ, ΡΠ΅ΠΌ Π±ΠΎΠ»ΡΡΠ΅ Π²ΠΎΠ·Π½ΠΈΠΊΠ°Π΅Ρ ΠΈΠ½Π΄ΡΠΊΡΠΈΠΎΠ½Π½ΡΠΉ ΡΠΎΠΊ Π² Π½Π΅ΠΉ, Π° Π·Π½Π°ΡΠΈΡ, ΠΈ ΠΠΠ‘ ΠΈΠ½Π΄ΡΠΊΡΠΈΠΈ.
ΠΠ°Π²ΠΈΡΠΈΠΌΠΎΡΡΡ Π½Π°ΠΏΡΠ°Π²Π»Π΅Π½ΠΈΡ ΠΈΠ½Π΄ΡΠΊΡΠΈΠΎΠ½Π½ΠΎΠ³ΠΎ ΡΠΎΠΊΠ° ΠΎΡ Ρ Π°ΡΠ°ΠΊΡΠ΅ΡΠ° ΠΈΠ·ΠΌΠ΅Π½Π΅Π½ΠΈΡ ΠΌΠ°Π³Π½ΠΈΡΠ½ΠΎΠ³ΠΎ ΠΏΠΎΠ»Ρ ΡΠ΅ΡΠ΅Π· Π·Π°ΠΌΠΊΠ½ΡΡΡΠΉ ΠΊΠΎΠ½ΡΡΡ Π² 1833 Π³. ΠΎΠΏΡΡΠ½ΡΠΌ ΠΏΡΡΠ΅ΠΌ ΡΡΡΠ°Π½ΠΎΠ²ΠΈΠ» ΡΡΡΡΠΊΠΈΠΉ ΡΠΈΠ·ΠΈΠΊ Π.Π₯.ΠΠ΅Π½Ρ.
Π‘ΠΎΠ³Π»Π°ΡΠ½ΠΎ ΠΏΡΠ°Π²ΠΈΠ»Ρ ΠΠ΅Π½ΡΠ°, Π²ΠΎΠ·Π½ΠΈΠΊΠ°ΡΡΠΈΠΉ Π² Π·Π°ΠΌΠΊΠ½ΡΡΠΎΠΌ ΠΊΠΎΠ½ΡΡΡΠ΅ ΠΈΠ½Π΄ΡΠΊΡΠΈΠΎΠ½Π½ΡΠΉ ΡΠΎΠΊ ΡΠ²ΠΎΠΈΠΌΒ ΠΌΠ°Π³Π½ΠΈΡΠ½ΡΠΌ Β ΠΏΠΎΠ»Π΅ΠΌ ΠΏΡΠΎΡΠΈΠ²ΠΎΠ΄Π΅ΠΉΡΡΠ²ΡΠ΅Ρ ΡΠΎΠΌΡ ΠΈΠ·ΠΌΠ΅Π½Π΅Π½ΠΈΡ Β ΠΌΠ°Π³Π½ΠΈΡΠ½ΠΎΠ³ΠΎ ΠΏΠΎΡΠΎΠΊΠ°, ΠΊΠΎΡΠΎΡΡΠΌ ΠΎΠ½Β Π²ΡΠ·Π²Π°Π½.
Β ΠΠΎΠ»Π΅Π΅ ΠΊΡΠ°ΡΠΊΠΎ ΡΡΠΎ ΠΏΡΠ°Π²ΠΈΠ»ΠΎ ΠΌΠΎΠΆΠ½ΠΎ ΡΡΠΎΡΠΌΡΠ»ΠΈΡΠΎΠ²Π°ΡΡ ΡΠ»Π΅Π΄ΡΡΡΠΈΠΌ ΠΎΠ±ΡΠ°Π·ΠΎΠΌ: ΠΈΠ½Π΄ΡΠΊΡΠΈΠΎΠ½Π½ΡΠΉ ΡΠΎΠΊ Π½Π°ΠΏΡΠ°Π²Π»Π΅Π½ ΡΠ°ΠΊ, ΡΡΠΎΠ±ΡΒ ΠΏΡΠ΅ΠΏΡΡΡΡΠ²ΠΎΠ²Π°ΡΡΒ ΠΏΡΠΈΡΠΈΠ½Π΅, Π΅Π³ΠΎ Π²ΡΠ·ΡΠ²Π°ΡΡΠ΅ΠΉ. ΠΡΠ°Π²ΠΈΠ»ΠΎ ΠΠ΅Π½ΡΠ° ΠΎΡΡΠ°ΠΆΠ°Π΅Ρ ΡΠΎΡ ΡΠΊΡΠΏΠ΅ΡΠΈΠΌΠ΅Π½ΡΠ°Π»ΡΠ½ΡΠΉ ΡΠ°ΠΊΡ, ΡΡΠΎ Β Β Β Π²ΡΠ΅Π³Π΄Π° ΠΈΠΌΠ΅ΡΡ ΠΏΡΠΎΡΠΈΠ²ΠΎΠΏΠΎΠ»ΠΎΠΆΠ½ΡΠ΅ Π·Π½Π°ΠΊΠΈ (Π·Π½Π°ΠΊ Β«ΠΌΠΈΠ½ΡΡΒ» Π² ΡΠΎΡΠΌΡΠ»Π΅ Π€Π°ΡΠ°Π΄Π΅Ρ).
ΠΠ΅Π½ΡΠ΅ΠΌ Π±ΡΠ» ΡΠΊΠΎΠ½ΡΡΡΡΠΈΡΠΎΠ²Π°Π½ ΠΏΡΠΈΠ±ΠΎΡ, ΠΏΡΠ΅Π΄ΡΡΠ°Π²Π»ΡΡΡΠΈΠΉΒ ΡΠΎΠ±ΠΎΠΉ Π΄Π²Π° Π°Π»ΡΠΌΠΈΠ½ΠΈΠ΅Π²ΡΡ ΠΊΠΎΠ»ΡΡΠ°, ΡΠΏΠ»ΠΎΡΠ½ΠΎΠ΅ ΠΈ ΡΠ°Π·ΡΠ΅Π·Π°Π½Π½ΠΎΠ΅, ΡΠΊΡΠ΅ΠΏΠ»Π΅Π½Π½ΡΠ΅ Π½Π° Π°Π»ΡΠΌΠΈΠ½ΠΈΠ΅Π²ΠΎΠΉ ΠΏΠ΅ΡΠ΅ΠΊΠ»Π°Π΄ΠΈΠ½Π΅. ΠΠ½ΠΈ ΠΌΠΎΠ³Π»ΠΈ Π²ΡΠ°ΡΠ°ΡΡΡΡ Π²ΠΎΠΊΡΡΠ³ ΠΎΡΠΈ, ΠΊΠ°ΠΊ ΠΊΠΎΡΠΎΠΌΡΡΠ»ΠΎ.
ΠΡΠΈ Π²Π½Π΅ΡΠ΅Π½ΠΈΠΈ ΠΌΠ°Π³Π½ΠΈΡΠ° Π² ΡΠΏΠ»ΠΎΡΠ½ΠΎΠ΅ ΠΊΠΎΠ»ΡΡΠΎ ΠΎΠ½ΠΎ Π½Π°ΡΠΈΠ½Π°Π»ΠΎ Β«ΡΠ±Π΅Π³Π°ΡΡΒ» ΠΎΡ ΠΌΠ°Π³Π½ΠΈΡΠ°, ΠΏΠΎΠ²ΠΎΡΠ°ΡΠΈΠ²Π°Ρ ΡΠΎΠΎΡΠ²Π΅ΡΡΡΠ²Π΅Π½Π½ΠΎ ΠΊΠΎΡΠΎΠΌΡΡΠ»ΠΎ. ΠΡΠΈ Π²ΡΠ½Π΅ΡΠ΅Π½ΠΈΠΈ ΠΌΠ°Π³Π½ΠΈΡΠ° ΠΈΠ· ΠΊΠΎΠ»ΡΡΠ° ΠΎΠ½ΠΎ ΡΡΡΠ΅ΠΌΠΈΠ»ΠΎΡΡ Β«Π΄ΠΎΠ³Π½Π°ΡΡΒ» ΠΌΠ°Π³Π½ΠΈΡ. ΠΡΠΈ Π΄Π²ΠΈΠΆΠ΅Π½ΠΈΠΈ ΠΆΠ΅ ΠΌΠ°Π³Π½ΠΈΡΠ° Π²Π½ΡΡΡΠΈ ΡΠ°Π·ΡΠ΅Π·Π°Π½Π½ΠΎΠ³ΠΎ ΠΊΠΎΠ»ΡΡΠ° Π½ΠΈΠΊΠ°ΠΊΠΎΠ³ΠΎ Π΄Π²ΠΈΠΆΠ΅Π½ΠΈΡ Π½Π΅ ΠΏΡΠΎΠΈΡΡ ΠΎΠ΄ΠΈΠ»ΠΎ.
ΠΠ΅Π½Ρ ΠΎΠ±ΡΡΡΠ½ΡΠ» ΠΎΠΏΡΡ ΡΠ΅ΠΌ, ΡΡΠΎ ΠΌΠ°Π³Π½ΠΈΡΠ½ΠΎΠ΅ ΠΏΠΎΠ»Π΅ ΠΈΠ½Π΄ΡΠΊΡΠΈΠΎΠ½Π½ΠΎΠ³ΠΎ ΡΠΎΠΊΠ° ΡΡΡΠ΅ΠΌΠΈΠ»ΠΎΡΡ ΠΊΠΎΠΌΠΏΠ΅Π½ΡΠΈΡΠΎΠ²Π°ΡΡ ΠΈΠ·ΠΌΠ΅Π½Π΅Π½ΠΈΠ΅ Π²Π½Π΅ΡΠ½Π΅Π³ΠΎ ΠΌΠ°Π³Π½ΠΈΡΠ½ΠΎΠ³ΠΎ ΠΏΠΎΡΠΎΠΊΠ°.
Π‘ΠΎΠ²Π΅Ρ
ΠΡΠ°Π²ΠΈΠ»ΠΎ ΠΠ΅Π½ΡΠ° ΠΈΠΌΠ΅Π΅Ρ Π³Π»ΡΠ±ΠΎΠΊΠΈΠΉ ΡΠΈΠ·ΠΈΡΠ΅ΡΠΊΠΈΠΉ ΡΠΌΡΡΠ» β ΠΎΠ½ΠΎ Π²ΡΡΠ°ΠΆΠ°Π΅Ρ Π·Π°ΠΊΠΎΠ½ ΡΠΎΡ ΡΠ°Π½Π΅Π½ΠΈΡ ΡΠ½Π΅ΡΠ³ΠΈΠΈ.
ΠΡΡΠΎΡΠ½ΠΈΠΊ: http://kaplio. ru/yavlenie-elektromagnitnoj-induktsii-magnitnyj-potok-zakon-elektromagnitnoj-induktsii-pravilo-lentsa/
ΠΠ°ΠΊΠΎΠ½ ΠΠΠ‘ ΠΈΠ½Π΄ΡΠΊΡΠΈΠΈ Π€Π°ΡΠ°Π΄Π΅Ρ Π΄Π»Ρ ΡΡΠ°Π½ΡΡΠΎΡΠΌΠ°ΡΠΎΡΠΎΠ²
ΠΠ»Π΅ΠΊΡΡΠΈΡΠ΅ΡΡΠ²ΠΎ ΠΎΠ±Π»Π°Π΄Π°Π΅Ρ ΡΠΏΠΎΡΠΎΠ±Π½ΠΎΡΡΡΡ Π³Π΅Π½Π΅ΡΠΈΡΠΎΠ²Π°ΡΡ ΠΌΠ°Π³Π½ΠΈΡΠ½ΠΎΠ΅ ΠΏΠΎΠ»Π΅. Π 1831 Π³ΠΎΠ΄Ρ Π. Π€Π°ΡΠ°Π΄Π΅ΠΉ Π²Π²Π΅Π» ΠΏΠΎΠ½ΡΡΠΈΠ΅ ΡΠ»Π΅ΠΊΡΡΠΎΠΌΠ°Π³Π½ΠΈΡΠ½Π°Ρ ΠΈΠ½Π΄ΡΠΊΡΠΈΡ. ΠΠ½ ΡΠΌΠΎΠ³ ΠΏΠΎΠ»ΡΡΠΈΡΡ Π² Π·Π°ΠΊΡΡΡΠΎΠΉ ΡΠΈΡΡΠ΅ΠΌΠ΅ ΠΏΡΠΎΠ²ΠΎΠ΄Π½ΠΈΠΊΠΎΠ² ΡΠ»Π΅ΠΊΡΡΠΈΡΠ΅ΡΡΠ²ΠΎ, ΠΏΠΎΡΠ²Π»ΡΡΡΠ΅Π΅ΡΡ ΠΏΡΠΈ ΠΈΠ·ΠΌΠ΅Π½Π΅Π½ΠΈΠΈ ΠΏΠΎΠΊΠ°Π·Π°ΡΠ΅Π»Π΅ΠΉ ΠΌΠ°Π³Π½ΠΈΡΠ½ΠΎΠ³ΠΎ ΠΏΠΎΡΠΎΠΊΠ°. Π€ΠΎΡΠΌΡΠ»Π° Π·Π°ΠΊΠΎΠ½Π° Π€Π°ΡΠ°Π΄Π΅Ρ Π΄Π°Π»Π° ΡΠΎΠ»ΡΠΎΠΊ Π΄Π»Ρ ΡΠ°Π·Π²ΠΈΡΠΈΡ ΡΠ»Π΅ΠΊΡΡΠΎΠ΄ΠΈΠ½Π°ΠΌΠΈΠΊΠΈ.
ΠΡΡΠΎΡΠΈΡ ΡΠ°Π·Π²ΠΈΡΠΈΡ
ΠΠΎΡΠ»Π΅ Π΄ΠΎΠΊΠ°Π·Π°ΡΠ΅Π»ΡΡΡΠ²Π° Π·Π°ΠΊΠΎΠ½Π° ΡΠ»Π΅ΠΊΡΡΠΎΠΌΠ°Π³Π½ΠΈΡΠ½ΠΎΠΉ ΠΈΠ½Π΄ΡΠΊΡΠΈΠΈ Π°Π½Π³Π»ΠΈΠΉΡΠΊΠΈΠΌ ΡΡΠ΅Π½ΡΠΌ Π. Π€Π°ΡΠ°Π΄Π΅Π΅ΠΌ Π½Π°Π΄ ΠΎΡΠΊΡΡΡΠΈΠ΅ΠΌ ΡΠ°Π±ΠΎΡΠ°Π»ΠΈ ΡΠΎΡΡΠΈΠΉΡΠΊΠΈΠ΅ ΡΡΠ΅Π½ΡΠ΅ Π. ΠΠ΅Π½Ρ ΠΈ Π. Π―ΠΊΠΎΠ±ΠΈ. ΠΠ»Π°Π³ΠΎΠ΄Π°ΡΡ ΠΈΡ ΡΡΡΠ΄Π°ΠΌ, ΡΠ΅Π³ΠΎΠ΄Π½Ρ ΡΠ°Π·ΡΠ°Π±ΠΎΡΠ°Π½Π½ΡΠΉ ΠΏΡΠΈΠ½ΡΠΈΠΏ ΠΏΠΎΠ»ΠΎΠΆΠ΅Π½ Π² ΠΎΡΠ½ΠΎΠ²Ρ ΡΡΠ½ΠΊΡΠΈΠΎΠ½ΠΈΡΠΎΠ²Π°Π½ΠΈΡ ΠΌΠ½ΠΎΠ³ΠΈΡ ΠΏΡΠΈΠ±ΠΎΡΠΎΠ² ΠΈ ΠΌΠ΅Ρ Π°Π½ΠΈΠ·ΠΌΠΎΠ².
ΠΡΠ½ΠΎΠ²Π½ΡΠΌΠΈ Π°Π³ΡΠ΅Π³Π°ΡΠ°ΠΌΠΈ, Π² ΠΊΠΎΡΠΎΡΡΡ ΠΏΡΠΈΠΌΠ΅Π½ΡΠ΅ΡΡΡ Π·Π°ΠΊΠΎΠ½ ΡΠ»Π΅ΠΊΡΡΠΎΠΌΠ°Π³Π½ΠΈΡΠ½ΠΎΠΉ ΠΈΠ½Π΄ΡΠΊΡΠΈΠΈ Π€Π°ΡΠ°Π΄Π΅Ρ, ΡΠ²Π»ΡΡΡΡΡ Π΄Π²ΠΈΠ³Π°ΡΠ΅Π»Ρ, ΡΡΠ°Π½ΡΡΠΎΡΠΌΠ°ΡΠΎΡ ΠΈ ΠΌΠ½ΠΎΠΆΠ΅ΡΡΠ²ΠΎ ΠΈΠ½ΡΡ ΠΏΡΠΈΠ±ΠΎΡΠΎΠ².
ΠΠ½Π΄ΡΠΊΡΠΈΠ΅ΠΉ ΡΠ»Π΅ΠΊΡΡΠΎΠΌΠ°Π³Π½ΠΈΡΠ½ΠΎ ΠΈΠΌΠ΅Π½ΡΠ΅ΡΡΡ ΠΈΠ½Π΄ΡΡΠΈΡΠΎΠ²Π°Π½ΠΈΠ΅ Π² Π·Π°ΠΌΠΊΠ½ΡΡΠΎΠΉ ΠΏΡΠΎΠ²ΠΎΠ΄ΡΡΠ΅ΠΉ ΡΠΈΡΡΠ΅ΠΌΠ΅ ΡΠ»Π΅ΠΊΡΡΠΈΡΠ΅ΡΠΊΠΎΠ³ΠΎ ΡΠΎΠΊΠ°.
Π’Π°ΠΊΠΎΠ΅ ΡΠ²Π»Π΅Π½ΠΈΠ΅ ΡΡΠ°Π½ΠΎΠ²ΠΈΡΡΡ Π²ΠΎΠ·ΠΌΠΎΠΆΠ½ΡΠΌ ΠΏΡΠΈ ΡΠΈΠ·ΠΈΡΠ΅ΡΠΊΠΎΠΌ ΠΏΠ΅ΡΠ΅Π΄Π²ΠΈΠΆΠ΅Π½ΠΈΠΈ ΡΠ΅ΡΠ΅Π· ΠΏΡΠΎΠ²ΠΎΠ΄Π½ΠΈΠΊΠΎΠ²ΡΡ ΡΠΈΡΡΠ΅ΠΌΡ ΠΌΠ°Π³Π½ΠΈΡΠ½ΠΎΠ³ΠΎ ΠΏΠΎΠ»Ρ. ΠΠ΅Ρ Π°Π½ΠΈΡΠ΅ΡΠΊΠΎΠ΅ Π΄Π΅ΠΉΡΡΠ²ΠΈΠ΅ Π²Π»Π΅ΡΠ΅Ρ Π·Π° ΡΠΎΠ±ΠΎΠΉ ΠΏΠΎΡΠ²Π»Π΅Π½ΠΈΠ΅ ΡΠ»Π΅ΠΊΡΡΠΈΡΠ΅ΡΡΠ²Π°. ΠΠ³ΠΎ ΠΏΡΠΈΠ½ΡΡΠΎ Π½Π°Π·ΡΠ²Π°ΡΡ ΠΈΠ½Π΄ΡΠΊΡΠΈΠΎΠ½Π½ΡΠΌ.
ΠΠΎ ΠΎΡΠΊΡΡΡΠΈΡ Π·Π°ΠΊΠΎΠ½Π° Π€Π°ΡΠ°Π΄Π΅Ρ ΡΠ΅Π»ΠΎΠ²Π΅ΡΠ΅ΡΡΠ²ΠΎ Π½Π΅ Π·Π½Π°Π»ΠΎ ΠΎΠ± ΠΈΠ½ΡΡ ΡΠΏΠΎΡΠΎΠ±Π°Ρ ΡΠΎΠ·Π΄Π°Π½ΠΈΡ ΡΠ»Π΅ΠΊΡΡΠΈΡΠ΅ΡΡΠ²Π°, ΠΊΡΠΎΠΌΠ΅ Π³Π°Π»ΡΠ²Π°Π½ΠΈΠΊΠΈ.
ΠΡΠ»ΠΈ ΡΠΊΠ²ΠΎΠ·Ρ ΠΏΡΠΎΠ²ΠΎΠ΄Π½ΠΈΠΊ ΠΏΡΠΎΠΏΡΡΡΠΈΡΡ ΠΌΠ°Π³Π½ΠΈΡΠ½ΠΎΠ΅ ΠΏΠΎΠ»Π΅, Π² Π½Π΅ΠΌ Π±ΡΠ΄Π΅Ρ Π²ΠΎΠ·Π½ΠΈΠΊΠ°ΡΡ ΠΠΠ‘ ΠΈΠ½Π΄ΡΠΊΡΠΈΠΈ. ΠΠ΅ Π΅ΡΠ΅ ΠΈΠΌΠ΅Π½ΡΡΡ ΡΠ»Π΅ΠΊΡΡΠΎΠ΄Π²ΠΈΠΆΡΡΠ΅ΠΉ ΡΠΈΠ»ΠΎΠΉ. ΠΡΠΈ ΠΏΠΎΠΌΠΎΡΠΈ ΡΡΠΎΠ³ΠΎ ΠΎΡΠΊΡΡΡΠΈΡ ΡΠ΄Π°Π΅ΡΡΡ ΠΏΡΠ΅Π΄ΡΡΠ°Π²ΠΈΡΡ Π² ΠΊΠΎΠ»ΠΈΡΠ΅ΡΡΠ²Π΅Π½Π½ΠΎΠΌ Π²ΡΡΠ°ΠΆΠ΅Π½ΠΈΠΈ ΠΏΠΎΠΊΠ°Π·Π°ΡΠ΅Π»Ρ.
ΠΠΏΡΡΠ½ΠΎΠ΅ Π΄ΠΎΠΊΠ°Π·Π°ΡΠ΅Π»ΡΡΡΠ²ΠΎ
ΠΡΠΎΠ²ΠΎΠ΄Ρ ΡΠ²ΠΎΠΈ ΠΈΡΡΠ»Π΅Π΄ΠΎΠ²Π°Π½ΠΈΡ, Π°Π½Π³Π»ΠΈΠΉΡΠΊΠΈΠΉ ΡΡΠ΅Π½ΡΠΉ ΡΡΡΠ°Π½ΠΎΠ²ΠΈΠ», ΡΡΠΎ ΠΈΠ½Π΄ΡΠΊΡΠΈΠΎΠ½Π½ΡΠΉ ΡΠΎΠΊ ΠΏΠΎΠ»ΡΡΠ°Π΅ΡΡΡ ΠΎΠ΄Π½ΠΈΠΌ ΠΈΠ· Π΄Π²ΡΡ ΡΠΏΠΎΡΠΎΠ±ΠΎΠ². Π ΠΏΠ΅ΡΠ²ΠΎΠΌ ΠΎΠΏΡΡΠ΅ ΠΎΠ½ ΠΏΠΎΡΠ²Π»ΡΠ΅ΡΡΡ ΠΏΡΠΈ Π΄Π²ΠΈΠΆΠ΅Π½ΠΈΠΈ ΡΠ°ΠΌΠΊΠΈ Π² ΠΌΠ°Π³Π½ΠΈΡΠ½ΠΎΠΌ ΠΏΠΎΠ»Π΅, ΡΠΎΠ·Π΄Π°Π²Π°Π΅ΠΌΠΎΠΌ Π½Π΅ΠΏΠΎΠ΄Π²ΠΈΠΆΠ½ΠΎΠΉ ΠΊΠ°ΡΡΡΠΊΠΎΠΉ. ΠΡΠΎΡΠΎΠΉ ΡΠΏΠΎΡΠΎΠ± ΠΏΡΠ΅Π΄ΠΏΠΎΠ»Π°Π³Π°Π΅Ρ Π½Π΅ΠΏΠΎΠ΄Π²ΠΈΠΆΠ½ΠΎΠ΅ ΠΏΠΎΠ»ΠΎΠΆΠ΅Π½ΠΈΠ΅ ΡΠ°ΠΌΠΊΠΈ. Π ΡΡΠΎΠΌ ΡΠΊΡΠΏΠ΅ΡΠΈΠΌΠ΅Π½ΡΠ΅ ΠΈΠ·ΠΌΠ΅Π½ΡΠ΅ΡΡΡ ΡΠΎΠ»ΡΠΊΠΎ ΠΏΠΎΠ»Π΅ ΠΊΠ°ΡΡΡΠΊΠΈ ΠΏΡΠΈ Π΅Π΅ Π΄Π²ΠΈΠΆΠ΅Π½ΠΈΠΈ ΠΈΠ»ΠΈ ΠΈΠ·ΠΌΠ΅Π½Π΅Π½ΠΈΠΈ ΡΠΈΠ»Ρ ΡΠΎΠΊΠ° Π² Π½Π΅ΠΉ.
ΠΠΏΡΡΡ Π€Π°ΡΠ°Π΄Π΅Ρ ΠΏΡΠΈΠ²Π΅Π»ΠΈ ΠΈΡΡΠ»Π΅Π΄ΠΎΠ²Π°ΡΠ΅Π»Ρ ΠΊ Π²ΡΠ²ΠΎΠ΄Ρ, ΡΡΠΎ ΠΏΡΠΈ Π³Π΅Π½Π΅ΡΠΈΡΠΎΠ²Π°Π½ΠΈΠΈ ΠΈΠ½Π΄ΡΠΊΡΠΈΠΎΠ½Π½ΠΎΠ³ΠΎ ΡΠΎΠΊΠ° ΠΏΡΠΎΠ²ΠΎΡΠΈΡΡΠ΅ΡΡΡ ΡΠ²Π΅Π»ΠΈΡΠ΅Π½ΠΈΠ΅ΠΌ ΠΈΠ»ΠΈ ΡΠΌΠ΅Π½ΡΡΠ΅Π½ΠΈΠ΅ΠΌ ΠΌΠ°Π³Π½ΠΈΡΠ½ΠΎΠ³ΠΎ ΠΏΠΎΡΠΎΠΊΠ° Π² ΡΠΈΡΡΠ΅ΠΌΠ΅. Π’Π°ΠΊΠΆΠ΅ ΠΎΠΏΡΡΡ Π€Π°ΡΠ°Π΄Π΅Ρ ΠΏΠΎΠ·Π²ΠΎΠ»ΠΈΠ»ΠΈ ΡΡΠ²Π΅ΡΠΆΠ΄Π°ΡΡ, ΡΡΠΎ Π·Π½Π°ΡΠ΅Π½ΠΈΠ΅ ΡΠ»Π΅ΠΊΡΡΠΈΡΠ΅ΡΡΠ²Π°, ΠΏΠΎΠ»ΡΡΠ΅Π½Π½ΠΎΠ³ΠΎ ΠΎΠΏΡΡΠ½ΡΠΌ ΠΏΡΡΠ΅ΠΌ, Π½Π΅ Π·Π°Π²ΠΈΡΠΈΡ ΠΎΡ ΠΌΠ΅ΡΠΎΠ΄ΠΎΠ»ΠΎΠ³ΠΈΠΈ, ΠΊΠΎΡΠΎΡΠΎΠΉ Π±ΡΠ» ΠΈΠ·ΠΌΠ΅Π½Π΅Π½ ΠΏΠΎΡΠΎΠΊ ΠΌΠ°Π³Π½ΠΈΡΠ½ΠΎΠΉ ΠΈΠ½Π΄ΡΠΊΡΠΈΠΈ. ΠΠ° ΠΏΠΎΠΊΠ°Π·Π°ΡΠ΅Π»Ρ Π²Π»ΠΈΡΠ΅Ρ ΡΠΎΠ»ΡΠΊΠΎ ΡΠΊΠΎΡΠΎΡΡΡ ΡΠ°ΠΊΠΎΠ³ΠΎ ΠΈΠ·ΠΌΠ΅Π½Π΅Π½ΠΈΡ.
ΠΠΎΠ»ΠΈΡΠ΅ΡΡΠ²Π΅Π½Π½ΠΎΠ΅ Π²ΡΡΠ°ΠΆΠ΅Π½ΠΈΠ΅
Π£ΡΡΠ°Π½ΠΎΠ²ΠΈΡΡ ΠΊΠΎΠ»ΠΈΡΠ΅ΡΡΠ²Π΅Π½Π½ΠΎΠ΅ Π·Π½Π°ΡΠ΅Π½ΠΈΠ΅ ΡΠ²Π»Π΅Π½ΠΈΡ ΡΠ»Π΅ΠΊΡΡΠΎΠΌΠ°Π³Π½ΠΈΡΠ½ΠΎΠΉ ΠΈΠ½Π΄ΡΠΊΡΠΈΠΈ ΠΏΠΎΠ·Π²ΠΎΠ»ΡΠ΅Ρ Π·Π°ΠΊΠΎΠ½ Π€Π°ΡΠ°Π΄Π΅Ρ. ΠΠ½ Π³Π»Π°ΡΠΈΡ, ΡΡΠΎ ΠΠΠ‘, ΠΎΠΏΡΠ΅Π΄Π΅Π»ΡΡΡΠ°ΡΡΡ Π² ΡΠΈΡΡΠ΅ΠΌΠ΅, ΠΌΠ΅Π½ΡΠ΅Ρ Π·Π½Π°ΡΠ΅Π½ΠΈΠ΅ ΠΏΡΠΎΠΏΠΎΡΡΠΈΠΎΠ½Π°Π»ΡΠ½ΠΎ ΡΠΊΠΎΡΠΎΡΡΠΈ ΠΏΠ΅ΡΠ΅ΠΌΠ΅ΡΠ΅Π½ΠΈΡ ΠΏΠΎΡΠΎΠΊΠ° Π² ΠΏΡΠΎΠ²ΠΎΠ΄Π½ΠΈΠΊΠ΅. Π€ΠΎΡΠΌΡΠ»Π° Π±ΡΠ΄Π΅Ρ ΠΈΠΌΠ΅ΡΡ ΡΠ°ΠΊΠΎΠΉ Π²ΠΈΠ΄:
ΠΡΡΠΈΡΠ°ΡΠ΅Π»ΡΠ½ΡΠΉ Π·Π½Π°ΠΊ ΡΠ²ΠΈΠ΄Π΅ΡΠ΅Π»ΡΡΡΠ²ΡΠ΅Ρ ΠΎ ΡΠΎΠΌ, ΡΡΠΎ ΠΠΠ‘ ΠΏΡΠ΅ΠΏΡΡΡΡΠ²ΡΠ΅Ρ ΠΏΠΎΡΠ²Π»Π΅Π½ΠΈΡ ΠΈΠ·ΠΌΠ΅Π½Π΅Π½ΠΈΠΉ Π²Π½ΡΡΡΠΈ ΠΊΠΎΠ½ΡΡΡΠ°. ΠΠ»Ρ ΡΠ΅ΡΠ΅Π½ΠΈΡ Π½Π΅ΠΊΠΎΡΠΎΡΡΡ Π·Π°Π΄Π°Ρ ΠΎΡΡΠΈΡΠ°ΡΠ΅Π»ΡΠ½ΡΠΉ Π·Π½Π°ΠΊ Π² ΡΠΎΡΠΌΡΠ»Π΅ Π½Π΅ ΡΡΠ°Π²ΡΡ. Π ΡΡΠΎΠΌ ΡΠ»ΡΡΠ°Π΅ ΡΠ΅Π·ΡΠ»ΡΡΠ°Ρ Π·Π°ΠΏΠΈΡΡΠ²Π°ΡΡ Π² Π²ΠΈΠ΄Π΅ ΠΌΠΎΠ΄ΡΠ»Ρ.
ΠΠ±ΡΠ°ΡΠΈΡΠ΅ Π²Π½ΠΈΠΌΠ°Π½ΠΈΠ΅
Π‘ΠΈΡΡΠ΅ΠΌΠ° ΠΌΠΎΠΆΠ΅Ρ Π²ΠΊΠ»ΡΡΠ°ΡΡ Π² ΡΠ΅Π±Ρ Π½Π΅ΡΠΊΠΎΠ»ΡΠΊΠΎ Π²ΠΈΡΠΊΠΎΠ². ΠΠΎΠ»ΠΈΡΠ΅ΡΡΠ²ΠΎ ΠΈΡ ΠΎΠ±ΠΎΠ·Π½Π°ΡΠ°Π΅ΡΡΡ Π»Π°ΡΠΈΠ½ΡΠΊΠΎΠΉ Π±ΡΠΊΠ²ΠΎΠΉ N. ΠΡΠ΅ ΡΠ»Π΅ΠΌΠ΅Π½ΡΡ ΠΊΠΎΠ½ΡΡΡΠ° ΠΏΡΠΎΠ½ΠΈΠ·ΡΠ²Π°ΡΡΡΡ Π΅Π΄ΠΈΠ½ΡΠΌ ΠΌΠ°Π³Π½ΠΈΡΠ½ΡΠΌ ΠΏΠΎΡΠΎΠΊΠΎΠΌ. ΠΠΠ‘ ΠΈΠ½Π΄ΡΠΊΡΠΈΠΈ Π±ΡΠ΄Π΅Ρ ΡΠ°ΡΡΡΠΈΡΡΠ²Π°ΡΡΡΡ ΡΠ°ΠΊ:
ΠΠΎΠ½ΡΡΠ½ΡΠΌ ΠΏΡΠΈΠΌΠ΅ΡΠΎΠΌ Π²ΠΎΡΡΠΎΠ·Π΄Π°Π½ΠΈΡ ΡΠ»Π΅ΠΊΡΡΠΈΡΠ΅ΡΡΠ²Π° Π² ΠΏΡΠΎΠ²ΠΎΠ΄Π½ΠΈΠΊΠ΅ ΡΡΠΈΡΠ°Π΅ΡΡΡ ΠΊΠ°ΡΡΡΠΊΠ°, ΡΠΊΠ²ΠΎΠ·Ρ ΠΊΠΎΡΠΎΡΡΡ ΠΏΠ΅ΡΠ΅ΠΌΠ΅ΡΠ°Π΅ΡΡΡ ΠΏΠΎΡΡΠΎΡΠ½Π½ΡΠΉ ΠΌΠ°Π³Π½ΠΈΡ.
Π Π°Π±ΠΎΡΠ° Π. ΠΠ΅Π½ΡΠ°
ΠΠ°ΠΏΡΠ°Π²Π»Π΅Π½Π½ΠΎΡΡΡ ΠΈΠ½Π΄ΡΠΊΡΠΈΠΎΠ½Π½ΠΎΠ³ΠΎ ΡΠΎΠΊΠ° ΠΏΡΠ΅Π΄ΠΎΡΡΠ°Π²Π»ΡΠ΅Ρ Π²ΠΎΠ·ΠΌΠΎΠΆΠ½ΠΎΡΡΡ ΠΎΠΏΡΠ΅Π΄Π΅Π»ΠΈΡΡ ΠΏΡΠ°Π²ΠΈΠ»ΠΎ ΠΠ΅Π½ΡΠ°. ΠΡΠ°ΡΠΊΠ°Ρ ΡΠΎΡΠΌΡΠ»ΠΈΡΠΎΠ²ΠΊΠ° Π·Π²ΡΡΠΈΡ Π΄ΠΎΡΡΠ°ΡΠΎΡΠ½ΠΎ ΠΏΡΠΎΡΡΠΎ. ΠΠΎΡΠ²Π»ΡΡΡΠΈΠΉΡΡ ΠΏΡΠΈ ΠΈΠ·ΠΌΠ΅Π½Π΅Π½ΠΈΠΈ ΠΏΠΎΠΊΠ°Π·Π°ΡΠ΅Π»Π΅ΠΉ ΠΏΠΎΠ»Ρ ΠΏΡΠΎΠ²ΠΎΠ΄Π½ΠΈΠΊΠΎΠ²ΠΎΠ³ΠΎ ΠΊΠΎΠ½ΡΡΡΠ° ΡΠΎΠΊ, ΠΏΡΠ΅ΠΏΡΡΡΡΠ²ΡΠ΅Ρ Π±Π»Π°Π³ΠΎΠ΄Π°ΡΡ ΡΠ²ΠΎΠ΅ΠΌΡ ΠΌΠ°Π³Π½ΠΈΡΠ½ΠΎΠΌΡ ΠΏΠΎΠ»Ρ ΡΠ°ΠΊΠΎΠΌΡ ΠΈΠ·ΠΌΠ΅Π½Π΅Π½ΠΈΡ.
ΠΡΠ»ΠΈ Π² ΠΊΠ°ΡΡΡΠΊΡ ΠΏΠΎΡΡΠ΅ΠΏΠ΅Π½Π½ΠΎ Π²Π²ΠΎΠ΄ΠΈΡΡ ΠΌΠ°Π³Π½ΠΈΡ, Π² Π½Π΅ΠΉ ΠΏΠΎΠ²ΡΡΠ°Π΅ΡΡΡ ΡΡΠΎΠ²Π΅Π½Ρ ΠΌΠ°Π³Π½ΠΈΡΠ½ΠΎΠ³ΠΎ ΠΏΠΎΡΠΎΠΊΠ°. Π‘ΠΎΠ³Π»Π°ΡΠ½ΠΎ ΠΏΡΠ°Π²ΠΈΠ»Ρ ΠΠ΅Π½ΡΠ°, ΠΌΠ°Π³Π½ΠΈΡΠ½ΠΎΠ΅ ΠΏΠΎΠ»Π΅ Π±ΡΠ΄Π΅Ρ ΠΈΠΌΠ΅ΡΡ Π½Π°ΠΏΡΠ°Π²Π»Π΅Π½ΠΈΠ΅ ΠΏΡΠΎΡΠΈΠ²ΠΎΠΏΠΎΠ»ΠΎΠΆΠ½ΠΎΠ΅ ΡΠ²Π΅Π»ΠΈΡΠ΅Π½ΠΈΡ ΠΏΠΎΠ»Ρ ΠΌΠ°Π³Π½ΠΈΡΠ°.
Π§ΡΠΎΠ±Ρ ΠΏΠΎΠ½ΡΡΡ ΡΡΡ Π½Π°ΠΏΡΠ°Π²Π»Π΅Π½Π½ΠΎΡΡΡ, Π½Π΅ΠΎΠ±Ρ ΠΎΠ΄ΠΈΠΌΠΎ ΡΠΌΠΎΡΡΠ΅ΡΡ Π½Π° ΠΌΠ°Π³Π½ΠΈΡ Ρ ΡΠ΅Π²Π΅ΡΠ½ΠΎΠΉ ΡΡΠΎΡΠΎΠ½Ρ. ΠΡΡΡΠ΄Π° Π±ΡΠ΄Π΅Ρ Π²ΠΊΡΡΡΠΈΠ²Π°ΡΡΡΡ Π±ΡΡΠ°Π²ΡΠΈΠΊ Π½Π°Π²ΡΡΡΠ΅ΡΡ ΡΠ΅Π²Π΅ΡΠ½ΠΎΠΌΡ ΠΏΠΎΠ»ΡΡΡ.
Π’ΠΎΠΊ Π±ΡΠ΄Π΅Ρ ΠΏΠ΅ΡΠ΅ΠΌΠ΅ΡΠ°ΡΡΡΡ Π² ΡΡΠΎΡΠΎΠ½Ρ Π΄Π²ΠΈΠΆΠ΅Π½ΠΈΡ ΡΠ°ΡΠΎΠ²ΠΎΠΉ ΡΡΡΠ΅Π»ΠΊΠΈ.
ΠΡΠ»ΠΈ ΠΌΠ°Π³Π½ΠΈΡ Π²ΡΠ²ΠΎΠ΄ΠΈΡΡΡ ΠΈΠ· ΡΠΈΡΡΠ΅ΠΌΡ, ΠΌΠ°Π³Π½ΠΈΡΠ½ΡΠΉ ΠΏΠΎΡΠΎΠΊ Π² Π½Π΅ΠΉ ΡΠΌΠ΅Π½ΡΡΠΈΡΡΡ. Π§ΡΠΎΠ±Ρ ΡΡΡΠ°Π½ΠΎΠ²ΠΈΡΡ Π½Π°ΠΏΡΠ°Π²Π»Π΅Π½ΠΈΠ΅ ΡΠΎΠΊΠ°, Π²ΡΠΊΡΡΡΠΈΠ²Π°Π΅ΡΡΡ Π±ΡΡΠ°Π²ΡΠΈΠΊ. ΠΡΠ°ΡΠ΅Π½ΠΈΡ Π±ΡΠ΄Π΅Ρ Π½Π°ΠΏΡΠ°Π²Π»Π΅Π½ΠΎ Π² ΠΎΠ±ΡΠ°ΡΠ½ΡΡ ΡΡΠΎΡΠΎΠ½Ρ ΠΏΠ΅ΡΠ΅ΠΌΠ΅ΡΠ΅Π½ΠΈΡ ΠΏΠΎ ΡΠΈΡΠ΅ΡΠ±Π»Π°ΡΡ ΡΠ°ΡΠΎΠ²ΠΎΠΉ ΡΡΡΠ΅Π»ΠΊΠΈ.
Π€ΠΎΡΠΌΡΠ»ΠΈΡΠΎΠ²ΠΊΠΈ ΠΠ΅Π½ΡΠ° ΠΏΡΠΈΠΎΠ±ΡΠ΅ΡΠ°ΡΡ Π±ΠΎΠ»ΡΡΠΎΠ΅ Π·Π½Π°ΡΠ΅Π½ΠΈΠ΅ Π΄Π»Ρ ΡΠΈΡΡΠ΅ΠΌΡ Ρ ΠΊΠΎΠ½ΡΡΡΠΎΠΌ Π·Π°ΠΌΠΊΠ½ΡΡΠΎΠ³ΠΎ ΡΠΈΠΏΠ° ΠΈ ΠΎΡΡΡΡΡΡΠ²ΡΡΡΠΈΠΌ ΡΠΎΠΏΡΠΎΡΠΈΠ²Π»Π΅Π½ΠΈΠ΅ΠΌ. ΠΠ³ΠΎ ΠΏΡΠΈΠ½ΡΡΠΎ ΠΈΠΌΠ΅Π½ΠΎΠ²Π°ΡΡ ΠΈΠ΄Π΅Π°Π»ΡΠ½ΡΠΌ ΠΊΠΎΠ½ΡΡΡΠΎΠΌ. ΠΠΎ ΠΏΡΠ°Π²ΠΈΠ»Ρ ΠΠ΅Π½ΡΠ°, Π² Π½Π΅ΠΌ Π½Π΅Π²ΠΎΠ·ΠΌΠΎΠΆΠ½ΠΎ ΡΠ²Π΅Π»ΠΈΡΠΈΡΡ ΠΈΠ»ΠΈ ΡΠΌΠ΅Π½ΡΡΠΈΡΡ ΠΌΠ°Π³Π½ΠΈΡΠ½ΡΠΉ ΠΏΠΎΡΠΎΠΊ.
ΠΠΎΠ½ΡΡΠΈΠ΅ ΡΠ°ΠΌΠΎΠΈΠ½Π΄ΡΠΊΡΠΈΠΈ
ΠΠ΅Π½Π΅ΡΠ°ΡΠΈΡ ΠΈΠ½Π΄ΡΠΊΡΠΈΠΈ Π² ΠΈΠ΄Π΅Π°Π»ΡΠ½ΠΎΠΉ ΡΠΈΡΡΠ΅ΠΌΠ΅, ΠΊΠΎΡΠΎΡΠΎΠ΅ ΠΈΠΌΠ΅Π΅Ρ ΠΌΠ΅ΡΡΠΎ ΠΏΡΠΈ ΠΏΠ°Π΄Π΅Π½ΠΈΠΈ ΠΈΠ»ΠΈ Π²ΠΎΠ·ΡΠ°ΡΡΠ°Π½ΠΈΠΈ ΡΠ»Π΅ΠΊΡΡΠΈΡΠ΅ΡΡΠ²Π° Π² ΠΏΡΠΎΠ²ΠΎΠ΄Π½ΠΈΠΊΠ΅, ΠΈΠΌΠ΅Π½ΡΠ΅ΡΡΡ ΡΠ°ΠΌΠΎΠΈΠ½Π΄ΡΠΊΡΠΈΠ΅ΠΉ.
ΠΠ°ΠΊΠΎΠ½ Π€Π°ΡΠ°Π΄Π΅Ρ Π΄Π»Ρ ΡΠ°ΠΌΠΎΠΈΠ½Π΄ΡΠΊΡΠΈΠΈ Π²ΡΡΠ°ΠΆΠ°Π΅ΡΡΡ ΡΠ°Π²Π΅Π½ΡΡΠ²ΠΎΠΌ, ΠΊΠΎΠ³Π΄Π° ΠΏΡΠΈ ΠΈΠ·ΠΌΠ΅Π½Π΅Π½ΠΈΠΈ ΡΠ»Π΅ΠΊΡΡΠΈΡΠ΅ΡΡΠ²Π° Π½Π΅ ΠΏΡΠΎΠΈΠ·ΠΎΡΠ»ΠΎ ΠΈΠ½ΡΡ ΠΈΠ·ΠΌΠ΅Π½Π΅Π½ΠΈΠΉ:
Π³Π΄Π΅ Π΅ β ΠΠΠ‘, L β ΠΈΠ½Π΄ΡΠΊΡΠΈΠ²Π½ΠΎΡΡΡ Π·Π°ΠΊΡΡΡΠΎΠΉ ΠΊΠ°ΡΡΡΠΊΠΈ, ΞI/Ξt β ΡΠΊΠΎΡΠΎΡΡΡ, Ρ ΠΊΠΎΡΠΎΡΠΎΠΉ ΠΏΡΠΎΠΈΡΡ ΠΎΠ΄ΡΡ ΠΈΠ·ΠΌΠ΅Π½Π΅Π½ΠΈΡ ΡΠΈΠ»Ρ ΡΠΎΠΊΠ°.
ΠΠ½Π΄ΡΠΊΡΠΈΠ²Π½ΠΎΡΡΡ
ΠΡΠ½ΠΎΡΠ΅Π½ΠΈΠ΅, ΠΊΠΎΡΠΎΡΠΎΠ΅ ΠΏΠΎΠΊΠ°Π·ΡΠ²Π°Π΅Ρ ΠΏΡΠΎΠΏΠΎΡΡΠΈΠΎΠ½Π°Π»ΡΠ½ΠΎΡΡΡ ΠΌΠ΅ΠΆΠ΄Ρ ΡΠ°ΠΊΠΈΠΌΠΈ ΠΊΠ°ΡΠ΅Π³ΠΎΡΠΈΡΠΌΠΈ, ΠΊΠ°ΠΊ ΡΠΈΠ»Π° ΡΠΎΠΊΠ° Π² ΠΏΡΠΎΠ²ΠΎΠ΄ΡΡΠ΅ΠΉ ΡΠΈΡΡΠ΅ΠΌΠ΅ ΠΈ ΠΌΠ°Π³Π½ΠΈΡΠ½ΡΠΌ ΠΏΠΎΡΠΎΠΊΠΎΠΌ ΠΈΠΌΠ΅Π½ΡΠ΅ΡΡΡ ΠΈΠ½Π΄ΡΠΊΡΠΈΠ²Π½ΠΎΡΡΡΡ. ΠΠ° ΠΏΠΎΠΊΠ°Π·Π°ΡΠ΅Π»Ρ ΠΈΠΌΠ΅Π΅Ρ Π²Π»ΠΈΡΠ½ΠΈΠ΅ ΡΠΈΠ·ΠΈΡΠ΅ΡΠΊΠΈΠ΅ Π³Π°Π±Π°ΡΠΈΡΡ ΠΊΠ°ΡΡΡΠΊΠΈ ΠΈ ΠΌΠ°Π³Π½ΠΈΡΠ½ΡΠ΅ Ρ Π°ΡΠ°ΠΊΡΠ΅ΡΠΈΡΡΠΈΠΊΠΈ ΡΡΠ΅Π΄Ρ. ΠΡΠ½ΠΎΡΠ΅Π½ΠΈΠ΅ ΠΎΠΏΠΈΡΡΠ²Π°Π΅ΡΡΡ ΡΠΎΡΠΌΡΠ»ΠΎΠΉ:
ΠΠ²ΠΈΠΆΡΡΠ΅Π΅ΡΡ Π² ΠΊΠΎΠ½ΡΡΡΠ΅ ΡΠ»Π΅ΠΊΡΡΠΈΡΠ΅ΡΡΠ²ΠΎ ΠΏΡΠΎΠ²ΠΎΡΠΈΡΡΠ΅Ρ ΠΏΠΎΡΠ²Π»Π΅Π½ΠΈΠ΅ ΠΌΠ°Π³Π½ΠΈΡΠ½ΠΎΠ³ΠΎ ΠΏΠΎΠ»Ρ. ΠΠ½ΠΎ ΠΏΡΠΎΠ½ΠΈΠ·ΡΠ²Π°Π΅Ρ ΡΠΎΠ±ΡΡΠ²Π΅Π½Π½ΡΠΉ ΠΏΡΠΎΠ²ΠΎΠ΄Π½ΠΈΠΊ ΠΈ Π²Π»Π΅ΡΠ΅Ρ ΠΏΠΎΡΠ²Π»Π΅Π½ΠΈΠ΅ ΡΠ²ΠΎΠ΅Π³ΠΎ ΠΏΠΎΡΠΎΠΊΠ° ΡΠΊΠ²ΠΎΠ·Ρ ΠΊΠΎΠ½ΡΡΡ. ΠΡΠΈΡΠ΅ΠΌ ΡΠΎΠ±ΡΡΠ²Π΅Π½Π½ΡΠΉ ΠΏΠΎΡΠΎΠΊ ΠΏΡΠΎΠΏΠΎΡΡΠΈΠΎΠ½Π°Π»Π΅Π½ ΡΠ»Π΅ΠΊΡΡΠΈΡΠ΅ΡΡΠ²Ρ, ΠΊΠΎΡΠΎΡΠ°Ρ Π΅Π³ΠΎ ΠΏΠΎΡΠΎΠΆΠ΄Π°Π΅Ρ:
Π€Ρ = L*I
ΠΠ½Π°ΡΠ΅Π½ΠΈΠ΅ ΠΈΠ½Π΄ΡΠΊΡΠΈΠ²Π½ΠΎΡΡΠΈ ΡΠ°ΠΊΠΆΠ΅ ΡΠΎΡΠΌΠΈΡΡΠ΅ΡΡΡ ΠΈΠ· Π·Π°ΠΊΠΎΠ½Π° Π€Π°ΡΠ°Π΄Π΅Ρ.
ΠΠ΅Π΄Π²ΠΈΠΆΠΈΠΌΠ°Ρ ΡΠΈΡΡΠ΅ΠΌΠ°
Π‘ΠΈΠ»Π° ΠΠΎΡΠ΅Π½ΡΠ° ΠΎΠ±ΡΡΡΠ½ΡΠ΅Ρ Π²ΠΎΠ·Π½ΠΈΠΊΠ½ΠΎΠ²Π΅Π½ΠΈΠ΅ ΠΠΠ‘ ΠΏΡΠΈ Π΄Π²ΠΈΠΆΠ΅Π½ΠΈΠΈ ΡΠΈΡΡΠ΅ΠΌΡ Π² ΠΏΠΎΠ»Π΅ ΡΠΎ Π·Π½Π°ΡΠ΅Π½ΠΈΠ΅ΠΌ ΠΏΠΎΡΡΠΎΡΠ½Π½ΡΠΌ. ΠΠ½Π΄ΡΠΊΡΠΈΠΎΠ½Π½Π°Ρ ΠΠΠ‘ ΠΈΠΌΠ΅Π΅Ρ ΡΠΏΠΎΡΠΎΠ±Π½ΠΎΡΡΡ Π²ΠΎΠ·Π½ΠΈΠΊΠ°ΡΡ ΠΈ ΠΏΡΠΈ Π½Π΅ΠΏΠΎΠ΄Π²ΠΈΠΆΠ½ΠΎΠΉ ΠΏΡΠΎΠ²ΠΎΠ΄ΡΡΠ΅ΠΉ ΡΠΈΡΡΠ΅ΠΌΠ΅, Π½Π°Ρ ΠΎΠ΄ΡΡΠ΅ΠΉΡΡ Π² ΠΏΠ΅ΡΠ΅ΠΌΠ΅Π½Π½ΠΎΠΌ ΠΌΠ°Π³Π½ΠΈΡΠ½ΠΎΠΌ ΠΏΠΎΠ»Π΅. Π‘ΠΈΠ»Π° ΠΠΎΡΠ΅Π½ΡΠ° Π² ΡΠ°ΠΊΠΎΠΌ ΠΏΡΠΈΠΌΠ΅ΡΠ΅ Π½Π΅ ΡΠΏΠΎΡΠΎΠ±Π½Π° ΠΎΠ±ΡΡΡΠ½ΠΈΡΡ ΠΏΠΎΡΠ²Π»Π΅Π½ΠΈΠ΅ ΠΠΠ‘ ΠΈΠ½Π΄ΡΠΊΡΠΈΠΈ.
ΠΠ°ΠΊΡΠ²Π΅Π»Π» Π΄Π»Ρ ΠΏΡΠΎΠ²ΠΎΠ΄ΡΡΠΈΡ ΡΠΈΡΡΠ΅ΠΌ Π½Π΅ΠΏΠΎΠ΄Π²ΠΈΠΆΠ½ΠΎΠ³ΠΎ ΡΠΈΠΏΠ° ΠΏΡΠ΅Π΄Π»ΠΎΠΆΠΈΠ» ΠΏΡΠΈΠΌΠ΅Π½ΡΡΡ ΠΎΡΠΎΠ±ΠΎΠ΅ ΡΡΠ°Π²Π½Π΅Π½ΠΈΠ΅. ΠΠ½ΠΎ ΠΎΠ±ΡΡΡΠ½ΡΠ΅Ρ Π²ΠΎΠ·Π½ΠΈΠΊΠ½ΠΎΠ²Π΅Π½ΠΈΠ΅ Π² ΡΠ°ΠΊΠΈΡ ΡΠΈΡΡΠ΅ΠΌΠ°Ρ ΠΠΠ‘.
ΠΠ»Π°Π²Π½ΡΠΌ ΠΏΡΠΈΠ½ΡΠΈΠΏΠΎΠΌ Π·Π°ΠΊΠΎΠ½Π° Π€Π°ΡΠ°Π΄Π΅Ρ-ΠΠ°ΠΊΡΠ²Π΅Π»Π»Π° ΡΠ²Π»ΡΠ΅ΡΡΡ ΡΠ°ΠΊΡ, ΡΡΠΎ ΠΏΠ΅ΡΠ΅ΠΌΠ΅Π½Π½ΠΎΠ΅ ΠΏΠΎΠ»Π΅ ΠΎΠ±ΡΠ°Π·ΡΠ΅Ρ Π² ΠΏΡΠΎΡΡΡΠ°Π½ΡΡΠ²Π΅ Π²ΠΎΠΊΡΡΠ³ ΡΠ΅Π±Ρ ΡΠ»Π΅ΠΊΡΡΠΈΡΠ΅ΡΠΊΠΎΠ΅ ΠΏΠΎΠ»Π΅.
ΠΠ°ΠΆΠ½ΠΎ
ΠΠ½ΠΎ Π²ΡΡΡΡΠΏΠ°Π΅Ρ ΡΠ°ΠΊΡΠΎΡΠΎΠΌ, ΠΏΡΠΎΠ²ΠΎΡΠΈΡΡΡΡΠΈΠΌ ΠΏΠΎΡΠ²Π»Π΅Π½ΠΈΠ΅ ΡΠΎΠΊΠ° ΠΈΠ½Π΄ΡΠΊΡΠΈΠΈ Π² Π½Π΅Π΄Π²ΠΈΠΆΠΈΠΌΠΎΠΉ ΡΠΈΡΡΠ΅ΠΌΠ΅. ΠΠ΅ΡΠ΅ΠΌΠ΅ΡΠ΅Π½ΠΈΠ΅ Π²Π΅ΠΊΡΠΎΡΠ° (Π) ΠΏΠΎ ΡΡΠ°ΡΠΈΠΎΠ½Π°ΡΠ½ΡΠΌ ΠΊΠΎΠ½ΡΡΡΠ°ΠΌ (L) ΡΠ²Π»ΡΠ΅ΡΡΡ ΠΠΠ‘:
ΠΡΠΈ Π½Π°Π»ΠΈΡΠΈΠΈ ΡΠΎΠΊΠ° ΠΏΠ΅ΡΠ΅ΠΌΠ΅Π½Π½ΠΎΠ³ΠΎ Π·Π½Π°ΡΠ΅Π½ΠΈΡ Π·Π°ΠΊΠΎΠ½Ρ Π€Π°ΡΠ°Π΄Π΅Ρ Π²ΠΎΠ΄ΡΡΡΡ Π² ΡΡΠ°Π²Π½Π΅Π½ΠΈΡ ΠΠ°ΠΊΡΠ²Π΅Π»Π»Π°. ΠΡΠΈΡΠ΅ΠΌ ΠΎΠ½ΠΈ ΠΌΠΎΠ³ΡΡ Π±ΡΡΡ ΠΏΡΠ΅Π΄ΡΡΠ°Π²Π»Π΅Π½Ρ ΠΊΠ°ΠΊ Π² Π΄ΠΈΡΡΠ΅ΡΠ΅Π½ΡΠΈΠ°Π»ΡΠ½ΠΎΠΉ ΡΠΎΡΠΌΠ΅, ΡΠ°ΠΊ ΠΈ Π² Π²ΠΈΠ΄Π΅ ΠΈΠ½ΡΠ΅Π³ΡΠ°Π»ΠΎΠ².
Π’ΡΡΠ΄Ρ Π² ΠΎΠ±Π»Π°ΡΡΠΈ ΡΠ»Π΅ΠΊΡΡΠΎΠ»ΠΈΠ·Π°
ΠΡΠΈ ΠΈΡΠΏΠΎΠ»ΡΠ·ΠΎΠ²Π°Π½ΠΈΠΈ Π·Π°ΠΊΠΎΠ½ΠΎΠ² Π€Π°ΡΠ°Π΄Π΅Ρ ΠΎΠΏΠΈΡΡΠ²Π°ΡΡΡΡ Π·Π°ΠΊΠΎΠ½ΠΎΠΌΠ΅ΡΠ½ΠΎΡΡΠΈ, ΠΊΠΎΡΠΎΡΡΠ΅ ΡΡΡΠ΅ΡΡΠ²ΡΡΡ ΠΏΡΠΈ ΡΠ»Π΅ΠΊΡΡΠΎΠ»ΠΈΠ·Π΅. ΠΡΠΎΡ ΠΏΡΠΎΡΠ΅ΡΡ Π·Π°ΠΊΠ»ΡΡΠ°Π΅ΡΡΡ Π² ΠΏΡΠ΅Π²ΡΠ°ΡΠ΅Π½ΠΈΠΈ Π²Π΅ΡΠ΅ΡΡΠ² Ρ ΡΠ°Π·Π½ΠΎΠΎΠ±ΡΠ°Π·Π½ΡΠΌΠΈ Ρ Π°ΡΠ°ΠΊΡΠ΅ΡΠΈΡΡΠΈΠΊΠ°ΠΌΠΈ. ΠΡΠΎ ΠΏΡΠΎΠΈΡΡ ΠΎΠ΄ΠΈΡ ΠΏΡΠΈ Π΄Π²ΠΈΠΆΠ΅Π½ΠΈΠΈ ΡΠ»Π΅ΠΊΡΡΠΈΡΠ΅ΡΡΠ²Π° ΡΠΊΠ²ΠΎΠ·Ρ ΡΠ»Π΅ΠΊΡΡΠΎΠ»ΠΈΡ.
ΠΡΠΈ Π·Π°ΠΊΠΎΠ½ΠΎΠΌΠ΅ΡΠ½ΠΎΡΡΠΈ Π±ΡΠ»ΠΈ Π΄ΠΎΠΊΠ°Π·Π°Π½Ρ Π. Π€Π°ΡΠ°Π΄Π΅Π΅ΠΌ Π² 1834 Π³ΠΎΠ΄Ρ. ΠΠ΅ΡΠ²ΠΎΠ΅ ΡΡΠ²Π΅ΡΠΆΠ΄Π΅Π½ΠΈΠ΅ Π³Π»Π°ΡΠΈΡ, ΡΡΠΎ ΠΌΠ°ΡΡΠ° Π²Π΅ΡΠ΅ΡΡΠ²Π°, ΠΊΠΎΡΠΎΡΠΎΠ΅ ΠΎΠ±ΡΠ°Π·ΡΠ΅ΡΡΡ Π½Π° ΡΠ»Π΅ΠΊΡΡΠΎΠ΄Π΅, ΠΌΠ΅Π½ΡΠ΅ΡΡΡ ΡΠΎΠΎΡΠ²Π΅ΡΡΡΠ²Π΅Π½Π½ΠΎ Π·Π°ΡΡΠ΄Ρ, ΠΏΠ΅ΡΠ΅ΠΌΠ΅ΡΠ΅Π½Π½ΠΎΠΌΡ ΡΠΊΠ²ΠΎΠ·Ρ ΡΠ»Π΅ΠΊΡΡΠΎΠ»ΠΈΡ.
ΠΡΠΎΡΠΎΠ΅ ΡΡΠ²Π΅ΡΠΆΠ΄Π΅Π½ΠΈΠ΅ Π³Π»Π°ΡΠΈΡ, ΡΡΠΎ ΡΠΊΠ²ΠΈΠ²Π°Π»Π΅Π½ΡΡ ΠΊΠΎΠΌΠΏΠΎΠ½Π΅Π½ΡΠΎΠ² Ρ ΡΠ°Π·Π½ΡΠΌΠΈ Ρ Π°ΡΠ°ΠΊΡΠ΅ΡΠΈΡΡΠΈΠΊΠ°ΠΌΠΈ ΠΏΡΠΎΠΏΠΎΡΡΠΈΠΎΠ½Π°Π»ΡΠ½Ρ Ρ ΠΈΠΌΠΈΡΠ΅ΡΠΊΠΈΠΌ ΡΠΊΠ²ΠΈΠ²Π°Π»Π΅Π½ΡΠ°ΠΌ ΡΡΠΈΡ ΠΊΠΎΠΌΠΏΠΎΠ½Π΅Π½ΡΠΎΠ².
ΠΠ±Π° ΠΏΡΠ΅Π΄ΡΡΠ°Π²Π»Π΅Π½Π½ΡΡ ΡΡΠ²Π΅ΡΠΆΠ΄Π΅Π½ΠΈΡ ΡΠΎΠ²ΠΌΠ΅ΡΠ°ΡΡΡΡ Π² ΠΎΠ±ΡΠ΅Π΄ΠΈΠ½Π΅Π½Π½ΡΠΉ Π·Π°ΠΊΠΎΠ½ Π€Π°ΡΠ°Π΄Π΅Ρ. ΠΠ· Π½Π΅Π³ΠΎ ΡΠ»Π΅Π΄ΡΠ΅Ρ, ΡΡΠΎ ΡΠΈΡΠ»ΠΎ Π€Π°ΡΠ°Π΄Π΅Ρ Π±ΡΠ΄Π΅Ρ ΡΠ°Π²Π½ΡΡΡΡΡ ΡΠ»Π΅ΠΊΡΡΠΈΡΠ΅ΡΡΠ²Ρ, ΡΠΏΠΎΡΠΎΠ±Π½ΠΎΠΌΡ Π²ΡΠ΄Π΅Π»ΠΈΡΡ Π½Π° ΡΠ»Π΅ΠΊΡΡΠΎΠ»ΠΈΡΠ΅ 1 ΠΌΠΎΠ»Ρ Π²Π΅ΡΠ΅ΡΡΠ²Π°. ΠΠ΅ ΡΠ°ΡΡΡΠΈΡΡΠ²Π°ΡΡ Π½Π° Π΅Π΄ΠΈΠ½ΠΈΡΡ Π²Π°Π»Π΅Π½ΡΠ½ΠΎΡΡΠΈ. ΠΠΌΠ΅Π½Π½ΠΎ ΠΏΠΎ ΠΎΠ±ΡΠ΅Π΄ΠΈΠ½Π΅Π½Π½ΠΎΠΉ ΡΠΎΡΠΌΡΠ»Π΅ Π² Π΄Π°Π»Π΅ΠΊΠΎΠΌ 1874 Π³ΠΎΠ΄Ρ Π±ΡΠ» Π²ΡΡΠΈΡΠ»Π΅Π½ Π·Π°ΡΡΠ΄ ΡΠ»Π΅ΠΊΡΡΠΎΠ½Π°.
ΠΠ°ΠΊΠΎΠ½Ρ ΡΠ»Π΅ΠΊΡΡΠΎΠ»ΠΈΠ·Π°, ΡΡΡΠ°Π½ΠΎΠ²Π»Π΅Π½Π½ΡΠ΅ Π€Π°ΡΠ°Π΄Π΅Π΅ΠΌ, ΡΠ΅ΡΡΠΈΡΠΎΠ²Π°Π»ΠΈΡΡ ΠΏΡΠΈ ΡΠ°Π·Π»ΠΈΡΠ½ΠΎΠΌ Π·Π½Π°ΡΠ΅Π½ΠΈΠΈ ΡΠΎΠΊΠ°, ΡΠ΅ΠΌΠΏΠ΅ΡΠ°ΡΡΡΡ, Π΄Π°Π²Π»Π΅Π½ΠΈΡ, Π° ΡΠ°ΠΊΠΆΠ΅ ΠΏΡΠΈ ΠΎΠ΄Π½ΠΎΠ²ΡΠ΅ΠΌΠ΅Π½Π½ΠΎΠΌ Π²ΡΠ΄Π΅Π»Π΅Π½ΠΈΠΈ Π΄Π²ΡΡ ΠΈ Π±ΠΎΠ»Π΅Π΅ Π²Π΅ΡΠ΅ΡΡΠ². ΠΠ»Π΅ΠΊΡΡΠΎΠ»ΠΈΠ· ΡΠ°ΠΊΠΆΠ΅ ΠΏΡΠΎΠ²ΠΎΠ΄ΠΈΠ»ΡΡ Π² ΡΠ°Π·Π½ΡΡ ΡΠ°ΡΠΏΠ»Π°Π²Π°Ρ ΠΈ ΡΠ°ΡΡΠ²ΠΎΡΠΈΡΠ΅Π»ΡΡ .
ΠΠΎΠ½ΡΠ΅Π½ΡΡΠ°ΡΠΈΡ ΡΠ»Π΅ΠΊΡΡΠΎΠ»ΠΈΡΠ° ΡΠ°ΠΊΠΆΠ΅ ΠΎΡΠ»ΠΈΡΠ°Π»Π°ΡΡ Π² ΡΠ°Π·Π½ΡΡ ΠΎΠΏΡΡΠ°Ρ . ΠΡΠΈ ΡΡΠΎΠΌ ΠΈΠ½ΠΎΠ³Π΄Π° Π½Π°Π±Π»ΡΠ΄Π°Π»ΠΈΡΡ Π½Π΅Π±ΠΎΠ»ΡΡΠΈΠ΅ ΠΎΡΠΊΠ»ΠΎΠ½Π΅Π½ΠΈΡ ΠΎΡ Π·Π°ΠΊΠΎΠ½Π° Π€Π°ΡΠ°Π΄Π΅Ρ. ΠΠ½ΠΈ ΠΎΠ±ΡΡΡΠ½ΡΡΡΡΡ ΡΠ»Π΅ΠΊΡΡΠΎΠ½Π½ΠΎΠΉ ΠΏΡΠΎΠ²ΠΎΠ΄ΠΈΠΌΠΎΡΡΡΡ ΡΠ»Π΅ΠΊΡΡΠΎΠ»ΠΈΡΠΎΠ², ΠΊΠΎΡΠΎΡΠ°Ρ ΠΎΠΏΡΠ΅Π΄Π΅Π»ΡΠ΅ΡΡΡ Π½Π°ΡΠ°Π²Π½Π΅ Ρ ΠΈΠΎΠ½Π½ΠΎΠΉ ΠΏΡΠΎΠ²ΠΎΠ΄ΠΈΠΌΠΎΡΡΡΡ.
ΠΡΠΊΡΡΡΠΈΡ, ΡΠ΄Π΅Π»Π°Π½Π½ΡΠ΅ Π°Π½Π³Π»ΠΈΠΉΡΠΊΠΈΠΌ ΡΠΈΠ·ΠΈΠΊΠΎΠΌ Π. Π€Π°ΡΠ°Π΄Π΅Π΅ΠΌ, ΠΏΠΎΠ·Π²ΠΎΠ»ΠΈΠ»ΠΈ ΠΎΠΏΠΈΡΠ°ΡΡ ΠΌΠ½ΠΎΠΆΠ΅ΡΡΠ²ΠΎ ΡΠ²Π»Π΅Π½ΠΈΠΉ. ΠΠ³ΠΎ Π·Π°ΠΊΠΎΠ½Ρ ΡΠ²Π»ΡΡΡΡΡ ΠΎΡΠ½ΠΎΠ²ΠΎΠΉ ΡΠΎΠ²ΡΠ΅ΠΌΠ΅Π½Π½ΠΎΠΉ ΡΠ»Π΅ΠΊΡΡΠΎΠ΄ΠΈΠ½Π°ΠΌΠΈΠΊΠΈ. ΠΠΎ ΡΡΠΎΠΌΡ ΠΏΡΠΈΠ½ΡΠΈΠΏΡ ΡΡΠ½ΠΊΡΠΈΠΎΠ½ΠΈΡΡΠ΅Ρ ΡΠ°Π·Π»ΠΈΡΠ½ΠΎΠ΅ ΡΠΎΠ²ΡΠ΅ΠΌΠ΅Π½Π½ΠΎΠ΅ ΠΎΠ±ΠΎΡΡΠ΄ΠΎΠ²Π°Π½ΠΈΠ΅.
ΠΡΡΠΎΡΠ½ΠΈΠΊ: https://ProTransformatory.ru/raschety/zakon-faradeya
ΠΠ°ΠΊΠΎΠ½ ΡΠ»Π΅ΠΊΡΡΠΎΠΌΠ°Π³Π½ΠΈΡΠ½ΠΎΠΉ ΠΈΠ½Π΄ΡΠΊΡΠΈΠΈ Π€Π°ΡΠ°Π΄Π΅Ρ
Π‘ΠΎΠ΄Π΅ΡΠΆΠ°Π½ΠΈΠ΅:
- ΠΡΡΠΎΡΠΈΡ ΠΎΡΠΊΡΡΡΠΈΡ
- ΠΠ°ΠΊΠΎΠ½Ρ ΡΠ»Π΅ΠΊΡΡΠΎΠΌΠ°Π³Π½ΠΈΡΠ½ΠΎΠΉ ΠΈΠ½Π΄ΡΠΊΡΠΈΠΈ
- ΠΠΈΠ΄Π΅ΠΎΡΡΠΎΠΊ
ΠΡΠ»ΠΈ Π²Π·ΡΡΡ Π·Π°ΠΌΠΊΠ½ΡΡΡΡ ΠΏΡΠΎΠ²ΠΎΠ΄ΡΡΡΡ ΡΠΈΡΡΠ΅ΠΌΡ ΠΈ ΡΠΎΠ·Π΄Π°ΡΡ Π² Π½Π΅ΠΉ ΡΡΠ»ΠΎΠ²ΠΈΡ Π΄Π»Ρ ΡΠΎΠ³ΠΎ ΡΡΠΎΠ±Ρ ΠΌΠ°Π³Π½ΠΈΡΠ½ΡΠΉ ΠΏΠΎΡΠΎΠΊ ΠΈΠ·ΠΌΠ΅Π½ΠΈΠ»ΡΡ Π² ΠΌΠ°Π³Π½ΠΈΡΠ½ΠΎΠΌ ΠΏΠΎΠ»Π΅, ΡΠΎ Π² ΡΠ΅Π·ΡΠ»ΡΡΠ°ΡΠ΅ ΡΡΠΈΡ Π΄Π²ΠΈΠΆΠ΅Π½ΠΈΠΉ ΠΏΠΎΡΠ²ΠΈΡΡΡ ΡΠ»Π΅ΠΊΡΡΠΈΡΠ΅ΡΠΊΠΈΠΉ ΡΠΎΠΊ. ΠΠ°Π½Π½ΠΎΠ΅ ΠΎΠ±ΡΡΠΎΡΡΠ΅Π»ΡΡΡΠ²ΠΎ ΠΎΠΏΠΈΡΡΠ²Π°Π΅Ρ Π·Π°ΠΊΠΎΠ½ ΡΠ»Π΅ΠΊΡΡΠΎΠΌΠ°Π³Π½ΠΈΡΠ½ΠΎΠΉ ΠΈΠ½Π΄ΡΠΊΡΠΈΠΈ Π€Π°ΡΠ°Π΄Π΅Ρ β Π°Π½Π³Π»ΠΈΠΉΡΠΊΠΎΠ³ΠΎ ΡΡΠ΅Π½ΠΎΠ³ΠΎ, ΠΊΠΎΡΠΎΡΡΠΉ ΠΏΡΠΈ ΠΏΡΠΎΠ²Π΅Π΄Π΅Π½ΠΈΠΈ ΠΎΠΏΡΡΠΎΠ² Π΄ΠΎΠ±ΠΈΠ»ΡΡ ΠΏΡΠ΅Π²ΡΠ°ΡΠ΅Π½ΠΈΡ ΠΌΠ°Π³Π½ΠΈΡΠ½ΠΎΠΉ ΡΠ½Π΅ΡΠ³ΠΈΠΈ Π² ΡΠ»Π΅ΠΊΡΡΠΈΡΠ΅ΡΡΠ²ΠΎ. ΠΠ½ΠΎ ΠΏΠΎΠ»ΡΡΠΈΠ»ΠΎ Π½Π°Π·Π²Π°Π½ΠΈΠ΅ ΠΈΠ½Π΄ΡΠΊΡΠΈΠΎΠ½Π½ΠΎΠ³ΠΎ, ΠΏΠΎΡΠΊΠΎΠ»ΡΠΊΡ Π΄ΠΎ ΡΠΎΠ³ΠΎ Π²ΡΠ΅ΠΌΠ΅Π½ΠΈ Π΅Π³ΠΎ ΠΌΠΎΠΆΠ½ΠΎ Π±ΡΠ»ΠΎ ΡΠΎΠ·Π΄Π°ΡΡ Π»ΠΈΡΡ Π³Π°Π»ΡΠ²Π°Π½ΠΈΡΠ΅ΡΠΊΠΈΠΌ ΠΏΡΡΠ΅ΠΌ.
ΠΡΡΠΎΡΠΈΡ ΠΎΡΠΊΡΡΡΠΈΡ
Π―Π²Π»Π΅Π½ΠΈΠ΅ ΡΠ»Π΅ΠΊΡΡΠΎΠΌΠ°Π³Π½ΠΈΡΠ½ΠΎΠΉ ΠΈΠ½Π΄ΡΠΊΡΠΈΠΈ Π±ΡΠ»ΠΎ ΠΎΡΠΊΡΡΡΠΎ ΡΡΠ°Π·Ρ Π΄Π²ΡΠΌΡ ΡΡΠ΅Π½ΡΠΌΠΈ. ΠΡΠΎ Π±ΡΠ»ΠΈ ΠΠ°ΠΉΠΊΠ» Π€Π°ΡΠ°Π΄Π΅ΠΉ ΠΈ ΠΠΆΠΎΠ·Π΅Ρ ΠΠ΅Π½ΡΠΈ, ΡΠ΄Π΅Π»Π°Π²ΡΠΈΠ΅ ΡΠ²ΠΎΠ΅ ΠΎΡΠΊΡΡΡΠΈΠ΅ Π² 1831 Π³ΠΎΠ΄Ρ. ΠΡΠ±Π»ΠΈΠΊΠ°ΡΠΈΡ Π€Π°ΡΠ°Π΄Π΅Π΅ΠΌ ΡΠ΅Π·ΡΠ»ΡΡΠ°ΡΠΎΠ² ΠΏΡΠΎΠ²Π΅Π΄Π΅Π½Π½ΡΡ ΡΠΊΡΠΏΠ΅ΡΠΈΠΌΠ΅Π½ΡΠΎΠ² Π±ΡΠ»Π° ΡΠ΄Π΅Π»Π°Π½Π° ΡΠ°Π½ΡΡΠ΅ Π΅Π³ΠΎ ΠΊΠΎΠ»Π»Π΅Π³ΠΈ, ΠΏΠΎΡΡΠΎΠΌΡ ΠΈΠ½Π΄ΡΠΊΡΠΈΡ ΡΠ²ΡΠ·ΡΠ²Π°ΡΡ ΠΈΠΌΠ΅Π½Π½ΠΎ Ρ ΡΡΠΈΠΌ ΡΡΠ΅Π½ΡΠΌ. Π Π΄Π°Π»ΡΠ½Π΅ΠΉΡΠ΅ΠΌ ΡΡΠΎ ΠΏΠΎΠ½ΡΡΠΈΠ΅ Π±ΡΠ»ΠΎ Π²ΠΊΠ»ΡΡΠ΅Π½ΠΎ Π² ΡΠΈΡΡΠ΅ΠΌΡ Π‘ΠΠ‘.
ΠΠ»Ρ Π΄Π΅ΠΌΠΎΠ½ΡΡΡΠ°ΡΠΈΠΈ ΡΠ²Π»Π΅Π½ΠΈΡ ΠΈΡΠΏΠΎΠ»ΡΠ·ΠΎΠ²Π°Π»ΡΡ ΠΆΠ΅Π»Π΅Π·Π½ΡΠΉ ΡΠΎΡ, Π½Π°ΠΏΠΎΠΌΠΈΠ½Π°ΡΡΠΈΠΉ ΠΊΠΎΠ½ΡΠΈΠ³ΡΡΠ°ΡΠΈΡ ΡΠΎΠ²ΡΠ΅ΠΌΠ΅Π½Π½ΠΎΠ³ΠΎ ΡΡΠ°Π½ΡΡΠΎΡΠΌΠ°ΡΠΎΡΠ°. ΠΡΠΎΡΠΈΠ²ΠΎΠΏΠΎΠ»ΠΎΠΆΠ½ΡΠ΅ ΡΡΠΎΡΠΎΠ½Ρ Π΅Π³ΠΎ Π±ΡΠ»ΠΈ ΠΎΠ±ΠΌΠΎΡΠ°Π½Ρ Π΄Π²ΡΠΌΡ ΠΏΡΠΎΠ²ΠΎΠ΄Π½ΠΈΠΊΠ°ΠΌΠΈ Ρ ΡΠ΅Π»ΡΡ ΠΈΡΠΏΠΎΠ»ΡΠ·ΠΎΠ²Π°Π½ΠΈΡ ΡΠ»Π΅ΠΊΡΡΠΎΠΌΠ°Π³Π½ΠΈΡΠ½ΡΡ ΡΠ²ΠΎΠΉΡΡΠ².
Π ΠΎΠ΄Π½ΠΎΠΌΡ ΠΈΠ· ΠΏΡΠΎΠ²ΠΎΠ΄ΠΎΠ² ΠΏΠΎΠ΄ΠΊΠ»ΡΡΠ°Π»ΡΡ ΡΠΎΠΊ, Π²ΡΠ·ΡΠ²Π°ΡΡΠΈΠΉ ΡΠ²ΠΎΠ΅ΠΎΠ±ΡΠ°Π·Π½ΡΡ ΡΠ»Π΅ΠΊΡΡΠΈΡΠ΅ΡΠΊΡΡ Π²ΠΎΠ»Π½Ρ ΠΏΡΠΈ ΠΏΡΠΎΡ ΠΎΠΆΠ΄Π΅Π½ΠΈΠΈ ΡΠΊΠ²ΠΎΠ·Ρ ΡΠΎΡ, ΠΈ Π½Π΅ΠΊΠΎΡΠΎΡΡΠΉ ΡΠ»Π΅ΠΊΡΡΠΈΡΠ΅ΡΠΊΠΈΠΉ Π²ΡΠΏΠ»Π΅ΡΠΊ Ρ ΠΏΡΠΎΡΠΈΠ²ΠΎΠΏΠΎΠ»ΠΎΠΆΠ½ΠΎΠΉ ΡΡΠΎΡΠΎΠ½Ρ. ΠΠ°Π»ΠΈΡΠΈΠ΅ ΡΠΎΠΊΠ° Π±ΡΠ»ΠΎ Π·Π°ΡΠΈΠΊΡΠΈΡΠΎΠ²Π°Π½ΠΎ Π³Π°Π»ΡΠ²Π°Π½ΠΎΠΌΠ΅ΡΡΠΎΠΌ. Π’ΠΎΡΠ½ΠΎ ΡΠ°ΠΊΠΎΠΉ ΠΆΠ΅ Π²ΡΠΏΠ»Π΅ΡΠΊ ΡΠ»Π΅ΠΊΡΡΠΈΡΠ΅ΡΡΠ²Π° Π½Π°Π±Π»ΡΠ΄Π°Π»ΡΡ ΠΈ Π² ΠΌΠΎΠΌΠ΅Π½Ρ ΠΎΡΠΊΠ»ΡΡΠ΅Π½ΠΈΡ ΠΏΡΠΎΠ²ΠΎΠ΄Π°.
Π‘ΠΎΠ²Π΅Ρ
ΠΠΎΡΡΠ΅ΠΏΠ΅Π½Π½ΠΎ Π±ΡΠ»ΠΈ ΠΎΠ±Π½Π°ΡΡΠΆΠ΅Π½Ρ ΠΈ Π΄ΡΡΠ³ΠΈΠ΅ ΡΠΎΡΠΌΡ ΠΏΡΠΎΡΠ²Π»Π΅Π½ΠΈΡ ΡΠ»Π΅ΠΊΡΡΠΎΠΌΠ°Π³Π½ΠΈΡΠ½ΠΎΠΉ ΠΈΠ½Π΄ΡΠΊΡΠΈΠΈ. ΠΡΠ°ΡΠΊΠΎΠ²ΡΠ΅ΠΌΠ΅Π½Π½ΠΎΠ΅ Π²ΠΎΠ·Π½ΠΈΠΊΠ½ΠΎΠ²Π΅Π½ΠΈΠ΅ ΡΠΎΠΊΠ° Π½Π°Π±Π»ΡΠ΄Π°Π»ΠΎΡΡ Π²ΠΎ Π²ΡΠ΅ΠΌΡ Π³Π΅Π½Π΅ΡΠ°ΡΠΈΠΈ Π΅Π³ΠΎ Π½Π° ΠΌΠ΅Π΄Π½ΠΎΠΌ Π΄ΠΈΡΠΊΠ΅, Π²ΡΠ°ΡΠ°ΡΡΠ΅ΠΌΡΡ Π²ΠΎΠ·Π»Π΅ ΠΌΠ°Π³Π½ΠΈΡΠ°. ΠΠ° ΡΠ°ΠΌΠΎΠΌ Π΄ΠΈΡΠΊΠ΅ Π±ΡΠ» ΡΡΡΠ°Π½ΠΎΠ²Π»Π΅Π½ ΡΠΊΠΎΠ»ΡΠ·ΡΡΠΈΠΉ ΡΠ»Π΅ΠΊΡΡΠΎΠΏΡΠΎΠ²ΠΎΠ΄.
ΠΠ°ΠΈΠ±ΠΎΠ»ΡΡΠΈΠ΅ ΠΏΡΠ΅Π΄ΡΡΠ°Π²Π»Π΅Π½ΠΈΠ΅ ΠΎ ΡΠΎΠΌ, ΡΡΠΎ ΡΠ°ΠΊΠΎΠ΅ ΠΈΠ½Π΄ΡΠΊΡΠΈΠ²Π½ΠΎΡΡΡ, Π΄Π°Π» ΡΠΊΡΠΏΠ΅ΡΠΈΠΌΠ΅Π½Ρ Ρ Π΄Π²ΡΠΌΡ ΠΊΠ°ΡΡΡΠΊΠ°ΠΌΠΈ. ΠΠ΄Π½Π° ΠΈΠ· Π½ΠΈΡ , Ρ ΠΌΠ΅Π½ΡΡΠΈΠΌΠΈ ΡΠ°Π·ΠΌΠ΅ΡΠ°ΠΌΠΈ, ΠΏΠΎΠ΄ΠΊΠ»ΡΡΠ΅Π½Π° ΠΊ ΠΆΠΈΠ΄ΠΊΠΎΡΡΠ½ΠΎΠΉ Π±Π°ΡΠ°ΡΠ΅Π΅, ΡΠ°ΡΠΏΠΎΠ»ΠΎΠΆΠ΅Π½Π½ΠΎΠΉ Π½Π° ΡΠΈΡΡΠ½ΠΊΠ΅ Ρ ΠΏΡΠ°Π²ΠΎΠΉ ΡΡΠΎΡΠΎΠ½Ρ. Π’Π°ΠΊΠΈΠΌ ΠΎΠ±ΡΠ°Π·ΠΎΠΌ, ΡΠ΅ΡΠ΅Π· ΡΡΡ ΠΊΠ°ΡΡΡΠΊΡ Π½Π°ΡΠΈΠ½Π°Π΅Ρ ΠΏΡΠΎΡΠ΅ΠΊΠ°ΡΡ ΡΠ»Π΅ΠΊΡΡΠΈΡΠ΅ΡΠΊΠΈΠΉ ΡΠΎΠΊ, ΠΏΠΎΠ΄ Π΄Π΅ΠΉΡΡΠ²ΠΈΠ΅ΠΌ ΠΊΠΎΡΠΎΡΠΎΠ³ΠΎ Π²ΠΎΠ·Π½ΠΈΠΊΠ°Π΅Ρ ΠΌΠ°Π³Π½ΠΈΡΠ½ΠΎΠ΅ ΠΏΠΎΠ»Π΅.
ΠΠΎΠ³Π΄Π° ΠΎΠ±Π΅ ΠΊΠ°ΡΡΡΠΊΠΈ Π½Π°Ρ ΠΎΠ΄ΡΡΡΡ Π² Π½Π΅ΠΏΠΎΠ΄Π²ΠΈΠΆΠ½ΠΎΠΌ ΠΏΠΎΠ»ΠΎΠΆΠ΅Π½ΠΈΠΈ ΠΎΡΠ½ΠΎΡΠΈΡΠ΅Π»ΡΠ½ΠΎ Π΄ΡΡΠ³ Π΄ΡΡΠ³Π°, Π½ΠΈΠΊΠ°ΠΊΠΈΡ ΡΠ²Π»Π΅Π½ΠΈΠΉ Π½Π΅ ΠΏΡΠΎΠΈΡΡ ΠΎΠ΄ΠΈΡ. ΠΠΎΠ³Π΄Π° Π½Π΅Π±ΠΎΠ»ΡΡΠ°Ρ ΠΊΠ°ΡΡΡΠΊΠ° Π½Π°ΡΠΈΠ½Π°Π΅Ρ Π΄Π²ΠΈΠ³Π°ΡΡΡΡ, ΡΠΎ Π΅ΡΡΡ Π²ΡΡ ΠΎΠ΄ΠΈΡΡ ΠΈΠ· Π±ΠΎΠ»ΡΡΠΎΠΉ ΠΊΠ°ΡΡΡΠΊΠΈ ΠΈΠ»ΠΈ Π²Ρ ΠΎΠ΄ΠΈΡΡ Π² Π½Π΅Π΅, Π½Π°ΡΡΡΠΏΠ°Π΅Ρ ΠΈΠ·ΠΌΠ΅Π½Π΅Π½ΠΈΠ΅ ΠΌΠ°Π³Π½ΠΈΡΠ½ΠΎΠ³ΠΎ ΠΏΠΎΡΠΎΠΊΠ°. Π ΡΠ΅Π·ΡΠ»ΡΡΠ°ΡΠ΅, Π² Π±ΠΎΠ»ΡΡΠΎΠΉ ΠΊΠ°ΡΡΡΠΊΠ΅ Π½Π°Π±Π»ΡΠ΄Π°Π΅ΡΡΡ ΠΏΠΎΡΠ²Π»Π΅Π½ΠΈΠ΅ ΡΠ»Π΅ΠΊΡΡΠΎΠ΄Π²ΠΈΠΆΡΡΠ΅ΠΉ ΡΠΈΠ»Ρ.
ΠΡΠΊΡΡΡΠΈΠ΅ Π€Π°ΡΠ°Π΄Π΅Ρ Π΄ΠΎΡΠ°Π±ΠΎΡΠ°Π» Π΄ΡΡΠ³ΠΎΠΉ ΡΡΠ΅Π½ΡΠΉ β ΠΠ°ΠΊΡΠ²Π΅Π»Π», ΠΊΠΎΡΠΎΡΡΠΉ ΠΎΠ±ΠΎΡΠ½ΠΎΠ²Π°Π» Π΅Π³ΠΎ ΠΌΠ°ΡΠ΅ΠΌΠ°ΡΠΈΡΠ΅ΡΠΊΠΈ, ΠΎΡΠΎΠ±ΡΠ°ΠΆΠ°Ρ Π΄Π°Π½Π½ΠΎΠ΅ ΡΠΈΠ·ΠΈΡΠ΅ΡΠΊΠΎΠ΅ ΡΠ²Π»Π΅Π½ΠΈΠ΅ Π΄ΠΈΡΡΠ΅ΡΠ΅Π½ΡΠΈΠ°Π»ΡΠ½ΡΠΌΠΈ ΡΡΠ°Π²Π½Π΅Π½ΠΈΡΠΌΠΈ. ΠΡΠ΅ ΠΎΠ΄Π½ΠΎΠΌΡ ΡΡΠ΅Π½ΠΎΠΌΡ-ΡΠΈΠ·ΠΈΠΊΡ β ΠΠ΅Π½ΡΡ ΡΠ΄Π°Π»ΠΎΡΡ ΠΎΠΏΡΠ΅Π΄Π΅Π»ΠΈΡΡ Π½Π°ΠΏΡΠ°Π²Π»Π΅Π½ΠΈΠ΅ ΡΠ»Π΅ΠΊΡΡΠΎΡΠΎΠΊΠ° ΠΈ ΠΠΠ‘, ΠΏΠΎΠ»ΡΡΠ΅Π½Π½ΡΡ ΠΏΠΎΠ΄ Π΄Π΅ΠΉΡΡΠ²ΠΈΠ΅ΠΌ ΡΠ»Π΅ΠΊΡΡΠΎΠΌΠ°Π³Π½ΠΈΡΠ½ΠΎΠΉ ΠΈΠ½Π΄ΡΠΊΡΠΈΠΈ.
ΠΠ°ΠΊΠΎΠ½Ρ ΡΠ»Π΅ΠΊΡΡΠΎΠΌΠ°Π³Π½ΠΈΡΠ½ΠΎΠΉ ΠΈΠ½Π΄ΡΠΊΡΠΈΠΈ
Π‘ΡΡΠ½ΠΎΡΡΡ ΡΠ»Π΅ΠΊΡΡΠΎΠΌΠ°Π³Π½ΠΈΡΠ½ΠΎΠΉ ΠΈΠ½Π΄ΡΠΊΡΠΈΠΈ ΠΎΠΏΡΠ΅Π΄Π΅Π»ΡΠ΅ΡΡΡ Π·Π°ΠΌΠΊΠ½ΡΡΡΠΌ ΠΊΠΎΠ½ΡΡΡΠΎΠΌ Ρ ΡΠ»Π΅ΠΊΡΡΠΎΠΏΡΠΎΠ²ΠΎΠ΄Π½ΠΎΡΡΡΡ, ΠΏΠ»ΠΎΡΠ°Π΄Ρ ΠΊΠΎΡΠΎΡΠΎΠ³ΠΎ ΠΏΡΠΎΠΏΡΡΠΊΠ°Π΅Ρ ΡΠ΅ΡΠ΅Π· ΡΠ΅Π±Ρ ΠΈΠ·ΠΌΠ΅Π½ΡΡΡΠΈΠΉΡΡ ΠΌΠ°Π³Π½ΠΈΡΠ½ΡΠΉ ΠΏΠΎΡΠΎΠΊ. Π ΡΡΠΎΡ ΠΌΠΎΠΌΠ΅Π½Ρ ΠΏΠΎΠ΄ Π²Π»ΠΈΡΠ½ΠΈΠ΅ΠΌ ΠΌΠ°Π³Π½ΠΈΡΠ½ΠΎΠ³ΠΎ ΠΏΠΎΡΠΎΠΊΠ° ΠΏΠΎΡΠ²Π»ΡΠ΅ΡΡΡ ΡΠ»Π΅ΠΊΡΡΠΎΠ΄Π²ΠΈΠΆΡΡΠ°Ρ ΡΠΈΠ»Π° Πi ΠΈ Π² ΠΊΠΎΠ½ΡΡΡΠ΅ Π½Π°ΡΠΈΠ½Π°Π΅Ρ ΡΠ΅ΡΡ ΡΠ»Π΅ΠΊΡΡΠΈΡΠ΅ΡΠΊΠΈΠΉ ΡΠΎΠΊ.
ΠΠ°ΠΊΠΎΠ½ Π€Π°ΡΠ°Π΄Π΅Ρ Π΄Π»Ρ ΡΠ»Π΅ΠΊΡΡΠΎΠΌΠ°Π³Π½ΠΈΡΠ½ΠΎΠΉ ΠΈΠ½Π΄ΡΠΊΡΠΈΠΈ Π·Π°ΠΊΠ»ΡΡΠ°Π΅ΡΡΡ Π² ΠΏΡΡΠΌΠΎΠΉ Π·Π°Π²ΠΈΡΠΈΠΌΠΎΡΡΠΈ ΠΠΠ‘ ΠΈ ΡΠΊΠΎΡΠΎΡΡΠΈ, ΡΠΎΡΡΠ°Π²Π»ΡΡΡΠΈΡ ΠΏΡΠΎΠΏΠΎΡΡΠΈΡ. ΠΠ°Π½Π½Π°Ρ ΡΠΊΠΎΡΠΎΡΡΡ ΠΏΡΠ΅Π΄ΡΡΠ°Π²Π»ΡΠ΅Ρ ΡΠΎΠ±ΠΎΠΉ Π²ΡΠ΅ΠΌΡ, Π² ΡΠ΅ΡΠ΅Π½ΠΈΠ΅ ΠΊΠΎΡΠΎΡΠΎΠ³ΠΎ ΠΌΠ°Π³Π½ΠΈΡΠ½ΡΠΉ ΠΏΠΎΡΠΎΠΊ ΠΏΠΎΠ΄Π²Π΅ΡΠ³Π°Π΅ΡΡΡ ΠΈΠ·ΠΌΠ΅Π½Π΅Π½ΠΈΡΠΌ.
ΠΠ°Π½Π½ΡΠΉ Π·Π°ΠΊΠΎΠ½ Π²ΡΡΠ°ΠΆΠ°Π΅ΡΡΡ ΡΠΎΡΠΌΡΠ»ΠΎΠΉ Πi = β βΠ€/βt, Π² ΠΊΠΎΡΠΎΡΠΎΠΉ Πi β Π·Π½Π°ΡΠ΅Π½ΠΈΠ΅ ΡΠ»Π΅ΠΊΡΡΠΎΠ΄Π²ΠΈΠΆΡΡΠ΅ΠΉ ΡΠΈΠ»Ρ, Π²ΠΎΠ·Π½ΠΈΠΊΠ°ΡΡΠ΅ΠΉ Π² ΠΊΠΎΠ½ΡΡΡΠ΅, Π° βΠ€/βt ΡΠ²Π»ΡΠ΅ΡΡΡ ΡΠΊΠΎΡΠΎΡΡΡΡ ΠΈΠ·ΠΌΠ΅Π½Π΅Π½ΠΈΡ ΠΌΠ°Π³Π½ΠΈΡΠ½ΠΎΠ³ΠΎ ΠΏΠΎΡΠΎΠΊΠ°. Π ΡΡΠΎΠΉ ΡΠΎΡΠΌΡΠ»Π΅ Π½Π΅ ΡΠΎΠ²ΡΠ΅ΠΌ ΠΏΠΎΠ½ΡΡΠ½ΡΠΌ ΠΎΡΡΠ°Π΅ΡΡΡ Π·Π½Π°ΠΊ Β«ΠΌΠΈΠ½ΡΡΒ», Π½ΠΎ Π΅ΠΌΡ ΡΠΎΠΆΠ΅ ΠΈΠΌΠ΅Π΅ΡΡΡ ΡΠ²ΠΎΠ΅ ΠΎΠ±ΡΡΡΠ½Π΅Π½ΠΈΠ΅.
Π ΡΠΎΠΎΡΠ²Π΅ΡΡΡΠ²ΠΈΠΈ Ρ ΠΏΡΠ°Π²ΠΈΠ»ΠΎΠΌ ΡΡΡΡΠΊΠΎΠ³ΠΎ ΡΡΠ΅Π½ΠΎΠ³ΠΎ ΠΠ΅Π½ΡΠ°, ΠΈΠ·ΡΡΠ°Π²ΡΠ΅Π³ΠΎ ΠΎΡΠΊΡΡΡΠΈΡ Π€Π°ΡΠ°Π΄Π΅Ρ, ΡΡΠΎΡ Π·Π½Π°ΠΊ ΠΎΡΠΎΠ±ΡΠ°ΠΆΠ°Π΅Ρ Π½Π°ΠΏΡΠ°Π²Π»Π΅Π½ΠΈΠ΅ ΠΠΠ‘, Π²ΠΎΠ·Π½ΠΈΠΊΠ°ΡΡΠ΅ΠΉ Π² ΠΊΠΎΠ½ΡΡΡΠ΅.
Π’ΠΎ Π΅ΡΡΡ, Π½Π°ΠΏΡΠ°Π²Π»Π΅Π½ΠΈΠ΅ ΠΈΠ½Π΄ΡΠΊΡΠΈΠΎΠ½Π½ΠΎΠ³ΠΎ ΡΠΎΠΊΠ° ΠΏΡΠΎΠΈΡΡ ΠΎΠ΄ΠΈΡ ΡΠ°ΠΊΠΈΠΌ ΠΎΠ±ΡΠ°Π·ΠΎΠΌ, ΡΡΠΎ ΡΠΎΠ·Π΄Π°Π²Π°Π΅ΠΌΡΠΉ ΠΈΠΌ ΠΌΠ°Π³Π½ΠΈΡΠ½ΡΠΉ ΠΏΠΎΡΠΎΠΊ Π½Π° ΠΏΠ»ΠΎΡΠ°Π΄ΠΈ, ΠΎΠ³ΡΠ°Π½ΠΈΡΠ΅Π½Π½ΠΎΠΉ ΠΊΠΎΠ½ΡΡΡΠΎΠΌ, ΠΏΡΠ΅ΠΏΡΡΡΡΠ²ΡΠ΅Ρ ΠΈΠ·ΠΌΠ΅Π½Π΅Π½ΠΈΡΠΌ, Π²ΡΠ·Π²Π°Π½Π½ΡΠΌ ΡΡΠΈΠΌ ΡΠΎΠΊΠΎΠΌ.
ΠΡΠΊΡΡΡΠΈΡ Π€Π°ΡΠ°Π΄Π΅Ρ Π±ΡΠ»ΠΈ Π΄ΠΎΡΠ°Π±ΠΎΡΠ°Π½Ρ ΠΠ°ΠΊΡΠ²Π΅Π»Π»ΠΎΠΌ, Ρ ΠΊΠΎΡΠΎΡΠΎΠ³ΠΎ ΡΠ΅ΠΎΡΠΈΡ ΡΠ»Π΅ΠΊΡΡΠΎΠΌΠ°Π³Π½ΠΈΡΠ½ΠΎΠ³ΠΎ ΠΏΠΎΠ»Ρ ΠΏΠΎΠ»ΡΡΠΈΠ»Π° Π½ΠΎΠ²ΡΠ΅ Π½Π°ΠΏΡΠ°Π²Π»Π΅Π½ΠΈΡ. Π ΡΠ΅Π·ΡΠ»ΡΡΠ°ΡΠ΅, ΠΏΠΎΡΠ²ΠΈΠ»ΡΡ Π·Π°ΠΊΠΎΠ½ Π€Π°ΡΠ°Π΄Π΅Ρ ΠΈ ΠΠ°ΠΊΡΠ²Π΅Π»Π»Π°, Π²ΡΡΠ°ΠΆΠ΅Π½Π½ΡΠΉ Π² ΡΠ»Π΅Π΄ΡΡΡΠΈΡ ΡΠΎΡΠΌΡΠ»Π°Ρ :
- Edl = -βΠ€/βt β ΠΎΡΠΎΠ±ΡΠ°ΠΆΠ°Π΅Ρ ΡΠ»Π΅ΠΊΡΡΠΎΠ΄Π²ΠΈΠΆΡΡΡΡ ΡΠΈΠ»Ρ.
- Hdl = -βN/βt β ΠΎΡΠΎΠ±ΡΠ°ΠΆΠ°Π΅Ρ ΠΌΠ°Π³Π½ΠΈΡΠΎΠ΄Π²ΠΈΠΆΡΡΡΡ ΡΠΈΠ»Ρ.
Π ΡΡΠΈΡ ΡΠΎΡΠΌΡΠ»Π°Ρ Π ΡΠΎΠΎΡΠ²Π΅ΡΡΡΠ²ΡΠ΅Ρ Π½Π°ΠΏΡΡΠΆΠ΅Π½Π½ΠΎΡΡΠΈ ΡΠ»Π΅ΠΊΡΡΠΈΡΠ΅ΡΠΊΠΎΠ³ΠΎ ΠΏΠΎΠ»Ρ Π½Π° ΠΎΠΏΡΠ΅Π΄Π΅Π»Π΅Π½Π½ΠΎΠΌ ΡΡΠ°ΡΡΠΊΠ΅ dl, Π ΡΠ²Π»ΡΠ΅ΡΡΡ Π½Π°ΠΏΡΡΠΆΠ΅Π½Π½ΠΎΡΡΡΡ ΠΌΠ°Π³Π½ΠΈΡΠ½ΠΎΠ³ΠΎ ΠΏΠΎΠ»Ρ Π½Π° ΡΡΠΎΠΌ ΠΆΠ΅ ΡΡΠ°ΡΡΠΊΠ΅, N β ΠΏΠΎΡΠΎΠΊ ΡΠ»Π΅ΠΊΡΡΠΈΡΠ΅ΡΠΊΠΎΠΉ ΠΈΠ½Π΄ΡΠΊΡΠΈΠΈ, t β ΠΏΠ΅ΡΠΈΠΎΠ΄ Π²ΡΠ΅ΠΌΠ΅Π½ΠΈ.
ΠΠ±Π° ΡΡΠ°Π²Π½Π΅Π½ΠΈΡ ΠΎΡΠ»ΠΈΡΠ°ΡΡΡΡ ΡΠΈΠΌΠΌΠ΅ΡΡΠΈΡΠ½ΠΎΡΡΡΡ, ΠΏΠΎΠ·Π²ΠΎΠ»ΡΡΡΠ΅ΠΉ ΡΠ΄Π΅Π»Π°ΡΡ Π²ΡΠ²ΠΎΠ΄, ΡΡΠΎ ΠΌΠ°Π³Π½ΠΈΡΠ½ΡΠ΅ ΠΈ ΡΠ»Π΅ΠΊΡΡΠΈΡΠ΅ΡΠΊΠΈΠ΅ ΡΠ²Π»Π΅Π½ΠΈΡ ΡΠ²ΡΠ·Π°Π½Ρ ΠΌΠ΅ΠΆΠ΄Ρ ΡΠΎΠ±ΠΎΠΉ. Π‘ ΡΠΈΠ·ΠΈΡΠ΅ΡΠΊΠΎΠΉ ΡΠΎΡΠΊΠΈ Π·ΡΠ΅Π½ΠΈΡ ΡΡΠΈ ΡΠΎΡΠΌΡΠ»Ρ ΠΎΠΏΡΠ΅Π΄Π΅Π»ΡΡΡ ΡΠ»Π΅Π΄ΡΡΡΠ΅Π΅:
- ΠΠ·ΠΌΠ΅Π½Π΅Π½ΠΈΡΠΌ Π² ΡΠ»Π΅ΠΊΡΡΠΈΡΠ΅ΡΠΊΠΎΠΌ ΠΏΠΎΠ»Π΅ Π²ΡΠ΅Π³Π΄Π° ΡΠΎΠΏΡΡΡΡΠ²ΡΠ΅Ρ ΠΎΠ±ΡΠ°Π·ΠΎΠ²Π°Π½ΠΈΠ΅ ΠΌΠ°Π³Π½ΠΈΡΠ½ΠΎΠ³ΠΎ ΠΏΠΎΠ»Ρ.
- ΠΠ·ΠΌΠ΅Π½Π΅Π½ΠΈΡ Π² ΠΌΠ°Π³Π½ΠΈΡΠ½ΠΎΠΌ ΠΏΠΎΠ»Π΅ Π²ΡΠ΅Π³Π΄Π° ΠΏΡΠΎΠΈΡΡ ΠΎΠ΄ΡΡ ΠΎΠ΄Π½ΠΎΠ²ΡΠ΅ΠΌΠ΅Π½Π½ΠΎ Ρ ΠΎΠ±ΡΠ°Π·ΠΎΠ²Π°Π½ΠΈΠ΅ΠΌ ΡΠ»Π΅ΠΊΡΡΠΈΡΠ΅ΡΠΊΠΎΠ³ΠΎ ΠΏΠΎΠ»Ρ.
ΠΠ·ΠΌΠ΅Π½ΡΡΡΠΈΠΉΡΡ ΠΌΠ°Π³Π½ΠΈΡΠ½ΡΠΉ ΠΏΠΎΡΠΎΠΊ, ΠΏΡΠΎΡ ΠΎΠ΄ΡΡΠΈΠΉ ΡΠΊΠ²ΠΎΠ·Ρ Π·Π°ΠΌΠΊΠ½ΡΡΡΡ ΠΊΠΎΠ½ΡΠΈΠ³ΡΡΠ°ΡΠΈΡ ΠΏΡΠΎΠ²ΠΎΠ΄ΡΡΠ΅Π³ΠΎ ΠΊΠΎΠ½ΡΡΡΠ°, ΠΏΡΠΈΠ²ΠΎΠ΄ΠΈΡ ΠΊ Π²ΠΎΠ·Π½ΠΈΠΊΠ½ΠΎΠ²Π΅Π½ΠΈΡ Π² ΡΡΠΎΠΌ ΠΊΠΎΠ½ΡΡΡΠ΅ ΡΠ»Π΅ΠΊΡΡΠΈΡΠ΅ΡΠΊΠΎΠ³ΠΎ ΡΠΎΠΊΠ°. ΠΡΠΎ ΠΎΡΠ½ΠΎΠ²Π½Π°Ρ ΡΠΎΡΠΌΡΠ»ΠΈΡΠΎΠ²ΠΊΠ° Π·Π°ΠΊΠΎΠ½Π° Π€Π°ΡΠ°Π΄Π΅Ρ. ΠΡΠ»ΠΈ ΠΈΠ·Π³ΠΎΡΠΎΠ²ΠΈΡΡ ΠΏΡΠΎΠ²ΠΎΠ»ΠΎΡΠ½ΡΡ ΡΠ°ΠΌΠΊΡ ΠΈ ΠΏΠΎΠΌΠ΅ΡΡΠΈΡΡ Π΅Π΅ Π²Π½ΡΡΡΠΈ Π²ΡΠ°ΡΠ°ΡΡΠ΅Π³ΠΎΡΡ ΠΌΠ°Π³Π½ΠΈΡΠ°, ΡΠΎ Π² ΡΠ°ΠΌΠΎΠΉ ΡΠ°ΠΌΠΊΠ΅ ΠΏΠΎΡΠ²ΠΈΡΡΡ ΡΠ»Π΅ΠΊΡΡΠΈΡΠ΅ΡΡΠ²ΠΎ.
ΠΡΠΎ ΠΈ Π±ΡΠ΄Π΅Ρ ΠΈΠ½Π΄ΡΠΊΡΠΈΠΎΠ½Π½ΡΠΉ ΡΠΎΠΊ, Π² ΠΏΠΎΠ»Π½ΠΎΠΌ ΡΠΎΠΎΡΠ²Π΅ΡΡΡΠ²ΠΈΠΈ Ρ ΡΠ΅ΠΎΡΠΈΠ΅ΠΉ ΠΈ Π·Π°ΠΊΠΎΠ½ΠΎΠΌ ΠΠ°ΠΉΠΊΠ»Π° Π€Π°ΡΠ°Π΄Π΅Ρ. ΠΠ·ΠΌΠ΅Π½Π΅Π½ΠΈΡ ΠΌΠ°Π³Π½ΠΈΡΠ½ΠΎΠ³ΠΎ ΠΏΠΎΡΠΎΠΊΠ°, ΠΏΡΠΎΡ ΠΎΠ΄ΡΡΠ΅Π³ΠΎ ΡΠ΅ΡΠ΅Π· ΠΊΠΎΠ½ΡΡΡ, ΠΌΠΎΠ³ΡΡ Π±ΡΡΡ ΠΏΡΠΎΠΈΠ·Π²ΠΎΠ»ΡΠ½ΡΠΌΠΈ.
Π‘Π»Π΅Π΄ΠΎΠ²Π°ΡΠ΅Π»ΡΠ½ΠΎ, ΡΠΎΡΠΌΡΠ»Π° βΠ€/βt Π±ΡΠ²Π°Π΅Ρ Π½Π΅ ΡΠΎΠ»ΡΠΊΠΎ Π»ΠΈΠ½Π΅ΠΉΠ½ΠΎΠΉ, Π° Π² ΠΎΠΏΡΠ΅Π΄Π΅Π»Π΅Π½Π½ΡΡ ΡΡΠ»ΠΎΠ²ΠΈΡΡ ΠΏΡΠΈΠ½ΠΈΠΌΠ°Π΅Ρ Π»ΡΠ±ΡΡ ΠΊΠΎΠ½ΡΠΈΠ³ΡΡΠ°ΡΠΈΡ. ΠΡΠ»ΠΈ ΠΈΠ·ΠΌΠ΅Π½Π΅Π½ΠΈΡ ΠΏΡΠΎΠΈΡΡ ΠΎΠ΄ΡΡ Π»ΠΈΠ½Π΅ΠΉΠ½ΠΎ, ΡΠΎ ΠΠΠ‘ ΡΠ»Π΅ΠΊΡΡΠΎΠΌΠ°Π³Π½ΠΈΡΠ½ΠΎΠΉ ΠΈΠ½Π΄ΡΠΊΡΠΈΠΈ, Π²ΠΎΠ·Π½ΠΈΠΊΠ°ΡΡΠ΅ΠΉ Π² ΠΊΠΎΠ½ΡΡΡΠ΅, Π±ΡΠ΄Π΅Ρ ΠΏΠΎΡΡΠΎΡΠ½Π½ΠΎΠΉ.
ΠΠ±ΡΠ°ΡΠΈΡΠ΅ Π²Π½ΠΈΠΌΠ°Π½ΠΈΠ΅
ΠΡΠ΅ΠΌΠ΅Π½Π½ΠΎΠΉ ΠΈΠ½ΡΠ΅ΡΠ²Π°Π» t ΡΡΠ°Π½ΠΎΠ²ΠΈΡΡΡ ΠΊΠ°ΠΊΠΈΠΌ ΡΠ³ΠΎΠ΄Π½ΠΎ, Π° ΠΎΡΠ½ΠΎΡΠ΅Π½ΠΈΠ΅ βΠ€/βt Π½Π΅ Π±ΡΠ΄Π΅Ρ Π·Π°Π²ΠΈΡΠ΅ΡΡ ΠΎΡ Π΅Π³ΠΎ ΠΏΡΠΎΠ΄ΠΎΠ»ΠΆΠΈΡΠ΅Π»ΡΠ½ΠΎΡΡΠΈ.
ΠΡΠ»ΠΈ ΠΆΠ΅ ΠΈΠ·ΠΌΠ΅Π½Π΅Π½ΠΈΡ ΠΌΠ°Π³Π½ΠΈΡΠ½ΠΎΠ³ΠΎ ΠΏΠΎΡΠΎΠΊΠ° ΠΏΡΠΈΠ½ΠΈΠΌΠ°ΡΡ Π±ΠΎΠ»Π΅Π΅ ΡΠ»ΠΎΠΆΠ½ΡΡ ΡΠΎΡΠΌΡ, ΡΠΎ ΠΠΠ‘ ΠΈΠ½Π΄ΡΠΊΡΠΈΠΈ ΡΠΆΠ΅ Π½Π΅ Π±ΡΠ΄Π΅Ρ ΠΏΠΎΡΡΠΎΡΠ½Π½ΠΎΠΉ, Π° Π±ΡΠ΄Π΅Ρ Π·Π°Π²ΠΈΡΠ΅ΡΡ ΠΎΡ Π΄Π°Π½Π½ΠΎΠ³ΠΎ ΠΏΡΠΎΠΌΠ΅ΠΆΡΡΠΊΠ° Π²ΡΠ΅ΠΌΠ΅Π½ΠΈ. Π ΡΡΠΎΠΌ ΡΠ»ΡΡΠ°Π΅ Π²ΡΠ΅ΠΌΠ΅Π½Π½ΠΎΠΉ ΠΈΠ½ΡΠ΅ΡΠ²Π°Π» ΡΠ°ΡΡΠΌΠ°ΡΡΠΈΠ²Π°Π΅ΡΡΡ Π² ΠΊΠ°ΡΠ΅ΡΡΠ²Π΅ Π±Π΅ΡΠΊΠΎΠ½Π΅ΡΠ½ΠΎ ΠΌΠ°Π»ΠΎΠΉ Π²Π΅Π»ΠΈΡΠΈΠ½Ρ ΠΈ ΡΠΎΠ³Π΄Π° ΡΠΎΠΎΡΠ½ΠΎΡΠ΅Π½ΠΈΠ΅ βΠ€/βt Ρ ΡΠΎΡΠΊΠΈ Π·ΡΠ΅Π½ΠΈΡ ΠΌΠ°ΡΠ΅ΠΌΠ°ΡΠΈΠΊΠΈ ΡΡΠ°Π½Π΅Ρ ΠΏΡΠΎΠΈΠ·Π²ΠΎΠ΄Π½ΠΎΠΉ ΠΎΡ ΠΈΠ·ΠΌΠ΅Π½ΡΡΡΠ΅Π³ΠΎΡΡ ΠΌΠ°Π³Π½ΠΈΡΠ½ΠΎΠ³ΠΎ ΠΏΠΎΡΠΎΠΊΠ°.
Π‘ΡΡΠ΅ΡΡΠ²ΡΠ΅Ρ Π΅ΡΠ΅ ΠΎΠ΄ΠΈΠ½ Π²Π°ΡΠΈΠ°Π½Ρ, ΡΡΠ°ΠΊΡΡΡΡΠΈΠΉ Π·Π°ΠΊΠΎΠ½ ΡΠ»Π΅ΠΊΡΡΠΎΠΌΠ°Π³Π½ΠΈΡΠ½ΠΎΠΉ ΠΈΠ½Π΄ΡΠΊΡΠΈΠΈ Π€Π°ΡΠ°Π΄Π΅Ρ. ΠΠ³ΠΎ ΠΊΡΠ°ΡΠΊΠ°Ρ ΡΠΎΡΠΌΡΠ»ΠΈΡΠΎΠ²ΠΊΠ° ΠΎΠ±ΡΡΡΠ½ΡΠ΅Ρ, ΡΡΠΎ Π΄Π΅ΠΉΡΡΠ²ΠΈΠ΅ ΠΏΠ΅ΡΠ΅ΠΌΠ΅Π½Π½ΠΎΠ³ΠΎ ΠΌΠ°Π³Π½ΠΈΡΠ½ΠΎΠ³ΠΎ ΠΏΠΎΠ»Ρ Π²ΡΠ·ΡΠ²Π°Π΅Ρ ΠΏΠΎΡΠ²Π»Π΅Π½ΠΈΠ΅ Π²ΠΈΡ ΡΠ΅Π²ΠΎΠ³ΠΎ ΡΠ»Π΅ΠΊΡΡΠΈΡΠ΅ΡΠΊΠΎΠ³ΠΎ ΠΏΠΎΠ»Ρ.
ΠΡΠΎΡ ΠΆΠ΅ Π·Π°ΠΊΠΎΠ½ ΠΌΠΎΠΆΠ½ΠΎ ΡΡΠ°ΠΊΡΠΎΠ²Π°ΡΡ ΠΊΠ°ΠΊ ΠΎΠ΄Π½Ρ ΠΈΠ· Ρ Π°ΡΠ°ΠΊΡΠ΅ΡΠΈΡΡΠΈΠΊ ΡΠ»Π΅ΠΊΡΡΠΎΠΌΠ°Π³Π½ΠΈΡΠ½ΠΎΠ³ΠΎ ΠΏΠΎΠ»Ρ: Π²Π΅ΠΊΡΠΎΡ Π½Π°ΠΏΡΡΠΆΠ΅Π½Π½ΠΎΡΡΠΈ ΠΏΠΎΠ»Ρ ΠΌΠΎΠΆΠ΅Ρ ΡΠΈΡΠΊΡΠ»ΠΈΡΠΎΠ²Π°ΡΡ ΠΏΠΎ Π»ΡΠ±ΠΎΠΌΡ ΠΈΠ· ΠΊΠΎΠ½ΡΡΡΠΎΠ² ΡΠΎ ΡΠΊΠΎΡΠΎΡΡΡΡ, ΡΠ°Π²Π½ΠΎΠΉ ΡΠΊΠΎΡΠΎΡΡΠΈ ΠΈΠ·ΠΌΠ΅Π½Π΅Π½ΠΈΡ ΠΌΠ°Π³Π½ΠΈΡΠ½ΠΎΠ³ΠΎ ΠΏΠΎΡΠΎΠΊΠ°, ΠΏΡΠΎΡ ΠΎΠ΄ΡΡΠ΅Π³ΠΎ ΡΠ΅ΡΠ΅Π· ΡΠΎΡ ΠΈΠ»ΠΈ ΠΈΠ½ΠΎΠΉ ΠΊΠΎΠ½ΡΡΡ.
ΠΡΡΠΎΡΠ½ΠΈΠΊ: https://electric-220.ru/news/zakon_ehlektromagnitnoj_indukcii_faradeja/2018-09-29-1576
ΠΠ°Π³Π½ΠΈΡΠ½ΡΠΉ ΠΏΠΎΡΠΎΠΊ ΠΈ ΡΠ»Π΅ΠΊΡΡΠΎΠΌΠ°Π³Π½ΠΈΡΠ½Π°Ρ ΠΈΠ½Π΄ΡΠΊΡΠΈΡ: ΡΠΈΠ·ΠΈΡΠ΅ΡΠΊΠΈΠ΅ ΡΠΎΡΠΌΡΠ»Ρ
ΠΡΠ»ΠΈ ΠΏΡΠΎΠ²ΠΎΠ΄Π½ΠΈΠΊ Π·Π°ΠΌΠΊΠ½ΡΡ, ΡΠΎ Π΅ΡΡΡ ΡΠ²Π»ΡΠ΅ΡΡΡ ΠΊΠΎΠ½ΡΡΡΠΎΠΌ, ΡΠΎ Π² Π½Π΅ΠΌ ΠΏΠΎΡΠ²Π»ΡΠ΅ΡΡΡ ΡΠΎΠΊ ΠΈΠ½Π΄ΡΠΊΡΠΈΠΈ. Π―Π²Π»Π΅Π½ΠΈΠ΅ Π±ΡΠ»ΠΎ ΠΎΡΠΊΡΡΡΠΎ Π² 1831 Π³. Π. Π€Π°ΡΠ°Π΄Π΅Π΅ΠΌ.
ΠΡΠ½ΠΎΠ²Π½ΠΎΠΉ Π·Π°ΠΊΠΎΠ½ ΡΠ»Π΅ΠΊΡΡΠΎΠΌΠ°Π³Π½ΠΈΡΠ½ΠΎΠΉ ΠΈΠ½Π΄ΡΠΊΡΠΈΠΈ
ΠΡΠ½ΠΎΠ²Π½ΠΎΠΉ ΡΠΎΡΠΌΡΠ»ΠΎΠΉ, ΠΏΡΠΈ ΠΏΠΎΠΌΠΎΡΠΈ ΠΊΠΎΡΠΎΡΠΎΠΉ ΠΎΠΏΡΠ΅Π΄Π΅Π»ΡΡΡ ΠΠΠ‘ ΠΈΠ½Π΄ΡΠΊΡΠΈΠΈ (), ΡΠ²Π»ΡΠ΅ΡΡΡ Π·Π°ΠΊΠΎΠ½ Π€Π°ΡΠ°Π΄Π΅Ρ β ΠΠ°ΠΊΡΠ²Π΅Π»Π»Π°, Π±ΠΎΠ»ΡΡΠ΅ ΠΈΠ·Π²Π΅ΡΡΠ½ΡΠΉ ΠΊΠ°ΠΊ ΠΎΡΠ½ΠΎΠ²Π½ΠΎΠΉ Π·Π°ΠΊΠΎΠ½ ΡΠ»Π΅ΠΊΡΡΠΎΠΌΠ°Π³Π½ΠΈΡΠ½ΠΎΠΉ ΠΈΠ½Π΄ΡΠΊΡΠΈΠΈ (ΠΈΠ»ΠΈ Π·Π°ΠΊΠΎΠ½ Π€Π°ΡΠ°Π΄Π΅Ρ).
Π ΡΠΎΠΎΡΠ²Π΅ΡΡΡΠ²ΠΈΠΈ Ρ Π΄Π°Π½Π½ΡΠΌ Π·Π°ΠΊΠΎΠ½ΠΎΠΌ, ΡΠ»Π΅ΠΊΡΡΠΎΠ΄Π²ΠΈΠΆΡΡΠ°Ρ ΡΠΈΠ»Π° ΠΈΠ½Π΄ΡΠΊΡΠΈΠΈ Π² ΠΊΠΎΠ½ΡΡΡΠ΅, Π½Π°Ρ ΠΎΠ΄ΡΡΠ΅ΠΌΡΡ Π² ΠΏΠ΅ΡΠ΅ΠΌΠ΅Π½Π½ΠΎΠΌ ΠΌΠ°Π³Π½ΠΈΡΠ½ΠΎΠΌ ΠΏΠΎΠ»Π΅, ΡΠ°Π²Π½Π° ΠΏΠΎ ΠΌΠΎΠ΄ΡΠ»Ρ ΠΈ ΠΏΡΠΎΡΠΈΠ²ΠΎΠΏΠΎΠ»ΠΎΠΆΠ½Π° ΠΏΠΎ Π·Π½Π°ΠΊΡ ΡΠΊΠΎΡΠΎΡΡΠΈ ΠΈΠ·ΠΌΠ΅Π½Π΅Π½ΠΈΡ ΠΌΠ°Π³Π½ΠΈΡΠ½ΠΎΠ³ΠΎ ΠΏΠΎΡΠΎΠΊΠ° () ΡΠ΅ΡΠ΅Π· ΠΏΠΎΠ²Π΅ΡΡ Π½ΠΎΡΡΡ, ΠΊΠΎΡΠΎΡΡΡ ΠΎΠ³ΡΠ°Π½ΠΈΡΠΈΠ²Π°Π΅Ρ ΡΠ°ΡΡΠΌΠ°ΡΡΠΈΠ²Π°Π΅ΠΌΡΠΉ ΠΊΠΎΠ½ΡΡΡ:
Β Β
Π³Π΄Π΅ β ΡΠΊΠΎΡΠΎΡΡΡ ΠΈΠ·ΠΌΠ΅Π½Π΅Π½ΠΈΡ ΠΌΠ°Π³Π½ΠΈΡΠ½ΠΎΠ³ΠΎ ΠΏΠΎΡΠΎΠΊΠ°. ΠΠΎΠ»Π½Π°Ρ ΠΏΡΠΎΠΈΠ·Π²ΠΎΠ΄Π½Π°Ρ ΠΏΡΠΈΡΡΡΡΡΠ²ΡΡΡΠ°Ρ Π² ΡΠΎΡΠΌΡΠ»Π΅ (1) ΠΎΡ Π²Π°ΡΡΠ²Π°Π΅Ρ Π²Π΅ΡΡ ΡΠΏΠ΅ΠΊΡΡ ΠΏΡΠΈΡΠΈΠ½ ΠΈΠ·ΠΌΠ΅Π½Π΅Π½ΠΈΡ ΠΌΠ°Π³Π½ΠΈΡΠ½ΠΎΠ³ΠΎ ΠΏΠΎΡΠΎΠΊΠ° ΡΠ΅ΡΠ΅Π· ΠΏΠΎΠ²Π΅ΡΡ Π½ΠΎΡΡΡ ΠΊΠΎΠ½ΡΡΡΠ°. ΠΠ½Π°ΠΊ ΠΌΠΈΠ½ΡΡ Π² ΡΠΎΡΠΌΡΠ»Π΅ (1) ΠΎΡΠ²Π΅ΡΠ°Π΅Ρ ΠΏΡΠ°Π²ΠΈΠ»Ρ ΠΠ΅Π½ΡΠ°. Π Π²ΠΈΠ΄Π΅ (1) ΡΠΎΡΠΌΡΠ»Π° ΠΠΠ‘ Π·Π°ΠΏΠΈΡΠ°Π½Π° Π΄Π»Ρ ΠΌΠ΅ΠΆΠ΄ΡΠ½Π°ΡΠΎΠ΄Π½ΠΎΠΉ ΡΠΈΡΡΠ΅ΠΌΡ Π΅Π΄ΠΈΠ½ΠΈΡ (Π‘Π), Π² Π΄ΡΡΠ³ΠΈΡ ΡΠΈΡΡΠ΅ΠΌΠ°Ρ Π²ΠΈΠ΄ Π·Π°ΠΊΠΎΠ½Π° ΠΌΠΎΠΆΠ΅Ρ ΠΎΡΠ»ΠΈΡΠ°ΡΡΡΡ.
ΠΡΠΈ ΡΠ°Π²Π½ΠΎΠΌΠ΅ΡΠ½ΠΎΠΌ ΠΈΠ·ΠΌΠ΅Π½Π΅Π½ΠΈΠΈ ΠΌΠ°Π³Π½ΠΈΡΠ½ΠΎΠ³ΠΎ ΠΏΠΎΡΠΎΠΊΠ° ΠΎΡΠ½ΠΎΠ²Π½ΠΎΠΉ Π·Π°ΠΊΠΎΠ½ ΡΠ»Π΅ΠΊΡΡΠΎΠΌΠ°Π³Π½ΠΈΡΠ½ΠΎΠΉ ΠΈΠ½Π΄ΡΠΊΡΠΈΠΈ Π·Π°ΠΏΠΈΡΡΠ²Π°ΡΡ ΠΊΠ°ΠΊ:
Β Β
Π€ΠΎΡΠΌΡΠ»Ρ ΠΠΠ‘ ΠΈΠ½Π΄ΡΠΊΡΠΈΠΈ Π΄Π»Ρ ΡΠ°ΡΡΠ½ΡΡ ΡΠ»ΡΡΠ°Π΅Π²
- ΠΠΠ‘ ΠΈΠ½Π΄ΡΠΊΡΠΈΠΈ Π΄Π»Ρ ΠΊΠΎΠ½ΡΡΡΠ° ΠΈΠΌΠ΅ΡΡΠ΅Π³ΠΎ N Π²ΠΈΡΠΊΠΎΠ², Π½Π°Ρ ΠΎΠ΄ΡΡΠ΅Π³ΠΎΡΡ Π² ΠΏΠ΅ΡΠ΅ΠΌΠ΅Π½Π½ΠΎΠΌ ΠΌΠ°Π³Π½ΠΈΡΠ½ΠΎΠΌ ΠΏΠΎΠ»Π΅ ΠΌΠΎΠΆΠ½ΠΎ Π½Π°ΠΉΡΠΈ ΠΊΠ°ΠΊ:
- Β Β
- Π³Π΄Π΅ β ΠΏΠΎΡΠΎΠΊΠΎΡΡΠ΅ΠΏΠ»Π΅Π½ΠΈΠ΅.
- ΠΡΠ»ΠΈ ΠΏΡΡΠΌΠΎΠ»ΠΈΠ½Π΅ΠΉΠ½ΡΠΉ ΠΏΡΠΎΠ²ΠΎΠ΄Π½ΠΈΠΊ Π΄Π²ΠΈΠΆΠ΅ΡΡΡ Π² ΠΎΠ΄Π½ΠΎΡΠΎΠ΄Π½ΠΎΠΌ ΠΌΠ°Π³Π½ΠΈΡΠ½ΠΎΠΌ ΠΏΠΎΠ»Π΅, ΡΠΎ Π² Π½Π΅ΠΌ ΠΏΠΎΡΠ²Π»ΡΠ΅ΡΡΡ ΠΠΠ‘ ΠΈΠ½Π΄ΡΠΊΡΠΈΠΈ, ΡΠ°Π²Π½Π°Ρ:
- Β Β
- Π³Π΄Π΅ v β ΡΠΊΠΎΡΠΎΡΡΡ Π΄Π²ΠΈΠΆΠ΅Π½ΠΈΡ ΠΏΡΠΎΠ²ΠΎΠ΄Π½ΠΈΠΊΠ°; l β Π΄Π»ΠΈΠ½Π° ΠΏΡΠΎΠ²ΠΎΠ΄Π½ΠΈΠΊΠ°; B β ΠΌΠΎΠ΄ΡΠ»Ρ Π²Π΅ΠΊΡΠΎΡΠ° ΠΌΠ°Π³Π½ΠΈΡΠ½ΠΎΠΉ ΠΈΠ½Π΄ΡΠΊΡΠΈΠΈ ΠΏΠΎΠ»Ρ; .
- Π Π°Π·Π½ΠΎΡΡΡ ΠΏΠΎΡΠ΅Π½ΡΠΈΠ°Π»ΠΎΠ² (U) Π½Π° ΠΊΠΎΠ½ΡΠ°Ρ ΠΏΡΡΠΌΠΎΠ³ΠΎ ΠΏΡΠΎΠ²ΠΎΠ΄Π½ΠΈΠΊΠ°, Π΄Π²ΠΈΠΆΡΡΠ΅Π³ΠΎΡΡ Π² ΠΎΠ΄Π½ΠΎΡΠΎΠ΄Π½ΠΎΠΌ ΠΌΠ°Π³Π½ΠΈΡΠ½ΠΎΠΌ ΠΏΠΎΠ»Π΅ Ρ ΠΏΠΎΡΡΠΎΡΠ½Π½ΠΎΠΉ ΡΠΊΠΎΡΠΎΡΡΡΡ Π±ΡΠ΄Π΅Ρ ΡΠ°Π²Π½Π°:
- Β Β
- Π³Π΄Π΅ β ΡΠ³ΠΎΠ» ΠΌΠ΅ΠΆΠ΄Ρ Π½Π°ΠΏΡΠ°Π²Π»Π΅Π½ΠΈΡΠΌΠΈ Π²Π΅ΠΊΡΠΎΡΠΎΠ² ΠΈ .
- ΠΡΠΈ Π²ΡΠ°ΡΠ΅Π½ΠΈΠΈ ΠΏΠ»ΠΎΡΠΊΠΎΠ³ΠΎ ΠΊΠΎΠ½ΡΡΡΠ° Ρ ΠΏΠΎΡΡΠΎΡΠ½Π½ΠΎΠΉ ΡΠΊΠΎΡΠΎΡΡΡΡ Π² ΠΎΠ΄Π½ΠΎΡΠΎΠ΄Π½ΠΎΠΌ ΠΌΠ°Π³Π½ΠΈΡΠ½ΠΎΠΌ ΠΏΠΎΠ»Π΅ Π²ΠΎΠΊΡΡΠ³ ΠΎΡΠΈ, ΠΊΠΎΡΠΎΡΠ°Ρ Π»Π΅ΠΆΠΈΡ Π² ΠΏΠ»ΠΎΡΠΊΠΎΡΡΠΈ ΠΊΠΎΠ½ΡΡΡΠ° Π² Π½Π΅ΠΌ ΠΏΠΎΡΠ²Π»ΡΠ΅ΡΡΡ ΠΠΠ‘ ΠΈΠ½Π΄ΡΠΊΡΠΈΠΈ, ΠΊΠΎΡΠΎΡΡΡ ΠΌΠΎΠΆΠ½ΠΎ Π²ΡΡΠΈΡΠ»ΠΈΡΡ ΠΊΠ°ΠΊ:
- Β Β
Π³Π΄Π΅ S β ΠΏΠ»ΠΎΡΠ°Π΄Ρ, ΠΊΠΎΡΠΎΡΡΡ ΠΎΠ³ΡΠ°Π½ΠΈΡΠΈΠ²Π°Π΅Ρ Π²ΠΈΡΠΎΠΊ; β ΠΏΠΎΡΠΎΠΊ ΡΠ°ΠΌΠΎΠΈΠ½Π΄ΡΠΊΡΠΈΠΈ Π²ΠΈΡΠΊΠ°; β ΡΠ³Π»ΠΎΠ²Π°Ρ ΡΠΊΠΎΡΠΎΡΡΡ; () β ΡΠ³ΠΎΠ» ΠΏΠΎΠ²ΠΎΡΠΎΡΠ° ΠΊΠΎΠ½ΡΡΡΠ°. ΠΠ΅ΠΎΠ±Ρ ΠΎΠ΄ΠΈΠΌΠΎ Π·Π°ΠΌΠ΅ΡΠΈΡΡ, ΡΡΠΎ ΡΠΎΡΠΌΡΠ»Π° (5) ΠΏΡΠΈΠΌΠ΅Π½ΠΈΠΌΠ°, Π² ΡΠ»ΡΡΠ°Π΅, Π΅ΡΠ»ΠΈ ΠΎΡΡ Π²ΡΠ°ΡΠ΅Π½ΠΈΡ ΡΠΎΡΡΠ°Π²Π»ΡΠ΅Ρ ΠΏΡΡΠΌΠΎΠΉ ΡΠ³ΠΎΠ» Ρ Π½Π°ΠΏΡΠ°Π²Π»Π΅Π½ΠΈΠ΅ΠΌ Π²Π΅ΠΊΡΠΎΡΠ° Π²Π½Π΅ΡΠ½Π΅Π³ΠΎ ΠΌΠ°Π³Π½ΠΈΡΠ½ΠΎΠ³ΠΎ ΠΏΠΎΠ»Ρ .
- ΠΡΠ»ΠΈ Π²ΡΠ°ΡΠ°ΡΡΠ°ΡΡΡ ΡΠ°ΠΌΠΊΠ° ΠΎΠ±Π»Π°Π΄Π°Π΅Ρ N Π²ΠΈΡΠΊΠ°ΠΌΠΈ, ΠΏΡΠΈ ΡΡΠΎΠΌ ΡΠ°ΠΌΠΎΠΈΠ½Π΄ΡΠΊΡΠΈΠ΅ΠΉ ΡΠ°ΡΡΠΌΠ°ΡΡΠΈΠ²Π°Π΅ΠΌΠΎΠΉ ΡΠΈΡΡΠ΅ΠΌΡ ΠΌΠΎΠΆΠ½ΠΎ ΠΏΡΠ΅Π½Π΅Π±ΡΠ΅ΡΡ, ΡΠΎ:
- Β Β
- ΠΡΠ»ΠΈ ΠΏΡΠΎΠ²ΠΎΠ΄Π½ΠΈΠΊ Π½Π΅ΠΏΠΎΠ΄Π²ΠΈΠΆΠ΅Π½ Π² ΠΏΠ΅ΡΠ΅ΠΌΠ΅Π½Π½ΠΎΠΌ ΠΌΠ°Π³Π½ΠΈΡΠ½ΠΎΠΌ ΠΏΠΎΠ»Π΅, ΡΠΎ ΠΠΠ‘ ΠΈΠ½Π΄ΡΠΊΡΠΈΠΈ ΠΌΠΎΠΆΠ½ΠΎ Π½Π°ΠΉΡΠΈ ΠΊΠ°ΠΊ:
- Β Β
ΠΡΠΈΠΌΠ΅ΡΡ ΡΠ΅ΡΠ΅Π½ΠΈΡ Π·Π°Π΄Π°Ρ ΠΏΠΎ ΡΠ΅ΠΌΠ΅ Β«ΠΠ»Π΅ΠΊΡΡΠΎΠΌΠ°Π³Π½ΠΈΡΠ½Π°Ρ ΠΈΠ½Π΄ΡΠΊΡΠΈΡΒ»
ΠΠΎΠ½ΡΠ°Π²ΠΈΠ»ΡΡ ΡΠ°ΠΉΡ? Π Π°ΡΡΠΊΠ°ΠΆΠΈ Π΄ΡΡΠ·ΡΡΠΌ! |
ΠΡΡΠΎΡΠ½ΠΈΠΊ: http://ru.solverbook.com/spravochnik/formuly-po-fizike/formuly-elektromagnitnoj-indukcii/
ΠΠ»Π΅ΠΊΡΡΠΎΠΌΠ°Π³Π½ΠΈΡΠ½Π°Ρ ΠΈΠ½Π΄ΡΠΊΡΠΈΡ. ΠΠ°Π³Π½ΠΈΡΠ½ΡΠΉ ΠΏΠΎΡΠΎΠΊ β ΠΠ»Π°ΡΡ!Π½Π°Ρ ΡΠΈΠ·ΠΈΠΊΠ°
Β«Π€ΠΈΠ·ΠΈΠΊΠ° β 11 ΠΊΠ»Π°ΡΡΒ»
ΠΠ»Π΅ΠΊΡΡΠΎΠΌΠ°Π³Π½ΠΈΡΠ½Π°Ρ ΠΈΠ½Π΄ΡΠΊΡΠΈΡ
ΠΠ½Π³Π»ΠΈΠΉΡΠΊΠΈΠΉ ΡΠΈΠ·ΠΈΠΊ ΠΠ°ΠΉΠΊΠ» Π€Π°ΡΠ°Π΄Π΅ΠΉ Π±ΡΠ» ΡΠ²Π΅ΡΠ΅Π½ Π² Π΅Π΄ΠΈΠ½ΠΎΠΉ ΠΏΡΠΈΡΠΎΠ΄Π΅ ΡΠ»Π΅ΠΊΡΡΠΈΡΠ΅ΡΠΊΠΈΡ ΠΈ ΠΌΠ°Π³Π½ΠΈΡΠ½ΡΡ ΡΠ²Π»Π΅Π½ΠΈΠΉ. ΠΠ·ΠΌΠ΅Π½ΡΡΡΠ΅Π΅ΡΡ Π²ΠΎ Π²ΡΠ΅ΠΌΠ΅Π½ΠΈ ΠΌΠ°Π³Π½ΠΈΡΠ½ΠΎΠ΅ ΠΏΠΎΠ»Π΅ ΠΏΠΎΡΠΎΠΆΠ΄Π°Π΅Ρ ΡΠ»Π΅ΠΊΡΡΠΈΡΠ΅ΡΠΊΠΎΠ΅ ΠΏΠΎΠ»Π΅, Π° ΠΈΠ·ΠΌΠ΅Π½ΡΡΡΠ΅Π΅ΡΡ ΡΠ»Π΅ΠΊΡΡΠΈΡΠ΅ΡΠΊΠΎΠ΅ ΠΏΠΎΠ»Π΅ β ΠΌΠ°Π³Π½ΠΈΡΠ½ΠΎΠ΅.
- Π 1831 Π³ΠΎΠ΄Ρ Π€Π°ΡΠ°Π΄Π΅ΠΉ ΠΎΡΠΊΡΡΠ» ΡΠ²Π»Π΅Π½ΠΈΠ΅ ΡΠ»Π΅ΠΊΡΡΠΎΠΌΠ°Π³Π½ΠΈΡΠ½ΠΎΠΉ ΠΈΠ½Π΄ΡΠΊΡΠΈΠΈ, Π»Π΅Π³ΡΠ΅Π΅ Π² ΠΎΡΠ½ΠΎΠ²Ρ ΡΡΡΡΠΎΠΉΡΡΠ²Π° Π³Π΅Π½Π΅ΡΠ°ΡΠΎΡΠΎΠ², ΠΏΡΠ΅Π²ΡΠ°ΡΠ°ΡΡΠΈΡ ΠΌΠ΅Ρ Π°Π½ΠΈΡΠ΅ΡΠΊΡΡ ΡΠ½Π΅ΡΠ³ΠΈΡ Π² ΡΠ½Π΅ΡΠ³ΠΈΡ ΡΠ»Π΅ΠΊΡΡΠΈΡΠ΅ΡΠΊΠΎΠ³ΠΎ ΡΠΎΠΊΠ°.
- Π―Π²Π»Π΅Π½ΠΈΠ΅ ΡΠ»Π΅ΠΊΡΡΠΎΠΌΠ°Π³Π½ΠΈΡΠ½ΠΎΠΉ ΠΈΠ½Π΄ΡΠΊΡΠΈΠΈ
- Π―Π²Π»Π΅Π½ΠΈΠ΅ ΡΠ»Π΅ΠΊΡΡΠΎΠΌΠ°Π³Π½ΠΈΡΠ½ΠΎΠΉ ΠΈΠ½Π΄ΡΠΊΡΠΈΠΈ β ΡΡΠΎ Π²ΠΎΠ·Π½ΠΈΠΊΠ½ΠΎΠ²Π΅Π½ΠΈΠΈ ΡΠ»Π΅ΠΊΡΡΠΈΡΠ΅ΡΠΊΠΎΠ³ΠΎ ΡΠΎΠΊΠ° Π² ΠΏΡΠΎΠ²ΠΎΠ΄ΡΡΠ΅ΠΌ ΠΊΠΎΠ½ΡΡΡΠ΅, ΠΊΠΎΡΠΎΡΡΠΉ Π»ΠΈΠ±ΠΎ ΠΏΠΎΠΊΠΎΠΈΡΡΡ Π² ΠΏΠ΅ΡΠ΅ΠΌΠ΅Π½Π½ΠΎΠΌ Π²ΠΎ Π²ΡΠ΅ΠΌΠ΅Π½ΠΈ ΠΌΠ°Π³Π½ΠΈΡΠ½ΠΎΠΌ ΠΏΠΎΠ»Π΅, Π»ΠΈΠ±ΠΎ Π΄Π²ΠΈΠΆΠ΅ΡΡΡ Π² ΠΏΠΎΡΡΠΎΡΠ½Π½ΠΎΠΌ ΠΌΠ°Π³Π½ΠΈΡΠ½ΠΎΠΌ ΠΏΠΎΠ»Π΅ ΡΠ°ΠΊΠΈΠΌ ΠΎΠ±ΡΠ°Π·ΠΎΠΌ, ΡΡΠΎ ΡΠΈΡΠ»ΠΎ Π»ΠΈΠ½ΠΈΠΉ ΠΌΠ°Π³Π½ΠΈΡΠ½ΠΎΠΉ ΠΈΠ½Π΄ΡΠΊΡΠΈΠΈ, ΠΏΡΠΎΠ½ΠΈΠ·ΡΠ²Π°ΡΡΠΈΡ ΠΊΠΎΠ½ΡΡΡ, ΠΌΠ΅Π½ΡΠ΅ΡΡΡ.
- ΠΠ»Ρ ΡΠ²ΠΎΠΈΡ ΠΌΠ½ΠΎΠ³ΠΎΡΠΈΡΠ»Π΅Π½Π½ΡΡ ΠΎΠΏΡΡΠΎΠ² Π€Π°ΡΠ°Π΄Π΅ΠΉ ΠΈΡΠΏΠΎΠ»ΡΠ·ΠΎΠ²Π°Π» Π΄Π²Π΅ ΠΊΠ°ΡΡΡΠΊΠΈ, ΠΌΠ°Π³Π½ΠΈΡ, Π²ΡΠΊΠ»ΡΡΠ°ΡΠ΅Π»Ρ, ΠΈΡΡΠΎΡΠ½ΠΈΠΊ ΠΏΠΎΡΡΠΎΡΠ½Π½ΠΎΠ³ΠΎ ΡΠΎΠΊΠ° ΠΈ Π³Π°Π»ΡΠ²Π°Π½ΠΎΠΌΠ΅ΡΡ.
ΠΠ»Π΅ΠΊΡΡΠΈΡΠ΅ΡΠΊΠΈΠΉ ΡΠΎΠΊ ΡΠΏΠΎΡΠΎΠ±Π΅Π½ Π½Π°ΠΌΠ°Π³Π½ΠΈΡΠΈΡΡ ΠΊΡΡΠΎΠΊ ΠΆΠ΅Π»Π΅Π·Π°. ΠΠ΅ ΠΌΠΎΠΆΠ΅Ρ Π»ΠΈ ΠΌΠ°Π³Π½ΠΈΡ Π²ΡΠ·Π²Π°ΡΡ ΠΏΠΎΡΠ²Π»Π΅Π½ΠΈΠ΅ ΡΠ»Π΅ΠΊΡΡΠΈΡΠ΅ΡΠΊΠΎΠ³ΠΎ ΡΠΎΠΊΠ°?
Π ΡΠ΅Π·ΡΠ»ΡΡΠ°ΡΠ΅ ΠΎΠΏΡΡΠΎΠ² Π€Π°ΡΠ°Π΄Π΅ΠΉ ΡΡΡΠ°Π½ΠΎΠ²ΠΈΠ» Π³Π»Π°Π²Π½ΡΠ΅ ΠΎΡΠΎΠ±Π΅Π½Π½ΠΎΡΡΠΈ ΡΠ²Π»Π΅Π½ΠΈΡ ΡΠ»Π΅ΠΊΡΡΠΎΠΌΠ°Π³Π½ΠΈΡΠ½ΠΎΠΉ ΠΈΠ½Π΄ΡΠΊΡΠΈΠΈ:
1). ΠΈΠ½Π΄ΡΠΊΡΠΈΠΎΠ½Π½ΡΠΉ ΡΠΎΠΊ Π²ΠΎΠ·Π½ΠΈΠΊΠ°Π΅Ρ Π² ΠΎΠ΄Π½ΠΎΠΉ ΠΈΠ· ΠΊΠ°ΡΡΡΠ΅ΠΊ Π² ΠΌΠΎΠΌΠ΅Π½Ρ Π·Π°ΠΌΡΠΊΠ°Π½ΠΈΡ ΠΈΠ»ΠΈ ΡΠ°Π·ΠΌΡΠΊΠ°Π½ΠΈΡ ΡΠ»Π΅ΠΊΡΡΠΈΡΠ΅ΡΠΊΠΎΠΉ ΡΠ΅ΠΏΠΈ Π΄ΡΡΠ³ΠΎΠΉ ΠΊΠ°ΡΡΡΠΊΠΈ, Π½Π΅ΠΏΠΎΠ΄Π²ΠΈΠΆΠ½ΠΎΠΉ ΠΎΡΠ½ΠΎΡΠΈΡΠ΅Π»ΡΠ½ΠΎ ΠΏΠ΅ΡΠ²ΠΎΠΉ.
2). ΠΈΠ½Π΄ΡΠΊΡΠΈΠΎΠ½Π½ΡΠΉ ΡΠΎΠΊ Π²ΠΎΠ·Π½ΠΈΠΊΠ°Π΅Ρ ΠΏΡΠΈ ΠΈΠ·ΠΌΠ΅Π½Π΅Π½ΠΈΠΈ ΡΠΈΠ»Ρ ΡΠΎΠΊΠ° Π² ΠΎΠ΄Π½ΠΎΠΉ ΠΈΠ· ΠΊΠ°ΡΡΡΠ΅ΠΊ Ρ ΠΏΠΎΠΌΠΎΡΡΡ ΡΠ΅ΠΎΡΡΠ°ΡΠ° 3). ΠΈΠ½Π΄ΡΠΊΡΠΈΠΎΠ½Π½ΡΠΉ ΡΠΎΠΊ Π²ΠΎΠ·Π½ΠΈΠΊΠ°Π΅Ρ ΠΏΡΠΈ Π΄Π²ΠΈΠΆΠ΅Π½ΠΈΠΈ ΠΊΠ°ΡΡΡΠ΅ΠΊ ΠΎΡΠ½ΠΎΡΠΈΡΠ΅Π»ΡΠ½ΠΎ Π΄ΡΡΠ³ Π΄ΡΡΠ³Π° 4). ΠΈΠ½Π΄ΡΠΊΡΠΈΠΎΠ½Π½ΡΠΉ ΡΠΎΠΊ Π²ΠΎΠ·Π½ΠΈΠΊΠ°Π΅Ρ ΠΏΡΠΈ Π΄Π²ΠΈΠΆΠ΅Π½ΠΈΠΈ ΠΏΠΎΡΡΠΎΡΠ½Π½ΠΎΠ³ΠΎ ΠΌΠ°Π³Π½ΠΈΡΠ° ΠΎΡΠ½ΠΎΡΠΈΡΠ΅Π»ΡΠ½ΠΎ ΠΊΠ°ΡΡΡΠΊΠΈ
ΠΡΠ²ΠΎΠ΄:
Π Π·Π°ΠΌΠΊΠ½ΡΡΠΎΠΌ ΠΏΡΠΎΠ²ΠΎΠ΄ΡΡΠ΅ΠΌ ΠΊΠΎΠ½ΡΡΡΠ΅ Π²ΠΎΠ·Π½ΠΈΠΊΠ°Π΅Ρ ΡΠΎΠΊ ΠΏΡΠΈ ΠΈΠ·ΠΌΠ΅Π½Π΅Π½ΠΈΠΈ ΡΠΈΡΠ»Π° Π»ΠΈΠ½ΠΈΠΉ ΠΌΠ°Π³Π½ΠΈΡΠ½ΠΎΠΉ ΠΈΠ½Π΄ΡΠΊΡΠΈΠΈ, ΠΏΡΠΎΠ½ΠΈΠ·ΡΠ²Π°ΡΡΠΈΡ ΠΏΠΎΠ²Π΅ΡΡ Π½ΠΎΡΡΡ, ΠΎΠ³ΡΠ°Π½ΠΈΡΠ΅Π½Π½ΡΡ ΡΡΠΈΠΌ ΠΊΠΎΠ½ΡΡΡΠΎΠΌ. Π ΡΠ΅ΠΌ Π±ΡΡΡΡΠ΅Π΅ ΠΌΠ΅Π½ΡΠ΅ΡΡΡ ΡΠΈΡΠ»ΠΎ Π»ΠΈΠ½ΠΈΠΉ ΠΌΠ°Π³Π½ΠΈΡΠ½ΠΎΠΉ ΠΈΠ½Π΄ΡΠΊΡΠΈΠΈ, ΡΠ΅ΠΌ Π±ΠΎΠ»ΡΡΠ΅ Π²ΠΎΠ·Π½ΠΈΠΊΠ°ΡΡΠΈΠΉ ΠΈΠ½Π΄ΡΠΊΡΠΈΠΎΠ½Π½ΡΠΉ ΡΠΎΠΊ.
ΠΡΠΈ ΡΡΠΎΠΌ Π½Π΅ Π²Π°ΠΆΠ½ΠΎ. ΡΡΠΎ ΡΠ²Π»ΡΠ΅ΡΡΡ ΠΏΡΠΈΡΠΈΠ½ΠΎΠΉ ΠΈΠ·ΠΌΠ΅Π½Π΅Π½ΠΈΡ ΡΠΈΡΠ»Π° Π»ΠΈΠ½ΠΈΠΉ ΠΌΠ°Π³Π½ΠΈΡΠ½ΠΎΠΉ ΠΈΠ½Π΄ΡΠΊΡΠΈΠΈ. ΠΡΠΎ ΠΌΠΎΠΆΠ΅Ρ Π±ΡΡΡ ΠΈ ΠΈΠ·ΠΌΠ΅Π½Π΅Π½ΠΈΠ΅ ΡΠΈΡΠ»Π° Π»ΠΈΠ½ΠΈΠΉ ΠΌΠ°Π³Π½ΠΈΡΠ½ΠΎΠΉ ΠΈΠ½Π΄ΡΠΊΡΠΈΠΈ, ΠΏΡΠΎΠ½ΠΈΠ·ΡΠ²Π°ΡΡΠΈΡ ΠΏΠΎΠ²Π΅ΡΡ Π½ΠΎΡΡΡ, ΠΎΠ³ΡΠ°Π½ΠΈΡΠ΅Π½Π½ΡΡ Π½Π΅ΠΏΠΎΠ΄Π²ΠΈΠΆΠ½ΡΠΌ ΠΏΡΠΎΠ²ΠΎΠ΄ΡΡΠΈΠΌ ΠΊΠΎΠ½ΡΡΡΠΎΠΌ, Π²ΡΠ»Π΅Π΄ΡΡΠ²ΠΈΠ΅ ΠΈΠ·ΠΌΠ΅Π½Π΅Π½ΠΈΡ ΡΠΈΠ»Ρ ΡΠΎΠΊΠ° Π² ΡΠΎΡΠ΅Π΄Π½Π΅ΠΉ ΠΊΠ°ΡΡΡΠΊΠ΅,
ΠΈ ΠΈΠ·ΠΌΠ΅Π½Π΅Π½ΠΈΠ΅ ΡΠΈΡΠ»Π° Π»ΠΈΠ½ΠΈΠΉ ΠΈΠ½Π΄ΡΠΊΡΠΈΠΈ Π²ΡΠ»Π΅Π΄ΡΡΠ²ΠΈΠ΅ Π΄Π²ΠΈΠΆΠ΅Π½ΠΈΡ ΠΊΠΎΠ½ΡΡΡΠ° Π² Π½Π΅ΠΎΠ΄Π½ΠΎΡΠΎΠ΄Π½ΠΎΠΌ ΠΌΠ°Π³Π½ΠΈΡΠ½ΠΎΠΌ ΠΏΠΎΠ»Π΅, Π³ΡΡΡΠΎΡΠ° Π»ΠΈΠ½ΠΈΠΉ ΠΊΠΎΡΠΎΡΠΎΠ³ΠΎ ΠΌΠ΅Π½ΡΠ΅ΡΡΡ Π² ΠΏΡΠΎΡΡΡΠ°Π½ΡΡΠ²Π΅, ΠΈ Ρ.Π΄.
ΠΠ°Π³Π½ΠΈΡΠ½ΡΠΉ ΠΏΠΎΡΠΎΠΊ β ΡΡΠΎ Ρ Π°ΡΠ°ΠΊΡΠ΅ΡΠΈΡΡΠΈΠΊΠ° ΠΌΠ°Π³Π½ΠΈΡΠ½ΠΎΠ³ΠΎ ΠΏΠΎΠ»Ρ, ΠΊΠΎΡΠΎΡΠ°Ρ Π·Π°Π²ΠΈΡΠΈΡ ΠΎΡ Π²Π΅ΠΊΡΠΎΡΠ° ΠΌΠ°Π³Π½ΠΈΡΠ½ΠΎΠΉ ΠΈΠ½Π΄ΡΠΊΡΠΈΠΈ Π²ΠΎ Π²ΡΠ΅Ρ ΡΠΎΡΠΊΠ°Ρ ΠΏΠΎΠ²Π΅ΡΡ Π½ΠΎΡΡΠΈ, ΠΎΠ³ΡΠ°Π½ΠΈΡΠ΅Π½Π½ΠΎΠΉ ΠΏΠ»ΠΎΡΠΊΠΈΠΌ Π·Π°ΠΌΠΊΠ½ΡΡΡΠΌ ΠΊΠΎΠ½ΡΡΡΠΎΠΌ.
ΠΡΡΡ ΠΏΠ»ΠΎΡΠΊΠΈΠΉ Π·Π°ΠΌΠΊΠ½ΡΡΡΠΉ ΠΏΡΠΎΠ²ΠΎΠ΄Π½ΠΈΠΊ (ΠΊΠΎΠ½ΡΡΡ), ΠΎΠ³ΡΠ°Π½ΠΈΡΠΈΠ²Π°ΡΡΠΈΠΉ ΠΏΠΎΠ²Π΅ΡΡ Π½ΠΎΡΡΡ ΠΏΠ»ΠΎΡΠ°Π΄ΡΡ S ΠΈ ΠΏΠΎΠΌΠ΅ΡΠ΅Π½Π½ΡΠΉ Π² ΠΎΠ΄Π½ΠΎΡΠΎΠ΄Π½ΠΎΠ΅ ΠΌΠ°Π³Π½ΠΈΡΠ½ΠΎΠ΅ ΠΏΠΎΠ»Π΅. ΠΠΎΡΠΌΠ°Π»Ρ (Π²Π΅ΠΊΡΠΎΡ, ΠΌΠΎΠ΄ΡΠ»Ρ ΠΊΠΎΡΠΎΡΠΎΠ³ΠΎ ΡΠ°Π²Π΅Π½ Π΅Π΄ΠΈΠ½ΠΈΡΠ΅) ΠΊ ΠΏΠ»ΠΎΡΠΊΠΎΡΡΠΈ ΠΏΡΠΎΠ²ΠΎΠ΄Π½ΠΈΠΊΠ° ΡΠΎΡΡΠ°Π²Π»ΡΠ΅Ρ ΡΠ³ΠΎΠ» Ξ± Ρ Π½Π°ΠΏΡΠ°Π²Π»Π΅Π½ΠΈΠ΅ΠΌ Π²Π΅ΠΊΡΠΎΡΠ° ΠΌΠ°Π³Π½ΠΈΡΠ½ΠΎΠΉ ΠΈΠ½Π΄ΡΠΊΡΠΈΠΈ .
- ΠΠ°Π³Π½ΠΈΡΠ½ΡΠΌ ΠΏΠΎΡΠΎΠΊΠΎΠΌ Π€ (ΠΏΠΎΡΠΎΠΊΠΎΠΌ Π²Π΅ΠΊΡΠΎΡΠ° ΠΌΠ°Π³Π½ΠΈΡΠ½ΠΎΠΉ ΠΈΠ½Π΄ΡΠΊΡΠΈΠΈ) ΡΠ΅ΡΠ΅Π· ΠΏΠΎΠ²Π΅ΡΡ Π½ΠΎΡΡΡ ΠΏΠ»ΠΎΡΠ°Π΄ΡΡ S Π½Π°Π·ΡΠ²Π°ΡΡ Π²Π΅Π»ΠΈΡΠΈΠ½Ρ, ΡΠ°Π²Π½ΡΡ ΠΏΡΠΎΠΈΠ·Π²Π΅Π΄Π΅Π½ΠΈΡ ΠΌΠΎΠ΄ΡΠ»Ρ Π²Π΅ΠΊΡΠΎΡΠ° ΠΌΠ°Π³Π½ΠΈΡΠ½ΠΎΠΉ ΠΈΠ½Π΄ΡΠΊΡΠΈΠΈ Π½Π° ΠΏΠ»ΠΎΡΠ°Π΄Ρ S ΠΈ ΠΊΠΎΡΠΈΠ½ΡΡ ΡΠ³Π»Π° Ξ± ΠΌΠ΅ΠΆΠ΄Ρ Π²Π΅ΠΊΡΠΎΡΠ°ΠΌΠΈ ΠΈ :
- Π€ = BScos Ξ±
- Π³Π΄Π΅ Πcos Ξ± = Πn β ΠΏΡΠΎΠ΅ΠΊΡΠΈΡ Π²Π΅ΠΊΡΠΎΡΠ° ΠΌΠ°Π³Π½ΠΈΡΠ½ΠΎΠΉ ΠΈΠ½Π΄ΡΠΊΡΠΈΠΈ Π½Π° Π½ΠΎΡΠΌΠ°Π»Ρ ΠΊ ΠΏΠ»ΠΎΡΠΊΠΎΡΡΠΈ ΠΊΠΎΠ½ΡΡΡΠ°. ΠΠΎΡΡΠΎΠΌΡ
- Π€ = BnS
- ΠΠ°Π³Π½ΠΈΡΠ½ΡΠΉ ΠΏΠΎΡΠΎΠΊ ΡΠ΅ΠΌ Π±ΠΎΠ»ΡΡΠ΅, ΡΠ΅ΠΌ Π±ΠΎΠ»ΡΡΠ΅ Πn ΠΈ S.
- ΠΠ°Π³Π½ΠΈΡΠ½ΡΠΉ ΠΏΠΎΡΠΎΠΊ Π·Π°Π²ΠΈΡΠΈΡ ΠΎΡ ΠΎΡΠΈΠ΅Π½ΡΠ°ΡΠΈΠΈ ΠΏΠΎΠ²Π΅ΡΡ Π½ΠΎΡΡΠΈ, ΠΊΠΎΡΠΎΡΡΡ ΠΏΡΠΎΠ½ΠΈΠ·ΡΠ²Π°Π΅Ρ ΠΌΠ°Π³Π½ΠΈΡΠ½ΠΎΠ΅ ΠΏΠΎΠ»Π΅.
- ΠΠ°Π³Π½ΠΈΡΠ½ΡΠΉ ΠΏΠΎΡΠΎΠΊ Π³ΡΠ°ΡΠΈΡΠ΅ΡΠΊΠΈ ΠΌΠΎΠΆΠ½ΠΎ ΠΈΡΡΠΎΠ»ΠΊΠΎΠ²Π°ΡΡ ΠΊΠ°ΠΊ Π²Π΅Π»ΠΈΡΠΈΠ½Ρ, ΠΏΡΠΎΠΏΠΎΡΡΠΈΠΎΠ½Π°Π»ΡΠ½ΡΡ ΡΠΈΡΠ»Ρ Π»ΠΈΠ½ΠΈΠΉ ΠΌΠ°Π³Π½ΠΈΡΠ½ΠΎΠΉ ΠΈΠ½Π΄ΡΠΊΡΠΈΠΈ, ΠΏΡΠΎΠ½ΠΈΠ·ΡΠ²Π°ΡΡΠΈΡ ΠΏΠΎΠ²Π΅ΡΡ Π½ΠΎΡΡΡ ΠΏΠ»ΠΎΡΠ°Π΄ΡΡ S.
ΠΠ΄ΠΈΠ½ΠΈΡΠ΅ΠΉ ΠΌΠ°Π³Π½ΠΈΡΠ½ΠΎΠ³ΠΎ ΠΏΠΎΡΠΎΠΊΠ° ΡΠ²Π»ΡΠ΅ΡΡΡ Π²Π΅Π±Π΅Ρ. ΠΠ°Π³Π½ΠΈΡΠ½ΡΠΉ ΠΏΠΎΡΠΎΠΊ Π² 1 Π²Π΅Π±Π΅Ρ (1 ΠΠ±) ΡΠΎΠ·Π΄Π°Π΅ΡΡΡ ΠΎΠ΄Π½ΠΎΡΠΎΠ΄Π½ΡΠΌ ΠΌΠ°Π³Π½ΠΈΡΠ½ΡΠΌ ΠΏΠΎΠ»Π΅ΠΌ Ρ ΠΈΠ½Π΄ΡΠΊΡΠΈΠ΅ΠΉ 1 Π’Π» ΡΠ΅ΡΠ΅Π· ΠΏΠΎΠ²Π΅ΡΡ Π½ΠΎΡΡΡ ΠΏΠ»ΠΎΡΠ°Π΄ΡΡ 1 ΠΌ2, ΡΠ°ΡΠΏΠΎΠ»ΠΎΠΆΠ΅Π½Π½ΡΡ ΠΏΠ΅ΡΠΏΠ΅Π½Π΄ΠΈΠΊΡΠ»ΡΡΠ½ΠΎ Π²Π΅ΠΊΡΠΎΡΡ ΠΌΠ°Π³Π½ΠΈΡΠ½ΠΎΠΉ ΠΈΠ½Π΄ΡΠΊΡΠΈΠΈ.
ΠΡΡΠΎΡΠ½ΠΈΠΊ: Β«Π€ΠΈΠ·ΠΈΠΊΠ° β 11 ΠΊΠ»Π°ΡΡΒ», ΡΡΠ΅Π±Π½ΠΈΠΊ ΠΡΠΊΠΈΡΠ΅Π², ΠΡΡ ΠΎΠ²ΡΠ΅Π², Π§Π°ΡΡΠ³ΠΈΠ½
Π‘Π»Π΅Π΄ΡΡΡΠ°Ρ ΡΡΡΠ°Π½ΠΈΡΠ° Β«ΠΠ°ΠΏΡΠ°Π²Π»Π΅Π½ΠΈΠ΅ ΠΈΠ½Π΄ΡΠΊΡΠΈΠΎΠ½Π½ΠΎΠ³ΠΎ ΡΠΎΠΊΠ°. ΠΡΠ°Π²ΠΈΠ»ΠΎ ΠΠ΅Π½ΡΠ°Β» ΠΠ°Π·Π°Π΄ Π² ΡΠ°Π·Π΄Π΅Π» Β«Π€ΠΈΠ·ΠΈΠΊΠ° β 11 ΠΊΠ»Π°ΡΡ, ΡΡΠ΅Π±Π½ΠΈΠΊ ΠΡΠΊΠΈΡΠ΅Π², ΠΡΡ ΠΎΠ²ΡΠ΅Π², Π§Π°ΡΡΠ³ΠΈΠ½Β»
ΠΠ»Π΅ΠΊΡΡΠΎΠΌΠ°Π³Π½ΠΈΡΠ½Π°Ρ ΠΈΠ½Π΄ΡΠΊΡΠΈΡ. Π€ΠΈΠ·ΠΈΠΊΠ°, ΡΡΠ΅Π±Π½ΠΈΠΊ Π΄Π»Ρ 11 ΠΊΠ»Π°ΡΡΠ° β ΠΠ»Π°ΡΡ!Π½Π°Ρ ΡΠΈΠ·ΠΈΠΊΠ°
ΠΠ»Π΅ΠΊΡΡΠΎΠΌΠ°Π³Π½ΠΈΡΠ½Π°Ρ ΠΈΠ½Π΄ΡΠΊΡΠΈΡ. ΠΠ°Π³Π½ΠΈΡΠ½ΡΠΉ ΠΏΠΎΡΠΎΠΊ β ΠΠ°ΠΏΡΠ°Π²Π»Π΅Π½ΠΈΠ΅ ΠΈΠ½Π΄ΡΠΊΡΠΈΠΎΠ½Π½ΠΎΠ³ΠΎ ΡΠΎΠΊΠ°. ΠΡΠ°Π²ΠΈΠ»ΠΎ ΠΠ΅Π½ΡΠ° β ΠΠ°ΠΊΠΎΠ½ ΡΠ»Π΅ΠΊΡΡΠΎΠΌΠ°Π³Π½ΠΈΡΠ½ΠΎΠΉ ΠΈΠ½Π΄ΡΠΊΡΠΈΠΈ β ΠΠΠ‘ ΠΈΠ½Π΄ΡΠΊΡΠΈΠΈ Π² Π΄Π²ΠΈΠΆΡΡΠΈΡ ΡΡ ΠΏΡΠΎΠ²ΠΎΠ΄Π½ΠΈΠΊΠ°Ρ . ΠΠ»Π΅ΠΊΡΡΠΎΠ΄ΠΈΠ½Π°ΠΌΠΈΡΠ΅ΡΠΊΠΈΠΉ ΠΌΠΈΠΊΡΠΎΡΠΎΠ½ β ΠΠΈΡ ΡΠ΅Π²ΠΎΠ΅ ΡΠ»Π΅ΠΊΡΡΠΈΡΠ΅ΡΠΊΠΎΠ΅ ΠΏΠΎΠ»Π΅ β Π‘Π°ΠΌΠΎΠΈΠ½Π΄ΡΠΊΡΠΈΡ. ΠΠ½Π΄ΡΠΊΡΠΈΠ²Π½ΠΎΡΡΡ. ΠΠ½Π΅ΡΠ³ΠΈΡ ΠΌΠ°Π³Π½ΠΈΡΠ½ΠΎΠ³ΠΎ ΠΏΠΎΠ»Ρ ΡΠΎΠΊΠ° β ΠΠ»Π΅ΠΊΡΡΠΎΠΌΠ°Π³Π½ΠΈΡΠ½ΠΎΠ΅ ΠΏΠΎΠ»Π΅ β ΠΡΠΈΠΌΠ΅ΡΡ ΡΠ΅ΡΠ΅Π½ΠΈΡ Π·Π°Π΄Π°Ρ β ΠΡΠ°ΡΠΊΠΈΠ΅ ΠΈΡΠΎΠ³ΠΈ Π³Π»Π°Π²Ρ
ΠΡΡΠΎΡΠ½ΠΈΠΊ: http://class-fizika.ru/11_7. html
ΠΠ°Π³Π½ΠΈΡΠ½ΡΠΉ ΠΏΠΎΡΠΎΠΊ (ΠΡΡΡΠΊΠΈΠ½ Π.Π‘.). ΠΠΈΠ΄Π΅ΠΎΡΡΠΎΠΊ. Π€ΠΈΠ·ΠΈΠΊΠ° 9 ΠΠ»Π°ΡΡ
ΠΡΠΎΠ΄ΠΎΠ»ΠΆΠ°Ρ ΠΈΠ·ΡΡΠ΅Π½ΠΈΠ΅ ΡΠ΅ΠΌΡ Β«ΠΠ»Π΅ΠΊΡΡΠΎΠΌΠ°Π³Π½ΠΈΡΠ½Π°Ρ ΠΈΠ½Π΄ΡΠΊΡΠΈΡΒ» Π΄Π°Π²Π°ΠΉΡΠ΅ Β ΠΏΠΎΠ΄ΡΠΎΠ±Π½Π΅Π΅ ΠΎΡΡΠ°Π½ΠΎΠ²ΠΈΡΡΡΡ Π½Π° ΡΠ°ΠΊΠΎΠΌ ΠΏΠΎΠ½ΡΡΠΈΠΈ, ΠΊΠ°ΠΊ ΠΌΠ°Π³Π½ΠΈΡΠ½ΡΠΉ ΠΏΠΎΡΠΎΠΊ.
ΠΡ ΡΠΆΠ΅ Π·Π½Π°Π΅ΡΠ΅, ΠΊΠ°ΠΊ ΠΎΠ±Π½Π°ΡΡΠΆΠΈΡΡ ΡΠ²Π»Π΅Π½ΠΈΠ΅ ΡΠ»Π΅ΠΊΡΡΠΎΠΌΠ°Π³Π½ΠΈΡΠ½ΠΎΠΉ ΠΈΠ½Π΄ΡΠΊΡΠΈΠΈ β Π΅ΡΠ»ΠΈ Π·Π°ΠΌΠΊΠ½ΡΡΡΠΉ ΠΏΡΠΎΠ²ΠΎΠ΄Π½ΠΈΠΊ ΠΏΠ΅ΡΠ΅ΡΠ΅ΠΊΠ°ΡΡ ΠΌΠ°Π³Π½ΠΈΡΠ½ΡΠ΅ Π»ΠΈΠ½ΠΈΠΈ, Π² ΡΡΠΎΠΌ ΠΏΡΠΎΠ²ΠΎΠ΄Π½ΠΈΠΊΠ΅ Π²ΠΎΠ·Π½ΠΈΠΊΠ°Π΅Ρ ΡΠ»Π΅ΠΊΡΡΠΈΡΠ΅ΡΠΊΠΈΠΉ ΡΠΎΠΊ. Π’Π°ΠΊΠΎΠΉ ΡΠΎΠΊ Π½Π°Π·ΡΠ²Π°Π΅ΡΡΡ ΠΈΠ½Π΄ΡΠΊΡΠΈΠΎΠ½Π½ΡΠΌ.
- Π’Π΅ΠΏΠ΅ΡΡ Π΄Π°Π²Π°ΠΉΡΠ΅ ΠΎΠ±ΡΡΠ΄ΠΈΠΌ, Π·Π° ΡΡΠ΅Ρ ΡΠ΅Π³ΠΎ ΠΎΠ±ΡΠ°Π·ΡΠ΅ΡΡΡ ΡΡΠΎΡ ΡΠ»Π΅ΠΊΡΡΠΈΡΠ΅ΡΠΊΠΈΠΉ ΡΠΎΠΊ ΠΈ ΡΡΠΎ ΡΠ²Π»ΡΠ΅ΡΡΡ Π³Π»Π°Π²Π½ΡΠΌ Π΄Π»Ρ ΡΠΎΠ³ΠΎ, ΡΡΠΎΠ±Ρ ΡΡΠΎΡ ΡΠΎΠΊ ΠΏΠΎΡΠ²ΠΈΠ»ΡΡ.
- ΠΡΠ΅ΠΆΠ΄Π΅ Π²ΡΠ΅Π³ΠΎ, ΠΎΠ±ΡΠ°ΡΠΈΠΌΡΡ ΠΊ ΠΎΠΏΡΡΡ Π€Π°ΡΠ°Π΄Π΅Ρ ΠΈ ΠΏΠΎΡΠΌΠΎΡΡΠΈΠΌ Π΅ΡΠ΅ ΡΠ°Π· Π½Π° Π΅Π³ΠΎ Π²Π°ΠΆΠ½ΡΠ΅ ΠΎΡΠΎΠ±Π΅Π½Π½ΠΎΡΡΠΈ.
- ΠΡΠ°ΠΊ, Ρ Π½Π°Ρ Π² Π½Π°Π»ΠΈΡΠΈΠΈ Π΅ΡΡΡ Π°ΠΌΠΏΠ΅ΡΠΌΠ΅ΡΡ, ΠΊΠ°ΡΡΡΠΊΠ° Ρ Π±ΠΎΠ»ΡΡΠΈΠΌ ΡΠΈΡΠ»ΠΎΠΌ Π²ΠΈΡΠΊΠΎΠ², ΠΊΠΎΡΠΎΡΠ°Ρ Π½Π°ΠΊΠΎΡΠΎΡΠΊΠΎ ΠΏΡΠΈΠΊΡΠ΅ΠΏΠ»Π΅Π½Π° ΠΊ ΡΡΠΎΠΌΡ Π°ΠΌΠΏΠ΅ΡΠΌΠ΅ΡΡΡ.
ΠΠ΅ΡΠ΅ΠΌ ΠΌΠ°Π³Π½ΠΈΡ, ΠΈ ΡΠΎΡΠ½ΠΎ ΡΠ°ΠΊ ΠΆΠ΅, ΠΊΠ°ΠΊ Π½Π° ΠΏΡΠ΅Π΄ΡΠ΄ΡΡΠ΅ΠΌ ΡΡΠΎΠΊΠ΅, ΠΎΠΏΡΡΠΊΠ°Π΅ΠΌ ΡΡΠΎΡ ΠΌΠ°Π³Π½ΠΈΡ Π²Π½ΡΡΡΡ ΠΊΠ°ΡΡΡΠΊΠΈ. Π‘ΡΡΠ΅Π»ΠΊΠ° ΠΎΡΠΊΠ»ΠΎΠ½ΡΠ΅ΡΡΡ, ΡΠΎ Π΅ΡΡΡ Π² Π΄Π°Π½Π½ΠΎΠΉ ΡΠ΅ΠΏΠΈ ΡΡΡΠ΅ΡΡΠ²ΡΠ΅Ρ ΡΠ»Π΅ΠΊΡΡΠΈΡΠ΅ΡΠΊΠΈΠΉ ΡΠΎΠΊ.
Π ΠΈΡ. 1. ΠΠΏΡΡ ΠΏΠΎ ΠΎΠ±Π½Π°ΡΡΠΆΠ΅Π½ΠΈΡ ΠΈΠ½Π΄ΡΠΊΡΠΈΠΎΠ½Π½ΠΎΠ³ΠΎ ΡΠΎΠΊΠ°
Π Π²ΠΎΡ ΠΊΠΎΠ³Π΄Π° ΠΌΠ°Π³Π½ΠΈΡ Π½Π°Ρ ΠΎΠ΄ΠΈΡΡΡ Π²Π½ΡΡΡΠΈ ΠΊΠ°ΡΡΡΠΊΠΈ ΡΠ»Π΅ΠΊΡΡΠΈΡΠ΅ΡΠΊΠΎΠ³ΠΎ ΡΠΎΠΊΠ° Π² ΡΠ΅ΠΏΠΈ Π½Π΅Ρ. ΠΠΎ ΡΡΠΎΠΈΡ ΡΠΎΠ»ΡΠΊΠΎ ΠΏΠΎΠΏΡΡΠ°ΡΡΡΡ ΡΡΠΎΡ ΠΌΠ°Π³Π½ΠΈΡ Π΄ΠΎΡΡΠ°ΡΡ ΠΈΠ· ΠΊΠ°ΡΡΡΠΊΠΈ, ΠΊΠ°ΠΊ Π² ΡΠ΅ΠΏΠΈ Π²Π½ΠΎΠ²Ρ ΠΏΠΎΡΠ²Π»ΡΠ΅ΡΡΡ ΡΠ»Π΅ΠΊΡΡΠΈΡΠ΅ΡΠΊΠΈΠΉ ΡΠΎΠΊ, Π½ΠΎ Π½Π°ΠΏΡΠ°Π²Π»Π΅Π½ΠΈΠ΅ ΡΡΠΎΠ³ΠΎ ΡΠΎΠΊΠ° ΠΈΠ·ΠΌΠ΅Π½ΡΠ΅ΡΡΡ Π½Π° ΠΏΡΠΎΡΠΈΠ²ΠΎΠΏΠΎΠ»ΠΎΠΆΠ½ΠΎΠ΅.
ΠΠ±ΡΠ°ΡΠΈΡΠ΅ Π²Π½ΠΈΠΌΠ°Π½ΠΈΠ΅ ΡΠ°ΠΊΠΆΠ΅ Π½Π° ΡΠΎ, ΡΡΠΎ Π·Π½Π°ΡΠ΅Π½ΠΈΠ΅ ΡΠ»Π΅ΠΊΡΡΠΈΡΠ΅ΡΠΊΠΎΠ³ΠΎ ΡΠΎΠΊΠ°, ΠΊΠΎΡΠΎΡΡΠΉ ΠΏΡΠΎΡΠ΅ΠΊΠ°Π΅Ρ Π² ΡΠ΅ΠΏΠΈ, Π·Π°Π²ΠΈΡΠΈΡ Π΅ΡΠ΅ ΠΈ ΠΎΡ ΡΠ²ΠΎΠΉΡΡΠ² ΡΠ°ΠΌΠΎΠ³ΠΎ ΠΌΠ°Π³Π½ΠΈΡΠ°. ΠΡΠ»ΠΈ Π²Π·ΡΡΡ Π΄ΡΡΠ³ΠΎΠΉ ΠΌΠ°Π³Π½ΠΈΡ ΠΈ ΠΏΡΠΎΠ΄Π΅Π»Π°ΡΡ ΡΠΎΡ ΠΆΠ΅ ΡΠΊΡΠΏΠ΅ΡΠΈΠΌΠ΅Π½Ρ, Π·Π½Π°ΡΠ΅Π½ΠΈΠ΅ ΡΠΎΠΊΠ° ΡΡΡΠ΅ΡΡΠ²Π΅Π½Π½ΠΎ ΠΌΠ΅Π½ΡΠ΅ΡΡΡ, Π² Π΄Π°Π½Π½ΠΎΠΌ ΡΠ»ΡΡΠ°Π΅ ΡΠΎΠΊ ΡΡΠ°Π½ΠΎΠ²ΠΈΡΡΡ ΠΌΠ΅Π½ΡΡΠ΅.
ΠΡΠΎΠ²Π΅Π΄Ρ ΡΠΊΡΠΏΠ΅ΡΠΈΠΌΠ΅Π½ΡΡ, ΠΌΠΎΠΆΠ½ΠΎ ΡΠ΄Π΅Π»Π°ΡΡ Π²ΡΠ²ΠΎΠ΄ ΠΎ ΡΠΎΠΌ, ΡΡΠΎ ΡΠ»Π΅ΠΊΡΡΠΈΡΠ΅ΡΠΊΠΈΠΉ ΡΠΎΠΊ, ΠΊΠΎΡΠΎΡΡΠΉ Π²ΠΎΠ·Π½ΠΈΠΊΠ°Π΅Ρ Π² Π·Π°ΠΌΠΊΠ½ΡΡΠΎΠΌ ΠΏΡΠΎΠ²ΠΎΠ΄Π½ΠΈΠΊΠ΅ (Π² ΠΊΠ°ΡΡΡΠΊΠ΅), ΡΠ²ΡΠ·Π°Π½ Ρ ΠΌΠ°Π³Π½ΠΈΡΠ½ΡΠΌ ΠΏΠΎΠ»Π΅ΠΌ ΠΏΠΎΡΡΠΎΡΠ½Π½ΠΎΠ³ΠΎ ΠΌΠ°Π³Π½ΠΈΡΠ°.
ΠΠ½ΡΠΌΠΈ ΡΠ»ΠΎΠ²Π°ΠΌΠΈ, ΡΠ»Π΅ΠΊΡΡΠΈΡΠ΅ΡΠΊΠΈΠΉ ΡΠΎΠΊ Π·Π°Π²ΠΈΡΠΈΡ ΠΎΡ ΠΊΠ°ΠΊΠΎΠΉ-ΡΠΎ Ρ Π°ΡΠ°ΠΊΡΠ΅ΡΠΈΡΡΠΈΠΊΠΈ ΠΌΠ°Π³Π½ΠΈΡΠ½ΠΎΠ³ΠΎ ΠΏΠΎΠ»Ρ. Π ΠΌΡ ΡΠΆΠ΅ Π²Π²Π΅Π»ΠΈ ΡΠ°ΠΊΡΡ Ρ Π°ΡΠ°ΠΊΡΠ΅ΡΠΈΡΡΠΈΠΊΡ β ΠΌΠ°Π³Π½ΠΈΡΠ½Π°Ρ ΠΈΠ½Π΄ΡΠΊΡΠΈΡ.
ΠΠ°ΠΏΠΎΠΌΠ½ΠΈΠΌ, ΡΡΠΎ ΠΌΠ°Π³Π½ΠΈΡΠ½Π°Ρ ΠΈΠ½Π΄ΡΠΊΡΠΈΡ ΠΎΠ±ΠΎΠ·Π½Π°ΡΠ°Π΅ΡΡΡ Π±ΡΠΊΠ²ΠΎΠΉ , ΡΡΠΎ β Π²Π΅ΠΊΡΠΎΡΠ½Π°Ρ Π²Π΅Π»ΠΈΡΠΈΠ½Π°. Π ΠΈΠ·ΠΌΠ΅ΡΡΠ΅ΡΡΡ ΠΌΠ°Π³Π½ΠΈΡΠ½Π°Ρ ΠΈΠ½Π΄ΡΠΊΡΠΈΡ Π² ΡΠ΅ΡΠ»Π°Ρ .
- [TΠ»] β Π’Π΅ΡΠ»Π° βΒ Π² ΡΠ΅ΡΡΡ Π΅Π²ΡΠΎΠΏΠ΅ΠΉΡΠΊΠΎΠ³ΠΎ ΠΈ Π°ΠΌΠ΅ΡΠΈΠΊΠ°Π½ΡΠΊΠΎΠ³ΠΎ ΡΡΠ΅Π½ΠΎΠ³ΠΎ ΠΠΈΠΊΠΎΠ»Ρ Π’Π΅ΡΠ»Π°.
- ΠΠ°Π³Π½ΠΈΡΠ½Π°Ρ ΠΈΠ½Π΄ΡΠΊΡΠΈΡ Ρ Π°ΡΠ°ΠΊΡΠ΅ΡΠΈΠ·ΡΠ΅Ρ Π΄Π΅ΠΉΡΡΠ²ΠΈΠ΅ ΠΌΠ°Π³Π½ΠΈΡΠ½ΠΎΠ³ΠΎ ΠΏΠΎΠ»Ρ Π½Π° ΠΏΡΠΎΠ²ΠΎΠ΄Π½ΠΈΠΊ Ρ ΡΠΎΠΊΠΎΠΌ, ΠΏΠΎΠΌΠ΅ΡΠ΅Π½Π½ΡΠΉ Π² ΡΡΠΎ ΠΏΠΎΠ»Π΅.
- ΠΠΎ, ΠΊΠΎΠ³Π΄Π° ΠΌΡ Π³ΠΎΠ²ΠΎΡΠΈΠΌ ΠΎΠ± ΡΠ»Π΅ΠΊΡΡΠΈΡΠ΅ΡΠΊΠΎΠΌ ΡΠΎΠΊΠ΅, ΡΠΎ Π΄ΠΎΠ»ΠΆΠ½Ρ ΠΏΠΎΠ½ΠΈΠΌΠ°ΡΡ, ΡΡΠΎ ΡΠ»Π΅ΠΊΡΡΠΈΡΠ΅ΡΠΊΠΈΠΉ ΡΠΎΠΊ, ΠΈ ΡΡΠΎ Π²Ρ Π·Π½Π°Π΅ΡΠ΅ ΠΈΠ· 8 ΠΊΠ»Π°ΡΡΠ°, Π²ΠΎΠ·Π½ΠΈΠΊΠ°Π΅Ρ ΠΏΠΎΠ΄ Π΄Π΅ΠΉΡΡΠ²ΠΈΠ΅ΠΌ ΡΠ»Π΅ΠΊΡΡΠΈΡΠ΅ΡΠΊΠΎΠ³ΠΎ ΠΏΠΎΠ»Ρ.
Π‘Π»Π΅Π΄ΠΎΠ²Π°ΡΠ΅Π»ΡΠ½ΠΎ, ΠΌΠΎΠΆΠ½ΠΎ ΡΠ΄Π΅Π»Π°ΡΡ Π²ΡΠ²ΠΎΠ΄ ΠΎ ΡΠΎΠΌ, ΡΡΠΎ ΡΠ»Π΅ΠΊΡΡΠΈΡΠ΅ΡΠΊΠΈΠΉ ΠΈΠ½Π΄ΡΠΊΡΠΈΠΎΠ½Π½ΡΠΉ ΡΠΎΠΊ ΠΏΠΎΡΠ²Π»ΡΠ΅ΡΡΡ Π·Π° ΡΡΠ΅Ρ ΡΠ»Π΅ΠΊΡΡΠΈΡΠ΅ΡΠΊΠΎΠ³ΠΎ ΠΏΠΎΠ»Ρ, ΠΊΠΎΡΠΎΡΡΠΉ Π² ΡΠ²ΠΎΡ ΠΎΡΠ΅ΡΠ΅Π΄Ρ ΠΎΠ±ΡΠ°Π·ΡΠ΅ΡΡΡ Π² ΡΠ΅Π·ΡΠ»ΡΡΠ°ΡΠ΅ Π΄Π΅ΠΉΡΡΠ²ΠΈΡ ΠΌΠ°Π³Π½ΠΈΡΠ½ΠΎΠ³ΠΎ ΠΏΠΎΠ»Ρ. Π ΡΠ°ΠΊΠ°Ρ Π²Π·Π°ΠΈΠΌΠΎΡΠ²ΡΠ·Ρ ΠΊΠ°ΠΊ ΡΠ°Π· ΠΎΡΡΡΠ΅ΡΡΠ²Π»ΡΠ΅ΡΡΡ Π·Π° ΡΡΠ΅Ρ ΠΌΠ°Π³Π½ΠΈΡΠ½ΠΎΠ³ΠΎ ΠΏΠΎΡΠΎΠΊΠ°.
Π§ΡΠΎ ΠΆΠ΅ ΡΠ°ΠΊΠΎΠ΅ ΠΌΠ°Π³Π½ΠΈΡΠ½ΡΠΉ ΠΏΠΎΡΠΎΠΊ?
ΠΠ°Π³Π½ΠΈΡΠ½ΡΠΉ ΠΏΠΎΡΠΎΠΊ ΠΎΠ±ΠΎΠ·Π½Π°ΡΠ°Π΅ΡΡΡ Π±ΡΠΊΠ²ΠΎΠΉ Π€ ΠΈ Π²ΡΡΠ°ΠΆΠ°Π΅ΡΡΡ Π² ΡΠ°ΠΊΠΈΡ Π΅Π΄ΠΈΠ½ΠΈΡΠ°Ρ , ΠΊΠ°ΠΊ Π²Π΅Π±Π΅Ρ, ΠΈ ΠΎΠ±ΠΎΠ·Π½Π°ΡΠ°Π΅ΡΡΡ [BΠ±].
ΠΠ°Π³Π½ΠΈΡΠ½ΡΠΉ ΠΏΠΎΡΠΎΠΊ ΠΌΠΎΠΆΠ½ΠΎ ΡΡΠ°Π²Π½ΠΈΡΡ Ρ ΠΏΠΎΡΠΎΠΊΠΎΠΌ ΠΆΠΈΠ΄ΠΊΠΎΡΡΠΈ, ΠΏΡΠΎΡΠ΅ΠΊΠ°ΡΡΠ΅ΠΉ ΡΠ΅ΡΠ΅Π· ΠΎΠ³ΡΠ°Π½ΠΈΡΠ΅Π½Π½ΡΡ ΠΏΠΎΠ²Π΅ΡΡ Π½ΠΎΡΡΡ. ΠΡΠ»ΠΈ Π²Π·ΡΡΡ ΡΡΡΠ±Ρ, ΠΈ Π² ΡΡΠΎΠΉ ΡΡΡΠ±Π΅ ΠΏΡΠΎΡΠ΅ΠΊΠ°Π΅Ρ ΠΆΠΈΠ΄ΠΊΠΎΡΡΡ, ΡΠΎ, ΡΠΎΠΎΡΠ²Π΅ΡΡΡΠ²Π΅Π½Π½ΠΎ, ΡΠ΅ΡΠ΅Π· ΠΏΠ»ΠΎΡΠ°Π΄Ρ ΡΠ΅ΡΠ΅Π½ΠΈΡ ΡΡΡΠ±Ρ Π±ΡΠ΄Π΅Ρ ΠΏΡΠΎΡΠ΅ΠΊΠ°ΡΡ ΠΎΠΏΡΠ΅Π΄Π΅Π»Π΅Π½Π½ΡΠΉ ΠΏΠΎΡΠΎΠΊ Π²ΠΎΠ΄Ρ.
ΠΠ°Π³Π½ΠΈΡΠ½ΡΠΉ ΠΏΠΎΡΠΎΠΊ ΠΏΠΎ ΡΠ°ΠΊΠΎΠΉ Π°Π½Π°Π»ΠΎΠ³ΠΈΠΈ Ρ Π°ΡΠ°ΠΊΡΠ΅ΡΠΈΠ·ΡΠ΅Ρ, ΠΊΠ°ΠΊΠΎΠ΅ ΠΊΠΎΠ»ΠΈΡΠ΅ΡΡΠ²ΠΎ ΠΌΠ°Π³Π½ΠΈΡΠ½ΡΡ Π»ΠΈΠ½ΠΈΠΉ Π±ΡΠ΄Π΅Ρ ΠΏΡΠΎΡ ΠΎΠ΄ΠΈΡΡ ΡΠ΅ΡΠ΅Π· ΠΎΠ³ΡΠ°Π½ΠΈΡΠ΅Π½Π½ΡΠΉ ΠΊΠΎΠ½ΡΡΡ. ΠΡΠΎΡ ΠΊΠΎΠ½ΡΡΡ ΡΡΠΎ ΠΈ Π΅ΡΡΡ ΠΏΠ»ΠΎΡΠ°Π΄ΠΊΠ°, ΠΎΠ³ΡΠ°Π½ΠΈΡΠ΅Π½Π½Π°Ρ ΠΏΡΠΎΠ²ΠΎΠ»ΠΎΡΠ½ΡΠΌ Π²ΠΈΡΠΊΠΎΠΌ ΠΈΠ»ΠΈ, ΠΌΠΎΠΆΠ΅Ρ Π±ΡΡΡ, ΠΊΠ°ΠΊΠΎΠΉ-Π»ΠΈΠ±ΠΎ Π΄ΡΡΠ³ΠΎΠΉ ΡΠΎΡΠΌΠΎΠΉ, ΠΏΡΠΈ ΡΡΠΎΠΌ ΠΎΠ±ΡΠ·Π°ΡΠ΅Π»ΡΠ½ΠΎ ΡΡΠ° ΠΏΠ»ΠΎΡΠ°Π΄Ρ β ΠΎΠ³ΡΠ°Π½ΠΈΡΠ΅Π½Π½Π°Ρ.
Π ΠΈΡ. 2. Π ΠΏΠ΅ΡΠ²ΠΎΠΌ ΡΠ»ΡΡΠ°Π΅ ΠΌΠ°Π³Π½ΠΈΡΠ½ΡΠΉ ΠΏΠΎΡΠΎΠΊ ΠΌΠ°ΠΊΡΠΈΠΌΠ°Π»Π΅Π½. ΠΠΎ Π²ΡΠΎΡΠΎΠΌ ΡΠ»ΡΡΠ°Π΅ β ΡΠ°Π²Π΅Π½ Π½ΡΠ»Ρ.
ΠΠ° ΡΠΈΡΡΠ½ΠΊΠ΅ ΠΈΠ·ΠΎΠ±ΡΠ°ΠΆΠ΅Π½Ρ Π΄Π²Π° Π²ΠΈΡΠΊΠ°. ΠΠ΄ΠΈΠ½ Π²ΠΈΡΠΎΠΊ β ΡΡΠΎ ΠΏΡΠΎΠ²ΠΎΠ»ΠΎΡΠ½ΡΠΉ Π²ΠΈΡΠΎΠΊ, ΡΠ΅ΡΠ΅Π· ΠΊΠΎΡΠΎΡΡΠΉ ΠΏΡΠΎΡ ΠΎΠ΄ΡΡ Π»ΠΈΠ½ΠΈΠΈ ΠΌΠ°Π³Π½ΠΈΡΠ½ΠΎΠΉ ΠΈΠ½Π΄ΡΠΊΡΠΈΠΈ. ΠΠ°ΠΊ Π²ΠΈΠ΄ΠΈΡΠ΅, ΡΡΠΈΡ Π»ΠΈΠ½ΠΈΠΉ Π·Π΄Π΅ΡΡ ΠΈΠ·ΠΎΠ±ΡΠ°ΠΆΠ΅Π½ΠΎ ΡΠ΅ΡΡΡΠ΅.
ΠΡΠ»ΠΈ Π±Ρ ΠΈΡ Π±ΡΠ»ΠΎ Π³ΠΎΡΠ°Π·Π΄ΠΎ Π±ΠΎΠ»ΡΡΠ΅, ΡΠΎ ΠΌΡ Π±Ρ Π³ΠΎΠ²ΠΎΡΠΈΠ»ΠΈ, ΡΡΠΎ ΠΌΠ°Π³Π½ΠΈΡΠ½ΡΠΉ ΠΏΠΎΡΠΎΠΊ Π±ΡΠ΄Π΅Ρ Π±ΠΎΠ»ΡΡΠΎΠΉ.
ΠΡΠ»ΠΈ Π±Ρ ΡΡΠΈΡ Π»ΠΈΠ½ΠΈΠΉ Π±ΡΠ»ΠΎ ΠΌΠ΅Π½ΡΡΠ΅, Π½Π°ΠΏΡΠΈΠΌΠ΅Ρ, ΠΌΡ Π±Ρ Π½Π°ΡΠΈΡΠΎΠ²Π°Π»ΠΈ ΠΎΠ΄Π½Ρ Π»ΠΈΠ½ΠΈΡ, ΡΠΎ ΡΠΎΠ³Π΄Π° Π±Ρ ΠΌΡ ΠΌΠΎΠ³Π»ΠΈ ΡΠΊΠ°Π·Π°ΡΡ, ΡΡΠΎ ΠΌΠ°Π³Π½ΠΈΡΠ½ΡΠΉ ΠΏΠΎΡΠΎΠΊ Π΄ΠΎΡΡΠ°ΡΠΎΡΠ½ΠΎ ΠΌΠ°Π», ΠΎΠ½ Π½Π΅Π±ΠΎΠ»ΡΡΠΎΠΉ.
Π Π΅ΡΠ΅ ΠΎΠ΄ΠΈΠ½ ΡΠ»ΡΡΠ°ΠΉ: ΡΠΎΠ³Π΄Π°, ΠΊΠΎΠ³Π΄Π° Π²ΠΈΡΠΎΠΊ ΡΠ°ΡΠΏΠΎΠ»Π°Π³Π°Π΅ΡΡΡ ΡΠ°ΠΊΠΈΠΌ ΠΎΠ±ΡΠ°Π·ΠΎΠΌ, ΡΡΠΎ ΡΠ΅ΡΠ΅Π· Π΅Π³ΠΎ ΠΏΠ»ΠΎΡΠ°Π΄Ρ Π½Π΅ ΠΏΡΠΎΡ ΠΎΠ΄ΡΡ ΠΌΠ°Π³Π½ΠΈΡΠ½ΡΠ΅ Π»ΠΈΠ½ΠΈΠΈ. Π’Π°ΠΊΠΎΠ΅ Π²ΠΏΠ΅ΡΠ°ΡΠ»Π΅Π½ΠΈΠ΅, ΡΡΠΎ Π»ΠΈΠ½ΠΈΠΈ ΠΌΠ°Π³Π½ΠΈΡΠ½ΠΎΠΉ ΠΈΠ½Π΄ΡΠΊΡΠΈΠΈ ΡΠΊΠΎΠ»ΡΠ·ΡΡ ΠΏΠΎ ΠΏΠΎΠ²Π΅ΡΡ Π½ΠΎΡΡΠΈ. Π ΡΡΠΎΠΌ ΡΠ»ΡΡΠ°Π΅ ΠΌΠΎΠΆΠ½ΠΎ ΡΠΊΠ°Π·Π°ΡΡ, ΡΡΠΎ ΠΌΠ°Π³Π½ΠΈΡΠ½ΡΠΉ ΠΏΠΎΡΠΎΠΊ ΠΎΡΡΡΡΡΡΠ²ΡΠ΅Ρ, Ρ.Π΅. Π½Π΅Ρ Π»ΠΈΠ½ΠΈΠΉ, ΠΊΠΎΡΠΎΡΡΠ΅ ΠΏΡΠΎΠ½ΠΈΠ·ΡΠ²Π°Π»ΠΈ Π±Ρ ΠΏΠΎΠ²Π΅ΡΡ Π½ΠΎΡΡΡ ΡΡΠΎΠ³ΠΎ ΠΊΠΎΠ½ΡΡΡΠ°.
ΠΠ°Π³Π½ΠΈΡΠ½ΡΠΉ ΠΏΠΎΡΠΎΠΊ Ρ Π°ΡΠ°ΠΊΡΠ΅ΡΠΈΠ·ΡΠ΅Ρ Π²Π΅ΡΡ ΠΌΠ°Π³Π½ΠΈΡ Π² ΡΠ΅Π»ΠΎΠΌ (Π»ΠΈΠ±ΠΎ Π΄ΡΡΠ³ΠΎΠΉ ΠΈΡΡΠΎΡΠ½ΠΈΠΊ ΠΌΠ°Π³Π½ΠΈΡΠ½ΠΎΠ³ΠΎ ΠΏΠΎΠ»Ρ). ΠΡΠ»ΠΈ ΠΌΠ°Π³Π½ΠΈΡΠ½Π°Ρ ΠΈΠ½Π΄ΡΠΊΡΠΈΡ Ρ Π°ΡΠ°ΠΊΡΠ΅ΡΠΈΠ·ΡΠ΅Ρ Π΄Π΅ΠΉΡΡΠ²ΠΈΠ΅ Π² ΠΊΠ°ΠΊΠΎΠΉ-ΡΠΎ ΠΎΠ΄Π½ΠΎΠΉ ΡΠΎΡΠΊΠ΅, ΡΠΎ ΠΌΠ°Π³Π½ΠΈΡΠ½ΡΠΉ ΠΏΠΎΡΠΎΠΊ β Π²Π΅ΡΡ ΠΌΠ°Π³Π½ΠΈΡ ΡΠ΅Π»ΠΈΠΊΠΎΠΌ.
ΠΠΎΠΆΠ½ΠΎ ΡΠΊΠ°Π·Π°ΡΡ ΠΎ ΡΠΎΠΌ, ΡΡΠΎ ΠΌΠ°Π³Π½ΠΈΡΠ½ΡΠΉ ΠΏΠΎΡΠΎΠΊ β ΡΡΠΎ Π²ΡΠΎΡΠ°Ρ ΠΎΡΠ΅Π½Ρ Π²Π°ΠΆΠ½Π°Ρ Ρ Π°ΡΠ°ΠΊΡΠ΅ΡΠΈΡΡΠΈΠΊΠ° ΠΌΠ°Π³Π½ΠΈΡΠ½ΠΎΠ³ΠΎ ΠΏΠΎΠ»Ρ.
ΠΡΠ»ΠΈ ΠΌΠ°Π³Π½ΠΈΡΠ½ΡΡ ΠΈΠ½Π΄ΡΠΊΡΠΈΡ Π½Π°Π·ΡΠ²Π°ΡΡ ΡΠΈΠ»ΠΎΠ²ΠΎΠΉ Ρ Π°ΡΠ°ΠΊΡΠ΅ΡΠΈΡΡΠΈΠΊΠΎΠΉ ΠΌΠ°Π³Π½ΠΈΡΠ½ΠΎΠ³ΠΎ ΠΏΠΎΠ»Ρ, ΡΠΎ ΠΌΠ°Π³Π½ΠΈΡΠ½ΡΠΉ ΠΏΠΎΡΠΎΠΊ β ΡΡΠΎ ΡΠ½Π΅ΡΠ³Π΅ΡΠΈΡΠ΅ΡΠΊΠ°Ρ Ρ Π°ΡΠ°ΠΊΡΠ΅ΡΠΈΡΡΠΈΠΊΠ° ΠΌΠ°Π³Π½ΠΈΡΠ½ΠΎΠ³ΠΎ ΠΏΠΎΠ»Ρ.
ΠΠ΅ΡΠ½ΡΠ²ΡΠΈΡΡ ΠΊ ΡΠΊΡΠΏΠ΅ΡΠΈΠΌΠ΅Π½ΡΠ°ΠΌ, ΠΌΠΎΠΆΠ½ΠΎ ΡΠΊΠ°Π·Π°ΡΡ ΠΎ ΡΠΎΠΌ, ΡΡΠΎ ΠΊΠ°ΠΆΠ΄ΡΠΉ Π²ΠΈΡΠΎΠΊ ΠΊΠ°ΡΡΡΠΊΠΈ ΠΌΠΎΠΆΠ½ΠΎ ΠΏΡΠ΅Π΄ΡΡΠ°Π²ΠΈΡΡ ΠΊΠ°ΠΊ ΠΎΡΠ΄Π΅Π»ΡΠ½ΡΠΉ Π·Π°ΠΌΠΊΠ½ΡΡΡΠΉ Π²ΠΈΡΠΎΠΊ. Π’ΠΎΡ ΡΠ°ΠΌΡΠΉ ΠΊΠΎΠ½ΡΡΡ, ΡΠ΅ΡΠ΅Π· ΠΊΠΎΡΠΎΡΡΠΉ ΠΈ Π±ΡΠ΄Π΅Ρ ΠΏΡΠΎΡ ΠΎΠ΄ΠΈΡΡ ΠΌΠ°Π³Π½ΠΈΡΠ½ΡΠΉ ΠΏΠΎΡΠΎΠΊ Π²Π΅ΠΊΡΠΎΡΠ° ΠΌΠ°Π³Π½ΠΈΡΠ½ΠΎΠΉ ΠΈΠ½Π΄ΡΠΊΡΠΈΠΈ. Π ΡΡΠΎΠΌ ΡΠ»ΡΡΠ°Π΅ Π±ΡΠ΄Π΅Ρ Π½Π°Π±Π»ΡΠ΄Π°ΡΡΡΡ ΠΈΠ½Π΄ΡΠΊΡΠΈΠΎΠ½Π½ΡΠΉ ΡΠ»Π΅ΠΊΡΡΠΈΡΠ΅ΡΠΊΠΈΠΉ ΡΠΎΠΊ.
Π’.ΠΎ., ΠΈΠΌΠ΅Π½Π½ΠΎ ΠΏΠΎΠ΄ Π΄Π΅ΠΉΡΡΠ²ΠΈΠ΅ΠΌ ΠΌΠ°Π³Π½ΠΈΡΠ½ΠΎΠ³ΠΎ ΠΏΠΎΡΠΎΠΊΠ° ΡΠΎΠ·Π΄Π°Π΅ΡΡΡ ΡΠ»Π΅ΠΊΡΡΠΈΡΠ΅ΡΠΊΠΎΠ΅ ΠΏΠΎΠ»Π΅ Π² Π·Π°ΠΌΠΊΠ½ΡΡΠΎΠΌ ΠΏΡΠΎΠ²ΠΎΠ΄Π½ΠΈΠΊΠ΅. Π ΡΠΆΠ΅ ΡΡΠΎ ΡΠ»Π΅ΠΊΡΡΠΈΡΠ΅ΡΠΊΠΎΠ΅ ΠΏΠΎΠ»Π΅ ΡΠΎΠ·Π΄Π°Π΅Ρ Π½Π΅ ΡΡΠΎ ΠΈΠ½ΠΎΠ΅, ΠΊΠ°ΠΊ ΡΠ»Π΅ΠΊΡΡΠΈΡΠ΅ΡΠΊΠΈΠΉ ΡΠΎΠΊ.
ΠΠ°Π²Π°ΠΉΡΠ΅ ΠΏΠΎΡΠΌΠΎΡΡΠΈΠΌ Π΅ΡΠ΅ ΡΠ°Π· Π½Π° ΡΠΊΡΠΏΠ΅ΡΠΈΠΌΠ΅Π½Ρ, ΠΈ ΡΠ΅ΠΏΠ΅ΡΡ, ΡΠΆΠ΅ Π·Π½Π°Ρ, ΡΡΠΎ ΡΡΡΠ΅ΡΡΠ²ΡΠ΅Ρ ΠΌΠ°Π³Π½ΠΈΡΠ½ΡΠΉ ΠΏΠΎΡΠΎΠΊ, ΠΏΠΎΡΠΌΠΎΡΡΠΈΠΌ Π½Π° ΡΠ²ΡΠ·Ρ ΠΌΠ°Π³Π½ΠΈΡΠ½ΠΎΠ³ΠΎ ΠΏΠΎΡΠΎΠΊΠ° ΠΈ Π·Π½Π°ΡΠ΅Π½ΠΈΠ΅ ΠΈΠ½Π΄ΡΠΊΡΠΈΠΎΠ½Π½ΠΎΠ³ΠΎ ΡΠ»Π΅ΠΊΡΡΠΈΡΠ΅ΡΠΊΠΎΠ³ΠΎ ΡΠΎΠΊΠ°.
ΠΠΎΠ·ΡΠΌΠ΅ΠΌ ΠΌΠ°Π³Π½ΠΈΡ ΠΈ Π΄ΠΎΡΡΠ°ΡΠΎΡΠ½ΠΎ ΠΌΠ΅Π΄Π»Π΅Π½Π½ΠΎ ΠΏΡΠΎΠΏΡΡΡΠΈΠΌ Π΅Π³ΠΎ ΡΠ΅ΡΠ΅Π· ΠΊΠ°ΡΡΡΠΊΡ. ΠΠ½Π°ΡΠ΅Π½ΠΈΠ΅ ΡΠ»Π΅ΠΊΡΡΠΈΡΠ΅ΡΠΊΠΎΠ³ΠΎ ΡΠΎΠΊΠ° ΠΌΠ΅Π½ΡΠ΅ΡΡΡ ΠΎΡΠ΅Π½Ρ Π½Π΅Π·Π½Π°ΡΠΈΡΠ΅Π»ΡΠ½ΠΎ.
ΠΡΠ»ΠΈ ΠΆΠ΅ ΠΏΠΎΠΏΡΡΠ°ΡΡΡΡ Π²ΡΡΠ°ΡΠΈΡΡ ΠΌΠ°Π³Π½ΠΈΡ Π±ΡΡΡΡΠΎ, ΡΠΎ Π·Π½Π°ΡΠ΅Π½ΠΈΠ΅ ΡΠ»Π΅ΠΊΡΡΠΈΡΠ΅ΡΠΊΠΎΠ³ΠΎ ΡΠΎΠΊΠ° Π±ΡΠ΄Π΅Ρ Π±ΠΎΠ»ΡΡΠ΅, ΡΠ΅ΠΌ Π² ΠΏΠ΅ΡΠ²ΠΎΠΌ ΡΠ»ΡΡΠ°Π΅.
Π Π΄Π°Π½Π½ΠΎΠΌ ΡΠ»ΡΡΠ°Π΅ ΡΠΎΠ»Ρ ΠΈΠ³ΡΠ°Π΅Ρ ΡΠΊΠΎΡΠΎΡΡΡ ΠΈΠ·ΠΌΠ΅Π½Π΅Π½ΠΈΡ ΠΌΠ°Π³Π½ΠΈΡΠ½ΠΎΠ³ΠΎ ΠΏΠΎΡΠΎΠΊΠ°. ΠΡΠ»ΠΈ ΠΈΠ·ΠΌΠ΅Π½Π΅Π½ΠΈΠ΅ ΡΠΊΠΎΡΠΎΡΡΠΈ ΠΌΠ°Π³Π½ΠΈΡΠ° Π±ΡΠ΄Π΅Ρ Π΄ΠΎΡΡΠ°ΡΠΎΡΠ½ΠΎ Π±ΠΎΠ»ΡΡΠΈΠΌ, Π·Π½Π°ΡΠΈΡ, ΠΈ ΠΈΠ½Π΄ΡΠΊΡΠΈΠΎΠ½Π½ΡΠΉ ΡΠΎΠΊ ΡΠΎΠΆΠ΅ Π±ΡΠ΄Π΅Ρ Π·Π½Π°ΡΠΈΡΠ΅Π»ΡΠ½ΡΠΌ.
Π ΡΠ΅Π·ΡΠ»ΡΡΠ°ΡΠ΅ ΡΠ°ΠΊΠΎΠ³ΠΎ ΡΠΎΠ΄Π° ΡΠΊΡΠΏΠ΅ΡΠΈΠΌΠ΅Π½ΡΠΎΠ² Π±ΡΠ»ΠΈ Π²ΡΡΠ²Π»Π΅Π½Ρ ΡΠ»Π΅Π΄ΡΡΡΠΈΠ΅ Π·Π°ΠΊΠΎΠ½ΠΎΠΌΠ΅ΡΠ½ΠΎΡΡΠΈ.
Π ΠΈΡ. 3. ΠΡ ΡΠ΅Π³ΠΎ Π·Π°Π²ΠΈΡΡΡ ΠΌΠ°Π³Π½ΠΈΡΠ½ΡΠΉ ΠΏΠΎΡΠΎΠΊ ΠΈ ΠΈΠ½Π΄ΡΠΊΡΠΈΠΎΠ½Π½ΡΠΉ ΡΠΎΠΊ
1. ΠΠ°Π³Π½ΠΈΡΠ½ΡΠΉ ΠΏΠΎΡΠΎΠΊ ΠΏΡΠΎΠΏΠΎΡΡΠΈΠΎΠ½Π°Π»Π΅Π½ ΠΌΠ°Π³Π½ΠΈΡΠ½ΠΎΠΉ ΠΈΠ½Π΄ΡΠΊΡΠΈΠΈ.
2. ΠΠ°Π³Π½ΠΈΡΠ½ΡΠΉ ΠΏΠΎΡΠΎΠΊ ΠΏΡΡΠΌΠΎ ΠΏΡΠΎΠΏΠΎΡΡΠΈΠΎΠ½Π°Π»Π΅Π½ ΠΏΠ»ΠΎΡΠ°Π΄ΠΈ ΠΏΠΎΠ²Π΅ΡΡ Π½ΠΎΡΡΠΈ ΠΊΠΎΠ½ΡΡΡΠ°, ΡΠ΅ΡΠ΅Π· ΠΊΠΎΡΠΎΡΡΠΉ ΠΏΡΠΎΡ ΠΎΠ΄ΡΡ Π»ΠΈΠ½ΠΈΠΈ ΠΌΠ°Π³Π½ΠΈΡΠ½ΠΎΠΉ ΠΈΠ½Π΄ΡΠΊΡΠΈΠΈ.
3. Π ΡΡΠ΅ΡΡΠ΅ β Π·Π°Π²ΠΈΡΠΈΠΌΠΎΡΡΡ ΠΌΠ°Π³Π½ΠΈΡΠ½ΠΎΠ³ΠΎ ΠΏΠΎΡΠΎΠΊΠ° ΠΎΡ ΡΠ³Π»Π° ΡΠ°ΡΠΏΠΎΠ»ΠΎΠΆΠ΅Π½ΠΈΡ ΠΊΠΎΠ½ΡΡΡΠ°. ΠΡ ΡΠΆΠ΅ ΠΎΠ±ΡΠ°ΡΠ°Π»ΠΈ Π²Π½ΠΈΠΌΠ°Π½ΠΈΠ΅ Π½Π° ΡΠΎ, ΡΡΠΎ, Π΅ΡΠ»ΠΈ ΠΏΠ»ΠΎΡΠ°Π΄Ρ ΠΊΠΎΠ½ΡΡΡΠ° ΡΠ΅ΠΌ ΠΈΠ»ΠΈ ΠΈΠ½ΡΠΌ ΠΎΠ±ΡΠ°Π·ΠΎΠΌ, ΡΡΠΎ ΠΎΠΊΠ°Π·ΡΠ²Π°Π΅Ρ Π²Π»ΠΈΡΠ½ΠΈΠ΅ Π½Π° Π½Π°Π»ΠΈΡΠΈΠ΅ ΠΈ Π²Π΅Π»ΠΈΡΠΈΠ½Ρ ΠΌΠ°Π³Π½ΠΈΡΠ½ΠΎΠ³ΠΎ ΠΏΠΎΡΠΎΠΊΠ°.
- Π’Π°ΠΊΠΈΠΌ ΠΎΠ±ΡΠ°Π·ΠΎΠΌ, ΠΌΠΎΠΆΠ½ΠΎ ΡΠΊΠ°Π·Π°ΡΡ, ΡΡΠΎ ΡΠΈΠ»Π° ΠΈΠ½Π΄ΡΠΊΡΠΈΠΎΠ½Π½ΠΎΠ³ΠΎ ΡΠΎΠΊΠ° ΠΏΡΡΠΌΠΎ ΠΏΡΠΎΠΏΠΎΡΡΠΈΠΎΠ½Π°Π»ΡΠ½Π° ΡΠΊΠΎΡΠΎΡΡΠΈ ΠΈΠ·ΠΌΠ΅Π½Π΅Π½ΠΈΡ ΠΌΠ°Π³Π½ΠΈΡΠ½ΠΎΠ³ΠΎ ΠΏΠΎΡΠΎΠΊΠ°.
- I~
- βΠ€ β ΡΡΠΎ ΠΈΠ·ΠΌΠ΅Π½Π΅Π½ΠΈΠ΅ ΠΌΠ°Π³Π½ΠΈΡΠ½ΠΎΠ³ΠΎ ΠΏΠΎΡΠΎΠΊΠ°.
- βt β ΡΡΠΎ Π²ΡΠ΅ΠΌΡ, Π² ΡΠ΅ΡΠ΅Π½ΠΈΠ΅ ΠΊΠΎΡΠΎΡΠΎΠ³ΠΎ ΠΈΠ·ΠΌΠ΅Π½ΡΠ΅ΡΡΡ ΠΌΠ°Π³Π½ΠΈΡΠ½ΡΠΉ ΠΏΠΎΡΠΎΠΊ.
- ΠΡΠ½ΠΎΡΠ΅Π½ΠΈΠ΅ β ΡΡΠΎ ΠΊΠ°ΠΊ ΡΠ°Π· ΠΈ Π΅ΡΡΡ ΡΠΊΠΎΡΠΎΡΡΡ ΠΈΠ·ΠΌΠ΅Π½Π΅Π½ΠΈΡ ΠΌΠ°Π³Π½ΠΈΡΠ½ΠΎΠ³ΠΎ ΠΏΠΎΡΠΎΠΊΠ°.
- ΠΡΡ ΠΎΠ΄Ρ ΠΈΠ· ΡΡΠΎΠΉ Π·Π°Π²ΠΈΡΠΈΠΌΠΎΡΡΠΈ, ΠΌΠΎΠΆΠ½ΠΎ ΡΠ΄Π΅Π»Π°ΡΡ Π²ΡΠ²ΠΎΠ΄, ΡΡΠΎ, Π½Π°ΠΏΡΠΈΠΌΠ΅Ρ, ΠΈΠ½Π΄ΡΠΊΡΠΈΠΎΠ½Π½ΡΠΉ ΡΠΎΠΊ ΠΌΠΎΠΆΠ΅Ρ Π±ΡΡΡ ΡΠΎΠ·Π΄Π°Π½ ΠΈ Π΄ΠΎΡΡΠ°ΡΠΎΡΠ½ΠΎ ΡΠ»Π°Π±ΡΠΌ ΠΌΠ°Π³Π½ΠΈΡΠΎΠΌ, Π½ΠΎ ΠΏΡΠΈ ΡΡΠΎΠΌ ΡΠΊΠΎΡΠΎΡΡΡ Π΄Π²ΠΈΠΆΠ΅Π½ΠΈΡ ΡΡΠΎΠ³ΠΎ ΠΌΠ°Π³Π½ΠΈΡΠ° Π΄ΠΎΠ»ΠΆΠ½Π° Π±ΡΡΡ ΠΎΡΠ΅Π½Ρ Π±ΠΎΠ»ΡΡΠΎΠΉ.
ΠΠ΅ΡΠ²ΡΠΌ ΡΠ΅Π»ΠΎΠ²Π΅ΠΊΠΎΠΌ, ΠΊΠΎΡΠΎΡΡΠΉ ΡΡΠΎΡ Π·Π°ΠΊΠΎΠ½ ΠΏΠΎΠ»ΡΡΠΈΠ», Π±ΡΠ» Π°Π½Π³Π»ΠΈΠΉΡΠΊΠΈΠΉ ΡΡΠ΅Π½ΡΠΉ Π. Π€Π°ΡΠ°Π΄Π΅ΠΉ. ΠΠΎΠ½ΡΡΠΈΠ΅ ΠΌΠ°Π³Π½ΠΈΡΠ½ΠΎΠ³ΠΎ ΠΏΠΎΡΠΎΠΊΠ° ΠΏΠΎΠ·Π²ΠΎΠ»ΡΠ΅Ρ Π³Π»ΡΠ±ΠΆΠ΅ Π²Π·Π³Π»ΡΠ½ΡΡΡ Π½Π° Π΅Π΄ΠΈΠ½ΡΡ ΠΏΡΠΈΡΠΎΠ΄Ρ ΡΠ»Π΅ΠΊΡΡΠΈΡΠ΅ΡΠΊΠΈΡ ΠΈ ΠΌΠ°Π³Π½ΠΈΡΠ½ΡΡ ΡΠ²Π»Π΅Π½ΠΈΠΉ.
Π‘ΠΏΠΈΡΠΎΠΊ Π΄ΠΎΠΏΠΎΠ»Π½ΠΈΡΠ΅Π»ΡΠ½ΠΎΠΉ Π»ΠΈΡΠ΅ΡΠ°ΡΡΡΡ:
ΠΠ»Π΅ΠΌΠ΅Π½ΡΠ°ΡΠ½ΡΠΉ ΡΡΠ΅Π±Π½ΠΈΠΊ ΡΠΈΠ·ΠΈΠΊΠΈ. ΠΠΎΠ΄ ΡΠ΅Π΄. Π.Π‘. ΠΠ°Π½Π΄ΡΠ±Π΅ΡΠ³Π°, Π’. 2. Π., 1974 Π―Π²ΠΎΡΡΠΊΠΈΠΉ Π.Π., ΠΠΈΠ½ΡΠΊΠΈΠΉ Π.Π., ΠΡΠ½ΠΎΠ²Ρ ΡΠΈΠ·ΠΈΠΊΠΈ, Ρ.2., Π. Π€ΠΈΠ·ΠΌΠ°ΡΠ»ΠΈΡ., 2003 Π ΡΠ°ΠΊ Π»ΠΈ Ρ ΠΎΡΠΎΡΠΎ Π·Π½Π°ΠΊΠΎΠΌΡ Π²Π°ΠΌ ΠΏΠΎΡΠΎΠΊΠΈ?// ΠΠ²Π°Π½Ρ. β 2009. β β 3. β Π‘. 32-33. ΠΠΊΡΠ΅Π½ΠΎΠ²ΠΈΡ Π. Π.
Π€ΠΈΠ·ΠΈΠΊΠ° Π² ΡΡΠ΅Π΄Π½Π΅ΠΉ ΡΠΊΠΎΠ»Π΅: Π’Π΅ΠΎΡΠΈΡ. ΠΠ°Π΄Π°Π½ΠΈΡ. Π’Π΅ΡΡΡ: Π£ΡΠ΅Π±. ΠΏΠΎΡΠΎΠ±ΠΈΠ΅ Π΄Π»Ρ ΡΡΡΠ΅ΠΆΠ΄Π΅Π½ΠΈΠΉ, ΠΎΠ±Π΅ΡΠΏΠ΅ΡΠΈΠ²Π°ΡΡΠΈΡ ΠΏΠΎΠ»ΡΡΠ΅Π½ΠΈΠ΅ ΠΎΠ±Ρ. ΡΡΠ΅Π΄, ΠΎΠ±ΡΠ°Π·ΠΎΠ²Π°Π½ΠΈΡ / Π. Π. ΠΠΊΡΠ΅Π½ΠΎΠ²ΠΈΡ, Π.Π.Π Π°ΠΊΠΈΠ½Π°, Π. Π‘. Π€Π°ΡΠΈΠ½ΠΎ; ΠΠΎΠ΄ ΡΠ΅Π΄. Π. Π‘. Π€Π°ΡΠΈΠ½ΠΎ. β ΠΠ½.: ΠΠ΄ΡΠΊΠ°ΡΡΡ i Π²ΡΡ Π°Π²Π°Π½Π½Π΅, 2004. β C.
344.
ΠΡΡΠΎΡΠ½ΠΈΠΊ: https://interneturok.ru/lesson/physics/9-klass/elektromagnitnye-yavleniya/magnitnyy-potok-2
ΠΠ»Π΅ΠΊΡΡΠΎΠΌΠ°Π³Π½ΠΈΡΠ½Π°Ρ ΠΈΠ½Π΄ΡΠΊΡΠΈΡ. | ΠΠ±ΡΠ΅Π΄ΠΈΠ½Π΅Π½ΠΈΠ΅ ΡΡΠΈΡΠ΅Π»Π΅ΠΉ Π‘Π°Π½ΠΊΡ-ΠΠ΅ΡΠ΅ΡΠ±ΡΡΠ³Π°
ΠΠ»Π΅ΠΊΡΡΠΎΠΌΠ°Π³Π½ΠΈΡΠ½Π°Ρ ΠΈΠ½Π΄ΡΠΊΡΠΈΡ | |
1831Β Π³. β Π. Π€Π°ΡΠ°Π΄Π΅ΠΉ ΠΎΠ±Π½Π°ΡΡΠΆΠΈΠ», ΡΡΠΎ Π² Π·Π°ΠΌΠΊΠ½ΡΡΠΎΠΌ ΠΏΡΠΎΠ²ΠΎΠ΄ΡΡΠ΅ΠΌ ΠΊΠΎΠ½ΡΡΡΠ΅ ΠΏΡΠΈ ΠΈΠ·ΠΌΠ΅Π½Π΅Π½ΠΈΠΈ ΠΌΠ°Π³Π½ΠΈΡΠ½ΠΎΠ³ΠΎ ΠΏΠΎΠ»Ρ Π²ΠΎΠ·Π½ΠΈΠΊΠ°Π΅Ρ ΡΠ°ΠΊ Π½Π°Π·ΡΠ²Π°Π΅ΠΌΡΠΉ ΠΈΠ½Π΄ΡΠΊΡΠΈΠΎΠ½Π½ΡΠΉ ΡΠΎΠΊ. (ΠΠ½Π΄ΡΠΊΡΠΈΡ, Π² Π΄Π°Π½Π½ΠΎΠΌ ΡΠ»ΡΡΠ°Π΅, β ΠΏΠΎΡΠ²Π»Π΅Π½ΠΈΠ΅, Π²ΠΎΠ·Π½ΠΈΠΊΠ½ΠΎΠ²Π΅Π½ΠΈΠ΅). | |
| |
| |
ΠΠ°ΠΊΠΎΠ½ ΡΠ»Π΅ΠΊΡΡΠΎΠΌΠ°Π³Π½ΠΈΡΠ½ΠΎΠΉ ΠΈΠ½Π΄ΡΠΊΡΠΈΠΈ ΠΡΠΈ Π²ΡΡΠΊΠΎΠΌ ΠΈΠ·ΠΌΠ΅Π½Π΅Π½ΠΈΠΈ ΠΌΠ°Π³Π½ΠΈΡΠ½ΠΎΠ³ΠΎ ΠΏΠΎΡΠΎΠΊΠ° ΡΠ΅ΡΠ΅Π· ΠΏΡΠΎΠ²ΠΎΠ΄ΡΡΠΈΠΉ Π·Π°ΠΌΠΊΠ½ΡΡΡΠΉ ΠΊΠΎΠ½ΡΡΡ Π² ΡΡΠΎΠΌ ΠΊΠΎΠ½ΡΡΡΠ΅ Π²ΠΎΠ·Π½ΠΈΠΊΠ°Π΅Ρ ΡΠ»Π΅ΠΊΡΡΠΈΡΠ΅ΡΠΊΠΈΠΉ ΡΠΎΠΊ.Β IΒ Π·Π°Π²ΠΈΡΠΈΡ ΠΎΡ ΡΠ²ΠΎΠΉΡΡΠ² ΠΊΠΎΠ½ΡΡΡΠ° (ΡΠΎΠΏΡΠΎΡΠΈΠ²Π»Π΅Π½ΠΈΠ΅):Β Β .Β Β eΒ Π½Π΅ Π·Π°Π²ΠΈΡΠΈΡ ΠΎΡ ΡΠ²ΠΎΠΉΡΡΠ² ΠΊΠΎΠ½ΡΡΡΠ°:Β . ΠΠΠ‘ ΠΈΠ½Π΄ΡΠΊΡΠΈΠΈ Π² Π·Π°ΠΌΠΊΠ½ΡΡΠΎΠΌ ΠΊΠΎΠ½ΡΡΡΠ΅ ΠΏΡΡΠΌΠΎ ΠΏΡΠΎΠΏΠΎΡΡΠΈΠΎΠ½Π°Π»ΡΠ½Π° ΡΠΊΠΎΡΠΎΡΡΠΈ ΠΈΠ·ΠΌΠ΅Π½Π΅Π½ΠΈΡ ΠΌΠ°Π³Π½ΠΈΡΠ½ΠΎΠ³ΠΎ ΠΏΠΎΡΠΎΠΊΠ° ΡΠ΅ΡΠ΅Π· ΠΏΠ»ΠΎΡΠ°Π΄Ρ, ΠΎΠ³ΡΠ°Π½ΠΈΡΠ΅Π½Π½ΡΡ ΡΡΠΈΠΌ ΠΊΠΎΠ½ΡΡΡΠΎΠΌ. | |
ΠΡΠ½ΠΎΠ²Π½ΡΠ΅ ΠΏΡΠΈΠΌΠ΅Π½Π΅Π½ΠΈΡ ΡΠ»Π΅ΠΊΡΡΠΎΠΌΠ°Π³Π½ΠΈΡΠ½ΠΎΠΉ ΠΈΠ½Π΄ΡΠΊΡΠΈΠΈ: Π³Π΅Π½Π΅ΡΠΈΡΠΎΠ²Π°Π½ΠΈΠ΅ ΡΠΎΠΊΠ° (ΠΈΠ½Π΄ΡΠΊΡΠΈΠΎΠ½Π½ΡΠ΅ Π³Π΅Π½Π΅ΡΠ°ΡΠΎΡΡ Π½Π° Π²ΡΠ΅Ρ ΡΠ»Π΅ΠΊΡΡΠΎΡΡΠ°Π½ΡΠΈΡΡ ,Β Π΄ΠΈΠ½Π°ΠΌΠΎΠΌΠ°ΡΠΈΠ½Ρ), ΡΡΠ°Π½ΡΡΠΎΡΠΌΠ°ΡΠΎΡΡ. | |
ΠΠΎΠ·Π½ΠΈΠΊΠ½ΠΎΠ²Π΅Π½ΠΈΠ΅ ΠΈΠ½Π΄ΡΠΊΡΠΈΠΎΠ½Π½ΠΎΠ³ΠΎ ΡΠΎΠΊΠ° β ΡΠ»Π΅Π΄ΡΡΠ²ΠΈΠ΅ Π·Π°ΠΊΠΎΠ½Π° ΡΠΎΡ
ΡΠ°Π½Π΅Π½ΠΈΡ ΡΠ½Π΅ΡΠ³ΠΈΠΈ! Π ΡΠ»ΡΡΠ°Π΅ 1: ΠΡΠΈ ΠΏΡΠΈΠ±Π»ΠΈΠΆΠ΅Π½ΠΈΠΈ ΠΌΠ°Π³Π½ΠΈΡΠ°, ΡΠ²Π΅Π»ΠΈΡΠ΅Π½ΠΈΠΈ ΡΠΎΠΊΠ°, Π·Π°ΠΌΡΠΊΠ°Π½ΠΈΠΈ ΡΠ΅ΠΏΠΈ:Β ;Β ΠΠ°Π³Π½ΠΈΡΠ½ΡΠΉ ΠΏΠΎΡΠΎΠΊΒ Π€ΒΒ βΒ ΞΠ€>0.Π§ΡΠΎΠ±Ρ ΠΊΠΎΠΌΠΏΠ΅Π½ΡΠΈΡΠΎΠ²Π°ΡΡ ΡΡΠΎ ΠΈΠ·ΠΌΠ΅Π½Π΅Π½ΠΈΠ΅ (ΡΠ²Π΅Π»ΠΈΡΠ΅Π½ΠΈΠ΅) Π²Π½Π΅ΡΠ½Π΅Π³ΠΎ ΠΏΠΎΠ»Ρ, Π½Π΅ΠΎΠ±Ρ ΠΎΠ΄ΠΈΠΌΠΎ ΠΌΠ°Π³Π½ΠΈΡΠ½ΠΎΠ΅ ΠΏΠΎΠ»Π΅, Π½Π°ΠΏΡΠ°Π²Π»Π΅Π½Π½ΠΎΠ΅ Π² ΡΡΠΎΡΠΎΠ½Ρ, ΠΏΡΠΎΡΠΈΠ²ΠΎΠΏΠΎΠ»ΠΎΠΆΠ½ΡΡ Π²Π½Π΅ΡΠ½Π΅ΠΌΡ ΠΏΠΎΠ»Ρ:Β , Π³Π΄Π΅Β Β β Ρ.Π½. ΠΈΠ½Π΄ΡΠΊΡΠΈΠΎΠ½Π½ΠΎΠ΅ ΠΌΠ°Π³Π½ΠΈΡΠ½ΠΎΠ΅ ΠΏΠΎΠ»Π΅. Π ΡΠ»ΡΡΠ°Π΅ 2: ΠΏΡΠΈ ΡΠ΄Π°Π»Π΅Π½ΠΈΠΈ ΠΌΠ°Π³Π½ΠΈΡΠ°, ΡΠΌΠ΅Π½ΡΡΠ΅Π½ΠΈΠΈ ΡΠΎΠΊΠ°, ΡΠ°Π·ΠΌΡΠΊΠ°Π½ΠΈΠΈ ΡΠ΅ΠΏΠΈ:Β .Β ΠΠ°Π³Π½ΠΈΡΠ½ΡΠΉ ΠΏΠΎΡΠΎΠΊΒ Π€Β Β βΒ ΞΠ€0). Π’ΠΎΠΊ Π² ΠΊΠΎΠ½ΡΡΡΠ΅ ΠΈΠΌΠ΅Π΅Ρ ΠΏΠΎΠ»ΠΎΠΆΠΈΡΠ΅Π»ΡΠ½ΠΎΠ΅ Π½Π°ΠΏΡΠ°Π²Π»Π΅Π½ΠΈΠ΅ (), Π΅ΡΠ»ΠΈΒ Β ΡΠΎΠ²ΠΏΠ°ΒΠ΄Π°Π΅ΡΒ ΡΒ ,Β Β Β (Ρ.Π΅.Β ΞΞ¦ |
ΠΡΡΠΎΡΠ½ΠΈΠΊ: https://www.eduspb.com/node/1776
ΠΠ°Π³Π½ΠΈΡΠ½ΡΠΉ ΠΏΠΎΡΠΎΠΊ
Π‘ΡΠ±Π±ΠΎΡΠΈΠ½ Π.Π.
ΠΠ°
ΠΊΠ°ΡΡΠΈΠ½ΠΊΠ΅ ΠΏΠΎΠΊΠ°Π·Π°Π½ΠΎ ΠΎΠ΄Π½ΠΎΡΠΎΠ΄Π½ΠΎΠ΅ ΠΌΠ°Π³Π½ΠΈΡΠ½ΠΎΠ΅
ΠΏΠΎΠ»Π΅. ΠΠ΄Π½ΠΎΡΠΎΠ΄Π½ΠΎΠ΅ ΠΎΠ·Π½Π°ΡΠ°Π΅Ρ ΠΎΠ΄ΠΈΠ½Π°ΠΊΠΎΠ²ΠΎΠ΅
Π²ΠΎ Π²ΡΠ΅Ρ
ΡΠΎΡΠΊΠ°Ρ
Π² Π΄Π°Π½Π½ΠΎΠΌ ΠΎΠ±ΡΠ΅ΠΌΠ΅. Π ΠΏΠΎΠ»Π΅
ΠΏΠΎΠΌΠ΅ΡΠ΅Π½Π° ΠΏΠΎΠ²Π΅ΡΡ
Π½ΠΎΡΡΡ Ρ ΠΏΠ»ΠΎΡΠ°Π΄ΡΡ S. ΠΠΈΠ½ΠΈΠΈ
ΠΏΠΎΠ»Ρ ΠΏΠ΅ΡΠ΅ΡΠ΅ΠΊΠ°ΡΡ ΠΏΠΎΠ²Π΅ΡΡ
Π½ΠΎΡΡΡ.
- ΠΠΏΡΠ΅Π΄Π΅Π»Π΅Π½ΠΈΠ΅
ΠΌΠ°Π³Π½ΠΈΡΠ½ΠΎΠ³ΠΎ ΠΏΠΎΡΠΎΠΊΠ°: - ΠΠ°Π³Π½ΠΈΡΠ½ΡΠΌ
ΠΏΠΎΡΠΎΠΊΠΎΠΌ Π€ ΡΠ΅ΡΠ΅Π· ΠΏΠΎΠ²Π΅ΡΡ Π½ΠΎΡΡΡ S Π½Π°Π·ΡΠ²Π°ΡΡ
ΠΊΠΎΠ»ΠΈΡΠ΅ΡΡΠ²ΠΎ Π»ΠΈΠ½ΠΈΠΉ Π²Π΅ΠΊΡΠΎΡΠ° ΠΌΠ°Π³Π½ΠΈΡΠ½ΠΎΠΉ
ΠΈΠ½Π΄ΡΠΊΡΠΈΠΈ B, ΠΏΡΠΎΡ ΠΎΠ΄ΡΡΠΈΡ ΡΠ΅ΡΠ΅Π· ΠΏΠΎΠ²Π΅ΡΡ Π½ΠΎΡΡΡ
S. - Π€ΠΎΡΠΌΡΠ»Π°
ΠΌΠ°Π³Π½ΠΈΡΠ½ΠΎΠ³ΠΎ ΠΏΠΎΡΠΎΠΊΠ°: - Π€
= BS cos Ξ± - Π·Π΄Π΅ΡΡ
Ξ± β ΡΠ³ΠΎΠ» ΠΌΠ΅ΠΆΠ΄Ρ Π½Π°ΠΏΡΠ°Π²Π»Π΅Π½ΠΈΠ΅ΠΌ Π²Π΅ΠΊΡΠΎΡΠ°
ΠΌΠ°Π³Π½ΠΈΡΠ½ΠΎΠΉ ΠΈΠ½Π΄ΡΠΊΡΠΈΠΈ B ΠΈ Π½ΠΎΡΠΌΠ°Π»ΡΡ ΠΊ
ΠΏΠΎΠ²Π΅ΡΡ Π½ΠΎΡΡΠΈ S.
ΠΠ·
ΡΠΎΡΠΌΡΠ»Ρ ΠΌΠ°Π³Π½ΠΈΡΠ½ΠΎΠ³ΠΎ ΠΏΠΎΡΠΎΠΊΠ° Π²ΠΈΠ΄Π½ΠΎ, ΡΡΠΎ
ΠΌΠ°ΠΊΡΠΈΠΌΠ°Π»ΡΠ½ΡΠΌ ΠΌΠ°Π³Π½ΠΈΡΠ½ΡΠΉ ΠΏΠΎΡΠΎΠΊ Π±ΡΠ΄Π΅Ρ ΠΏΡΠΈ
cos Ξ± = 1, Π° ΡΡΠΎ ΡΠ»ΡΡΠΈΡΡΡ, ΠΊΠΎΠ³Π΄Π° Π²Π΅ΠΊΡΠΎΡ B
ΠΏΠ°ΡΠ°Π»Π»Π΅Π»Π΅Π½ Π½ΠΎΡΠΌΠ°Π»ΠΈ ΠΊ ΠΏΠΎΠ²Π΅ΡΡ
Π½ΠΎΡΡΠΈ S.
ΠΠΈΠ½ΠΈΠΌΠ°Π»ΡΠ½ΡΠΌ ΠΌΠ°Π³Π½ΠΈΡΠ½ΡΠΉ ΠΏΠΎΡΠΎΠΊ Π±ΡΠ΄Π΅Ρ ΠΏΡΠΈ
cos Ξ± = 0, ΡΡΠΎ Π±ΡΠ΄Π΅Ρ, ΠΊΠΎΠ³Π΄Π° Π²Π΅ΠΊΡΠΎΡ B
ΠΏΠ΅ΡΠΏΠ΅Π½Π΄ΠΈΠΊΡΠ»ΡΡΠ΅Π½ Π½ΠΎΡΠΌΠ°Π»ΠΈ ΠΊ ΠΏΠΎΠ²Π΅ΡΡ
Π½ΠΎΡΡΠΈ
S, Π²Π΅Π΄Ρ Π² ΡΡΠΎΠΌ ΡΠ»ΡΡΠ°Π΅ Π»ΠΈΠ½ΠΈΠΈ Π²Π΅ΠΊΡΠΎΡΠ° B
Π±ΡΠ΄ΡΡ ΡΠΊΠΎΠ»ΡΠ·ΠΈΡΡ ΠΏΠΎ ΠΏΠΎΠ²Π΅ΡΡ
Π½ΠΎΡΡΠΈ S, Π½Π΅
ΠΏΠ΅ΡΠ΅ΡΠ΅ΠΊΠ°Ρ Π΅Ρ.
Π
ΠΏΠΎ ΠΎΠΏΡΠ΅Π΄Π΅Π»Π΅Π½ΠΈΡ ΠΌΠ°Π³Π½ΠΈΡΠ½ΠΎΠ³ΠΎ ΠΏΠΎΡΠΎΠΊΠ°
ΡΡΠΈΡΡΠ²Π°ΡΡΡΡ ΡΠΎΠ»ΡΠΊΠΎ ΡΠ΅ Π»ΠΈΠ½ΠΈΠΈ Π²Π΅ΠΊΡΠΎΡΠ°
ΠΌΠ°Π³Π½ΠΈΡΠ½ΠΎΠΉ ΠΈΠ½Π΄ΡΠΊΡΠΈΠΈ, ΠΊΠΎΡΠΎΡΡΠ΅ ΠΏΠ΅ΡΠ΅ΡΠ΅ΠΊΠ°ΡΡ
Π΄Π°Π½Π½ΡΡ ΠΏΠΎΠ²Π΅ΡΡ
Π½ΠΎΡΡΡ.
ΠΠ·ΠΌΠ΅ΡΡΠ΅ΡΡΡ
ΠΌΠ°Π³Π½ΠΈΡΠ½ΡΠΉ ΠΏΠΎΡΠΎΠΊ Π² Π²Π΅Π±Π΅ΡΠ°Ρ
(Π²ΠΎΠ»ΡΡ-ΡΠ΅ΠΊΡΠ½Π΄Π°Ρ
):
1 Π²Π± = 1 Π² * Ρ. ΠΡΠΎΠΌΠ΅ ΡΠΎΠ³ΠΎ, Π΄Π»Ρ ΠΈΠ·ΠΌΠ΅ΡΠ΅Π½ΠΈΡ
ΠΌΠ°Π³Π½ΠΈΡΠ½ΠΎΠ³ΠΎ ΠΏΠΎΡΠΎΠΊΠ° ΠΏΡΠΈΠΌΠ΅Π½ΡΡΡ ΠΌΠ°ΠΊΡΠ²Π΅Π»Π»:
1 Π²Π± = 108Β ΠΌΠΊΡ.
Π‘ΠΎΠΎΡΠ²Π΅ΡΡΡΠ²Π΅Π½Π½ΠΎ 1 ΠΌΠΊΡ = 10-8Β Π²Π±.
ΠΠ°Π³Π½ΠΈΡΠ½ΡΠΉ
ΠΏΠΎΡΠΎΠΊ ΡΠ²Π»ΡΠ΅ΡΡΡ ΡΠΊΠ°Π»ΡΡΠ½ΠΎΠΉ Π²Π΅Π»ΠΈΡΠΈΠ½ΠΎΠΉ.
ΠΠΠΠ ΠΠΠ―
ΠΠΠΠΠΠ’ΠΠΠΠ ΠΠΠΠ― Π’ΠΠΠ
ΠΠΎΠΊΡΡΠ³
ΠΏΡΠΎΠ²ΠΎΠ΄Π½ΠΈΠΊΠ° Ρ ΡΠΎΠΊΠΎΠΌ ΡΡΡΠ΅ΡΡΠ²ΡΠ΅Ρ ΠΌΠ°Π³Π½ΠΈΡΠ½ΠΎΠ΅
ΠΏΠΎΠ»Π΅, ΠΊΠΎΡΠΎΡΠΎΠ΅ ΠΎΠ±Π»Π°Π΄Π°Π΅Ρ ΡΠ½Π΅ΡΠ³ΠΈΠ΅ΠΉ.
ΠΡΠΊΡΠ΄Π°
ΠΎΠ½Π° Π±Π΅ΡΠ΅ΡΡΡ? ΠΡΡΠΎΡΠ½ΠΈΠΊ ΡΠΎΠΊΠ°, Π²ΠΊΠ»ΡΡΠ΅Π½Π½ΡΠΉ
Π² ΡΠ».ΡΠ΅ΠΏΡ, ΠΎΠ±Π»Π°Π΄Π°Π΅Ρ Π·Π°ΠΏΠ°ΡΠΎΠΌ ΡΠ½Π΅ΡΠ³ΠΈΠΈ.
Π
ΠΌΠΎΠΌΠ΅Π½Ρ Π·Π°ΠΌΡΠΊΠ°Π½ΠΈΡ ΡΠ».ΡΠ΅ΠΏΠΈ ΠΈΡΡΠΎΡΠ½ΠΈΠΊ ΡΠΎΠΊΠ°
ΡΠ°ΡΡ
ΠΎΠ΄ΡΠ΅Ρ ΡΠ°ΡΡΡ ΡΠ²ΠΎΠ΅ΠΉ ΡΠ½Π΅ΡΠ³ΠΈΠΈ Π½Π°
ΠΏΡΠ΅ΠΎΠ΄ΠΎΠ»Π΅Π½ΠΈΠ΅ Π΄Π΅ΠΉΡΡΠ²ΠΈΡ Π²ΠΎΠ·Π½ΠΈΠΊΠ°ΡΡΠ΅ΠΉ ΠΠΠ‘
ΡΠ°ΠΌΠΎΠΈΠ½Π΄ΡΠΊΡΠΈΠΈ.
ΠΡΠ° ΡΠ°ΡΡΡ ΡΠ½Π΅ΡΠ³ΠΈΠΈ,
Π½Π°Π·ΡΠ²Π°Π΅ΠΌΠ°Ρ ΡΠΎΠ±ΡΡΠ²Π΅Π½Π½ΠΎΠΉ ΡΠ½Π΅ΡΠ³ΠΈΠ΅ΠΉ ΡΠΎΠΊΠ°,
ΠΈ ΠΈΠ΄Π΅Ρ Π½Π° ΠΎΠ±ΡΠ°Π·ΠΎΠ²Π°Π½ΠΈΠ΅ ΠΌΠ°Π³Π½ΠΈΡΠ½ΠΎΠ³ΠΎ
ΠΏΠΎΠ»Ρ. ΠΠ½Π΅ΡΠ³ΠΈΡ ΠΌΠ°Π³Π½ΠΈΡΠ½ΠΎΠ³ΠΎ ΠΏΠΎΠ»Ρ
ΡΠ°Π²Π½Π°Β ΡΠΎΠ±ΡΡΠ²Π΅Π½Π½ΠΎΠΉ
ΡΠ½Π΅ΡΠ³ΠΈΠΈ ΡΠΎΠΊΠ°.
Π‘ΠΎΠ±ΡΡΠ²Π΅Π½Π½Π°Ρ
ΡΠ½Π΅ΡΠ³ΠΈΡ ΡΠΎΠΊΠ° ΡΠΈΡΠ»Π΅Π½Π½ΠΎ ΡΠ°Π²Π½Π° ΡΠ°Π±ΠΎΡΠ΅,
ΠΊΠΎΡΠΎΡΡΡ Π΄ΠΎΠ»ΠΆΠ΅Π½ ΡΠΎΠ²Π΅ΡΡΠΈΡΡ ΠΈΡΡΠΎΡΠ½ΠΈΠΊ ΡΠΎΠΊΠ°
Π΄Π»Ρ ΠΏΡΠ΅ΠΎΠ΄ΠΎΠ»Π΅Π½ΠΈΡ ΠΠΠ‘ ΡΠ°ΠΌΠΎΠΈΠ½Π΄ΡΠΊΡΠΈΠΈ, ΡΡΠΎΠ±Ρ
ΡΠΎΠ·Π΄Π°ΡΡ ΡΠΎΠΊ Π² ΡΠ΅ΠΏΠΈ.
ΠΠ½Π΅ΡΠ³ΠΈΡ
ΠΌΠ°Π³Π½ΠΈΡΠ½ΠΎΠ³ΠΎ ΠΏΠΎΠ»Ρ, ΡΠΎΠ·Π΄Π°Π½Π½ΠΎΠ³ΠΎ ΡΠΎΠΊΠΎΠΌ, ΠΏΡΡΠΌΠΎ
ΠΏΡΠΎΠΏΠΎΡΡΠΈΠΎΠ½Π°Π»ΡΠ½Π° ΠΊΠ²Π°Π΄ΡΠ°ΡΡ ΡΠΈΠ»Ρ ΡΠΎΠΊΠ°.
ΠΡΠ΄Π°
ΠΏΡΠΎΠΏΠ°Π΄Π°Π΅Ρ ΡΠ½Π΅ΡΠ³ΠΈΡ ΠΌΠ°Π³Π½ΠΈΡΠ½ΠΎΠ³ΠΎ ΠΏΠΎΠ»Ρ ΠΏΠΎΡΠ»Π΅
ΠΏΡΠ΅ΠΊΡΠ°ΡΠ΅Π½ΠΈΡ ΡΠΎΠΊΠ°? β Π²ΡΠ΄Π΅Π»ΡΠ΅ΡΡΡ ( ΠΏΡΠΈ
ΡΠ°Π·ΠΌΡΠΊΠ°Π½ΠΈΠΈ ΡΠ΅ΠΏΠΈ Ρ Π΄ΠΎΡΡΠ°ΡΠΎΡΠ½ΠΎ Π±ΠΎΠ»ΡΡΠΎΠΉ
ΡΠΈΠ»ΠΎΠΉ ΡΠΎΠΊΠ° Π²ΠΎΠ·ΠΌΠΎΠΆΠ½ΠΎ Π²ΠΎΠ·Π½ΠΈΠΊΠ½ΠΎΠ²Π΅Π½ΠΈΠ΅ ΠΈΡΠΊΡΡ
ΠΈΠ»ΠΈ Π΄ΡΠ³ΠΈ)
4.1. ΠΠ°ΠΊΠΎΠ½ ΡΠ»Π΅ΠΊΡΡΠΎΠΌΠ°Π³Π½ΠΈΡΠ½ΠΎΠΉ ΠΈΠ½Π΄ΡΠΊΡΠΈΠΈ. Π‘Π°ΠΌΠΎΠΈΠ½Π΄ΡΠΊΡΠΈΡ. ΠΠ½Π΄ΡΠΊΡΠΈΠ²Π½ΠΎΡΡΡ
ΠΡΠ½ΠΎΠ²Π½ΡΠ΅
ΡΠΎΡΠΌΡΠ»Ρ
Β·Β Β Β Β Β Β Β Β Β Β Β Β ΠΠ°ΠΊΠΎΠ½
ΡΠ»Π΅ΠΊΡΡΠΎΠΌΠ°Π³Π½ΠΈΡΠ½ΠΎΠΉ ΠΈΠ½Π΄ΡΠΊΡΠΈΠΈ (Π·Π°ΠΊΠΎΠ½
Π€Π°ΡΠ°Π΄Π΅Ρ):
Π³Π΄Π΅Β βΒ ΡΠ΄ΡΒ ΠΈΠ½Π΄ΡΠΊΡΠΈΠΈ;β
ΠΏΠΎΠ»Π½ΡΠΉ ΠΌΠ°Π³Π½ΠΈΡΠ½ΡΠΉ ΠΏΠΎΡΠΎΠΊ (ΠΏΠΎΡΠΎΠΊΠΎΡΡΠ΅ΠΏΠ»Π΅Π½ΠΈΠ΅).
Β·Β Β Β Β Β Β Β Β Β Β Β Β ΠΠ°Π³Π½ΠΈΡΠ½ΡΠΉ
ΠΏΠΎΡΠΎΠΊ, ΡΠΎΠ·Π΄Π°Π²Π°Π΅ΠΌΡΠΉ ΡΠΎΠΊΠΎΠΌ Π² ΠΊΠΎΠ½ΡΡΡΠ΅,
Π³Π΄Π΅Β Β β
ΠΈΠ½Π΄ΡΠΊΡΠΈΠ²Π½ΠΎΡΡΡ ΠΊΠΎΠ½ΡΡΡΠ°;β
ΡΠΈΠ»Π° ΡΠΎΠΊΠ°.
Β·Β Β Β Β Β Β Β Β Β Β Β Β ΠΠ°ΠΊΠΎΠ½
Π€Π°ΡΠ°Π΄Π΅Ρ ΠΏΡΠΈΠΌΠ΅Π½ΠΈΡΠ΅Π»ΡΠ½ΠΎ ΠΊ ΡΠ°ΠΌΠΎΠΈΠ½Π΄ΡΠΊΡΠΈΠΈ
Β·Β Β Β Β Β Β Β Β Β Β Β Β ΠΠ΄ΡΒ ΠΈΠ½Π΄ΡΠΊΡΠΈΠΈ,Β Π²ΠΎΠ·Π½ΠΈΠΊΠ°ΡΡΠ°ΡΒ ΠΏΡΠΈ
Π²ΡΠ°ΡΠ΅Π½ΠΈΠΈ ΡΠ°ΠΌΠΊΠΈ Ρ ΡΠΎΠΊΠΎΠΌ Π² ΠΌΠ°Π³Π½ΠΈΡΠ½ΠΎΠΌ ΠΏΠΎΠ»Π΅,
Π³Π΄Π΅Β β
ΠΈΠ½Π΄ΡΠΊΡΠΈΡ ΠΌΠ°Π³Π½ΠΈΡΠ½ΠΎΠ³ΠΎ ΠΏΠΎΠ»Ρ;β
ΠΏΠ»ΠΎΡΠ°Π΄Ρ ΡΠ°ΠΌΠΊΠΈ;β
ΡΠ³Π»ΠΎΠ²Π°Ρ ΡΠΊΠΎΡΠΎΡΡΡ Π²ΡΠ°ΡΠ΅Π½ΠΈΡ.
Β·Β Β Β Β Β Β Β Β Β Β Β Β ΠΠ½Π΄ΡΠΊΡΠΈΠ²Π½ΠΎΡΡΡ
ΡΠΎΠ»Π΅Π½ΠΎΠΈΠ΄Π°
Π³Π΄Π΅Β β
ΠΌΠ°Π³Π½ΠΈΡΠ½Π°Ρ ΠΏΠΎΡΡΠΎΡΠ½Π½Π°Ρ;
pngΒ» width=Β»19β³>β
ΠΌΠ°Π³Π½ΠΈΡΠ½Π°Ρ ΠΏΡΠΎΠ½ΠΈΡΠ°Π΅ΠΌΠΎΡΡΡ Π²Π΅ΡΠ΅ΡΡΠ²Π°;β
ΡΠΈΡΠ»ΠΎ Π²ΠΈΡΠΊΠΎΠ² ΡΠΎΠ»Π΅Π½ΠΎΠΈΠ΄Π°;
pngΒ» width=Β»17β³>β
ΠΏΠ»ΠΎΡΠ°Π΄Ρ ΡΠ΅ΡΠ΅Π½ΠΈΡ Π²ΠΈΡΠΊΠ°;β
Π΄Π»ΠΈΠ½Π° ΡΠΎΠ»Π΅Π½ΠΎΠΈΠ΄Π°.
Β·Β Β Β Β Β Β Β Β Β Β Β Β Π‘ΠΈΠ»Π°
ΡΠΎΠΊΠ° ΠΏΡΠΈ ΡΠ°Π·ΠΌΡΠΊΠ°Π½ΠΈΠΈ ΡΠ΅ΠΏΠΈ
Π³Π΄Π΅Β β
ΡΡΡΠ°Π½ΠΎΠ²ΠΈΠ²ΡΠ°ΡΡΡ Π² ΡΠ΅ΠΏΠΈ ΡΠΈΠ»Π° ΡΠΎΠΊΠ°;
pngΒ» width=Β»17β³>β
ΠΈΠ½Π΄ΡΠΊΡΠΈΠ²Π½ΠΎΡΡΡ ΠΊΠΎΠ½ΡΡΡΠ°,β
ΡΠΎΠΏΡΠΎΡΠΈΠ²Π»Π΅Π½ΠΈΠ΅ ΠΊΠΎΠ½ΡΡΡΠ°;
pngΒ» width=Β»11β³>β
Π²ΡΠ΅ΠΌΡ ΡΠ°Π·ΠΌΡΠΊΠ°Π½ΠΈΡ.
Β·Β Β Β Β Β Β Β Β Β Β Β Β Π‘ΠΈΠ»Π°
ΡΠΎΠΊΠ° ΠΏΡΠΈ Π·Π°ΠΌΡΠΊΠ°Π½ΠΈΠΈ ΡΠ΅ΠΏΠΈ
Β·Β Β Β Β Β Β Β Β Β Β Β Β ΠΡΠ΅ΠΌΡ
ΡΠ΅Π»Π°ΠΊΡΠ°ΡΠΈΠΈ
ΠΡΠΈΠΌΠ΅ΡΡ
ΡΠ΅ΡΠ΅Π½ΠΈΡ Π·Π°Π΄Π°Ρ
ΠΡΠΈΠΌΠ΅Ρ
1.
ΠΠ°Π³Π½ΠΈΡΠ½ΠΎΠ΅
ΠΏΠΎΠ»Π΅ ΠΈΠ·ΠΌΠ΅Π½ΡΠ΅ΡΡΡ ΠΏΠΎ Π·Π°ΠΊΠΎΠ½ΡΒ ,
Π³Π΄Π΅=
15 ΠΌΠ’Π»,.
Β Π
ΠΌΠ°Π³Π½ΠΈΡΠ½ΠΎΠ΅ ΠΏΠΎΠ»Π΅ ΠΏΠΎΠΌΠ΅ΡΠ΅Π½ ΠΊΡΡΠ³ΠΎΠ²ΠΎΠΉ
ΠΏΡΠΎΠ²ΠΎΠ΄ΡΡΠΈΠΉ Π²ΠΈΡΠΎΠΊ ΡΠ°Π΄ΠΈΡΡΠΎΠΌΒ =Β 20
ΡΠΌΒ ΠΏΠΎΠ΄ ΡΠ³Π»ΠΎΠΌΠΊ
Π½Π°ΠΏΡΠ°Π²Π»Π΅Π½ΠΈΡ ΠΏΠΎΠ»Ρ (Π² Π½Π°ΡΠ°Π»ΡΠ½ΡΠΉ ΠΌΠΎΠΌΠ΅Π½Ρ
Π²ΡΠ΅ΠΌΠ΅Π½ΠΈ).
ΠΠ°ΠΉΡΠΈΒ ΡΠ΄ΡΒ ΠΈΠ½Π΄ΡΠΊΡΠΈΠΈ,Β Π²ΠΎΠ·Π½ΠΈΠΊΠ°ΡΡΡΡΒ Π²
Π²ΠΈΡΠΊΠ΅ Π² ΠΌΠΎΠΌΠ΅Π½Ρ Π²ΡΠ΅ΠΌΠ΅Π½ΠΈ=
5 Ρ.
Π Π΅ΡΠ΅Π½ΠΈΠ΅
ΠΠΎ
Π·Π°ΠΊΠΎΠ½Ρ ΡΠ»Π΅ΠΊΡΡΠΎΠΌΠ°Π³Π½ΠΈΡΠ½ΠΎΠΉ ΠΈΠ½Π΄ΡΠΊΡΠΈΠΈΒ Π²ΠΎΠ·Π½ΠΈΠΊΠ°ΡΡΠ°ΡΒ Π²
Π²ΠΈΡΠΊΠ΅Β ΡΠ΄ΡΒ ΠΈΠ½Π΄ΡΠΊΡΠΈΠΈΒ ,
Π³Π΄Π΅β
Β ΠΌΠ°Π³Π½ΠΈΡΠ½ΡΠΉ ΠΏΠΎΡΠΎΠΊ, ΡΡΠ΅ΠΏΠ»Π΅Π½Π½ΡΠΉ Π² Π²ΠΈΡΠΊΠ΅.
Π³Π΄Π΅Β β
ΠΏΠ»ΠΎΡΠ°Π΄Ρ Π²ΠΈΡΠΊΠ°,;
pngΒ» width=Β»19β³>βΒ ΡΠ³ΠΎΠ»
ΠΌΠ΅ΠΆΠ΄Ρ Π½Π°ΠΏΡΠ°Π²Π»Π΅Π½ΠΈΠ΅ΠΌ Π²Π΅ΠΊΡΠΎΡΠ° ΠΌΠ°Π³Π½ΠΈΡΠ½ΠΎΠΉ
ΠΈΠ½Π΄ΡΠΊΡΠΈΠΈΠΈ
Π½ΠΎΡΠΌΠ°Π»ΡΡ ΠΊ ΠΊΠΎΠ½ΡΡΡΡ:
pngΒ» width=Β»159β³>.
ΠΠΎΠ΄ΡΡΠ°Π²ΠΈΠΌ
ΡΠΈΡΠ»ΠΎΠ²ΡΠ΅ Π·Π½Π°ΡΠ΅Π½ΠΈΡ:Β =
15 ΠΌΠ’Π»,,=Β 20
ΡΠΌΒ =Β Β =Β 0,2 ΠΌ,.
ΠΡΠΈΠΌΠ΅Ρ 2Π ΠΎΠ΄Π½ΠΎΡΠΎΠ΄Π½ΠΎΠΌ ΠΌΠ°Π³Π½ΠΈΡΠ½ΠΎΠΌ ΠΏΠΎΠ»Π΅ Ρ ΠΈΠ½Π΄ΡΠΊΡΠΈΠ΅ΠΉΒ = 0,2 Π’Π» ΡΠ°ΡΠΏΠΎΠ»ΠΎΠΆΠ΅Π½Π° ΠΏΡΡΠΌΠΎΡΠ³ΠΎΠ»ΡΠ½Π°Ρ ΡΠ°ΠΌΠΊΠ°, ΠΏΠΎΠ΄Π²ΠΈΠΆΠ½Π°Ρ ΡΡΠΎΡΠΎΠ½Π° ΠΊΠΎΡΠΎΡΠΎΠΉ Π΄Π»ΠΈΠ½ΠΎΠΉ=Β 0,2 ΠΌΒ ΠΏΠ΅ΡΠ΅ΠΌΠ΅ΡΠ°Π΅ΡΡΡ ΡΠΎ ΡΠΊΠΎΡΠΎΡΡΡΡ= 25 ΠΌ/ΡΒ ΠΏΠ΅ΡΠΏΠ΅Π½Π΄ΠΈΠΊΡΠ»ΡΡΠ½ΠΎΒ Π»ΠΈΠ½ΠΈΡΠΌΒ ΠΈΠ½Π΄ΡΠΊΡΠΈΠΈ ΠΏΠΎΠ»Ρ (ΡΠΈΡ. 42). ΠΠΏΡΠ΅Π΄Π΅Π»ΠΈΡΡΒ ΡΠ΄ΡΒ ΠΈΠ½Π΄ΡΠΊΡΠΈΠΈ,Β Π²ΠΎΠ·Π½ΠΈΠΊΠ°ΡΡΡΡΒ Π² ΠΊΠΎΠ½ΡΡΡΠ΅.Π Π΅ΡΠ΅Π½ΠΈΠ΅ΠΡΠΈ Π΄Π²ΠΈΠΆΠ΅Π½ΠΈΠΈ ΠΏΡΠΎΠ²ΠΎΠ΄Π½ΠΈΠΊΠ° ΠΠ Π² ΠΌΠ°Π³Π½ΠΈΡΠ½ΠΎΠΌ ΠΏΠΎΠ»Π΅ ΠΏΠ»ΠΎΡΠ°Π΄Ρ ΡΠ°ΠΌΠΊΠΈ ΡΠ²Π΅Π»ΠΈΡΠΈΠ²Π°Π΅ΡΡΡ, ΡΠ»Π΅Π΄ΠΎΠ²Π°ΡΠ΅Π»ΡΠ½ΠΎ, Π²ΠΎΠ·ΡΠ°ΡΡΠ°Π΅Ρ ΠΌΠ°Π³Π½ΠΈΡΠ½ΡΠΉ ΠΏΠΎΡΠΎΠΊ ΡΠΊΠ²ΠΎΠ·Ρ ΡΠ°ΠΌΠΊΡ ΠΈ Π²ΠΎΠ·Π½ΠΈΠΊΠ°Π΅ΡΒ ΡΠ΄ΡΒ ΠΈΠ½Π΄ΡΠΊΡΠΈΠΈ. |
ΠΠΎ
Π·Π°ΠΊΠΎΠ½Ρ Π€Π°ΡΠ°Π΄Π΅ΡΒ ,
Β Π³Π΄Π΅
pngΒ» width=Β»120β³>,
ΡΠΎΠ³Π΄Π°,
Π½ΠΎ
pngΒ» width=Β»58β³>,
ΠΏΠΎΡΡΠΎΠΌΡ.
ΠΠ½Π°ΠΊ
Β«β» показΡΠ²Π°Π΅Ρ, ΡΡΠΎΒ ΡΠ΄ΡΒ ΠΈΠ½Π΄ΡΠΊΡΠΈΠΈ
ΠΈ ΠΈΠ½Π΄ΡΠΊΡΠΈΠΎΠ½Π½ΡΠΉ ΡΠΎΠΊ Π½Π°ΠΏΡΠ°Π²Π»Π΅Π½Ρ ΠΏΡΠΎΡΠΈΠ²
ΡΠ°ΡΠΎΠ²ΠΎΠΉ ΡΡΡΠ΅Π»ΠΊΠΈ.
Π‘ΠΠΠΠΠΠΠ£ΠΠ¦ΠΠ―
ΠΠ°ΠΆΠ΄ΡΠΉ
ΠΏΡΠΎΠ²ΠΎΠ΄Π½ΠΈΠΊ, ΠΏΠΎ ΠΊΠΎΡΠΎΡΠΎΠΌΡ ΠΏΡΠΎΡΠ΅ΠΊΠ°Π΅Ρ ΡΠ».ΡΠΎΠΊ,
Π½Π°Ρ
ΠΎΠ΄ΠΈΡΡΡ Π² ΡΠΎΠ±ΡΡΠ²Π΅Π½Π½ΠΎΠΌ ΠΌΠ°Π³Π½ΠΈΡΠ½ΠΎΠΌ ΠΏΠΎΠ»Π΅.
ΠΡΠΈ
ΠΈΠ·ΠΌΠ΅Π½Π΅Π½ΠΈΠΈ ΡΠΈΠ»Ρ ΡΠΎΠΊΠ° Π² ΠΏΡΠΎΠ²ΠΎΠ΄Π½ΠΈΠΊΠ΅ ΠΌΠ΅Π½ΡΠ΅ΡΡΡ
ΠΌ.ΠΏΠΎΠ»Π΅, Ρ.Π΅. ΠΈΠ·ΠΌΠ΅Π½ΡΠ΅ΡΡΡ ΠΌΠ°Π³Π½ΠΈΡΠ½ΡΠΉ ΠΏΠΎΡΠΎΠΊ,
ΡΠΎΠ·Π΄Π°Π²Π°Π΅ΠΌΡΠΉ ΡΡΠΈΠΌ ΡΠΎΠΊΠΎΠΌ. ΠΠ·ΠΌΠ΅Π½Π΅Π½ΠΈΠ΅
ΠΌΠ°Π³Π½ΠΈΡΠ½ΠΎΠ³ΠΎ ΠΏΠΎΡΠΎΠΊΠ° Π²Π΅Π΄Π΅Ρ Π² Π²ΠΎΠ·Π½ΠΈΠΊΠ½ΠΎΠ²Π΅Π½ΠΈΡ
Π²ΠΈΡ
ΡΠ΅Π²ΠΎΠ³ΠΎ ΡΠ».
ΠΏΠΎΠ»Ρ ΠΈ Π² ΡΠ΅ΠΏΠΈ ΠΏΠΎΡΠ²Π»ΡΠ΅ΡΡΡ
ΠΠΠ‘ ΠΈΠ½Π΄ΡΠΊΡΠΈΠΈ.Β ΠΡΠΎ
ΡΠ²Π»Π΅Π½ΠΈΠ΅ Π½Π°Π·ΡΠ²Π°Π΅ΡΡΡ ΡΠ°ΠΌΠΎΠΈΠ½Π΄ΡΠΊΡΠΈΠ΅ΠΉ.Π‘Π°ΠΌΠΎΠΈΠ½Π΄ΡΠΊΡΠΈΡΒ β
ΡΠ²Π»Π΅Π½ΠΈΠ΅ Π²ΠΎΠ·Π½ΠΈΠΊΠ½ΠΎΠ²Π΅Π½ΠΈΡ ΠΠΠ‘ ΠΈΠ½Π΄ΡΠΊΡΠΈΠΈ Π²
ΡΠ».ΡΠ΅ΠΏΠΈ Π² ΡΠ΅Π·ΡΠ»ΡΡΠ°ΡΠ΅ ΠΈΠ·ΠΌΠ΅Π½Π΅Π½ΠΈΡ ΡΠΈΠ»Ρ
ΡΠΎΠΊΠ°.
ΠΠΎΠ·Π½ΠΈΠΊΠ°ΡΡΠ°Ρ ΠΏΡΠΈ ΡΡΠΎΠΌ ΠΠΠ‘
Π½Π°Π·ΡΠ²Π°Π΅ΡΡΡΒ ΠΠΠ‘
ΡΠ°ΠΌΠΎΠΈΠ½Π΄ΡΠΊΡΠΈΠΈ
ΠΡΠΎΡΠ²Π»Π΅Π½ΠΈΠ΅
ΡΠ²Π»Π΅Π½ΠΈΡ ΡΠ°ΠΌΠΎΠΈΠ½Π΄ΡΠΊΡΠΈΠΈ
ΠΠ°ΠΌΡΠΊΠ°Π½ΠΈΠ΅
ΡΠ΅ΠΏΠΈ ΠΡΠΈ
Π·Π°ΠΌΡΠΊΠ°Π½ΠΈΠΈ Π² ΡΠ».
ΡΠ΅ΠΏΠΈ Π½Π°ΡΠ°ΡΡΠ°Π΅Ρ ΡΠΎΠΊ, ΡΡΠΎ
Π²ΡΠ·ΡΠ²Π°Π΅Ρ Π² ΠΊΠ°ΡΡΡΠΊΠ΅ ΡΠ²Π΅Π»ΠΈΡΠ΅Π½ΠΈΠ΅ ΠΌΠ°Π³Π½ΠΈΡΠ½ΠΎΠ³ΠΎ
ΠΏΠΎΡΠΎΠΊΠ°, Π²ΠΎΠ·Π½ΠΈΠΊΠ°Π΅Ρ Π²ΠΈΡ
ΡΠ΅Π²ΠΎΠ΅ ΡΠ».ΠΏΠΎΠ»Π΅,
Π½Π°ΠΏΡΠ°Π²Π»Π΅Π½Π½ΠΎΠ΅ ΠΏΡΠΎΡΠΈΠ² ΡΠΎΠΊΠ°, Ρ.Π΅.
Π² ΠΊΠ°ΡΡΡΠΊΠ΅
Π²ΠΎΠ·Π½ΠΈΠΊΠ°Π΅Ρ ΠΠΠ‘ ΡΠ°ΠΌΠΎΠΈΠ½Π΄ΡΠΊΡΠΈΠΈ, ΠΏΡΠ΅ΠΏΡΡΡΡΠ²ΡΡΡΠ°Ρ
Π½Π°ΡΠ°ΡΡΠ°Π½ΠΈΡ ΡΠΎΠΊΠ° Π² ΡΠ΅ΠΏΠΈ ( Π²ΠΈΡ
ΡΠ΅Π²ΠΎΠ΅ ΠΏΠΎΠ»Π΅
ΡΠΎΡΠΌΠΎΠ·ΠΈΡ ΡΠ»Π΅ΠΊΡΡΠΎΠ½Ρ).
Π ΡΠ΅Π·ΡΠ»ΡΡΠ°ΡΠ΅Π1
Π·Π°Π³ΠΎΡΠ°Π΅ΡΡΡ ΠΏΠΎΠ·ΠΆΠ΅,Β ΡΠ΅ΠΌ
Π2.
Π Π°Π·ΠΌΡΠΊΠ°Π½ΠΈΠ΅
ΡΠ΅ΠΏΠΈ ΠΡΠΈ
ΡΠ°Π·ΠΌΡΠΊΠ°Π½ΠΈΠΈ ΡΠ».ΡΠ΅ΠΏΠΈ ΡΠΎΠΊ ΡΠ±ΡΠ²Π°Π΅Ρ, Π²ΠΎΠ·Π½ΠΈΠΊΠ°Π΅Ρ
ΡΠΌΠ΅Π½ΡΡΠ΅Π½ΠΈΠ΅ ΠΌ.ΠΏΠΎΡΠΎΠΊΠ° Π² ΠΊΠ°ΡΡΡΠΊΠ΅, Π²ΠΎΠ·Π½ΠΈΠΊΠ°Π΅Ρ
Π²ΠΈΡ
ΡΠ΅Π²ΠΎΠ΅ ΡΠ». ΠΏΠΎΠ»Π΅, Π½Π°ΠΏΡΠ°Π²Π»Π΅Π½Π½ΠΎΠ΅ ΠΊΠ°ΠΊ ΡΠΎΠΊ
( ΡΡΡΠ΅ΠΌΡΡΠ΅Π΅ΡΡ ΡΠΎΡ
ΡΠ°Π½ΠΈΡΡ ΠΏΡΠ΅ΠΆΠ½ΡΡ ΡΠΈΠ»Ρ
ΡΠΎΠΊΠ°) , Ρ.Π΅.
Π² ΠΊΠ°ΡΡΡΠΊΠ΅ Π²ΠΎΠ·Π½ΠΈΠΊΠ°Π΅Ρ ΠΠΠ‘
ΡΠ°ΠΌΠΎΠΈΠ½Π΄ΡΠΊΡΠΈΠΈ, ΠΏΠΎΠ΄Π΄Π΅ΡΠΆΠΈΠ²Π°ΡΡΠ°Ρ ΡΠΎΠΊ Π²
ΡΠ΅ΠΏΠΈ.
Π ΡΠ΅Π·ΡΠ»ΡΡΠ°ΡΠ΅ Π ΠΏΡΠΈ Π²ΡΠΊΠ»ΡΡΠ΅Π½ΠΈΠΈΒ ΡΡΠΊΠΎ
Π²ΡΠΏΡΡ
ΠΈΠ²Π°Π΅Ρ. ΠΡΠ²ΠΎΠ΄ Π²
ΡΠ»Π΅ΠΊΡΡΠΎΡΠ΅Ρ
Π½ΠΈΠΊΠ΅ ΡΠ²Π»Π΅Π½ΠΈΠ΅ ΡΠ°ΠΌΠΎΠΈΠ½Π΄ΡΠΊΡΠΈΠΈ
ΠΏΡΠΎΡΠ²Π»ΡΠ΅ΡΡΡ ΠΏΡΠΈ Π·Π°ΠΌΡΠΊΠ°Π½ΠΈΠΈ ΡΠ΅ΠΏΠΈ (ΡΠ».ΡΠΎΠΊ
Π½Π°ΡΠ°ΡΡΠ°Π΅Ρ ΠΏΠΎΡΡΠ΅ΠΏΠ΅Π½Π½ΠΎ) ΠΈ ΠΏΡΠΈ ΡΠ°Π·ΠΌΡΠΊΠ°Π½ΠΈΠΈ
ΡΠ΅ΠΏΠΈ (ΡΠ».ΡΠΎΠΊ ΠΏΡΠΎΠΏΠ°Π΄Π°Π΅Ρ Π½Π΅ ΡΡΠ°Π·Ρ).
ΠΠΠΠ£ΠΠ’ΠΠΠΠΠ‘Π’Π¬
ΠΠ΄ΠΈΠ½ΠΈΡΡ
ΠΈΠ·ΠΌΠ΅ΡΠ΅Π½ΠΈΡ ΠΈΠ½Π΄ΡΠΊΡΠΈΠ²Π½ΠΎΡΡΠΈΒ Π²
ΡΠΈΡΡΠ΅ΠΌΠ΅ Π‘Π:
ΠΠ½Π΄ΡΠΊΡΠΈΠ²Π½ΠΎΡΡΡ
ΠΊΠ°ΡΡΡΠΊΠΈ Π·Π°Π²ΠΈΡΠΈΡ ΠΎΡ:
ΡΠΈΡΠ»Π° Π²ΠΈΡΠΊΠΎΠ²,
ΡΠ°Π·ΠΌΠ΅ΡΠΎΠ² ΠΈ ΡΠΎΡΠΌΡ ΠΊΠ°ΡΡΡΠΊΠΈ ΠΈ ΠΎΡ ΠΎΡΠ½ΠΎΡΠΈΡΠ΅Π»ΡΠ½ΠΎΠΉ
ΠΌΠ°Π³Π½ΠΈΡΠ½ΠΎΠΉ ΠΏΡΠΎΠ½ΠΈΡΠ°Π΅ΠΌΠΎΡΡΠΈ ΡΡΠ΅Π΄ΡΒ
(
Π²ΠΎΠ·ΠΌΠΎΠΆΠ΅Π½ ΡΠ΅ΡΠ΄Π΅ΡΠ½ΠΈΠΊ).
ΠΠΠ‘
Π‘ΠΠΠΠΠΠΠ£ΠΠ¦ΠΠ
ΠΠΠ‘
ΡΠ°ΠΌΠΎΠΈΠ½Π΄ΡΠΊΡΠΈΠΈ ΠΏΡΠ΅ΠΏΡΡΡΡΠ²ΡΠ΅Ρ Π½Π°ΡΠ°ΡΡΠ°Π½ΠΈΡ
ΡΠΈΠ»Ρ ΡΠΎΠΊΠ° ΠΏΡΠΈ Π²ΠΊΠ»ΡΡΠ΅Π½ΠΈΠΈ ΡΠ΅ΠΏΠΈ ΠΈ ΡΠ±ΡΠ²Π°Π½ΠΈΡ
ΡΠΈΠ»Ρ ΡΠΎΠΊΠ° ΠΏΡΠΈ ΡΠ°Π·ΠΌΡΠΊΠ°Π½ΠΈΠΈ ΡΠ΅ΠΏΠΈ.
ΠΠ»Ρ
Ρ
Π°ΡΠ°ΠΊΡΠ΅ΡΠΈΡΡΠΈΠΊΠΈ Π½Π°ΠΌΠ°Π³Π½ΠΈΡΠ΅Π½Π½ΠΎΡΡΠΈ Π²Π΅ΡΠ΅ΡΡΠ²Π°
Π² ΠΌΠ°Π³Π½ΠΈΡΠ½ΠΎΠΌ ΠΏΠΎΠ»Π΅ ΠΈΡΠΏΠΎΠ»ΡΠ·ΡΠ΅ΡΡΡΠΌΠ°Π³Π½ΠΈΡΠ½ΡΠΉ
ΠΌΠΎΠΌΠ΅Π½Ρ (Π ΠΌ).Β ΠΠ½
ΡΠΈΡΠ»Π΅Π½Π½ΠΎ ΡΠ°Π²Π΅Π½ ΠΌΠ΅Ρ
Π°Π½ΠΈΡΠ΅ΡΠΊΠΎΠΌΡ ΠΌΠΎΠΌΠ΅Π½ΡΡ,
ΠΈΡΠΏΡΡΡΠ²Π°Π΅ΠΌΠΎΠΌΡ Π²Π΅ΡΠ΅ΡΡΠ²ΠΎΠΌ Π² ΠΌΠ°Π³Π½ΠΈΡΠ½ΠΎΠΌ
ΠΏΠΎΠ»Π΅ Ρ ΠΈΠ½Π΄ΡΠΊΡΠΈΠ΅ΠΉ Π² 1 Π’Π».
- ΠΠ°Π³Π½ΠΈΡΠ½ΡΠΉ
ΠΌΠΎΠΌΠ΅Π½Ρ Π΅Π΄ΠΈΠ½ΠΈΡΡ ΠΎΠ±ΡΠ΅ΠΌΠ° Π²Π΅ΡΠ΅ΡΡΠ²Π°
Ρ Π°ΡΠ°ΠΊΡΠ΅ΡΠΈΠ·ΡΠ΅Ρ Π΅Π³ΠΎΒ Π½Π°ΠΌΠ°Π³Π½ΠΈΡΠ΅Π½Π½ΠΎΡΡΡ
β I,
ΠΎΠΏΡΠ΅Π΄Π΅Π»ΡΠ΅ΡΡΡ ΠΏΠΎ ΡΠΎΡΠΌΡΠ»Π΅: - I=Β Π ΠΌΒ /V,
(2.4) - Π³Π΄Π΅Β VΒ β
ΠΎΠ±ΡΠ΅ΠΌ Π²Π΅ΡΠ΅ΡΡΠ²Π°. - ΠΠ°ΠΌΠ°Π³Π½ΠΈΡΠ΅Π½Π½ΠΎΡΡΡ
Π² ΡΠΈΡΡΠ΅ΠΌΠ΅ Π‘Π ΠΈΠ·ΠΌΠ΅ΡΡΠ΅ΡΡΡ, ΠΊΠ°ΠΊ ΠΈ Π½Π°ΠΏΡΡΠΆΠ΅Π½Π½ΠΎΡΡΡ,
Π²Β Π/ΠΌ,
Π²Π΅Π»ΠΈΡΠΈΠ½Π° Π²Π΅ΠΊΡΠΎΡΠ½Π°Ρ. - ΠΠ°Π³Π½ΠΈΡΠ½ΡΠ΅
ΡΠ²ΠΎΠΉΡΡΠ²Π° Π²Π΅ΡΠ΅ΡΡΠ² Ρ Π°ΡΠ°ΠΊΡΠ΅ΡΠΈΠ·ΡΡΡΡΡΒ ΠΎΠ±ΡΠ΅ΠΌΠ½ΠΎΠΉ
ΠΌΠ°Π³Π½ΠΈΡΠ½ΠΎΠΉ Π²ΠΎΡΠΏΡΠΈΠΈΠΌΡΠΈΠ²ΠΎΡΡΡΡΒ βΒ cΠΎΒ ,Β Π²Π΅Π»ΠΈΡΠΈΠ½Π°
Π±Π΅Π·ΡΠ°Π·ΠΌΠ΅ΡΠ½Π°Ρ.
ΠΡΠ»ΠΈ
ΠΊΠ°ΠΊΠΎΠ΅-Π»ΠΈΠ±ΠΎ ΡΠ΅Π»ΠΎ ΠΏΠΎΠΌΠ΅ΡΡΠΈΡΡ Π² ΠΌΠ°Π³Π½ΠΈΡΠ½ΠΎΠ΅
ΠΏΠΎΠ»Π΅ Ρ ΠΈΠ½Π΄ΡΠΊΡΠΈΠ΅ΠΉΒ Π,
ΡΠΎ ΠΏΡΠΎΠΈΡΡ
ΠΎΠ΄ΠΈΡ Π΅Π³ΠΎ Π½Π°ΠΌΠ°Π³Π½ΠΈΡΠΈΠ²Π°Π½ΠΈΠ΅.
ΠΡΠ»Π΅Π΄ΡΡΠ²ΠΈΠ΅ ΡΡΠΎΠ³ΠΎ ΡΠ΅Π»ΠΎ ΡΠΎΠ·Π΄Π°Π΅Ρ ΡΠ²ΠΎΠ΅
ΡΠΎΠ±ΡΡΠ²Π΅Π½Π½ΠΎΠ΅ ΠΌΠ°Π³Π½ΠΈΡΠ½ΠΎΠ΅ ΠΏΠΎΠ»Π΅ Ρ ΠΈΠ½Π΄ΡΠΊΡΠΈΠ΅ΠΉΒ Π‘,
ΠΊΠΎΡΠΎΡΠΎΠ΅ Π²Π·Π°ΠΈΠΌΠΎΠ΄Π΅ΠΉΡΡΠ²ΡΠ΅Ρ Ρ Π½Π°ΠΌΠ°Π³Π½ΠΈΡΠΈΠ²Π°ΡΡΠΈΠΌ
ΠΏΠΎΠ»Π΅ΠΌ.
- Π
ΡΡΠΎΠΌ ΡΠ»ΡΡΠ°Π΅ Π²Π΅ΠΊΡΠΎΡ ΠΈΠ½Π΄ΡΠΊΡΠΈΠΈ Π² ΡΡΠ΅Π΄Π΅Β (Π)Π±ΡΠ΄Π΅Ρ
ΡΠ»Π°Π³Π°ΡΡΡΡ ΠΈΠ· Π²Π΅ΠΊΡΠΎΡΠΎΠ²: - Π
= ΠΒ +
Π‘(Π·Π½Π°ΠΊ
Π²Π΅ΠΊΡΠΎΡΠ° ΠΎΠΏΡΡΠ΅Π½), (2. 5) - Π³Π΄Π΅Β Π‘Β βΠΈΠ½Π΄ΡΠΊΡΠΈΡ
ΡΠΎΠ±ΡΡΠ²Π΅Π½Π½ΠΎΠ³ΠΎ ΠΌΠ°Π³Π½ΠΈΡΠ½ΠΎΠ³ΠΎ ΠΏΠΎΠ»Ρ
Π½Π°ΠΌΠ°Π³Π½ΠΈΡΠΈΠ²ΡΠ΅Π³ΠΎΡΡ Π²Π΅ΡΠ΅ΡΡΠ²Π°. - ΠΠ½Π΄ΡΠΊΡΠΈΡ
ΡΠΎΠ±ΡΡΠ²Π΅Π½Π½ΠΎΠ³ΠΎ ΠΏΠΎΠ»Ρ ΠΎΠΏΡΠ΅Π΄Π΅Π»ΡΠ΅ΡΡΡ ΠΌΠ°Π³Π½ΠΈΡΠ½ΡΠΌΠΈ
ΡΠ²ΠΎΠΉΡΡΠ²Π°ΠΌΠΈ Π²Π΅ΡΠ΅ΡΡΠ²Π°, ΠΊΠΎΡΠΎΡΡΠ΅ Ρ Π°ΡΠ°ΠΊΡΠ΅ΡΠΈΠ·ΡΡΡΡΡ
ΠΎΠ±ΡΠ΅ΠΌΠ½ΠΎΠΉ ΠΌΠ°Π³Π½ΠΈΡΠ½ΠΎΠΉ Π²ΠΎΡΠΏΡΠΈΠΈΠΌΡΠΈΠ²ΠΎΡΡΡΡ
βΒ cΠΎ,
ΡΠΏΡΠ°Π²Π΅Π΄Π»ΠΈΠ²ΠΎ Π²ΡΡΠ°ΠΆΠ΅Π½ΠΈΠ΅:Π‘Β =Β cΠΎΒ Π0Β (2.6) - Π Π°Π·Π΄Π΅Π»ΠΈΠΌ
Π½Π°Β m0Β Π²ΡΡΠ°ΠΆΠ΅Π½ΠΈΠ΅
(2.6): - Π‘/
mΠΎ=Β cΠΎΒ Π0Β /m - ΠΠΎΠ»ΡΡΠΈΠΌ:Β Π‘Β =Β cΠΎΒ ΠΒ ,Β (2.7)
Π½ΠΎΒ Π‘Β ΠΎΠΏΡΠ΅Π΄Π΅Π»ΡΠ΅Ρ
Π½Π°ΠΌΠ°Π³Π½ΠΈΡΠ΅Π½Π½ΠΎΡΡΡ Π²Π΅ΡΠ΅ΡΡΠ²Π°Β I,
Ρ.Π΅.Β Π‘Β =Β I,
ΡΠΎΠ³Π΄Π° ΠΈΠ· (2.7):
I
= cΠΎΠ.
(2.8)
Π’Π°ΠΊΠΈΠΌ
ΠΎΠ±ΡΠ°Π·ΠΎΠΌ, Π΅ΡΠ»ΠΈ Π²Π΅ΡΠ΅ΡΡΠ²ΠΎ Π½Π°Ρ
ΠΎΠ΄ΠΈΡΡΡ Π²ΠΎ
Π²Π½Π΅ΡΠ½Π΅ΠΌ ΠΌΠ°Π³Π½ΠΈΡΠ½ΠΎΠΌ ΠΏΠΎΠ»Π΅ Ρ Π½Π°ΠΏΡΡΠΆΠ΅Π½Π½ΠΎΡΡΡΡΠ,
ΡΠΎ Π²Π½ΡΡΡΠΈ Π½Π΅Π³ΠΎ ΠΈΠ½Π΄ΡΠΊΡΠΈΡ ΠΎΠΏΡΠ΅Π΄Π΅Π»ΡΠ΅ΡΡΡ
Π²ΡΡΠ°ΠΆΠ΅Π½ΠΈΠ΅ΠΌ:
Π=ΠΒ +
Π‘Β =
mΠΒ +mΠ‘Β =
m0Β (ΠΒ +
I)Β (2. 9)
ΠΠΎΡΠ»Π΅Π΄Π½Π΅Π΅
Π²ΡΡΠ°ΠΆΠ΅Π½ΠΈΠ΅ ΡΡΡΠΎΠ³ΠΎ ΡΠΏΡΠ°Π²Π΅Π΄Π»ΠΈΠ²ΠΎ, ΠΊΠΎΠ³Π΄Π°
ΡΠ΅ΡΠ΄Π΅ΡΠ½ΠΈΠΊ (Π²Π΅ΡΠ΅ΡΡΠ²ΠΎ) Π½Π°Ρ
ΠΎΠ΄ΠΈΡΡΡ ΠΏΠΎΠ»Π½ΠΎΡΡΡΡ
Π²ΠΎ Π²Π½Π΅ΡΠ½Π΅ΠΌ ΠΎΠ΄Π½ΠΎΡΠΎΠ΄Π½ΠΎΠΌ ΠΌΠ°Π³Π½ΠΈΡΠ½ΠΎΠΌ ΠΏΠΎΠ»Π΅
(Π·Π°ΠΌΠΊΠ½ΡΡΡΠΉ ΡΠΎΡ, Π±Π΅ΡΠΊΠΎΠ½Π΅ΡΠ½ΠΎ Π΄Π»ΠΈΠ½Π½ΡΠΉ
ΡΠΎΠ»Π΅Π½ΠΎΠΈΠ΄ ΠΈ Ρ.Π΄.).
ΠΡΡΠΎΡΠ½ΠΈΠΊ: https://studfile.net/preview/5582906/page:11/
ΠΠ°Π³Π½Π΅ΡΠΈΠ·ΠΌ β Π€ΠΈΠ·ΠΈΠΊΠ° β Π’Π΅ΠΎΡΠΈΡ, ΡΠ΅ΡΡΡ, ΡΠΎΡΠΌΡΠ»Ρ ΠΈ Π·Π°Π΄Π°ΡΠΈ β ΠΠ±ΡΡΠ΅Π½ΠΈΠ΅ Π€ΠΈΠ·ΠΈΠΊΠ΅, ΠΠ½Π»Π°ΠΉΠ½ ΠΏΠΎΠ΄Π³ΠΎΡΠΎΠ²ΠΊΠ° ΠΊ Π¦Π’ ΠΈ ΠΠΠ
ΠΡΠ½ΠΎΠ²Π½ΡΠ΅ ΡΠ΅ΠΎΡΠ΅ΡΠΈΡΠ΅ΡΠΊΠΈΠ΅ ΡΠ²Π΅Π΄Π΅Π½ΠΈΡ
Π‘ΠΈΠ»Π° ΠΠΌΠΏΠ΅ΡΠ°
Π ΠΎΠ³Π»Π°Π²Π»Π΅Π½ΠΈΡβ¦
ΠΠ°ΡΡΠΆΠ΅Π½Π½ΡΠ΅ ΡΠ΅Π»Π° ΡΠΏΠΎΡΠΎΠ±Π½Ρ ΡΠΎΠ·Π΄Π°Π²Π°ΡΡ ΠΊΡΠΎΠΌΠ΅ ΡΠ»Π΅ΠΊΡΡΠΈΡΠ΅ΡΠΊΠΎΠ³ΠΎ Π΅ΡΠ΅ ΠΎΠ΄ΠΈΠ½ Π²ΠΈΠ΄ ΠΏΠΎΠ»Ρ. ΠΡΠ»ΠΈ Π·Π°ΡΡΠ΄Ρ Π΄Π²ΠΈΠΆΡΡΡΡ, ΡΠΎ Π² ΠΏΡΠΎΡΡΡΠ°Π½ΡΡΠ²Π΅ Π²ΠΎΠΊΡΡΠ³ Π½ΠΈΡ ΡΠΎΠ·Π΄Π°Π΅ΡΡΡ ΠΎΡΠΎΠ±ΡΠΉ Π²ΠΈΠ΄ ΠΌΠ°ΡΠ΅ΡΠΈΠΈ, Π½Π°Π·ΡΠ²Π°Π΅ΠΌΡΠΉ ΠΌΠ°Π³Π½ΠΈΡΠ½ΡΠΌ ΠΏΠΎΠ»Π΅ΠΌ.
Π‘Π»Π΅Π΄ΠΎΠ²Π°ΡΠ΅Π»ΡΠ½ΠΎ, ΡΠ»Π΅ΠΊΡΡΠΈΡΠ΅ΡΠΊΠΈΠΉ ΡΠΎΠΊ, ΠΏΡΠ΅Π΄ΡΡΠ°Π²Π»ΡΡΡΠΈΠΉ ΡΠΎΠ±ΠΎΠΉ ΡΠΏΠΎΡΡΠ΄ΠΎΡΠ΅Π½Π½ΠΎΠ΅ Π΄Π²ΠΈΠΆΠ΅Π½ΠΈΠ΅ Π·Π°ΡΡΠ΄ΠΎΠ², ΡΠΎΠΆΠ΅ ΡΠΎΠ·Π΄Π°Π΅Ρ ΠΌΠ°Π³Π½ΠΈΡΠ½ΠΎΠ΅ ΠΏΠΎΠ»Π΅. ΠΠ°ΠΊ ΠΈ ΡΠ»Π΅ΠΊΡΡΠΈΡΠ΅ΡΠΊΠΎΠ΅ ΠΏΠΎΠ»Π΅, ΠΌΠ°Π³Π½ΠΈΡΠ½ΠΎΠ΅ ΠΏΠΎΠ»Π΅ Π½Π΅ ΠΎΠ³ΡΠ°Π½ΠΈΡΠ΅Π½ΠΎ Π² ΠΏΡΠΎΡΡΡΠ°Π½ΡΡΠ²Π΅, ΡΠ°ΡΠΏΡΠΎΡΡΡΠ°Π½ΡΠ΅ΡΡΡ ΠΎΡΠ΅Π½Ρ Π±ΡΡΡΡΠΎ, Π½ΠΎ Π²ΡΠ΅ ΠΆΠ΅ Ρ ΠΊΠΎΠ½Π΅ΡΠ½ΠΎΠΉ ΡΠΊΠΎΡΠΎΡΡΡΡ.
ΠΠ³ΠΎ ΠΌΠΎΠΆΠ½ΠΎ ΠΎΠ±Π½Π°ΡΡΠΆΠΈΡΡ ΡΠΎΠ»ΡΠΊΠΎ ΠΏΠΎ Π΄Π΅ΠΉΡΡΠ²ΠΈΡ Π½Π° Π΄Π²ΠΈΠΆΡΡΠΈΠ΅ΡΡ Π·Π°ΡΡΠΆΠ΅Π½Π½ΡΠ΅ ΡΠ΅Π»Π° (ΠΈ, ΠΊΠ°ΠΊ ΡΠ»Π΅Π΄ΡΡΠ²ΠΈΠ΅, ΡΠΎΠΊΠΈ).
ΠΠ»Ρ ΠΎΠΏΠΈΡΠ°Π½ΠΈΡ ΠΌΠ°Π³Π½ΠΈΡΠ½ΠΎΠ³ΠΎ ΠΏΠΎΠ»Ρ Π½Π΅ΠΎΠ±Ρ ΠΎΠ΄ΠΈΠΌΠΎ Π²Π²Π΅ΡΡΠΈ ΡΠΈΠ»ΠΎΠ²ΡΡ Ρ Π°ΡΠ°ΠΊΡΠ΅ΡΠΈΡΡΠΈΠΊΡ ΠΏΠΎΠ»Ρ, Π°Π½Π°Π»ΠΎΠ³ΠΈΡΠ½ΡΡ Π²Π΅ΠΊΡΠΎΡΡ Π½Π°ΠΏΡΡΠΆΠ΅Π½Π½ΠΎΡΡΠΈ E ΡΠ»Π΅ΠΊΡΡΠΈΡΠ΅ΡΠΊΠΎΠ³ΠΎ ΠΏΠΎΠ»Ρ. Π’Π°ΠΊΠΎΠΉ Ρ Π°ΡΠ°ΠΊΡΠ΅ΡΠΈΡΡΠΈΠΊΠΎΠΉ ΡΠ²Π»ΡΠ΅ΡΡΡ Π²Π΅ΠΊΡΠΎΡ B ΠΌΠ°Π³Π½ΠΈΡΠ½ΠΎΠΉ ΠΈΠ½Π΄ΡΠΊΡΠΈΠΈ.
Π ΡΠΈΡΡΠ΅ΠΌΠ΅ Π΅Π΄ΠΈΠ½ΠΈΡ Π‘Π Π·Π° Π΅Π΄ΠΈΠ½ΠΈΡΡ ΠΌΠ°Π³Π½ΠΈΡΠ½ΠΎΠΉ ΠΈΠ½Π΄ΡΠΊΡΠΈΠΈ ΠΏΡΠΈΠ½ΡΡ 1 Π’Π΅ΡΠ»Π° (Π’Π»).
Β ΠΡΠ»ΠΈ Π² ΠΌΠ°Π³Π½ΠΈΡΠ½ΠΎΠ΅ ΠΏΠΎΠ»Π΅ Ρ ΠΈΠ½Π΄ΡΠΊΡΠΈΠ΅ΠΉ B ΠΏΠΎΠΌΠ΅ΡΡΠΈΡΡ ΠΏΡΠΎΠ²ΠΎΠ΄Π½ΠΈΠΊ Π΄Π»ΠΈΠ½ΠΎΠΉ l Ρ ΡΠΎΠΊΠΎΠΌ I, ΡΠΎ Π½Π° Π½Π΅Π³ΠΎ Π±ΡΠ΄Π΅Ρ Π΄Π΅ΠΉΡΡΠ²ΠΎΠ²Π°ΡΡ ΡΠΈΠ»Π°, Π½Π°Π·ΡΠ²Π°Π΅ΠΌΠ°Ρ ΡΠΈΠ»ΠΎΠΉ ΠΠΌΠΏΠ΅ΡΠ°, ΠΊΠΎΡΠΎΡΠ°Ρ Π²ΡΡΠΈΡΠ»ΡΠ΅ΡΡΡ ΠΏΠΎ ΡΠΎΡΠΌΡΠ»Π΅:
Π³Π΄Π΅: Π β ΠΈΠ½Π΄ΡΠΊΡΠΈΡ ΠΌΠ°Π³Π½ΠΈΡΠ½ΠΎΠ³ΠΎ ΠΏΠΎΠ»Ρ, I β ΡΠΈΠ»Π° ΡΠΎΠΊΠ° Π² ΠΏΡΠΎΠ²ΠΎΠ΄Π½ΠΈΠΊΠ΅, l β Π΅Π³ΠΎ Π΄Π»ΠΈΠ½Π°.Β Π‘ΠΈΠ»Π° ΠΠΌΠΏΠ΅ΡΠ° Π½Π°ΠΏΡΠ°Π²Π»Π΅Π½Π° ΠΏΠ΅ΡΠΏΠ΅Π½Π΄ΠΈΠΊΡΠ»ΡΡΠ½ΠΎ Π²Π΅ΠΊΡΠΎΡΡ ΠΌΠ°Π³Π½ΠΈΡΠ½ΠΎΠΉ ΠΈΠ½Π΄ΡΠΊΡΠΈΠΈ ΠΈ Π½Π°ΠΏΡΠ°Π²Π»Π΅Π½ΠΈΡ ΡΠΎΠΊΠ°, ΡΠ΅ΠΊΡΡΠ΅Π³ΠΎ ΠΏΠΎ ΠΏΡΠΎΠ²ΠΎΠ΄Π½ΠΈΠΊΡ.Β
ΠΠ»Ρ ΠΎΠΏΡΠ΅Π΄Π΅Π»Π΅Π½ΠΈΡ Π½Π°ΠΏΡΠ°Π²Π»Π΅Π½ΠΈΡ ΡΠΈΠ»Ρ ΠΠΌΠΏΠ΅ΡΠ° ΠΎΠ±ΡΡΠ½ΠΎ ΠΈΡΠΏΠΎΠ»ΡΠ·ΡΡΡ ΠΏΡΠ°Π²ΠΈΠ»ΠΎ Β«ΠΠ΅Π²ΠΎΠΉ ΡΡΠΊΠΈΒ»: Π΅ΡΠ»ΠΈ ΡΠ°ΡΠΏΠΎΠ»ΠΎΠΆΠΈΡΡ Π»Π΅Π²ΡΡ ΡΡΠΊΡ ΡΠ°ΠΊ, ΡΡΠΎΠ±Ρ Π»ΠΈΠ½ΠΈΠΈ ΠΈΠ½Π΄ΡΠΊΡΠΈΠΈ Π²Ρ ΠΎΠ΄ΠΈΠ»ΠΈ Π² Π»Π°Π΄ΠΎΠ½Ρ, Π° Π²ΡΡΡΠ½ΡΡΡΠ΅ ΠΏΠ°Π»ΡΡΡ Π±ΡΠ»ΠΈ Π½Π°ΠΏΡΠ°Π²Π»Π΅Π½Ρ Π²Π΄ΠΎΠ»Ρ ΡΠΎΠΊΠ°, ΡΠΎ ΠΎΡΠ²Π΅Π΄Π΅Π½Π½ΡΠΉ Π±ΠΎΠ»ΡΡΠΎΠΉ ΠΏΠ°Π»Π΅Ρ ΡΠΊΠ°ΠΆΠ΅Ρ Π½Π°ΠΏΡΠ°Π²Π»Π΅Π½ΠΈΠ΅ ΡΠΈΠ»Ρ ΠΠΌΠΏΠ΅ΡΠ°, Π΄Π΅ΠΉΡΡΠ²ΡΡΡΠ΅ΠΉ Π½Π° ΠΏΡΠΎΠ²ΠΎΠ΄Π½ΠΈΠΊ (ΡΠΌ. ΡΠΈΡΡΠ½ΠΎΠΊ).
ΠΡΠ»ΠΈ ΡΠ³ΠΎΠ» Ξ± ΠΌΠ΅ΠΆΠ΄Ρ Π½Π°ΠΏΡΠ°Π²Π»Π΅Π½ΠΈΡΠΌΠΈ Π²Π΅ΠΊΡΠΎΡΠ° ΠΌΠ°Π³Π½ΠΈΡΠ½ΠΎΠΉ ΠΈΠ½Π΄ΡΠΊΡΠΈΠΈ ΠΈ ΡΠΎΠΊΠ° Π² ΠΏΡΠΎΠ²ΠΎΠ΄Π½ΠΈΠΊΠ΅ ΠΎΡΠ»ΠΈΡΠ΅Π½ ΠΎΡ 90Β°, ΡΠΎ Π΄Π»Ρ ΠΎΠΏΡΠ΅Π΄Π΅Π»Π΅Π½ΠΈΡ Π½Π°ΠΏΡΠ°Π²Π»Π΅Π½ΠΈΡ ΡΠΈΠ»Ρ ΠΠΌΠΏΠ΅ΡΠ° Π½Π°Π΄ΠΎ Π²Π·ΡΡΡ ΡΠΎΡΡΠ°Π²Π»ΡΡΡΡΡ ΠΌΠ°Π³Π½ΠΈΡΠ½ΠΎΠ³ΠΎ ΠΏΠΎΠ»Ρ, ΠΊΠΎΡΠΎΡΠ°Ρ ΠΏΠ΅ΡΠΏΠ΅Π½Π΄ΠΈΠΊΡΠ»ΡΡΠ½Π° Π½Π°ΠΏΡΠ°Π²Π»Π΅Π½ΠΈΡ ΡΠΎΠΊΠ°. Π Π΅ΡΠ°ΡΡ Π·Π°Π΄Π°ΡΠΈ ΡΡΠΎΠΉ ΡΠ΅ΠΌΡ Π½ΡΠΆΠ½ΠΎ ΡΠ°ΠΊ ΠΆΠ΅Β ΠΊΠ°ΠΊ ΠΈ Π² Π΄ΠΈΠ½Π°ΠΌΠΈΠΊΠ΅ ΠΈΠ»ΠΈ ΡΡΠ°ΡΠΈΠΊΠ΅, Ρ.Π΅. ΡΠ°ΡΠΏΠΈΡΠ°Π² ΡΠΈΠ»Ρ ΠΏΠΎ ΠΎΡΡΠΌ ΠΊΠΎΠΎΡΠ΄ΠΈΠ½Π°Ρ ΠΈΠ»ΠΈ ΡΠΊΠ»Π°Π΄ΡΠ²Π°Ρ ΡΠΈΠ»Ρ ΠΏΠΎ ΠΏΡΠ°Π²ΠΈΠ»Π°ΠΌ ΡΠ»ΠΎΠΆΠ΅Π½ΠΈΡ Π²Π΅ΠΊΡΠΎΡΠΎΠ².
ΠΠΎΠΌΠ΅Π½Ρ ΡΠΈΠ», Π΄Π΅ΠΉΡΡΠ²ΡΡΡΠΈΡ Π½Π° ΡΠ°ΠΌΠΊΡ Ρ ΡΠΎΠΊΠΎΠΌ
ΠΡΡΡΡ ΡΠ°ΠΌΠΊΠ° Ρ ΡΠΎΠΊΠΎΠΌ Π½Π°Ρ ΠΎΠ΄ΠΈΡΡΡ Π² ΠΌΠ°Π³Π½ΠΈΡΠ½ΠΎΠΌ ΠΏΠΎΠ»Π΅, ΠΏΡΠΈΡΡΠΌ ΠΏΠ»ΠΎΡΠΊΠΎΡΡΡ ΡΠ°ΠΌΠΊΠΈ ΠΏΠ΅ΡΠΏΠ΅Π½Π΄ΠΈΠΊΡΠ»ΡΡΠ½Π° ΠΏΠΎΠ»Ρ. Π‘ΠΈΠ»Ρ ΠΠΌΠΏΠ΅ΡΠ° Π±ΡΠ΄ΡΡ ΡΠΆΠΈΠΌΠ°ΡΡ ΡΠ°ΠΌΠΊΡ, Π° ΠΈΡ ΡΠ°Π²Π½ΠΎΠ΄Π΅ΠΉΡΡΠ²ΡΡΡΠ°Ρ Π±ΡΠ΄Π΅Ρ ΡΠ°Π²Π½Π° Π½ΡΠ»Ρ.
ΠΡΠ»ΠΈ ΠΏΠΎΠΌΠ΅Π½ΡΡΡ Π½Π°ΠΏΡΠ°Π²Π»Π΅Π½ΠΈΠ΅ ΡΠΎΠΊΠ°, ΡΠΎ ΡΠΈΠ»Ρ ΠΠΌΠΏΠ΅ΡΠ° ΠΏΠΎΠΌΠ΅Π½ΡΡΡ ΡΠ²ΠΎΡ Π½Π°ΠΏΡΠ°Π²Π»Π΅Π½ΠΈΠ΅, ΠΈ ΡΠ°ΠΌΠΊΠ° Π±ΡΠ΄Π΅Ρ Π½Π΅ ΡΠΆΠΈΠΌΠ°ΡΡΡΡ, Π° ΡΠ°ΡΡΡΠ³ΠΈΠ²Π°ΡΡΡΡ.Β ΠΡΠ»ΠΈ Π»ΠΈΠ½ΠΈΠΈ ΠΌΠ°Π³Π½ΠΈΡΠ½ΠΎΠΉ ΠΈΠ½Π΄ΡΠΊΡΠΈΠΈ Π»Π΅ΠΆΠ°Ρ Π² ΠΏΠ»ΠΎΡΠΊΠΎΡΡΠΈ ΡΠ°ΠΌΠΊΠΈ, ΡΠΎ Π²ΠΎΠ·Π½ΠΈΠΊΠ°Π΅Ρ Π²ΡΠ°ΡΠ°ΡΠ΅Π»ΡΠ½ΡΠΉ ΠΌΠΎΠΌΠ΅Π½Ρ ΡΠΈΠ» ΠΠΌΠΏΠ΅ΡΠ°.
ΠΡΠ°ΡΠ°ΡΠ΅Π»ΡΠ½ΡΠΉ ΠΌΠΎΠΌΠ΅Π½Ρ ΡΠΈΠ» ΠΠΌΠΏΠ΅ΡΠ° ΡΠ°Π²Π΅Π½:
Π³Π΄Π΅: S β ΠΏΠ»ΠΎΡΠ°Π΄Ρ ΡΠ°ΠΌΠΊΠΈ, Ξ± β ΡΠ³ΠΎΠ» ΠΌΠ΅ΠΆΠ΄Ρ Π½ΠΎΡΠΌΠ°Π»ΡΡ ΠΊ ΡΠ°ΠΌΠΊΠ΅ ΠΈ Π²Π΅ΠΊΡΠΎΡΠΎΠΌ ΠΌΠ°Π³Π½ΠΈΡΠ½ΠΎΠΉ ΠΈΠ½Π΄ΡΠΊΡΠΈΠΈ (Π½ΠΎΡΠΌΠ°Π»Ρ β Π²Π΅ΠΊΡΠΎΡ, ΠΏΠ΅ΡΠΏΠ΅Π½Π΄ΠΈΠΊΡΠ»ΡΡΠ½ΡΠΉ ΠΏΠ»ΠΎΡΠΊΠΎΡΡΠΈ ΡΠ°ΠΌΠΊΠΈ), N β ΠΊΠΎΠ»ΠΈΡΠ΅ΡΡΠ²ΠΎ Π²ΠΈΡΠΊΠΎΠ², B β ΠΈΠ½Π΄ΡΠΊΡΠΈΡ ΠΌΠ°Π³Π½ΠΈΡΠ½ΠΎΠ³ΠΎ ΠΏΠΎΠ»Ρ, I β ΡΠΈΠ»Π° ΡΠΎΠΊΠ° Π² ΡΠ°ΠΌΠΊΠ΅.
Π‘ΠΈΠ»Π° ΠΠΎΡΠ΅Π½ΡΠ°
Π ΠΎΠ³Π»Π°Π²Π»Π΅Π½ΠΈΡβ¦
Π‘ΠΈΠ»Π° ΠΠΌΠΏΠ΅ΡΠ°, Π΄Π΅ΠΉΡΡΠ²ΡΡΡΠ°Ρ Π½Π° ΠΎΡΡΠ΅Π·ΠΎΠΊ ΠΏΡΠΎΠ²ΠΎΠ΄Π½ΠΈΠΊΠ° Π΄Π»ΠΈΠ½ΠΎΠΉ Ξl Ρ ΡΠΈΠ»ΠΎΠΉ ΡΠΎΠΊΠ° I, Π½Π°Ρ ΠΎΠ΄ΡΡΠΈΠΉΡΡ Π² ΠΌΠ°Π³Π½ΠΈΡΠ½ΠΎΠΌ ΠΏΠΎΠ»Π΅ B ΠΌΠΎΠΆΠ΅Ρ Π±ΡΡΡ Π²ΡΡΠ°ΠΆΠ΅Π½Π° ΡΠ΅ΡΠ΅Π· ΡΠΈΠ»Ρ, Π΄Π΅ΠΉΡΡΠ²ΡΡΡΠΈΠ΅ Π½Π° ΠΎΡΠ΄Π΅Π»ΡΠ½ΡΠ΅ Π½ΠΎΡΠΈΡΠ΅Π»ΠΈ Π·Π°ΡΡΠ΄Π°. ΠΡΠΈ ΡΠΈΠ»Ρ Π½Π°Π·ΡΠ²Π°ΡΡ ΡΠΈΠ»Π°ΠΌΠΈ ΠΠΎΡΠ΅Π½ΡΠ°. Π‘ΠΈΠ»Π° ΠΠΎΡΠ΅Π½ΡΠ°, Π΄Π΅ΠΉΡΡΠ²ΡΡΡΠ°Ρ Π½Π° ΡΠ°ΡΡΠΈΡΡ Ρ Π·Π°ΡΡΠ΄ΠΎΠΌ q Π² ΠΌΠ°Π³Π½ΠΈΡΠ½ΠΎΠΌ ΠΏΠΎΠ»Π΅ B, Π΄Π²ΠΈΠ³Π°ΡΡΡΡΡΡ ΡΠΎ ΡΠΊΠΎΡΠΎΡΡΡΡ v, Π²ΡΡΠΈΡΠ»ΡΠ΅ΡΡΡ ΠΏΠΎ ΡΠ»Π΅Π΄ΡΡΡΠ΅ΠΉ ΡΠΎΡΠΌΡΠ»Π΅:
Π£Π³ΠΎΠ» Ξ± Π² ΡΡΠΎΠΌ Π²ΡΡΠ°ΠΆΠ΅Π½ΠΈΠΈ ΡΠ°Π²Π΅Π½ ΡΠ³Π»Ρ ΠΌΠ΅ΠΆΠ΄Ρ ΡΠΊΠΎΡΠΎΡΡΡΡ ΠΈ Π²Π΅ΠΊΡΠΎΡΠΎΠΌ ΠΌΠ°Π³Π½ΠΈΡΠ½ΠΎΠΉ ΠΈΠ½Π΄ΡΠΊΡΠΈΠΈ. ΠΠ°ΠΏΡΠ°Π²Π»Π΅Π½ΠΈΠ΅ ΡΠΈΠ»Ρ ΠΠΎΡΠ΅Π½ΡΠ°, Π΄Π΅ΠΉΡΡΠ²ΡΡΡΠ΅ΠΉ Π½Π° ΠΏΠΎΠ»ΠΎΠΆΠΈΡΠ΅Π»ΡΠ½ΠΎ Π·Π°ΡΡΠΆΠ΅Π½Π½ΡΡ ΡΠ°ΡΡΠΈΡΡ, ΡΠ°ΠΊ ΠΆΠ΅, ΠΊΠ°ΠΊ ΠΈ Π½Π°ΠΏΡΠ°Π²Π»Π΅Π½ΠΈΠ΅ ΡΠΈΠ»Ρ ΠΠΌΠΏΠ΅ΡΠ°, ΠΌΠΎΠΆΠ΅Ρ Π±ΡΡΡ Π½Π°ΠΉΠ΄Π΅Π½ΠΎ ΠΏΠΎ ΠΏΡΠ°Π²ΠΈΠ»Ρ Π»Π΅Π²ΠΎΠΉ ΡΡΠΊΠΈ ΠΈΠ»ΠΈ ΠΏΠΎ ΠΏΡΠ°Π²ΠΈΠ»Ρ Π±ΡΡΠ°Π²ΡΠΈΠΊΠ° (ΠΊΠ°ΠΊ ΠΈ ΡΠΈΠ»Π° ΠΠΌΠΏΠ΅ΡΠ°).
ΠΠ΅ΠΊΡΠΎΡ ΠΌΠ°Π³Π½ΠΈΡΠ½ΠΎΠΉ ΠΈΠ½Π΄ΡΠΊΡΠΈΠΈ Π½ΡΠΆΠ½ΠΎ ΠΌΡΡΠ»Π΅Π½Π½ΠΎ Π²ΠΎΡΠΊΠ½ΡΡΡ Π² Π»Π°Π΄ΠΎΠ½Ρ Π»Π΅Π²ΠΎΠΉ ΡΡΠΊΠΈ, ΡΠ΅ΡΡΡΠ΅ ΡΠΎΠΌΠΊΠ½ΡΡΡΡ ΠΏΠ°Π»ΡΡΠ° Π½Π°ΠΏΡΠ°Π²ΠΈΡΡ ΠΏΠΎ ΡΠΊΠΎΡΠΎΡΡΠΈ Π΄Π²ΠΈΠΆΠ΅Π½ΠΈΡ Π·Π°ΡΡΠΆΠ΅Π½Π½ΠΎΠΉ ΡΠ°ΡΡΠΈΡΡ, Π° ΠΎΡΠΎΠ³Π½ΡΡΡΠΉ Π±ΠΎΠ»ΡΡΠΎΠΉ ΠΏΠ°Π»Π΅Ρ ΠΏΠΎΠΊΠ°ΠΆΠ΅Ρ Π½Π°ΠΏΡΠ°Π²Π»Π΅Π½ΠΈΠ΅ ΡΠΈΠ»Ρ ΠΠΎΡΠ΅Π½ΡΠ°.
ΠΡΠ»ΠΈ ΡΠ°ΡΡΠΈΡΠ° ΠΈΠΌΠ΅Π΅Ρ ΠΎΡΡΠΈΡΠ°ΡΠ΅Π»ΡΠ½ΡΠΉ Π·Π°ΡΡΠ΄, ΡΠΎ Π½Π°ΠΏΡΠ°Π²Π»Π΅Π½ΠΈΠ΅ ΡΠΈΠ»Ρ ΠΠΎΡΠ΅Π½ΡΠ°, Π½Π°ΠΉΠ΄Π΅Π½Π½ΠΎΠ΅ ΠΏΠΎ ΠΏΡΠ°Π²ΠΈΠ»Ρ Π»Π΅Π²ΠΎΠΉ ΡΡΠΊΠΈ, Π½Π°Π΄ΠΎ Π±ΡΠ΄Π΅Ρ Π·Π°ΠΌΠ΅Π½ΠΈΡΡ Π½Π° ΠΏΡΠΎΡΠΈΠ²ΠΎΠΏΠΎΠ»ΠΎΠΆΠ½ΠΎΠ΅.
Π‘ΠΈΠ»Π° ΠΠΎΡΠ΅Π½ΡΠ° Π½Π°ΠΏΡΠ°Π²Π»Π΅Π½Π° ΠΏΠ΅ΡΠΏΠ΅Π½Π΄ΠΈΠΊΡΠ»ΡΡΠ½ΠΎ Π²Π΅ΠΊΡΠΎΡΠ°ΠΌ ΡΠΊΠΎΡΠΎΡΡΠΈ ΠΈ ΠΈΠ½Π΄ΡΠΊΡΠΈΠΈ ΠΌΠ°Π³Π½ΠΈΡΠ½ΠΎΠ³ΠΎ ΠΏΠΎΠ»Ρ. ΠΡΠΈ Π΄Π²ΠΈΠΆΠ΅Π½ΠΈΠΈ Π·Π°ΡΡΠΆΠ΅Π½Π½ΠΎΠΉ ΡΠ°ΡΡΠΈΡΡ Π² ΠΌΠ°Π³Π½ΠΈΡΠ½ΠΎΠΌ ΠΏΠΎΠ»Π΅ ΡΠΈΠ»Π° ΠΠΎΡΠ΅Π½ΡΠ° ΡΠ°Π±ΠΎΡΡ Π½Π΅ ΡΠΎΠ²Π΅ΡΡΠ°Π΅Ρ. ΠΠΎΡΡΠΎΠΌΡ ΠΌΠΎΠ΄ΡΠ»Ρ Π²Π΅ΠΊΡΠΎΡΠ° ΡΠΊΠΎΡΠΎΡΡΠΈ ΠΏΡΠΈ Π΄Π²ΠΈΠΆΠ΅Π½ΠΈΠΈ ΡΠ°ΡΡΠΈΡΡ Π½Π΅ ΠΈΠ·ΠΌΠ΅Π½ΡΠ΅ΡΡΡ.
ΠΡΠ»ΠΈ Π·Π°ΡΡΠΆΠ΅Π½Π½Π°Ρ ΡΠ°ΡΡΠΈΡΠ° Π΄Π²ΠΈΠΆΠ΅ΡΡΡ Π² ΠΎΠ΄Π½ΠΎΡΠΎΠ΄Π½ΠΎΠΌ ΠΌΠ°Π³Π½ΠΈΡΠ½ΠΎΠΌ ΠΏΠΎΠ»Π΅ ΠΏΠΎΠ΄ Π΄Π΅ΠΉΡΡΠ²ΠΈΠ΅ΠΌ ΡΠΈΠ»Ρ ΠΠΎΡΠ΅Π½ΡΠ°, Π° Π΅Π΅ ΡΠΊΠΎΡΠΎΡΡΡ Π»Π΅ΠΆΠΈΡ Π² ΠΏΠ»ΠΎΡΠΊΠΎΡΡΠΈ, ΠΏΠ΅ΡΠΏΠ΅Π½Π΄ΠΈΠΊΡΠ»ΡΡΠ½ΠΎΠΉ Π²Π΅ΠΊΡΠΎΡΡ ΠΈΠ½Π΄ΡΠΊΡΠΈΠΈ ΠΌΠ°Π³Π½ΠΈΡΠ½ΠΎΠ³ΠΎ ΠΏΠΎΠ»Ρ, ΡΠΎ ΡΠ°ΡΡΠΈΡΠ° Π±ΡΠ΄Π΅Ρ Π΄Π²ΠΈΠ³Π°ΡΡΡΡ ΠΏΠΎ ΠΎΠΊΡΡΠΆΠ½ΠΎΡΡΠΈ, ΡΠ°Π΄ΠΈΡΡ ΠΊΠΎΡΠΎΡΠΎΠΉ ΠΌΠΎΠΆΠ½ΠΎ Π²ΡΡΠΈΡΠ»ΠΈΡΡ ΠΏΠΎ ΡΠ»Π΅Π΄ΡΡΡΠ΅ΠΉ ΡΠΎΡΠΌΡΠ»Π΅:
Π‘ΠΈΠ»Π° ΠΠΎΡΠ΅Π½ΡΠ° Π² ΡΡΠΎΠΌ ΡΠ»ΡΡΠ°Π΅ ΠΈΠ³ΡΠ°Π΅Ρ ΡΠΎΠ»Ρ ΡΠ΅Π½ΡΡΠΎΡΡΡΠ΅ΠΌΠΈΡΠ΅Π»ΡΠ½ΠΎΠΉ ΡΠΈΠ»Ρ. ΠΠ΅ΡΠΈΠΎΠ΄ ΠΎΠ±ΡΠ°ΡΠ΅Π½ΠΈΡ ΡΠ°ΡΡΠΈΡΡ Π² ΠΎΠ΄Π½ΠΎΡΠΎΠ΄Π½ΠΎΠΌ ΠΌΠ°Π³Π½ΠΈΡΠ½ΠΎΠΌ ΠΏΠΎΠ»Π΅ ΡΠ°Π²Π΅Π½:
ΠΠΎΡΠ»Π΅Π΄Π½Π΅Π΅ Π²ΡΡΠ°ΠΆΠ΅Π½ΠΈΠ΅ ΠΏΠΎΠΊΠ°Π·ΡΠ²Π°Π΅Ρ, ΡΡΠΎ Π΄Π»Ρ Π·Π°ΡΡΠΆΠ΅Π½Π½ΡΡ ΡΠ°ΡΡΠΈΡ Π·Π°Π΄Π°Π½Π½ΠΎΠΉ ΠΌΠ°ΡΡΡ m ΠΏΠ΅ΡΠΈΠΎΠ΄ ΠΎΠ±ΡΠ°ΡΠ΅Π½ΠΈΡ (Π° Π·Π½Π°ΡΠΈΡ ΠΈ ΡΠ°ΡΡΠΎΡΠ°, ΠΈ ΡΠ³Π»ΠΎΠ²Π°Ρ ΡΠΊΠΎΡΠΎΡΡΡ) Π½Π΅ Π·Π°Π²ΠΈΡΠΈΡ ΠΎΡ ΡΠΊΠΎΡΠΎΡΡΠΈ (ΡΠ»Π΅Π΄ΠΎΠ²Π°ΡΠ΅Π»ΡΠ½ΠΎ, ΠΈ ΠΎΡ ΠΊΠΈΠ½Π΅ΡΠΈΡΠ΅ΡΠΊΠΎΠΉ ΡΠ½Π΅ΡΠ³ΠΈΠΈ) ΠΈ ΡΠ°Π΄ΠΈΡΡΠ° ΡΡΠ°Π΅ΠΊΡΠΎΡΠΈΠΈ R.
Π’Π΅ΠΎΡΠΈΡ ΠΎ ΠΌΠ°Π³Π½ΠΈΡΠ½ΠΎΠΌ ΠΏΠΎΠ»Π΅
Π ΠΎΠ³Π»Π°Π²Π»Π΅Π½ΠΈΡβ¦
ΠΠ°Π³Π½ΠΈΡΠ½ΠΎΠ΅ Π²Π·Π°ΠΈΠΌΠΎΠ΄Π΅ΠΉΡΡΠ²ΠΈΠ΅ ΡΠΎΠΊΠΎΠ²
ΠΡΠ»ΠΈ ΠΏΠΎ Π΄Π²ΡΠΌ ΠΏΠ°ΡΠ°Π»Π»Π΅Π»ΡΠ½ΡΠΌ ΠΏΡΠΎΠ²ΠΎΠ΄Π°ΠΌ ΠΈΠ΄ΡΡ ΡΠΎΠΊ Π² ΠΎΠ΄Π½ΠΎΠΌ Π½Π°ΠΏΡΠ°Π²Π»Π΅Π½ΠΈΠΈ, ΡΠΎ ΠΎΠ½ΠΈ ΠΏΡΠΈΡΡΠ³ΠΈΠ²Π°ΡΡΡΡ; Π΅ΡΠ»ΠΈ Π² ΠΏΡΠΎΡΠΈΠ²ΠΎΠΏΠΎΠ»ΠΎΠΆΠ½ΡΡ Π½Π°ΠΏΡΠ°Π²Π»Π΅Π½ΠΈΡΡ , ΡΠΎ ΠΎΡΡΠ°Π»ΠΊΠΈΠ²Π°ΡΡΡΡ. ΠΠ°ΠΊΠΎΠ½ΠΎΠΌΠ΅ΡΠ½ΠΎΡΡΠΈ ΡΡΠΎΠ³ΠΎ ΡΠ²Π»Π΅Π½ΠΈΡ Π±ΡΠ»ΠΈ ΡΠΊΡΠΏΠ΅ΡΠΈΠΌΠ΅Π½ΡΠ°Π»ΡΠ½ΠΎ ΡΡΡΠ°Π½ΠΎΠ²Π»Π΅Π½Ρ ΠΠΌΠΏΠ΅ΡΠΎΠΌ.
Β ΠΠ·Π°ΠΈΠΌΠΎΠ΄Π΅ΠΉΡΡΠ²ΠΈΠ΅ ΡΠΎΠΊΠΎΠ² Π²ΡΠ·ΡΠ²Π°Π΅ΡΡΡ ΠΈΡ ΠΌΠ°Π³Π½ΠΈΡΠ½ΡΠΌΠΈ ΠΏΠΎΠ»ΡΠΌΠΈ: ΠΌΠ°Π³Π½ΠΈΡΠ½ΠΎΠ΅ ΠΏΠΎΠ»Π΅ ΠΎΠ΄Π½ΠΎΠ³ΠΎ ΡΠΎΠΊΠ° Π΄Π΅ΠΉΡΡΠ²ΡΠ΅Ρ ΡΠΈΠ»ΠΎΠΉ ΠΠΌΠΏΠ΅ΡΠ° Π½Π° Π΄ΡΡΠ³ΠΎΠΉ ΡΠΎΠΊ ΠΈ Π½Π°ΠΎΠ±ΠΎΡΠΎΡ.
Β ΠΠΏΡΡΡ ΠΏΠΎΠΊΠ°Π·Π°Π»ΠΈ, ΡΡΠΎ ΠΌΠΎΠ΄ΡΠ»Ρ ΡΠΈΠ»Ρ, Π΄Π΅ΠΉΡΡΠ²ΡΡΡΠ΅ΠΉ Π½Π° ΠΎΡΡΠ΅Π·ΠΎΠΊ Π΄Π»ΠΈΠ½ΠΎΠΉ Ξl ΠΊΠ°ΠΆΠ΄ΠΎΠ³ΠΎ ΠΈΠ· ΠΏΡΠΎΠ²ΠΎΠ΄Π½ΠΈΠΊΠΎΠ², ΠΏΡΡΠΌΠΎ ΠΏΡΠΎΠΏΠΎΡΡΠΈΠΎΠ½Π°Π»Π΅Π½ ΡΠΈΠ»Π°ΠΌ ΡΠΎΠΊΠ° I1 ΠΈ I2 Π² ΠΏΡΠΎΠ²ΠΎΠ΄Π½ΠΈΠΊΠ°Ρ , Π΄Π»ΠΈΠ½Π΅ ΠΎΡΡΠ΅Π·ΠΊΠ° Ξl ΠΈ ΠΎΠ±ΡΠ°ΡΠ½ΠΎ ΠΏΡΠΎΠΏΠΎΡΡΠΈΠΎΠ½Π°Π»Π΅Π½ ΡΠ°ΡΡΡΠΎΡΠ½ΠΈΡ R ΠΌΠ΅ΠΆΠ΄Ρ Π½ΠΈΠΌΠΈ:
Π³Π΄Π΅: ΞΌ0 β ΠΏΠΎΡΡΠΎΡΠ½Π½Π°Ρ Π²Π΅Π»ΠΈΡΠΈΠ½Π°, ΠΊΠΎΡΠΎΡΡΡ Π½Π°Π·ΡΠ²Π°ΡΡ ΠΌΠ°Π³Π½ΠΈΡΠ½ΠΎΠΉ ΠΏΠΎΡΡΠΎΡΠ½Π½ΠΎΠΉ. ΠΠ²Π΅Π΄Π΅Π½ΠΈΠ΅ ΠΌΠ°Π³Π½ΠΈΡΠ½ΠΎΠΉ ΠΏΠΎΡΡΠΎΡΠ½Π½ΠΎΠΉ Π² Π‘Π ΡΠΏΡΠΎΡΠ°Π΅Ρ Π·Π°ΠΏΠΈΡΡ ΡΡΠ΄Π° ΡΠΎΡΠΌΡΠ». ΠΠ΅ ΡΠΈΡΠ»Π΅Π½Π½ΠΎΠ΅ Π·Π½Π°ΡΠ΅Π½ΠΈΠ΅ ΡΠ°Π²Π½ΠΎ:
ΞΌ0 = 4ΟΒ·10β7 H/A2 β 1,26Β·10β6 H/A2.
Π‘ΡΠ°Π²Π½ΠΈΠ²Π°Ρ ΠΏΡΠΈΠ²Π΅Π΄Π΅Π½Π½ΠΎΠ΅ ΡΠΎΠ»ΡΠΊΠΎ ΡΡΠΎ Π²ΡΡΠ°ΠΆΠ΅Π½ΠΈΠ΅ Π΄Π»Ρ ΡΠΈΠ»Ρ Π²Π·Π°ΠΈΠΌΠΎΠ΄Π΅ΠΉΡΡΠ²ΠΈΡ Π΄Π²ΡΡ ΠΏΡΠΎΠ²ΠΎΠ΄Π½ΠΈΠΊΠΎΠ² Ρ ΡΠΎΠΊΠΎΠΌ ΠΈ Π²ΡΡΠ°ΠΆΠ΅Π½ΠΈΠ΅ Π΄Π»Ρ ΡΠΈΠ»Ρ ΠΠΌΠΏΠ΅ΡΠ° Π½Π΅ΡΡΡΠ΄Π½ΠΎ ΠΏΠΎΠ»ΡΡΠΈΡΡ Π²ΡΡΠ°ΠΆΠ΅Π½ΠΈΠ΅ Π΄Π»Ρ ΠΈΠ½Π΄ΡΠΊΡΠΈΠΈ ΠΌΠ°Π³Π½ΠΈΡΠ½ΠΎΠ³ΠΎ ΠΏΠΎΠ»Ρ ΡΠΎΠ·Π΄Π°Π²Π°Π΅ΠΌΠΎΠ³ΠΎ ΠΊΠ°ΠΆΠ΄ΡΠΌ ΠΈΠ· ΠΏΡΡΠΌΠΎΠ»ΠΈΠ½Π΅ΠΉΠ½ΡΡ ΠΏΡΠΎΠ²ΠΎΠ΄Π½ΠΈΠΊΠΎΠ² Ρ ΡΠΎΠΊΠΎΠΌ Π½Π° ΡΠ°ΡΡΡΠΎΡΠ½ΠΈΠΈ R ΠΎΡ Π½Π΅Π³ΠΎ:
Π³Π΄Π΅: ΞΌ β ΠΌΠ°Π³Π½ΠΈΡΠ½Π°Ρ ΠΏΡΠΎΠ½ΠΈΡΠ°Π΅ΠΌΠΎΡΡΡ Π²Π΅ΡΠ΅ΡΡΠ²Π° (ΠΎΠ± ΡΡΠΎΠΌ ΡΡΡΡ Π½ΠΈΠΆΠ΅). ΠΡΠ»ΠΈ ΡΠΎΠΊ ΠΏΡΠΎΡΠ΅ΠΊΠ°Π΅Ρ ΠΏΠΎ ΠΊΡΡΠ³ΠΎΠ²ΠΎΠΌΡ Π²ΠΈΡΠΊΡ, ΡΠΎ Π² ΡΠ΅Π½ΡΡΠ΅ Π²ΠΈΡΠΊΠ° ΠΈΠ½Π΄ΡΠΊΡΠΈΡ ΠΌΠ°Π³Π½ΠΈΡΠ½ΠΎΠ³ΠΎ ΠΏΠΎΠ»Ρ ΠΎΠΏΡΠ΅Π΄Π΅Π»ΡΠ΅ΡΡΡ ΠΏΠΎ ΡΠΎΡΠΌΡΠ»Π΅:
Π‘ΠΈΠ»ΠΎΠ²ΡΠΌΠΈ Π»ΠΈΠ½ΠΈΡΠΌΠΈ ΠΌΠ°Π³Π½ΠΈΡΠ½ΠΎΠ³ΠΎ ΠΏΠΎΠ»Ρ Π½Π°Π·ΡΠ²Π°ΡΡ Π»ΠΈΠ½ΠΈΠΈ, ΠΏΠΎ ΠΊΠ°ΡΠ°ΡΠ΅Π»ΡΠ½ΡΠΌ ΠΊ ΠΊΠΎΡΠΎΡΡΠΌ ΡΠ°ΡΠΏΠΎΠ»Π°Π³Π°ΡΡΡΡ ΠΌΠ°Π³Π½ΠΈΡΠ½ΡΠ΅ ΡΡΡΠ΅Π»ΠΊΠΈ.Β ΠΠ°Π³Π½ΠΈΡΠ½ΠΎΠΉ ΡΡΡΠ΅Π»ΠΊΠΎΠΉ Π½Π°Π·ΡΠ²Π°ΡΡ Π΄Π»ΠΈΠ½Π½ΡΠΉ ΠΈ ΡΠΎΠ½ΠΊΠΈΠΉ ΠΌΠ°Π³Π½ΠΈΡ, Π΅Π³ΠΎ ΠΏΠΎΠ»ΡΡΠ° ΡΠΎΡΠ΅ΡΠ½Ρ. ΠΠΎΠ΄Π²Π΅ΡΠ΅Π½Π½Π°Ρ Π½Π° Π½ΠΈΡΠΈ ΠΌΠ°Π³Π½ΠΈΡΠ½Π°Ρ ΡΡΡΠ΅Π»ΠΊΠ° Π²ΡΠ΅Π³Π΄Π° ΠΏΠΎΠ²ΠΎΡΠ°ΡΠΈΠ²Π°Π΅ΡΡΡ Π² ΠΎΠ΄Π½Ρ ΡΡΠΎΡΠΎΠ½Ρ. ΠΡΠΈ ΡΡΠΎΠΌ ΠΎΠ΄ΠΈΠ½ Π΅Ρ ΠΊΠΎΠ½Π΅Ρ Π½Π°ΠΏΡΠ°Π²Π»Π΅Π½ Π² ΡΡΠΎΡΠΎΠ½Ρ ΡΠ΅Π²Π΅ΡΠ°, Π²ΡΠΎΡΠΎΠΉ β Π½Π° ΡΠ³.
ΠΡΡΡΠ΄Π° Π½Π°Π·Π²Π°Π½ΠΈΠ΅ ΠΏΠΎΠ»ΡΡΠΎΠ²: ΡΠ΅Π²Π΅ΡΠ½ΡΠΉ (N) ΠΈ ΡΠΆΠ½ΡΠΉ (S). ΠΠ°Π³Π½ΠΈΡΡ Π²ΡΠ΅Π³Π΄Π° ΠΈΠΌΠ΅ΡΡ Π΄Π²Π° ΠΏΠΎΠ»ΡΡΠ°: ΡΠ΅Π²Π΅ΡΠ½ΡΠΉ (ΠΎΠ±ΠΎΠ·Π½Π°ΡΠ°Π΅ΡΡΡ ΡΠΈΠ½ΠΈΠΌ ΡΠ²Π΅ΡΠΎΠΌ ΠΈΠ»ΠΈ Π±ΡΠΊΠ²ΠΎΠΉ N) ΠΈ ΡΠΆΠ½ΡΠΉ (ΠΊΡΠ°ΡΠ½ΡΠΌ ΡΠ²Π΅ΡΠΎΠΌ ΠΈΠ»ΠΈ Π±ΡΠΊΠ²ΠΎΠΉ S). ΠΠ°Π³Π½ΠΈΡΡ Π²Π·Π°ΠΈΠΌΠΎΠ΄Π΅ΠΉΡΡΠ²ΡΡΡ ΡΠ°ΠΊ ΠΆΠ΅, ΠΊΠ°ΠΊ ΠΈ Π·Π°ΡΡΠ΄Ρ: ΠΎΠ΄Π½ΠΎΠΈΠΌΠ΅Π½Π½ΡΠ΅ ΠΏΠΎΠ»ΡΡΠ° ΠΎΡΡΠ°Π»ΠΊΠΈΠ²Π°ΡΡΡΡ, Π° ΡΠ°Π·Π½ΠΎΠΈΠΌΠ΅Π½Π½ΡΠ΅ β ΠΏΡΠΈΡΡΠ³ΠΈΠ²Π°ΡΡΡΡ. ΠΠ΅Π²ΠΎΠ·ΠΌΠΎΠΆΠ½ΠΎ ΠΏΠΎΠ»ΡΡΠΈΡΡ ΠΌΠ°Π³Π½ΠΈΡ Ρ ΠΎΠ΄Π½ΠΈΠΌ ΠΏΠΎΠ»ΡΡΠΎΠΌ.
ΠΠ°ΠΆΠ΅ Π΅ΡΠ»ΠΈ ΠΌΠ°Π³Π½ΠΈΡ ΡΠ°Π·Π»ΠΎΠΌΠ°ΡΡ, ΡΠΎ Ρ ΠΊΠ°ΠΆΠ΄ΠΎΠΉ ΡΠ°ΡΡΠΈ Π±ΡΠ΄Π΅Ρ ΠΏΠΎ Π΄Π²Π° ΡΠ°Π·Π½ΡΡ ΠΏΠΎΠ»ΡΡΠ°.
ΠΠ΅ΠΊΡΠΎΡ ΠΌΠ°Π³Π½ΠΈΡΠ½ΠΎΠΉ ΠΈΠ½Π΄ΡΠΊΡΠΈΠΈ
ΠΠ΅ΠΊΡΠΎΡ ΠΌΠ°Π³Π½ΠΈΡΠ½ΠΎΠΉ ΠΈΠ½Π΄ΡΠΊΡΠΈΠΈ β Π²Π΅ΠΊΡΠΎΡΠ½Π°Ρ ΡΠΈΠ·ΠΈΡΠ΅ΡΠΊΠ°Ρ Π²Π΅Π»ΠΈΡΠΈΠ½Π°, ΡΠ²Π»ΡΡΡΠ°ΡΡΡ Ρ Π°ΡΠ°ΠΊΡΠ΅ΡΠΈΡΡΠΈΠΊΠΎΠΉ ΠΌΠ°Π³Π½ΠΈΡΠ½ΠΎΠ³ΠΎ ΠΏΠΎΠ»Ρ, ΡΠΈΡΠ»Π΅Π½Π½ΠΎ ΡΠ°Π²Π½Π°Ρ ΡΠΈΠ»Π΅, Π΄Π΅ΠΉΡΡΠ²ΡΡΡΠ΅ΠΉ Π½Π° ΡΠ»Π΅ΠΌΠ΅Π½Ρ ΡΠΎΠΊΠ° Π² 1 Π ΠΈ Π΄Π»ΠΈΠ½ΠΎΠΉ 1 ΠΌ, Π΅ΡΠ»ΠΈ Π½Π°ΠΏΡΠ°Π²Π»Π΅Π½ΠΈΠ΅ ΡΠΈΠ»ΠΎΠ²ΠΎΠΉ Π»ΠΈΠ½ΠΈΠΈ ΠΏΠ΅ΡΠΏΠ΅Π½Π΄ΠΈΠΊΡΠ»ΡΡΠ½ΠΎ ΠΏΡΠΎΠ²ΠΎΠ΄Π½ΠΈΠΊΡ. ΠΠ±ΠΎΠ·Π½Π°ΡΠ°Π΅ΡΡΡ Π, Π΅Π΄ΠΈΠ½ΠΈΡΠ° ΠΈΠ·ΠΌΠ΅ΡΠ΅Π½ΠΈΡ β 1 Π’Π΅ΡΠ»Π°. 1 Π’Π» β ΠΎΡΠ΅Π½Ρ Π±ΠΎΠ»ΡΡΠ°Ρ Π²Π΅Π»ΠΈΡΠΈΠ½Π°, ΠΏΠΎΡΡΠΎΠΌΡ Π² ΡΠ΅Π°Π»ΡΠ½ΡΡ ΠΌΠ°Π³Π½ΠΈΡΠ½ΡΡ ΠΏΠΎΠ»ΡΡ ΠΌΠ°Π³Π½ΠΈΡΠ½ΡΡ ΠΈΠ½Π΄ΡΠΊΡΠΈΡ ΠΈΠ·ΠΌΠ΅ΡΡΡΡ Π² ΠΌΠ’Π».
ΠΠ΅ΠΊΡΠΎΡ ΠΌΠ°Π³Π½ΠΈΡΠ½ΠΎΠΉ ΠΈΠ½Π΄ΡΠΊΡΠΈΠΈ Π½Π°ΠΏΡΠ°Π²Π»Π΅Π½ ΠΏΠΎ ΠΊΠ°ΡΠ°ΡΠ΅Π»ΡΠ½ΠΎΠΉ ΠΊ ΡΠΈΠ»ΠΎΠ²ΡΠΌ Π»ΠΈΠ½ΠΈΡΠΌ, Ρ.Π΅. ΡΠΎΠ²ΠΏΠ°Π΄Π°Π΅Ρ Ρ Π½Π°ΠΏΡΠ°Π²Π»Π΅Π½ΠΈΠ΅ΠΌ ΡΠ΅Π²Π΅ΡΠ½ΠΎΠ³ΠΎ ΠΏΠΎΠ»ΡΡΠ° ΠΌΠ°Π³Π½ΠΈΡΠ½ΠΎΠΉ ΡΡΡΠ΅Π»ΠΊΠΈ, ΠΏΠΎΠΌΠ΅ΡΡΠ½Π½ΠΎΠΉ Π² Π΄Π°Π½Π½ΠΎΠ΅ ΠΌΠ°Π³Π½ΠΈΡΠ½ΠΎΠ΅ ΠΏΠΎΠ»Π΅.Β ΠΠ°ΠΏΡΠ°Π²Π»Π΅Π½ΠΈΠ΅ Π²Π΅ΠΊΡΠΎΡΠ° ΠΌΠ°Π³Π½ΠΈΡΠ½ΠΎΠΉ ΠΈΠ½Π΄ΡΠΊΡΠΈΠΈ Π½Π΅ ΡΠΎΠ²ΠΏΠ°Π΄Π°Π΅Ρ Ρ Π½Π°ΠΏΡΠ°Π²Π»Π΅Π½ΠΈΠ΅ΠΌ ΡΠΈΠ»Ρ, Π΄Π΅ΠΉΡΡΠ²ΡΡΡΠ΅ΠΉ Π½Π° ΠΏΡΠΎΠ²ΠΎΠ΄Π½ΠΈΠΊ, ΠΏΠΎΡΡΠΎΠΌΡ ΡΠΈΠ»ΠΎΠ²ΡΠ΅ Π»ΠΈΠ½ΠΈΠΈ ΠΌΠ°Π³Π½ΠΈΡΠ½ΠΎΠ³ΠΎ ΠΏΠΎΠ»Ρ, ΡΡΡΠΎΠ³ΠΎ Π³ΠΎΠ²ΠΎΡΡ, ΡΠΈΠ»ΠΎΠ²ΡΠΌΠΈ Π½Π΅ ΡΠ²Π»ΡΡΡΡΡ.
Π‘ΠΈΠ»ΠΎΠ²Π°Ρ Π»ΠΈΠ½ΠΈΡ ΠΌΠ°Π³Π½ΠΈΡΠ½ΠΎΠ³ΠΎ ΠΏΠΎΠ»Ρ ΠΏΠΎΡΡΠΎΡΠ½Π½ΡΡ ΠΌΠ°Π³Π½ΠΈΡΠΎΠ² Π½Π°ΠΏΡΠ°Π²Π»Π΅Π½Π° ΠΏΠΎ ΠΎΡΠ½ΠΎΡΠ΅Π½ΠΈΡ ΠΊ ΡΠ°ΠΌΠΈΠΌ ΠΌΠ°Π³Π½ΠΈΡΠ°ΠΌ ΡΠ°ΠΊ, ΠΊΠ°ΠΊ ΠΏΠΎΠΊΠ°Π·Π°Π½ΠΎ Π½Π° ΡΠΈΡΡΠ½ΠΊΠ΅:
Π ΡΠ»ΡΡΠ°Π΅ ΠΌΠ°Π³Π½ΠΈΡΠ½ΠΎΠ³ΠΎ ΠΏΠΎΠ»Ρ ΡΠ»Π΅ΠΊΡΡΠΈΡΠ΅ΡΠΊΠΎΠ³ΠΎ ΡΠΎΠΊΠ° Π΄Π»Ρ ΠΎΠΏΡΠ΅Π΄Π΅Π»Π΅Π½ΠΈΡ Π½Π°ΠΏΡΠ°Π²Π»Π΅Π½ΠΈΡ ΡΠΈΠ»ΠΎΠ²ΡΡ Π»ΠΈΠ½ΠΈΠΉ ΠΈΡΠΏΠΎΠ»ΡΠ·ΡΡΡ ΠΏΡΠ°Π²ΠΈΠ»ΠΎ Β«ΠΡΠ°Π²ΠΎΠΉ ΡΡΠΊΠΈΒ»: Π΅ΡΠ»ΠΈ Π²Π·ΡΡΡ ΠΏΡΠΎΠ²ΠΎΠ΄Π½ΠΈΠΊ Π² ΠΏΡΠ°Π²ΡΡ ΡΡΠΊΡ ΡΠ°ΠΊ, ΡΡΠΎΠ±Ρ Π±ΠΎΠ»ΡΡΠΎΠΉ ΠΏΠ°Π»Π΅Ρ Π±ΡΠ» Π½Π°ΠΏΡΠ°Π²Π»Π΅Π½ ΠΏΠΎ ΡΠΎΠΊΡ, ΡΠΎ ΡΠ΅ΡΡΡΠ΅ ΠΏΠ°Π»ΡΡΠ°, ΠΎΠ±Ρ Π²Π°ΡΡΠ²Π°ΡΡΠΈΠ΅ ΠΏΡΠΎΠ²ΠΎΠ΄Π½ΠΈΠΊ, ΠΏΠΎΠΊΠ°Π·ΡΠ²Π°ΡΡ Π½Π°ΠΏΡΠ°Π²Π»Π΅Π½ΠΈΠ΅ ΡΠΈΠ»ΠΎΠ²ΡΡ Π»ΠΈΠ½ΠΈΠΉ Π²ΠΎΠΊΡΡΠ³ ΠΏΡΠΎΠ²ΠΎΠ΄Π½ΠΈΠΊΠ°:
Π ΡΠ»ΡΡΠ°Π΅ ΠΏΡΡΠΌΠΎΠ³ΠΎ ΡΠΎΠΊΠ° Π»ΠΈΠ½ΠΈΠΈ ΠΌΠ°Π³Π½ΠΈΡΠ½ΠΎΠΉ ΠΈΠ½Π΄ΡΠΊΡΠΈΠΈ β ΠΎΠΊΡΡΠΆΠ½ΠΎΡΡΠΈ, ΠΏΠ»ΠΎΡΠΊΠΎΡΡΠΈ ΠΊΠΎΡΠΎΡΡΡ ΠΏΠ΅ΡΠΏΠ΅Π½Π΄ΠΈΠΊΡΠ»ΡΡΠ½Ρ ΡΠΎΠΊΡ. ΠΠ΅ΠΊΡΠΎΡΠ° ΠΌΠ°Π³Π½ΠΈΡΠ½ΠΎΠΉ ΠΈΠ½Π΄ΡΠΊΡΠΈΠΈ Π½Π°ΠΏΡΠ°Π²Π»Π΅Π½Ρ ΠΏΠΎ ΠΊΠ°ΡΠ°ΡΠ΅Π»ΡΠ½ΠΎΠΉ ΠΊ ΠΎΠΊΡΡΠΆΠ½ΠΎΡΡΠΈ.
Π‘ΠΎΠ»Π΅Π½ΠΎΠΈΠ΄ β Π½Π°ΠΌΠΎΡΠ°Π½Π½ΡΠΉ Π½Π° ΡΠΈΠ»ΠΈΠ½Π΄ΡΠΈΡΠ΅ΡΠΊΡΡ ΠΏΠΎΠ²Π΅ΡΡ Π½ΠΎΡΡΡ ΠΏΡΠΎΠ²ΠΎΠ΄Π½ΠΈΠΊ, ΠΏΠΎ ΠΊΠΎΡΠΎΡΠΎΠΌΡ ΡΠ΅ΡΡΡ ΡΠ»Π΅ΠΊΡΡΠΈΡΠ΅ΡΠΊΠΈΠΉ ΡΠΎΠΊ I. ΠΠ°Π³Π½ΠΈΡΠ½ΠΎΠ΅ ΠΏΠΎΠ»Π΅ ΡΠΎΠ»Π΅Π½ΠΎΠΈΠ΄Π° ΠΏΠΎΠ΄ΠΎΠ±Π½ΠΎ ΠΏΠΎΠ»Ρ ΠΏΡΡΠΌΠΎΠ³ΠΎ ΠΏΠΎΡΡΠΎΡΠ½Π½ΠΎΠ³ΠΎ ΠΌΠ°Π³Π½ΠΈΡΠ°. ΠΠ½ΡΡΡΠΈ ΡΠΎΠ»Π΅Π½ΠΎΠΈΠ΄Π° Π΄Π»ΠΈΠ½ΠΎΠΉ lΒ ΠΈ ΠΊΠΎΠ»ΠΈΡΠ΅ΡΡΠ²ΠΎΠΌ Π²ΠΈΡΠΊΠΎΠ² N ΡΠΎΠ·Π΄Π°Π΅ΡΡΡ ΠΎΠ΄Π½ΠΎΡΠΎΠ΄Π½ΠΎΠ΅ ΠΌΠ°Π³Π½ΠΈΡΠ½ΠΎΠ΅ ΠΏΠΎΠ»Π΅ Ρ ΠΈΠ½Π΄ΡΠΊΡΠΈΠ΅ΠΉ (Π΅Π³ΠΎ Π½Π°ΠΏΡΠ°Π²Π»Π΅Π½ΠΈΠ΅ ΡΠ°ΠΊΠΆΠ΅ ΠΎΠΏΡΠ΅Π΄Π΅Π»ΡΠ΅ΡΡΡ ΠΏΡΠ°Π²ΠΈΠ»ΠΎΠΌ ΠΏΡΠ°Π²ΠΎΠΉ ΡΡΠΊΠΈ):
ΠΠΈΠ½ΠΈΠΈ ΠΌΠ°Π³Π½ΠΈΡΠ½ΠΎΠ³ΠΎ ΠΏΠΎΠ»Ρ ΠΈΠΌΠ΅ΡΡ Π²ΠΈΠ΄ Π·Π°ΠΌΠΊΠ½ΡΡΡΡ Π»ΠΈΠ½ΠΈΠΉ β ΡΡΠΎ ΠΎΠ±ΡΠ΅Π΅ ΡΠ²ΠΎΠΉΡΡΠ²ΠΎ Π²ΡΠ΅Ρ ΠΌΠ°Π³Π½ΠΈΡΠ½ΡΡ Π»ΠΈΠ½ΠΈΠΉ. Π’Π°ΠΊΠΎΠ΅ ΠΏΠΎΠ»Π΅ Π½Π°Π·ΡΠ²Π°ΡΡ Π²ΠΈΡ ΡΠ΅Π²ΡΠΌ. Π ΡΠ»ΡΡΠ°Π΅ ΠΏΠΎΡΡΠΎΡΠ½Π½ΡΡ ΠΌΠ°Π³Π½ΠΈΡΠΎΠ² Π»ΠΈΠ½ΠΈΠΈ Π½Π΅ ΠΎΠΊΠ°Π½ΡΠΈΠ²Π°ΡΡΡΡ Π½Π° ΠΏΠΎΠ²Π΅ΡΡ Π½ΠΎΡΡΠΈ, Π° ΠΏΡΠΎΠ½ΠΈΠΊΠ°ΡΡ Π²Π½ΡΡΡΡ ΠΌΠ°Π³Π½ΠΈΡΠ° ΠΈ Π·Π°ΠΌΡΠΊΠ°ΡΡΡΡ Π²Π½ΡΡΡΠΈ. ΠΡΠΎ ΡΠ°Π·Π»ΠΈΡΠΈΠ΅ ΡΠ»Π΅ΠΊΡΡΠΈΡΠ΅ΡΠΊΠΎΠ³ΠΎ ΠΈ ΠΌΠ°Π³Π½ΠΈΡΠ½ΠΎΠ³ΠΎ ΠΏΠΎΠ»Π΅ΠΉ ΠΎΠ±ΡΡΡΠ½ΡΠ΅ΡΡΡ ΡΠ΅ΠΌ, ΡΡΠΎ, Π² ΠΎΡΠ»ΠΈΡΠΈΠ΅ ΠΎΡ ΡΠ»Π΅ΠΊΡΡΠΈΡΠ΅ΡΠΊΠΈΡ , ΠΌΠ°Π³Π½ΠΈΡΠ½ΡΡ Π·Π°ΡΡΠ΄ΠΎΠ² Π½Π΅ ΡΡΡΠ΅ΡΡΠ²ΡΠ΅Ρ.
ΠΠ°Π³Π½ΠΈΡΠ½ΡΠ΅ ΡΠ²ΠΎΠΉΡΡΠ²Π° Π²Π΅ΡΠ΅ΡΡΠ²Π°
ΠΡΠ΅ Π²Π΅ΡΠ΅ΡΡΠ²Π° ΠΎΠ±Π»Π°Π΄Π°ΡΡ ΠΌΠ°Π³Π½ΠΈΡΠ½ΡΠΌΠΈ ΡΠ²ΠΎΠΉΡΡΠ²Π°ΠΌΠΈ. ΠΠ°Π³Π½ΠΈΡΠ½ΡΠ΅ ΡΠ²ΠΎΠΉΡΡΠ²Π° Π²Π΅ΡΠ΅ΡΡΠ²Π° Ρ Π°ΡΠ°ΠΊΡΠ΅ΡΠΈΠ·ΡΡΡΡΡ ΠΎΡΠ½ΠΎΡΠΈΡΠ΅Π»ΡΠ½ΠΎΠΉ ΠΌΠ°Π³Π½ΠΈΡΠ½ΠΎΠΉ ΠΏΡΠΎΠ½ΠΈΡΠ°Π΅ΠΌΠΎΡΡΡΡ ΞΌ, Π΄Π»Ρ ΠΊΠΎΡΠΎΡΠΎΠΉ Π²Π΅ΡΠ½ΠΎ ΡΠ»Π΅Π΄ΡΡΡΠ΅Π΅:
ΠΠ°Π½Π½Π°Ρ ΡΠΎΡΠΌΡΠ»Π° Π²ΡΡΠ°ΠΆΠ°Π΅Ρ ΡΠΎΠΎΡΠ²Π΅ΡΡΡΠ²ΠΈΠ΅ Π²Π΅ΠΊΡΠΎΡΠ° ΠΌΠ°Π³Π½ΠΈΡΠ½ΠΎΠΉ ΠΈΠ½Π΄ΡΠΊΡΠΈΠΈ ΠΏΠΎΠ»Ρ Π² Π²Π°ΠΊΡΡΠΌΠ΅ ΠΈ Π² Π΄Π°Π½Π½ΠΎΠΉ ΡΡΠ΅Π΄Π΅. Π ΠΎΡΠ»ΠΈΡΠΈΠ΅ ΠΎΡ ΡΠ»Π΅ΠΊΡΡΠΈΡΠ΅ΡΠΊΠΎΠ³ΠΎ, ΠΏΡΠΈ ΠΌΠ°Π³Π½ΠΈΡΠ½ΠΎΠΌ Π²Π·Π°ΠΈΠΌΠΎΠ΄Π΅ΠΉΡΡΠ²ΠΈΠΈ Π² ΡΡΠ΅Π΄Π΅ ΠΌΠΎΠΆΠ½ΠΎ Π½Π°Π±Π»ΡΠ΄Π°ΡΡ ΠΈ ΡΡΠΈΠ»Π΅Π½ΠΈΠ΅, ΠΈ ΠΎΡΠ»Π°Π±Π»Π΅Π½ΠΈΠ΅ Π²Π·Π°ΠΈΠΌΠΎΠ΄Π΅ΠΉΡΡΠ²ΠΈΡ ΠΏΠΎ ΡΡΠ°Π²Π½Π΅Π½ΠΈΡ Ρ Π²Π°ΠΊΡΡΠΌΠΎΠΌ, Ρ ΠΊΠΎΡΠΎΡΠΎΠ³ΠΎ ΠΌΠ°Π³Π½ΠΈΡΠ½Π°Ρ ΠΏΡΠΎΠ½ΠΈΡΠ°Π΅ΠΌΠΎΡΡΡΒ ΞΌ = 1.
Π£ Π΄ΠΈΠ°ΠΌΠ°Π³Π½Π΅ΡΠΈΠΊΠΎΠ²Β ΠΌΠ°Π³Π½ΠΈΡΠ½Π°Ρ ΠΏΡΠΎΠ½ΠΈΡΠ°Π΅ΠΌΠΎΡΡΡΒ ΞΌΒ Π½Π΅ΠΌΠ½ΠΎΠ³ΠΎ ΠΌΠ΅Π½ΡΡΠ΅ Π΅Π΄ΠΈΠ½ΠΈΡΡ. ΠΡΠΈΠΌΠ΅ΡΡ: Π²ΠΎΠ΄Π°, Π°Π·ΠΎΡ, ΡΠ΅ΡΠ΅Π±ΡΠΎ, ΠΌΠ΅Π΄Ρ, Π·ΠΎΠ»ΠΎΡΠΎ. ΠΡΠΈ Π²Π΅ΡΠ΅ΡΡΠ²Π° Π½Π΅ΡΠΊΠΎΠ»ΡΠΊΠΎ ΠΎΡΠ»Π°Π±Π»ΡΡΡ ΠΌΠ°Π³Π½ΠΈΡΠ½ΠΎΠ΅ ΠΏΠΎΠ»Π΅. ΠΠ°ΡΠ°ΠΌΠ°Π³Π½Π΅ΡΠΈΠΊΠΈ β ΠΊΠΈΡΠ»ΠΎΡΠΎΠ΄, ΠΏΠ»Π°ΡΠΈΠ½Π°, ΠΌΠ°Π³Π½ΠΈΠΉ β Π½Π΅ΡΠΊΠΎΠ»ΡΠΊΠΎ ΡΡΠΈΠ»ΠΈΠ²Π°ΡΡ ΠΏΠΎΠ»Π΅, ΠΈΠΌΠ΅Ρ ΞΌΒ Π½Π΅ΠΌΠ½ΠΎΠ³ΠΎ Π±ΠΎΠ»ΡΡΠ΅ Π΅Π΄ΠΈΠ½ΠΈΡΡ.
Π£ ΡΠ΅ΡΡΠΎΠΌΠ°Π³Π½Π΅ΡΠΈΠΊΠΎΠ² β ΠΆΠ΅Π»Π΅Π·ΠΎ, Π½ΠΈΠΊΠ΅Π»Ρ, ΠΊΠΎΠ±Π°Π»ΡΡ β ΞΌΒ >> 1. ΠΠ°ΠΏΡΠΈΠΌΠ΅Ρ, Ρ ΠΆΠ΅Π»Π΅Π·Π° ΞΌ βΒ 25000.
ΠΠ°Π³Π½ΠΈΡΠ½ΡΠΉ ΠΏΠΎΡΠΎΠΊ. ΠΠ»Π΅ΠΊΡΡΠΎΠΌΠ°Π³Π½ΠΈΡΠ½Π°Ρ ΠΈΠ½Π΄ΡΠΊΡΠΈΡ
Π ΠΎΠ³Π»Π°Π²Π»Π΅Π½ΠΈΡβ¦
Π―Π²Π»Π΅Π½ΠΈΠ΅ ΡΠ»Π΅ΠΊΡΡΠΎΠΌΠ°Π³Π½ΠΈΡΠ½ΠΎΠΉ ΠΈΠ½Π΄ΡΠΊΡΠΈΠΈ Π±ΡΠ»ΠΎ ΠΎΡΠΊΡΡΡΠΎ Π²ΡΠ΄Π°ΡΡΠΈΠΌΡΡ Π°Π½Π³Π»ΠΈΠΉΡΠΊΠΈΠΌ ΡΠΈΠ·ΠΈΠΊΠΎΠΌ Π.Π€Π°ΡΠ°Π΄Π΅Π΅ΠΌ Π² 1831 Π³ΠΎΠ΄Ρ. ΠΠ½ΠΎ Π·Π°ΠΊΠ»ΡΡΠ°Π΅ΡΡΡ Π² Π²ΠΎΠ·Π½ΠΈΠΊΠ½ΠΎΠ²Π΅Π½ΠΈΠΈ ΡΠ»Π΅ΠΊΡΡΠΈΡΠ΅ΡΠΊΠΎΠ³ΠΎ ΡΠΎΠΊΠ° Π² Π·Π°ΠΌΠΊΠ½ΡΡΠΎΠΌ ΠΏΡΠΎΠ²ΠΎΠ΄ΡΡΠ΅ΠΌ ΠΊΠΎΠ½ΡΡΡΠ΅ ΠΏΡΠΈ ΠΈΠ·ΠΌΠ΅Π½Π΅Π½ΠΈΠΈ Π²ΠΎ Π²ΡΠ΅ΠΌΠ΅Π½ΠΈ ΠΌΠ°Π³Π½ΠΈΡΠ½ΠΎΠ³ΠΎ ΠΏΠΎΡΠΎΠΊΠ°, ΠΏΡΠΎΠ½ΠΈΠ·ΡΠ²Π°ΡΡΠ΅Π³ΠΎ ΠΊΠΎΠ½ΡΡΡ. ΠΠ°Π³Π½ΠΈΡΠ½ΡΠΌ ΠΏΠΎΡΠΎΠΊΠΎΠΌ Ξ¦ ΡΠ΅ΡΠ΅Π· ΠΏΠ»ΠΎΡΠ°Π΄Ρ S ΠΊΠΎΠ½ΡΡΡΠ° Π½Π°Π·ΡΠ²Π°ΡΡ Π²Π΅Π»ΠΈΡΠΈΠ½Ρ:
Π³Π΄Π΅: B β ΠΌΠΎΠ΄ΡΠ»Ρ Π²Π΅ΠΊΡΠΎΡΠ° ΠΌΠ°Π³Π½ΠΈΡΠ½ΠΎΠΉ ΠΈΠ½Π΄ΡΠΊΡΠΈΠΈ, Ξ± β ΡΠ³ΠΎΠ» ΠΌΠ΅ΠΆΠ΄Ρ Π²Π΅ΠΊΡΠΎΡΠΎΠΌ ΠΌΠ°Π³Π½ΠΈΡΠ½ΠΎΠΉ ΠΈΠ½Π΄ΡΠΊΡΠΈΠΈ B ΠΈ Π½ΠΎΡΠΌΠ°Π»ΡΡ (ΠΏΠ΅ΡΠΏΠ΅Π½Π΄ΠΈΠΊΡΠ»ΡΡΠΎΠΌ) ΠΊ ΠΏΠ»ΠΎΡΠΊΠΎΡΡΠΈ ΠΊΠΎΠ½ΡΡΡΠ°, S β ΠΏΠ»ΠΎΡΠ°Π΄Ρ ΠΊΠΎΠ½ΡΡΡΠ°, N β ΠΊΠΎΠ»ΠΈΡΠ΅ΡΡΠ²ΠΎ Π²ΠΈΡΠΊΠΎΠΌ Π² ΠΊΠΎΠ½ΡΡΡΠ΅. ΠΠ΄ΠΈΠ½ΠΈΡΠ° ΠΌΠ°Π³Π½ΠΈΡΠ½ΠΎΠ³ΠΎ ΠΏΠΎΡΠΎΠΊΠ° Π² ΡΠΈΡΡΠ΅ΠΌΠ΅ Π‘Π Π½Π°Π·ΡΠ²Π°Π΅ΡΡΡ ΠΠ΅Π±Π΅ΡΠΎΠΌ (ΠΠ±).
- Π€Π°ΡΠ°Π΄Π΅ΠΉ ΡΠΊΡΠΏΠ΅ΡΠΈΠΌΠ΅Π½ΡΠ°Π»ΡΠ½ΠΎ ΡΡΡΠ°Π½ΠΎΠ²ΠΈΠ», ΡΡΠΎ ΠΏΡΠΈ ΠΈΠ·ΠΌΠ΅Π½Π΅Π½ΠΈΠΈ ΠΌΠ°Π³Π½ΠΈΡΠ½ΠΎΠ³ΠΎ ΠΏΠΎΡΠΎΠΊΠ° Π² ΠΏΡΠΎΠ²ΠΎΠ΄ΡΡΠ΅ΠΌ ΠΊΠΎΠ½ΡΡΡΠ΅ Π²ΠΎΠ·Π½ΠΈΠΊΠ°Π΅Ρ ΠΠΠ‘ ΠΈΠ½Π΄ΡΠΊΡΠΈΠΈΒ Ξ΅ΠΈΠ½Π΄, ΡΠ°Π²Π½Π°Ρ ΡΠΊΠΎΡΠΎΡΡΠΈ ΠΈΠ·ΠΌΠ΅Π½Π΅Π½ΠΈΡ ΠΌΠ°Π³Π½ΠΈΡΠ½ΠΎΠ³ΠΎ ΠΏΠΎΡΠΎΠΊΠ° ΡΠ΅ΡΠ΅Π· ΠΏΠΎΠ²Π΅ΡΡ Π½ΠΎΡΡΡ, ΠΎΠ³ΡΠ°Π½ΠΈΡΠ΅Π½Π½ΡΡ ΠΊΠΎΠ½ΡΡΡΠΎΠΌ, Π²Π·ΡΡΠΎΠΉ ΡΠΎ Π·Π½Π°ΠΊΠΎΠΌ ΠΌΠΈΠ½ΡΡ:
- ΠΠ·ΠΌΠ΅Π½Π΅Π½ΠΈΠ΅ ΠΌΠ°Π³Π½ΠΈΡΠ½ΠΎΠ³ΠΎ ΠΏΠΎΡΠΎΠΊΠ°, ΠΏΡΠΎΠ½ΠΈΠ·ΡΠ²Π°ΡΡΠ΅Π³ΠΎ Π·Π°ΠΌΠΊΠ½ΡΡΡΠΉ ΠΊΠΎΠ½ΡΡΡ, ΠΌΠΎΠΆΠ΅Ρ ΠΏΡΠΎΠΈΡΡ ΠΎΠ΄ΠΈΡΡ ΠΏΠΎ Π΄Π²ΡΠΌ Π²ΠΎΠ·ΠΌΠΎΠΆΠ½ΡΠΌ ΠΏΡΠΈΡΠΈΠ½Π°ΠΌ.
- ΠΠ°Π³Π½ΠΈΡΠ½ΡΠΉ ΠΏΠΎΡΠΎΠΊ ΠΈΠ·ΠΌΠ΅Π½ΡΠ΅ΡΡΡ Π²ΡΠ»Π΅Π΄ΡΡΠ²ΠΈΠ΅ ΠΏΠ΅ΡΠ΅ΠΌΠ΅ΡΠ΅Π½ΠΈΡ ΠΊΠΎΠ½ΡΡΡΠ° ΠΈΠ»ΠΈ Π΅Π³ΠΎ ΡΠ°ΡΡΠ΅ΠΉ Π² ΠΏΠΎΡΡΠΎΡΠ½Π½ΠΎΠΌ Π²ΠΎ Π²ΡΠ΅ΠΌΠ΅Π½ΠΈ ΠΌΠ°Π³Π½ΠΈΡΠ½ΠΎΠΌ ΠΏΠΎΠ»Π΅. ΠΡΠΎ ΡΠ»ΡΡΠ°ΠΉ, ΠΊΠΎΠ³Π΄Π° ΠΏΡΠΎΠ²ΠΎΠ΄Π½ΠΈΠΊΠΈ, Π° Π²ΠΌΠ΅ΡΡΠ΅ Ρ Π½ΠΈΠΌΠΈ ΠΈ ΡΠ²ΠΎΠ±ΠΎΠ΄Π½ΡΠ΅ Π½ΠΎΡΠΈΡΠ΅Π»ΠΈ Π·Π°ΡΡΠ΄Π°, Π΄Π²ΠΈΠΆΡΡΡΡ Π² ΠΌΠ°Π³Π½ΠΈΡΠ½ΠΎΠΌ ΠΏΠΎΠ»Π΅. ΠΠΎΠ·Π½ΠΈΠΊΠ½ΠΎΠ²Π΅Π½ΠΈΠ΅ ΠΠΠ‘ ΠΈΠ½Π΄ΡΠΊΡΠΈΠΈ ΠΎΠ±ΡΡΡΠ½ΡΠ΅ΡΡΡ Π΄Π΅ΠΉΡΡΠ²ΠΈΠ΅ΠΌ ΡΠΈΠ»Ρ ΠΠΎΡΠ΅Π½ΡΠ° Π½Π° ΡΠ²ΠΎΠ±ΠΎΠ΄Π½ΡΠ΅ Π·Π°ΡΡΠ΄Ρ Π² Π΄Π²ΠΈΠΆΡΡΠΈΡ ΡΡ ΠΏΡΠΎΠ²ΠΎΠ΄Π½ΠΈΠΊΠ°Ρ . Π‘ΠΈΠ»Π° ΠΠΎΡΠ΅Π½ΡΠ° ΠΈΠ³ΡΠ°Π΅Ρ Π² ΡΡΠΎΠΌ ΡΠ»ΡΡΠ°Π΅ ΡΠΎΠ»Ρ ΡΡΠΎΡΠΎΠ½Π½Π΅ΠΉ ΡΠΈΠ»Ρ.
- ΠΡΠΎΡΠ°Ρ ΠΏΡΠΈΡΠΈΠ½Π° ΠΈΠ·ΠΌΠ΅Π½Π΅Π½ΠΈΡ ΠΌΠ°Π³Π½ΠΈΡΠ½ΠΎΠ³ΠΎ ΠΏΠΎΡΠΎΠΊΠ°, ΠΏΡΠΎΠ½ΠΈΠ·ΡΠ²Π°ΡΡΠ΅Π³ΠΎ ΠΊΠΎΠ½ΡΡΡ, β ΠΈΠ·ΠΌΠ΅Π½Π΅Π½ΠΈΠ΅ Π²ΠΎ Π²ΡΠ΅ΠΌΠ΅Π½ΠΈ ΠΌΠ°Π³Π½ΠΈΡΠ½ΠΎΠ³ΠΎ ΠΏΠΎΠ»Ρ ΠΏΡΠΈ Π½Π΅ΠΏΠΎΠ΄Π²ΠΈΠΆΠ½ΠΎΠΌ ΠΊΠΎΠ½ΡΡΡΠ΅.
ΠΡΠΈ ΡΠ΅ΡΠ΅Π½ΠΈΠΈ Π·Π°Π΄Π°Ρ Π²Π°ΠΆΠ½ΠΎ ΡΡΠ°Π·Ρ ΠΎΠΏΡΠ΅Π΄Π΅Π»ΠΈΡΡ Π·Π° ΡΡΠ΅Ρ ΡΠ΅Π³ΠΎ ΠΌΠ΅Π½ΡΠ΅ΡΡΡ ΠΌΠ°Π³Π½ΠΈΡΠ½ΡΠΉ ΠΏΠΎΡΠΎΠΊ. ΠΠΎΠ·ΠΌΠΎΠΆΠ½ΠΎ ΡΡΠΈ Π²Π°ΡΠΈΠ°Π½ΡΠ°:
- ΠΠ΅Π½ΡΠ΅ΡΡΡ ΠΌΠ°Π³Π½ΠΈΡΠ½ΠΎΠ΅ ΠΏΠΎΠ»Π΅.
- ΠΠ΅Π½ΡΠ΅ΡΡΡ ΠΏΠ»ΠΎΡΠ°Π΄Ρ ΠΊΠΎΠ½ΡΡΡΠ°.
- ΠΠ΅Π½ΡΠ΅ΡΡΡ ΠΎΡΠΈΠ΅Π½ΡΠ°ΡΠΈΡ ΡΠ°ΠΌΠΊΠΈ ΠΎΡΠ½ΠΎΡΠΈΡΠ΅Π»ΡΠ½ΠΎ ΠΏΠΎΠ»Ρ.
ΠΡΠΈ ΡΡΠΎΠΌ ΠΏΡΠΈ ΡΠ΅ΡΠ΅Π½ΠΈΠΈ Π·Π°Π΄Π°Ρ ΠΎΠ±ΡΡΠ½ΠΎ ΡΡΠΈΡΠ°ΡΡ ΠΠΠ‘ ΠΏΠΎ ΠΌΠΎΠ΄ΡΠ»Ρ. ΠΠ±ΡΠ°ΡΠΈΠΌ Π²Π½ΠΈΠΌΠ°Π½ΠΈΠ΅ ΡΠ°ΠΊΠΆΠ΅ Π²Π½ΠΈΠΌΠ°Π½ΠΈΠ΅ Π½Π° ΠΎΠ΄ΠΈΠ½ ΡΠ°ΡΡΠ½ΡΠΉ ΡΠ»ΡΡΠ°ΠΉ, Π² ΠΊΠΎΡΠΎΡΠΎΠΌ ΠΏΡΠΎΠΈΡΡ ΠΎΠ΄ΠΈΡ ΡΠ²Π»Π΅Π½ΠΈΠ΅ ΡΠ»Π΅ΠΊΡΡΠΎΠΌΠ°Π³Π½ΠΈΡΠ½ΠΎΠΉ ΠΈΠ½Π΄ΡΠΊΡΠΈΠΈ. ΠΡΠ°ΠΊ, ΠΌΠ°ΠΊΡΠΈΠΌΠ°Π»ΡΠ½ΠΎΠ΅ Π·Π½Π°ΡΠ΅Π½ΠΈΠ΅ ΠΠΠ‘ ΠΈΠ½Π΄ΡΠΊΡΠΈΠΈ Π² ΠΊΠΎΠ½ΡΡΡΠ΅ ΡΠΎΡΡΠΎΡΡΠ΅ΠΌ ΠΈΠ· N Π²ΠΈΡΠΊΠΎΠ², ΠΏΠ»ΠΎΡΠ°Π΄ΡΡ S, Π²ΡΠ°ΡΠ°ΡΡΠ΅ΠΌΡΡ Ρ ΡΠ³Π»ΠΎΠ²ΠΎΠΉ ΡΠΊΠΎΡΠΎΡΡΡΡ Ο Π² ΠΌΠ°Π³Π½ΠΈΡΠ½ΠΎΠΌ ΠΏΠΎΠ»Π΅ Ρ ΠΈΠ½Π΄ΡΠΊΡΠΈΠ΅ΠΉ Π:
ΠΠ²ΠΈΠΆΠ΅Π½ΠΈΠ΅ ΠΏΡΠΎΠ²ΠΎΠ΄Π½ΠΈΠΊΠ° Π² ΠΌΠ°Π³Π½ΠΈΡΠ½ΠΎΠΌ ΠΏΠΎΠ»Π΅
Π ΠΎΠ³Π»Π°Π²Π»Π΅Π½ΠΈΡβ¦
ΠΡΠΈ Π΄Π²ΠΈΠΆΠ΅Π½ΠΈΠΈ ΠΏΡΠΎΠ²ΠΎΠ΄Π½ΠΈΠΊΠ° Π΄Π»ΠΈΠ½ΠΎΠΉ l Π² ΠΌΠ°Π³Π½ΠΈΡΠ½ΠΎΠΌ ΠΏΠΎΠ»Π΅ B ΡΠΎ ΡΠΊΠΎΡΠΎΡΡΡΡ v Π½Π° Π΅Π³ΠΎ ΠΊΠΎΠ½ΡΠ°Ρ Π²ΠΎΠ·Π½ΠΈΠΊΠ°Π΅Ρ ΡΠ°Π·Π½ΠΎΡΡΡ ΠΏΠΎΡΠ΅Π½ΡΠΈΠ°Π»ΠΎΠ², Π²ΡΠ·Π²Π°Π½Π½Π°Ρ Π΄Π΅ΠΉΡΡΠ²ΠΈΠ΅ΠΌ ΡΠΈΠ»Ρ ΠΠΎΡΠ΅Π½ΡΠ° Π½Π° ΡΠ²ΠΎΠ±ΠΎΠ΄Π½ΡΠ΅ ΡΠ»Π΅ΠΊΡΡΠΎΠ½Ρ Π² ΠΏΡΠΎΠ²ΠΎΠ΄Π½ΠΈΠΊΠ΅. ΠΡΡ ΡΠ°Π·Π½ΠΎΡΡΡ ΠΏΠΎΡΠ΅Π½ΡΠΈΠ°Π»ΠΎΠ² (ΡΡΡΠΎΠ³ΠΎ Π³ΠΎΠ²ΠΎΡΡ, ΠΠΠ‘) Π½Π°Ρ ΠΎΠ΄ΡΡ ΠΏΠΎ ΡΠΎΡΠΌΡΠ»Π΅:
Π³Π΄Π΅: Ξ± β ΡΠ³ΠΎΠ», ΠΊΠΎΡΠΎΡΡΠΉ ΠΈΠ·ΠΌΠ΅ΡΡΠ΅ΡΡΡ ΠΌΠ΅ΠΆΠ΄Ρ Π½Π°ΠΏΡΠ°Π²Π»Π΅Π½ΠΈΠ΅ΠΌ ΡΠΊΠΎΡΠΎΡΡΠΈ ΠΈ Π²Π΅ΠΊΡΠΎΡΠ° ΠΌΠ°Π³Π½ΠΈΡΠ½ΠΎΠΉ ΠΈΠ½Π΄ΡΠΊΡΠΈΠΈ. Π Π½Π΅ΠΏΠΎΠ΄Π²ΠΈΠΆΠ½ΡΡ ΡΠ°ΡΡΡΡ ΠΊΠΎΠ½ΡΡΡΠ° ΠΠΠ‘ Π½Π΅ Π²ΠΎΠ·Π½ΠΈΠΊΠ°Π΅Ρ.
ΠΡΠ»ΠΈΒ ΡΡΠ΅ΡΠΆΠ΅Π½Ρ Π΄Π»ΠΈΠ½ΠΎΠΉ L Π²ΡΠ°ΡΠ°Π΅ΡΡΡ Π² ΠΌΠ°Π³Π½ΠΈΡΠ½ΠΎΠΌ ΠΏΠΎΠ»Π΅ Π Π²ΠΎΠΊΡΡΠ³ ΠΎΠ΄Π½ΠΎΠ³ΠΎ ΠΈΠ· ΡΠ²ΠΎΠΈΡ ΠΊΠΎΠ½ΡΠΎΠ² Ρ ΡΠ³Π»ΠΎΠ²ΠΎΠΉ ΡΠΊΠΎΡΠΎΡΡΡΡ Ο, ΡΠΎ Π½Π° Π΅Π³ΠΎ ΠΊΠΎΠ½ΡΠ°Ρ Π²ΠΎΠ·Π½ΠΈΠΊΠ½Π΅Ρ ΡΠ°Π·Π½ΠΎΡΡΡ ΠΏΠΎΡΠ΅Π½ΡΠΈΠ°Π»ΠΎΠ² (ΠΠΠ‘), ΠΊΠΎΡΠΎΡΡΡ ΠΌΠΎΠΆΠ½ΠΎ ΡΠ°ΡΡΡΠΈΡΠ°ΡΡ ΠΏΠΎ ΡΠΎΡΠΌΡΠ»Π΅:
ΠΠ½Π΄ΡΠΊΡΠΈΠ²Π½ΠΎΡΡΡ.
Π‘Π°ΠΌΠΎΠΈΠ½Π΄ΡΠΊΡΠΈΡ. ΠΠ½Π΅ΡΠ³ΠΈΡ ΠΌΠ°Π³Π½ΠΈΡΠ½ΠΎΠ³ΠΎ ΠΏΠΎΠ»ΡΠ ΠΎΠ³Π»Π°Π²Π»Π΅Π½ΠΈΡβ¦
Π‘Π°ΠΌΠΎΠΈΠ½Π΄ΡΠΊΡΠΈΡ ΡΠ²Π»ΡΠ΅ΡΡΡ Π²Π°ΠΆΠ½ΡΠΌ ΡΠ°ΡΡΠ½ΡΠΌ ΡΠ»ΡΡΠ°Π΅ΠΌ ΡΠ»Π΅ΠΊΡΡΠΎΠΌΠ°Π³Π½ΠΈΡΠ½ΠΎΠΉ ΠΈΠ½Π΄ΡΠΊΡΠΈΠΈ, ΠΊΠΎΠ³Π΄Π° ΠΈΠ·ΠΌΠ΅Π½ΡΡΡΠΈΠΉΡΡ ΠΌΠ°Π³Π½ΠΈΡΠ½ΡΠΉ ΠΏΠΎΡΠΎΠΊ, Π²ΡΠ·ΡΠ²Π°ΡΡΠΈΠΉ ΠΠΠ‘ ΠΈΠ½Π΄ΡΠΊΡΠΈΠΈ, ΡΠΎΠ·Π΄Π°Π΅ΡΡΡ ΡΠΎΠΊΠΎΠΌ Π² ΡΠ°ΠΌΠΎΠΌ ΠΊΠΎΠ½ΡΡΡΠ΅.
ΠΡΠ»ΠΈ ΡΠΎΠΊ Π² ΡΠ°ΡΡΠΌΠ°ΡΡΠΈΠ²Π°Π΅ΠΌΠΎΠΌ ΠΊΠΎΠ½ΡΡΡΠ΅ ΠΏΠΎ ΠΊΠ°ΠΊΠΈΠΌ-ΡΠΎ ΠΏΡΠΈΡΠΈΠ½Π°ΠΌ ΠΈΠ·ΠΌΠ΅Π½ΡΠ΅ΡΡΡ, ΡΠΎ ΠΈΠ·ΠΌΠ΅Π½ΡΠ΅ΡΡΡ ΠΈ ΠΌΠ°Π³Π½ΠΈΡΠ½ΠΎΠ΅ ΠΏΠΎΠ»Π΅ ΡΡΠΎΠ³ΠΎ ΡΠΎΠΊΠ°, Π°, ΡΠ»Π΅Π΄ΠΎΠ²Π°ΡΠ΅Π»ΡΠ½ΠΎ, ΠΈ ΡΠΎΠ±ΡΡΠ²Π΅Π½Π½ΡΠΉ ΠΌΠ°Π³Π½ΠΈΡΠ½ΡΠΉ ΠΏΠΎΡΠΎΠΊ, ΠΏΡΠΎΠ½ΠΈΠ·ΡΠ²Π°ΡΡΠΈΠΉ ΠΊΠΎΠ½ΡΡΡ. Π ΠΊΠΎΠ½ΡΡΡΠ΅ Π²ΠΎΠ·Π½ΠΈΠΊΠ°Π΅Ρ ΠΠΠ‘ ΡΠ°ΠΌΠΎΠΈΠ½Π΄ΡΠΊΡΠΈΠΈ, ΠΊΠΎΡΠΎΡΠ°Ρ ΡΠΎΠ³Π»Π°ΡΠ½ΠΎ ΠΏΡΠ°Π²ΠΈΠ»Ρ ΠΠ΅Π½ΡΠ° ΠΏΡΠ΅ΠΏΡΡΡΡΠ²ΡΠ΅Ρ ΠΈΠ·ΠΌΠ΅Π½Π΅Π½ΠΈΡ ΡΠΎΠΊΠ° Π² ΠΊΠΎΠ½ΡΡΡΠ΅.
Π‘ΠΎΠ±ΡΡΠ²Π΅Π½Π½ΡΠΉ ΠΌΠ°Π³Π½ΠΈΡΠ½ΡΠΉ ΠΏΠΎΡΠΎΠΊ Ξ¦, ΠΏΡΠΎΠ½ΠΈΠ·ΡΠ²Π°ΡΡΠΈΠΉ ΠΊΠΎΠ½ΡΡΡ ΠΈΠ»ΠΈ ΠΊΠ°ΡΡΡΠΊΡ Ρ ΡΠΎΠΊΠΎΠΌ, ΠΏΡΠΎΠΏΠΎΡΡΠΈΠΎΠ½Π°Π»Π΅Π½ ΡΠΈΠ»Π΅ ΡΠΎΠΊΠ° I:
ΠΠΎΡΡΡΠΈΡΠΈΠ΅Π½Ρ ΠΏΡΠΎΠΏΠΎΡΡΠΈΠΎΠ½Π°Π»ΡΠ½ΠΎΡΡΠΈ L Π² ΡΡΠΎΠΉ ΡΠΎΡΠΌΡΠ»Π΅ Π½Π°Π·ΡΠ²Π°Π΅ΡΡΡ ΠΊΠΎΡΡΡΠΈΡΠΈΠ΅Π½ΡΠΎΠΌ ΡΠ°ΠΌΠΎΠΈΠ½Π΄ΡΠΊΡΠΈΠΈ ΠΈΠ»ΠΈ ΠΈΠ½Π΄ΡΠΊΡΠΈΠ²Π½ΠΎΡΡΡΡ ΠΊΠ°ΡΡΡΠΊΠΈ. ΠΠ΄ΠΈΠ½ΠΈΡΠ° ΠΈΠ½Π΄ΡΠΊΡΠΈΠ²Π½ΠΎΡΡΠΈ Π² Π‘Π Π½Π°Π·ΡΠ²Π°Π΅ΡΡΡ ΠΠ΅Π½ΡΠΈ (ΠΠ½).
ΠΠ°ΠΏΠΎΠΌΠ½ΠΈΡΠ΅: ΠΈΠ½Π΄ΡΠΊΡΠΈΠ²Π½ΠΎΡΡΡ ΠΊΠΎΠ½ΡΡΡΠ° Π½Π΅ Π·Π°Π²ΠΈΡΠΈΡ Π½ΠΈ ΠΎΡ ΠΌΠ°Π³Π½ΠΈΡΠ½ΠΎΠ³ΠΎ ΠΏΠΎΡΠΎΠΊΠ°, Π½ΠΈ ΠΎΡ ΡΠΈΠ»Ρ ΡΠΎΠΊΠ° Π² Π½Π΅ΠΌ, Π° ΠΎΠΏΡΠ΅Π΄Π΅Π»ΡΠ΅ΡΡΡ ΡΠΎΠ»ΡΠΊΠΎ ΡΠΎΡΠΌΠΎΠΉ ΠΈ ΡΠ°Π·ΠΌΠ΅ΡΠ°ΠΌΠΈ ΠΊΠΎΠ½ΡΡΡΠ°, Π° ΡΠ°ΠΊΠΆΠ΅ ΡΠ²ΠΎΠΉΡΡΠ²Π°ΠΌΠΈ ΠΎΠΊΡΡΠΆΠ°ΡΡΠ΅ΠΉ ΡΡΠ΅Π΄Ρ. ΠΠΎΡΡΠΎΠΌΡ ΠΏΡΠΈ ΠΈΠ·ΠΌΠ΅Π½Π΅Π½ΠΈΠΈ ΡΠΈΠ»Ρ ΡΠΎΠΊΠ° Π² ΠΊΠΎΠ½ΡΡΡΠ΅ ΠΈΠ½Π΄ΡΠΊΡΠΈΠ²Π½ΠΎΡΡΡ ΠΎΡΡΠ°Π΅ΡΡΡ Π½Π΅ΠΈΠ·ΠΌΠ΅Π½Π½ΠΎΠΉ. ΠΠ½Π΄ΡΠΊΡΠΈΠ²Π½ΠΎΡΡΡ ΠΊΠ°ΡΡΡΠΊΠΈ ΠΌΠΎΠΆΠ½ΠΎ ΡΠ°ΡΡΡΠΈΡΠ°ΡΡ ΠΏΠΎ ΡΠΎΡΠΌΡΠ»Π΅:
- Π³Π΄Π΅: n β ΠΊΠΎΠ½ΡΠ΅Π½ΡΡΠ°ΡΠΈΡ Π²ΠΈΡΠΊΠΎΠ² Π½Π° Π΅Π΄ΠΈΠ½ΠΈΡΡ Π΄Π»ΠΈΠ½Ρ ΠΊΠ°ΡΡΡΠΊΠΈ:
- ΠΠΠ‘ ΡΠ°ΠΌΠΎΠΈΠ½Π΄ΡΠΊΡΠΈΠΈ, Π²ΠΎΠ·Π½ΠΈΠΊΠ°ΡΡΠ°Ρ Π² ΠΊΠ°ΡΡΡΠΊΠ΅ Ρ ΠΏΠΎΡΡΠΎΡΠ½Π½ΡΠΌ Π·Π½Π°ΡΠ΅Π½ΠΈΠ΅ΠΌ ΠΈΠ½Π΄ΡΠΊΡΠΈΠ²Π½ΠΎΡΡΠΈ, ΡΠΎΠ³Π»Π°ΡΠ½ΠΎ ΡΠΎΡΠΌΡΠ»Π΅ Π€Π°ΡΠ°Π΄Π΅Ρ ΡΠ°Π²Π½Π°:
- ΠΡΠ°ΠΊ ΠΠΠ‘ ΡΠ°ΠΌΠΎΠΈΠ½Π΄ΡΠΊΡΠΈΠΈ ΠΏΡΡΠΌΠΎ ΠΏΡΠΎΠΏΠΎΡΡΠΈΠΎΠ½Π°Π»ΡΠ½Π° ΠΈΠ½Π΄ΡΠΊΡΠΈΠ²Π½ΠΎΡΡΠΈ ΠΊΠ°ΡΡΡΠΊΠΈ ΠΈ ΡΠΊΠΎΡΠΎΡΡΠΈ ΠΈΠ·ΠΌΠ΅Π½Π΅Π½ΠΈΡ ΡΠΈΠ»Ρ ΡΠΎΠΊΠ° Π² Π½Π΅ΠΉ.
ΠΠ°Π³Π½ΠΈΡΠ½ΠΎΠ΅ ΠΏΠΎΠ»Π΅ ΠΎΠ±Π»Π°Π΄Π°Π΅Ρ ΡΠ½Π΅ΡΠ³ΠΈΠ΅ΠΉ. ΠΠΎΠ΄ΠΎΠ±Π½ΠΎ ΡΠΎΠΌΡ, ΠΊΠ°ΠΊ Π² Π·Π°ΡΡΠΆΠ΅Π½Π½ΠΎΠΌ ΠΊΠΎΠ½Π΄Π΅Π½ΡΠ°ΡΠΎΡΠ΅ ΠΈΠΌΠ΅Π΅ΡΡΡ Π·Π°ΠΏΠ°Ρ ΡΠ»Π΅ΠΊΡΡΠΈΡΠ΅ΡΠΊΠΎΠΉ ΡΠ½Π΅ΡΠ³ΠΈΠΈ, Π² ΠΊΠ°ΡΡΡΠΊΠ΅, ΠΏΠΎ Π²ΠΈΡΠΊΠ°ΠΌ ΠΊΠΎΡΠΎΡΠΎΠΉ ΠΏΡΠΎΡΠ΅ΠΊΠ°Π΅Ρ ΡΠΎΠΊ, ΠΈΠΌΠ΅Π΅ΡΡΡ Π·Π°ΠΏΠ°Ρ ΠΌΠ°Π³Π½ΠΈΡΠ½ΠΎΠΉ ΡΠ½Π΅ΡΠ³ΠΈΠΈ. ΠΠ½Π΅ΡΠ³ΠΈΡ WΠΌ ΠΌΠ°Π³Π½ΠΈΡΠ½ΠΎΠ³ΠΎ ΠΏΠΎΠ»Ρ ΠΊΠ°ΡΡΡΠΊΠΈ Ρ ΠΈΠ½Π΄ΡΠΊΡΠΈΠ²Π½ΠΎΡΡΡΡ L, ΡΠΎΠ·Π΄Π°Π²Π°Π΅ΠΌΠΎΠ³ΠΎ ΡΠΎΠΊΠΎΠΌ I, ΠΌΠΎΠΆΠ΅Ρ Π±ΡΡΡ ΡΠ°ΡΡΡΠΈΡΠ°Π½Π° ΠΏΠΎ ΠΎΠ΄Π½ΠΎΠΉ ΠΈΠ· ΡΠΎΡΠΌΡΠ» (ΠΎΠ½ΠΈ ΡΠ»Π΅Π΄ΡΡΡ Π΄ΡΡΠ³ ΠΈΠ· Π΄ΡΡΠ³Π° Ρ ΡΡΡΡΠΎΠΌ ΡΠΎΡΠΌΡΠ»Ρ Ξ¦ = LI):
- Π‘ΠΎΠΎΡΠ½Π΅ΡΡ ΡΠΎΡΠΌΡΠ»Ρ Π΄Π»Ρ ΡΠ½Π΅ΡΠ³ΠΈΠΈ ΠΌΠ°Π³Π½ΠΈΡΠ½ΠΎΠ³ΠΎ ΠΏΠΎΠ»Ρ ΠΊΠ°ΡΡΡΠΊΠΈ Ρ Π΅Ρ Π³Π΅ΠΎΠΌΠ΅ΡΡΠΈΡΠ΅ΡΠΊΠΈΠΌΠΈ ΡΠ°Π·ΠΌΠ΅ΡΠ°ΠΌΠΈ ΠΌΠΎΠΆΠ½ΠΎ ΠΏΠΎΠ»ΡΡΠΈΡΡ ΡΠΎΡΠΌΡΠ»Ρ Π΄Π»Ρ ΠΎΠ±ΡΠ΅ΠΌΠ½ΠΎΠΉ ΠΏΠ»ΠΎΡΠ½ΠΎΡΡΠΈ ΡΠ½Π΅ΡΠ³ΠΈΠΈ ΠΌΠ°Π³Π½ΠΈΡΠ½ΠΎΠ³ΠΎ ΠΏΠΎΠ»Ρ (ΠΈΠ»ΠΈ ΡΠ½Π΅ΡΠ³ΠΈΠΈ Π΅Π΄ΠΈΠ½ΠΈΡΡ ΠΎΠ±ΡΡΠΌΠ°):
ΠΡΠ°Π²ΠΈΠ»ΠΎ ΠΠ΅Π½ΡΠ°
Π ΠΎΠ³Π»Π°Π²Π»Π΅Π½ΠΈΡβ¦
ΠΠ½Π΅ΡΡΠΈΡ β ΡΠ²Π»Π΅Π½ΠΈΠ΅, ΠΏΡΠΎΠΈΡΡ ΠΎΠ΄ΡΡΠ΅Π΅ ΠΈ Π² ΠΌΠ΅Ρ Π°Π½ΠΈΠΊΠ΅ (ΠΏΡΠΈ ΡΠ°Π·Π³ΠΎΠ½Π΅ Π°Π²ΡΠΎΠΌΠΎΠ±ΠΈΠ»Ρ ΠΌΡ ΠΎΡΠΊΠ»ΠΎΠ½ΡΠ΅ΠΌΡΡ Π½Π°Π·Π°Π΄, ΠΏΡΠΎΡΠΈΠ²ΠΎΠ΄Π΅ΠΉΡΡΠ²ΡΡ ΡΠ²Π΅Π»ΠΈΡΠ΅Π½ΠΈΡ ΡΠΊΠΎΡΠΎΡΡΠΈ, Π° ΠΏΡΠΈ ΡΠΎΡΠΌΠΎΠΆΠ΅Π½ΠΈΠΈ ΠΎΡΠΊΠ»ΠΎΠ½ΡΠ΅ΠΌΡΡ Π²ΠΏΠ΅ΡΡΠ΄, ΠΏΡΠΎΡΠΈΠ²ΠΎΠ΄Π΅ΠΉΡΡΠ²ΡΡ ΡΠΌΠ΅Π½ΡΡΠ΅Π½ΠΈΡ ΡΠΊΠΎΡΠΎΡΡΠΈ), ΠΈ Π² ΠΌΠΎΠ»Π΅ΠΊΡΠ»ΡΡΠ½ΠΎΠΉ ΡΠΈΠ·ΠΈΠΊΠ΅ (ΠΏΡΠΈ Π½Π°Π³ΡΠ΅Π²Π°Π½ΠΈΠΈ ΠΆΠΈΠ΄ΠΊΠΎΡΡΠΈ ΡΠ²Π΅Π»ΠΈΡΠΈΠ²Π°Π΅ΡΡΡ ΡΠΊΠΎΡΠΎΡΡΡ ΠΈΡΠΏΠ°ΡΠ΅Π½ΠΈΡ, ΡΠ°ΠΌΡΠ΅ Π±ΡΡΡΡΡΠ΅ ΠΌΠΎΠ»Π΅ΠΊΡΠ»Ρ ΠΏΠΎΠΊΠΈΠ΄Π°ΡΡ ΠΆΠΈΠ΄ΠΊΠΎΡΡΡ, ΡΠΌΠ΅Π½ΡΡΠ°Ρ ΡΠΊΠΎΡΠΎΡΡΡ Π½Π°Π³ΡΠ΅Π²Π°Π½ΠΈΡ) ΠΈ ΡΠ°ΠΊ Π΄Π°Π»Π΅Π΅.
Π ΡΠ»Π΅ΠΊΡΡΠΎΠΌΠ°Π³Π½Π΅ΡΠΈΠ·ΠΌΠ΅ ΠΈΠ½Π΅ΡΡΠΈΡ ΠΏΡΠΎΡΠ²Π»ΡΠ΅ΡΡΡ Π² ΠΏΡΠΎΡΠΈΠ²ΠΎΠ΄Π΅ΠΉΡΡΠ²ΠΈΠΈ ΠΈΠ·ΠΌΠ΅Π½Π΅Π½ΠΈΡ ΠΌΠ°Π³Π½ΠΈΡΠ½ΠΎΠ³ΠΎ ΠΏΠΎΡΠΎΠΊΠ°, ΠΏΡΠΎΠ½ΠΈΠ·ΡΠ²Π°ΡΡΠ΅Π³ΠΎ ΠΊΠΎΠ½ΡΡΡ. ΠΡΠ»ΠΈ ΠΌΠ°Π³Π½ΠΈΡΠ½ΡΠΉ ΠΏΠΎΡΠΎΠΊ Π½Π°ΡΠ°ΡΡΠ°Π΅Ρ, ΡΠΎ Π²ΠΎΠ·Π½ΠΈΠΊΠ°ΡΡΠΈΠΉ Π² ΠΊΠΎΠ½ΡΡΡΠ΅ ΠΈΠ½Π΄ΡΠΊΡΠΈΠΎΠ½Π½ΡΠΉ ΡΠΎΠΊ Π½Π°ΠΏΡΠ°Π²Π»Π΅Π½ ΡΠ°ΠΊ, ΡΡΠΎΠ±Ρ ΠΏΡΠ΅ΠΏΡΡΡΡΠ²ΠΎΠ²Π°ΡΡ Π½Π°ΡΠ°ΡΡΠ°Π½ΠΈΡ ΠΌΠ°Π³Π½ΠΈΡΠ½ΠΎΠ³ΠΎ ΠΏΠΎΡΠΎΠΊΠ°, Π° Π΅ΡΠ»ΠΈ ΠΌΠ°Π³Π½ΠΈΡΠ½ΡΠΉ ΠΏΠΎΡΠΎΠΊ ΡΠ±ΡΠ²Π°Π΅Ρ, ΡΠΎ Π²ΠΎΠ·Π½ΠΈΠΊΠ°ΡΡΠΈΠΉ Π² ΠΊΠΎΠ½ΡΡΡΠ΅ ΠΈΠ½Π΄ΡΠΊΡΠΈΠΎΠ½Π½ΡΠΉ ΡΠΎΠΊ Π½Π°ΠΏΡΠ°Π²Π»Π΅Π½ ΡΠ°ΠΊ, ΡΡΠΎΠ±Ρ ΠΏΡΠ΅ΠΏΡΡΡΡΠ²ΠΎΠ²Π°ΡΡ ΡΠ±ΡΠ²Π°Π½ΠΈΡ ΠΌΠ°Π³Π½ΠΈΡΠ½ΠΎΠ³ΠΎ ΠΏΠΎΡΠΎΠΊΠ°.
ΠΡΠ°Π²ΠΈΠ»ΠΎ ΠΠ΅Π½ΡΠ° Π΄Π»Ρ ΠΎΠΏΡΠ΅Π΄Π΅Π»Π΅Π½ΠΈΡ Π½Π°ΠΏΡΠ°Π²Π»Π΅Π½ΠΈΡ ΠΈΠ½Π΄ΡΠΊΡΠΈΠΎΠ½Π½ΠΎΠ³ΠΎ ΡΠΎΠΊΠ°: Π²ΠΎΠ·Π½ΠΈΠΊΠ°ΡΡΠΈΠΉ Π² ΠΊΠΎΠ½ΡΡΡΠ΅ ΠΈΠ½Π΄ΡΠΊΡΠΈΠΎΠ½Π½ΡΠΉ ΡΠΎΠΊ ΠΈΠΌΠ΅Π΅Ρ ΡΠ°ΠΊΠΎΠ΅ Π½Π°ΠΏΡΠ°Π²Π»Π΅Π½ΠΈΠ΅, ΡΡΠΎ ΡΠΎΠ·Π΄Π°Π²Π°Π΅ΠΌΠΎΠ΅ ΠΈΠΌ ΠΌΠ°Π³Π½ΠΈΡΠ½ΠΎΠ΅ ΠΏΠΎΠ»Π΅ ΠΏΡΠ΅ΠΏΡΡΡΡΠ²ΡΠ΅Ρ ΠΈΠ·ΠΌΠ΅Π½Π΅Π½ΠΈΡ ΠΌΠ°Π³Π½ΠΈΡΠ½ΠΎΠ³ΠΎ ΠΏΠΎΡΠΎΠΊΠ°, ΠΊΠΎΡΠΎΡΠΎΠ΅ Π²ΡΠ·ΡΠ²Π°Π»ΠΎ ΡΡΠΎΡ ΡΠΎΠΊ.
ΠΡΡΠΎΡΠ½ΠΈΠΊ: https://educon.by/index.php/materials/phys/magnetizm
13.4: ΠΠΠ‘ Π΄Π²ΠΈΠΆΠ΅Π½ΠΈΡ — Π€ΠΈΠ·ΠΈΠΊΠ° LibreTexts
- ΠΠΎΡΠ»Π΅Π΄Π½Π΅Π΅ ΠΎΠ±Π½ΠΎΠ²Π»Π΅Π½ΠΈΠ΅
- Π‘ΠΎΡ ΡΠ°Π½ΠΈΡΡ ΠΊΠ°ΠΊ PDF
- ΠΠ΄Π΅Π½ΡΠΈΡΠΈΠΊΠ°ΡΠΎΡ ΡΡΡΠ°Π½ΠΈΡΡ
- 4430
- OpenStax
- OpenStax
Π¦Π΅Π»ΠΈ ΠΎΠ±ΡΡΠ΅Π½ΠΈΡ
Π ΠΊΠΎΠ½ΡΡ ΡΡΠΎΠ³ΠΎ ΡΠ°Π·Π΄Π΅Π»Π° Π²Ρ ΡΠΌΠΎΠΆΠ΅ΡΠ΅:
- ΠΠΏΡΠ΅Π΄Π΅Π»ΡΡΡ Π²Π΅Π»ΠΈΡΠΈΠ½Ρ ΠΠΠ‘ ΠΈΠ½Π΄ΡΠΊΡΠΈΠΈ Π² ΠΏΡΠΎΠ²ΠΎΠ΄Π΅, Π΄Π²ΠΈΠΆΡΡΠ΅ΠΌΡΡ Ρ ΠΏΠΎΡΡΠΎΡΠ½Π½ΠΎΠΉ ΡΠΊΠΎΡΠΎΡΡΡΡ ΡΠ΅ΡΠ΅Π· ΠΌΠ°Π³Π½ΠΈΡΠ½ΠΎΠ΅ ΠΏΠΎΠ»Π΅
- ΠΠ±ΡΡΠ΄ΠΈΡΠ΅ ΠΏΡΠΈΠΌΠ΅ΡΡ, ΠΈΡΠΏΠΎΠ»ΡΠ·ΡΡΡΠΈΠ΅ ΠΠΠ‘ Π΄Π²ΠΈΠΆΠ΅Π½ΠΈΡ, ΡΠ°ΠΊΠΈΠ΅ ΠΊΠ°ΠΊ ΡΠ΅Π»ΡΡΠΎΠ²Π°Ρ ΠΏΡΡΠΊΠ° ΠΈ ΠΏΡΠΈΠ²ΡΠ·Π°Π½Π½ΡΠΉ ΡΠΏΡΡΠ½ΠΈΠΊ
ΠΠ°Π³Π½ΠΈΡΠ½ΡΠΉ ΠΏΠΎΡΠΎΠΊ Π·Π°Π²ΠΈΡΠΈΡ ΠΎΡ ΡΡΠ΅Ρ ΡΠ°ΠΊΡΠΎΡΠΎΠ²: ΡΠΈΠ»Ρ ΠΌΠ°Π³Π½ΠΈΡΠ½ΠΎΠ³ΠΎ ΠΏΠΎΠ»Ρ, ΠΏΠ»ΠΎΡΠ°Π΄ΠΈ, ΡΠ΅ΡΠ΅Π· ΠΊΠΎΡΠΎΡΡΡ ΠΏΡΠΎΡ ΠΎΠ΄ΡΡ ΡΠΈΠ»ΠΎΠ²ΡΠ΅ Π»ΠΈΠ½ΠΈΠΈ, ΠΈ ΠΎΡΠΈΠ΅Π½ΡΠ°ΡΠΈΠΈ ΠΏΠΎΠ»Ρ Ρ ΠΏΠ»ΠΎΡΠ°Π΄ΡΡ ΠΏΠΎΠ²Π΅ΡΡ Π½ΠΎΡΡΠΈ. ΠΡΠ»ΠΈ ΠΊΠ°ΠΊΠ°Ρ-Π»ΠΈΠ±ΠΎ ΠΈΠ· ΡΡΠΈΡ Π²Π΅Π»ΠΈΡΠΈΠ½ ΠΈΠ·ΠΌΠ΅Π½ΡΠ΅ΡΡΡ, ΠΏΡΠΎΠΈΡΡ ΠΎΠ΄ΠΈΡ ΡΠΎΠΎΡΠ²Π΅ΡΡΡΠ²ΡΡΡΠ΅Π΅ ΠΈΠ·ΠΌΠ΅Π½Π΅Π½ΠΈΠ΅ ΠΌΠ°Π³Π½ΠΈΡΠ½ΠΎΠ³ΠΎ ΠΏΠΎΡΠΎΠΊΠ°. ΠΠΎ ΡΠΈΡ ΠΏΠΎΡ ΠΌΡ ΡΠ°ΡΡΠΌΠ°ΡΡΠΈΠ²Π°Π»ΠΈ ΡΠΎΠ»ΡΠΊΠΎ ΠΈΠ·ΠΌΠ΅Π½Π΅Π½ΠΈΡ ΠΏΠΎΡΠΎΠΊΠ° ΠΈΠ·-Π·Π° ΠΈΠ·ΠΌΠ΅Π½ΡΡΡΠ΅Π³ΠΎΡΡ ΠΏΠΎΠ»Ρ. Π’Π΅ΠΏΠ΅ΡΡ ΠΌΡ ΡΠ°ΡΡΠΌΠΎΡΡΠΈΠΌ Π΄ΡΡΠ³ΡΡ Π²ΠΎΠ·ΠΌΠΎΠΆΠ½ΠΎΡΡΡ: ΠΈΠ·ΠΌΠ΅Π½Π΅Π½ΠΈΠ΅ ΠΎΠ±Π»Π°ΡΡΠΈ, ΡΠ΅ΡΠ΅Π· ΠΊΠΎΡΠΎΡΡΡ ΠΏΡΠΎΡ ΠΎΠ΄ΡΡ ΡΠΈΠ»ΠΎΠ²ΡΠ΅ Π»ΠΈΠ½ΠΈΠΈ, Π²ΠΊΠ»ΡΡΠ°Ρ ΠΈΠ·ΠΌΠ΅Π½Π΅Π½ΠΈΠ΅ ΠΎΡΠΈΠ΅Π½ΡΠ°ΡΠΈΠΈ ΠΎΠ±Π»Π°ΡΡΠΈ.
ΠΠ²Π° ΠΏΡΠΈΠΌΠ΅ΡΠ° ΡΠ°ΠΊΠΎΠ³ΠΎ ΠΈΠ·ΠΌΠ΅Π½Π΅Π½ΠΈΡ ΠΏΠΎΡΠΎΠΊΠ° ΠΏΡΠ΅Π΄ΡΡΠ°Π²Π»Π΅Π½Ρ Π½Π° ΡΠΈΡΡΠ½ΠΊΠ΅ \(\PageIndex{1}\). Π ΡΠ°ΡΡΠΈ (Π°) ΠΏΠΎΡΠΎΠΊ ΡΠ΅ΡΠ΅Π· ΠΏΡΡΠΌΠΎΡΠ³ΠΎΠ»ΡΠ½ΡΡ ΠΏΠ΅ΡΠ»Ρ ΡΠ²Π΅Π»ΠΈΡΠΈΠ²Π°Π΅ΡΡΡ ΠΏΠΎ ΠΌΠ΅ΡΠ΅ Π΅Π΅ ΠΏΠ΅ΡΠ΅ΠΌΠ΅ΡΠ΅Π½ΠΈΡ Π² ΠΌΠ°Π³Π½ΠΈΡΠ½ΠΎΠ΅ ΠΏΠΎΠ»Π΅, Π° Π² ΡΠ°ΡΡΠΈ (Π±) ΠΏΠΎΡΠΎΠΊ ΡΠ΅ΡΠ΅Π· Π²ΡΠ°ΡΠ°ΡΡΡΡΡΡ ΠΊΠ°ΡΡΡΠΊΡ ΠΈΠ·ΠΌΠ΅Π½ΡΠ΅ΡΡΡ Π² Π·Π°Π²ΠΈΡΠΈΠΌΠΎΡΡΠΈ ΠΎΡ ΡΠ³Π»Π° \(\theta\).
Π ΠΈΡΡΠ½ΠΎΠΊ \(\PageIndex{1}\): (a) ΠΈΠ·ΠΌΠ΅Π½Π΅Π½ΠΈΠ΅ ΠΌΠ°Π³Π½ΠΈΡΠ½ΠΎΠ³ΠΎ ΠΏΠΎΡΠΎΠΊΠ° ΠΏΠΎ ΠΌΠ΅ΡΠ΅ ΡΠΎΠ³ΠΎ, ΠΊΠ°ΠΊ ΠΏΠ΅ΡΠ»Ρ ΠΏΠ΅ΡΠ΅ΠΌΠ΅ΡΠ°Π΅ΡΡΡ Π² ΠΌΠ°Π³Π½ΠΈΡΠ½ΠΎΠ΅ ΠΏΠΎΠ»Π΅; Π±) ΠΈΠ·ΠΌΠ΅Π½Π΅Π½ΠΈΠ΅ ΠΌΠ°Π³Π½ΠΈΡΠ½ΠΎΠ³ΠΎ ΠΏΠΎΡΠΎΠΊΠ° ΠΏΡΠΈ Π²ΡΠ°ΡΠ΅Π½ΠΈΠΈ ΠΏΠ΅ΡΠ»ΠΈ Π² ΠΌΠ°Π³Π½ΠΈΡΠ½ΠΎΠΌ ΠΏΠΎΠ»Π΅.ΠΠ½ΡΠ΅ΡΠ΅ΡΠ½ΠΎ ΠΎΡΠΌΠ΅ΡΠΈΡΡ, ΡΡΠΎ ΡΠΎ, ΡΡΠΎ ΠΌΡ Π²ΠΎΡΠΏΡΠΈΠ½ΠΈΠΌΠ°Π΅ΠΌ ΠΊΠ°ΠΊ ΠΏΡΠΈΡΠΈΠ½Ρ ΠΊΠΎΠ½ΠΊΡΠ΅ΡΠ½ΠΎΠ³ΠΎ ΠΈΠ·ΠΌΠ΅Π½Π΅Π½ΠΈΡ ΠΏΠΎΡΠΎΠΊΠ°, Π½Π° ΡΠ°ΠΌΠΎΠΌ Π΄Π΅Π»Π΅ Π·Π°Π²ΠΈΡΠΈΡ ΠΎΡ Π²ΡΠ±ΡΠ°Π½Π½ΠΎΠΉ Π½Π°ΠΌΠΈ ΡΠΈΡΡΠ΅ΠΌΡ ΠΎΡΡΡΠ΅ΡΠ°. ΠΠ°ΠΏΡΠΈΠΌΠ΅Ρ, Π΅ΡΠ»ΠΈ Π²Ρ Π½Π°Ρ ΠΎΠ΄ΠΈΡΠ΅ΡΡ Π² ΡΠΎΡΡΠΎΡΠ½ΠΈΠΈ ΠΏΠΎΠΊΠΎΡ ΠΎΡΠ½ΠΎΡΠΈΡΠ΅Π»ΡΠ½ΠΎ Π΄Π²ΠΈΠΆΡΡΠΈΡ ΡΡ ΠΊΠ°ΡΡΡΠ΅ΠΊ Π½Π° ΡΠΈΡΡΠ½ΠΊΠ΅ \(\PageIndex{1b}\), Π²Ρ ΡΠ²ΠΈΠ΄ΠΈΡΠ΅, ΡΡΠΎ ΠΏΠΎΡΠΎΠΊ ΠΌΠ΅Π½ΡΠ΅ΡΡΡ ΠΈΠ·-Π·Π° ΠΈΠ·ΠΌΠ΅Π½ΡΡΡΠ΅Π³ΠΎΡΡ ΠΌΠ°Π³Π½ΠΈΡΠ½ΠΎΠ³ΠΎ ΠΏΠΎΠ»Ρ β Π² ΡΠ°ΡΡΠΈ (Π°) ΠΏΠΎΠ»Π΅ ΠΏΠ΅ΡΠ΅ΠΌΠ΅ΡΠ°Π΅ΡΡΡ ΡΠ»Π΅Π²Π° Π½Π°ΠΏΡΠ°Π²ΠΎ. ΠΏΡΡΠΌΠΎ Π² Π²Π°ΡΠ΅ΠΉ ΡΠΈΡΡΠ΅ΠΌΠ΅ ΠΎΡΡΡΠ΅ΡΠ°, Π° Π² ΡΠ°ΡΡΠΈ (b) ΠΏΠΎΠ»Π΅ Π²ΡΠ°ΡΠ°Π΅ΡΡΡ. Π§Π°ΡΡΠΎ ΠΌΠΎΠΆΠ½ΠΎ ΠΎΠΏΠΈΡΠ°ΡΡ ΠΈΠ·ΠΌΠ΅Π½Π΅Π½ΠΈΠ΅ ΠΏΠΎΡΠΎΠΊΠ° ΡΠ΅ΡΠ΅Π· ΠΊΠ°ΡΡΡΠΊΡ, ΠΊΠΎΡΠΎΡΠ°Ρ Π΄Π²ΠΈΠΆΠ΅ΡΡΡ Π² ΠΎΠ΄Π½ΠΎΠΉ ΠΊΠΎΠ½ΠΊΡΠ΅ΡΠ½ΠΎΠΉ ΡΠΈΡΡΠ΅ΠΌΠ΅ ΠΎΡΡΡΠ΅ΡΠ°, Ρ ΡΠΎΡΠΊΠΈ Π·ΡΠ΅Π½ΠΈΡ ΠΈΠ·ΠΌΠ΅Π½Π΅Π½ΠΈΡ ΠΌΠ°Π³Π½ΠΈΡΠ½ΠΎΠ³ΠΎ ΠΏΠΎΠ»Ρ Π²ΠΎ Π²ΡΠΎΡΠΎΠΉ ΡΠΈΡΡΠ΅ΠΌΠ΅ ΠΎΡΡΡΠ΅ΡΠ°, Π³Π΄Π΅ ΠΊΠ°ΡΡΡΠΊΠ° Π½Π΅ΠΏΠΎΠ΄Π²ΠΈΠΆΠ½Π°. ΠΠ΄Π½Π°ΠΊΠΎ Π²ΠΎΠΏΡΠΎΡΡ ΡΠΈΡΡΠ΅ΠΌΡ ΠΎΡΡΡΠ΅ΡΠ°, ΡΠ²ΡΠ·Π°Π½Π½ΡΠ΅ Ρ ΠΌΠ°Π³Π½ΠΈΡΠ½ΡΠΌ ΠΏΠΎΡΠΎΠΊΠΎΠΌ, Π²ΡΡ ΠΎΠ΄ΡΡ Π·Π° ΡΠ°ΠΌΠΊΠΈ ΡΡΠΎΠ³ΠΎ ΡΡΠ΅Π±Π½ΠΈΠΊΠ°. ΠΡ ΠΈΠ·Π±Π΅ΠΆΠΈΠΌ ΡΠ°ΠΊΠΈΡ ΡΠ»ΠΎΠΆΠ½ΠΎΡΡΠ΅ΠΉ, Π²ΡΠ΅Π³Π΄Π° ΡΠ°Π±ΠΎΡΠ°Ρ Π² ΡΠΈΡΡΠ΅ΠΌΠ΅ ΠΎΡΡΡΠ΅ΡΠ°, ΠΏΠΎΠΊΠΎΡΡΠ΅ΠΉΡΡ ΠΎΡΠ½ΠΎΡΠΈΡΠ΅Π»ΡΠ½ΠΎ Π»Π°Π±ΠΎΡΠ°ΡΠΎΡΠΈΠΈ, ΠΈ ΠΎΠ±ΡΡΡΠ½ΡΡ ΠΈΠ·ΠΌΠ΅Π½Π΅Π½ΠΈΡ ΠΏΠΎΡΠΎΠΊΠ° Π»ΠΈΠ±ΠΎ ΠΈΠ·-Π·Π° ΠΈΠ·ΠΌΠ΅Π½ΡΡΡΠ΅Π³ΠΎΡΡ ΠΏΠΎΠ»Ρ, Π»ΠΈΠ±ΠΎ ΠΈΠ·-Π·Π° ΠΈΠ·ΠΌΠ΅Π½ΡΡΡΠ΅ΠΉΡΡ ΠΏΠ»ΠΎΡΠ°Π΄ΠΈ.
Π’Π΅ΠΏΠ΅ΡΡ Π΄Π°Π²Π°ΠΉΡΠ΅ ΠΏΠΎΡΠΌΠΎΡΡΠΈΠΌ Π½Π° ΡΠΎΠΊΠΎΠΏΡΠΎΠ²ΠΎΠ΄ΡΡΠΈΠΉ ΡΡΠ΅ΡΠΆΠ΅Π½Ρ, Π²ΡΡΠ½ΡΡΡΠΉ Π² ΡΠ΅ΠΏΡ, ΠΈΠ·ΠΌΠ΅Π½ΡΡΡΡΡ ΠΌΠ°Π³Π½ΠΈΡΠ½ΡΠΉ ΠΏΠΎΡΠΎΠΊ. ΠΠ»ΠΎΡΠ°Π΄Ρ, ΠΎΠ³ΡΠ°Π½ΠΈΡΠ΅Π½Π½Π°Ρ ΡΡ Π΅ΠΌΠΎΠΉ MNOP Π½Π° ΡΠΈΡΡΠ½ΠΊΠ΅ \(\PageIndex{2}\), ΡΠΎΡΡΠ°Π²Π»ΡΠ΅Ρ Π»ΠΊ x ΠΈ ΠΏΠ΅ΡΠΏΠ΅Π½Π΄ΠΈΠΊΡΠ»ΡΡΠ½Π° ΠΌΠ°Π³Π½ΠΈΡΠ½ΠΎΠΌΡ ΠΏΠΎΠ»Ρ, ΠΏΠΎΡΡΠΎΠΌΡ ΠΌΡ ΠΌΠΎΠΆΠ΅ΠΌ ΡΠΏΡΠΎΡΡΠΈΡΡ ΠΈΠ½ΡΠ΅Π³ΡΠΈΡΠΎΠ²Π°Π½ΠΈΠ΅ \(\Phi _{\mathrm{m}} =\int_{S} \overrightarrow{\mathbf{B}} \cdot \hat{\mathbf{n}} d A\) Π² ΠΏΡΠΎΠΈΠ·Π²Π΅Π΄Π΅Π½ΠΈΠ΅ ΠΌΠ°Π³Π½ΠΈΡΠ½ΠΎΠ³ΠΎ ΠΏΠΎΠ»Ρ ΠΈ ΠΏΠ»ΠΎΡΠ°Π΄ΠΈ. Π’Π°ΠΊΠΈΠΌ ΠΎΠ±ΡΠ°Π·ΠΎΠΌ, ΠΌΠ°Π³Π½ΠΈΡΠ½ΡΠΉ ΠΏΠΎΡΠΎΠΊ ΡΠ΅ΡΠ΅Π· ΠΎΡΠΊΡΡΡΡΡ ΠΏΠΎΠ²Π΅ΡΡ Π½ΠΎΡΡΡ ΡΠ°Π²Π΅Π½
\[\Phi_m = Blx. \]
Π‘ B ΠΈ l ΠΏΠΎΡΡΠΎΡΠ½Π½Ρ, Π° ΡΠΊΠΎΡΠΎΡΡΡ ΡΡΠ΅ΡΠΆΠ½Ρ ΡΠ°Π²Π½Π° \(v = dx/dt\), ΡΠ΅ΠΏΠ΅ΡΡ ΠΌΡ ΠΌΠΎΠΆΠ΅ΠΌ ΠΏΠ΅ΡΠ΅ΡΠΎΡΠΌΡΠ»ΠΈΡΠΎΠ²Π°ΡΡ Π·Π°ΠΊΠΎΠ½ Π€Π°ΡΠ°Π΄Π΅Ρ, ΡΡΠ°Π²Π½Π΅Π½ΠΈΠ΅ 13.2.2, Π΄Π»Ρ Π²Π΅Π»ΠΈΡΠΈΠ½Ρ ΠΠΠ‘ Π² ΡΠ΅ΡΠΌΠΈΠ½Π°Ρ Π΄Π²ΠΈΠΆΡΡΠ΅ΠΉΡΡ ΠΏΡΠΎΠ²ΠΎΠ΄ΡΡΠ΅ΠΉ ΡΡΠ΅ΡΠΆΠ΅Π½Ρ ΠΊΠ°ΠΊ
\[\epsilon = \frac{d\Phi_m}{dt} = Bl \frac{dx}{dt} = Blv.\]
Π’ΠΎΠΊ, ΠΈΠ½Π΄ΡΡΠΈΡΠΎΠ²Π°Π½Π½ΡΠΉ Π² ΡΠ΅ΠΏΠΈ, ΡΠ°Π²Π΅Π½ ΠΠΠ‘, Π΄Π΅Π»Π΅Π½Π½ΠΎΠΉ Π½Π° ΡΠΎΠΏΡΠΎΡΠΈΠ²Π»Π΅Π½ΠΈΠ΅ ΠΈΠ»ΠΈ
\[I = \frac{Blv}{R}.\]
ΠΡΠΎΠΌΠ΅ ΡΠΎΠ³ΠΎ, Π½Π°ΠΏΡΠ°Π²Π»Π΅Π½ΠΈΠ΅ ΠΠΠ‘ ΠΈΠ½Π΄ΡΠΊΡΠΈΠΈ ΡΠ΄ΠΎΠ²Π»Π΅ΡΠ²ΠΎΡΡΠ΅Ρ Π·Π°ΠΊΠΎΠ½Ρ ΠΠ΅Π½ΡΠ°, Π² ΡΠ΅ΠΌ ΠΌΠΎΠΆΠ½ΠΎ ΡΠ±Π΅Π΄ΠΈΡΡΡΡ, ΡΠ°ΡΡΠΌΠΎΡΡΠ΅Π² ΡΠΈΡΡΠ½ΠΎΠΊ.
ΠΡΠΎΡ ΡΠ°ΡΡΠ΅Ρ ΠΠΠ‘, Π²ΡΠ·Π²Π°Π½Π½ΠΎΠΉ Π΄Π²ΠΈΠΆΠ΅Π½ΠΈΠ΅ΠΌ, Π½Π΅ ΠΎΠ³ΡΠ°Π½ΠΈΡΠΈΠ²Π°Π΅ΡΡΡ ΡΡΠ΅ΡΠΆΠ½Π΅ΠΌ, Π΄Π²ΠΈΠΆΡΡΠΈΠΌΡΡ ΠΏΠΎ ΡΠΎΠΊΠΎΠΏΡΠΎΠ²ΠΎΠ΄ΡΡΠΈΠΌ ΡΠ΅Π»ΡΡΠ°ΠΌ. ΠΡΠΏΠΎΠ»ΡΠ·ΡΡ \(\vec{F} = q\vec{v} \times \vec{B}\) Π² ΠΊΠ°ΡΠ΅ΡΡΠ²Π΅ ΠΎΡΠΏΡΠ°Π²Π½ΠΎΠΉ ΡΠΎΡΠΊΠΈ, ΠΌΠΎΠΆΠ½ΠΎ ΠΏΠΎΠΊΠ°Π·Π°ΡΡ, ΡΡΠΎ \(\epsilon = — d\Phi_m/dt\) Π²ΡΠΏΠΎΠ»Π½ΡΠ΅ΡΡΡ Π΄Π»Ρ Π»ΡΠ±ΠΎΠ³ΠΎ ΠΈΠ·ΠΌΠ΅Π½Π΅Π½ΠΈΠ΅ ΠΏΠΎΡΠΎΠΊΠ°, Π²ΡΠ·Π²Π°Π½Π½ΠΎΠ΅ Π΄Π²ΠΈΠΆΠ΅Π½ΠΈΠ΅ΠΌ ΠΏΡΠΎΠ²ΠΎΠ΄Π½ΠΈΠΊΠ°. ΠΡ Π²ΠΈΠ΄Π΅Π»ΠΈ Π² Π·Π°ΠΊΠΎΠ½Π΅ Π€Π°ΡΠ°Π΄Π΅Ρ, ΡΡΠΎ ΠΠΠ‘, ΠΈΠ½Π΄ΡΡΠΈΡΠΎΠ²Π°Π½Π½Π°Ρ ΠΈΠ·ΠΌΠ΅Π½ΡΡΡΠΈΠΌΡΡ Π²ΠΎ Π²ΡΠ΅ΠΌΠ΅Π½ΠΈ ΠΌΠ°Π³Π½ΠΈΡΠ½ΡΠΌ ΠΏΠΎΠ»Π΅ΠΌ, ΠΏΠΎΠ΄ΡΠΈΠ½ΡΠ΅ΡΡΡ ΡΠΎΠΌΡ ΠΆΠ΅ ΡΠΎΠΎΡΠ½ΠΎΡΠ΅Π½ΠΈΡ, ΠΊΠΎΡΠΎΡΠΎΠ΅ ΡΠ²Π»ΡΠ΅ΡΡΡ Π·Π°ΠΊΠΎΠ½ΠΎΠΌ Π€Π°ΡΠ°Π΄Π΅Ρ. Π’Π°ΠΊΠΈΠΌ ΠΎΠ±ΡΠ°Π·ΠΎΠΌ, Π·Π°ΠΊΠΎΠ½ Π€Π°ΡΠ°Π΄Π΅Ρ ΡΠΏΡΠ°Π²Π΅Π΄Π»ΠΈΠ² Π΄Π»Ρ Π²ΡΠ΅Ρ ΠΈΠ·ΠΌΠ΅Π½Π΅Π½ΠΈΠΉ ΠΏΠΎΡΠΎΠΊΠ° , Π½Π΅Π·Π°Π²ΠΈΡΠΈΠΌΠΎ ΠΎΡ ΡΠΎΠ³ΠΎ, Π²ΡΠ·Π²Π°Π½Ρ Π»ΠΈ ΠΎΠ½ΠΈ ΠΈΠ·ΠΌΠ΅Π½ΡΡΡΠΈΠΌΡΡ ΠΌΠ°Π³Π½ΠΈΡΠ½ΡΠΌ ΠΏΠΎΠ»Π΅ΠΌ, Π΄Π²ΠΈΠΆΠ΅Π½ΠΈΠ΅ΠΌ ΠΈΠ»ΠΈ ΠΈΡ ΠΊΠΎΠΌΠ±ΠΈΠ½Π°ΡΠΈΠ΅ΠΉ. 92}{R}.\]
ΠΡΠΈ ΡΠΎΠ±Π»ΡΠ΄Π΅Π½ΠΈΠΈ ΠΏΡΠΈΠ½ΡΠΈΠΏΠ° ΡΠΎΡ ΡΠ°Π½Π΅Π½ΠΈΡ ΡΠ½Π΅ΡΠ³ΠΈΠΈ ΠΏΡΠΎΠΈΠ·Π²ΠΎΠ΄ΠΈΠΌΠ°Ρ ΠΈ ΡΠ°ΡΡΠ΅ΠΈΠ²Π°Π΅ΠΌΠ°Ρ ΠΌΠΎΡΠ½ΠΎΡΡΠΈ ΡΠ°Π²Π½Ρ.
ΠΡΠΎΡ ΠΏΡΠΈΠ½ΡΠΈΠΏ ΠΌΠΎΠΆΠ½ΠΎ ΡΠ²ΠΈΠ΄Π΅ΡΡ Π² ΡΠ°Π±ΠΎΡΠ΅ ΡΠ΅Π»ΡΡΠΎΠ²ΠΎΠΉ ΠΏΡΡΠΊΠΈ. Π Π΅Π»ΡΡΠΎΠ²Π°Ρ ΠΏΡΡΠΊΠ° β ΡΡΠΎ ΡΠ»Π΅ΠΊΡΡΠΎΠΌΠ°Π³Π½ΠΈΡΠ½Π°Ρ ΠΏΡΡΠΊΠΎΠ²Π°Ρ ΡΡΡΠ°Π½ΠΎΠ²ΠΊΠ°, Π² ΠΊΠΎΡΠΎΡΠΎΠΉ ΠΈΡΠΏΠΎΠ»ΡΠ·ΡΠ΅ΡΡΡ ΡΡΡΡΠΎΠΉΡΡΠ²ΠΎ, ΠΏΠΎΠ΄ΠΎΠ±Π½ΠΎΠ΅ ΠΏΠΎΠΊΠ°Π·Π°Π½Π½ΠΎΠΌΡ Π½Π° ΡΠΈΡΡΠ½ΠΊΠ΅ \(\PageIndex{2}\), ΡΡ Π΅ΠΌΠ°ΡΠΈΡΠ½ΠΎ ΠΏΠΎΠΊΠ°Π·Π°Π½Π½ΠΎΠ΅ Π½Π° ΡΠΈΡΡΠ½ΠΊΠ΅ \(\PageIndex{3}\). ΠΡΠΎΠ²ΠΎΠ΄ΡΡΠΈΠΉ ΡΡΠ΅ΡΠΆΠ΅Π½Ρ Π·Π°ΠΌΠ΅Π½ΡΠ΅ΡΡΡ ΡΠ½Π°ΡΡΠ΄ΠΎΠΌ ΠΈΠ»ΠΈ ΠΎΡΡΠΆΠΈΠ΅ΠΌ Π΄Π»Ρ ΡΡΡΠ΅Π»ΡΠ±Ρ. ΠΠΎ ΡΠΈΡ ΠΏΠΎΡ ΠΌΡ ΡΠ»ΡΡΠ°Π»ΠΈ ΡΠΎΠ»ΡΠΊΠΎ ΠΎ ΡΠΎΠΌ, ΠΊΠ°ΠΊ Π΄Π²ΠΈΠΆΠ΅Π½ΠΈΠ΅ Π²ΡΠ·ΡΠ²Π°Π΅Ρ ΠΠΠ‘. Π ΡΠ΅Π»ΡΡΠΎΠ²ΠΎΠΉ ΠΏΡΡΠΊΠ΅ ΠΎΠΏΡΠΈΠΌΠ°Π»ΡΠ½ΠΎΠ΅ ΠΎΡΠΊΠ»ΡΡΠ΅Π½ΠΈΠ΅/ΡΠΌΠ΅Π½ΡΡΠ΅Π½ΠΈΠ΅ ΠΌΠ°Π³Π½ΠΈΡΠ½ΠΎΠ³ΠΎ ΠΏΠΎΠ»Ρ ΡΠΌΠ΅Π½ΡΡΠ°Π΅Ρ ΠΏΠΎΡΠΎΠΊ ΠΌΠ΅ΠΆΠ΄Ρ ΡΠ΅Π»ΡΡΠ°ΠΌΠΈ, Π²ΡΠ·ΡΠ²Π°Ρ ΠΏΡΠΎΡΠ΅ΠΊΠ°Π½ΠΈΠ΅ ΡΠΎΠΊΠ° Π² ΡΡΠ΅ΡΠΆΠ½Π΅ (ΡΠΊΠΎΡΠ΅), ΠΊΠΎΡΠΎΡΡΠΉ ΡΠ΄Π΅ΡΠΆΠΈΠ²Π°Π΅Ρ ΡΠ½Π°ΡΡΠ΄. ΠΡΠΎΡ ΡΠΎΠΊ ΡΠ΅ΡΠ΅Π· ΡΠΊΠΎΡΡ ΠΈΡΠΏΡΡΡΠ²Π°Π΅Ρ ΠΌΠ°Π³Π½ΠΈΡΠ½ΡΡ ΡΠΈΠ»Ρ ΠΈ Π΄Π²ΠΈΠΆΠ΅ΡΡΡ Π²ΠΏΠ΅ΡΠ΅Π΄. Π Π΅Π»ΡΡΠΎΠ²ΡΠ΅ ΠΏΡΡΠΊΠΈ, ΠΎΠ΄Π½Π°ΠΊΠΎ, Π½Π΅ ΠΈΡΠΏΠΎΠ»ΡΠ·ΡΡΡΡΡ ΡΠΈΡΠΎΠΊΠΎ Π² Π²ΠΎΠΎΡΡΠΆΠ΅Π½Π½ΡΡ ΡΠΈΠ»Π°Ρ ΠΈΠ·-Π·Π° Π²ΡΡΠΎΠΊΠΎΠΉ ΡΡΠΎΠΈΠΌΠΎΡΡΠΈ ΠΏΡΠΎΠΈΠ·Π²ΠΎΠ΄ΡΡΠ²Π° ΠΈ Π±ΠΎΠ»ΡΡΠΈΡ ΡΠΎΠΊΠΎΠ²: ΡΡΠ΅Π±ΡΠ΅ΡΡΡ ΠΏΠΎΡΡΠΈ ΠΎΠ΄ΠΈΠ½ ΠΌΠΈΠ»Π»ΠΈΠΎΠ½ Π°ΠΌΠΏΠ΅Ρ Π΄Π»Ρ ΠΏΡΠΎΠΈΠ·Π²ΠΎΠ΄ΡΡΠ²Π° ΡΠ½Π΅ΡΠ³ΠΈΠΈ, Π΄ΠΎΡΡΠ°ΡΠΎΡΠ½ΠΎΠΉ Π΄Π»Ρ ΡΠΎΠ³ΠΎ, ΡΡΠΎΠ±Ρ ΡΠ΅Π»ΡΡΠΎΠ²Π°Ρ ΠΏΡΡΠΊΠ° Π±ΡΠ»Π° ΡΡΡΠ΅ΠΊΡΠΈΠ²Π½ΡΠΌ ΠΎΡΡΠΆΠΈΠ΅ΠΌ.
Π ΠΈΡΡΠ½ΠΎΠΊ \(\PageIndex{3}\): Π’ΠΎΠΊ ΠΏΠΎ Π΄Π²ΡΠΌ ΡΠ΅Π»ΡΡΠ°ΠΌ ΡΠΎΠ»ΠΊΠ°Π΅Ρ ΡΠΎΠΊΠΎΠΏΡΠΎΠ²ΠΎΠ΄ΡΡΠΈΠΉ ΡΠ½Π°ΡΡΠ΄ Π²ΠΏΠ΅ΡΠ΅Π΄ Π·Π° ΡΡΠ΅Ρ ΡΠΎΠ·Π΄Π°Π²Π°Π΅ΠΌΠΎΠΉ ΠΌΠ°Π³Π½ΠΈΡΠ½ΠΎΠΉ ΡΠΈΠ»Ρ.ΠΡ ΠΌΠΎΠΆΠ΅ΠΌ ΡΠ°ΡΡΡΠΈΡΠ°ΡΡ ΠΠΠ‘, Π²ΡΠ·Π²Π°Π½Π½ΡΡ Π΄Π²ΠΈΠΆΠ΅Π½ΠΈΠ΅ΠΌ, Ρ ΠΏΠΎΠΌΠΎΡΡΡ Π·Π°ΠΊΠΎΠ½Π° Π€Π°ΡΠ°Π΄Π΅Ρ , Π΄Π°ΠΆΠ΅ Π΅ΡΠ»ΠΈ ΡΠ°ΠΊΡΠΈΡΠ΅ΡΠΊΠ°Ρ Π·Π°ΠΌΠΊΠ½ΡΡΠ°Ρ ΡΠ΅ΠΏΡ ΠΎΡΡΡΡΡΡΠ²ΡΠ΅Ρ . ΠΡ ΠΏΡΠΎΡΡΠΎ ΠΏΡΠ΅Π΄ΡΡΠ°Π²Π»ΡΠ΅ΠΌ Π·Π°ΠΌΠΊΠ½ΡΡΡΡ ΠΎΠ±Π»Π°ΡΡΡ, Π³ΡΠ°Π½ΠΈΡΠ° ΠΊΠΎΡΠΎΡΠΎΠΉ Π²ΠΊΠ»ΡΡΠ°Π΅Ρ Π² ΡΠ΅Π±Ρ Π΄Π²ΠΈΠΆΡΡΠΈΠΉΡΡ ΠΏΡΠΎΠ²ΠΎΠ΄Π½ΠΈΠΊ, Π²ΡΡΠΈΡΠ»ΡΠ΅ΠΌ \(\Phi_m\), Π° Π·Π°ΡΠ΅ΠΌ Π½Π°Ρ ΠΎΠ΄ΠΈΠΌ ΠΠΠ‘ ΠΏΠΎ Π·Π°ΠΊΠΎΠ½Ρ Π€Π°ΡΠ°Π΄Π΅Ρ. ΠΠ°ΠΏΡΠΈΠΌΠ΅Ρ, ΠΌΡ ΠΌΠΎΠΆΠ΅ΠΌ ΠΏΠΎΠ·Π²ΠΎΠ»ΠΈΡΡ Π΄Π²ΠΈΠΆΡΡΠ΅ΠΌΡΡΡ ΡΡΠ΅ΡΠΆΠ½Ρ Π½Π° ΡΠΈΡΡΠ½ΠΊΠ΅ \(\PageIndex{4}\) Π±ΡΡΡ ΠΎΠ΄Π½ΠΎΠΉ ΡΡΠΎΡΠΎΠ½ΠΎΠΉ Π²ΠΎΠΎΠ±ΡΠ°ΠΆΠ°Π΅ΠΌΠΎΠΉ ΠΏΡΡΠΌΠΎΡΠ³ΠΎΠ»ΡΠ½ΠΎΠΉ ΠΎΠ±Π»Π°ΡΡΠΈ, ΠΏΡΠ΅Π΄ΡΡΠ°Π²Π»Π΅Π½Π½ΠΎΠΉ ΠΏΡΠ½ΠΊΡΠΈΡΠ½ΡΠΌΠΈ Π»ΠΈΠ½ΠΈΡΠΌΠΈ. ΠΠ»ΠΎΡΠ°Π΄Ρ ΠΏΡΡΠΌΠΎΡΠ³ΠΎΠ»ΡΠ½ΠΈΠΊΠ° Π»ΠΊ , ΠΏΠΎΡΡΠΎΠΌΡ ΠΌΠ°Π³Π½ΠΈΡΠ½ΡΠΉ ΠΏΠΎΡΠΎΠΊ ΡΠ΅ΡΠ΅Π· Π½Π΅Π³ΠΎ ΡΠ°Π²Π΅Π½ \(\Phi_m = Blx\). ΠΠΈΡΡΠ΅ΡΠ΅Π½ΡΠΈΡΡΡ ΡΡΠΎ ΡΡΠ°Π²Π½Π΅Π½ΠΈΠ΅, ΠΏΠΎΠ»ΡΡΠ°Π΅ΠΌ
\[\frac{d\Phi_m}{dt} = Bl\frac{dx}{dt} = Blv,\]
, ΡΡΠΎ ΠΈΠ΄Π΅Π½ΡΠΈΡΠ½ΠΎ ΡΠ°Π·Π½ΠΎΡΡΠΈ ΠΏΠΎΡΠ΅Π½ΡΠΈΠ°Π»ΠΎΠ² ΠΌΠ΅ΠΆΠ΄Ρ ΠΊΠΎΠ½ΡΠ°ΠΌΠΈ ΡΡΠ΅ΡΠΆΠ½Ρ, ΠΊΠΎΡΠΎΡΡΡ ΠΌΡ ΠΎΠΏΡΠ΅Π΄Π΅Π»ΠΈΠ»ΠΈ ΡΠ°Π½Π΅Π΅.
Π ΠΈΡΡΠ½ΠΎΠΊ \(\PageIndex{4}\): Π‘ ΠΏΠΎΠΊΠ°Π·Π°Π½Π½ΡΠΌ Π²ΠΎΠΎΠ±ΡΠ°ΠΆΠ°Π΅ΠΌΡΠΌ ΠΏΡΡΠΌΠΎΡΠ³ΠΎΠ»ΡΠ½ΠΈΠΊΠΎΠΌ ΠΌΡ ΠΌΠΎΠΆΠ΅ΠΌ ΠΈΡΠΏΠΎΠ»ΡΠ·ΠΎΠ²Π°ΡΡ Π·Π°ΠΊΠΎΠ½ Π€Π°ΡΠ°Π΄Π΅Ρ Π΄Π»Ρ ΡΠ°ΡΡΠ΅ΡΠ° ΠΠΠ‘ ΠΈΠ½Π΄ΡΠΊΡΠΈΠΈ Π² Π΄Π²ΠΈΠΆΡΡΠ΅ΠΌΡΡ ΡΡΠ΅ΡΠΆΠ½Π΅.ΠΠΠ‘ Π΄Π²ΠΈΠΆΠ΅Π½ΠΈΡ Π² ΡΠ»Π°Π±ΠΎΠΌ ΠΌΠ°Π³Π½ΠΈΡΠ½ΠΎΠΌ ΠΏΠΎΠ»Π΅ ΠΠ΅ΠΌΠ»ΠΈ ΠΎΠ±ΡΡΠ½ΠΎ Π½Π΅ ΠΎΡΠ΅Π½Ρ Π²Π΅Π»ΠΈΠΊΠΈ, ΠΈΠ½Π°ΡΠ΅ ΠΌΡ Π·Π°ΠΌΠ΅ΡΠΈΠ»ΠΈ Π±Ρ Π½Π°ΠΏΡΡΠΆΠ΅Π½ΠΈΠ΅ Π²Π΄ΠΎΠ»Ρ ΠΌΠ΅ΡΠ°Π»Π»ΠΈΡΠ΅ΡΠΊΠΈΡ ΡΡΠ΅ΡΠΆΠ½Π΅ΠΉ, ΡΠ°ΠΊΠΈΡ ΠΊΠ°ΠΊ ΠΎΡΠ²Π΅ΡΡΠΊΠ°, ΠΏΡΠΈ ΠΎΠ±ΡΡΠ½ΡΡ Π΄Π²ΠΈΠΆΠ΅Π½ΠΈΡΡ . ΠΠ°ΠΏΡΠΈΠΌΠ΅Ρ, ΠΏΡΠΎΡΡΠΎΠΉ ΡΠ°ΡΡΠ΅Ρ ΠΠΠ‘ Π΄Π²ΠΈΠΆΠ΅Π½ΠΈΡ ΡΡΠ΅ΡΠΆΠ½Ρ Π΄Π»ΠΈΠ½ΠΎΠΉ 1,0 ΠΌ, Π΄Π²ΠΈΠΆΡΡΠ΅Π³ΠΎΡΡ ΡΠΎ ΡΠΊΠΎΡΠΎΡΡΡΡ 3,0 ΠΌ/Ρ ΠΏΠ΅ΡΠΏΠ΅Π½Π΄ΠΈΠΊΡΠ»ΡΡΠ½ΠΎ ΠΏΠΎΠ»Ρ ΠΠ΅ΠΌΠ»ΠΈ, Π΄Π°Π΅Ρ 9{-5} Π’)(1,0 Π», ΠΌ)(3,0 Π», ΠΌ/Ρ) = 150 Π», ΠΌΠΊΠ.\]
ΠΡΠΎ Π½Π΅Π±ΠΎΠ»ΡΡΠΎΠ΅ Π·Π½Π°ΡΠ΅Π½ΠΈΠ΅ ΡΠΎΠΎΡΠ²Π΅ΡΡΡΠ²ΡΠ΅Ρ ΠΎΠΏΡΡΡ. ΠΠ΄Π½Π°ΠΊΠΎ Π΅ΡΡΡ ΡΡΡΠ΅ΠΊΡΠ½ΠΎΠ΅ ΠΈΡΠΊΠ»ΡΡΠ΅Π½ΠΈΠ΅. Π 1992 ΠΈ 1996 Π³ΠΎΠ΄Π°Ρ Ρ ΠΏΠΎΠΌΠΎΡΡΡ ΠΊΠΎΡΠΌΠΈΡΠ΅ΡΠΊΠΎΠ³ΠΎ ΠΊΠΎΡΠ°Π±Π»Ρ «Π¨Π°ΡΡΠ»» Π±ΡΠ»ΠΈ ΠΏΡΠ΅Π΄ΠΏΡΠΈΠ½ΡΡΡ ΠΏΠΎΠΏΡΡΠΊΠΈ ΡΠΎΠ·Π΄Π°ΡΡ Π±ΠΎΠ»ΡΡΠΈΠ΅ ΠΠΠ‘ Π΄Π²ΠΈΠΆΠ΅Π½ΠΈΡ. ΠΡΠΈΠ²ΡΠ·Π°Π½Π½ΡΠΉ ΡΠΏΡΡΠ½ΠΈΠΊ Π΄ΠΎΠ»ΠΆΠ΅Π½ Π±ΡΠ» Π±ΡΡΡ Π²ΡΠΏΡΡΠ΅Π½ ΠΏΠΎ ΠΏΡΠΎΠ²ΠΎΠ΄Ρ Π΄Π»ΠΈΠ½ΠΎΠΉ 20 ΠΊΠΌ, ΠΊΠ°ΠΊ ΠΏΠΎΠΊΠ°Π·Π°Π½ΠΎ Π½Π° ΡΠΈΡΡΠ½ΠΊΠ΅ \(\PageIndex{5}\), ΡΡΠΎΠ±Ρ ΡΠΎΠ·Π΄Π°ΡΡ ΠΠΠ‘ 5 ΠΊΠ, Π΄Π²ΠΈΠ³Π°ΡΡΡ Ρ ΠΎΡΠ±ΠΈΡΠ°Π»ΡΠ½ΠΎΠΉ ΡΠΊΠΎΡΠΎΡΡΡΡ ΡΠ΅ΡΠ΅Π· ΠΏΠΎΠ»Π΅ ΠΠ΅ΠΌΠ»ΠΈ. ΠΡΡ ΠΠΠ‘ ΠΌΠΎΠΆΠ½ΠΎ Π±ΡΠ»ΠΎ Π±Ρ ΠΈΡΠΏΠΎΠ»ΡΠ·ΠΎΠ²Π°ΡΡ Π΄Π»Ρ ΠΏΡΠ΅ΠΎΠ±ΡΠ°Π·ΠΎΠ²Π°Π½ΠΈΡ ΡΠ°ΡΡΠΈ ΠΊΠΈΠ½Π΅ΡΠΈΡΠ΅ΡΠΊΠΎΠΉ ΠΈ ΠΏΠΎΡΠ΅Π½ΡΠΈΠ°Π»ΡΠ½ΠΎΠΉ ΡΠ½Π΅ΡΠ³ΠΈΠΈ ΡΠ°ΡΡΠ»Π° Π² ΡΠ»Π΅ΠΊΡΡΠΈΡΠ΅ΡΠΊΡΡ ΡΠ½Π΅ΡΠ³ΠΈΡ, Π΅ΡΠ»ΠΈ Π±Ρ ΡΠ΄Π°Π»ΠΎΡΡ ΡΠΎΠ·Π΄Π°ΡΡ ΠΏΠΎΠ»Π½ΡΡ ΡΠ΅ΠΏΡ. Π§ΡΠΎΠ±Ρ Π·Π°Π²Π΅ΡΡΠΈΡΡ ΡΠ΅ΠΏΡ, Π½Π΅ΠΏΠΎΠ΄Π²ΠΈΠΆΠ½Π°Ρ ΠΈΠΎΠ½ΠΎΡΡΠ΅ΡΠ° Π΄ΠΎΠ»ΠΆΠ½Π° Π±ΡΠ»Π° ΠΎΠ±Π΅ΡΠΏΠ΅ΡΠΈΡΡ ΠΎΠ±ΡΠ°ΡΠ½ΡΠΉ ΠΏΡΡΡ, ΠΏΠΎ ΠΊΠΎΡΠΎΡΠΎΠΌΡ ΠΌΠΎΠ³ ΡΠ΅ΡΡ ΡΠΎΠΊ. (ΠΠΎΠ½ΠΎΡΡΠ΅ΡΠ° β ΡΡΠΎ ΡΠ°Π·ΡΠ΅ΠΆΠ΅Π½Π½Π°Ρ ΠΈ ΡΠ°ΡΡΠΈΡΠ½ΠΎ ΠΈΠΎΠ½ΠΈΠ·ΠΈΡΠΎΠ²Π°Π½Π½Π°Ρ Π°ΡΠΌΠΎΡΡΠ΅ΡΠ° Π½Π° ΠΎΡΠ±ΠΈΡΠ°Π»ΡΠ½ΡΡ Π²ΡΡΠΎΡΠ°Ρ . ΠΠ½Π° ΠΏΡΠΎΠ²ΠΎΠ΄ΠΈΡ ΠΈΠ·-Π·Π° ΠΈΠΎΠ½ΠΈΠ·Π°ΡΠΈΠΈ. ΠΠΎΠ½ΠΎΡΡΠ΅ΡΠ° Π²ΡΠΏΠΎΠ»Π½ΡΠ΅Ρ ΡΡ ΠΆΠ΅ ΡΡΠ½ΠΊΡΠΈΡ, ΡΡΠΎ ΠΈ ΡΡΠ°ΡΠΈΠΎΠ½Π°ΡΠ½ΡΠ΅ ΡΠ΅Π»ΡΡΡ ΠΈ ΡΠΎΠ΅Π΄ΠΈΠ½ΠΈΡΠ΅Π»ΡΠ½ΡΠΉ ΡΠ΅Π·ΠΈΡΡΠΎΡ Π½Π° ΡΠΈΡΡΠ½ΠΊΠ΅ \(\PageIndex{3}\), Π±Π΅Π· ΠΊΠΎΡΠΎΡΡΡ Π½Π΅ Π±ΡΠ»ΠΎ Π±Ρ Π½Π΅ ΠΌΠΎΠΆΠ΅Ρ Π±ΡΡΡ ΠΏΠΎΠ»Π½ΠΎΠΉ ΡΠ΅ΠΏΡΡ.) Π’ΠΎΠΊ Π² ΠΊΠ°Π±Π΅Π»Π΅ Π·Π° ΡΡΠ΅Ρ ΠΌΠ°Π³Π½ΠΈΡΠ½ΠΎΠΉ ΡΠΈΠ»Ρ \(F = IlBsin \, \theta\) Π²ΡΠΏΠΎΠ»Π½ΡΠ΅Ρ ΡΠ°Π±ΠΎΡΡ, ΠΊΠΎΡΠΎΡΠ°Ρ ΡΠΌΠ΅Π½ΡΡΠ°Π΅Ρ ΠΊΠΈΠ½Π΅ΡΠΈΡΠ΅ΡΠΊΡΡ ΠΈ ΠΏΠΎΡΠ΅Π½ΡΠΈΠ°Π»ΡΠ½ΡΡ ΡΠ½Π΅ΡΠ³ΠΈΡ ΡΠ΅Π»Π½ΠΎΠΊΠ° ΠΈ ΠΏΠΎΠ·Π²ΠΎΠ»ΡΠ΅Ρ ΠΏΡΠ΅ΠΎΠ±ΡΠ°Π·ΠΎΠ²Π°ΡΡ Π΅Π΅ Π² ΡΠ»Π΅ΠΊΡΡΠΈΡΠ΅ΡΠΊΡΡ ΡΠ½Π΅ΡΠ³ΠΈΡ. ΠΠ±Π° ΡΠ΅ΡΡΠ° ΠΎΠΊΠ°Π·Π°Π»ΠΈΡΡ Π½Π΅ΡΠ΄Π°ΡΠ½ΡΠΌΠΈ. Π ΠΏΠ΅ΡΠ²ΠΎΠΌ ΠΊΠ°Π±Π΅Π»Ρ Π±ΠΎΠ»ΡΠ°Π»ΡΡ ΠΈ Π΅Π³ΠΎ ΠΌΠΎΠΆΠ½ΠΎ Π±ΡΠ»ΠΎ ΠΏΡΠΎΡΡΠ½ΡΡΡ ΡΠΎΠ»ΡΠΊΠΎ Π½Π° ΠΏΠ°ΡΡ ΡΠΎΡΠ΅Π½ ΠΌΠ΅ΡΡΠΎΠ²; Π²ΠΎ Π²ΡΠΎΡΠΎΠΌ ΡΡΠΎΡ ΠΎΠ±ΠΎΡΠ²Π°Π»ΡΡ ΠΏΡΠΈ ΠΏΠΎΡΡΠΈ ΠΏΠΎΠ»Π½ΠΎΠΌ Π²ΡΡΡΠ³ΠΈΠ²Π°Π½ΠΈΠΈ. ΠΡΠΈΠΌΠ΅Ρ \(\PageIndex{1}\) ΡΠΊΠ°Π·ΡΠ²Π°Π΅Ρ Π½Π° ΠΏΡΠΈΠ½ΡΠΈΠΏΠΈΠ°Π»ΡΠ½ΡΡ Π²ΠΎΠ·ΠΌΠΎΠΆΠ½ΠΎΡΡΡ.
Π ΠΈΡΡΠ½ΠΎΠΊ \(\PageIndex{5}\): ΠΠΠ‘ Π΄Π²ΠΈΠΆΠ΅Π½ΠΈΡ ΠΊΠ°ΠΊ ΠΏΡΠ΅ΠΎΠ±ΡΠ°Π·ΠΎΠ²Π°Π½ΠΈΠ΅ ΡΠ»Π΅ΠΊΡΡΠΎΡΠ½Π΅ΡΠ³ΠΈΠΈ Π΄Π»Ρ ΠΊΠΎΡΠΌΠΈΡΠ΅ΡΠΊΠΎΠ³ΠΎ ΡΠ΅Π»Π½ΠΎΠΊΠ° ΠΏΠΎΡΠ»ΡΠΆΠΈΠ»Π° ΠΌΠΎΡΠΈΠ²ΠΎΠΌ Π΄Π»Ρ ΡΠΊΡΠΏΠ΅ΡΠΈΠΌΠ΅Π½ΡΠ° Ρ ΠΏΡΠΈΠ²ΡΠ·Π°Π½Π½ΡΠΌ ΡΠΏΡΡΠ½ΠΈΠΊΠΎΠΌ. ΠΡΠ»ΠΎ ΠΏΡΠ΅Π΄ΡΠΊΠ°Π·Π°Π½ΠΎ, ΡΡΠΎ ΠΠΠ‘ Π½Π°ΠΏΡΡΠΆΠ΅Π½ΠΈΠ΅ΠΌ 5 ΠΊΠ Π±ΡΠ΄Π΅Ρ ΠΈΠ½Π΄ΡΡΠΈΡΠΎΠ²Π°ΡΡΡΡ Π² 20-ΠΊΠΈΠ»ΠΎΠΌΠ΅ΡΡΠΎΠ²ΠΎΠΌ ΡΡΠΎΡΠ΅ ΠΏΡΠΈ Π΄Π²ΠΈΠΆΠ΅Π½ΠΈΠΈ Ρ ΠΎΡΠ±ΠΈΡΠ°Π»ΡΠ½ΠΎΠΉ ΡΠΊΠΎΡΠΎΡΡΡΡ Π² ΠΌΠ°Π³Π½ΠΈΡΠ½ΠΎΠΌ ΠΏΠΎΠ»Π΅ ΠΠ΅ΠΌΠ»ΠΈ. {-5} Π’Π»\) ΠΌΠ°Π³Π½ΠΈΡΠ½ΠΎΠ³ΠΎ ΠΏΠΎΠ»Ρ. 9ΠΎ\) ΠΈ, ΡΠ»Π΅Π΄ΠΎΠ²Π°ΡΠ΅Π»ΡΠ½ΠΎ, \(sin\, \theta = 1\).ΠΡΠΈΠΌΠ΅Ρ \(\PageIndex{2}\): ΠΌΠ΅ΡΠ°Π»Π»ΠΈΡΠ΅ΡΠΊΠΈΠΉ ΡΡΠ΅ΡΠΆΠ΅Π½Ρ, Π²ΡΠ°ΡΠ°ΡΡΠΈΠΉΡΡ Π² ΠΌΠ°Π³Π½ΠΈΡΠ½ΠΎΠΌ ΠΏΠΎΠ»Π΅
Π§Π°ΡΡΡ (a) ΡΠΈΡΡΠ½ΠΊΠ° \(\PageIndex{6}\) ΠΏΠΎΠΊΠ°Π·ΡΠ²Π°Π΅Ρ ΠΌΠ΅ΡΠ°Π»Π»ΠΈΡΠ΅ΡΠΊΠΈΠΉ ΡΡΠ΅ΡΠΆΠ΅Π½Ρ OS , Π³ΠΎΡΠΈΠ·ΠΎΠ½ΡΠ°Π»ΡΠ½Π°Ρ ΠΏΠ»ΠΎΡΠΊΠΎΡΡΡ Π²ΠΎΠΊΡΡΠ³ ΡΠΎΡΠΊΠΈ O . Π‘ΡΠ΅ΡΠΆΠ΅Π½Ρ ΡΠΊΠΎΠ»ΡΠ·ΠΈΡ ΠΏΠΎ ΠΏΡΠΎΠ²ΠΎΠ»ΠΎΠΊΠ΅, ΠΎΠ±ΡΠ°Π·ΡΡΡΠ΅ΠΉ Π΄ΡΠ³Ρ ΠΎΠΊΡΡΠΆΠ½ΠΎΡΡΠΈ PST ΡΠ°Π΄ΠΈΡΡΠΎΠΌ r . Π‘ΠΈΡΡΠ΅ΠΌΠ° Π½Π°Ρ ΠΎΠ΄ΠΈΡΡΡ Π² ΠΏΠΎΡΡΠΎΡΠ½Π½ΠΎΠΌ ΠΌΠ°Π³Π½ΠΈΡΠ½ΠΎΠΌ ΠΏΠΎΠ»Π΅ \(\vec{B}\), ΠΊΠΎΡΠΎΡΠΎΠ΅ Π½Π°ΠΏΡΠ°Π²Π»Π΅Π½ΠΎ Π·Π° ΠΏΡΠ΅Π΄Π΅Π»Ρ ΡΡΡΠ°Π½ΠΈΡΡ. (a) ΠΡΠ»ΠΈ Π²Ρ Π²ΡΠ°ΡΠ°Π΅ΡΠ΅ ΡΡΠ΅ΡΠΆΠ΅Π½Ρ Ρ ΠΏΠΎΡΡΠΎΡΠ½Π½ΠΎΠΉ ΡΠ³Π»ΠΎΠ²ΠΎΠΉ ΡΠΊΠΎΡΠΎΡΡΡΡ \(\omega\), ΠΊΠ°ΠΊΠΎΠ²Π° ΡΠΈΠ»Π° ΡΠΎΠΊΠ° I Π² Π·Π°ΠΌΠΊΠ½ΡΡΠΎΠΌ ΠΊΠΎΠ½ΡΡΡΠ΅ ΠΠΠ‘Π ? ΠΡΠ΅Π΄ΠΏΠΎΠ»ΠΎΠΆΠΈΠΌ, ΡΡΠΎ ΡΠ΅Π·ΠΈΡΡΠΎΡ R ΠΎΠ±Π΅ΡΠΏΠ΅ΡΠΈΠ²Π°Π΅Ρ Π²ΡΠ΅ ΡΠΎΠΏΡΠΎΡΠΈΠ²Π»Π΅Π½ΠΈΠ΅ Π² Π·Π°ΠΌΠΊΠ½ΡΡΠΎΠΌ ΠΊΠΎΠ½ΡΡΡΠ΅. Π±) ΠΡΡΠΈΡΠ»ΠΈΡΠ΅ ΡΠ°Π±ΠΎΡΡ, ΡΠΎΠ²Π΅ΡΡΠ°Π΅ΠΌΡΡ Π²Π°ΠΌΠΈ ΠΏΡΠΈ Π²ΡΠ°ΡΠ΅Π½ΠΈΠΈ ΡΡΠ΅ΡΠΆΠ½Ρ Π² Π΅Π΄ΠΈΠ½ΠΈΡΡ Π²ΡΠ΅ΠΌΠ΅Π½ΠΈ, ΠΈ ΠΏΠΎΠΊΠ°ΠΆΠΈΡΠ΅, ΡΡΠΎ ΠΎΠ½Π° ΡΠ°Π²Π½Π° ΠΌΠΎΡΠ½ΠΎΡΡΠΈ, ΡΠ°ΡΡΠ΅ΠΈΠ²Π°Π΅ΠΌΠΎΠΉ Π½Π° ΡΠ΅Π·ΠΈΡΡΠΎΡΠ΅.
Π ΠΈΡΡΠ½ΠΎΠΊ \(\PageIndex{6}\): (a) ΠΠΎΠ½Π΅Ρ Π²ΡΠ°ΡΠ°ΡΡΠ΅Π³ΠΎΡΡ ΠΌΠ΅ΡΠ°Π»Π»ΠΈΡΠ΅ΡΠΊΠΎΠ³ΠΎ ΡΡΠ΅ΡΠΆΠ½Ρ ΡΠΊΠΎΠ»ΡΠ·ΠΈΡ ΠΏΠΎ ΠΊΡΡΠ³Π»ΠΎΠΉ ΠΏΡΠΎΠ²ΠΎΠ»ΠΎΠΊΠ΅ Π² Π³ΠΎΡΠΈΠ·ΠΎΠ½ΡΠ°Π»ΡΠ½ΠΎΠΉ ΠΏΠ»ΠΎΡΠΊΠΎΡΡΠΈ. (Π±) ΠΠ½Π΄ΡΡΠΈΡΠΎΠ²Π°Π½Π½ΡΠΉ ΡΠΎΠΊ Π² ΡΡΠ΅ΡΠΆΠ½Π΅. (c) ΠΠ°Π³Π½ΠΈΡΠ½Π°Ρ ΡΠΈΠ»Π° Π½Π° Π±Π΅ΡΠΊΠΎΠ½Π΅ΡΠ½ΠΎ ΠΌΠ°Π»ΠΎΠΌ ΠΎΡΡΠ΅Π·ΠΊΠ΅ ΡΠΎΠΊΠ°. 92\omega}{2R}.\] ΠΠΎ ΠΌΠ΅ΡΠ΅ ΡΠ²Π΅Π»ΠΈΡΠ΅Π½ΠΈΡ \(\theta\) ΡΠ²Π΅Π»ΠΈΡΠΈΠ²Π°Π΅ΡΡΡ ΠΈ ΠΏΠΎΡΠΎΠΊ ΡΠ΅ΡΠ΅Π· ΠΏΠ΅ΡΠ»Ρ ΠΈΠ·-Π·Π° \(\vec{B}\). Π§ΡΠΎΠ±Ρ ΠΏΡΠΎΡΠΈΠ²ΠΎΠ΄Π΅ΠΉΡΡΠ²ΠΎΠ²Π°ΡΡ ΡΡΠΎΠΌΡ ΡΠ²Π΅Π»ΠΈΡΠ΅Π½ΠΈΡ, ΠΌΠ°Π³Π½ΠΈΡΠ½ΠΎΠ΅ ΠΏΠΎΠ»Π΅ ΠΈΠ·-Π·Π° ΠΈΠ½Π΄ΡΡΠΈΡΠΎΠ²Π°Π½Π½ΠΎΠ³ΠΎ ΡΠΎΠΊΠ° Π΄ΠΎΠ»ΠΆΠ½ΠΎ Π±ΡΡΡ Π½Π°ΠΏΡΠ°Π²Π»Π΅Π½ΠΎ Π² ΡΡΡΠ°Π½ΠΈΡΡ Π² ΠΎΠ±Π»Π°ΡΡΠΈ, ΠΎΠ³ΡΠ°Π½ΠΈΡΠ΅Π½Π½ΠΎΠΉ ΠΏΠ΅ΡΠ»Π΅ΠΉ. Π‘Π»Π΅Π΄ΠΎΠ²Π°ΡΠ΅Π»ΡΠ½ΠΎ, ΠΊΠ°ΠΊ ΠΏΠΎΠΊΠ°Π·Π°Π½ΠΎ Π² ΡΠ°ΡΡΠΈ (b) ΡΠΈΡΡΠ½ΠΊΠ° \(\PageIndex{6}\), ΡΠΎΠΊ ΡΠΈΡΠΊΡΠ»ΠΈΡΡΠ΅Ρ ΠΏΠΎ ΡΠ°ΡΠΎΠ²ΠΎΠΉ ΡΡΡΠ΅Π»ΠΊΠ΅.ΠΠ½Π°ΡΠ΅Π½ΠΈΠ΅
ΠΠ»ΡΡΠ΅ΡΠ½Π°ΡΠΈΠ²Π½ΡΠΉ ΡΠΏΠΎΡΠΎΠ± ΡΠ°ΡΡΠΌΠΎΡΡΠ΅Π½ΠΈΡ ΠΠΠ‘ ΠΈΠ½Π΄ΡΠΊΡΠΈΠΈ ΠΈΠ· Π·Π°ΠΊΠΎΠ½Π° Π€Π°ΡΠ°Π΄Π΅Ρ ΡΠΎΡΡΠΎΠΈΡ Π² ΠΈΠ½ΡΠ΅Π³ΡΠΈΡΠΎΠ²Π°Π½ΠΈΠΈ Π² ΠΏΡΠΎΡΡΡΠ°Π½ΡΡΠ²Π΅, Π° Π½Π΅ Π²ΠΎ Π²ΡΠ΅ΠΌΠ΅Π½ΠΈ. ΠΠ΄Π½Π°ΠΊΠΎ ΡΠ΅ΡΠ΅Π½ΠΈΠ΅ Π±ΡΠ΄Π΅Ρ ΡΠ°ΠΊΠΈΠΌ ΠΆΠ΅. ΠΠΠ‘ Π΄Π²ΠΈΠΆΠ΅Π½ΠΈΡ ΡΠ°Π²Π½Π°
\[|\epsilon| = \int Bvdl.\]
Π‘ΠΊΠΎΡΠΎΡΡΡ ΠΌΠΎΠΆΠ΅Ρ Π±ΡΡΡ Π·Π°ΠΏΠΈΡΠ°Π½Π° ΠΊΠ°ΠΊ ΠΏΡΠΎΠΈΠ·Π²Π΅Π΄Π΅Π½ΠΈΠ΅ ΡΠ³Π»ΠΎΠ²ΠΎΠΉ ΡΠΊΠΎΡΠΎΡΡΠΈ Π½Π° ΡΠ°Π΄ΠΈΡΡ, Π° Π΄ΠΈΡΡΠ΅ΡΠ΅Π½ΡΠΈΠ°Π»ΡΠ½Π°Ρ Π΄Π»ΠΈΠ½Π° Π·Π°ΠΏΠΈΡΠ°Π½Π° ΠΊΠ°ΠΊ 92,\], ΡΡΠΎ ΡΠ²Π»ΡΠ΅ΡΡΡ ΡΠ΅ΠΌ ΠΆΠ΅ ΡΠ΅ΡΠ΅Π½ΠΈΠ΅ΠΌ, ΡΡΠΎ ΠΈ ΡΠ°Π½ΡΡΠ΅.
ΠΡΠΈΠΌΠ΅Ρ \(\PageIndex{3}\): ΠΏΡΡΠΌΠΎΡΠ³ΠΎΠ»ΡΠ½Π°Ρ ΠΊΠ°ΡΡΡΠΊΠ°, Π²ΡΠ°ΡΠ°ΡΡΠ°ΡΡΡ Π² ΠΌΠ°Π³Π½ΠΈΡΠ½ΠΎΠΌ ΠΏΠΎΠ»Π΅ = B\hat{j}\), ΠΊΠ°ΠΊ ΠΏΠΎΠΊΠ°Π·Π°Π½ΠΎ Π½Π° ΡΠΈΡΡΠ½ΠΊΠ΅ \(\PageIndex{7}\). ΠΠ°ΡΡΡΠΊΠ° Π²ΡΠ°ΡΠ°Π΅ΡΡΡ Π²ΠΎΠΊΡΡΠ³ ΠΎΡΠΈ
z ΡΠ΅ΡΠ΅Π· Π΅Π΅ ΡΠ΅Π½ΡΡ Ρ ΠΏΠΎΡΡΠΎΡΠ½Π½ΠΎΠΉ ΡΠ³Π»ΠΎΠ²ΠΎΠΉ ΡΠΊΠΎΡΠΎΡΡΡΡ \(\omega\). ΠΠΎΠ»ΡΡΠΈΡΠ΅ Π²ΡΡΠ°ΠΆΠ΅Π½ΠΈΠ΅ Π΄Π»Ρ ΠΠΠ‘ ΠΈΠ½Π΄ΡΠΊΡΠΈΠΈ Π² ΠΊΠ°ΡΡΡΠΊΠ΅. Π ΠΈΡΡΠ½ΠΎΠΊ \(\PageIndex{7}\): ΠΏΡΡΠΌΠΎΡΠ³ΠΎΠ»ΡΠ½Π°Ρ ΠΊΠ°ΡΡΡΠΊΠ°, Π²ΡΠ°ΡΠ°ΡΡΠ°ΡΡΡ Π² ΠΎΠ΄Π½ΠΎΡΠΎΠ΄Π½ΠΎΠΌ ΠΌΠ°Π³Π½ΠΈΡΠ½ΠΎΠΌ ΠΏΠΎΠ»Π΅.Π‘ΡΡΠ°ΡΠ΅Π³ΠΈΡ
Π‘ΠΎΠ³Π»Π°ΡΠ½ΠΎ Π΄ΠΈΠ°Π³ΡΠ°ΠΌΠΌΠ΅ ΡΠ³ΠΎΠ» ΠΌΠ΅ΠΆΠ΄Ρ ΠΏΠ΅ΡΠΏΠ΅Π½Π΄ΠΈΠΊΡΠ»ΡΡΠΎΠΌ ΠΊ ΠΏΠΎΠ²Π΅ΡΡ Π½ΠΎΡΡΠΈ ((\hat{n}\)) ΠΈ ΠΌΠ°Π³Π½ΠΈΡΠ½ΡΠΌ ΠΏΠΎΠ»Π΅ΠΌ \((\vec{B})\) ΡΠ°Π²Π΅Π½ \(\theta\ ). Π‘ΠΊΠ°Π»ΡΡΠ½ΠΎΠ΅ ΠΏΡΠΎΠΈΠ·Π²Π΅Π΄Π΅Π½ΠΈΠ΅ \(B \cdot \hat{n}\) ΡΠΏΡΠΎΡΠ°Π΅ΡΡΡ ΡΠΎΠ»ΡΠΊΠΎ Π΄ΠΎ \(cos \, \theta\) ΠΊΠΎΠΌΠΏΠΎΠ½Π΅Π½ΡΠ° ΠΌΠ°Π³Π½ΠΈΡΠ½ΠΎΠ³ΠΎ ΠΏΠΎΠ»Ρ, Π° ΠΈΠΌΠ΅Π½Π½ΠΎ ΡΠ°ΠΌ, Π³Π΄Π΅ ΠΌΠ°Π³Π½ΠΈΡΠ½ΠΎΠ΅ ΠΏΠΎΠ»Π΅ ΠΏΡΠΎΠ΅ΡΠΈΡΡΠ΅ΡΡΡ Π½Π° Π΅Π΄ΠΈΠ½ΠΈΡΠ½ΡΠΉ Π²Π΅ΠΊΡΠΎΡ ΠΏΠ»ΠΎΡΠ°Π΄ΠΈ \(\hat {Π½}\). ΠΠ΅Π»ΠΈΡΠΈΠ½Π° ΠΌΠ°Π³Π½ΠΈΡΠ½ΠΎΠ³ΠΎ ΠΏΠΎΠ»Ρ ΠΈ ΠΏΠ»ΠΎΡΠ°Π΄Ρ ΠΏΠ΅ΡΠ»ΠΈ ΡΠΈΠΊΡΠΈΡΡΡΡΡΡ Π²ΠΎ Π²ΡΠ΅ΠΌΠ΅Π½ΠΈ, ΡΡΠΎ ΡΠΏΡΠΎΡΠ°Π΅Ρ ΠΈΠ½ΡΠ΅Π³ΡΠΈΡΠΎΠ²Π°Π½ΠΈΠ΅. ΠΠΠ‘ ΠΈΠ½Π΄ΡΠΊΡΠΈΠΈ Π·Π°ΠΏΠΈΡΡΠ²Π°Π΅ΡΡΡ ΠΏΠΎ Π·Π°ΠΊΠΎΠ½Ρ Π€Π°ΡΠ°Π΄Π΅Ρ.
Π Π΅ΡΠ΅Π½ΠΈΠ΅
ΠΠΎΠ³Π΄Π° ΠΊΠ°ΡΡΡΠΊΠ° Π½Π°Ρ ΠΎΠ΄ΠΈΡΡΡ Π² ΡΠ°ΠΊΠΎΠΌ ΠΏΠΎΠ»ΠΎΠΆΠ΅Π½ΠΈΠΈ, ΡΡΠΎ Π΅Π΅ Π²Π΅ΠΊΡΠΎΡ Π½ΠΎΡΠΌΠ°Π»ΠΈ \(\hat{n}\) ΠΎΠ±ΡΠ°Π·ΡΠ΅Ρ ΡΠ³ΠΎΠ» \(\theta\) Ρ ΠΌΠ°Π³Π½ΠΈΡΠ½ΡΠΌ ΠΏΠΎΠ»Π΅ΠΌ \(\vec{B}\) ΠΌΠ°Π³Π½ΠΈΡΠ½ΡΠΉ ΠΏΠΎΡΠΎΠΊ ΡΠ΅ΡΠ΅Π· ΠΎΠ΄ΠΈΠ½ Π²ΠΈΡΠΎΠΊ ΠΊΠ°ΡΡΡΠΊΠΈ ΡΠ°Π²Π΅Π½
\[\Phi_m = \int_S \vec{B} \cdot \hat{n} dA = BA \, cos \, \theta.\]
ΠΠ· Π·Π°ΠΊΠΎΠ½Π° Π€Π°ΡΠ°Π΄Π΅Ρ , ΠΠΠ‘ Π² ΠΊΠ°ΡΡΡΠΊΠ΅ ΡΠ°Π²Π½Π°
\[\epsilon = — N\frac{d\Phi_m}{dt} = NB A \, sin \, \theta \frac{d\theta}{dt}.\]
ΠΠΎΡΡΠΎΡΠ½Π½Π°Ρ ΡΠ³Π»ΠΎΠ²Π°Ρ ΡΠΊΠΎΡΠΎΡΡΡ ΡΠ°Π²Π½Π° \(\omega = d\theta /dt\). Π£Π³ΠΎΠ» \(\theta\) ΠΏΡΠ΅Π΄ΡΡΠ°Π²Π»ΡΠ΅Ρ ΠΈΠ·ΠΌΠ΅Π½Π΅Π½ΠΈΠ΅ Π²ΠΎ Π²ΡΠ΅ΠΌΠ΅Π½ΠΈ ΡΠ³Π»ΠΎΠ²ΠΎΠΉ ΡΠΊΠΎΡΠΎΡΡΠΈ ΠΈΠ»ΠΈ \(\omega t\). ΠΡΠΎ ΠΈΠ·ΠΌΠ΅Π½ΡΠ΅Ρ ΡΡΠ½ΠΊΡΠΈΡ Π½Π° Π²ΡΠ΅ΠΌΠ΅Π½Π½ΠΎΠ΅ ΠΏΡΠΎΡΡΡΠ°Π½ΡΡΠ²ΠΎ, Π° Π½Π΅ Π½Π° \(\theta\). Π‘Π»Π΅Π΄ΠΎΠ²Π°ΡΠ΅Π»ΡΠ½ΠΎ, ΠΠΠ‘ ΠΈΠ½Π΄ΡΠΊΡΠΈΠΈ ΠΈΠ·ΠΌΠ΅Π½ΡΠ΅ΡΡΡ ΡΠΈΠ½ΡΡΠΎΠΈΠ΄Π°Π»ΡΠ½ΠΎ ΡΠΎ Π²ΡΠ΅ΠΌΠ΅Π½Π΅ΠΌ ΠΏΠΎ Π·Π°ΠΊΠΎΠ½Ρ 9.0034
\[\epsilon = \epsilon_0 \, sin \, \omega t,\], Π³Π΄Π΅ \(\epsilon_0 = NBA \omega\).
ΠΠ½Π°ΡΠΈΠΌΠΎΡΡΡ
ΠΡΠ»ΠΈ Π±Ρ Π½Π°ΠΏΡΡΠΆΠ΅Π½Π½ΠΎΡΡΡ ΠΌΠ°Π³Π½ΠΈΡΠ½ΠΎΠ³ΠΎ ΠΏΠΎΠ»Ρ ΠΈΠ»ΠΈ ΠΏΠ»ΠΎΡΠ°Π΄Ρ ΠΏΠ΅ΡΠ»ΠΈ ΡΠ°ΠΊΠΆΠ΅ ΠΈΠ·ΠΌΠ΅Π½ΡΠ»ΠΈΡΡ Π²ΠΎ Π²ΡΠ΅ΠΌΠ΅Π½ΠΈ, ΡΡΠΈ ΠΏΠ΅ΡΠ΅ΠΌΠ΅Π½Π½ΡΠ΅ Π½Π΅Π»ΡΠ·Ρ Π±ΡΠ»ΠΎ Π±Ρ Π²ΡΠ²Π΅ΡΡΠΈ ΠΈΠ· ΠΏΡΠΎΠΈΠ·Π²ΠΎΠ΄Π½ΠΎΠΉ ΠΏΠΎ Π²ΡΠ΅ΠΌΠ΅Π½ΠΈ Π΄Π»Ρ ΠΏΡΠΎΡΡΠΎΠ³ΠΎ ΡΠ΅ΡΠ΅Π½ΠΈΡ, ΠΊΠ°ΠΊ ΠΏΠΎΠΊΠ°Π·Π°Π½ΠΎ. ΠΡΠΎΡ ΠΏΡΠΈΠΌΠ΅Ρ ΡΠ²Π»ΡΠ΅ΡΡΡ ΠΎΡΠ½ΠΎΠ²ΠΎΠΉ Π΄Π»Ρ ΡΠ»Π΅ΠΊΡΡΠΈΡΠ΅ΡΠΊΠΎΠ³ΠΎ Π³Π΅Π½Π΅ΡΠ°ΡΠΎΡΠ°, ΠΏΠΎΡΠΊΠΎΠ»ΡΠΊΡ ΠΌΡ Π΄Π°Π΄ΠΈΠΌ ΠΏΠΎΠ»Π½ΠΎΠ΅ ΠΎΠ±ΡΡΠΆΠ΄Π΅Π½ΠΈΠ΅ Π² ΡΠ°Π·Π΄Π΅Π»Π΅ Β«ΠΡΠΈΠ»ΠΎΠΆΠ΅Π½ΠΈΡ Π·Π°ΠΊΠΎΠ½Π° ΠΡΡΡΠΎΠ½Π°Β».
Π£ΠΏΡΠ°ΠΆΠ½Π΅Π½ΠΈΠ΅ \(\PageIndex{1}\)
92 \omega/2\), Ρ O ΠΏΡΠΈ Π±ΠΎΠ»Π΅Π΅ Π²ΡΡΠΎΠΊΠΎΠΌ ΠΏΠΎΡΠ΅Π½ΡΠΈΠ°Π»Π΅, ΡΠ΅ΠΌ SΠ£ΠΏΡΠ°ΠΆΠ½Π΅Π½ΠΈΠ΅ \(\PageIndex{2}\)
Π‘ΡΠ΅ΡΠΆΠ΅Π½Ρ Π΄Π»ΠΈΠ½ΠΎΠΉ 10 ΡΠΌ Π΄Π²ΠΈΠΆΠ΅ΡΡΡ ΡΠΎ ΡΠΊΠΎΡΠΎΡΡΡΡ 10 ΠΌ/Ρ ΠΏΠ΅ΡΠΏΠ΅Π½Π΄ΠΈΠΊΡΠ»ΡΡΠ½ΠΎ ΡΠ΅ΡΠ΅Π· ΠΌΠ°Π³Π½ΠΈΡΠ½ΠΎΠ΅ ΠΏΠΎΠ»Π΅ ΡΠΈΠ»ΠΎΠΉ 1,5 Π’Π». Π§Π΅ΠΌΡ ΡΠ°Π²Π½Π° ΡΠ°Π·Π½ΠΎΡΡΡ ΠΏΠΎΡΠ΅Π½ΡΠΈΠ°Π»ΠΎΠ² ΠΌΠ΅ΠΆΠ΄Ρ ΠΊΠΎΠ½ΡΠ°ΠΌΠΈ ΡΡΠ΅ΡΠΆΠ½Ρ?
- ΠΡΠ²Π΅ΡΠΈΡΡ
1,5 Π
Π‘ΡΠΌΡΡΠ» ΠΠΆ. ΠΠΈΠ½Π³ (ΠΠΎΡΡΠ΄Π°ΡΡΡΠ²Π΅Π½Π½ΡΠΉ ΡΠ½ΠΈΠ²Π΅ΡΡΠΈΡΠ΅Ρ Π’ΡΡΠΌΡΠ½Π°), ΠΠΆΠ΅ΡΡ Π‘Π°Π½Π½ΠΈ (Π£Π½ΠΈΠ²Π΅ΡΡΠΈΡΠ΅Ρ ΠΠΎΠΉΠΎΠ»Ρ ΠΡΡΠΈΠΌΠ°ΡΠ½Ρ) ΠΈ ΠΠΈΠ»Π» ΠΠΎΠ΅Π±ΡΒ ΠΈ ΠΌΠ½ΠΎΠ³ΠΈΠ΅ Π΄ΡΡΠ³ΠΈΠ΅ Π°Π²ΡΠΎΡΡ. ΠΡΠ° ΡΠ°Π±ΠΎΡΠ° Π½Π°Ρ ΠΎΠ΄ΠΈΡΡΡ ΠΏΠΎΠ΄ Π»ΠΈΡΠ΅Π½Π·ΠΈΠ΅ΠΉ OpenStax University Physics Π² ΡΠΎΠΎΡΠ²Π΅ΡΡΡΠ²ΠΈΠΈ Ρ Π»ΠΈΡΠ΅Π½Π·ΠΈΠ΅ΠΉ Creative Commons Attribution License (4.0).
ΠΡΠ° ΡΡΡΠ°Π½ΠΈΡΠ° ΠΏΠΎΠ΄ Π½Π°Π·Π²Π°Π½ΠΈΠ΅ΠΌ 13.4: Motional Emf ΡΠ°ΡΠΏΡΠΎΡΡΡΠ°Π½ΡΠ΅ΡΡΡ ΠΏΠΎΠ΄ Π»ΠΈΡΠ΅Π½Π·ΠΈΠ΅ΠΉ CC BY 4.0 ΠΈ Π±ΡΠ»Π° ΡΠΎΠ·Π΄Π°Π½Π°, ΠΈΠ·ΠΌΠ΅Π½Π΅Π½Π° ΠΈ/ΠΈΠ»ΠΈ ΠΊΡΡΠΈΡΠΎΠ²Π°Π½Π° OpenStax Ρ ΠΈΡΠΏΠΎΠ»ΡΠ·ΠΎΠ²Π°Π½ΠΈΠ΅ΠΌ ΠΈΡΡ ΠΎΠ΄Π½ΠΎΠ³ΠΎ ΠΊΠΎΠ½ΡΠ΅Π½ΡΠ°, ΠΊΠΎΡΠΎΡΡΠΉ Π±ΡΠ» ΠΎΡΡΠ΅Π΄Π°ΠΊΡΠΈΡΠΎΠ²Π°Π½ Π² ΡΠΎΠΎΡΠ²Π΅ΡΡΡΠ²ΠΈΠΈ ΡΠΎ ΡΡΠΈΠ»Π΅ΠΌ ΠΈ ΡΡΠ°Π½Π΄Π°ΡΡΠ°ΠΌΠΈ ΠΏΠ»Π°ΡΡΠΎΡΠΌΡ LibreTexts; ΠΏΠΎΠ΄ΡΠΎΠ±Π½Π°Ρ ΠΈΡΡΠΎΡΠΈΡ ΡΠ΅Π΄Π°ΠΊΡΠΈΡΠΎΠ²Π°Π½ΠΈΡ Π΄ΠΎΡΡΡΠΏΠ½Π° ΠΏΠΎ Π·Π°ΠΏΡΠΎΡΡ.
- ΠΠ°Π²Π΅ΡΡ
- ΠΡΠ»Π° Π»ΠΈ ΡΡΠ° ΡΡΠ°ΡΡΡ ΠΏΠΎΠ»Π΅Π·Π½ΠΎΠΉ?
- Π’ΠΈΠΏ ΠΈΠ·Π΄Π΅Π»ΠΈΡ
- Π Π°Π·Π΄Π΅Π» ΠΈΠ»ΠΈ Π‘ΡΡΠ°Π½ΠΈΡΠ°
- ΠΠ²ΡΠΎΡ
- ΠΠΏΠ΅Π½Π‘ΡΠ°ΠΊΡ
- ΠΠΈΡΠ΅Π½Π·ΠΈΡ
- Π‘Π‘ BY
- ΠΠ΅ΡΡΠΈΡ Π»ΠΈΡΠ΅Π½Π·ΠΈΠΈ
- 4,0
- ΠΡΠΎΠ³ΡΠ°ΠΌΠΌΠ° OER ΠΈΠ»ΠΈ Publisher
- ΠΠΏΠ΅Π½Π‘ΡΠ°ΠΊΡ
- ΠΠΎΠΊΠ°Π·Π°ΡΡ ΠΎΠ³Π»Π°Π²Π»Π΅Π½ΠΈΠ΅
- Π½Π΅Ρ
- Π’Π΅Π³ΠΈ
- ΠΠΠ‘ Π΄Π²ΠΈΠΆΠ΅Π½ΠΈΡ
- ΠΈΡΡΠΎΡΠ½ΠΈΠΊ@https://openstax. org/details/books/university-physics-volume-2
ΠΠ°ΠΊΠΎΠ½ ΡΠ»Π΅ΠΊΡΡΠΎΠΌΠ°Π³Π½ΠΈΡΠ½ΠΎΠΉ ΠΈΠ½Π΄ΡΠΊΡΠΈΠΈ Π€Π°ΡΠ°Π΄Π΅Ρ
ΠΠ°ΠΊΠΎΠ½ ΡΠ»Π΅ΠΊΡΡΠΎΠΌΠ°Π³Π½ΠΈΡΠ½ΠΎΠΉ ΠΈΠ½Π΄ΡΠΊΡΠΈΠΈ Π€Π°ΡΠ°Π΄Π΅Ρ ΠΏΠΎΠΊΠ°Π·ΡΠ²Π°Π΅Ρ Π²Π·Π°ΠΈΠΌΠΎΡΠ²ΡΠ·Ρ ΠΌΠ΅ΠΆΠ΄Ρ ΠΌΠ°Π³Π½ΠΈΡΠ½ΡΠΌ ΠΏΠΎΠ»Π΅ΠΌ, ΡΠ»Π΅ΠΊΡΡΠΈΡΠ΅ΡΠΊΠΈΠΌ ΡΠΎΠΊΠΎΠΌ ΠΈ ΡΠ»Π΅ΠΊΡΡΠΎΠ΄Π²ΠΈΠΆΡΡΠ΅ΠΉ ΡΠΈΠ»ΠΎΠΉ (ΠΠΠ‘).
Π ΠΏΡΠΈΡΠΎΠ΄Π΅ ΡΡΡΠ΅ΡΡΠ²ΡΠ΅Ρ Π΄Π²Π° Π²Π·Π°ΠΈΠΌΠΎΠ·Π°Π²ΠΈΡΠΈΠΌΡΡ ΡΠ²Π»Π΅Π½ΠΈΡ, Π° ΠΈΠΌΠ΅Π½Π½ΠΎ ΡΠ»Π΅ΠΊΡΡΠΈΡΠ΅ΡΡΠ²ΠΎ ΠΈ ΠΌΠ°Π³Π½Π΅ΡΠΈΠ·ΠΌ . ΠΠ»Π΅ΠΊΡΡΠΎΠΌΠ°Π³Π½Π΅ΡΠΈΠ·ΠΌ β ΡΡΠΎ ΡΠ°Π·Π΄Π΅Π» ΡΠΈΠ·ΠΈΠΊΠΈ, ΠΊΠΎΡΠΎΡΡΠΉ Π·Π°Π½ΠΈΠΌΠ°Π΅ΡΡΡ ΠΈΠ·ΡΡΠ΅Π½ΠΈΠ΅ΠΌ ΡΡΡΠ΅ΠΊΡΠΎΠ² ΡΠ»Π΅ΠΊΡΡΠΈΡΠ΅ΡΡΠ²Π° ΠΈ ΠΌΠ°Π³Π½Π΅ΡΠΈΠ·ΠΌΠ° ΠΈ ΠΈΡ Π·Π°Π²ΠΈΡΠΈΠΌΠΎΡΡΠΈ Π΄ΡΡΠ³ ΠΎΡ Π΄ΡΡΠ³Π°.
Π Π½Π°ΡΡΠΎΡΡΠ΅Π΅ Π²ΡΠ΅ΠΌΡ Ρ ΠΎΡΠΎΡΠΎ ΠΈΠ·Π²Π΅ΡΡΠ΅Π½ ΡΠΎΡ ΡΠ°ΠΊΡ, ΡΡΠΎ ΠΈΠ·ΠΌΠ΅Π½ΡΡΡΠ΅Π΅ΡΡ ΠΌΠ°Π³Π½ΠΈΡΠ½ΠΎΠ΅ ΠΏΠΎΠ»Π΅ ΠΌΠΎΠΆΠ΅Ρ ΡΠΎΠ·Π΄Π°Π²Π°ΡΡ ΠΠΠ‘ (ΡΠ»Π΅ΠΊΡΡΠΎΠ΄Π²ΠΈΠΆΡΡΡΡ ΡΠΈΠ»Ρ), Π° Π΄Π²ΠΈΠΆΡΡΠΈΠΉΡΡ Π·Π°ΡΡΠ΄ (ΠΈΠ»ΠΈ ΡΠ»Π΅ΠΊΡΡΠΈΡΠ΅ΡΠΊΠΈΠΉ ΡΠΎΠΊ) ΠΌΠΎΠΆΠ΅Ρ ΡΠΎΠ·Π΄Π°Π²Π°ΡΡ ΠΌΠ°Π³Π½ΠΈΡΠ½ΠΎΠ΅ ΠΏΠΎΠ»Π΅. ΠΠ΄Π½Π°ΠΊΠΎ ΡΡΠΈ ΠΊΠΎΠ½ΡΠ΅ΠΏΡΠΈΠΈ ΡΠ²Π»ΡΡΡΡΡ ΡΠ΅Π·ΡΠ»ΡΡΠ°ΡΠΎΠΌ Π½Π΅ΡΠΊΠΎΠ»ΡΠΊΠΈΡ ΡΠΊΡΠΏΠ΅ΡΠΈΠΌΠ΅Π½ΡΠΎΠ², ΠΏΡΠΎΠ²Π΅Π΄Π΅Π½Π½ΡΡ ΠΌΠ½ΠΎΠ³ΠΈΠΌΠΈ ΡΠΈΠ·ΠΈΠΊΠ°ΠΌΠΈ ΠΈ ΠΈΠ½ΠΆΠ΅Π½Π΅ΡΠ°ΠΌΠΈ.
ΠΠ΄Π½ΠΈΠΌ ΠΈΠ· Π½ΠΈΡ ΡΠ²Π»ΡΠ΅ΡΡΡ ΠΠ°ΠΉΠΊΠ» Π€Π°ΡΠ°Π΄Π΅ΠΉ , ΠΊΠΎΡΠΎΡΡΠΉ ΡΠ°Π·ΡΠ°Π±ΠΎΡΠ°Π» Π΄Π²Π° ΠΎΡΠ½ΠΎΠ²Π½ΡΡ Π·Π°ΠΊΠΎΠ½Π° ΡΠ»Π΅ΠΊΡΡΠΎΠΌΠ°Π³Π½ΠΈΡΠ½ΠΎΠΉ ΠΈΠ½Π΄ΡΠΊΡΠΈΠΈ Π² 1831 Π³ΠΎΠ΄Ρ. ΠΡΠΈ Π·Π°ΠΊΠΎΠ½Ρ ΡΠΈΡΠΎΠΊΠΎ ΠΈΠ·Π²Π΅ΡΡΠ½Ρ ΠΊΠ°ΠΊ ΠΠ΅ΡΠ²ΡΠΉ Π·Π°ΠΊΠΎΠ½ Π€Π°ΡΠ°Π΄Π΅Ρ ΠΈ ΠΡΠΎΡΠΎΠΉ Π·Π°ΠΊΠΎΠ½ Π€Π°ΡΠ°Π΄Π΅Ρ ΡΠ»Π΅ΠΊΡΡΠΎΠΌΠ°Π³Π½ΠΈΡΠ½ΠΎΠΉ ΠΈΠ½Π΄ΡΠΊΡΠΈΠΈ . ΠΡΠΈ Π΄Π²Π° Π·Π°ΠΊΠΎΠ½Π° ΠΎΠ±ΡΡΡΠ½ΡΡΡ, ΠΊΠ°ΠΊ ΠΈΠ·ΠΌΠ΅Π½ΡΡΡΠ΅Π΅ΡΡ ΠΌΠ°Π³Π½ΠΈΡΠ½ΠΎΠ΅ ΠΏΠΎΠ»Π΅ ΡΠΎΠ·Π΄Π°Π΅Ρ ΠΠΠ‘ Π² Π΄Π²ΠΈΠΆΡΡΠ΅ΠΌΡΡ ΠΏΡΠΎΠ²ΠΎΠ΄Π½ΠΈΠΊΠ΅.
Π§ΡΠΎ ΡΠ°ΠΊΠΎΠ΅ ΡΠ»Π΅ΠΊΡΡΠΎΠΌΠ°Π³Π½ΠΈΡΠ½Π°Ρ ΠΈΠ½Π΄ΡΠΊΡΠΈΡ?ΠΡΠΈ ΠΈΠ·ΠΌΠ΅Π½Π΅Π½ΠΈΠΈ ΠΌΠ°Π³Π½ΠΈΡΠ½ΠΎΠ³ΠΎ ΠΏΠΎΠ»Ρ, ΡΠ²ΡΠ·Π°Π½Π½ΠΎΠ³ΠΎ Ρ ΠΏΡΠΎΠ²ΠΎΠ΄Π½ΠΈΠΊΠΎΠΌ, Π² ΠΏΡΠΎΠ²ΠΎΠ΄Π½ΠΈΠΊΠ΅ ΠΈΠ½Π΄ΡΡΠΈΡΡΠ΅ΡΡΡ ΡΠ»Π΅ΠΊΡΡΠΎΠ΄Π²ΠΈΠΆΡΡΠ°Ρ ΡΠΈΠ»Π° (ΠΠΠ‘), ΡΡΠΎ ΡΠ²Π»Π΅Π½ΠΈΠ΅ ΠΈΠ·Π²Π΅ΡΡΠ½ΠΎ ΠΊΠ°ΠΊ ΡΠ»Π΅ΠΊΡΡΠΎΠΌΠ°Π³Π½ΠΈΡΠ½Π°Ρ ΠΈΠ½Π΄ΡΠΊΡΠΈΡ . ΠΠ»Π΅ΠΊΡΡΠΎΠΌΠ°Π³Π½ΠΈΡΠ½Π°Ρ ΠΈΠ½Π΄ΡΠΊΡΠΈΡ ΡΠ²Π»ΡΠ΅ΡΡΡ Π½Π°ΠΈΠ±ΠΎΠ»Π΅Π΅ ΡΡΠ½Π΄Π°ΠΌΠ΅Π½ΡΠ°Π»ΡΠ½ΡΠΌ ΠΏΡΠΈΠ½ΡΠΈΠΏΠΎΠΌ, Π½Π° ΠΊΠΎΡΠΎΡΠΎΠΌ ΡΠ΅Π³ΠΎΠ΄Π½Ρ ΡΠ°Π±ΠΎΡΠ°ΡΡ ΠΌΠ½ΠΎΠ³ΠΈΠ΅ ΡΠ»Π΅ΠΊΡΡΠΈΡΠ΅ΡΠΊΠΈΠ΅ ΡΡΡΡΠΎΠΉΡΡΠ²Π°, ΡΠ°ΠΊΠΈΠ΅ ΠΊΠ°ΠΊ Π΄Π²ΠΈΠ³Π°ΡΠ΅Π»ΠΈ, Π³Π΅Π½Π΅ΡΠ°ΡΠΎΡΡ, ΠΈΠ·ΠΌΠ΅ΡΠΈΡΠ΅Π»ΡΠ½ΡΠ΅ ΠΏΡΠΈΠ±ΠΎΡΡ ΠΈ Ρ. Π΄.
ΠΠ°ΠΊΠΎΠ½ ΡΠ»Π΅ΠΊΡΡΠΎΠΌΠ°Π³Π½ΠΈΡΠ½ΠΎΠΉ ΠΈΠ½Π΄ΡΠΊΡΠΈΠΈ Π€Π°ΡΠ°Π΄Π΅ΡΠΠ΅ΡΠ²ΡΠΉ Π·Π°ΠΊΠΎΠ½ ΡΠ»Π΅ΠΊΡΡΠΎΠΌΠ°Π³Π½ΠΈΡΠ½ΠΎΠΉ ΠΈΠ½Π΄ΡΠΊΡΠΈΠΈ Π€Π°ΡΠ°Π΄Π΅Ρ ΠΎΠΏΠΈΡΡΠ²Π°Π΅Ρ ΠΈΠ½Π΄ΡΠΊΡΠΈΡ ΡΠ»Π΅ΠΊΡΡΠΎΠ΄Π²ΠΈΠΆΡΡΠ΅ΠΉ ΡΠΈΠ»Ρ Π² ΠΏΡΠΎΠ²ΠΎΠ΄Π½ΠΈΠΊΠ΅, Π° Π²ΡΠΎΡΠΎΠΉ Π·Π°ΠΊΠΎΠ½ Π€Π°ΡΠ°Π΄Π΅Ρ Π΄Π°Π΅Ρ Π²Π΅Π»ΠΈΡΠΈΠ½Ρ ΡΠ»Π΅ΠΊΡΡΠΎΠ΄Π²ΠΈΠΆΡΡΠ΅ΠΉ ΡΠΈΠ»Ρ, ΠΈΠ½Π΄ΡΡΠΈΡΠΎΠ²Π°Π½Π½ΠΎΠΉ Π² ΠΏΡΠΎΠ²ΠΎΠ΄Π½ΠΈΠΊΠ΅.
ΠΠ²Π° ΡΠΈΠ·ΠΈΠΊΠ° ΠΠ°ΠΉΠΊΠ» Π€Π°ΡΠ°Π΄Π΅ΠΉ ΠΈ ΠΠΆΠΎΠ·Π΅Ρ ΠΠ΅Π½ΡΠΈ ΠΏΡΠΎΠ²Π΅Π»ΠΈ Π΄Π»ΠΈΠ½Π½ΡΡ ΡΠ΅ΡΠΈΡ ΡΠΊΡΠΏΠ΅ΡΠΈΠΌΠ΅Π½ΡΠΎΠ². ΠΠ· Π½Π°Π±Π»ΡΠ΄Π΅Π½ΠΈΠΉ Π·Π° ΡΡΠΈΠΌΠΈ ΡΠΊΡΠΏΠ΅ΡΠΈΠΌΠ΅Π½ΡΠ°ΠΌΠΈ Π€Π°ΡΠ°Π΄Π΅ΠΉ ΠΎΠΏΠΈΡΠ°Π», ΡΡΠΎ ΠΊΠΎΠ³Π΄Π° ΠΈΠ·ΠΌΠ΅Π½ΡΡΡΠ΅Π΅ΡΡ ΠΌΠ°Π³Π½ΠΈΡΠ½ΠΎΠ΅ ΠΏΠΎΠ»Π΅ ΡΠΎΠ΅Π΄ΠΈΠ½ΡΠ΅ΡΡΡ Ρ ΠΏΡΠΎΠ²ΠΎΠ΄Π½ΠΈΠΊΠΎΠΌ, Π² ΠΏΡΠΎΠ²ΠΎΠ΄Π½ΠΈΠΊΠ΅ ΠΈΠ½Π΄ΡΡΠΈΡΡΠ΅ΡΡΡ ΠΠΠ‘.
ΠΠ΅ΡΠ²ΡΠΉ Π·Π°ΠΊΠΎΠ½ Π€Π°ΡΠ°Π΄Π΅ΡΠΠ΅ΡΠ²ΡΠΉ Π·Π°ΠΊΠΎΠ½ Π€Π°ΡΠ°Π΄Π΅Ρ ΡΠ»Π΅ΠΊΡΡΠΎΠΌΠ°Π³Π½ΠΈΡΠ½ΠΎΠΉ ΠΈΠ½Π΄ΡΠΊΡΠΈΠΈ ΡΡΠ²Π΅ΡΠΆΠ΄Π°Π΅Ρ, ΡΡΠΎ Β« ΠΡΡΠΊΠΈΠΉ ΡΠ°Π·, ΠΊΠΎΠ³Π΄Π° ΠΌΠ°Π³Π½ΠΈΡΠ½ΠΎΠ΅ ΠΏΠΎΠ»Π΅, ΡΠ²ΡΠ·Π°Π½Π½ΠΎΠ΅ Ρ ΠΏΡΠΎΠ²ΠΎΠ΄Π½ΠΈΠΊΠΎΠΌ, ΠΈΠ·ΠΌΠ΅Π½ΡΠ΅ΡΡΡ, Π² ΠΏΡΠΎΠ²ΠΎΠ΄Π½ΠΈΠΊΠ΅ ΠΈΠ½Π΄ΡΡΠΈΡΡΠ΅ΡΡΡ ΡΠ»Π΅ΠΊΡΡΠΎΠ΄Π²ΠΈΠΆΡΡΠ°Ρ ΡΠΈΠ»Π° (ΠΠΠ‘) Β».
ΠΡΠ»ΠΈ ΡΠ΅ΠΏΡ ΠΏΡΠΎΠ²ΠΎΠ΄Π½ΠΈΠΊΠ° Π·Π°ΠΌΠΊΠ½ΡΡΠ°, ΠΏΠΎ ΠΏΡΠΎΠ²ΠΎΠ΄Π½ΠΈΠΊΡ Π½Π°ΡΠΈΠ½Π°Π΅Ρ ΡΠ΅ΡΡ ΡΠΎΠΊ, Π½Π°Π·ΡΠ²Π°Π΅ΠΌΡΠΉ ΠΈΠ½Π΄ΡΠΊΡΠΈΠ²Π½ΡΠΌ ΡΠΎΠΊΠΎΠΌ.
ΠΠ·ΠΌΠ΅Π½Π΅Π½ΠΈΠ΅ ΠΌΠ°Π³Π½ΠΈΡΠ½ΠΎΠ³ΠΎ ΠΏΠΎΠ»Ρ, ΡΠ²ΡΠ·Π°Π½Π½ΠΎΠ³ΠΎ Ρ ΠΏΡΠΎΠ²ΠΎΠ΄Π½ΠΈΠΊΠΎΠΌ, ΠΌΠΎΠΆΠ½ΠΎ Π²ΡΠ·Π²Π°ΡΡ Π΄Π²ΡΠΌΡ ΡΠΏΠΎΡΠΎΠ±Π°ΠΌΠΈ:
- ΠΡΡΠ΅ΠΌ ΠΏΠ΅ΡΠ΅ΠΌΠ΅ΡΠ΅Π½ΠΈΡ ΠΏΡΠΎΠ²ΠΎΠ΄Π½ΠΈΠΊΠ° ΠΎΡΠ½ΠΎΡΠΈΡΠ΅Π»ΡΠ½ΠΎ ΡΡΠ°ΡΠΈΠΎΠ½Π°ΡΠ½ΠΎΠ³ΠΎ ΠΌΠ°Π³Π½ΠΈΡΠ½ΠΎΠ³ΠΎ ΠΏΠΎΠ»Ρ.
- ΠΡΡΠ΅ΠΌ ΠΏΠ΅ΡΠ΅ΠΌΠ΅ΡΠ΅Π½ΠΈΡ ΠΌΠ°Π³Π½ΠΈΡΠ½ΠΎΠ³ΠΎ ΠΏΠΎΠ»Ρ, ΠΊΠΎΠ³Π΄Π° ΠΏΡΠΎΠ²ΠΎΠ΄Π½ΠΈΠΊ Π½Π΅ΠΏΠΎΠ΄Π²ΠΈΠΆΠ΅Π½.
ΠΡΠΎΡΠΎΠΉ Π·Π°ΠΊΠΎΠ½ Π€Π°ΡΠ°Π΄Π΅Ρ ΡΡΠ²Π΅ΡΠΆΠ΄Π°Π΅Ρ, ΡΡΠΎ Β« Π²Π΅Π»ΠΈΡΠΈΠ½Π° ΠΠΠ‘ ΠΈΠ½Π΄ΡΠΊΡΠΈΠΈ Π² ΠΏΡΠΎΠ²ΠΎΠ΄Π½ΠΈΠΊΠ΅ ΡΠ°Π²Π½Π° ΡΠΊΠΎΡΠΎΡΡΠΈ ΠΈΠ·ΠΌΠ΅Π½Π΅Π½ΠΈΡ ΠΌΠ°Π³Π½ΠΈΡΠ½ΠΎΠ³ΠΎ ΠΏΠΎΡΠΎΠΊΠ°, ΡΠ²ΡΠ·Π°Π½Π½ΠΎΠ³ΠΎ Ρ ΠΏΡΠΎΠ²ΠΎΠ΄Π½ΠΈΠΊΠΎΠΌ Β».
ΠΡΠ²ΠΎΠ΄ Π·Π°ΠΊΠΎΠ½Π° Π€Π°ΡΠ°Π΄Π΅ΡΠ Π°ΡΡΠΌΠΎΡΡΠΈΠΌ ΠΊΠ°ΡΡΡΠΊΡ Ρ N ΠΏΡΠΎΠ²ΠΎΠ΄Π½ΠΈΠΊΠ°ΠΌΠΈ ΠΈ ΠΌΠ°Π³Π½ΠΈΡ Π΄Π²ΠΈΠΆΠ΅ΡΡΡ ΠΊ ΠΊΠ°ΡΡΡΠΊΠ΅, Π·Π°ΡΠ΅ΠΌ
Π Π½Π°ΡΠ°Π»ΡΠ½ΠΎΠΌ ΠΏΠΎΠ»ΠΎΠΆΠ΅Π½ΠΈΠΈ ΠΏΠΎΡΠΎΠΊΠΎΡΡΠ΅ΠΏΠ»Π΅Π½ΠΈΠ΅ Ρ ΠΊΠ°ΡΡΡΠΊΠΎΠΉ ΡΠ°Π²Π½ΠΎ
Π ΠΊΠΎΠ½Π΅ΡΠ½ΠΎΠΌ ΡΠΎΡΡΠΎΡΠ½ΠΈΠΈ ΠΏΠΎΡΠΎΠΊΠΎΡΡΠ΅ΠΏΠ»Π΅Π½ΠΈΠ΅ Ρ ΠΊΠ°ΡΡΡΠΊΠΎΠΉ
Π‘Π»Π΅Π΄ΠΎΠ²Π°ΡΠ΅Π»ΡΠ½ΠΎ, ΠΈΠ·ΠΌΠ΅Π½Π΅Π½ΠΈΠ΅ ΠΏΠΎΡΠΎΠΊΠΎΡΡΠ΅ΠΏΠ»Π΅Π½ΠΈΡ ΡΠ°Π²Π½ΠΎΠ’Π΅ΠΏΠ΅ΡΡ ΡΠΊΠΎΡΠΎΡΡΡ ΠΈΠ·ΠΌΠ΅Π½Π΅Π½ΠΈΡ ΡΡΠΎΠ³ΠΎ ΠΏΠΎΡΠΎΠΊΠΎΡΡΠ΅ΠΏΠ»Π΅Π½ΠΈΡ ΡΠ°Π²Π½Π°
Π‘ΠΎΠ³Π»Π°ΡΠ½ΠΎ Π²ΡΠΎΡΠΎΠΌΡ Π·Π°ΠΊΠΎΠ½Ρ ΡΠ»Π΅ΠΊΡΡΠΎΠΌΠ°Π³Π½ΠΈΡΠ½ΠΎΠΉ ΠΈΠ½Π΄ΡΠΊΡΠΈΠΈ Π€Π°ΡΠ°Π΄Π΅Ρ, ΠΠΠ‘, ΠΈΠ½Π΄ΡΡΠΈΡΡΠ΅ΠΌΠ°Ρ Π² ΠΏΡΠΎΠ²ΠΎΠ΄Π½ΠΈΠΊΠ΅, ΡΠ°Π²Π½Π° ΡΠΊΠΎΡΠΎΡΡΠΈ ΠΈΠ·ΠΌΠ΅Π½Π΅Π½ΠΈΡ ΠΏΠΎΡΠΎΠΊΠΎΡΡΠ΅ΠΏΠ»Π΅Π½ΠΈΡ.
Π Π΄ΠΈΡΡΠ΅ΡΠ΅Π½ΡΠΈΠ°Π»ΡΠ½ΠΎΠΉ ΡΠΎΡΠΌΠ΅,
ΠΠ΄Π΅, π = 2 — 1 , ΠΏΠΎΠ»Π½ΠΎΠ΅ ΠΈΠ·ΠΌΠ΅Π½Π΅Π½ΠΈΠ΅ ΠΌΠ°Π³Π½ΠΈΡΠ½ΠΎΠ³ΠΎ ΠΏΠΎΡΠΎΠΊΠ°.
ΠΡΠΈΠΌΠ΅Π½Π΅Π½ΠΈΠ΅ Π·Π°ΠΊΠΎΠ½ΠΎΠ² ΡΠ»Π΅ΠΊΡΡΠΎΠΌΠ°Π³Π½ΠΈΡΠ½ΠΎΠΉ ΠΈΠ½Π΄ΡΠΊΡΠΈΠΈ Π€Π°ΡΠ°Π΄Π΅ΡΠΠ°ΠΊΠΎΠ½Ρ Π€Π°ΡΠ°Π΄Π΅Ρ ΡΠ²Π»ΡΡΡΡΡ Π½Π°ΠΈΠ±ΠΎΠ»Π΅Π΅ ΡΡΠ½Π΄Π°ΠΌΠ΅Π½ΡΠ°Π»ΡΠ½ΡΠΌΠΈ Π·Π°ΠΊΠΎΠ½Π°ΠΌΠΈ ΡΠ»Π΅ΠΊΡΡΠΎΠΌΠ°Π³Π½Π΅ΡΠΈΠ·ΠΌΠ°. ΠΡΠΈ Π·Π°ΠΊΠΎΠ½Ρ ΡΠΈΡΠΎΠΊΠΎ ΠΈΡΠΏΠΎΠ»ΡΠ·ΡΡΡΡΡ Π²ΠΎ ΠΌΠ½ΠΎΠ³ΠΈΡ ΠΎΠ±Π»Π°ΡΡΡΡ ΡΠ»Π΅ΠΊΡΡΠΎΡΠ΅Ρ Π½ΠΈΠΊΠΈ, ΡΠ°ΠΊΠΈΡ ΠΊΠ°ΠΊ ΡΠ»Π΅ΠΊΡΡΠΈΡΠ΅ΡΠΊΠΈΠ΅ ΠΌΠ°ΡΠΈΠ½Ρ, ΠΈΠ·ΠΌΠ΅ΡΠΈΡΠ΅Π»ΡΠ½ΡΠ΅ ΠΏΡΠΈΠ±ΠΎΡΡ, Π² ΠΌΠ΅Π΄ΠΈΡΠΈΠ½Π΅ Π΄Π»Ρ Π΄ΠΈΠ°Π³Π½ΠΎΡΡΠΈΠΊΠΈ Π·Π°Π±ΠΎΠ»Π΅Π²Π°Π½ΠΈΠΉ ΠΈ Ρ. Π΄. ΠΠΎΡ Π½Π΅ΠΊΠΎΡΠΎΡΡΠ΅ ΡΠ°ΡΠΏΡΠΎΡΡΡΠ°Π½Π΅Π½Π½ΡΠ΅ ΠΏΡΠΈΠΌΠ΅Π½Π΅Π½ΠΈΡ Π·Π°ΠΊΠΎΠ½ΠΎΠ² Π€Π°ΡΠ°Π΄Π΅Ρ:
- Π’ΡΠ°Π½ΡΡΠΎΡΠΌΠ°ΡΠΎΡ ΡΠ°Π±ΠΎΡΠ°Π΅Ρ ΠΏΠΎ ΠΏΡΠΈΠ½ΡΠΈΠΏΡ ΡΠ»Π΅ΠΊΡΡΠΎΠΌΠ°Π³Π½ΠΈΡΠ½ΠΎΠΉ ΠΈΠ½Π΄ΡΠΊΡΠΈΠΈ. ΠΠΎΡΡΠΎΠΌΡ Π·Π°ΠΊΠΎΠ½ Π€Π°ΡΠ°Π΄Π΅Ρ ΠΌΠΎΠΆΠ΅Ρ ΡΠ»ΡΠΆΠΈΡΡ Π΄Π»Ρ Π΅Π³ΠΎ Π°Π½Π°Π»ΠΈΠ·Π°.
- ΠΡΠΎΠΈΠ·Π²ΠΎΠ΄ΡΡΠ²ΠΎ ΡΠ»Π΅ΠΊΡΡΠΎΡΠ½Π΅ΡΠ³ΠΈΠΈ Π³Π΅Π½Π΅ΡΠ°ΡΠΎΡΠΎΠΌ ΡΠ°ΠΊΠΆΠ΅ ΠΎΡΠ½ΠΎΠ²Π°Π½ΠΎ Π½Π° Π·Π°ΠΊΠΎΠ½Π°Ρ ΡΠ»Π΅ΠΊΡΡΠΎΠΌΠ°Π³Π½ΠΈΡΠ½ΠΎΠΉ ΠΈΠ½Π΄ΡΠΊΡΠΈΠΈ Π€Π°ΡΠ°Π΄Π΅Ρ.
- ΠΠ°ΠΊΠΎΠ½Ρ Π€Π°ΡΠ°Π΄Π΅Ρ ΡΠ°ΠΊΠΆΠ΅ Π»Π΅ΠΆΠ°Ρ Π² ΠΎΡΠ½ΠΎΠ²Π΅ ΡΠ°Π±ΠΎΡΡ ΠΌΠ½ΠΎΠ³ΠΈΡ Π΄ΡΡΠ³ΠΈΡ ΠΏΡΠΈΠ±ΠΎΡΠΎΠ², ΡΠ°ΠΊΠΈΡ ΠΊΠ°ΠΊ ΠΈΠ½Π΄ΡΠΊΡΠΈΠΎΠ½Π½ΡΠ΅ ΠΏΠ»ΠΈΡΡ, ΡΠ»Π΅ΠΊΡΡΠΎΠΌΠ°Π³Π½ΠΈΡΠ½ΡΠ΅ ΡΠ°ΡΡ ΠΎΠ΄ΠΎΠΌΠ΅ΡΡ, ΡΠ»Π΅ΠΊΡΡΠΎΠ³ΠΈΡΠ°ΡΡ, ΡΠ»Π΅ΠΊΡΡΠΎΡΠΊΡΠΈΠΏΠΊΠΈ ΠΈ Ρ. Π΄.
ΠΠ°ΡΡΡΠΊΠ° Ρ 700 Π²ΠΈΡΠΊΠ°ΠΌΠΈ ΡΠ°Π·Π²ΠΈΠ²Π°Π΅Ρ ΡΡΠ΅Π΄Π½Π΅Π΅ ΠΈΠ½Π΄ΡΡΠΈΡΡΠ΅ΠΌΠΎΠ΅ Π½Π°ΠΏΡΡΠΆΠ΅Π½ΠΈΠ΅ 50 Π. ΠΠ°ΠΊΠΈΠΌ Π΄ΠΎΠ»ΠΆΠ½ΠΎ Π±ΡΡΡ ΠΈΠ·ΠΌΠ΅Π½Π΅Π½ΠΈΠ΅ ΠΌΠ°Π³Π½ΠΈΡΠ½ΠΎΠ³ΠΎ ΠΏΠΎΡΠΎΠΊΠ°, ΡΡΠΎΠ±Ρ Π²ΠΎΠ·Π½ΠΈΠΊΠ»ΠΎ ΡΠ°ΠΊΠΎΠ΅ Π½Π°ΠΏΡΡΠΆΠ΅Π½ΠΈΠ΅, Π΅ΡΠ»ΠΈ ΠΈΠ½ΡΠ΅ΡΠ²Π°Π» Π²ΡΠ΅ΠΌΠ΅Π½ΠΈ Π΄Π»Ρ ΡΡΠΎΠ³ΠΎ ΠΈΠ·ΠΌΠ΅Π½Π΅Π½ΠΈΡ ΡΠΎΡΡΠ°Π²Π»ΡΠ΅Ρ 0,7 ΡΠ΅ΠΊΡΠ½Π΄Ρ.
ΠΠ°Π½Π½ΡΠ΅ :
Π’Π΅ΠΏΠ΅ΡΡ ΠΏΠΎ Π·Π°ΠΊΠΎΠ½Ρ Π€Π°ΡΠ°Π΄Π΅Ρ ΠΏΠΎΠ»ΡΡΠ°Π΅ΠΌ
ΠΠ°Π³Π½ΠΈΡΠ½ΡΠΉ ΠΏΠΎΡΠΎΠΊ, ΡΠ²ΡΠ·Π°Π½Π½ΡΠΉ Ρ ΠΊΠ°ΡΡΡΠΊΠΎΠΉ, ΠΈΠΌΠ΅ΡΡΠ΅ΠΉ 250 Π²ΠΈΡΠΊΠΎΠ², ΠΈΠ·ΠΌΠ΅Π½ΡΠ΅ΡΡΡ Ρ 1,4 ΠΠ± Π΄ΠΎ 2 ΠΠ± Π·Π° 0,45 ΡΠ΅ΠΊΡΠ½Π΄Ρ. ΠΡΡΠΈΡΠ»ΠΈΡΠ΅ ΠΠΠ‘ ΠΈΠ½Π΄ΡΠΊΡΠΈΠΈ Π² ΠΊΠ°ΡΡΡΠΊΠ΅.
ΠΠ°Π½Π½ΡΠ΅ Π΄Π°Π½Π½ΡΠ΅,
Π’Π΅ΠΏΠ΅ΡΡ, ΡΠΎΠ³Π»Π°ΡΠ½ΠΎ Π·Π°ΠΊΠΎΠ½Ρ Π€Π°ΡΠ°Π΄Π΅Ρ, Ρ Π½Π°Ρ Π΅ΡΡΡ,
Π§ΠΈΡΠ°ΡΡ Π΄Π°Π»Π΅Π΅
ΠΠΎΡ ΠΎΠΆΠΈΠ΅ ΠΏΠΎΡΡΡ:
ΠΠΎΠ΄ΠΏΠΈΡΡΠ²Π°ΠΉΡΠ΅ΡΡ ΠΈ Π»Π°ΠΉΠΊΠ°ΠΉΡΠ΅ Π½Π°Ρ:
ΠΠ»Π΅ΠΊΡΡΠΎΠΌΠ°Π³Π½ΠΈΡΠ½Π°Ρ ΠΈΠ½Π΄ΡΠΊΡΠΈΡ.
ΠΠΠ‘ Π² Π΄Π²ΠΈΠΆΡΡΠ΅ΠΌΡΡ ΠΏΡΠΎΠ²ΠΎΠ΄Π½ΠΈΠΊΠ΅. ΠΠ°ΠΊΠΎΠ½ Π€Π°ΡΠ°Π΄Π΅Ρ. ΠΠ°ΠΊΠΎΠ½ ΠΠ΅Π½ΡΠ°. Π‘Π°ΠΌΠΎΠΈΠ½Π΄ΡΠΊΡΠΈΡ. ΠΠΠ‘ ΡΠ°ΠΌΠΎΠΈΠ½Π΄ΡΠΊΡΠΈΠΈ. Π‘Π°ΠΌΠΎΠΈΠ½Π΄ΡΠΊΡΠΈΡ ΠΊΠ°ΡΡΡΠΊΠΈ ΠΈΠ· n Π²ΠΈΡΠΊΠΎΠ². ΠΠ½Π΅ΡΠ³ΠΈΡ, Π·Π°ΠΏΠ°ΡΠ΅Π½Π½Π°Ρ Π² ΠΈΠ½Π΄ΡΠΊΡΠΎΡΠ΅. ΠΠ»Π΅ΠΊΡΡΠΈΡΠ΅ΡΠΊΠΈΠ΅ ΠΊΠΎΠ»Π΅Π±Π°Π½ΠΈΡ. ΠΠ»Π΅ΠΊΡΡΠΎΠ³Π΅Π½Π΅ΡΠ°ΡΠΎΡ, Π΄Π²ΠΈΠ³Π°ΡΠ΅Π»Ρ. ΠΠ»Π΅ΠΊΡΡΠΎΠΌΠ°Π³Π½ΠΈΡΠ½Π°Ρ ΠΈΠ½Π΄ΡΠΊΡΠΈΡ. ΠΠΠ‘ Π² Π΄Π²ΠΈΠΆΡΡΠ΅ΠΌΡΡ ΠΏΡΠΎΠ²ΠΎΠ΄Π½ΠΈΠΊΠ΅. ΠΠ°ΠΊΠΎΠ½ Π€Π°ΡΠ°Π΄Π΅Ρ. ΠΠ°ΠΊΠΎΠ½ ΠΠ΅Π½ΡΠ°. Π‘Π°ΠΌΠΎΠΈΠ½Π΄ΡΠΊΡΠΈΡ. ΠΠΠ‘ ΡΠ°ΠΌΠΎΠΈΠ½Π΄ΡΠΊΡΠΈΠΈ. Π‘Π°ΠΌΠΎΠΈΠ½Π΄ΡΠΊΡΠΈΡ ΠΊΠ°ΡΡΡΠΊΠΈ ΠΈΠ· n Π²ΠΈΡΠΊΠΎΠ². ΠΠ½Π΅ΡΠ³ΠΈΡ, Ρ ΡΠ°Π½ΡΡΠ°ΡΡΡ Π² ΠΈΠ½Π΄ΡΠΊΡΠΎΡ. ΠΠ»Π΅ΠΊΡΡΠΈΡΠ΅ΡΠΊΠΈΠ΅ ΠΊΠΎΠ»Π΅Π±Π°Π½ΠΈΡ. ΠΠ»Π΅ΠΊΡΡΠΎΠ³Π΅Π½Π΅ΡΠ°ΡΠΎΡ, Π΄Π²ΠΈΠ³Π°ΡΠ΅Π»Ρ.SolitaryRoad.com
ΠΠ»Π°Π΄Π΅Π»Π΅Ρ ΡΠ°ΠΉΡΠ°: ΠΠΆΠ΅ΠΉΠΌΡ ΠΠΈΠ»Π»Π΅Ρ
[ ΠΠΎΠΌ ] [ ΠΠ²Π΅ΡΡ ] [ ΠΠ½ΡΠΎΡΠΌΠ°ΡΠΈΡ ] [ ΠΠΎΡΡΠ° ]
ΠΠ»Π΅ΠΊΡΡΠΎΠΌΠ°Π³Π½ΠΈΡΠ½Π°Ρ ΠΈΠ½Π΄ΡΠΊΡΠΈΡ. ΠΠΠ‘, Π½Π°Π²Π΅Π΄Π΅Π½Π½Π°Ρ Π² Π΄Π²ΠΈΠΆΡΡΠ΅ΠΌΡΡ ΠΏΡΠΎΠ²ΠΎΠ΄Π½ΠΈΠΊ. ΠΠ°ΠΊΠΎΠ½ Π€Π°ΡΠ°Π΄Π΅Ρ. ΠΠ°ΠΊΠΎΠ½ ΠΠ΅Π½ΡΠ°. Π‘Π°ΠΌΠΎΠΈΠ½Π΄ΡΠΊΡΠΈΡ. ΠΠΠ‘ ΡΠ°ΠΌΠΎΠΈΠ½Π΄ΡΠΊΡΠΈΠΈ. Π‘Π°ΠΌΠΎΠΈΠ½Π΄ΡΠΊΡΠΈΡ ΠΊΠ°ΡΡΡΠΊΠΈ ΠΈΠ· n ΠΎΠ±ΠΎΡΠΎΡΠΎΠ². ΠΠ½Π΅ΡΠ³ΠΈΡ, Π·Π°ΠΏΠ°ΡΠ΅Π½Π½Π°Ρ Π² ΠΈΠ½Π΄ΡΠΊΡΠΎΡΠ΅. ΠΠ»Π΅ΠΊΡΡΠΈΡΠ΅ΡΠΊΠΈΠΉ ΠΊΠΎΠ»Π΅Π±Π°Π½ΠΈΡ. ΠΠ»Π΅ΠΊΡΡΠΎΠ³Π΅Π½Π΅ΡΠ°ΡΠΎΡ, Π΄Π²ΠΈΠ³Π°ΡΠ΅Π»Ρ.
ΠΠ»Π΅ΠΊΡΡΠΎΠΌΠ°Π³Π½ΠΈΡΠ½Π°Ρ ΠΈΠ½Π΄ΡΠΊΡΠΈΡ. Π Π²Π΅Π»ΠΈΠΊΠ°Ρ Π²Π΅Ρ Π° ΠΏΡΠΎΠΈΠ·ΠΎΡΠ»Π°, ΠΊΠΎΠ³Π΄Π° ΠΠ°Π½Ρ ΠΡΠΈΡΡΠΈΠ°Π½ ΠΡΡΡΠ΅Π΄ ΠΎΡΠΊΡΡΠ» Π² 1819 Π³.Π° ΡΠ²ΡΠ·Ρ ΠΌΠ΅ΠΆΠ΄Ρ ΡΠ»Π΅ΠΊΡΡΠΈΡΠ΅ΡΡΠ²ΠΎΠΌ ΠΈ ΠΌΠ°Π³Π½Π΅ΡΠΈΠ·ΠΌ Π² Π²ΠΈΠ΄Π΅ ΠΌΠ°Π³Π½Π΅ΡΠΈΠ·ΠΌΠ° ΠΏΠΎΠ»Ρ Π²Π±Π»ΠΈΠ·ΠΈ ΠΏΡΠΎΠ²ΠΎΠ΄Π½ΠΈΠΊΠ° Ρ ΡΠΎΠΊΠΎΠΌ. ΠΡΠΎΠΈΠ·ΠΎΡΠ»Π° Π΅ΡΠ΅ ΠΎΠ΄Π½Π° Π²Π°ΠΆΠ½Π°Ρ Π²Π΅Ρ Π° 12 Π»Π΅Ρ ΡΠΏΡΡΡΡ, Π² 1831 Π³ΠΎΠ΄Ρ, ΠΊΠΎΠ³Π΄Π° ΠΠ°ΠΉΠΊΠ» Π€Π°ΡΠ°Π΄Π΅ΠΉ ΠΎΡΠΊΡΡΠ» Π΅ΡΠ΅ ΠΎΠ΄ΠΈΠ½ ΡΠ²Π»Π΅Π½ΠΈΠ΅, ΡΠ²ΡΠ·Π°Π½Π½ΠΎΠ΅ Ρ ΡΠ»Π΅ΠΊΡΡΠΈΡΠ΅ΡΡΠ²ΠΎΠΌ ΠΈ ΠΌΠ°Π³Π½Π΅ΡΠΈΠ·ΠΌ: ΠΎΠ½ ΠΎΡΠΊΡΡΠ» ΡΠ²Π»Π΅Π½ΠΈΠ΅, Π½Π°Π·ΡΠ²Π°Π΅ΠΌΠΎΠ΅ ΡΠ»Π΅ΠΊΡΡΠΎΠΌΠ°Π³Π½ΠΈΡΠ½ΡΠΌ ΠΈΠ½Π΄ΡΠΊΡΠΈΡ. ΠΡΠΎ ΠΎΡΠΊΡΡΡΠΈΠ΅ ΡΠ΄Π΅Π»Π°Π»ΠΎ Π²ΠΎΠ·ΠΌΠΎΠΆΠ½ΡΠΉ ΡΠΏΠΎΡΠΎΠ± Π³Π΅Π½Π΅ΡΠ°ΡΠΈΠΈ Π±ΠΎΠ»ΡΡΠΈΡ ΠΎΠ±ΡΠ΅ΠΌΠΎΠ² ΡΠ»Π΅ΠΊΡΡΠΎΡΠ½Π΅ΡΠ³ΠΈΠΈ Π·Π° ΡΡΠ΅Ρ ΠΌΠ΅Ρ Π°Π½ΠΈΡΠ΅ΡΠΊΠΈΠ΅ ΡΡΠ΅Π΄ΡΡΠ²Π° Π² Π²ΠΈΠ΄Π΅ ΡΠ»Π΅ΠΊΡΡΠΈΡΠ΅ΡΠΊΠΈΠΉ Π³Π΅Π½Π΅ΡΠ°ΡΠΎΡ — ΠΊΠΎΡΠΎΡΡΠΉ Π·Π°ΡΠ΅ΠΌ ΡΠΎΠ²Π΅ΡΡΠΈΠ» Π²Π΅Π»ΠΈΠΊΡΡ ΡΠ΅Π²ΠΎΠ»ΡΡΠΈΡ Π² Π½Π°ΡΠ΅ΠΉ ΠΎΠ±ΡΠ°Π· ΠΆΠΈΠ·Π½ΠΈ Π² ΡΠΎΡΠΌΠ΅ Π½Π°ΡΠ΅Π³ΠΎ Π²Π΅ΠΊΠ° ΡΠ»Π΅ΠΊΡΡΠΈΡΠ΅ΡΡΠ²ΠΎ. ΠΠ°Π²Π°ΠΉΡΠ΅ ΡΠ·Π½Π°Π΅ΠΌ Π±ΠΎΠ»ΡΡΠ΅ ΠΎΠ± ΡΡΠΎΠΌ ΡΠ²Π»Π΅Π½ΠΈΠΈ, ΠΊΠΎΡΠΎΡΠΎΠ΅ ΠΎΠ½ ΠΎΡΠΊΡΡΠ». ΠΠ°Π²Π°ΠΉΡΠ΅ ΠΏΠΎΠ΄ΠΊΠ»ΡΡΠΈΠΌ ΠΏΡΠΎΠ²ΠΎΠ΄ΡΡΠΈΠΉ ΡΡΠ΅ΡΠΆΠ΅Π½Ρ C ΠΊ ΡΡΠ²ΡΡΠ²ΠΈΡΠ΅Π»ΡΠ½ΠΎΠΌΡ Π³Π°Π»ΡΠ²Π°Π½ΠΎΠΌΠ΅ΡΡΡ, ΠΊΠ°ΠΊ ΠΏΠΎΠΊΠ°Π·Π°Π½ΠΎ Π½Π° ΡΠΈΡ. 1, ΠΈ ΠΏΡΠΎΠΏΡΡΡΠΈΡΠ΅ ΡΡΠ΅ΡΠΆΠ΅Π½Ρ ΠΌΠ΅ΠΆΠ΄Ρ ΠΏΠΎΠ»ΡΡΠ° ΠΏΠΎΠ΄ΠΊΠΎΠ²ΠΎΠΎΠ±ΡΠ°Π·Π½ΠΎΠ³ΠΎ ΠΌΠ°Π³Π½ΠΈΡΠ°. ΠΠΎΠ³Π΄Π° ΠΌΡ Π΄Π΅Π»Π°Π΅ΠΌ ΡΡΠΎ, ΠΏΡΠΎΠΈΡΡ ΠΎΠ΄ΠΈΡ ΠΎΡΠΊΠ»ΠΎΠ½Π΅Π½ΠΈΠ΅ ΡΡΡΠ΅Π»ΠΊΠΈ Π³Π°Π»ΡΠ²Π°Π½ΠΎΠΌΠ΅ΡΡΠ°, Ρ ΡΠΊΠ°Π·Π°Π½ΠΈΠ΅ΠΌ ΡΠΎΠΊΠ°. ΠΠ°ΠΊΠΎΠ΅ Π·Π°ΠΌΠ΅ΡΠ°ΡΠ΅Π»ΡΠ½ΠΎΠ΅ ΡΠ²Π»Π΅Π½ΠΈΠ΅! ΠΡΠΎ Π±Ρ ΠΎΠΆΠΈΠ΄Π°Π» ΡΠ°ΠΊΠΎΠ³ΠΎ?! ΠΠΎΠ³Π΄Π° ΡΡΠ΅ΡΠΆΠ΅Π½Ρ ΡΠ΄Π΅ΡΠΆΠΈΠ²Π°Π΅ΡΡΡ Π½Π΅ΠΏΠΎΠ΄Π²ΠΈΠΆΠ½ΠΎ Π² ΠΏΠΎΠ»Π΅, ΡΠΎΠΊ Π½Π΅ ΡΠ΅ΡΠ΅Ρ. Π’Π΅ΠΊΡΡΠΈΠΉ ΡΠ΅ΡΠ΅Ρ ΡΠΎΠ»ΡΠΊΠΎ ΡΠΎΠ³Π΄Π°, ΠΊΠΎΠ³Π΄Π° ΡΡΠ΅ΡΠΆΠ΅Π½Ρ Π΄Π²ΠΈΠΆΠ΅ΡΡΡ Π²Π½ΡΡΡΠΈ ΠΌΠ°Π³Π½ΠΈΡΠ½ΠΎΠ³ΠΎ ΠΏΠΎΠ»Π΅. ΠΠΎΠ³Π΄Π° ΡΡΠ΅ΡΠΆΠ΅Π½Ρ Π΄Π²ΠΈΠΆΠ΅ΡΡΡ Π²Π²Π΅ΡΡ Π² ΠΏΡΠ΅Π΄Π΅Π»Π°Ρ ΠΏΠΎΠ»Ρ, ΡΠΎΠΊ ΡΠ΅ΡΠ΅Ρ Π² Π½Π°ΠΏΡΠ°Π²Π»Π΅Π½ΠΈΠΈ, ΠΏΡΠΎΡΠΈΠ²ΠΎΠΏΠΎΠ»ΠΎΠΆΠ½ΠΎΠΌ Π΅Π³ΠΎ ΡΠ΅ΡΠ΅Ρ ΠΏΡΠΈ Π΄Π²ΠΈΠΆΠ΅Π½ΠΈΠΈ ΡΡΠ΅ΡΠΆΠ½Ρ Π²Π½ΠΈΠ·. ΠΡΠΎΠΌΠ΅ ΡΠΎΠ³ΠΎ, ΠΌΡ ΠΎΠ±Π½Π°ΡΡΠΆΠΈΡΡ, ΡΡΠΎ ΡΠ΅ΠΌ Π±ΡΡΡΡΠ΅Π΅ ΡΡΠ΅ΡΠΆΠ΅Π½Ρ ΠΏΡΠΎΡ ΠΎΠ΄ΠΈΡ ΡΠ΅ΡΠ΅Π· ΠΏΠΎΠ»Π΅, ΡΠ΅ΠΌ Π±ΠΎΠ»ΡΡΠ΅ ΠΎΡΠΊΠ»ΠΎΠ½Π΅Π½ΠΈΠ΅ ΠΈΠ³Π»Ρ. Π’Π°ΠΊΠΈΠΌ ΠΎΠ±ΡΠ°Π·ΠΎΠΌ Π±ΡΡΡΡΠΎΠ΅ ΠΏΠ΅ΡΠ΅ΠΌΠ΅ΡΠ΅Π½ΠΈΠ΅ ΡΠ΄ΠΈΠ»ΠΈΡΠ° ΠΏΠΎ ΠΏΠΎΠ»Ρ Π΄Π°Π΅Ρ Π±ΠΎΠ»ΡΡΠΈΠΉ ΡΠΎΠΊ. ΠΠ΅ΡΠ΅ΠΌΠ΅ΡΡΠΈΠΌ ΡΡΠ΅ΡΠΆΠ΅Π½Ρ ΠΌΠ΅ΠΆΠ΄Ρ ΠΏΠΎΠ»ΡΡΠ°ΠΌΠΈ ΠΏΠ°ΡΠ°Π»Π»Π΅Π»ΡΠ½ΠΎ ΡΠΈΠ»ΠΎΠ²ΡΠΌ Π»ΠΈΠ½ΠΈΡΠΌ. ΠΠ΅Ρ ΡΠΎΠΊ ΡΠ΅ΡΠ΅Ρ, ΠΊΠΎΠ³Π΄Π° ΠΌΡ Π΄Π΅Π»Π°Π΅ΠΌ ΡΡΠΎ. Π’ΠΎΠΊ ΡΠ΅ΡΠ΅Ρ ΡΠΎΠ»ΡΠΊΠΎ ΡΠΎΠ³Π΄Π°, ΠΊΠΎΠ³Π΄Π° ΠΌΡ ΠΏΠ΅ΡΠ΅ΡΠ΅ΠΊΠ°Π΅ΠΌ Π»ΠΈΠ½ΠΈΠΈ ΠΏΠΎΡΠΎΠΊΠ°. ΠΠΎΠ·Π²ΠΎΠ»Ρ Π½Π°ΠΌ ΡΠ°ΡΡΠΌΠΎΡΡΠ΅ΡΡ Π΄ΡΡΠ³ΠΎΠΉ ΡΠΊΡΠΏΠ΅ΡΠΈΠΌΠ΅Π½Ρ. ΠΠΎΠ΄ΠΊΠ»ΡΡΠΈΠΌ Π³Π°Π»ΡΠ²Π°Π½ΠΎΠΌΠ΅ΡΡ ΠΊ ΠΊΠ°ΡΡΡΠΊΠ΅ Ρ ΠΈΠ·ΠΎΠ»ΠΈΡΠΎΠ²Π°Π½Π½ΡΠΌ ΠΏΡΠΎΠ²ΠΎΠ΄ΠΎΠΌ ΡΠ°ΠΊ, ΠΊΠ°ΠΊ ΠΏΠΎΠΊΠ°Π·Π°Π½ΠΎ Π½Π° ΡΠΈΡ. 2, ΠΈ Π²ΡΡΠ°Π²ΡΡΠ΅ ΡΡΠ΅ΡΠΆΠ½Π΅Π²ΠΎΠΉ ΠΌΠ°Π³Π½ΠΈΡ Π²Π½ΡΡΡΡ ΠΎΡΠ²Π΅ΡΡΡΠΈΡ Π² ΠΊΠ°ΡΡΡΠΊΠ΅. ΠΠΏΡΡΡ ΠΆΠ΅, ΡΡΡΠ΅Π»ΠΊΠ° Π³Π°Π»ΡΠ²Π°Π½ΠΎΠΌΠ΅ΡΡΠ° ΠΎΡΠΊΠ»ΠΎΠ½ΡΠ΅ΡΡΡ, ΡΠΊΠ°Π·ΡΠ²Π°Ρ Π½Π° Π½Π°Π»ΠΈΡΠΈΠ΅ ΡΠΎΠΊΠ°. ΠΡΠΈ ΠΈΠ·Π²Π»Π΅ΡΠ΅Π½ΠΈΠΈ ΠΌΠ°Π³Π½ΠΈΡΠ° Π³Π°Π»ΡΠ²Π°Π½ΠΎΠΌΠ΅ΡΡ ΠΏΠΎΠΊΠ°Π·ΡΠ²Π°Π΅Ρ ΡΠΎΠΊ Π² ΠΎΠ±ΡΠ°ΡΠ½ΠΎΠΌ Π½Π°ΠΏΡΠ°Π²Π»Π΅Π½ΠΈΠΈ. Π§Π΅ΠΌ Π±ΡΡΡΡΠ΅Π΅ ΠΎΠ½ ΠΏΠΎΠ³ΡΡΠΆΠ°Π΅ΡΡΡ, ΡΠ΅ΠΌ ΡΠΈΠ»ΡΠ½Π΅Π΅ ΠΏΡΠΎΠΈΠ·Π²ΠΎΠ΄ΠΈΠΌΡΠΉ ΡΠΎΠΊ. ΠΠΎΠ³Π΄Π° ΡΠΈΠ»ΠΎΠ²ΡΠ΅ Π»ΠΈΠ½ΠΈΠΈ ΠΌΠ°Π³Π½ΠΈΡΠ° ΠΏΠ΅ΡΠ΅ΡΠ΅ΠΊΠ°ΡΡ ΠΏΡΠΎΠ²ΠΎΠ»ΠΎΠΊΡ Π² ΠΊΠ°ΡΡΡΠΊΠ°Ρ , ΡΠΎΠΊ Π²ΡΡΠ°Π±Π°ΡΡΠ²Π°Π΅ΡΡΡ.
Β
ΠΠΎΠ³Π΄Π° ΠΏΡΠΎΠ²ΠΎΠ΄Π½ΠΈΠΊ ΠΏΠ΅ΡΠ΅ΡΠ΅ΠΊΠ°Π΅Ρ Π»ΠΈΠ½ΠΈΠΈ ΠΌΠ°Π³Π½ΠΈΡΠ½ΠΎΠ³ΠΎ ΠΏΠΎΡΠΎΠΊΠ° ΠΈΠ»ΠΈ ΠΊΠΎΠ³Π΄Π° ΠΏΠΎΠ»Π΅ ΠΌΠ°Π³Π½ΠΈΡΠ½ΠΎΠ³ΠΎ ΠΏΠΎΡΠΎΠΊΠ° ΠΈΠ·ΠΌΠ΅Π½ΡΠ΅ΡΡΡ ΠΏΠΎ ΡΠΈΠ»Π΅ Π²ΠΎΠΊΡΡΠ³ ΠΏΡΠΎΠ²ΠΎΠ΄Π½ΠΈΠΊΠ° Π²ΠΎΠ·Π½ΠΈΠΊΠ°Π΅Ρ (ΠΈΠ½Π΄ΡΡΠΈΡΡΠ΅ΡΡΡ) ΠΠΠ‘ Π² ΠΏΡΠΎΠ²ΠΎΠ΄Π½ΠΈΠΊ. ΠΡΠ° ΠΠΠ‘ Π½Π°Π·ΡΠ²Π°Π΅ΡΡΡ ΠΠΠ‘ ΠΈΠ½Π΄ΡΠΊΡΠΈΠΈ. ΠΡΠ»ΠΈ ΠΏΡΠΎΠ²ΠΎΠ΄Π½ΠΈΠΊ ΡΠ²Π»ΡΠ΅ΡΡΡ ΡΠ°ΡΡΡΡ ΡΠ΅ΠΏΠΈ, ΠΊΠ°ΠΊ Π² ΠΏΡΠΈΠ²Π΅Π΄Π΅Π½Π½ΠΎΠΌ Π²ΡΡΠ΅ ΡΠ»ΡΡΠ°ΡΡ , ΠΊΠΎΠ³Π΄Π° ΠΎΠ½ ΠΏΠΎΠ΄ΠΊΠ»ΡΡΠ΅Π½ ΠΊ Π³Π°Π»ΡΠ²Π°Π½ΠΎΠΌΠ΅ΡΡΡ, ΡΡΠ° ΠΠΠ‘ ΠΏΡΠΎΠΈΠ·Π²ΠΎΠ΄ΠΈΡ ΡΠΎΠΊ. Π’ΠΎΠΊ Π½Π°Π·ΡΠ²Π°Π΅ΡΡΡ ΠΈΠ½Π΄ΡΡΠΈΡΠΎΠ²Π°Π½Π½ΡΠΌ Π’Π΅ΠΊΡΡΠΈΠΉ. Π―Π²Π»Π΅Π½ΠΈΠ΅, ΠΎ ΠΊΠΎΡΠΎΡΠΎΠΌ ΠΌΡ Π³ΠΎΠ²ΠΎΡΠΈΠΌ, Π½Π°Π·ΡΠ²Π°Π΅ΡΡΡ ΡΠ»Π΅ΠΊΡΡΠΎΠΌΠ°Π³Π½ΠΈΡΠ½ΠΎΠΉ ΠΈΠ½Π΄ΡΠΊΡΠΈΠ΅ΠΉ.
ΠΠ°Π²Π°ΠΉΡΠ΅ ΡΠ°ΡΡΠΌΠΎΡΡΠΈΠΌ Π΅ΡΠ΅ ΠΎΠ΄ΠΈΠ½ ΡΠΊΡΠΏΠ΅ΡΠΈΠΌΠ΅Π½Ρ. ΠΠ° ΡΠΈΡ. 3 Π±ΠΎΠ»ΡΡΠΎΠΉ Π΄Π΅ΡΠ΅Π²ΡΠ½Π½Π°Ρ ΡΠΏΡΠ»Ρ, Π½Π°ΠΌΠΎΡΠ°Π½Π½Π°Ρ Ρ Π±ΠΎΠ»ΡΡΠΈΠΌ ΠΊΠΎΠ»ΠΈΡΠ΅ΡΡΠ²ΠΎΠΌ Π²ΠΈΡΠΊΠΎΠ² ΡΠΎΠ½ΠΊΠΈΠΉ ΠΈΠ·ΠΎΠ»ΠΈΡΠΎΠ²Π°Π½Π½ΡΠΉ ΠΏΡΠΎΠ²ΠΎΠ΄ ΠΏΡΠΈΡΠΎΠ΅Π΄ΠΈΠ½ΡΡΡ ΠΊ Π³Π°Π»ΡΠ²Π°Π½ΠΎΠΌΠ΅ΡΡΡ. Π Π½Π΅Π±ΠΎΠ»ΡΡΠ°Ρ ΠΊΠ°ΡΡΡΠΊΠ°, Π½Π°ΠΌΠΎΡΠ°Π½Π½Π°Ρ Π½Π° Π½Π΅ΡΠΊΠΎΠ»ΡΠΊΠΎ Π²ΠΈΡΠΊΠΎΠ² ΠΈΠ·ΠΎΠ»ΠΈΡΠΎΠ²Π°Π½Π½ΠΎΠ³ΠΎ ΠΏΡΠΎΠ²ΠΎΠ΄Π° ΡΠΎΠ΅Π΄ΠΈΠ½Π΅Π½ ΠΏΠΎΡΠ»Π΅Π΄ΠΎΠ²Π°ΡΠ΅Π»ΡΠ½ΠΎ Ρ ΡΡΡ ΠΈΠΌ ΡΠ»Π΅ΠΌΠ΅Π½ΡΠΎΠΌ ΠΈ ΠΊΠΎΠ½ΡΠ°ΠΊΡΠ½ΡΠΌ ΠΊΠ»ΡΡΠΎΠΌ. ΠΠΎΠΌΠ΅ΡΡΠΈΠΌ ΠΌΠ°Π»Π΅Π½ΡΠΊΡΡ ΠΊΠ°ΡΡΡΠΊΡ Π²Π½ΡΡΡΡ Π±ΠΎΠ»ΡΡΠΎΠΉ ΠΊΠ°ΡΡΡΠΊΠΈ. ΠΈ Π½Π°ΠΆΠΌΠΈΡΠ΅ ΠΊΠΎΠ½ΡΠ°ΠΊΡΠ½ΡΡ ΠΊΠ»Π°Π²ΠΈΡΡ, Π·Π°ΠΌΡΠΊΠ°Ρ ΡΠ΅ΠΏΡ. Π‘ΡΡΠ΅Π»ΠΊΠ° Π³Π°Π»ΡΠ²Π°Π½ΠΎΠΌΠ΅ΡΡΠ° ΠΎΡΠΊΠ»ΠΎΠ½ΠΈΡΡΡ, ΠΏΠΎΠΊΠ°Π·ΡΠ²Π°Ρ, ΡΡΠΎ Π² Π±ΠΎΠ»ΡΡΠΎΠΉ ΠΊΠ°ΡΡΡΠΊΠ΅ ΠΈΠ½Π΄ΡΡΠΈΡΡΠ΅ΡΡΡ ΡΠΎΠΊ. Π§ΡΠΎ ΠΏΡΠΎΠΈΠ·ΠΎΡΠ»ΠΎ? ΠΠΎΠ³Π΄Π° ΠΌΡ Π·Π°ΠΊΡΡΠ»ΠΈΡΡ ΠΏΠ΅ΡΠ΅ΠΊΠ»ΡΡΠ°ΡΠ΅Π»Ρ ΠΈ ΡΠΎΠΊ Π½Π°ΡΠ°Π» ΡΠ΅ΡΡ Π² ΠΌΠ°Π»Π΅Π½ΡΠΊΠΎΠΉ ΠΊΠ°ΡΡΡΠΊΠ΅, ΡΡΠΎΡ ΡΠΎΠΊ Π²ΡΠ·Π²Π°Π» ΠΌΠ°Π³Π½ΠΈΡΠ½ΠΎΠ΅ ΠΏΠΎΠ»Π΅ ΡΠ°Π·Π²ΠΈΠ²Π°ΡΡΡΡ Π²ΠΎΠΊΡΡΠ³ Π½Π΅Π³ΠΎ, ΠΈ ΡΡΠΎ ΡΠ°Π·Π²ΠΈΠ²Π°ΡΡΠ΅Π΅ΡΡ ΠΌΠ°Π³Π½ΠΈΡΠ½ΠΎΠ΅ ΠΏΠΎΠ»Π΅ ΠΈΠ½Π΄ΡΡΠΈΡΡΠ΅Ρ ΠΠΠ‘ ΠΈ ΡΠΎΠΊ Π² Π±ΠΎΠ»ΡΡΠΎΠΉ ΠΊΠ°ΡΡΡΠΊΠ΅. ΠΡΠ»ΠΈ ΠΊΠ»ΡΡ Π·Π°ΠΊΡΡΡΡΠΌ, ΠΈΠ½Π΄ΡΠΊΡΠΈΠΎΠ½Π½ΡΠΉ ΡΠΎΠΊ Π² Π±ΠΎΠ»ΡΡΠΎΠΉ ΠΊΠ°ΡΡΡΠΊΠ΅ Π²ΡΠΊΠΎΡΠ΅ ΠΏΡΠ΅ΠΊΡΠ°ΡΠ°Π΅ΡΡΡ. ΠΡΠΈ ΡΠ°Π·ΡΡΠ²Π΅ ΡΠ΅ΠΏΠΈ ΡΠΈΠ»Π° ΠΌΠ°Π³Π½ΠΈΡΠ½ΠΎΠ³ΠΎ ΠΏΠΎΠ»Ρ Π±ΡΡΡΡΠΎ ΠΏΠ°Π΄Π°Π΅Ρ Π΄ΠΎ Π½ΡΠ»Ρ ΠΈ Π² ΠΏΡΠΎΠ²ΠΎΠ΄Π½ΠΈΠΊΠ΅ ΡΠ΅ΡΠ΅Ρ ΠΈΠ½Π΄ΡΡΠΈΡΠΎΠ²Π°Π½Π½ΡΠΉ ΡΠΎΠΊ. ΠΏΡΠΎΠΈΡΡ ΠΎΠ΄ΠΈΡ ΠΏΡΠΎΡΠΈΠ²ΠΎΠΏΠΎΠ»ΠΎΠΆΠ½ΠΎΠ΅ Π½Π°ΠΏΡΠ°Π²Π»Π΅Π½ΠΈΠ΅. Π ΠΎΠ±ΠΎΠΈΡ ΡΠ»ΡΡΠ°ΡΡ ΠΈΠ½Π΄ΡΠΊΡΠΈΠΎΠ½Π½ΡΠΉ ΡΠΎΠΊ ΠΏΠ΅ΡΠ΅ΡΡΠ°Π½Π΅Ρ ΡΠ΅ΡΡ, ΠΊΠΎΠ³Π΄Π° ΠΌΠ°Π³Π½ΠΈΡΠ½ΠΎΠ΅ ΠΏΠΎΠ»Π΅ ΠΏΠ΅ΡΠ΅ΡΡΠ°Π½Π΅Ρ ΠΈΠ·ΠΌΠ΅Π½ΡΡΡΡΡ.
Π’ΠΎ, ΡΡΠΎ ΠΌΡ ΡΠΎΠ»ΡΠΊΠΎ ΡΡΠΎ ΠΎΠΏΠΈΡΠ°Π»ΠΈ, ΠΏΡΠ΅Π΄ΡΡΠ°Π²Π»ΡΠ΅Ρ ΡΠΎΠ±ΠΎΠΉ ΡΠΏΠΎΡΠΎΠ± ΠΈΡΠΏΠΎΠ»ΡΠ·ΠΎΠ²Π°Π½ΠΈΡ ΠΎΠ΄Π½ΠΎΠ³ΠΎ ΡΠΎΠΊ Π΄Π»Ρ ΠΏΡΠΎΠΈΠ·Π²ΠΎΠ΄ΡΡΠ²Π° Π΄ΡΡΠ³ΠΎΠ³ΠΎ. ΠΠ°Π»Π΅Π½ΡΠΊΠ°Ρ ΠΊΠ°ΡΡΡΠΊΠ°, ΠΊΠΎΡΠΎΡΠ°Ρ ΠΏΠΎΠ΄ΠΊΠ»ΡΡΠ΅Π½Π½ΡΠΉ ΠΊ Π²Π½Π΅ΡΠ½Π΅ΠΌΡ ΠΈΡΡΠΎΡΠ½ΠΈΠΊΡ ΠΏΠΈΡΠ°Π½ΠΈΡ, Π½Π°Π·ΡΠ²Π°Π΅ΡΡΡ ΠΎΡΠ½ΠΎΠ²Π½ΡΠΌ ΠΊΠ°ΡΡΡΠΊΠ°. ΠΠΎΠ»ΡΡΠ°Ρ ΠΊΠ°ΡΡΡΠΊΠ°, Π² ΠΊΠΎΡΠΎΡΠΎΠΉ Π²ΠΎΠ·Π½ΠΈΠΊΠ°Π΅Ρ ΠΈΠ½Π΄ΡΠΊΡΠΈΠΎΠ½Π½ΡΠΉ ΡΠΎΠΊ Π½Π°Π·ΡΠ²Π°Π΅ΡΡΡ Π²ΡΠΎΡΠΈΡΠ½ΠΎΠΉ ΠΊΠ°ΡΡΡΠΊΠΎΠΉ.
ΠΠΎΡΠ΅ΠΌΡ Π² Π΄Π²ΠΈΠΆΡΡΠ΅ΠΌΡΡ ΠΏΡΠΎΠ²ΠΎΠ΄Π½ΠΈΠΊΠ΅ Π²ΠΎΠ·Π½ΠΈΠΊΠ°Π΅Ρ ΠΠΠ‘ Π² ΠΌΠ°Π³Π½ΠΈΡΠ½ΠΎΠΌ ΠΏΠΎΠ»Π΅? ΠΠ°ΠΊΠΎΠ²Π° ΠΏΡΠΈΡΠΈΠ½Π° ΡΡΠΎΠ³ΠΎ ΡΠ²Π»Π΅Π½ΠΈΠ΅, ΠΊΠΎΡΠΎΡΠΎΠ΅ ΠΌΡ ΡΠΎΠ»ΡΠΊΠΎ ΡΡΠΎ Π½Π°Π±Π»ΡΠ΄Π°Π»ΠΈ? ΠΠΎΡΠ΅ΠΌΡ ΠΠΠ‘ Π²ΠΎΠ·Π½ΠΈΠΊΠ°Π΅Ρ Π² ΠΏΡΠΎΠ²ΠΎΠ΄Π½ΠΈΠΊΠ΅, ΠΊΠΎΠ³Π΄Π° ΠΎΠ½ ΠΏΠ΅ΡΠ΅ΡΠ΅ΠΊΠ°Π΅Ρ ΡΠΈΠ»ΠΎΠ²ΡΠ΅ Π»ΠΈΠ½ΠΈΠΈ Π² ΠΌΠ°Π³Π½ΠΈΡΠ½ΠΎΠΌ ΠΏΠΎΠ»Π΅? ΠΡΠ²Π΅Ρ Π½Π° ΡΡΠΎ ΠΠΎΠΏΡΠΎΡ Π²ΠΎΡΡ ΠΎΠ΄ΠΈΡ ΠΊ Π΄ΡΡΠ³ΠΎΠΌΡ ΠΌΠ°Π³Π½ΠΈΡΠ½ΠΎΠΌΡ ΡΠ²Π»Π΅Π½ΠΈΡ, ΠΊΠΎΡΠΎΡΠΎΠ΅ ΠΌΡ ΡΠΆΠ΅ ΠΎΠ±ΡΡΠΆΠ΄Π°Π»ΠΈ, Π° ΠΈΠΌΠ΅Π½Π½ΠΎ: ΠΠΎΠ³Π΄Π° Π΄Π²ΠΈΠΆΡΡΠΈΠΉΡΡ Π·Π°ΡΡΠ΄ ΠΏΠ΅ΡΠ΅ΡΠ΅ΠΊΠ°Π΅Ρ ΡΠΈΠ»ΠΎΠ²ΡΠ΅ Π»ΠΈΠ½ΠΈΠΈ ΠΌΠ°Π³Π½ΠΈΡΠ½ΠΎΠ³ΠΎ ΠΏΠΎΠ»Ρ, Π½Π° Π½Π΅Π³ΠΎ Π΄Π΅ΠΉΡΡΠ²ΡΠ΅Ρ ΡΠΈΠ»Π° ΠΎΠΏΡΠ΅Π΄Π΅Π»ΡΠ΅ΡΡΡ Π²ΡΡΠ°ΠΆΠ΅Π½ΠΈΠ΅ΠΌ F = qvΓB. Π§ΡΠΎΠ±Ρ ΡΠ²ΠΈΠ΄Π΅ΡΡ, ΡΡΠΎ ΠΏΡΠΎΠΈΡΡ ΠΎΠ΄ΠΈΡ, ΡΠ°ΡΡΠΌΠΎΡΡΠΈΠΌ ΡΠΈΡ. 4, Π³Π΄Π΅ ΠΊΡΠ΅ΡΡΡ ΠΏΡΠ΅Π΄ΡΡΠ°Π²Π»ΡΡΡ ΠΏΠΎΠ»Π΅ ΠΏΠΎΡΠΎΠΊΠ° B Π½Π°ΠΏΡΠ°Π²Π»Π΅Π½ ΠΎΡ ΡΠΈΡΠ°ΡΠ΅Π»Ρ. ΠΠΎΠ³Π΄Π° ΠΏΡΠΎΠ²ΠΎΠ΄ΡΡΠΈΠΉ ΡΡΠ΅ΡΠΆΠ΅Π½Ρ ab Π΄Π²ΠΈΠΆΠ΅ΡΡΡ Π²ΠΏΡΠ°Π²ΠΎ ΡΠΎ ΡΠΊΠΎΡΠΎΡΡΡΡ v, ΡΠΎΠ³Π΄Π° ΠΊΠ°ΠΆΠ΄ΡΠΉ Π·Π°ΡΡΠ΄ Π²Π½ΡΡΡΠΈ ΡΡΠ΅ΡΠΆΠ½Ρ Π΄Π²ΠΈΠΆΠ΅ΡΡΡ Π²ΠΏΡΠ°Π²ΠΎ, ΠΏΠ΅ΡΠ΅ΡΠ΅ΠΊΠ°Ρ Π»ΠΈΠ½ΠΈΠΈ ΠΏΠΎΡΠΎΠΊΠ°, ΡΠΎ ΡΠΊΠΎΡΠΎΡΡΡΡ v. Π‘Π»Π΅Π΄ΡΡΠ²ΠΈΠ΅ΠΌ ΡΡΠΎΠ³ΠΎ ΡΠ²Π»ΡΠ΅ΡΡΡ ΡΠΎ, ΡΡΠΎ ΡΠΈΠ»Π° F = qv Γ B Π΄Π΅ΠΉΡΡΠ²ΡΠ΅Ρ Π½Π° ΠΊΠ°ΠΆΠ΄ΡΠΉ Π·Π°ΡΡΠ΄ Π²Π½ΡΡΡΠΈ ΡΡΠ΅ΡΠΆΠ½Ρ — Π³Π΄Π΅ Π½Π°ΠΏΡΠ°Π²Π»Π΅Π½ΠΈΠ΅ Π²Π΅ΠΊΡΠΎΡΠ° F β Π²Π΄ΠΎΠ»Ρ ΡΡΠ΅ΡΠΆΠ½Ρ, Π½Π°ΠΏΡΠ°Π²Π»Π΅Π½Π½ΠΎΠ³ΠΎ ΠΎΡ b ΠΊ a. ΠΡΠ° ΡΠΈΠ»Π° ΡΠΎΡΡΠ°Π²Π»ΡΠ΅Ρ ΠΠΠ‘ Π²Π½ΡΡΡΠΈ ΡΡΠ΅ΡΠΆΠ½Ρ, ΡΡΡΠ΅ΠΌΡΡΠ°ΡΡΡ ΡΠΎΠ·Π΄Π°ΡΡ ΡΠΎΠΊ ΠΎΡ b ΠΊ a. ΠΡΠ»ΠΈ ΡΠΈΠ½Π° ΡΠ²Π»ΡΠ΅ΡΡΡ ΡΠ°ΡΡΡΡ ΡΠ΅ΠΏΠΈ ΠΈΠ»ΠΈ ΠΏΠΎΠ΄ΠΊΠ»ΡΡΠ΅Π½ ΠΊ Π³Π°Π»ΡΠ²Π°Π½ΠΎΠΌΠ΅ΡΡΡ, ΠΊΠ°ΠΊ Π½Π° ΡΠΈΡ. 1 Π²ΡΡΠ΅, ΡΡΠ° ΠΠΠ‘ Π²ΡΠ·ΠΎΠ²Π΅Ρ ΠΏΡΠΎΡΠ΅ΠΊΠ°Π½ΠΈΠ΅ ΡΠΎΠΊΠ°. ΠΡΠ»ΠΈ Π±Π°Ρ Π½Π΅ ΡΠ²Π»ΡΠ΅ΡΡΡ ΡΠ°ΡΡΡΡ ΡΠ΅ΠΏΠΈ, ΡΠΎ ΠΏΡΠΎΠΈΠ·ΠΎΠΉΠ΄Π΅Ρ ΡΠΎ, ΡΡΠΎ Π²ΡΠ΅ ΡΠ²ΠΎΠ±ΠΎΠ΄Π½ΡΠ΅ ΡΠ»Π΅ΠΊΡΡΠΎΠ½Ρ Π² ΡΡΠ΅ΡΠΆΠ½Π΅ Π±ΡΠ΄ΡΡ Π΄Π²ΠΈΠ³Π°ΡΡΡΡ ΠΊ ΠΊΠΎΠ½ΡΡ b, Π΄Π΅Π»Π°Ρ ΠΊΠΎΠ½Π΅Ρ b ΠΎΡΡΠΈΡΠ°ΡΠ΅Π»ΡΠ½ΡΠΌ, Π° ΠΊΠΎΠ½Π΅Ρ ΠΏΠΎΠ»ΠΎΠΆΠΈΡΠ΅Π»ΡΠ½ΡΠΌ.
ΠΠΠ‘, Π½Π°Π²ΠΎΠ΄ΠΈΠΌΠ°Ρ Π² Π΄Π²ΠΈΠΆΡΡΠ΅ΠΌΡΡ ΠΏΡΠΎΠ²ΠΎΠ΄Π½ΠΈΠΊΠ΅. ΠΠΠ‘ ΠΈΠ½Π΄ΡΠΊΡΠΈΠΈ Π² ΠΏΡΡΠΌΠΎΠΌ ΠΏΡΠΎΠ²ΠΎΠ΄Π½ΠΈΠΊΠ΅ Π΄Π»ΠΈΠ½ΠΎΠΉ l Π΄Π²ΠΈΠΆΠ΅ΡΡΡ ΡΠΎ ΡΠΊΠΎΡΠΎΡΡΡΡ v ΠΏΠ΅ΡΠΏΠ΅Π½Π΄ΠΈΠΊΡΠ»ΡΡΠ½ΠΎ ΠΌΠ°Π³Π½ΠΈΡΠ½ΠΎΠΌΡ ΠΏΠΎΠ»Ρ B ΡΠ°Π²Π½ΠΎ
Β
1)Β Β Β Β Β Β E = B l v
Π³Π΄Π΅ B, l ΠΈ v Π²Π·Π°ΠΈΠΌΠ½ΠΎ ΠΏΠ΅ΡΠΏΠ΅Π½Π΄ΠΈΠΊΡΠ»ΡΡΠ½Ρ. ΠΠΠ‘ Π²ΡΡΠ°ΠΆΠ°Π΅ΡΡΡ Π² Π²ΠΎΠ»ΡΡΠ°Ρ , ΠΊΠΎΠ³Π΄Π° B Π²ΡΡΠ°ΠΆΠ°Π΅ΡΡΡ Π² Π²Π΅Π±Π΅ΡΠ°Ρ /ΠΌ 2 , l ΠΌΠ΅ΡΡΠΎΠ², Π° v Π² ΠΌ/ΡΠ΅ΠΊ.
ΠΡΠ»ΠΈ Π²Π΅ΠΊΡΠΎΡ ΡΠΊΠΎΡΠΎΡΡΠΈ v ΡΠΎΡΡΠ°Π²Π»ΡΠ΅Ρ ΡΠ³ΠΎΠ» ΞΈ Ρ Π½Π°ΠΏΡΠ°Π²Π»Π΅Π½ΠΈΠ΅ΠΌ ΠΌΠ°Π³Π½ΠΈΡΠ½ΠΎΠ³ΠΎ ΠΏΠΎΠ»Ρ, 1) ΡΡΠ°Π½ΠΎΠ²ΠΈΡΡΡ
Β
2)Β Β Β Β Β Β E = B l v sin ΞΈ
________________________________________________________________________________
ΠΠΎΠΊΠ°Π·Π°ΡΡ. ΠΠΠ‘, ΠΈΠ½Π΄ΡΡΠΈΡΡΠ΅ΠΌΠ°Ρ Π² ΠΏΡΡΠΌΠΎΠΉ ΠΏΡΠΎΠ²ΠΎΠ΄ Π΄Π»ΠΈΠ½ΠΎΠΉ Π» Π΄Π²ΠΈΠΆΠ΅ΡΡΡ ΡΠΎ ΡΠΊΠΎΡΠΎΡΡΡΡ v ΠΏΠ΅ΡΠΏΠ΅Π½Π΄ΠΈΠΊΡΠ»ΡΡΠ½ΠΎ ΠΌΠ°Π³Π½ΠΈΡΠ½ΠΎΠΌΡ ΠΏΠΎΠ»Ρ B ΡΡΠΎ
Β
Π = Π Π» v
ΠΠΎΠΊΠ°Π·Π°ΡΠ΅Π»ΡΡΡΠ²ΠΎ. ΠΠΎ ΠΎΠΏΡΠ΅Π΄Π΅Π»Π΅Π½ΠΈΡ E = dW/dq. Π’ΠΎ Π΅ΡΡΡ ΠΠΠ‘ β ΡΡΠΎ ΡΠ°Π±ΠΎΡΠ°, ΡΠΎΠ²Π΅ΡΡΠ΅Π½Π½Π°Ρ Π½Π°Π΄ ΠΎΠ±ΠΎΡΠΎΡΠ½ΡΠΉ Π·Π°ΡΡΠ΄ Π½Π° Π΅Π΄ΠΈΠ½ΠΈΡΡ Π·Π°ΡΡΠ΄ (ΠΊΡΠ»ΠΎΠ½), ΡΠΌΠ΅ΡΠ΅Π½Π½ΡΠΉ Π·Π° ΡΠΎΡΠΊΠ° ΡΠ΅ΠΏΠΈ. Π Π°ΡΡΠΌΠΎΡΡΠΈΠΌ ΡΠΈΡ. 5, Π½Π° ΠΊΠΎΡΠΎΡΠΎΠΌ Π΄Π²ΠΈΠΆΡΡΠΈΠΉΡΡ ΠΏΡΠΎΠ²ΠΎΠ΄Π½ΠΈΠΊ ab Π΄Π»ΠΈΠ½Ρ Π» Π³ΠΎΡΠΊΠΈ Π²Π΄ΠΎΠ»Ρ Π½Π΅ΠΏΠΎΠ΄Π²ΠΈΠΆΠ½ΠΎΠ³ΠΎ U-ΠΎΠ±ΡΠ°Π·Π½ΠΎΠ³ΠΎ ΠΏΡΠΎΠ²ΠΎΠ΄Π½ΠΈΠΊΠ°, Π³Π΄Π΅ ΠΏΠ΅ΡΠ»Ρ Π½Π°Ρ ΠΎΠ΄ΠΈΡΡΡ Π² ΠΏΠ»ΠΎΡΠΊΠΎΡΡΠΈ, ΠΏΠ΅ΡΠΏΠ΅Π½Π΄ΠΈΠΊΡΠ»ΡΡΠ½ΠΎΠΉ ΠΌΠ°Π³Π½ΠΈΡΠ½ΠΎΠΌΡ ΠΏΠΎΠ»Π΅ B. ΠΡΠ»ΠΈ ΠΏΡΠΎΠ²ΠΎΠ΄Π½ΠΈΠΊ ab Π΄Π²ΠΈΠΆΠ΅ΡΡΡ Π²ΠΏΡΠ°Π²ΠΎ ΡΠΎ ΡΠΊΠΎΡΠΎΡΡΡΡ v, ΡΠΎ Π² ΠΊΠΎΠ½ΡΡΡΠ΅ adcb ΠΏΠΎΡΠ΅ΡΠ΅Ρ ΡΠΎΠΊ I. ΠΠΎΠΌΠ½Ρ, ΡΡΠΎ ΠΌΠ°Π³Π½ΠΈΡΠ½ΠΎΠ΅ ΠΏΠΎΠ»Π΅ Π΄Π΅ΠΉΡΡΠ²ΡΠ΅Ρ Ρ ΡΠΈΠ»ΠΎΠΉ F = Π» IB Π½Π° Π΄Π»ΠΈΠ½Π½ΡΡ ΠΏΡΡΠΌΡΡ ΠΏΡΠΎΠ²ΠΎΠ΄ΡΡΡΡ ΡΠΎΠΊ ΠΏΡΠΎΠ²ΠΎΠ΄Π½ΠΈΠΊΠ°, ΠΏΠ΅ΡΠΏΠ΅Π½Π΄ΠΈΠΊΡΠ»ΡΡΠ½ΠΎΠ³ΠΎ ΠΏΠΎΠ»Ρ, Π·Π°ΠΌΠ΅ΡΠΈΠΌ, ΡΡΠΎ ΡΠΎΠΊ I, Π΄Π²ΠΈΠΆΡΡΠΈΠΉΡΡ ΠΏΠΎ Π΄Π²ΠΈΠΆΡΡΠ΅ΠΉΡΡ ΠΏΡΠΎΠ²ΠΎΠ΄Π½ΠΈΠΊ ab Π²ΡΠ·ΠΎΠ²Π΅Ρ Π±ΠΎΠΊΠΎΠ²ΡΡ ΡΡΠ³Ρ Π²Π»Π΅Π²ΠΎ Π½Π° ab
Β
F = Π» IB
ΠΠ·-Π·Π° ΡΡΠΎΠΉ Π±ΠΎΠΊΠΎΠ²ΠΎΠΉ ΡΡΠ³ΠΈ ΡΡΠ΅Π±ΡΠ΅ΡΡΡ Π²Π½Π΅ΡΠ½ΡΡ ΡΠΈΠ»Π°, ΠΎΠ±Π΅ΡΠΏΠ΅ΡΠΈΠ²Π°Π΅ΠΌΠ°Ρ Π½Π΅ΠΊΠΎΡΠΎΡΡΠΌ ΡΠ°Π±ΠΎΡΠΈΠΌ Π°Π³Π΅Π½ΡΠΎΠΌ, ΡΡΠΎΠ±Ρ ΠΏΠΎΠ΄Π΄Π΅ΡΠΆΠΈΠ²Π°ΡΡ Π΄Π²ΠΈΠΆΠ΅Π½ΠΈΠ΅. Π Π°Π±ΠΎΡΠ°, ΡΠΎΠ²Π΅ΡΡΠ°Π΅ΠΌΠ°Ρ ΡΡΠΈΠΌ Π°Π³Π΅Π½ΡΠΎΠΌ, Π΅ΡΡΡ ΡΠ°Π±ΠΎΡΠ°, ΡΠΎΠ²Π΅ΡΡΠ°Π΅ΠΌΠ°Ρ Π½Π°Π΄ ΡΠΈΡΠΊΡΠ»ΠΈΡΡΡΡΠΈΠΌ Π·Π°ΡΡΠ΄ΠΎΠΌ. ΠΠ΄Π΅ΡΡ ΠΏΡΠΎΠΈΡΡ ΠΎΠ΄ΠΈΡ ΠΏΡΡΠΌΠΎΠ΅ ΠΏΡΠ΅ΠΎΠ±ΡΠ°Π·ΠΎΠ²Π°Π½ΠΈΠ΅ ΠΌΠ΅Ρ Π°Π½ΠΈΡΠ΅ΡΠΊΠΎΠΉ ΡΠ½Π΅ΡΠ³ΠΈΠΈ Π² ΡΠ»Π΅ΠΊΡΡΠΈΡΠ΅ΡΠΊΡΡ.
Π Π°ΡΡΡΠΎΡΠ½ΠΈΠ΅, ΠΏΡΠΎΠΉΠ΄Π΅Π½Π½ΠΎΠ΅ Π·Π° Π²ΡΠ΅ΠΌΡ t, ΡΠ°Π²Π½ΠΎ
.Β Β Β Β Β Β Β Β Β Β Β Β ds = vdt
ΠΈ ΠΏΡΠΎΠ΄Π΅Π»Π°Π½Π½Π°Ρ ΡΠ°Π±ΠΎΡΠ° ΡΠ°Π²Π½Π°
Β Β Β Β Β Β Β Β Β Β Β Β dW = Fds = Π» IBβvdt
ΠΡΠΎΠΈΠ·Π²Π΅Π΄Π΅Π½ΠΈΠ΅ I ΠΈ dt ΡΠ°Π²Π½ΠΎ ΡΠΌΠ΅ΡΠ΅Π½Π½ΠΎΠΌΡ Π·Π° ΡΡΠΎ Π²ΡΠ΅ΠΌΡ Π·Π°ΡΡΠ΄Ρ dq, ΠΏΠΎΡΡΠΎΠΌΡ
Β
Π΄Π = Π±ΡΠ»ΡΠ²Π°Ρ
ΠΈΠ»ΠΈ
Β
dW/dq = Π±ΡΠ»ΡΠ²Π°Ρ
ΠΠΎΡΠΊΠΎΠ»ΡΠΊΡ E = dW/dq,
Β
E = ΠΡΠ»ΡΠ²Π°Ρ
________________________________________________________________________________
ΠΠ°ΠΊΠΎΠ½ ΡΠ»Π΅ΠΊΡΡΠΎΠΌΠ°Π³Π½ΠΈΡΠ½ΠΎΠΉ ΠΈΠ½Π΄ΡΠΊΡΠΈΠΈ Π€Π°ΡΠ°Π΄Π΅Ρ. ΠΠ»Π΅ΠΊΡΡΠΎΠ΄Π²ΠΈΠΆΡΡΠ°Ρ ΡΠΈΠ»Π° E, ΠΈΠ½Π΄ΡΡΠΈΡΡΠ΅ΠΌΠ°Ρ Π² ΠΊΠ°ΠΆΠ΄ΡΠΉ Π²ΠΈΡΠΎΠΊ ΠΏΡΠΎΠ²ΠΎΠ΄Π° Π² Π»ΡΠ±ΠΎΠΉ ΡΠ΅ΠΏΠΈ, ΡΠΎΠ΄Π΅ΡΠΆΠ°ΡΠ΅ΠΉ ΠΏΠ΅ΡΠ»ΠΈ (ΠΊΠ°ΠΊ ΠΊΠ°ΡΡΡΠΊΠ°), ΡΠ²ΡΠ·Π°Π½ ΡΠΎ ΡΠΊΠΎΡΠΎΡΡΡΡ ΠΈΠ·ΠΌΠ΅Π½Π΅Π½ΠΈΡ Π²ΠΎ Π²ΡΠ΅ΠΌΠ΅Π½ΠΈ ΠΌΠ°Π³Π½ΠΈΡΠ½ΡΠΉ ΠΏΠΎΡΠΎΠΊ Ξ¦ ΡΠ΅ΡΠ΅Π· Π½Π΅Π³ΠΎ Π½Π°
Π ΡΠ»ΡΡΠ°Π΅ ΠΊΠ°ΡΡΡΠΊΠΈ ΠΈΠ· n Π²ΠΈΡΠΊΠΎΠ² (ΡΠΎΠ»Π΅Π½ΠΎΠΈΠ΄Π° ΠΈΠ»ΠΈ ΡΠΎΡΠΎΠΈΠ΄Π°) Π² ΠΊΠ°ΠΆΠ΄ΠΎΠΌ Π²ΠΈΡΠΊΠ΅ ΠΈΠ½Π΄ΡΡΠΈΡΡΠ΅ΡΡΡ ΠΠΠ‘, Π° ΠΏΠΎΡΠΊΠΎΠ»ΡΠΊΡ Π²ΠΈΡΠΊΠΈ ΠΏΠΎΡΠ»Π΅Π΄ΠΎΠ²Π°ΡΠ΅Π»ΡΠ½ΠΎ, ΠΎΠ±ΡΠ°Ρ ΠΠΠ‘
ΠΡΠΈΠΌΠ΅Ρ 1. Π Π°ΡΡΠΌΠΎΡΡΠΈΠΌ Π΅ΡΠ΅ ΡΠ°Π· ΡΡ Π΅ΠΌΡ ΡΠΈΡ. 5. ΠΠΎΠ³Π΄Π° ΠΏΡΠΎΠ²ΠΎΠ΄Π½ΠΈΠΊ ab Π΄Π²ΠΈΠΆΠ΅ΡΡΡ ΠΊ ΡΠΏΡΠ°Π²Π° Π½Π° ΡΠ°ΡΡΡΠΎΡΠ½ΠΈΠΈ ds ΠΏΠ»ΠΎΡΠ°Π΄Ρ ΠΏΠΎΠΏΠ΅ΡΠ΅ΡΠ½ΠΎΠ³ΠΎ ΡΠ΅ΡΠ΅Π½ΠΈΡ Π·Π°ΠΌΠΊΠ½ΡΡΠΎΠ³ΠΎ ΠΊΠΎΠ½ΡΡΡΠ° abcd ΡΠ²Π΅Π»ΠΈΡΠΈΠ²Π°Π΅ΡΡΡ Π½Π°
Β Β Β Β Β Β Β Β Β Β Β Β dA = l ds
, Π° ΠΈΠ·ΠΌΠ΅Π½Π΅Π½ΠΈΠ΅ ΠΌΠ°Π³Π½ΠΈΡΠ½ΠΎΠ³ΠΎ ΠΏΠΎΡΠΎΠΊΠ° ΡΠ΅ΡΠ΅Π· ΠΏΠ»ΠΎΡΠ°Π΄Ρ, ΠΎΠ³ΡΠ°Π½ΠΈΡΠ΅Π½Π½ΡΡ ΠΊΠΎΠ½ΡΡΡΠΎΠΌ, ΡΠ°Π²Π½ΠΎ
.            dΦ = BdA = l Bds
ΠΡΠ»ΠΈ ΠΌΡ ΡΠ°Π·Π΄Π΅Π»ΠΈΠΌ ΠΎΠ±Π΅ ΡΠ°ΡΡΠΈ ΡΡΠΎΠ³ΠΎ ΡΡΠ°Π²Π½Π΅Π½ΠΈΡ Π½Π° dt, ΠΌΡ ΠΏΠΎΠ»ΡΡΠΈΠΌ
ΠΈΠ»ΠΈ
, ΡΡΠΎ ΡΠΎΠ³Π»Π°ΡΡΠ΅ΡΡΡ Ρ ΠΏΡΠ½ΠΊΡΠΎΠΌ 3), Π·Π° ΠΈΡΠΊΠ»ΡΡΠ΅Π½ΠΈΠ΅ΠΌ Π·Π½Π°ΠΊΠ°. ΠΡΠΈΡΠΈΠ½Π° ΡΠ°Π·Π½ΠΈΡΡ Π² Π·Π½Π°ΠΊΠ΅ ΡΠ²ΡΠ·Π°Π½Π° ΡΠΎ Π·Π½Π°ΠΊΠΎΠΌ ΡΠΎΠ³Π»Π°ΡΠ΅Π½ΠΈΡ, ΠΊΠΎΡΠΎΡΡΠ΅ Π½Π΅ΠΎΠ±Ρ ΠΎΠ΄ΠΈΠΌΠΎ ΠΈΡΠΏΠΎΠ»ΡΠ·ΠΎΠ²Π°ΡΡ Π² ΠΎΡΠ½ΠΎΡΠ΅Π½ΠΈΠΈ Π½Π°ΠΏΡΠ°Π²Π»Π΅Π½ΠΈΡ ΡΠΎΠΊΠ°, Π½Π°ΠΏΡΠ°Π²Π»Π΅Π½ΠΈΡ ΠΏΠΎΡΠΎΠΊΠ° ΠΈ Ρ. Π΄. Ρ. Π΅. ΡΠΊΠ². 3) Π²Π΅ΡΠ½ΠΎ Π²ΡΡΠ΅ΠΏΡΠΈΠ²Π΅Π΄Π΅Π½Π½ΠΎΠ΅, ΡΡΠΈΡΡΠ²Π°Ρ ΡΠΎΠΎΡΠ²Π΅ΡΡΡΠ²ΡΡΡΠΈΠΉ Π·Π½Π°ΠΊ ΡΠΎΠ³Π»Π°ΡΠ΅Π½ΠΈΡ.
ΠΡΠΈΠΌΠ΅Ρ 2. Π Π°ΡΡΠΌΠΎΡΡΠΈΠΌ ΡΠΎΡΠΎΠΈΠ΄Π°Π»ΡΠ½ΡΡ ΠΎΠ±ΠΌΠΎΡΠΊΡ ΡΠΈΡ. 6 ΡΠ²ΡΠ·Π°Π½Ρ Ρ ΠΏΡΠΎΠ²ΠΎΠ΄ΡΡΠΈΠΌ ΠΊΠΎΠ»ΡΡΠΎΠΌ, ΠΊΠ°ΠΊ ΠΏΠΎΠΊΠ°Π·Π°Π½ΠΎ Π½Π° ΡΠΈΡΡΠ½ΠΊΠ΅. ΠΡΠ΅Π΄ΠΏΠΎΠ»ΠΎΠΆΠΈΠΌ, ΠΌΡ ΡΠΎΠ·Π΄Π°ΡΡ ΠΌΠ°Π³Π½ΠΈΡΠ½ΠΎΠ΅ ΠΏΠΎΠ»Π΅ Π² ΡΠΎΡΠΎΠΈΠ΄Π°Π»ΡΠ½ΠΎΠΉ ΠΎΠ±ΠΌΠΎΡΠΊΠ΅ ΠΈ Π·Π°ΡΠ΅ΠΌ Π²Π°ΡΡΠΈΡΠΎΠ²Π°ΡΡ ΠΏΠΎΠ»Π΅, ΠΈΠ·ΠΌΠ΅Π½ΡΡ ΡΠΎΠΊ Π² ΠΎΠ±ΠΌΠΎΡΠΊΠΈ. ΠΡ Π·Π½Π°Π΅ΠΌ, ΡΡΠΎ Π²Π΅ΡΡ ΠΌΠ°Π³Π½ΠΈΡΠ½ΡΠΉ ΠΏΠΎΡΠΎΠΊ Π·Π°ΠΊΠ»ΡΡΠ΅Π½ Π²Π½ΡΡΡΠΈ ΠΎΠ±ΠΌΠΎΡΠΊΠΈ. Π’Π°ΠΊΠΈΠΌ ΠΎΠ±ΡΠ°Π·ΠΎΠΌ, ΠΏΡΠΎΠ²Π΅Π΄Π΅Π½ΠΈΠ΅ ΠΊΠΎΠ»ΡΡΠΎ Π½Π΅ ΡΠΎΠ»ΡΠΊΠΎ Π½Π΅ Π΄Π²ΠΈΠΆΠ΅ΡΡΡ Π² ΠΌΠ°Π³Π½ΠΈΡΠ½ΠΎΠΌ ΠΏΠΎΠ»Π΅, ΠΎΠ½ΠΎ Π΄Π°ΠΆΠ΅ Π½Π΅ Π½Π°Ρ ΠΎΠ΄ΠΈΡΡΡ Π² ΠΌΠ°Π³Π½ΠΈΡΠ½ΠΎΠΌ ΠΏΠΎΠ»Π΅. Π’Π΅ΠΌ Π½Π΅ ΠΌΠ΅Π½Π΅Π΅ ΡΠΊΡΠΏΠ΅ΡΠΈΠΌΠ΅Π½Ρ ΠΏΠΎΠΊΠ°ΠΆΠ΅ΠΌ, ΡΡΠΎ Π΅ΡΠ»ΠΈ ΠΌΡ ΠΈΠ·ΠΌΠ΅Π½ΠΈΠΌ ΡΠΎΠΊ Π² ΠΎΠ±ΠΌΠΎΡΠΊΠ°Ρ , Π² ΠΊΠΎΠ»ΡΡΠ΅ Π²ΠΎΠ·Π½ΠΈΠΊΠ½Π΅Ρ ΠΠΠ‘.
ΠΡΠΎ ΠΏΠΎΠΊΠ°Π·ΡΠ²Π°Π΅Ρ, ΡΡΠΎ ΠΠΠ‘ ΠΌΠΎΠΆΠ΅Ρ ΡΠΎΠ·Π΄Π°Π²Π°ΡΡΡΡ Π² ΠΏΡΠΎΠ²ΠΎΠ΄Π½ΠΈΠΊΠ΅, Π΄Π°ΠΆΠ΅ Π΅ΡΠ»ΠΈ ΠΏΡΠΎΠ²ΠΎΠ΄Π½ΠΈΠΊ Π½Π΅ Π»Π΅ΠΆΠΈΡ Π² ΠΌΠ°Π³Π½ΠΈΡΠ½ΠΎΠ΅ ΠΏΠΎΠ»Π΅.
Β
ΠΠ°ΠΏΡΠ°Π²Π»Π΅Π½ΠΈΠ΅ ΠΈΠ½Π΄ΡΡΠΈΡΠΎΠ²Π°Π½Π½ΠΎΠ³ΠΎ ΡΠΎΠΊΠ°. ΠΠ°ΠΊΠΎΠ½ ΠΠ΅Π½ΡΠ°. ΠΠ½Π΄ΡΡΠΈΡΠΎΠ²Π°Π½Π½ΡΠΉ ΡΠΎΠΊ ΡΠ΅ΡΠ΅Ρ Π² ΡΠ°ΠΊΠΎΠΌ Π½Π°ΠΏΡΠ°Π²Π»Π΅Π½ΠΈΠ΅, ΡΡΠΎΠ±Ρ ΠΏΡΠΎΡΠΈΠ²ΠΎΠ΄Π΅ΠΉΡΡΠ²ΠΎΠ²Π°ΡΡ ΡΠ²ΠΎΠΈΠΌ ΡΠ»Π΅ΠΊΡΡΠΎΠΌΠ°Π³Π½ΠΈΡΠ½ΡΠΌ Π΄Π΅ΠΉΡΡΠ²ΠΈΠ΅ΠΌ Π΄Π²ΠΈΠΆΠ΅Π½ΠΈΡ ΠΈΠ»ΠΈ ΠΏΡΠΈΡΠΈΠ½Π΅, Π²ΡΠ·ΡΠ²Π°ΡΡΠ΅ΠΉ Π΅Π³ΠΎ.
Π Π°Π½Π΅Π΅ ΠΌΡ ΠΏΡΠΈΠ²ΠΎΠ΄ΠΈΠ»ΠΈ ΠΏΡΠΈΠΌΠ΅Ρ ΡΠΎΠ³ΠΎ, ΠΊΠ°ΠΊ ΠΌΠΎΠΆΠ½ΠΎ ΠΏΠΎΠ»ΡΡΠΈΡΡ ΠΈΠ½Π΄ΡΠΊΡΠΈΠΎΠ½Π½ΡΠΉ ΡΠΎΠΊ, ΠΏΠΎΠ³ΡΡΠΆΠ°Ρ ΡΡΠ΅ΡΠΆΠ΅Π½Ρ ΠΌΠ°Π³Π½ΠΈΡ Π²Π½ΠΈΠ· Π² ΠΊΠ°ΡΡΡΠΊΡ, ΡΠΎΠ΅Π΄ΠΈΠ½Π΅Π½Π½ΡΡ Ρ Π³Π°Π»ΡΠ²Π°Π½ΠΎΠΌΠ΅ΡΡΠΎΠΌ, ΠΏΠΎΠΊΠ°Π·Π°Π½Π½ΡΠΌ Π½Π° ΡΠΈΡ. 2. ΠΡΠ»ΠΈ ΡΠ΅Π²Π΅ΡΠ½ΡΠΉ ΠΏΠΎΠ»ΡΡ ΠΌΠ°Π³Π½ΠΈΡ Π²Π΄Π°Π²Π»ΠΈΠ²Π°Π΅ΡΡΡ Π² ΠΊΠ°ΡΡΡΠΊΡ, ΠΈΠ½Π΄ΡΡΠΈΡΠΎΠ²Π°Π½Π½ΡΠΉ ΡΠΎΠΊ, Π²ΠΎΠ·Π½ΠΈΠΊΠ°ΡΡΠΈΠΉ Π² ΠΊΠ°ΡΡΡΠΊΠ΅, ΠΏΠΎΡΠ΅ΡΠ΅Ρ Π² ΡΠ°ΠΊΠΎΠ΅ Π½Π°ΠΏΡΠ°Π²Π»Π΅Π½ΠΈΠ΅, ΡΡΠΎΠ±Ρ ΡΠΎΠ·Π΄Π°ΡΡ ΡΠ΅Π²Π΅ΡΠ½ΡΠΉ ΠΏΠΎΠ»ΡΡ Π² Π²Π΅ΡΡ Π½Π΅ΠΉ ΡΠ°ΡΡΠΈ ΠΊΠ°ΡΡΡΠΊΠΈ, ΡΡΠΎΠ±Ρ ΠΏΡΠΎΡΠΈΠ²ΠΎΠ΄Π΅ΠΉΡΡΠ²ΠΎΠ²Π°ΡΡ Π΄Π΅ΠΉΡΡΠ²ΠΈΡ. ΠΠ²Π° ΡΠ΅Π²Π΅ΡΠ½ΡΠ΅ ΠΏΠΎΠ»ΡΡΠ° ΠΎΡΡΠ°Π»ΠΊΠΈΠ²Π°ΡΡΡΡ Π΄ΡΡΠ³ ΠΎΡ Π΄ΡΡΠ³Π°, ΠΈ ΡΡΠ΅Π±ΡΠ΅ΡΡΡ ΡΠ°Π±ΠΎΡΠ°, ΡΡΠΎΠ±Ρ Π²ΡΠΎΠ»ΠΊΠ½ΡΡΡ ΠΏΠΎΠ»ΡΡ Π² ΠΊΠ°ΡΡΡΠΊΡ. ΠΠ°ΠΊ ΡΠΎΠ»ΡΠΊΠΎ Π±Π°Ρ ΠΌΠ°Π³Π½ΠΈΡ Π±ΡΠ» Π²Π΄Π°Π²Π»Π΅Π½ Π² ΠΊΠ°ΡΡΡΠΊΡ, ΠΈ ΠΌΡ ΠΏΡΡΠ°Π΅ΠΌΡΡ Π²ΡΡΠ°ΡΠΈΡΡ Π΅Π³ΠΎ, ΡΠΎΠΊ ΠΌΠ΅Π½ΡΠ΅ΡΡΡ Π½Π° ΠΏΡΠΎΡΠΈΠ²ΠΎΠΏΠΎΠ»ΠΎΠΆΠ½ΡΠΉ ΠΈ ΠΎΠ±ΡΠ°Π·ΡΠ΅ΡΡΡ ΡΠΆΠ½ΡΠΉ ΠΏΠΎΠ»ΡΡ, ΠΊΠΎΡΠΎΡΡΠΉ ΠΏΡΠΈΡΡΠ³ΠΈΠ²Π°Π΅Ρ ΡΠ΅Π²Π΅ΡΠ½ΡΠΉ ΠΏΠΎΠ»ΡΡ ΠΈ ΠΏΡΠΎΡΠΈΠ²ΠΎΡΡΠΎΠΈΡ Π΅Π³ΠΎ Π²ΡΡΡΠ³ΠΈΠ²Π°Π½ΠΈΡ.
Π’Π°ΠΊΠΈΠΌ ΠΆΠ΅ ΠΎΠ±ΡΠ°Π·ΠΎΠΌ, Π΅ΡΠ»ΠΈ ΠΌΡ ΠΏΠΎΠΏΡΡΠ°Π΅ΠΌΡΡ Π²Π΄Π°Π²ΠΈΡΡ ΡΠΆΠ½ΡΠΉ ΠΏΠΎΠ»ΡΡ ΠΌΠ°Π³Π½ΠΈΡΠ° Π² ΠΊΠ°ΡΡΡΠΊΡ, Π²Π΅ΡΡ Π½ΡΡ ΡΠ°ΡΡΡ ΠΊΠ°ΡΡΡΠΊΠ° ΡΠ°Π·ΠΎΠ²ΡΠ΅Ρ ΡΠΆΠ½ΡΠΉ ΠΏΠΎΠ»ΡΡ, ΡΡΠΎΠ±Ρ ΠΏΡΠΎΡΠΈΠ²ΠΎΠ΄Π΅ΠΉΡΡΠ²ΠΎΠ²Π°ΡΡ Π΄Π΅ΠΉΡΡΠ²ΠΈΡ. ΠΡΡΡΠ΄Π° ΠΌΡ Π²ΠΈΠ΄ΠΈΠΌ, ΡΡΠΎ ΡΠ½Π΅ΡΠ³ΠΈΡ ΠΈΠ½Π΄ΡΡΠΈΡΠΎΠ²Π°Π½Π½ΡΠΉ ΡΠΎΠΊ, Π²ΠΎΠ·Π½ΠΈΠΊΠ°ΡΡΠΈΠΉ ΠΏΡΠΈ ΡΠ»Π΅ΠΊΡΡΠΎΠΌΠ°Π³Π½ΠΈΡΠ½ΠΎΠΉ ΠΈΠ½Π΄ΡΠΊΡΠΈΠΈ, Π½Π΅ ΠΎΡΠ²ΠΎΠ±ΠΎΠΆΠ΄Π°Π΅ΡΡΡ. ΠΠ½ΠΎ ΠΏΡΠΎΠΈΠ·ΠΎΡΠ»ΠΎ ΠΎΡ ΡΠ°Π±ΠΎΡΠ°, ΠΊΠΎΡΠΎΡΠ°Ρ ΡΠ΄Π΅Π»Π°Π½Π°. ΠΡΠ΅ΠΎΠ±ΡΠ°Π·ΠΎΠ²Π°Π½ΠΈΠ΅ ΠΌΠ΅Ρ Π°Π½ΠΈΡΠ΅ΡΠΊΠΎΠΉ ΡΠ½Π΅ΡΠ³ΠΈΠΈ Π² ΡΠ»Π΅ΠΊΡΡΠΈΡΠ΅ΡΠΊΡΡ ΠΏΡΠΎΠΈΡΡ ΠΎΠ΄ΠΈΡ Π² ΡΠ΅Π½ΠΎΠΌΠ΅Π½.
ΠΡΠ»ΠΈ ΠΈΠ½Π΄ΡΡΠΈΡΠΎΠ²Π°Π½Π½ΡΠΉ ΡΠΎΠΊ Π² ΠΊΠ°ΡΡΡΠΊΠ΅ Π²ΡΠ·Π²Π°Π½ ΡΠ²Π΅Π»ΠΈΡΠ΅Π½ΠΈΠ΅ΠΌ ΠΏΠΎΡΠΎΠΊΠ° ΡΠ΅ΡΠ΅Π· ΠΊΠ°ΡΡΡΠΊΡ, ΠΈΠ½Π΄ΡΡΠΈΡΠΎΠ²Π°Π½Π½ΡΠΉ ΡΠΎΠΊ ΠΈΠΌΠ΅Π΅Ρ ΡΠ°ΠΊΠΎΠ΅ Π½Π°ΠΏΡΠ°Π²Π»Π΅Π½ΠΈΠ΅, ΡΡΠΎ Π²ΠΎΠ·Π½ΠΈΠΊΠ°ΡΡ ΠΌΠ°Π³Π½ΠΈΡΠ½ΡΠ΅ Π»ΠΈΠ½ΠΈΠΈ, Π½Π°ΠΏΡΠ°Π²Π»Π΅Π½Π½ΡΠ΅ Π² ΠΏΡΠΎΡΠΈΠ²ΠΎΠΏΠΎΠ»ΠΎΠΆΠ½ΡΡ ΡΡΠΎΡΠΎΠ½Ρ ΠΎΡ Π»ΠΈΠ½ΠΈΠΈ ΠΈΡΡ ΠΎΠ΄Π½ΠΎΠ³ΠΎ ΠΏΠΎΠ»Ρ. ΠΡΠ»ΠΈ Π΄Π²ΠΈΠΆΡΡΠ°ΡΡΡ ΠΏΡΠΎΠ²ΠΎΠ»ΠΎΠΊΠ° ΠΏΠ΅ΡΠ΅ΡΠ΅ΠΊΠ°Π΅Ρ ΠΌΠ°Π³Π½ΠΈΡΠ½ΡΠΉ ΠΏΠΎΡΠΎΠΊ, ΠΈΠ½Π΄ΡΠΊΡΠΈΠΎΠ½Π½ΡΠΉ ΡΠΎΠΊ Π½Π°Ρ ΠΎΠ΄ΠΈΡΡΡ Π² ΡΠ°ΠΊΠΎΠΉ Π½Π°ΠΏΡΠ°Π²Π»Π΅Π½ΠΈΠΈ, ΡΡΠΎΠ±Ρ ΡΠΎΠ·Π΄Π°ΡΡ ΠΌΠ°Π³Π½ΠΈΡΠ½ΠΎΠ΅ ΠΏΠΎΠ»Π΅, ΠΏΡΠΎΡΠΈΠ²ΠΎΠ΄Π΅ΠΉΡΡΠ²ΡΡΡΠ΅Π΅ Π΄Π²ΠΈΠΆΠ΅Π½ΠΈΡ.
Π‘Π°ΠΌΠΎΠΈΠ½Π΄ΡΠΊΡΠΈΡ. ΠΡΠ΅Π΄ΠΏΠΎΠ»ΠΎΠΆΠΈΠΌ, ΠΌΡ ΠΏΠΎΡΠ»Π΅Π΄ΠΎΠ²Π°ΡΠ΅Π»ΡΠ½ΠΎ ΠΏΠΎΠ΄ΠΊΠ»ΡΡΠ°Π΅ΠΌ ΠΌΠ°Π»Π΅Π½ΡΠΊΡΡ Π»Π°ΠΌΠΏΡ Ρ Π²ΡΠΊΠ»ΡΡΠ°ΡΠ΅Π»Π΅ΠΌ ΠΈ ΡΡΡ ΠΈΠΌ ΡΠ»Π΅ΠΌΠ΅Π½ΡΠΎΠΌ. ΠΠΎΠ³Π΄Π° ΠΌΡ Π·Π°ΠΌΡΠΊΠ°Π΅ΠΌ Π²ΡΠΊΠ»ΡΡΠ°ΡΠ΅Π»Ρ, Π»Π°ΠΌΠΏΠ° Π·Π°Π³ΠΎΡΠ°Π΅ΡΡΡ ΡΡΠ°Π·Ρ. ΠΠΎΠ³Π΄Π° ΠΌΡ ΡΠ°Π·ΠΌΡΠΊΠ°Π΅ΠΌ Π²ΡΠΊΠ»ΡΡΠ°ΡΠ΅Π»Ρ, Π»Π°ΠΌΠΏΠ° ΡΡΠ°Π·Ρ Π³Π°ΡΠ½Π΅Ρ. Π’Π΅ΠΏΠ΅ΡΡ Π²ΠΊΠ»ΡΡΠΈΠΌ Π² ΡΠ΅ΠΏΡ ΠΊΠ°ΡΡΡΠΊΡ (Π½Π°ΠΏΡΠΈΠΌΠ΅Ρ, ΠΏΠΎΠΊΠ°Π·Π°Π½Π½ΡΡ Π½Π° ΡΠΈΡ. 2). Π’Π΅ΠΏΠ΅ΡΡ, ΠΊΠΎΠ³Π΄Π° ΠΌΡ Π·Π°ΠΌΡΠΊΠ°Π΅ΠΌ Π²ΡΠΊΠ»ΡΡΠ°ΡΠ΅Π»Ρ, ΠΌΡ ΠΎΠ±Π½Π°ΡΡΠΆΠΈΠ²Π°Π΅ΠΌ, ΡΡΠΎ Π»Π°ΠΌΠΏΠ° Π·Π°Π³ΠΎΡΠ°Π΅ΡΡΡ Π½Π΅ ΡΠ°ΠΊ Π±ΡΡΡΡΠΎ. ΠΠΎΠ³Π΄Π° ΠΌΡ ΡΠ°Π·ΠΎΠΌΠΊΠ½ΠΈΡΠ΅ Π²ΡΠΊΠ»ΡΡΠ°ΡΠ΅Π»Ρ, Π»Π°ΠΌΠΏΠ° Π³ΠΎΡΠΈΡ Π΄ΠΎΠ»ΡΡΠ΅ ΠΈ ΡΡΡΠΊΠ½Π΅Π΅Ρ ΠΏΠ΅ΡΠ΅Π΄ ΡΠ΅ΠΌ, ΠΊΠ°ΠΊ ΠΏΠΎΠ³Π°ΡΠ½ΡΡΡ. ΠΠ±ΡΡΡΠ½Π΅Π½ΠΈΠ΅ ΠΏΠΎΡΠΊΠΎΠ»ΡΠΊΡ ΡΠ°ΠΊΠΎΠ΅ ΠΏΠΎΠ²Π΅Π΄Π΅Π½ΠΈΠ΅ Π·Π°ΠΊΠ»ΡΡΠ°Π΅ΡΡΡ Π² ΡΠ²Π»Π΅Π½ΠΈΠΈ, Π½Π°Π·ΡΠ²Π°Π΅ΠΌΠΎΠΌ ΡΠ°ΠΌΠΎΠΈΠ½Π΄ΡΠΊΡΠΈΠ΅ΠΉ, ΡΠ²Π»Π΅Π½ΠΈΠΈ, ΠΎΡΠΊΡΡΡΠΎΠΌ ΠΠΌΠ΅ΡΠΈΠΊΠ°Π½ΡΠΊΠΈΠΉ ΡΠΈΠ·ΠΈΠΊ ΠΠΆΠΎΠ·Π΅Ρ ΠΠ΅Π½ΡΠΈ (179 Π³.7 — 1878). Π Π½Π°ΡΠ΅ΠΌ ΡΠΊΡΠΏΠ΅ΡΠΈΠΌΠ΅Π½ΡΠ΅ ΠΊΠ°ΡΡΡΠΊΠ° Π·Π°ΠΌΠ΅Π΄Π»ΡΠ»Π° Π½Π°ΡΠ°ΡΡΠ°Π½ΠΈΠ΅ ΡΠΎΠΊΠ°, ΠΊΠΎΠ³Π΄Π° ΠΌΡ Π·Π°ΠΌΡΠΊΠ°Π»ΠΈ Π²ΡΠΊΠ»ΡΡΠ°ΡΠ΅Π»Ρ, Π° Π·Π°ΡΠ΅ΠΌ Π·Π°ΠΌΠ΅Π΄Π»ΡΠ»ΠΈ ΡΠ³Π°ΡΠ°Π½ΠΈΠ΅ ΡΠΎΠΊΠ°, ΠΊΠΎΠ³Π΄Π° ΠΌΡ ΠΎΡΠΊΡΡΠ» ΠΏΠ΅ΡΠ΅ΠΊΠ»ΡΡΠ°ΡΠ΅Π»Ρ. ΠΠΎΡΠ΅ΠΌΡ ΡΡΠΎ ΡΠ»ΡΡΠΈΠ»ΠΎΡΡ? ΠΡΠΎ ΠΏΡΠΎΠΈΠ·ΠΎΡΠ»ΠΎ ΠΈΠ·-Π·Π° Π²ΡΡΡΠ΅ΡΠ½ΠΎΠΉ ΠΠΠ‘, ΠΏΡΠΎΡΠΈΠ²ΠΎΠ΄Π΅ΠΉΡΡΠ²ΡΡΡΠ΅ΠΉ ΠΠΠ‘, Π²ΠΎΠ·Π½ΠΈΠΊΠ°ΡΡΠ°Ρ Π² ΠΊΠ°ΡΡΡΠΊΠ΅, Π·Π°ΠΌΠ΅Π΄Π»ΡΠ»Π° Π½Π°ΡΠ°ΡΡΠ°Π½ΠΈΠ΅ ΠΈ Π·Π°ΡΡΡ Π°Π½ΠΈΠ΅ ΡΠΎΠΊΠ°. ΠΠ΄Π΅ Π±Ρ ΠΎΡΠΊΡΠ΄Π° Π²Π·ΡΠ»Π°ΡΡ ΡΡΠ° ΠΏΡΠΎΡΠΈΠ²ΠΎ-ΠΠΠ‘? ΠΠΎΡΠ΅ΠΌΡ ΡΡΠΎ ΠΏΡΠΎΠΈΠ·ΠΎΡΠ»ΠΎ? ΠΡΠ²Π΅Ρ Π·Π°ΠΊΠ»ΡΡΠ°Π΅ΡΡΡ Π² ΡΠΎΠΌ, ΡΡΠΎ ΡΡΠΎ ΠΠΠ‘ ΡΠ°ΠΌΠΎΠΈΠ½Π΄ΡΠΊΡΠΈΠΈ, ΠΠΠ‘, Π²ΠΎΠ·Π½ΠΈΠΊΠ°ΡΡΠ°Ρ ΠΈΠ·-Π·Π° Π΅Π³ΠΎ ΡΠΎΠ±ΡΡΠ²Π΅Π½Π½ΠΎΠ³ΠΎ ΠΈΠ·ΠΌΠ΅Π½ΡΡΡΠ΅Π³ΠΎΡΡ ΠΌΠ°Π³Π½ΠΈΡΠ½ΠΎΠ³ΠΎ ΠΏΠΎΠ»Ρ, ΡΠΎΠΏΡΠΎΠ²ΠΎΠΆΠ΄Π°ΡΡΠ΅Π³ΠΎ Π½Π°ΡΠ°ΡΡΠ°Π½ΠΈΠ΅ ΠΈΠ»ΠΈ Π·Π°ΡΡΡ Π°Π½ΠΈΠ΅ Π΅Π³ΠΎ ΡΠΎΠΊΠ°. ΠΡΡΠ΅, ΠΈΡΠΏΠΎΠ»ΡΠ·ΡΡ ΠΏΠ΅ΡΠ²ΠΈΡΠ½ΡΡ ΠΊΠ°ΡΡΡΠΊΡ ΠΈ Π²ΡΠΎΡΠΈΡΠ½ΡΡ ΠΊΠ°ΡΡΡΠΊΡ, ΠΌΡ ΠΏΠΎΠΊΠ°Π·Π°Π»ΠΈ, ΠΊΠ°ΠΊ ΡΠΎΠΊ Π² ΠΏΠ΅ΡΠ²ΠΈΡΠ½ΠΎΠΉ ΠΊΠ°ΡΡΡΠΊΠ΅ ΠΌΠΎΠΆΠ΅Ρ ΠΈΠ½Π΄ΡΡΠΈΡΠΎΠ²Π°ΡΡ ΡΠΎΠΊ Π²ΠΎ Π²ΡΠΎΡΠΈΡΠ½ΠΎΠΉ ΠΎΠ±ΠΌΠΎΡΠΊΠ΅. ΠΠΎΠ³Π΄Π° ΠΏΠ΅ΡΠ²ΠΈΡΠ½Π°Ρ ΡΠ΅ΠΏΡ Π±ΡΠ»Π° Π·Π°ΠΌΠΊΠ½ΡΡΠ°, Π° Π΅Π΅ ΡΠΎΠΊ Π±ΡΠ» Π΅Π³ΠΎ ΡΠ°Π·Π²ΠΈΠ²Π°ΡΡΠ΅Π΅ΡΡ ΠΌΠ°Π³Π½ΠΈΡΠ½ΠΎΠ΅ ΠΏΠΎΠ»Π΅ Π²ΡΠ·ΡΠ²Π°Π»ΠΎ ΠΠΠ‘ ΠΈΠ½Π΄ΡΠΊΡΠΈΠΈ ΠΈ ΠΈΠ½Π΄ΡΠΊΡΠΈΠΎΠ½Π½ΡΠΉ ΡΠΎΠΊ Π² Π²ΡΠΎΡΠΈΡΠ½Π°Ρ ΠΊΠ°ΡΡΡΠΊΠ°. ΠΡ, ΡΠ²Π΅Π»ΠΈΡΠΈΠ²Π°ΡΡΠΈΠΉΡΡ ΡΠΎΠΊ Π² ΠΊΠ°ΡΡΡΠΊΠ΅ Π²ΡΠ·ΡΠ²Π°Π΅Ρ Π½Π°ΡΠ°ΡΡΠ°Π½ΠΈΠ΅ ΠΌΠ°Π³Π½ΠΈΡΠ½ΠΎΠ³ΠΎ ΠΏΠΎΠ»Ρ, ΠΊΠΎΡΠΎΡΠΎΠ΅ ΠΈΠ½Π΄ΡΡΠΈΡΡΠ΅Ρ Π²ΡΠΎΡΠ°Ρ ΠΏΡΠΎΡΠΈΠ²ΠΎΠ΄Π΅ΠΉΡΡΠ²ΡΡΡΠ°Ρ ΠΠΠ‘ Π² ΡΠ°ΠΌΠΎΠΉ ΠΊΠ°ΡΡΡΠΊΠ΅. ΠΠΎ Π·Π°ΠΊΠΎΠ½Ρ ΠΠ΅Π½ΡΠ° ΡΡΠ° Π²ΡΠΎΡΠ°Ρ ΠΠΠ‘ ΠΈΠ½Π΄ΡΠΊΡΠΈΠΈ Π±ΡΠ΄Π΅Ρ ΠΏΡΠΎΡΠΈΠ²ΠΎΠ΄Π΅ΠΉΡΡΠ²ΠΎΠ²Π°ΡΡ Π΄Π΅ΠΉΡΡΠ²ΠΈΠ΅ ΠΏΠ΅ΡΠ²ΠΎΠΉ ΠΠΠ‘. ΠΠ°ΠΊΠΎΠ½ ΡΠ»Π΅ΠΊΡΡΠΎΠΌΠ°Π³Π½ΠΈΡΠ½ΠΎΠΉ ΠΈΠ½Π΄ΡΠΊΡΠΈΠΈ Π³Π»Π°ΡΠΈΡ, ΡΡΠΎ ΠΠΠ‘ ΠΈΠ½Π΄ΡΡΠΈΡΡΠ΅ΡΡΡ Π² Π»ΡΠ±ΠΎΠΌ ΡΠ΅ΠΏΡ, Π² ΠΊΠΎΡΠΎΡΠΎΠΉ ΠΌΠ°Π³Π½ΠΈΡΠ½ΡΠΉ ΠΏΠΎΡΠΎΠΊ ΠΈΠ·ΠΌΠ΅Π½ΡΠ΅ΡΡΡ. ΠΠ·ΠΌΠ΅Π½ΡΡΡΠ΅Π΅ΡΡ ΠΌΠ°Π³Π½ΠΈΡΠ½ΠΎΠ΅ ΠΏΠΎΠ»Π΅ Π²ΠΎΠΊΡΡΠ³ ΠΏΡΠΎΠ²ΠΎΠ΄Π½ΠΈΠΊΠ° Π² ΠΏΡΠΎΠ²ΠΎΠ΄Π½ΠΈΠΊΠ΅ Π²ΠΎΠ·Π½ΠΈΠΊΠ°Π΅Ρ ΠΠΠ‘ ΠΈΠ½Π΄ΡΠΊΡΠΈΠΈ. ΠΡΡΠΎΡΠ½ΠΈΠΊ ΡΡΠΎΠ³ΠΎ ΠΌΠ°Π³Π½ΠΈΡΠ½ΠΎΠ³ΠΎ ΠΏΠΎΠ»Ρ Π½Π΅ ΠΈΠΌΠ΅Π΅Ρ Π·Π½Π°ΡΠ΅Π½ΠΈΡ. ΠΡΠΎ ΠΌΠΎΠΆΠ΅Ρ Π±ΡΡΡ ΠΎΡ ΡΠ°ΠΌΠΎΠ³ΠΎ ΠΏΡΠΎΠ²ΠΎΠ΄Π½ΠΈΠΊΠ°. Π‘Π»Π΅Π΄ΠΎΠ²Π°ΡΠ΅Π»ΡΠ½ΠΎ, Π»ΡΠ±Π°Ρ ΡΠ΅ΠΏΡ, Π² ΠΊΠΎΡΠΎΡΠΎΠΉ ΠΏΡΠΈΡΡΡΡΡΠ²ΡΠ΅Ρ ΠΏΠ΅ΡΠ΅ΠΌΠ΅Π½Π½ΡΠΉ ΡΠΎΠΊ Π½Π°Π²Π΅Π»Π° Π² Π½Π΅ΠΌ ΠΠΠ‘ ΠΈΠ·-Π·Π° ΠΈΠ·ΠΌΠ΅Π½Π΅Π½ΠΈΡ ΡΠΎΠ±ΡΡΠ²Π΅Π½Π½ΠΎΠ³ΠΎ ΠΌΠ°Π³Π½ΠΈΡΠ½ΠΎΠ³ΠΎ ΠΏΠΎΠ»Ρ.
ΠΠΠ‘ ΡΠ°ΠΌΠΎΠΈΠ½Π΄ΡΠΊΡΠΈΠΈ ΠΏΡΠΎΠΏΠΎΡΡΠΈΠΎΠ½Π°Π»ΡΠ½Π° ΡΠΊΠΎΡΠΎΡΡΠΈ ΠΈΠ·ΠΌΠ΅Π½Π΅Π½ΠΈΡ ΡΠΎΠΊΠ° Π²ΠΎ Π²ΡΠ΅ΠΌΠ΅Π½ΠΈ. ΠΠΠ‘ ΡΠ°ΠΌΠΎΠΈΠ½Π΄ΡΠΊΡΠΈΠΈ E ΡΠ΅ΠΏΠΈ ΠΏΡΠΎΠΏΠΎΡΡΠΈΠΎΠ½Π°Π»ΡΠ½Π° ΡΠΊΠΎΡΠΎΡΡΠΈ ΠΈΠ·ΠΌΠ΅Π½Π΅Π½ΠΈΡ Π²ΠΎ Π²ΡΠ΅ΠΌΠ΅Π½ΠΈ ΡΠΎΠΊΠ° I Π² ΡΠ΅ΠΏΠΈ. ΡΡ Π΅ΠΌΠ°
, Π³Π΄Π΅ L β ΠΊΠΎΠ½ΡΡΠ°Π½ΡΠ°, Π½Π°Π·ΡΠ²Π°Π΅ΠΌΠ°Ρ ΡΠΎΠ±ΡΡΠ²Π΅Π½Π½ΠΎΠΉ ΠΈΠ½Π΄ΡΠΊΡΠΈΠ²Π½ΠΎΡΡΡΡ ΡΠ΅ΠΏΠΈ. ΠΡΡΠΈΡΠ°ΡΠ΅Π»ΡΠ½ΡΠΉ Π·Π½Π°ΠΊ ΡΠΊΠ°Π·ΡΠ²Π°Π΅Ρ Π½Π° ΡΠΎ, ΡΡΠΎ ΠΠΠ‘ ΡΠ°ΠΌΠΎΠΈΠ½Π΄ΡΠΊΡΠΈΠΈ — ΡΡΠΎ ΠΎΠ±ΡΠ°ΡΠ½Π°Ρ ΠΠΠ‘, ΠΊΠΎΡΠΎΡΠ°Ρ ΠΏΡΠΎΡΠΈΠ²ΠΎΠ΄Π΅ΠΉΡΡΠ²ΡΠ΅Ρ ΠΈΠ·ΠΌΠ΅Π½Π΅Π½ΠΈΡ ΡΠΎΠΊΠ°, ΠΊΠΎΡΠΎΡΡΠΉ Π΅Π΅ ΠΏΡΠΎΠΈΠ·Π²ΠΎΠ΄ΠΈΡ.
ΠΠΎΠ³Π΄Π° E Π² Π²ΠΎΠ»ΡΡΠ°Ρ ΠΈ ββdI/dt Π² Π°ΠΌΠΏΠ΅Ρ/Ρ, L Π² Π³Π΅Π½ΡΠΈ. Π‘ΠΎΠ±ΡΡΠ²Π΅Π½Π½Π°Ρ ΠΈΠ½Π΄ΡΠΊΡΠΈΠ²Π½ΠΎΡΡΡ ΡΠ΅ΠΏΠΈ ΡΠ°Π²Π½Π° 1 ΠΠ΅Π½ΡΠΈ, Π΅ΡΠ»ΠΈ Π² Π½Π΅ΠΌ ΠΈΠ½Π΄ΡΡΠΈΡΡΠ΅ΡΡΡ ΠΠΠ‘ 1 Π²ΠΎΠ»ΡΡ ΠΏΡΠΈ ΠΈΠ·ΠΌΠ΅Π½Π΅Π½ΠΈΠΈ ΡΠΎΠΊΠ° ΡΠΎ ΡΠΊΠΎΡΠΎΡΡΡΡ 1 Π°ΠΌΠΏΠ΅Ρ/ΡΠ΅ΠΊ.
Π‘ΠΎΠ±ΡΡΠ²Π΅Π½Π½Π°Ρ ΠΈΠ½Π΄ΡΠΊΡΠΈΠ²Π½ΠΎΡΡΡ L ΡΠ΅ΠΏΠΈ Π·Π°Π²ΠΈΡΠΈΡ ΠΎΡ ΡΠ°Π·ΠΌΠ΅ΡΠ°, ΡΠΎΡΠΌΡ, ΠΊΠΎΠ»ΠΈΡΠ΅ΡΡΠ²Π° Π²ΠΈΡΠΊΠΎΠ² ΠΈ Ρ. Π΄. ΡΠ΅ΠΏΠΈ. ΠΡΠΎ ΡΠ°ΠΊΠΆΠ΅ Π·Π°Π²ΠΈΡΠΈΡ ΠΎΡ ΠΌΠ°Π³Π½ΠΈΡΠ½ΡΡ ΡΠ²ΠΎΠΉΡΡΠ² Π»ΡΠ±ΡΡ ΠΌΠ°ΡΠ΅ΡΠΈΠ°Π»ΠΎΠ², Π² ΠΊΠΎΡΠΎΡΡΡ ΡΡΡΠ΅ΡΡΠ²ΡΡΡ ΠΌΠ°Π³Π½ΠΈΡΠ½ΡΠ΅ ΠΏΠΎΠ»Ρ. ΡΠ°ΠΌΠΎΠΈΠ½Π΄ΡΠΊΡΠΈΡ ΡΠΎΠ»Π΅Π½ΠΎΠΈΠ΄Π° Π·Π°Π΄Π°Π½Π½ΡΡ ΡΠ°Π·ΠΌΠ΅ΡΠΎΠ² Π½Π°ΠΌΠ½ΠΎΠ³ΠΎ Π±ΠΎΠ»ΡΡΠ΅, Π΅ΡΠ»ΠΈ ΠΎΠ½ ΠΈΠΌΠ΅Π΅Ρ ΠΆΠ΅Π»Π΅Π·Π½ΡΠΉ ΡΠ΅ΡΠ΄Π΅ΡΠ½ΠΈΠΊ, ΡΠ΅ΠΌ Π΅ΡΠ»ΠΈ Π±Ρ ΠΎΠ½ Π½Π°Ρ ΠΎΠ΄ΠΈΡΡΡ Π² Π²Π°ΠΊΡΡΠΌΠ΅.
ΠΠ°Ρ. ΠΠ½Π΄ΡΠΊΡΠΎΡ. Π¦Π΅ΠΏΡ ΠΈΠ»ΠΈ ΡΠ°ΡΡΡ ΡΠ΅ΠΏΠΈ, ΠΊΠΎΡΠΎΡΠ°Ρ ΠΈΠΌΠ΅Π΅Ρ ΠΈΠ½Π΄ΡΠΊΡΠΈΠ²Π½ΠΎΡΡΡ.
Π‘Ρ Π΅ΠΌΠ°ΡΠΈΡΠ΅ΡΠΊΠΎΠ΅ ΠΎΠ±ΠΎΠ·Π½Π°ΡΠ΅Π½ΠΈΠ΅ ΠΊΠ°ΡΡΡΠΊΠΈ ΠΈΠ½Π΄ΡΠΊΡΠΈΠ²Π½ΠΎΡΡΠΈ ΠΏΠΎΠΊΠ°Π·Π°Π½ΠΎ Π½Π° ΡΠΈΡ. 7.
ΠΠ°ΠΏΡΠ°Π²Π»Π΅Π½ΠΈΠ΅ ΠΠΠ‘ ΡΠ°ΠΌΠΎΠΈΠ½Π΄ΡΠΊΡΠΈΠΈ. ΠΠ°ΠΏΡΠ°Π²Π»Π΅Π½ΠΈΠ΅ ΠΠΠ‘ ΡΠ°ΠΌΠΎΠΈΠ½Π΄ΡΠΊΡΠΈΠΈ Π½Π°Ρ ΠΎΠ΄ΠΈΡΡΡ ΠΏΠΎ ΡΠΎΡΠΌΡΠ»Π΅ ΠΠ΅Π½ΡΠ°. Π·Π°ΠΊΠΎΠ½. ΠΡΠΈΡΠΈΠ½Π° ΠΠΠ‘ — ΡΡΠΎ ΡΠ²Π΅Π»ΠΈΡΠ΅Π½ΠΈΠ΅ ΠΈΠ»ΠΈ ΡΠΌΠ΅Π½ΡΡΠ°ΡΡΠΈΠΉΡΡ ΡΠΎΠΊ. ΠΡΠ»ΠΈ ΡΠΎΠΊ ΡΠ²Π΅Π»ΠΈΡΠΈΠ²Π°Π΅ΡΡΡ, Π½Π°ΠΏΡΠ°Π²Π»Π΅Π½ΠΈΠ΅ ΠΠΠ‘ ΠΈΠ½Π΄ΡΠΊΡΠΈΠΈ Π½Π°ΠΏΡΠΎΡΠΈΠ² ΡΠΎΠ³ΠΎ, ΡΡΠΎ ΡΠ»Π΅ΠΊΡΡΠΈΡΠ΅ΡΠΊΠΈΠΉ ΡΠΎΠΊ. ΠΡΠ»ΠΈ ΡΠΎΠΊ ΡΠΌΠ΅Π½ΡΡΠ°Π΅ΡΡΡ, Π½Π°ΠΏΡΠ°Π²Π»Π΅Π½ΠΈΠ΅ ΠΠΠ‘ ΡΠ°ΠΊΠ°Ρ ΠΆΠ΅, ΠΊΠ°ΠΊ ΡΡΠΎ Ρ ΡΠΎΠΊΠ°. Π‘ΠΌ. ΡΠΈΡ. 8. ΠΡΠΎΡΠΈΠ²ΠΎΠ΄Π΅ΠΉΡΡΠ²ΡΠ΅Ρ ΠΈΠ·ΠΌΠ΅Π½Π΅Π½ΠΈΠ΅ ΡΠΎΠΊΠ°, Π° Π½Π΅ ΡΠ°ΠΌ ΡΠΎΠΊ. ΠΏΠΎ ΠΠΠ‘ ΠΈΠ½Π΄ΡΠΊΡΠΈΠΈ.
Π‘Π°ΠΌΠΎΠΈΠ½Π΄ΡΠΊΡΠΈΡ ΠΊΠ°ΡΡΡΠΊΠΈ ΠΈΠ· n Π²ΠΈΡΠΊΠΎΠ². Π‘ΠΎΠ±ΡΡΠ²Π΅Π½Π½Π°Ρ ΠΈΠ½Π΄ΡΠΊΡΠΈΠ²Π½ΠΎΡΡΡ L ΠΊΠ°ΡΡΡΠΊΠΈ ΠΈΠ· n Π²ΠΈΡΠΊΠΎΠ² (ΡΠΎΠ»Π΅Π½ΠΎΠΈΠ΄ ΠΈΠ»ΠΈ ΡΠΎΡΠΎΠΈΠ΄) Π·Π°Π΄Π°Π΅ΡΡΡ
Β
, Π³Π΄Π΅ Ξ¦ β ΠΏΠΎΡΠΎΠΊ Π² ΠΊΠ°ΡΡΡΠΊΠ΅, Π° I β ΡΠΎΠΊ. ΠΠ΅Π»ΠΈΡΠΈΠ½Π° nΞ¦ Π½Π°Π·ΡΠ²Π°Π΅ΡΡΡ ΠΏΠΎΡΠΎΠΊΠΎΡΡΠ΅ΠΏΠ»Π΅Π½ΠΈΠ΅ΠΌ ΠΊΠ°ΡΡΡΠΊΠ°. Π’Π°ΠΊΠΈΠΌ ΠΎΠ±ΡΠ°Π·ΠΎΠΌ, ΡΠΎΠ±ΡΡΠ²Π΅Π½Π½Π°Ρ ΠΈΠ½Π΄ΡΠΊΡΠΈΠ²Π½ΠΎΡΡΡ L ΠΏΡΠ΅Π΄ΡΡΠ°Π²Π»ΡΠ΅Ρ ΡΠΎΠ±ΠΎΠΉ ΠΏΠΎΡΠΎΠΊΠΎΡΡΠ΅ΠΏΠ»Π΅Π½ΠΈΠ΅ Π½Π° Π°ΠΌΠΏΠ΅Ρ.
ΠΡΠ²ΠΎΠ΄. ΠΠΎ Π·Π°ΠΊΠΎΠ½Ρ ΡΠ»Π΅ΠΊΡΡΠΎΠΌΠ°Π³Π½ΠΈΡΠ½ΠΎΠΉ ΠΈΠ½Π΄ΡΠΊΡΠΈΠΈ Π€Π°ΡΠ°Π΄Π΅Ρ E = n(dΞ¦/dt). ΠΠ· 5) E = -L(dI/dt). Π’Π°ΠΊΠΈΠΌ ΠΎΠ±ΡΠ°Π·ΠΎΠΌ, L(dI/dt) = n(dΞ¦/dt), LdI = ndΞ¦ ΠΈ L = (nΞ¦)/I.
ΠΡΠ»ΠΈ ΡΠΎΠ»Π΅Π½ΠΎΠΈΠ΄ ΠΈΠ· n Π²ΠΈΡΠΊΠΎΠ² ΠΈΠΌΠ΅Π΅Ρ ΠΏΡΠΎΠ½ΠΈΡΠ°Π΅ΠΌΠΎΡΡΡ ΡΠ΅ΡΠ΄Π΅ΡΠ½ΠΈΠΊΠ° Β΅, Π΄Π»ΠΈΠ½Ρ l ΠΈ ΠΏΠΎΠΏΠ΅ΡΠ΅ΡΠ½ΠΎΠ΅ ΡΠ΅ΡΠ΅Π½ΠΈΠ΅ ΠΏΠ»ΠΎΡΠ°Π΄Ρ Π, Π΅Π³ΠΎ ΡΠΎΠ±ΡΡΠ²Π΅Π½Π½Π°Ρ ΠΈΠ½Π΄ΡΠΊΡΠΈΠ²Π½ΠΎΡΡΡ ΡΠ°Π²Π½Π°
ΠΡΠ²ΠΎΠ΄. Ξ¦ = BA, B = (ΠΌΠΊΠ½I)/ l ΠΈ
ΠΠ°ΡΠ°ΡΡΠ°Π½ΠΈΠ΅ ΠΈ Π·Π°ΡΡΡ Π°Π½ΠΈΠ΅ ΡΠΎΠΊΠ° Π² ΡΠ΅ΠΏΡ, ΡΠΎΠ΄Π΅ΡΠΆΠ°ΡΠ°Ρ ΠΈΠ½Π΄ΡΠΊΡΠΈΠ²Π½ΠΎΡΡΡ ΠΈ ΡΠΎΠΏΡΠΎΡΠΈΠ²Π»Π΅Π½ΠΈΠ΅. ΠΠΎΠ³Π΄Π° ΠΏΠ΅ΡΠ΅ΠΊΠ»ΡΡΠ°ΡΠ΅Π»Ρ Π·Π°ΠΌΠΊΠ½ΡΡ Π½Π° ΡΠ΅ΠΏΡ, ΡΠΎΠ΄Π΅ΡΠΆΠ°ΡΠ°Ρ ΠΈΠ½Π΄ΡΠΊΡΠΈΠ²Π½ΠΎΡΡΡ, ΡΠΎΠΊ Π½Π΅ ΠΏΠΎΠ΄Π½ΠΈΠΌΠ΅ΡΡΡ Π΄ΠΎ ΡΠ²ΠΎΠ΅Π³ΠΎ ΠΊΠΎΠ½Π΅ΡΠ½ΠΎΠ³ΠΎ ΡΡΠ°ΡΠΈΠΎΠ½Π°ΡΠ½ΠΎΠ³ΠΎ Π·Π½Π°ΡΠ΅Π½ΠΈΡ Π½Π΅ΠΏΠΎΡΡΠ΅Π΄ΡΡΠ²Π΅Π½Π½ΠΎ ΠΈΠ·-Π·Π° ΠΎΠ±ΡΠ°ΡΠ½ΠΎΠΉ ΠΠΠ‘ ΠΈΠ½Π΄ΡΠΊΡΠΈΠ²Π½ΠΎΡΡΡ, Π½ΠΎ Π±ΡΠ΄Π΅Ρ ΡΠ°ΡΡΠΈ ΡΠΎ ΡΠΊΠΎΡΠΎΡΡΡΡ, Π·Π°Π²ΠΈΡΠΈΡ ΠΎΡ ΠΈΠ½Π΄ΡΠΊΡΠΈΠ²Π½ΠΎΡΡΠΈ ΠΈ ΡΠΎΠΏΡΠΎΡΠΈΠ²Π»Π΅Π½ΠΈΡ ΡΡ Π΅ΠΌΠ°. ΠΠ°Π½Π° ΠΏΠΎΡΠ»Π΅Π΄ΠΎΠ²Π°ΡΠ΅Π»ΡΠ½Π°Ρ ΡΠ΅ΠΏΡ, ΡΠΎΡΡΠΎΡΡΠ°Ρ Π±Π°ΡΠ°ΡΠ΅ΠΈ ΠΠΠ‘ V, Π±Π΅Π· ΡΠΎΠΏΡΠΎΡΠΈΠ²Π»Π΅Π½ΠΈΡ ΠΈΠ½Π΄ΡΠΊΡΠΎΡ L ΠΈ Π½Π΅ΠΈΠ½Π΄ΡΠΊΡΠΈΠ²Π½ΠΎΠ΅ ΡΠΎΠΏΡΠΎΡΠΈΠ²Π»Π΅Π½ΠΈΠ΅ R ΠΊΠ°ΠΊ ΠΏΠΎΠΊΠ°Π·Π°Π½ΠΎ Π½Π° ΡΠΈΡ. 9, ΡΠ΅ΠΊΡΡΠΈΠΉ i Π² ΠΌΠΎΠΌΠ΅Π½Ρ Π²ΡΠ΅ΠΌΠ΅Π½ΠΈ t ΠΎΠΏΡΠ΅Π΄Π΅Π»ΡΠ΅ΡΡΡ ΠΊΠ°ΠΊ
, Π³Π΄Π΅ I Ρ β ΡΠΎΠΊ Π² ΡΡΡΠ°Π½ΠΎΠ²ΠΈΠ²ΡΠ΅ΠΌΡΡ ΡΠ΅ΠΆΠΈΠΌΠ΅, Π·Π°Π΄Π°Π½Π½ΡΠΉ I Ρ = V/R ΠΈ Π·Π°ΠΌΡΠΊΠ°Π½ΠΈΠ΅ ΠΊΠ»ΡΡΠ° ΠΏΡΠΎΠΈΡΡ ΠΎΠ΄ΠΈΡ Π² ΠΌΠΎΠΌΠ΅Π½Ρ Π²ΡΠ΅ΠΌΠ΅Π½ΠΈ t = 0.
ΠΠΎΠΊΠ°Π·Π°ΡΠ΅Π»ΡΡΡΠ²ΠΎ
ΠΡΠ°ΡΠΈΠΊ ΡΡΠΎΠ³ΠΎ ΡΡΠ°Π²Π½Π΅Π½ΠΈΡ ΠΏΠΎΠΊΠ°Π·Π°Π½ Π½Π° ΡΠΈΡ. 10. (Π°). Π’ΠΎΠΊ Π±ΡΡΡΡΠΎ Π½Π°ΡΠ°ΡΡΠ°Π΅Ρ, Π° Π·Π°ΡΠ΅ΠΌ Π±ΠΎΠ»Π΅Π΅ ΠΌΠ΅Π΄Π»Π΅Π½Π½ΠΎ, Π°ΡΠΈΠΌΠΏΡΠΎΡΠΈΡΠ΅ΡΠΊΠΈ ΠΏΡΠΈΠ±Π»ΠΈΠΆΠ°ΡΡΡ ΠΊ ΡΠΈΠ½Π°Π»Ρ Π·Π½Π°ΡΠ΅Π½ΠΈΠ΅ I Ρ = V/R.
ΠΠΎΡΡΠΎΡΠ½Π½Π°Ρ Π²ΡΠ΅ΠΌΠ΅Π½ΠΈ. ΠΠΎΡΡΠΎΡΠ½Π½Π°Ρ Π²ΡΠ΅ΠΌΠ΅Π½ΠΈ ΡΠ΅ΠΏΠΈ ΠΎΠΏΡΠ΅Π΄Π΅Π»ΡΠ΅ΡΡΡ ΠΊΠ°ΠΊ ΠΌΠΎΠΌΠ΅Π½Ρ Π²ΡΠ΅ΠΌΠ΅Π½ΠΈ, Π² ΠΊΠΎΡΠΎΡΡΠΉ Rt/L =1, ΠΈΠ»ΠΈ ΠΊΠΎΠ³Π΄Π°
ΠΠΎΠ³Π΄Π° t =L/R, ΡΠΎΠΊ i ΡΠΎΡΡΠ°Π²Π»ΡΠ΅Ρ ΠΎΠΊΠΎΠ»ΠΎ 63% ΠΎΡ Π΅Π³ΠΎ ΠΊΠΎΠ½Π΅ΡΠ½ΠΎΠ΅ Π·Π½Π°ΡΠ΅Π½ΠΈΠ΅ I s . ΠΠ»Ρ ΡΠ΅ΠΏΠΈ Ρ Π·Π°Π΄Π°Π½Π½ΡΠΌ ΡΠΎΠΏΡΠΎΡΠΈΠ²Π»Π΅Π½ΠΈΠ΅, Π²ΡΠ΅ΠΌΡ, Π½Π΅ΠΎΠ±Ρ ΠΎΠ΄ΠΈΠΌΠΎΠ΅ Π΄Π»Ρ Π΄ΠΎΡΡΠΈΠΆΠ΅Π½ΠΈΡ ΡΡΠΎΠ³ΠΎ Π·Π½Π°ΡΠ΅Π½ΠΈΡ ΡΠ΅ΠΌ Π΄Π»ΠΈΠ½Π½Π΅Π΅, ΡΠ΅ΠΌ Π±ΠΎΠ»ΡΡΠ΅ ΠΈΠ½Π΄ΡΠΊΡΠΈΠ²Π½ΠΎΡΡΡ.
Π€ΠΎΡΠΌΡΠ»Π° Π·Π°ΡΡΡ Π°ΡΡΠ΅Π³ΠΎ ΡΠΎΠΊΠ°. Π΅ΡΠ»ΠΈ Π΅ΡΡΡ ΡΡΡΠ°Π½ΠΎΠ²ΠΈΠ²ΡΠΈΠΉΡΡ ΡΠΎΠΊ I Ρ Π² ΡΠ΅ΠΏΠΈ ΡΠΈΡ. 9 ΠΈ ΠΏΠ΅ΡΠ΅ΠΊΠ»ΡΡΠ°ΡΠ΅Π»Ρ ΠΎΡΠΊΡΡΡ, ΡΠΏΠ°Π΄ ΡΠΎΠΊΠ° ΠΏΠΎΠΊΠ°Π·Π°Π½ΠΎ Π½Π° ΡΠΈΡ. 10(Π±). ΠΡΠΎ ΡΠΎΡΠ½Π°Ρ ΠΎΠ±ΡΠ°ΡΠ½Π°Ρ ΡΡΠΎΡΠΎΠ½Π° Π ΠΈΡ. 10(Π°). Π€ΠΎΡΠΌΡΠ»Π° ΡΠ°ΡΠΏΠ°Π΄Π°: Β Β Β Β Β Β Β Β
ΠΠΎΡΡΠΎΡΠ½Π½Π°Ρ Π²ΡΠ΅ΠΌΠ΅Π½ΠΈ L/R β ΡΡΠΎ Π²ΡΠ΅ΠΌΡ, Π² ΡΠ΅ΡΠ΅Π½ΠΈΠ΅ ΠΊΠΎΡΠΎΡΠΎΠ³ΠΎ ΡΠΎΠΊ ΡΠΌΠ΅Π½ΡΡΠ°Π΅ΡΡΡ Π΄ΠΎ 1/e ΠΎΡ Π΅Π³ΠΎ ΠΏΠ΅ΡΠ²ΠΎΠ½Π°ΡΠ°Π»ΡΠ½ΠΎΠ³ΠΎ Π·Π½Π°ΡΠ΅Π½ΠΈΡ.
ΠΠ½Π΅ΡΠ³ΠΈΡ, Π½Π°ΠΊΠΎΠΏΠ»Π΅Π½Π½Π°Ρ Π² ΠΈΠ½Π΄ΡΠΊΡΠΎΡΠ΅. ΠΠ½Π΅ΡΠ³ΠΈΡ, Π·Π°ΠΏΠ°ΡΠ΅Π½Π½Π°Ρ Π² ΠΈΠ½Π΄ΡΠΊΡΠΎΡΠ΅ (ΡΠΎΠ»Π΅Π½ΠΎΠΈΠ΄Π΅, ΡΠΎΡΠΎΠΈΠ΄Π΅ ΠΈ Ρ. ΠΏΡΠΈ ΡΡΡΠ°Π½ΠΎΠ²ΠΈΠ²ΡΠ΅ΠΌΡΡ ΡΠΎΠΊΠ΅ Π² Π½Π΅ΠΌ
ΠΠΎΠΊΠ°Π·Π°ΡΠ΅Π»ΡΡΡΠ²ΠΎ
ΠΠ·Π°ΠΈΠΌΠ½Π°Ρ ΠΈΠ½Π΄ΡΠΊΡΠΈΠ²Π½ΠΎΡΡΡ. ΠΠΎΠ³Π΄Π° ΡΠΎΠΊ Π² ΠΌΠ΅Π½ΡΠ΅ΡΡΡ ΠΏΠ΅ΡΠ²ΠΈΡΠ½Π°Ρ ΡΠ΅ΠΏΡ (Π½Π°ΠΏΡΠΈΠΌΠ΅Ρ, ΠΏΠ΅ΡΠ²ΠΈΡΠ½Π°Ρ ΠΊΠ°ΡΡΡΠΊΠ°), Π² ΡΠΎΡΠ΅Π΄Π½Π΅ΠΉ Π²ΡΠΎΡΠΈΡΠ½ΠΎΠΉ ΠΎΠ±ΠΌΠΎΡΠΊΠ΅ ΠΈΠ½Π΄ΡΡΠΈΡΡΠ΅ΡΡΡ ΠΠΠ‘ ΡΠ΅ΠΏΡ (ΠΈΠ»ΠΈ Π²ΡΠΎΡΠΈΡΠ½Π°Ρ ΠΊΠ°ΡΡΡΠΊΠ°), ΠΊΠΎΡΠΎΡΠ°Ρ Π²Π·Π°ΠΈΠΌΠΎΡΠ²ΡΠ·Π°Π½Π° Π»ΡΠ±ΠΎΠΉ ΡΠ°ΡΡΡΡ ΠΏΠ΅ΡΠ²ΠΈΡΠ½ΠΎΠ³ΠΎ ΠΏΠΎΡΠΎΠΊΠ°. ΠΠ½Π΄ΡΡΠΈΡΠΎΠ²Π°Π½Π½ΡΠΉ Π²ΡΠΎΡΠΈΡΠ½Π°Ρ ΠΠΠ‘ E 2 ΠΏΡΠΎΠΏΠΎΡΡΠΈΠΎΠ½Π°Π»ΡΠ½Π° ΡΠΊΠΎΡΠΎΡΡΠΈ Π²ΡΠ΅ΠΌΠ΅Π½ΠΈ ΠΈΠ·ΠΌΠ΅Π½Π΅Π½ΠΈΡ ΠΏΠ΅ΡΠ²ΠΈΡΠ½ΠΎΠ³ΠΎ ΡΠΎΠΊΠ°, d I 1 /dt
, Π³Π΄Π΅ M β ΠΏΠΎΡΡΠΎΡΠ½Π½Π°Ρ, Π½Π°Π·ΡΠ²Π°Π΅ΠΌΠ°Ρ Π²Π·Π°ΠΈΠΌΠ½ΠΎΠΉ ΠΈΠ½Π΄ΡΠΊΡΠΈΠ²Π½ΠΎΡΡΡΡ ΡΠΈΡΡΠ΅ΠΌΠ°. ΠΡΠ»ΠΈ E Π² Π²ΠΎΠ»ΡΡΠ°Ρ ΠΈ ββd I 1 /dt Π² Π°ΠΌΠΏΠ΅Ρ/Ρ, M Π² ΠΠ΅Π½ΡΠΈ.
ΠΠ°ΡΡΠ΄ ΠΈ ΡΠ°Π·ΡΡΠ΄ ΠΊΠΎΠ½Π΄Π΅Π½ΡΠ°ΡΠΎΡΠ° ΡΠ΅ΡΠ΅Π· ΡΠ΅Π·ΠΈΡΡΠΎΡ. ΠΠΎΠ³Π΄Π° ΠΏΠ΅ΡΠ΅ΠΊΠ»ΡΡΠ°ΡΠ΅Π»Ρ Π·Π°ΠΌΠΊΠ½ΡΡ Π½Π° ΡΠ΅ΠΏΡ, ΡΠΎΠ΄Π΅ΡΠΆΠ°ΡΠ°Ρ Π΅ΠΌΠΊΠΎΡΡΡ ΠΈ ΡΠΎΠΏΡΠΎΡΠΈΠ²Π»Π΅Π½ΠΈΠ΅, Π·Π°ΡΡΠ΄ Π½Π° ΠΊΠΎΠ½Π΄Π΅Π½ΡΠ°ΡΠΎΡΠ΅ Π½Π΅ Π΄ΠΎΡΡΠΈΠ³Π°Π΅Ρ ΡΠ²ΠΎΠ΅Π³ΠΎ ΠΊΠΎΠ½Π΅ΡΠ½ΠΎΠ³ΠΎ Π·Π½Π°ΡΠ΅Π½ΠΈΡ ΡΡΠ°Π·Ρ, Π½ΠΎ ΠΏΡΠΈΠ±Π»ΠΈΠΆΠ°Π΅ΡΡΡ ΠΊ ΡΡΠΎΠΌΡ Π·Π½Π°ΡΠ΅Π½ΠΈΡ Π² ΡΠΎΠΌ ΠΆΠ΅ ΡΠ°ΠΊ ΠΆΠ΅, ΠΊΠ°ΠΊ ΡΠΎΠΊ Π² ΡΠ΅ΠΏΠΈ, ΡΠΎΠ΄Π΅ΡΠΆΠ°ΡΠ΅ΠΉ ΠΈΠ½Π΄ΡΠΊΡΠΈΠ²Π½ΠΎΡΡΡ ΠΈ ΡΠΎΠΏΡΠΎΡΠΈΠ²Π»Π΅Π½ΠΈΠ΅.
Π ΡΠ΅ΠΏΠΈ ΡΠΈΡ. 11, ΡΠΎΠ΄Π΅ΡΠΆΠ°ΡΠ΅ΠΉ ΡΠΎΠΏΡΠΎΡΠΈΠ²Π»Π΅Π½ΠΈΠ΅ R, Π΅ΠΌΠΊΠΎΡΡΡ C ΠΈ ΠΈΡΡΠΎΡΠ½ΠΈΠΊ ΠΠΠ‘ E, ΠΏΡΡΡΡ q ΠΏΡΠ΅Π΄ΡΡΠ°Π²Π»ΡΠ΅Ρ ΡΠΎΠ±ΠΎΠΉ Π·Π°ΡΡΠ΄ ΠΊΠΎΠ½Π΄Π΅Π½ΡΠ°ΡΠΎΡΠ° Π² ΠΎΠΏΡΠ΅Π΄Π΅Π»Π΅Π½Π½ΡΠΉ ΠΌΠΎΠΌΠ΅Π½Ρ ΠΏΠΎΡΠ»Π΅ ΠΏΠ΅ΡΠ΅ΠΊΠ»ΡΡΠ°ΡΠ΅Π»Ρ S Π·Π°ΠΌΠΊΠ½ΡΡ, ΠΈ ΠΏΡΡΡΡ i Π±ΡΠ΄Π΅Ρ ΡΠΎΠΊΠΎΠΌ Π² ΡΠ΅ΠΏΠΈ Π² ΡΠΎΡ ΠΌΠΎΠΌΠ΅Π½Ρ. Π’ΠΎΠ³Π΄Π° ΡΡΠΌΠΌΠ° Π½Π°ΡΠΈΡΠ»Π΅Π½ΠΈΡ Π½Π° ΠΊΠΎΠ½Π΄Π΅Π½ΡΠ°ΡΠΎΡ Π² Π½Π΅ΠΊΠΎΡΠΎΡΡΠΉ ΠΏΠΎΡΠ»Π΅Π΄ΡΡΡΠΈΠΉ ΠΌΠΎΠΌΠ΅Π½Ρ Π²ΡΠ΅ΠΌΠ΅Π½ΠΈ t ΠΎΠΏΡΠ΅Π΄Π΅Π»ΡΠ΅ΡΡΡ ΠΊΠ°ΠΊ
, Π³Π΄Π΅ Q = C E.
Β
ΠΠΎΠΊΠ°Π·Π°ΡΠ΅Π»ΡΡΡΠ²ΠΎ
Β
ΠΡΠ°ΡΠΈΠΊ ΡΡΠ°Π²Π½Π΅Π½ΠΈΡ ΠΏΠΎΠΊΠ°Π·Π°Π½ Π½Π° ΡΠΈΡ. 12(Π°). ΠΠΎΡΡΠΎΡΠ½Π½Π°Ρ Π²ΡΠ΅ΠΌΠ΅Π½ΠΈ ΡΠ΅ΠΏΠΈ ΡΠ°Π²Π½Π° RC.
Π€ΠΎΡΠΌΡΠ»Π° Π΄Π»Ρ Π²ΡΠΏΠΈΡΠΊΠΈ. ΠΡΠ»ΠΈ ΠΊΠΎΠ½Π΄Π΅Π½ΡΠ°ΡΠΎΡ ΠΏΠ΅ΡΠ²ΠΎΠ½Π°ΡΠ°Π»ΡΠ½ΠΎ Π·Π°ΡΡΠΆΠ΅Π½, Π° Π·Π°ΡΠ΅ΠΌ ΡΠ°Π·ΡΡΠΆΠ΅Π½ ΡΠ΅ΡΠ΅Π· ΡΠΎΠΏΡΠΎΡΠΈΠ²Π»Π΅Π½ΠΈΠ΅ R, Π·Π°ΡΡΠ΄ ΡΠΌΠ΅Π½ΡΡΠ°Π΅ΡΡΡ ΡΠΎ Π²ΡΠ΅ΠΌΠ΅Π½Π΅ΠΌ ΠΏΠΎ
ΠΡΠ°ΡΠΈΠΊ ΠΏΠΎΠΊΠ°Π·Π°Π½ Π½Π° ΡΠΈΡ. 12(b).
ΠΠ»Π΅ΠΊΡΡΠΈΡΠ΅ΡΠΊΠΈΠ΅ ΠΊΠΎΠ»Π΅Π±Π°Π½ΠΈΡ Π² LC-ΠΊΠΎΠ½ΡΡΡΠ΅. ΠΠ° ΡΠΈΡ. 13 (Π°) ΠΏΠΎΠΊΠ°Π·Π°Π½Π° ΡΠ΅ΠΏΡ, ΡΠΎΠ΄Π΅ΡΠΆΠ°ΡΠ°Ρ Π·Π°ΡΡΠΆΠ΅Π½Π½ΡΠΉ ΠΊΠΎΠ½Π΄Π΅Π½ΡΠ°ΡΠΎΡ C, ΠΊΠ°ΡΡΡΠΊΠ° ΠΈΠ½Π΄ΡΠΊΡΠΈΠ²Π½ΠΎΡΡΠΈ L ΠΏΡΠ΅Π½Π΅Π±ΡΠ΅ΠΆΠΈΠΌΠΎ ΠΌΠ°Π»ΠΎΠ³ΠΎ ΡΠΎΠΏΡΠΎΡΠΈΠ²Π»Π΅Π½ΠΈΡ ΠΈ ΠΏΠ΅ΡΠ΅ΠΊΠ»ΡΡΠ°ΡΠ΅Π»Ρ S. Π Π°ΡΡΠΌΠΎΡΡΠΈΠΌ ΡΠ΅ΠΏΠ΅ΡΡ ΠΊΠΎΠ»Π΅Π±Π°ΡΠ΅Π»ΡΠ½ΠΎΠ΅ ΠΏΠΎΠ²Π΅Π΄Π΅Π½ΠΈΠ΅, ΠΊΠΎΡΠΎΡΠΎΠ΅ Π²ΠΎΠ·Π½ΠΈΠΊΠ°Π΅Ρ, ΠΊΠΎΠ³Π΄Π° ΠΊΠΎΠ½Π΄Π΅Π½ΡΠ°ΡΠΎΡ ΡΠ°Π·ΡΡΠΆΠ°Π΅ΡΡΡ ΠΏΡΠΈ Π·Π°ΠΌΡΠΊΠ°Π½ΠΈΠΈ ΠΊΠ»ΡΡΠ°. Π ΠΌΠΎΠΌΠ΅Π½Ρ Π·Π°ΠΌΡΠΊΠ°Π½ΠΈΡ ΠΏΠ΅ΡΠ΅ΠΊΠ»ΡΡΠ°ΡΠ΅Π»Ρ, ΠΊΠΎΠ½Π΄Π΅Π½ΡΠ°ΡΠΎΡ Π½Π°ΡΠΈΠ½Π°Π΅Ρ ΡΠ°Π·ΡΡΠΆΠ°ΡΡΡΡ ΡΠ΅ΡΠ΅Π· ΠΈΠ½Π΄ΡΠΊΡΠΎΡ. ΠΠ°ΠΊ ΡΡΠΎ ΡΠ°Π·ΡΡΠΆΠ°Π΅ΡΡΡ, ΡΠΎΠ·Π΄Π°Π΅ΡΡΡ ΠΌΠ°Π³Π½ΠΈΡΠ½ΠΎΠ΅ ΠΏΠΎΠ»Π΅ Π² ΠΈΠ½Π΄ΡΠΊΡΠΎΡΠ΅ ΠΈ Ρ ΡΠ°Π½ΠΈΡΡΡ ΡΠ½Π΅ΡΠ³ΠΈΡ ΠΊΠΎΠ½Π΄Π΅Π½ΡΠ°ΡΠΎΡΠ° ΡΠ°Π²Π½Π° ΠΏΠ΅ΡΠ΅Π΄Π°Π΅ΡΡΡ Π½Π° ΠΈΠ½Π΄ΡΠΊΡΠΎΡ. ΠΠΎΠ³Π΄Π° ΠΎΠ½ ΠΏΠΎΠ»Π½ΠΎΡΡΡΡ ΡΠ°Π·ΡΡΠ΄ΠΈΠ»ΡΡ, Ρ. ΡΠ°Π·Π½ΠΎΡΡΡ ΠΏΠΎΡΠ΅Π½ΡΠΈΠ°Π»ΠΎΠ² ΠΌΠ΅ΠΆΠ΄Ρ Π΅Π³ΠΎ ΡΠ΅ΡΠΌΠΈΠ½Π°Π»ΠΎΠ² ΡΠΌΠ΅Π½ΡΡΠΈΠ»ΠΎΡΡ Π΄ΠΎ Π½ΡΠ»Ρ. Π‘ΠΌ. ΡΠΈΡ. 13 (Π±). ΠΠ°Π³Π½ΠΈΡΠ½ΡΠΉ ΠΏΠΎΠ»Π΅ ΠΈΠ½Π΄ΡΠΊΡΠΎΡΠ° ΡΠ΅ΠΏΠ΅ΡΡ Π½Π°ΡΠΈΠ½Π°Π΅ΡΡΡ ΡΠΌΠ΅Π½ΡΡΠ°Π΅ΡΡΡ, ΠΈΠ½Π΄ΡΡΠΈΡΡΡ ΠΠΠ‘ Π² ΠΈΠ½Π΄ΡΠΊΡΠΎΡ Π² ΡΠΎΠΌ ΠΆΠ΅ Π½Π°ΠΏΡΠ°Π²Π»Π΅Π½ΠΈΠΈ, ΡΡΠΎ ΠΈ ΡΠ»Π΅ΠΊΡΡΠΈΡΠ΅ΡΠΊΠΈΠΉ ΡΠΎΠΊ. ΠΠ»Π΅ΠΊΡΡΠΈΡΠ΅ΡΠΊΠΈΠΉ ΡΠΎΠΊ ΡΠ»Π΅Π΄ΠΎΠ²Π°ΡΠ΅Π»ΡΠ½ΠΎ, ΡΠΎΡ ΡΠ°Π½ΡΠ΅ΡΡΡ, Π½ΠΎ Ρ ΡΠΌΠ΅Π½ΡΡΠ°Π΅ΡΡΡ ΠΏΠΎ Π²Π΅Π»ΠΈΡΠΈΠ½Π΅ Π΄ΠΎ ΡΠ΅Ρ ΠΏΠΎΡ, ΠΏΠΎΠΊΠ° ΠΌΠ°Π³Π½ΠΈΡΠ½ΠΎΠ΅ ΠΏΠΎΠ»Π΅ ΠΈΡΡΠ΅Π·Π»ΠΎ ΠΈ ΠΊΠΎΠ½Π΄Π΅Π½ΡΠ°ΡΠΎΡ Π±ΡΠ» Π·Π°ΡΡΠΆΠ΅Π½ Π² Π½Π°ΠΏΡΠ°Π²Π»Π΅Π½ΠΈΠΈ, ΠΏΡΠΎΡΠΈΠ²ΠΎΠΏΠΎΠ»ΠΎΠΆΠ½ΠΎΠΌ Π΅Π³ΠΎ ΠΏΠ΅ΡΠ²ΠΎΠ½Π°ΡΠ°Π»ΡΠ½ΠΎΠΉ ΠΏΠΎΠ»ΡΡΠ½ΠΎΡΡΠΈ, ΠΊΠ°ΠΊ ΠΏΠΎΠΊΠ°Π·Π°Π½ΠΎ Π½Π° ΡΠΈΡ. 13 (Ρ). Π’Π΅ΠΏΠ΅ΡΡ ΠΏΡΠΎΡΠ΅ΡΡ ΠΏΠΎΠ²ΡΠΎΡΡΠ΅ΡΡΡ Π² ΠΎΠ±ΡΠ°ΡΠ½ΠΎΠΌ Π½Π°ΠΏΡΠ°Π²Π»Π΅Π½ΠΈΠΈ ΠΈ ΠΏΡΠΈ ΠΎΡΡΡΡΡΡΠ²ΠΈΠΈ ΡΠ½Π΅ΡΠ³ΠΈΠΈ. ΠΏΠΎΡΠ΅ΡΡ Π·Π°ΡΡΠ΄Ρ Π½Π° ΠΊΠΎΠ½Π΄Π΅Π½ΡΠ°ΡΠΎΡΠ΅ Π±ΡΠ΄ΡΡ ΡΠΊΠ°ΡΠΊΠ°ΠΌΠΈ Π²Π²Π΅ΡΡ ΠΈ Π²Π½ΠΈΠ· Π½Π° Π½Π΅ΠΎΠΏΡΠ΅Π΄Π΅Π»Π΅Π½Π½ΡΠΉ ΡΡΠΎΠΊ.
Β
Π§Π°ΡΡΠΎΡΠ° ΡΠ»Π΅ΠΊΡΡΠΈΡΠ΅ΡΠΊΠΈΡ ΠΊΠΎΠ»Π΅Π±Π°Π½ΠΈΠΉ ΡΠ΅ΠΏΠΈ, ΡΠΎΠ΄Π΅ΡΠΆΠ°ΡΠ΅ΠΉ ΠΈΠ½Π΄ΡΠΊΡΠΈΠ²Π½ΠΎΡΡΡ ΠΈ Π΅ΠΌΠΊΠΎΡΡΡ ΡΠΎΠ»ΡΠΊΠΎ Π²ΡΡΠΈΡΠ»ΡΠ΅ΡΡΡ ΡΠΎΡΠ½ΠΎ ΡΠ°ΠΊ ΠΆΠ΅, ΠΊΠ°ΠΊ ΡΠ°ΡΡΠΎΡΠ° ΠΊΠΎΠ»Π΅Π±Π°Π½ΠΈΠΉ ΡΠ΅Π»Π° ΠΌΠ°ΡΡΠΎΠΉ m ΠΏΠΎΠ΄Π²Π΅ΡΠ΅Π½ Π½Π° ΠΏΡΡΠΆΠΈΠ½Π΅ Ρ ΠΏΠΎΡΡΠΎΡΠ½Π½ΠΎΠΉ ΡΠΈΠ»Ρ k. Π§Π°ΡΡΠΎΡΠ° ΠΊΠΎΠ»Π΅Π±Π°Π½ΠΈΠΉ ΡΠ΅Π»Π° Π½Π° Π²Π΅ΡΠ½Π°
, Π° ΡΠ°ΡΡΠΎΡΠ° ΡΠ»Π΅ΠΊΡΡΠΈΡΠ΅ΡΠΊΠΈΡ ΠΊΠΎΠ»Π΅Π±Π°Π½ΠΈΠΉ ΡΠ°Π²Π½Π°
.Β Β Β Β Β Β Β Β Β Β Β Β Β Β Β Β Β Β Β Β Β Β Β Β Β Β Β Β Β Β Β Β Β Β Β Β Β Β Β Β Β Β Β Β Β Β
ΠΡΠ° ΡΠ°ΡΡΠΎΡΠ° Π½Π°Π·ΡΠ²Π°Π΅ΡΡΡ Π΅ΡΡΠ΅ΡΡΠ²Π΅Π½Π½ΠΎΠΉ ΡΠ°ΡΡΠΎΡΠ° ΡΠ΅ΠΏΠΈ LC.
Π ΡΡΠΎΠΉ Π·Π°Π΄Π°ΡΠ΅ ΠΈ Π²ΠΎ ΠΌΠ½ΠΎΠ³ΠΈΡ ΠΏΡΠΎΠ±Π»Π΅ΠΌΡ ΠΏΠΎ ΡΠΈΠ·ΠΈΠΊΠ΅ Π΅ΡΡΡ ΡΠΈΠ»ΡΠ½Π°Ρ ΠΏΠ°ΡΠ°Π»Π»Π΅Π»ΠΈΠ·ΠΌ ΠΌΠ΅ΠΆΠ΄Ρ
ΠΌΠ΅Ρ Π°Π½ΠΈΡΠ΅ΡΠΊΠΈΠΉ ΡΠΈΡΡΠ΅ΠΌΡ, Π°ΠΊΡΡΡΠΈΡΠ΅ΡΠΊΠΈΠ΅ ΡΠΈΡΡΠ΅ΠΌΡ ΠΈ ΡΠ»Π΅ΠΊΡΡΠΈΡΠ΅ΡΠΊΠΈΠ΅ ΡΠΈΡΡΠ΅ΠΌΡ Π² ΡΡΠΎΠΌ ΡΠ°Π·Π»ΠΈΡΠ½ΠΎΠΌ ΠΏΡΠΎΠ±Π»Π΅ΠΌΡ ΡΠ²ΠΎΠ΄ΡΡΡΡ ΠΊ ΡΠΎΠΌΡ ΠΆΠ΅ Π½Π°Π±ΠΎΡΡ Π΄ΠΈΡΡΠ΅ΡΠ΅Π½ΡΠΈΠ°Π»ΡΠ½ΡΠ΅ ΡΡΠ°Π²Π½Π΅Π½ΠΈΡ ΠΈ ΡΠ΅ΡΠ΅Π½ΠΈΡ ΡΠ΅ ΠΆΠ΅, Π·Π° ΠΈΡΠΊΠ»ΡΡΠ΅Π½ΠΈΠ΅ΠΌ Π·Π½Π°ΡΠ΅Π½ΠΈΡ ΠΏΠ΅ΡΠ΅ΠΌΠ΅Π½Π½ΡΡ . ΠΠΎΡΠΎΠΌΡ ΡΡΠΎ ΡΡΠΎΠ³ΠΎ ΠΏΠ°ΡΠ°Π»Π»Π΅Π»ΠΈΠ·ΠΌΠ°, ΠΈΠ½ΠΎΠ³Π΄Π° ΠΌΠΎΠΆΠ½ΠΎ ΡΠ΅ΡΠΈΡΡ ΡΠ»ΠΎΠΆΠ½ΡΡ ΠΌΠ΅Ρ Π°Π½ΠΈΡΠ΅ΡΠΊΠ°Ρ ΠΈΠ»ΠΈ Π°ΠΊΡΡΡΠΈΡΠ΅ΡΠΊΠ°Ρ ΠΏΡΠΎΠ±Π»Π΅ΠΌΠ° ΡΡΡΠ°Π½ΠΎΠ²ΠΊΠ° Π°Π½Π°Π»ΠΎΠ³ΠΈΡΠ½ΠΎΠΉ ΡΠ»Π΅ΠΊΡΡΠΈΠΊΠΈ ΡΠ΅ΠΏΠ΅ΠΉ ΠΈ ΠΈΠ·ΠΌΠ΅ΡΠ΅Π½ΠΈΡ ΡΠΎΠΊΠΎΠ² ΠΈ Π½Π°ΠΏΡΡΠΆΠ΅Π½ΠΈΡ, ΡΠΎΠΎΡΠ²Π΅ΡΡΡΠ²ΡΡΡΠΈΠ΅ ΠΆΠ΅Π»Π°Π΅ΠΌΡΠ΅ ΠΌΠ΅Ρ Π°Π½ΠΈΡΠ΅ΡΠΊΠΈΠ΅ ΠΈ Π°ΠΊΡΡΡΠΈΡΠ΅ΡΠΊΠΈΠ΅ Π½Π΅ΠΈΠ·Π²Π΅ΡΡΠ½ΡΠ΅.
Β Β Β Β Β Β Β Β Β Β Β Β Β Β Β Β Β Β Β Β Β Β Β Β Β Β Β Β Β Β Β Β Β Β Β Β Β Β Β Β Β Β Β Β Β Β
Π‘ ΡΠ½Π΅ΡΠ³Π΅ΡΠΈΡΠ΅ΡΠΊΠΎΠΉ ΡΠΎΡΠΊΠΈ Π·ΡΠ΅Π½ΠΈΡ ΠΊΠΎΠ»Π΅Π±Π°Π½ΠΈΡ Π²ΡΡΠ΅ΡΠΊΠ°Π·Π°Π½Π½ΠΎΠΉ ΡΠ΅ΠΏΠΈ LC Π·Π°ΠΊΠ»ΡΡΠ°ΡΡΡΡ Π² ΠΏΠ΅ΡΠ΅Π΄Π°ΡΠ΅ ΡΠ½Π΅ΡΠ³ΠΈΠΈ ΠΎΠ±ΡΠ°ΡΠ½ΠΎ ΠΈ Π΄Π°Π»Π΅Π΅ ΠΎΡ ΡΠ»Π΅ΠΊΡΡΠΈΡΠ΅ΡΠΊΠΎΠ³ΠΎ ΠΏΠΎΠ»Ρ ΠΊΠΎΠ½Π΄Π΅Π½ΡΠ°ΡΠΎΡ ΠΊ ΠΌΠ°Π³Π½ΠΈΡΠ½ΠΎΠΌΡ ΠΏΠΎΠ»Ρ ΠΈΠ½Π΄ΡΠΊΡΠΎΡ Ρ ΠΏΠΎΠ»Π½ΠΎΠΉ ΡΠ½Π΅ΡΠ³ΠΈΠ΅ΠΉ ΡΠ²ΡΠ·Π°Π½Π½ΡΠΉ Ρ ΠΎΡΡΠ°Π²ΡΠ΅ΠΉΡΡ ΡΠ΅ΠΏΡΡ ΠΏΠΎΡΡΠΎΡΠ½Π½ΡΠΉ; ΡΠ²Π»Π΅Π½ΠΈΠ΅ Π°Π½Π°Π»ΠΎΠ³ΠΈΡΠ½ΠΎΠ΅ ΠΊ ΠΏΠ΅ΡΠ΅Π΄Π°ΡΠ΅ ΡΠ½Π΅ΡΠ³ΠΈΠΈ Π² ΠΊΠΎΠ»Π΅Π±Π°ΡΠ΅Π»ΡΠ½Π°Ρ ΠΌΠ΅Ρ Π°Π½ΠΈΡΠ΅ΡΠΊΠ°Ρ ΡΠΈΡΡΠ΅ΠΌΠ° ΠΈΠ· ΠΊΠΈΠ½Π΅ΡΠΈΡΠ΅ΡΠΊΠ°Ρ Π² ΠΏΠΎΡΠ΅Π½ΡΠΈΠ°Π»ΡΠ½ΡΡ ΠΈ Π½Π°ΠΎΠ±ΠΎΡΠΎΡ.
ΠΠ½Π΅ΡΠ³ΠΈΡ ΠΊΠΎΠ½Π΄Π΅Π½ΡΠ°ΡΠΎΡΠ° ΠΏΡΠΈ Π»ΡΠ±ΠΎΠΌ ΠΌΠΎΠΌΠ΅Π½Ρ ΡΠ°Π²Π΅Π½ Β½ (q 2 /C), Π° ΠΈΠ½Π΄ΡΠΊΡΠΎΡΠ° Β½ Li 2 . Π‘Π»Π΅Π΄ΠΎΠ²Π°ΡΠ΅Π»ΡΠ½ΠΎ,
Β
, Π³Π΄Π΅ q an i β ΠΌΠ³Π½ΠΎΠ²Π΅Π½Π½ΡΠ΅ Π·Π½Π°ΡΠ΅Π½ΠΈΡ, Π° Q ΠΈ I β ΠΌΠ°ΠΊΡΠΈΠΌΠ°Π»ΡΠ½ΡΠ΅ Π·Π°ΡΡΠ΄ ΠΈ ΡΠΎΠΊ, ΡΠΎΠΎΡΠ²Π΅ΡΡΡΠ²Π΅Π½Π½ΠΎ.
ΠΡΡΠ΅ΠΊΡ ΡΠΎΠΏΡΠΎΡΠΈΠ²Π»Π΅Π½ΠΈΡ Π² ΠΊΠΎΠ»Π΅Π±Π°ΡΠ΅Π»ΡΠ½ΠΎΠΌ ΠΊΠΎΠ½ΡΡΡΠ΅ Π·Π°ΠΊΠ»ΡΡΠ°Π΅ΡΡΡ Π² ΡΠΎΠΌ, ΡΡΠΎΠ±Ρ ΠΎΡΠ²ΠΎΠ΄ΠΈΡΡ ΡΠ½Π΅ΡΠ³ΠΈΡ ΠΊΠΎΠ½ΡΡΡΠ° ΠΈ ΠΏΡΠ΅ΠΎΠ±ΡΠ°Π·ΠΎΠ²Π°ΡΡ Π΅Π³ΠΎ Π² ΡΠ΅ΠΏΠ»ΠΎ. Π’Π°ΠΊΠΈΠΌ ΠΎΠ±ΡΠ°Π·ΠΎΠΌ, ΡΠΎΠΏΡΠΎΡΠΈΠ²Π»Π΅Π½ΠΈΠ΅ ΠΈΠ³ΡΠ°Π΅Ρ Π² ΡΠ»Π΅ΠΊΡΡΠΈΡΠ΅ΡΠΊΠΎΠΉ ΡΠ΅ΠΏΠΈ ΡΡ ΠΆΠ΅ ΡΠΎΠ»Ρ, ΡΡΠΎ ΠΈ ΡΡΠ΅Π½ΠΈΠ΅ Π² ΡΠ»Π΅ΠΊΡΡΠΈΡΠ΅ΡΠΊΠΎΠΉ ΡΠ΅ΠΏΠΈ. ΠΌΠ΅Ρ Π°Π½ΠΈΡΠ΅ΡΠΊΠ°Ρ ΡΠΈΡΡΠ΅ΠΌΠ°.
ΠΠ΅ΡΠ΅ΠΌΠ΅Π½Π½ΡΠΉ ΡΠΎΠΊ ΠΈ ΠΏΠΎΡΡΠΎΡΠ½Π½ΡΠΉ ΡΠΎΠΊ. Π’ΠΎΠΊ, ΠΊΠΎΡΠΎΡΡΠΉ ΡΠ΅ΡΠ΅Ρ Π² ΠΎΠ΄Π½ΠΎΠΌ Π½Π°ΠΏΡΠ°Π²Π»Π΅Π½ΠΈΠΈ Π² ΡΠ΅ΡΠ΅Π½ΠΈΠ΅ ΡΠ°ΡΡΡ ΡΠΈΠΊΠ»Π° ΠΈ Π² ΠΏΡΠΎΡΠΈΠ²ΠΎΠΏΠΎΠ»ΠΎΠΆΠ½ΠΎΠΌ Π½Π°ΠΏΡΠ°Π²Π»Π΅Π½ΠΈΠΈ Π² ΡΠ΅ΡΠ΅Π½ΠΈΠ΅ ΠΎΡΡΠ°Π»ΡΠ½ΠΎΠΉ ΡΠ°ΡΡΠΈ ΡΠΈΠΊΠ»Π° Π½Π°Π·ΡΠ²Π°Π΅ΡΡΡ ΡΠ΅ΡΠ΅Π΄ΠΎΠ²Π°Π½ΠΈΠ΅ΠΌ Π’Π΅ΠΊΡΡΠΈΠΉ. ΠΠ΅ΡΠ΅ΠΌΠ΅Π½Π½ΡΠΉ ΡΠΎΠΊ — ΡΡΠΎ ΡΠΈΠΏ ΡΠΎΠΊΠ°, ΠΊΠΎΡΠΎΡΡΠΉ ΠΎΠ±ΡΡΠ½ΠΎ ΠΏΡΠ΅Π΄ΠΎΡΡΠ°Π²Π»ΡΠ΅ΡΡΡ ΡΠ½Π΅ΡΠ³Π΅ΡΠΈΡΠ΅ΡΠΊΠΈΠΌΠΈ ΠΊΠΎΠΌΠΏΠ°Π½ΠΈΡΠΌΠΈ. ΠΠΎΠ³Π΄Π° ΡΠΎΠΊ ΡΠ΅ΡΠ΅Ρ ΡΠΎΠ»ΡΠΊΠΎ Π² ΠΎΠ΄Π½ΠΎΠΌ Π½Π°ΠΏΡΠ°Π²Π»Π΅Π½ΠΈΠΈ, ΠΎΠ½ Π½Π°Π·ΡΠ²Π°Π΅ΡΡΡ ΠΏΠΎΡΡΠΎΡΠ½Π½ΡΠΌ ΡΠΎΠΊΠΎΠΌ. ΠΡΠΎ Π²ΠΈΠ΄ ΡΠΎΠΊΠ° ΠΎΡΠ½Π°ΡΠ΅Π½ ΡΡΡ ΠΈΠΌΠΈ ΡΠ»Π΅ΠΌΠ΅Π½ΡΠ°ΠΌΠΈ ΠΈ Π°ΠΊΠΊΡΠΌΡΠ»ΡΡΠΎΡΠ½ΡΠΌΠΈ Π±Π°ΡΠ°ΡΠ΅ΡΠΌΠΈ.
ΠΠ΅Π½Π΅ΡΠ°ΡΠΎΡ ΠΏΠ΅ΡΠ΅ΠΌΠ΅Π½Π½ΠΎΠ³ΠΎ ΡΠΎΠΊΠ°. ΠΡΠ½ΠΎΠ²Π½ΠΎΠΉ ΠΏΡΠΈΠ½ΡΠΈΠΏ ΡΠ°Π±ΠΎΡΡ Π³Π΅Π½Π΅ΡΠ°ΡΠΎΡ ΠΏΠ΅ΡΠ΅ΠΌΠ΅Π½Π½ΠΎΠ³ΠΎ ΡΠΎΠΊΠ° ΠΏΠΎΠΊΠ°Π·Π°Π½ Π½Π° Π ΠΈΡ. 14(Π°). ΠΠ»ΠΎΡΠ½ΠΎ Π½Π°ΠΌΠΎΡΠ°Π½Π½ΡΠΉ ΠΏΡΡΠΌΠΎΡΠ³ΠΎΠ»ΡΠ½ΡΠΉ ΠΊΠ°ΡΡΡΠΊΠ° abcd ΠΈΠ· n Π²ΠΈΡΠΊΠΎΠ² Π²ΡΠ°ΡΠ°Π΅ΡΡΡ Π²ΠΎΠΊΡΡΠ³ ΠΎΡΠΈ OO, ΠΊΠΎΡΠΎΡΡΠΉ ΠΏΠ΅ΡΠΏΠ΅Π½Π΄ΠΈΠΊΡΠ»ΡΡΠ΅Π½ ΡΠ°Π²Π½ΠΎΠΌΠ΅ΡΠ½ΠΎΠΌΡ ΠΌΠ°Π³Π½ΠΈΡΠ½ΠΎΠ΅ ΠΏΠΎΠ»Π΅ Ρ ΠΏΠ»ΠΎΡΠ½ΠΎΡΡΡΡ ΠΏΠΎΡΠΎΠΊΠ° B. As ab ΠΈ cd ΠΊΠ°ΡΡΡΠΊΠΈ abcd ΡΠ°Π·ΡΠ΅Π·Π°ΡΡ Π»ΠΈΠ½ΠΈΠΈ ΠΌΠ°Π³Π½ΠΈΡΠ½ΠΎΠ³ΠΎ ΠΏΠΎΡΠΎΠΊΠ°, ΡΠ»Π΅ΠΊΡΡΠΈΡΠ΅ΡΠΊΠΈΠΉ ΡΠΎΠΊ ΠΏΡΠΎΠΈΠ·Π²ΠΎΠ΄ΠΈΡΡΡ Π² ΡΠΎΠΎΡΠ²Π΅ΡΡΡΠ²ΠΈΠΈ Ρ ΡΡΠ°Π²Π½Π΅Π½ΠΈΠ΅ΠΌ 2) Π²ΡΡΠ΅
Β
17)Β Β Β Β Β Β E = B l v sin ΞΈ
, Π³Π΄Π΅ l β Π΄Π»ΠΈΠ½Π° ab (ΠΈΠ»ΠΈ cd) ΠΈ ΞΈ ΡΠ³ΠΎΠ», ΠΏΠΎΠ΄ ΠΊΠΎΡΠΎΡΡΠΌ Π²Π΅ΠΊΡΠΎΡ ΡΠΊΠΎΡΠΎΡΡΠΈ v bc ΡΠΎΠ²ΠΏΠ°Π΄Π°Π΅Ρ Ρ ΠΏΠΎΠ»Π΅ΠΌ ΠΏΠΎΡΠΎΠΊΠ° B. Π‘ΠΌ. ΡΠΈΡ. 15(b). ΠΠ°ΡΡΡΠΊΠ° Π½Π°ΠΌΠΎΡΠ°Π½Π° Π½Π° ΠΆΠ΅Π»Π΅Π·Π½ΡΠΉ ΡΠΈΠ»ΠΈΠ½Π΄Ρ, Π½Π°Π·ΡΠ²Π°Π΅ΠΌΡΠΉ ΡΠΊΠΎΡΡ (ΡΠ±ΠΎΡΠΊΠ° ΠΈΠ· ΠΊΠ°ΡΡΡΠΊΠΈ ΠΈ ΡΠΈΠ»ΠΈΠ½Π΄ΡΠ° ΡΠ°ΠΊΠΆΠ΅ Π½Π°Π·ΡΠ²Π°Π΅ΡΡΡ ΡΠΊΠΎΡΠ΅ΠΌ). ΠΠ΅ΡΠΎΠ΄, Ρ ΠΏΠΎΠΌΠΎΡΡΡ ΠΊΠΎΡΠΎΡΠΎΠ³ΠΎ ΡΠ»Π΅ΠΊΡΡΠΈΡΠ΅ΡΠΊΠΈΠΉ ΡΠΎΠΊ ΠΎΡ ΠΊΠ°ΡΡΡΠΊΠΈ ΠΏΠ΅ΡΠ΅Π΄Π°Π΅ΡΡΡ Π²ΠΎ Π²Π½Π΅ΡΠ½ΡΡ ΡΠ΅ΠΏΡ ΡΠ΅ΡΠ΅Π· ΡΠΎΠΊΠΎΡΡΠ΅ΠΌΠ½ΡΠ΅ ΠΊΠΎΠ»ΡΡΠ° ΠΈ ΡΠ΅ΡΠΊΠΈ ΠΊΠ°ΠΊ ΠΏΠΎΠΊΠ°Π·Π°Π½ΠΎ Π½Π° ΡΠΈΡ. 14 (Π±). ΠΡΠ²ΠΎΠ΄Ρ ΠΊΠ°ΡΡΡΠΊΠΈ ΠΏΡΠΈΠΏΠ°ΡΠ½Ρ ΠΊ Π»Π°ΡΡΠ½Π½ΡΠΌ ΠΊΠΎΠ½ΡΠ°ΠΊΡΠ½ΡΠΌ ΠΊΠΎΠ»ΡΡΠ°ΠΌ ΠΈ ΡΠ΅ΡΠΊΠ°ΠΌ. ΡΠΎΡΡΠΎΡΡΠΈΠ΅ ΠΈΠ· ΠΌΠ΅ΡΠ°Π»Π»ΠΈΡΠ΅ΡΠΊΠΈΡ ΠΏΠΎΠ»ΠΎΡ ΠΈΠ»ΠΈ ΡΠ³ΠΎΠ»ΡΠ½ΡΡ Π±Π»ΠΎΠΊΠΎΠ², ΡΠ»Π΅Π³ΠΊΠ° ΠΎΠΏΠΈΡΠ°ΡΡΡΡ Π½Π° ΡΠΎΠΊΠΎΡΡΠ΅ΠΌΠ½ΡΠ΅ ΠΊΠΎΠ»ΡΡΠ°, ΠΊΠΎΠ³Π΄Π° ΠΎΠ½ΠΈ Π²ΡΠ°ΡΠ°ΡΡΡΡ.
Β
ΠΠΠ‘ Π²ΡΠ°ΡΠ°ΡΡΠ΅ΠΉΡΡ ΠΊΠ°ΡΡΡΠΊΠΈ. ΠΠΠ‘ Π²ΡΠ°ΡΠ°ΡΡΠ΅ΠΉΡΡ ΠΊΠ°ΡΡΡΠΊΠΈ ΠΎΠΏΡΠ΅Π΄Π΅Π»ΡΠ΅ΡΡΡ Π²ΡΡΠ°ΠΆΠ΅Π½ΠΈΠ΅ΠΌ
.Β
18)Β Β Β Β E = E max sin 2Οft
Π³Π΄Π΅
19)Β Β Β Β E ΠΌΠ°ΠΊΡ. = nBAΟ
ΠΈ
Β Β Β Β Β Β Β Β Β Β Β Β E max β ΠΌΠ°ΠΊΡΠΈΠΌΠ°Π»ΡΠ½Π°Ρ ΠΠΠ‘
Β Β Β Β Β Β Β Β Β Β Β Β n —- ΠΊΠΎΠ»ΠΈΡΠ΅ΡΡΠ²ΠΎ Π²ΠΈΡΠΊΠΎΠ² Π² ΠΊΠ°ΡΡΡΠΊΠ΅
Β Β Β Β Β Β Β Β Β Β Β Β B β Π½Π°ΠΏΡΡΠΆΠ΅Π½Π½ΠΎΡΡΡ ΠΌΠ°Π³Π½ΠΈΡΠ½ΠΎΠ³ΠΎ ΠΏΠΎΠ»Ρ
Β Β Β Β Β Β Β Β Β Β Β Β A β ΠΏΠ»ΠΎΡΠ°Π΄Ρ ΠΎΠ΄Π½ΠΎΠΉ ΠΏΠ΅ΡΠ»ΠΈ ΠΈΠ»ΠΈ Π²ΠΈΡΠΊΠ° ΠΊΠ°ΡΡΡΠΊΠΈ [A = lw Π½Π° ΡΠΈΡ. 14, Π°)]
Β Β Β Β Β Β Β Β Β Β Β Β Ο β ΡΠΊΠΎΡΠΎΡΡΡ Π²ΡΠ°ΡΠ΅Π½ΠΈΡ ΠΊΠ°ΡΡΡΠΊΠΈ (ΡΠ°Π΄/ΡΠ΅ΠΊ)
Β Β Β Β Β Β Β Β Β Β Β Β f β ΡΠ°ΡΡΠΎΡΠ° Π²ΡΠ°ΡΠ΅Π½ΠΈΡ (ΠΎΠ±ΠΎΡΠΎΡΠΎΠ² Π² ΡΠ΅ΠΊΡΠ½Π΄Ρ)
ΠΡΠ°ΡΠΈΠΊ 18) ΠΏΠΎΠΊΠ°Π·Π°Π½ Π½Π° ΡΠΈΡ. 16.
______________________________________________________________________________Β Β Β
ΠΡΠ²ΠΎΠ΄ ΡΠΎΡΠΌΡΠ»Ρ. ΠΠ· ΡΠΈΡ. 15 (Π±) ΠΌΡ Π²ΠΈΠ΄ΠΈΠΌ, ΡΡΠΎ ΡΠΊΠΎΡΠΎΡΡΡ v bc (ΡΠΎΠΎΡΠ²Π΅ΡΡΡΠ²ΡΡΡΠ°Ρ ΡΠΊΠΎΡΠΎΡΡΡ ΡΠΎΡΠΊΠΈ c) ΡΠ²ΡΠ·Π°Π½Π° ΡΠΎ ΡΠΊΠΎΡΠΎΡΡΡΡ Π²ΡΠ°ΡΠ΅Π½ΠΈΡ Ο ΡΠΎΠΎΡΠ½ΠΎΡΠ΅Π½ΠΈΠ΅ΠΌ
Π’Π°ΠΊΠΈΠΌ ΠΎΠ±ΡΠ°Π·ΠΎΠΌ, 17) ΡΡΠ°Π½ΠΎΠ²ΠΈΡΡΡ
Β
21)Β Β Β Β E = Β½ B l Οw sin ΞΈ
, ΠΊΠΎΡΠΎΡΡΠΉ ΠΏΡΠ΅Π΄ΡΡΠ°Π²Π»ΡΠ΅Ρ ΠΠΠ‘, ΡΠΎΠ·Π΄Π°Π²Π°Π΅ΠΌΡΡ ΠΎΠ΄ΠΈΠ½ΠΎΡΠ½ΡΠΌ ΠΏΡΠΎΠ²ΠΎΠ΄ΠΎΠΌ Π΄Π»ΠΈΠ½ΠΎΠΉ l, ΠΏΠ΅ΡΠ΅ΡΠ΅ΠΊΠ°ΡΡΠΈΠΌ ΠΌΠ°Π³Π½ΠΈΡΠ½ΠΎΠ΅ ΠΏΠΎΠ»Π΅ B. ΠΠΎΡΠΊΠΎΠ»ΡΠΊΡ ΠΈ bc, ΠΈ ad Π³Π΅Π½Π΅ΡΠΈΡΡΡΡ ΠΠΠ‘, Π° ΡΠΈΡΠ»ΠΎ Π²ΠΈΡΠΊΠΎΠ² ΡΠ°Π²Π½ΠΎ n, 21) ΡΡΠ°Π½ΠΎΠ²ΠΈΡΡΡ ΡΠ°Π²Π½ΡΠΌ
.Β
22)Β Β Β Β E = nB l Οw sin ΞΈ
ΠΠ»ΠΎΡΠ°Π΄Ρ ΠΊΠΎΠ½ΡΡΡΠ° A ΡΠ°Π²Π½Π° A = l w, ΠΏΠΎΡΡΠΎΠΌΡ 22) ΠΌΠΎΠΆΠ½ΠΎ Π·Π°ΠΏΠΈΡΠ°ΡΡ ΠΊΠ°ΠΊ
Β
23)Β Β Β Β E = nB A Ο sin ΞΈ
ΠΠ΅Π³ΠΊΠΎ ΠΏΠΎΠΊΠ°Π·Π°ΡΡ, ΡΡΠΎ 23) ΠΎΡΠ½ΠΎΡΠΈΡΡΡ ΠΊ ΠΊΠ°ΡΡΡΠΊΠ΅ Π»ΡΠ±ΠΎΠΉ ΡΠΎΡΠΌΡ, Π²ΡΠ°ΡΠ°ΡΡΠ΅ΠΉΡΡ Π²ΠΎΠΊΡΡΠ³ ΠΎΡΠΈ, ΠΏΠ΅ΡΠΏΠ΅Π½Π΄ΠΈΠΊΡΠ»ΡΡΠ½ΠΎΠΉ ΠΎΠ΄Π½ΠΎΡΠΎΠ΄Π½ΠΎΠ΅ ΠΌΠ°Π³Π½ΠΈΡΠ½ΠΎΠ΅ ΠΏΠΎΠ»Π΅.
ΠΠΠ‘ ΠΌΠ°ΠΊΡΠΈΠΌΠ°Π»ΡΠ½Π°, ΠΊΠΎΠ³Π΄Π° ΠΏΠ»ΠΎΡΠΊΠΎΡΡΡ ΠΊΠ°ΡΡΡΠΊΠΈ ΠΏΠ°ΡΠ°Π»Π»Π΅Π»ΡΠ½Π° ΠΏΠΎΠ»Ρ, ΠΈ ΡΠ°Π²Π½Π° Π½ΡΠ»Ρ, ΠΊΠΎΠ³Π΄Π° ΠΎΠ½Π° ΠΏΠ΅ΡΠΏΠ΅Π½Π΄ΠΈΠΊΡΠ»ΡΡΠ½ΠΎ ΠΏΠΎΠ»Ρ. ΠΠ°ΠΊΡΠΈΠΌΠ°Π»ΡΠ½Π°Ρ ΠΠΠ‘ ΠΎΠΏΡΠ΅Π΄Π΅Π»ΡΠ΅ΡΡΡ ΡΠΈΡΠ»ΠΎΠΌ
.Β
24)Β Β Β Β E ΠΌΠ°ΠΊΡ. = nBAΟ
Π’Π°ΠΊΠΈΠΌ ΠΎΠ±ΡΠ°Π·ΠΎΠΌ, 23) ΠΌΠΎΠΆΠ½ΠΎ Π·Π°ΠΏΠΈΡΠ°ΡΡ ΠΊΠ°ΠΊ
Β
25)Β Β Β Β E = E max sin ΞΈ
ΠΡΠ»ΠΈ Ο ΠΏΠΎΡΡΠΎΡΠ½Π½Π°Ρ,
Β
26)Β Β Β Β ΞΈ = ΟtΒ Β Β Β Β Β Β Β Β Β Β ΠΈΠ»ΠΈΒ Β Β Β Β Β ΞΈ = 2Οft
ΠΈ
Β
27)Β Β Β Β Π = Π ΠΌΠ°ΠΊΡ sin Οt = E ΠΌΠ°ΠΊΡ sin 2Οft
______________________________________________________________________________Β Β
ΠΠ°Π³Π½Π΅ΡΠΎ. Π‘Π°ΠΌΡΠΉ ΠΏΡΠΎΡΡΠΎΠΉ ΡΠΈΠΏ Π³Π΅Π½Π΅ΡΠ°ΡΠΎΡΠ° ΠΏΠ΅ΡΠ΅ΠΌΠ΅Π½Π½ΠΎΠ³ΠΎ ΡΠΎΠΊΠ° β ΡΡΠΎ Π³Π΅Π½Π΅ΡΠ°ΡΠΎΡ, Π² ΠΊΠΎΡΠΎΡΠΎΠΌ ΠΈΡΠΏΠΎΠ»ΡΠ·ΡΠ΅ΡΡΡ ΠΏΠΎΡΡΠΎΡΠ½Π½ΡΠΉ ΠΌΠ°Π³Π½ΠΈΡ Π΄Π»Ρ ΡΠΎΠ·Π΄Π°Π½ΠΈΡ ΠΌΠ°Π³Π½ΠΈΡΠ½ΠΎΠ³ΠΎ ΠΏΠΎΠ»Ρ. Π‘ΠΌ. ΡΠΈΡ. 14 (Π±). ΠΡΠΎΡ ΡΠΈΠΏ ΡΠ»Π΅ΠΊΡΡΠΎΠ³Π΅Π½Π΅ΡΠ°ΡΠΎΡΠ° Π½Π°Π·ΡΠ²Π°Π΅ΡΡΡ ΠΌΠ°Π³Π½Π΅ΡΠΎ. ΠΠ°Π³Π½Π΅ΡΠΎ ΠΈΡΠΏΠΎΠ»ΡΠ·ΡΡΡΡΡ Π΄Π»Ρ ΠΏΠΎΠ΄Π°ΡΠΈ ΠΏΠ΅ΡΠ΅ΠΌΠ΅Π½Π½ΠΎΠ³ΠΎ ΡΠΎΠΊΠ° Π½Π° ΡΠ²Π΅ΡΠΈ Π·Π°ΠΆΠΈΠ³Π°Π½ΠΈΡ Π² Π³Π°Π·ΠΎΠ½ΠΎΠΊΠΎΡΠΈΠ»ΠΊΠΈ, ΠΌΠΎΡΠΎΡΠΈΠΊΠ»Ρ, ΠΏΠΎΠ΄Π²Π΅ΡΠ½ΡΠ΅ ΠΌΠΎΡΠΎΡΡ ΠΈ Π² Π½Π΅ΠΊΠΎΡΠΎΡΡΡ ΡΠ°ΠΌΠΎΠ»Π΅ΡΠ°Ρ .
ΠΠΎΠΌΠΌΠ΅ΡΡΠ΅ΡΠΊΠΈΠ΅ Π³Π΅Π½Π΅ΡΠ°ΡΠΎΡΡ ΠΏΠ΅ΡΠ΅ΠΌΠ΅Π½Π½ΠΎΠ³ΠΎ ΡΠΎΠΊΠ°. ΠΡΠΏΠΎΠ»ΡΠ·ΠΎΠ²Π°Π½ΠΈΠ΅ ΠΏΠΎΡΡΠΎΡΠ½Π½ΡΡ ΠΌΠ°Π³Π½ΠΈΡΠΎΠ² ΠΎΠ³ΡΠ°Π½ΠΈΡΠΈΠ²Π°Π΅Ρ ΠΊΠΎΠ»ΠΈΡΠ΅ΡΡΠ²ΠΎ ΡΠ»Π΅ΠΊΡΡΠΎΡΠ½Π΅ΡΠ³ΠΈΠΈ, ΠΊΠΎΡΠΎΡΠΎΠ΅ ΠΌΠΎΠΆΠ΅Ρ Π±ΡΡΡ ΠΏΡΠΎΠΈΠ·Π²Π΅Π΄Π΅Π½ΠΎ. ΠΠ»Π΅ΠΊΡΡΠΎΠΌΠ°Π³Π½ΠΈΡ ΠΌΠΎΠΆΠ΅Ρ ΠΏΡΠΎΠΈΠ·Π²ΠΎΠ΄ΠΈΡΡ Π³ΠΎΡΠ°Π·Π΄ΠΎ Π±ΠΎΠ»Π΅Π΅ ΡΠΈΠ»ΡΠ½ΠΎΠ΅ ΠΏΠΎΠ»Ρ, ΡΠ΅ΠΌ ΠΏΠΎΡΡΠΎΡΠ½Π½ΡΠΉ ΠΌΠ°Π³Π½ΠΈΡ. Π’Π°ΠΊΠΈΠΌ ΠΎΠ±ΡΠ°Π·ΠΎΠΌ, ΠΊΠΎΠΌΠΌΠ΅ΡΡΠ΅ΡΠΊΠΈΠ΅ Π³Π΅Π½Π΅ΡΠ°ΡΠΎΡΡ ΠΏΠ΅ΡΠ΅ΠΌΠ΅Π½Π½ΠΎΠ³ΠΎ ΡΠΎΠΊΠ° ΠΈΡΠΏΠΎΠ»ΡΠ·ΡΡΡ ΡΠ»Π΅ΠΊΡΡΠΎΠΌΠ°Π³Π½ΠΈΡΡ Π΄Π»Ρ ΠΏΡΠΎΠΈΠ·Π²ΠΎΠ΄ΡΡ ΠΌΠ°Π³Π½ΠΈΡΠ½ΠΎΠ΅ ΠΏΠΎΠ»Π΅.
ΠΡΠΎΠΌΡΡΠ»Π΅Π½Π½ΡΠΉ Π³Π΅Π½Π΅ΡΠ°ΡΠΎΡ ΠΏΠ΅ΡΠ΅ΠΌΠ΅Π½Π½ΠΎΠ³ΠΎ ΡΠΎΠΊΠ° ΡΠΎΡΡΠΎΠΈΡ ΠΈΠ· ΡΡΠ΅Ρ ΠΎΡΠ½ΠΎΠ²Π½ΡΡ ΡΠ°ΡΡΠ΅ΠΉ:
(1) ΠΠ»Π΅ΠΊΡΡΠΎΠΌΠ°Π³Π½ΠΈΡΡ Π²ΠΎΠ·Π±ΡΠΆΠ΄Π΅Π½ΠΈΡ, ΡΠΎΠ·Π΄Π°ΡΡΠΈΠ΅ ΠΌΠ°Π³Π½ΠΈΡΠ½ΠΎΠ΅ ΠΏΠΎΠ»Π΅.
(2) ΠΡΠΌΠ°ΡΡΡΠ°, ΡΠΎΡΡΠΎΡΡΠ°Ρ ΠΈΠ· Π±ΠΎΠ»ΡΡΠΎΠ³ΠΎ ΠΊΠΎΠ»ΠΈΡΠ΅ΡΡΠ²Π° ΠΊΠ°ΡΡΡΠΊΠΈ ΠΈΠ·ΠΎΠ»ΠΈΡΠΎΠ²Π°Π½Π½ΠΎΠ³ΠΎ ΠΏΡΠΎΠ²ΠΎΠ΄Π°.
(3) ΠΠΎΠ½ΡΠ°ΠΊΡΠ½ΡΠ΅ ΠΊΠΎΠ»ΡΡΠ° ΠΈ ΡΠ΅ΡΠΊΠΈ.
ΠΠ΅Π½Π΅ΡΠ°ΡΠΎΡ ΠΏΠΎΡΡΠΎΡΠ½Π½ΠΎΠ³ΠΎ ΡΠΎΠΊΠ°. ΠΡΡΠΌΠΎΠΉ Π³Π΅Π½Π΅ΡΠ°ΡΠΎΡ ΡΠΎΠΊΠ° ΠΎΡΠ»ΠΈΡΠ°Π΅ΡΡΡ ΠΎΡ Π³Π΅Π½Π΅ΡΠ°ΡΠΎΡΠ° ΠΏΠ΅ΡΠ΅ΠΌΠ΅Π½Π½ΠΎΠ³ΠΎ ΡΠΎΠΊΠ° Π³Π΅Π½Π΅ΡΠ°ΡΠΎΡ ΡΠΎΠ»ΡΠΊΠΎ Π² ΠΎΠ΄Π½ΠΎΠΉ Π΄Π΅ΡΠ°Π»ΠΈ. ΠΠΌΠ΅ΡΡΠΎ ΠΈΡΠΏΠΎΠ»ΡΠ·ΠΎΠ²Π°Π½ΠΈΡ ΡΠΊΠΎΠ»ΡΠΆΠ΅Π½ΠΈΡ ΠΊΠΎΠ»ΡΡΠ° Π΄Π»Ρ ΠΏΠ΅ΡΠ΅Π΄Π°ΡΠΈ ΡΠΎΠΊΠ° ΠΎΡ ΠΊΠ°ΡΡΡΠ΅ΠΊ ΠΊ Π²Π½Π΅ΡΠ½Π΅ΠΉ ΡΠ΅ΠΏΠΈ, ΠΎΠ½ ΠΈΡΠΏΠΎΠ»ΡΠ·ΡΠ΅Ρ ΡΠ°Π·ΡΠ΅ΠΌΠ½ΠΎΠ΅ ΠΊΠΎΠ»ΡΡΠΎ, Π½Π°Π·ΡΠ²Π°Π΅ΠΌΠΎΠ΅ ΠΊΠΎΠΌΠΌΡΡΠ°ΡΠΎΡ. Π‘ΠΌ. ΡΠΈΡ. 17. ΠΠΎΠ³Π΄Π° ΠΊΠ°ΡΡΡΠΊΠ° Π²ΡΠ°ΡΠ°Π΅ΡΡΡ, ΡΠ΅ΡΠΊΠΈ ΠΏΠ΅ΡΠ΅ΠΌΠ΅ΡΠ°ΡΡΡΡ ΠΈΠ· ΠΎΠ΄Π½ΠΎΠΉ ΡΠ°ΡΡΠΈ ΡΠ°Π·ΡΠ΅Π·Π½ΠΎΠ³ΠΎ ΠΊΠΎΠ»ΡΡΠ° Π² Π΄ΡΡΠ³ΡΡ ΠΊΠ°ΠΊ ΡΠ°Π· Π² ΡΠΎΡ ΠΌΠΎΠΌΠ΅Π½Ρ, ΠΊΠΎΠ³Π΄Π° ΡΠΎΠΊ ΠΌΠ΅Π½ΡΠ΅Ρ Π½Π°ΠΏΡΠ°Π²Π»Π΅Π½ΠΈΠ΅, ΡΠΎΠ·Π΄Π°Π²Π°Ρ ΡΠΎΠΊ, ΠΊΠΎΡΠΎΡΡΠΉ Π²ΡΠ³Π»ΡΠ΄ΠΈΡ ΠΊΠ°ΠΊ ΠΏΠΎΠΊΠ°Π·Π°Π½ΠΎ Π½Π° ΡΠΈΡ. 18 (Π°). Π ΡΠΎΠ²ΡΠ΅ΠΌΠ΅Π½Π½ΠΎΠΌ ΠΎΠΊΡΡΠ³Π΅ ΠΠΎΠ»ΡΠΌΠ±ΠΈΡ Π³Π΅Π½Π΅ΡΠ°ΡΠΎΡΠΎΠ² ΡΠΊΠΎΡΡ ΡΠΎΡΡΠΎΠΈΡ ΠΈΠ· ΠΌΠ½ΠΎΠΆΠ΅ΡΡΠ²Π° ΠΊΠ°ΡΡΡΠ΅ΠΊ ΡΠΎΠ΅Π΄ΠΈΠ½Π΅Π½Ρ ΠΏΠΎΡΠ»Π΅Π΄ΠΎΠ²Π°ΡΠ΅Π»ΡΠ½ΠΎ. ΠΠ° ΡΠΈΡ. 18 (c) ΠΏΠΎΠΊΠ°Π·Π°Π½ Π²ΡΠ²ΠΎΠ΄ ΡΡΠ΅Ρ Π²ΠΈΡΠΊΠΎΠ²ΡΠΉ ΡΠΊΠΎΡΡ. ΠΡΠ° Π°ΡΠΌΠ°ΡΡΡΠ° ΠΈΠΌΠ΅Π΅Ρ Π΄Π²Π° ΠΊΠΎΠ»Π»Π΅ΠΊΡΠΎΡΠ½ΡΠ΅ ΡΡΠ΅ΡΠΆΠ½ΠΈ Π΄Π»Ρ ΠΊΠ°ΠΆΠ΄ΠΎΠΉ ΠΊΠ°ΡΡΡΠΊΠΈ, Π²ΡΠ΅Π³ΠΎ ΡΠ΅ΡΡΡ ΡΡΠ΅ΡΠΆΠ½Π΅ΠΉ. ΠΠ½ΠΈ Π΅ΡΡΡ
ΡΠ°ΡΠΏΠΎΠ»ΠΎΠΆΠ΅Π½Ρ ΡΠ°ΠΊΠΈΠΌ ΠΎΠ±ΡΠ°Π·ΠΎΠΌ, ΡΡΠΎ ΠΊΠ°ΠΆΠ΄ΡΠΉ Π½Π°Π±ΠΎΡ ΡΡΠ΅ΡΠΆΠ½Π΅ΠΉ ΠΊΠΎΠ½ΡΠ°ΠΊΡΠΈΡΡΠ΅Ρ Ρ ΡΠ΅ΡΠΊΠΈ Π² ΡΠΎ Π²ΡΠ΅ΠΌΡ, ΠΊΠΎΠ³Π΄Π° ΡΠΎΠΎΡΠ²Π΅ΡΡΡΠ²ΡΡΡΠ°Ρ ΠΊΠ°ΡΡΡΠΊΠ° ΠΏΡΠΎΡ ΠΎΠ΄ΡΡΠΈΠΉ ΡΠ΅ΡΠ΅Π· Π½Π°ΠΈΠ±ΠΎΠ»ΡΡΠ΅Π΅ ΠΊΠΎΠ»ΠΈΡΠ΅ΡΡΠ²ΠΎ Π»ΠΈΠ½ΠΈΠΉ ΠΏΠΎΡΠΎΠΊΠ°. Π’Π°ΠΊΠΈΠΌ ΠΎΠ±ΡΠ°Π·ΠΎΠΌ, ΠΊΠ°ΡΡΡΠΊΠ° 1 Π½Π° ΡΠΈΡΡΠ½ΠΊΠ΅ ΠΏΠΎΠ΄Π°Π΅Ρ ΡΠ²ΠΎΠΉ Π²ΡΡ ΠΎΠ΄ Π½Π° ΡΠ΅ΡΠΊΠΈ Π΄Π»Ρ ΠΏΠ΅ΡΠ²ΡΡ 60 Π³ΡΠ°Π΄ΡΡΠΎΠ² Π²ΡΠ°ΡΠ΅Π½ΠΈΡ. ΠΡΠΎ ΡΠΎΠ³Π΄Π° ΠΎΡΠΊΠ»ΡΡΠ°Π΅ΡΡΡ ΠΎΡ ΡΠ΅ΠΏΠΈ, ΠΈ ΠΊΠ°ΡΡΡΠΊΠ° 2 ΠΎΠ±Π΅ΡΠΏΠ΅ΡΠΈΠ²Π°Π΅Ρ Π²ΡΡ ΠΎΠ΄ Π΄Π»Ρ ΡΠ»Π΅Π΄ΡΡΡΠΈΠ΅ 60 Π³ΡΠ°Π΄ΡΡΠΎΠ² ΠΈ Ρ. Π΄. ΠΠΠ‘, ΡΠΎΠ·Π΄Π°Π²Π°Π΅ΠΌΠ°Ρ ΠΊΠ°ΡΡΡΠΊΠΎΠΉ 1 ΠΏΠΎΡΠ»Π΅ Π²ΡΡΠ΅Π·Π°Π½ΠΈΡ ΠΏΠΎΠΊΠ°Π·Π°Π½ΠΎ Π½Π° ΡΠΈΡΡΠ½ΠΊΠ΅ ΠΏΡΠ½ΠΊΡΠΈΡΠ½ΡΠ΅ Π»ΠΈΠ½ΠΈΠΈ. ΠΡΠΏΠΎΠ»ΡΠ·ΡΡ ΠΌΠ½ΠΎΠΆΠ΅ΡΡΠ²ΠΎ ΠΊΠ°ΡΡΡΠ΅ΠΊ, ΠΌΠΎΠΆΠ½ΠΎ ΠΏΠΎΠ»ΡΡΠΈΡΡ Π΄ΠΎΡΡΠ°ΡΠΎΡΠ½ΠΎ ΡΡΠ°Π±ΠΈΠ»ΡΠ½ΠΎΠ΅ Π²ΡΡ ΠΎΠ΄Π½ΠΎΠ΅ Π½Π°ΠΏΡΡΠΆΠ΅Π½ΠΈΠ΅.
ΠΠΎΠΊΠ°Π·Π°Π½ Π°Π²ΡΠΎΠΌΠΎΠ±ΠΈΠ»ΡΠ½ΡΠΉ Π³Π΅Π½Π΅ΡΠ°ΡΠΎΡ Π² ΡΠ°Π·ΡΠ΅Π·Π΅. Π½Π° ΡΠΈΡ. 19.
ΠΠ»Π΅ΠΊΡΡΠΎΠ΄Π²ΠΈΠ³Π°ΡΠ΅Π»Ρ. ΠΡΠ½ΠΎΠ²Π½Π°Ρ ΠΎΠΏΠ΅ΡΠ°ΡΠΈΠΎΠ½Π½Π°Ρ ΠΡΠΈΠ½ΡΠΈΠΏ ΡΠ°Π±ΠΎΡΡ ΡΠ»Π΅ΠΊΡΡΠΎΠ΄Π²ΠΈΠ³Π°ΡΠ΅Π»Ρ ΠΎΡΠ½ΠΎΠ²Π°Π½ Π½Π° ΠΎΠ΄Π½ΠΎΠΌ ΠΏΡΠΎΡΡΠΎΠΌ ΡΠ°ΠΊΡ: ΠΏΡΠΎΠ²ΠΎΠ΄Π½ΠΈΠΊ Ρ ΡΠΎΠΊΠΎΠΌ Π² ΠΌΠ°Π³Π½ΠΈΡΠ½ΠΎΠΌ ΠΏΠΎΠ»Π΅ Π±ΡΠ΄Π΅Ρ ΠΈΡΠΏΡΡΠ°ΡΡ Π±ΠΎΠΊΠΎΠ²ΠΎΠΉ ΡΠΎΠ»ΡΠΎΠΊ Ρ ΠΏΠΎΠ»Ρ. ΠΠ°ΠΊ ΡΠ»Π΅Π΄ΡΡΠ²ΠΈΠ΅, ΠΏΠ΅ΡΠ»Ρ Ρ ΡΠΎΠΊΠΎΠΌ Π² ΠΌΠ°Π³Π½ΠΈΡΠ½ΠΎΠΌ ΠΏΠΎΠ»Π΅ Π±ΡΠ΄Π΅Ρ ΠΈΡΠΏΡΡΡΠ²Π°ΡΡ ΠΊΡΡΡΡΡΠΈΠΉ ΠΌΠΎΠΌΠ΅Π½Ρ. ΠΠ»Π΅ΠΊΡΡΠΈΡΠ΅ΡΠΊΠΈΠΉ Π΄Π²ΠΈΠ³Π°ΡΠ΅Π»Ρ Π½Π° ΡΠ°ΠΌΠΎΠΌ Π΄Π΅Π»Π΅ ΠΏΡΠΎΡΡΠΎ Π³Π΅Π½Π΅ΡΠ°ΡΠΎΡ ΠΏΠΎΡΡΠΎΡΠ½Π½ΠΎΠ³ΠΎ ΡΠΎΠΊΠ°, ΡΠ°Π±ΠΎΡΠ°ΡΡΠΈΠΉ Π² ΠΎΠ±ΡΠ°ΡΠ½ΠΎΠΌ Π½Π°ΠΏΡΠ°Π²Π»Π΅Π½ΠΈΠΈ. Π Π³Π΅Π½Π΅ΡΠ°ΡΠΎΡΠ΅ ΠΏΠΎΡΡΠΎΡΠ½Π½ΠΎΠ³ΠΎ ΡΠΎΠΊΠ° ΠΊΠ°ΡΡΡΠΊΠ° ΠΏΡΠΎΠΊΡΡΡΠΈΠ²Π°Π΅ΡΡΡ ΡΠ΅ΡΠ΅Π· Π½Π΅ΠΊΠΎΡΠΎΡΠΎΠ΅ ΠΌΠ΅Ρ Π°Π½ΠΈΡΠ΅ΡΠΊΠΈΠ΅ ΡΡΠ΅Π΄ΡΡΠ²Π°, ΡΠ°ΠΊΠΈΠ΅ ΠΊΠ°ΠΊ Π³Π°Π·ΠΎΠ²Π°Ρ ΡΡΡΠ±ΠΈΠ½Π°, ΠΈ ΠΏΡΠΎΠΈΠ·Π²ΠΎΠ΄ΠΈΡΡΡ ΡΠ»Π΅ΠΊΡΡΠΈΡΠ΅ΡΡΠ²ΠΎ. ΠΡΠ»ΠΈ ΠΆΠ΅ ΠΊΠΎΡΠΌΠΈΡΡ ΡΠ»Π΅ΠΊΡΡΠΈΡΠ΅ΡΡΠ²ΠΎ ΠΊ Π³Π΅Π½Π΅ΡΠ°ΡΠΎΡΡ, ΡΡΠΎ Π·Π°ΡΡΠ°Π²ΠΈΡ ΠΊΠ°ΡΡΡΠΊΡ Π²ΡΠ°ΡΠ°ΡΡΡΡ. ΠΡΠ»ΠΈ ΠΏΠΎΠ΄Π°ΡΡ ΡΠ»Π΅ΠΊΡΡΠΈΡΠ΅ΡΡΠ²ΠΎ Π½Π° ΠΏΠΎΡΡΠΎΡΠ½Π½ΡΠΉ ΡΠΎΠΊ Π³Π΅Π½Π΅ΡΠ°ΡΠΎΡΠ΅, ΠΏΠΎΠΊΠ°Π·Π°Π½Π½ΠΎΠΌ Π½Π° ΡΠΈΡ. 17, ΠΊΠ°ΡΡΡΠΊΠ° Π±ΡΠ΄Π΅Ρ Π²ΡΠ°ΡΠ°ΡΡΡΡ.
ΠΠ²Π° ΡΠΈΠΏΠ° Π΄Π²ΠΈΠ³Π°ΡΠ΅Π»Π΅ΠΉ ΠΏΠΎΡΡΠΎΡΠ½Π½ΠΎΠ³ΠΎ ΡΠΎΠΊΠ°. ΠΠ²ΠΈΠ³Π°ΡΠ΅Π»ΠΈ ΠΏΠΎΡΡΠΎΡΠ½Π½ΠΎΠ³ΠΎ ΡΠΎΠΊΠ° ΠΌΠΎΠ³ΡΡ Π±ΡΡΡ Ρ ΠΏΠ°ΡΠ°Π»Π»Π΅Π»ΡΠ½ΠΎΠΉ ΠΈΠ»ΠΈ ΠΏΠΎΡΠ»Π΅Π΄ΠΎΠ²Π°ΡΠ΅Π»ΡΠ½ΠΎΠΉ ΠΎΠ±ΠΌΠΎΡΠΊΠΎΠΉ. Π Π΄Π²ΠΈΠ³Π°ΡΠ΅Π»Π΅ Ρ ΠΏΠ°ΡΠ°Π»Π»Π΅Π»ΡΠ½ΡΠΌ Π²ΠΎΠ·Π±ΡΠΆΠ΄Π΅Π½ΠΈΠ΅ΠΌ ΠΎΠ±ΠΌΠΎΡΠΊΠΈ ΠΌΠ°Π³Π½ΠΈΡΠ° Π²ΠΎΠ·Π±ΡΠΆΠ΄Π΅Π½ΠΈΡ ΠΈ ΠΎΠ±ΠΌΠΎΡΠΊΠΈ ΡΠΊΠΎΡΡ ΡΠΎΠ΅Π΄ΠΈΠ½Π΅Π½Ρ ΠΏΠ°ΡΠ°Π»Π»Π΅Π»ΡΠ½ΠΎ, ΡΠΎΠ³Π΄Π° ΠΊΠ°ΠΊ Π² Π΄Π²ΠΈΠ³Π°ΡΠ΅Π»Π΅ Ρ ΠΏΠΎΡΠ»Π΅Π΄ΠΎΠ²Π°ΡΠ΅Π»ΡΠ½ΡΠΌ Π²ΠΎΠ·Π±ΡΠΆΠ΄Π΅Π½ΠΈΠ΅ΠΌ ΠΎΠ½ΠΈ ΡΠΎΠ΅Π΄ΠΈΠ½Π΅Π½Ρ ΠΏΠΎΡΠ»Π΅Π΄ΠΎΠ²Π°ΡΠ΅Π»ΡΠ½ΠΎ. ΠΠ²Π° Π’ΠΈΠΏΡ ΠΈΠΌΠ΅ΡΡ ΡΠ°Π·Π½ΡΠ΅ ΡΠΊΡΠΏΠ»ΡΠ°ΡΠ°ΡΠΈΠΎΠ½Π½ΡΠ΅ Ρ Π°ΡΠ°ΠΊΡΠ΅ΡΠΈΡΡΠΈΠΊΠΈ. ΠΠ²ΠΈΠ³Π°ΡΠ΅Π»Ρ Ρ ΠΏΠ°ΡΠ°Π»Π»Π΅Π»ΡΠ½ΠΎΠΉ ΠΎΠ±ΠΌΠΎΡΠΊΠΎΠΉ ΠΈΠΌΠ΅Π΅Ρ ΡΠΎ ΠΏΡΠ΅ΠΈΠΌΡΡΠ΅ΡΡΠ²ΠΎ, ΡΡΠΎ Π΅Π³ΠΎ ΡΠΊΠΎΡΠΎΡΡΡ Π½Π΅ ΡΠΈΠ»ΡΠ½ΠΎ Π·Π°Π²ΠΈΡΠΈΡ ΠΎΡ ΠΏΠ΅ΡΠ΅ΠΌΠ΅Π½Π½ΡΡ Π½Π°Π³ΡΡΠ·ΠΎΠΊ, Π² ΡΠΎ Π²ΡΠ΅ΠΌΡ ΠΊΠ°ΠΊ ΡΠΊΠΎΡΠΎΡΡΡ ΠΏΠΎΡΠ»Π΅Π΄ΠΎΠ²Π°ΡΠ΅Π»ΡΠ½ΠΎΠ³ΠΎ Π΄Π²ΠΈΠ³Π°ΡΠ΅Π»Ρ Π·Π°Π²ΠΈΡΠΈΡ. ΠΠ° Ρ Π΄ΡΡΠ³ΠΎΠΉ ΡΡΠΎΡΠΎΠ½Ρ, ΠΊΡΡΡΡΡΠΈΠΉ ΠΌΠΎΠΌΠ΅Π½Ρ Π΄Π²ΠΈΠ³Π°ΡΠ΅Π»Ρ Ρ ΠΏΠΎΡΠ»Π΅Π΄ΠΎΠ²Π°ΡΠ΅Π»ΡΠ½ΡΠΌ Π²ΠΎΠ·Π±ΡΠΆΠ΄Π΅Π½ΠΈΠ΅ΠΌ ΠΏΡΠΈ ΠΏΡΡΠΊΠ΅ Π½Π°ΠΌΠ½ΠΎΠ³ΠΎ Π±ΠΎΠ»ΡΡΠ΅, ΡΠ΅ΠΌ Ρ ΠΏΠ°ΡΠ°Π»Π»Π΅Π»ΡΠ½ΠΎΠ³ΠΎ Π΄Π²ΠΈΠ³Π°ΡΠ΅Π»Ρ. ΠΌΠΎΡΠΎΡ. ΠΠΎΡΠ»Π΅Π΄ΠΎΠ²Π°ΡΠ΅Π»ΡΠ½ΡΠΉ Π΄Π²ΠΈΠ³Π°ΡΠ΅Π»Ρ ΠΈΡΠΏΠΎΠ»ΡΠ·ΡΠ΅ΡΡΡ, ΠΊΠΎΠ³Π΄Π° Π΄Π²ΠΈΠ³Π°ΡΠ΅Π»Ρ Π΄ΠΎΠ»ΠΆΠ΅Π½ Π·Π°ΠΏΡΡΠΊΠ°ΡΡΡΡ ΠΏΠΎΠ΄ Π±ΠΎΠ»ΡΡΠΎΠΉ Π½Π°Π³ΡΡΠ·ΠΊΠΎΠΉ.
ΠΠ²ΠΈΠ³Π°ΡΠ΅Π»ΠΈ ΡΠ΅ΡΠΈΠΈΠΌΠΎΠ³ΡΡ ΡΠ°Π±ΠΎΡΠ°ΡΡ ΠΊΠ°ΠΊ Π½Π° ΠΏΠ΅ΡΠ΅ΠΌΠ΅Π½Π½ΠΎΠΌ, ΡΠ°ΠΊ ΠΈ Π½Π° ΠΏΠΎΡΡΠΎΡΠ½Π½ΠΎΠΌ ΡΠΎΠΊΠ΅. Π’Π΅ΡΠ΅Π½ΠΈΠ΅ ΠΌΠ΅Π½ΡΠ΅Ρ Π½Π°ΠΏΡΠ°Π²Π»Π΅Π½ΠΈΠ΅ Π½Π° ΠΎΠ΄Π½ΠΎΠ²ΡΠ΅ΠΌΠ΅Π½Π½ΠΎ ΠΊΠ°ΠΊ Π² ΠΏΠΎΠ»Π΅, ΡΠ°ΠΊ ΠΈ Π² ΡΠΊΠΎΡΠ΅. Π‘Π΅ΡΠΈΠΉΠ½ΡΠ΅ Π΄Π²ΠΈΠ³Π°ΡΠ΅Π»ΠΈ ΠΈΡΠΏΠΎΠ»ΡΠ·ΡΡΡΡΡ Π΄Π»Ρ Π²Π΅Π½ΡΠΈΠ»ΡΡΠΎΡΠΎΠ², ΡΠ²Π΅ΠΉΠ½ΡΡ ΠΌΠ°ΡΠΈΠ½, ΠΏΡΠ»Π΅ΡΠΎΡΡ ΠΈ Π΄Ρ.
ΠΡΠΈΠ½ΡΠΈΠΏΠΈΠ°Π»ΡΠ½Π°Ρ ΡΡ Π΅ΠΌΠ° Π΄Π²ΠΈΠ³Π°ΡΠ΅Π»Ρ ΠΏΠΎΡΡΠΎΡΠ½Π½ΠΎΠ³ΠΎ ΡΠΎΠΊΠ° ΠΏΠΎΠΊΠ°Π·Π°Π½Π° Π½Π° ΡΠΈΡ. 20. Π―ΠΊΠΎΡΡ Π ΠΏΡΠ΅Π΄ΡΡΠ°Π²Π»ΡΠ΅Ρ ΡΠΎΠ±ΠΎΠΉ ΡΠΈΠ»ΠΈΠ½Π΄Ρ ΠΈΠ· ΠΌΡΠ³ΠΊΠΎΠΉ ΡΡΠ°Π»ΠΈ, ΡΡΡΠ°Π½ΠΎΠ²Π»Π΅Π½Π½ΠΎΠΉ Π½Π° Π²Π°Π»Ρ, ΡΡΠΎΠ±Ρ ΠΎΠ½ ΠΌΠΎΠ³ Π²ΡΠ°ΡΠ°ΡΡΡΡ ΠΈ ΡΠΎΠ΄Π΅ΡΠΆΠ°ΡΠΈΠ΅ ΠΏΡΠΎΠ΄ΠΎΠ»ΡΠ½ΡΠ΅ ΠΏΠ°Π·Ρ, Π² ΠΊΠΎΡΠΎΡΡΠ΅ Π·Π°Π΄Π΅Π»Π°Π½Ρ ΠΌΠ΅Π΄Π½ΡΠ΅ ΠΏΡΠΎΠ²ΠΎΠ΄Π½ΠΈΠΊΠΈ C. Π’ΠΎΠΊ Π½Π°ΠΏΡΠ°Π²Π»ΡΠ΅ΡΡΡ Π² ΠΈ ΠΈΠ· ΡΡΠΈΡ ΠΏΡΠΎΠ²ΠΎΠ΄Π½ΠΈΠΊΠΎΠ² ΡΠ΅ΡΠ΅Π· Π³ΡΠ°ΡΠΈΡΠΎΠ²ΡΠ΅ ΡΠ΅ΡΠΊΠΈ ΠΊΠ°ΡΠ°ΡΡΡ ΡΠ΅Π³ΠΌΠ΅Π½ΡΠΈΡΠΎΠ²Π°Π½Π½ΠΎΠ³ΠΎ ΡΠΈΠ»ΠΈΠ½Π΄ΡΠ° Π½Π° Π²Π°Π» Π½Π°Π·ΡΠ²Π°Π΅ΡΡΡ ΠΊΠΎΠ»Π»Π΅ΠΊΡΠΎΡΠΎΠΌ.
Π’ΠΎΠΊ Π² ΠΊΠ°ΡΡΡΠΊΠ°Ρ Π²ΠΎΠ·Π±ΡΠΆΠ΄Π΅Π½ΠΈΡ F, F ΡΠΎΠ·Π΄Π°Π΅Ρ ΠΌΠ°Π³Π½ΠΈΡΠ½ΠΎΠ΅ ΠΏΠΎΠ»Π΅, ΡΡΡΠ΅ΡΡΠ²Π΅Π½Π½ΠΎ ΡΠ°Π΄ΠΈΠ°Π»ΡΠ½ΠΎΠ΅ Π² Π·Π°Π·ΠΎΡΠ΅ ΠΌΠ΅ΠΆΠ΄Ρ ΠΎΠ½ΠΈ ΠΈ Π°ΡΠΌΠ°ΡΡΡΠ°. ΠΠΎΡΠΎΡΠ½Π°Ρ ΡΠ°ΠΌΠ° Π, Π ΠΎΠ±Π΅ΡΠΏΠ΅ΡΠΈΠ²Π°Π΅Ρ ΠΏΡΡΡ Π΄Π»Ρ ΠΌΠ°Π³Π½ΠΈΡΠ½ΠΎΠ³ΠΎ ΠΏΠΎΠ»Ρ
ΠΠ±ΡΠ°ΡΠ½Π°Ρ ΠΠΠ‘ Π΄Π²ΠΈΠ³Π°ΡΠ΅Π»Ρ. ΠΠ°ΡΡΡΠΊΠ° Π΄Π²ΠΈΠ³Π°ΡΠ΅Π»Ρ Π²ΡΠ°ΡΠ°Π΅ΡΡΡ Π² ΠΌΠ°Π³Π½ΠΈΡΠ½ΠΎΠ΅ ΠΏΠΎΠ»Π΅ ΠΈ, ΡΠ»Π΅Π΄ΠΎΠ²Π°ΡΠ΅Π»ΡΠ½ΠΎ, ΡΠΎΠ·Π΄Π°Π΅Ρ ΠΠΠ‘ ΠΊΠΎΡΠΎΡΠΎΠΌΡ ΠΏΠΎ Π·Π°ΠΊΠΎΠ½Ρ ΠΠ΅Π½ΡΠ° ΠΏΡΠΎΡΠΈΠ²ΠΎΡΡΠΎΠΈΡ Π²ΠΏΠ΅ΡΠ°ΡΠ»Π΅Π½Π½ΠΎΠ΅ Ρ.Π΄.Ρ. ΠΠ½Π° Π½Π°Π·ΡΠ²Π°Π΅ΡΡΡ ΠΎΠ±ΡΠ°ΡΠ½ΠΎΠΉ ΠΠΠ‘. ΠΡΡΠ΅ΠΊΡΠΈΠ²Π½ΠΎΠ΅ Π½Π°ΠΏΡΡΠΆΠ΅Π½ΠΈΠ΅ ΠΊΠΎΡΠΎΡΠ°Ρ ΠΏΡΠΈΠ²ΠΎΠ΄ΠΈΡ Π² Π΄Π²ΠΈΠΆΠ΅Π½ΠΈΠ΅ Π΄Π²ΠΈΠ³Π°ΡΠ΅Π»Ρ, ΡΠ°Π²Π½Π° ΡΠ°Π·Π½ΠΈΡΠ΅ ΠΌΠ΅ΠΆΠ΄Ρ ΠΏΡΠΈΠ»ΠΎΠΆΠ΅Π½Π½ΠΎΠΉ ΠΠΠ‘ ΠΈ ΠΎΠ±ΡΠ°ΡΠ½ΠΎΠΉ ΠΠΠ‘. Π’Π°ΠΊΠΈΠΌ ΠΎΠ±ΡΠ°Π·ΠΎΠΌ, Π²ΠΎ Π²ΡΠ΅ΠΌΡ ΡΠ°Π±ΠΎΡΡ ΠΊΠ°ΠΆΠ΄ΡΠΉ Π΄Π²ΠΈΠ³Π°ΡΠ΅Π»Ρ ΡΠ°ΠΊΠΆΠ΅ Π³Π΅Π½Π΅ΡΠ°ΡΠΎΡ.
ΠΡΠΎΡΠΈΠ²ΠΎ-ΠΠΠ‘ Π΄Π²ΠΈΠ³Π°ΡΠ΅Π»Ρ, ΡΠ°Π±ΠΎΡΠ°ΡΡΠ΅Π³ΠΎ Π±Π΅Π· Π½Π°Π³ΡΡΠ·ΠΊΠΈ Π±ΡΠ΄Π΅Ρ ΠΏΠΎΡΡΠΈ ΡΠ°Π²Π½Π° ΠΏΡΠΈΠ»ΠΎΠΆΠ΅Π½Π½ΠΎΠΉ ΠΠΠ‘. ΠΠΎΡΡΠ°ΡΠΎΡΠ½ΠΎ ΡΠΎΠΊ ΡΠ΅ΡΠ΅Ρ ΡΠ΅ΡΠ΅Π· Π΄Π²ΠΈΠ³Π°ΡΠ΅Π»Ρ, ΡΡΠΎΠ±Ρ ΠΏΡΠ΅ΠΎΠ΄ΠΎΠ»Π΅ΡΡ ΡΡΠ΅Π½ΠΈΠ΅. ΠΠΎΠ΄ Π½Π°Π³ΡΡΠ·ΠΊΠΎΠΉ ΠΎΠ±ΡΠ°ΡΠ½Π°Ρ ΠΠΠ‘ ΠΏΠ°Π΄Π°Π΅Ρ ΠΈ ΠΏΡΠΎΡΠ΅ΠΊΠ°Π΅Ρ Π±ΠΎΠ»ΡΡΠΈΠΉ ΡΠΎΠΊ ΡΠ΅ΡΠ΅Π· Π°ΡΠΌΠ°ΡΡΡΡ, ΡΡΠΎΠ±Ρ ΡΠΏΡΠ°Π²ΠΈΡΡΡΡ Ρ Π½Π°Π³ΡΡΠ·ΠΊΠΎΠΉ.
Β
Π‘ΡΡΠ»ΠΊΠΈ
1. Π‘ΠΈΡΡ, ΠΠ΅ΠΌΠ°Π½ΡΠΊΠΈΠΉ. Π£Π½ΠΈΠ²Π΅ΡΡΠΈΡΠ΅ΡΡΠΊΠ°Ρ ΡΠΈΠ·ΠΈΠΊΠ°
2. Π‘Π΅ΠΌΠ°Ρ, ΠΠ°Ρ. Π€ΠΈΠ·ΠΈΠΊΠ°.
3. Π’ΡΠΏΠΎΠΉ, ΠΠ΅ΡΠΊΠ°Π»Ρ, ΠΡΡΠΊΡ. Π‘ΠΎΠ²ΡΠ΅ΠΌΠ΅Π½Π½Π°Ρ ΡΠΈΠ·ΠΈΠΊΠ°.
Β
ΠΡΠ΅ ΠΎΡ SolitaryRoad.com:
ΠΡΡΡ ΠΡΡΠΈΠ½Ρ ΠΈ ΠΠΈΠ·Π½ΠΈ
ΠΠΎΠΆΡΠ΅ ΠΏΠΎΡΠ»Π°Π½ΠΈΠ΅ ΠΌΠΈΡΡ
ΠΠΈΡΡΡ Π₯ΡΠΈΡΡΠΎΡ ΠΈ ΠΠ³ΠΎ Π£ΡΠ΅Π½ΠΈΠ΅
ΠΡΠ΄ΡΡΠ΅ ΡΠ»ΠΎΠ²Π°
ΠΡΡΡ ΠΏΡΠΎΡΠ²Π΅ΡΠ»Π΅Π½ΠΈΡ, ΠΌΡΠ΄ΡΠΎΡΡΠΈ ΠΈ ΠΏΠΎΠ½ΠΈΠΌΠ°Π½ΠΈΡ
ΠΡΡΡ ΠΈΡΡΠΈΠ½Π½ΠΎΠ³ΠΎ Ρ ΡΠΈΡΡΠΈΠ°Π½ΡΡΠ²Π°
ΠΠΌΠ΅ΡΠΈΠΊΠ°, ΠΊΠΎΡΡΡΠΌΠΏΠΈΡΠΎΠ²Π°Π½Π½Π°Ρ, ΡΠ°Π·Π²ΡΠ°ΡΠ½Π°Ρ, Π±Π΅ΡΡΠΎΠ²Π΅ΡΡΠ½Π°Ρ ΡΡΡΠ°Π½Π°
Π ΡΠ΅ΡΡΠ½ΠΎΡΡΠΈ ΠΈ Π΅Π΅ ΠΎΡΡΡΡΡΡΠ²ΠΈΠΈ
ΠΡΠΈΡΠ΅ΡΠΈΠ΅ΠΌ Ρ ΡΠΈΡΡΠΈΠ°Π½ΡΡΠ²Π° ΡΠ΅Π»ΠΎΠ²Π΅ΠΊΠ° ΡΠ²Π»ΡΠ΅ΡΡΡ ΡΠΎ, ΡΡΠΎ ΠΎΠ½ Π΅ΡΡΡ
ΠΡΠΎ ΠΏΠΎΠΏΠ°Π΄Π΅Ρ Π² ΡΠ°ΠΉ?
ΠΡΡΡΠ΅Π΅ Π»ΠΈΡΠΎ
Π Π²Π΅ΡΠ΅ ΠΈ Π΄Π΅Π»Π°Ρ
ΠΠ΅Π²ΡΠ½ΠΎΡΡΠΎ ΠΏΡΡΡ ΠΏΡΠΎΡΠ΅Π½ΡΠΎΠ² ΠΏΡΠΎΠ±Π»Π΅ΠΌ, Ρ ΠΊΠΎΡΠΎΡΡΠΌΠΈ Π±ΠΎΠ»ΡΡΠΈΠ½ΡΡΠ²ΠΎ Π»ΡΠ΄Π΅ΠΉ ΠΏΡΠΈΡΠ»ΠΈ ΠΎΡ Π»ΠΈΡΠ½ΠΎΠΉ Π³Π»ΡΠΏΠΎΡΡΠΈ
ΠΠΈΠ±Π΅ΡΠ°Π»ΠΈΠ·ΠΌ, ΡΠΎΡΠΈΠ°Π»ΠΈΠ·ΠΌ ΠΈ ΡΠΎΠ²ΡΠ΅ΠΌΠ΅Π½Π½ΠΎΠ΅ Π³ΠΎΡΡΠ΄Π°ΡΡΡΠ²ΠΎ Π²ΡΠ΅ΠΎΠ±ΡΠ΅Π³ΠΎ Π±Π»Π°Π³ΠΎΡΠΎΡΡΠΎΡΠ½ΠΈΡ
ΠΠ΅Π»Π°Π½ΠΈΠ΅ ΠΏΡΠΈΡΠΈΠ½ΠΈΡΡ Π²ΡΠ΅Π΄, ΠΌΠΎΡΠΈΠ² ΠΏΠΎΠ²Π΅Π΄Π΅Π½ΠΈΡ
Π£ΡΠ΅Π½ΠΈΠ΅ ΡΡΠΎ:
Π ΡΠΎΠ²ΡΠ΅ΠΌΠ΅Π½Π½ΠΎΠΌ ΠΈΠ½ΡΠ΅Π»Π»Π΅ΠΊΡΡΠ°Π»ΠΈΠ·ΠΌΠ΅
Π Π³ΠΎΠΌΠΎΡΠ΅ΠΊΡΡΠ°Π»ΠΈΠ·ΠΌΠ΅
Π ΡΠ°ΠΌΠΎΠ΄ΠΎΡΡΠ°ΡΠΎΡΠ½ΠΎΠΉ Π·Π°Π³ΠΎΡΠΎΠ΄Π½ΠΎΠΉ ΠΆΠΈΠ·Π½ΠΈ, ΠΏΡΠΈΡΡΠ°Π΄Π΅Π±Π½ΠΎΠΌ Ρ ΠΎΠ·ΡΠΉΡΡΠ²Π΅
ΠΡΠΈΠ½ΡΠΈΠΏΡ ΠΆΠΈΠ·Π½ΠΈ
Π’Π΅ΠΌΠ°ΡΠΈΡΠ΅ΡΠΊΠΈΠ΅ ΠΏΠΎΡΠ»ΠΎΠ²ΠΈΡΡ, ΠΏΠΎΡΡΠ΅Π½ΠΈΡ, Π¦ΠΈΡΠ°ΡΡ. ΠΠ±ΡΠΈΠ΅ ΠΏΠΎΠ³ΠΎΠ²ΠΎΡΠΊΠΈ. ΠΠ»ΡΠΌΠ°Π½Π°Ρ Π±Π΅Π΄Π½ΠΎΠ³ΠΎ Π ΠΈΡΠ°ΡΠ΄Π°.
ΠΠΌΠ΅ΡΠΈΠΊΠ° ΡΠ±ΠΈΠ»Π°ΡΡ Ρ ΠΏΡΡΠΈ
ΠΠ΅ΠΉΡΡΠ²ΠΈΡΠ΅Π»ΡΠ½ΠΎ Π±ΠΎΠ»ΡΡΠΈΠ΅ Π³ΡΠ΅Ρ ΠΈ
Π’Π΅ΠΎΡΠΈΡ ΡΠΎΡΠΌΠΈΡΠΎΠ²Π°Π½ΠΈΡ Ρ Π°ΡΠ°ΠΊΡΠ΅ΡΠ°
ΠΠΎΡΠ°Π»ΡΠ½ΠΎΠ΅ ΠΈΠ·Π²ΡΠ°ΡΠ΅Π½ΠΈΠ΅
Π’Ρ ΡΠΎ, ΡΡΠΎ ΡΡ Π΅ΡΡ
ΠΡΠ΄ΠΈ ΠΊΠ°ΠΊ ΡΠ°Π΄ΠΈΠΎΡΡΠ½Π΅ΡΡ — ΠΎΠ½ΠΈ Π²ΡΠ±ΠΈΡΠ°ΡΡ ΠΈ ΡΠ»ΡΡΠ°ΡΡ ΠΎΠ΄Π½Ρ Π΄Π»ΠΈΠ½Ρ Π²ΠΎΠ»Π½Ρ ΠΈ ΠΈΠ³Π½ΠΎΡΠΈΡΠΎΠ²Π°ΡΡ ΠΎΡΡΠ°Π»ΡΠ½ΡΠ΅
ΠΡΠΈΡΠΈΠ½Π° ΡΠ΅ΡΡ Ρ Π°ΡΠ°ΠΊΡΠ΅ΡΠ° — ΠΏΠΎ ΠΡΠΈΡΡΠΎΡΠ΅Π»Ρ
ΠΡΠΈ Π²Π΅ΡΠΈ ΠΈΠ΄ΡΡ Π²ΠΌΠ΅ΡΡΠ΅
Π’Π΅Π»Π΅Π²ΠΈΠ΄Π΅Π½ΠΈΠ΅
ΠΡ ΡΠΎ, ΡΡΠΎ ΠΌΡ Π΅Π΄ΠΈΠΌ — ΠΆΠΈΠ²Π΅ΠΌ ΠΏΠΎ Π΄ΠΈΡΡΠΈΠΏΠ»ΠΈΠ½Π΅ Π΄ΠΈΠ΅ΡΡ
ΠΠ·Π±Π΅Π³Π°Π½ΠΈΠ΅ ΠΏΡΠΎΠ±Π»Π΅ΠΌ ΠΈ Π½Π΅ΠΏΡΠΈΡΡΠ½ΠΎΡΡΠ΅ΠΉ Π² ΠΆΠΈΠ·Π½ΠΈ
Π ΠΎΠ»Ρ ΠΏΡΠΈΠ²ΡΡΠΊΠΈ Π² ΡΠΎΡΠΌΠΈΡΠΎΠ²Π°Π½ΠΈΠΈ Ρ Π°ΡΠ°ΠΊΡΠ΅ΡΠ°.
ΠΡΡΠΈΠ½Π½ΡΠΉ Ρ ΡΠΈΡΡΠΈΠ°Π½ΠΈΠ½
Π§ΡΠΎ ΡΠ°ΠΊΠΎΠ΅ ΠΈΡΡΠΈΠ½Π½ΠΎΠ΅ Ρ ΡΠΈΡΡΠΈΠ°Π½ΡΡΠ²ΠΎ?
ΠΠΈΡΠ½ΡΠ΅ ΠΊΠ°ΡΠ΅ΡΡΠ²Π° ΠΈΡΡΠΈΠ½Π½ΠΎΠ³ΠΎ Ρ ΡΠΈΡΡΠΈΠ°Π½ΠΈΠ½Π°
Π§ΡΠΎ ΠΎΠΏΡΠ΅Π΄Π΅Π»ΡΠ΅Ρ Ρ Π°ΡΠ°ΠΊΡΠ΅Ρ ΡΠ΅Π»ΠΎΠ²Π΅ΠΊΠ°?
ΠΡΠ±ΠΎΠ²Ρ ΠΊ ΠΠΎΠ³Ρ ΠΈ Π»ΡΠ±ΠΎΠ²Ρ ΠΊ Π΄ΠΎΠ±ΡΠΎΠ΄Π΅ΡΠ΅Π»ΠΈ ΡΠ΅ΡΠ½ΠΎ ΡΠ²ΡΠ·Π°Π½Ρ
ΠΡΠΎΠ³ΡΠ»ΠΊΠ° ΠΏΠΎ ΠΎΠ΄ΠΈΠ½ΠΎΠΊΠΎΠΉ Π΄ΠΎΡΠΎΠ³Π΅
ΠΠ½ΡΠ΅Π»Π»Π΅ΠΊΡΡΠ°Π»ΡΠ½ΠΎΠ΅ Π½Π΅ΡΠ°Π²Π΅Π½ΡΡΠ²ΠΎ ΠΌΠ΅ΠΆΠ΄Ρ Π»ΡΠ΄ΡΠΌΠΈ ΠΈ Π²Π»Π°ΡΡΡΡ Π² Ρ ΠΎΡΠΎΡΠΈΡ ΠΏΡΠΈΠ²ΡΡΠΊΠ°Ρ
ΠΠ½ΡΡΡΡΠΌΠ΅Π½ΡΡ ΡΠ°ΡΠ°Π½Ρ. Π’Π°ΠΊΡΠΈΠΊΠ° ΠΈ Π£Π»ΠΎΠ²ΠΊΠΈ, ΠΈΡΠΏΠΎΠ»ΡΠ·ΡΠ΅ΠΌΡΠ΅ ΠΡΡΠ²ΠΎΠ»ΠΎΠΌ.
Π ΡΠ΅Π°ΠΊΡΠΈΠΈ Π½Π° ΠΎΠ±ΠΈΠ΄Ρ
ΠΠ°ΡΡΠΎΡΡΠ°Ρ Ρ ΡΠΈΡΡΠΈΠ°Π½ΡΠΊΠ°Ρ Π²Π΅ΡΠ°
ΠΡΡΠ΅ΡΡΠ²Π΅Π½Π½ΡΠΉ ΠΏΡΡΡ β Π½Π΅Π΅ΡΡΠ΅ΡΡΠ²Π΅Π½Π½ΡΠΉ ΠΏΡΡΡ
ΠΡΠ΄ΡΠΎΡΡΡ, Π Π°Π·ΡΠΌ ΠΈ ΠΠΎΠ±ΡΠΎΠ΄Π΅ΡΠ΅Π»Ρ ΡΠ΅ΡΠ½ΠΎ ΡΠ²ΡΠ·Π°Π½Ρ
ΠΠ½Π°Π½ΠΈΠ΅ ΠΎΠ΄Π½ΠΎ, ΠΌΡΠ΄ΡΠΎΡΡΡ Π΄ΡΡΠ³ΠΎΠ΅
ΠΠΎΠΈ Π²Π·Π³Π»ΡΠ΄Ρ Π½Π° Ρ ΡΠΈΡΡΠΈΠ°Π½ΡΡΠ²ΠΎ Π² ΠΠΌΠ΅ΡΠΈΠΊΠ΅
Π‘Π°ΠΌΠΎΠ΅ Π³Π»Π°Π²Π½ΠΎΠ΅ Π² ΠΆΠΈΠ·Π½ΠΈ ΡΡΠΎ ΠΏΠΎΠ½ΠΈΠΌΠ°Π½ΠΈΠ΅
ΠΡΠ΅Π½ΠΊΠ° Π»ΡΠ΄Π΅ΠΉ
ΠΡ Π²ΡΠ΅ ΠΏΡΠΈΠΌΠ΅ΡΡ — Ρ ΠΎΡΠΎΡΠΎ ΡΡΠΎ ΠΈΠ»ΠΈ ΠΏΠ»ΠΎΡ ΠΎ
Π’Π΅Π»Π΅Π²ΠΈΠ΄Π΅Π½ΠΈΠ΅ — Π΄ΡΡ ΠΎΠ²Π½ΡΠΉ ΡΠ΄
ΠΠ΅ΡΠ²ΠΎΠ΄Π²ΠΈΠ³Π°ΡΠ΅Π»Ρ, ΠΊΠΎΡΠΎΡΡΠΉ ΡΠ΅ΡΠ°Π΅Ρ, Β«ΠΡΠΎ ΠΌΡ Π΅ΡΡΡΒ»
ΠΡΠΊΡΠ΄Π° Π±Π΅ΡΡΡΡΡ Π½Π°ΡΠΈ Π²Π·Π³Π»ΡΠ΄Ρ, Π²Π·Π³Π»ΡΠ΄Ρ ΠΈ ΡΠ΅Π½Π½ΠΎΡΡΠΈ?
ΠΡΠ΅Ρ β Π΄Π΅Π»ΠΎ ΡΠ΅ΡΡΠ΅Π·Π½ΠΎΠ΅. ΠΠ°ΠΊΠ°Π·Π°Π½ΠΈΠ΅ Π·Π° ΡΡΠΎ Π½Π°ΡΡΠΎΡΡΠ΅Π΅. ΠΠ΄ ΡΠ΅Π°Π»Π΅Π½.
Π‘Π°ΠΌΠΎΡΡΠΎΡΡΠ΅Π»ΡΠ½Π°Ρ Π΄ΠΈΡΡΠΈΠΏΠ»ΠΈΠ½Π° ΠΈ ΡΠ΅Π³Π»Π°ΠΌΠ΅Π½ΡΠ°ΡΠΈΡ
ΠΠΎΡΡΠΈΠΆΠ΅Π½ΠΈΠ΅ ΡΡΠ°ΡΡΡΡ Π² ΠΆΠΈΠ·Π½ΠΈ — Π²ΠΎΠΏΡΠΎΡ ΠΏΡΠ°Π²ΠΈΠ»ΡΠ½ΡΡ ΡΡΡΠ°ΡΠ΅Π³ΠΈΠΉ
Π‘Π°ΠΌΠΎΠ΄ΠΈΡΡΠΈΠΏΠ»ΠΈΠ½Π°
Π‘Π°ΠΌΠΎΠΎΠ±Π»Π°Π΄Π°Π½ΠΈΠ΅, ΡΠ΄Π΅ΡΠΆΠ°Π½Π½ΠΎΡΡΡ, ΡΠ°ΠΌΠΎΠ΄ΠΈΡΡΠΈΠΏΠ»ΠΈΠ½Π° β ΠΎΡΠ½ΠΎΠ²Π° ΡΡΠΎΠ»ΡΠΊΠΈΡ Π²Π΅ΡΠ΅ΠΉ Π² ΠΆΠΈΠ·Π½ΠΈ.
ΠΡ Π½Π°ΡΠΈ ΠΏΡΠΈΠ²ΡΡΠΊΠΈ
Π§ΡΠΎ ΡΠΎΡΠΌΠΈΡΡΠ΅Ρ Π½ΡΠ°Π²ΡΡΠ²Π΅Π½Π½ΡΠΉ Ρ Π°ΡΠ°ΠΊΡΠ΅Ρ?
[ ΠΠΎΠΌ ] [ ΠΠ²Π΅ΡΡ ] [ ΠΠ½ΡΠΎΡΠΌΠ°ΡΠΈΡ ] [ ΠΠΎΡΡΠ° ]
ΠΠ°ΠΊ ΠΎΠ½ΠΈ ΡΠΏΡΠ°Π²Π»ΡΡΡ ΠΏΠ΅ΡΠ΅ΠΊΡΠ΅ΡΡΠ½ΡΠΌΠΈ ΠΏΠΎΠΌΠ΅Ρ Π°ΠΌΠΈ ΠΈ ΡΠ»Π΅ΠΊΡΡΠΎΠΌΠ°Π³Π½ΠΈΡΠ½ΡΠΌΠΈ ΠΏΠΎΠΌΠ΅Ρ Π°ΠΌΠΈ?
ΠΠ»ΡΡΠ΅Π²ΡΠ΅ Π²ΡΠ²ΠΎΠ΄Ρ
ΠΠ°ΠΊΠΎΠ½ ΠΠ΅Π½ΡΠ° ΠΈ Π·Π°ΠΊΠΎΠ½ Π€Π°ΡΠ°Π΄Π΅Ρ ΡΠΎΠΎΠ±ΡΠ°ΡΡ Π½Π°ΠΌ Π΄Π²Π΅ Π²Π°ΠΆΠ½ΡΠ΅ Π²Π΅ΡΠΈ ΠΎ ΡΠΎΠΌ, ΠΊΠ°ΠΊ ΠΈΠ·ΠΌΠ΅Π½ΡΡΡΠ΅Π΅ΡΡ ΠΌΠ°Π³Π½ΠΈΡΠ½ΠΎΠ΅ ΠΏΠΎΠ»Π΅ Π²Π·Π°ΠΈΠΌΠΎΠ΄Π΅ΠΉΡΡΠ²ΡΠ΅Ρ Ρ ΠΏΠ΅ΡΠ»Π΅ΠΉ ΠΏΡΠΎΠ²ΠΎΠ΄Π½ΠΈΠΊΠ°.
ΠΡΠΈ Π΄Π²Π° ΡΡΠ½Π΄Π°ΠΌΠ΅Π½ΡΠ°Π»ΡΠ½ΡΡ ΡΠΈΠ·ΠΈΡΠ΅ΡΠΊΠΈΡ Π·Π°ΠΊΠΎΠ½Π° ΠΎΠ±ΡΠ΅Π΄ΠΈΠ½ΡΡΡΡΡ, ΡΡΠΎΠ±Ρ ΡΠΏΡΠ°Π²Π»ΡΡΡ ΡΠ΅ΠΌ, ΠΊΠ°ΠΊ ΠΌΠ°Π³Π½ΠΈΡΠ½ΡΠ΅ ΠΏΠΎΠ»Ρ Π³Π΅Π½Π΅ΡΠΈΡΡΡΡΡΡ ΠΏΡΠΎΠ²ΠΎΠ΄Π½ΠΈΠΊΠ°ΠΌΠΈ, ΠΏΠΎ ΠΊΠΎΡΠΎΡΡΠΌ ΡΠ΅ΡΠ΅Ρ ΠΏΠΎΡΡΠΎΡΠ½Π½ΡΠΉ ΠΈΠ»ΠΈ ΠΏΠ΅ΡΠ΅ΠΌΠ΅Π½Π½ΡΠΉ ΡΠΎΠΊ.
ΠΠ°ΠΊΠΎΠ½ ΠΠ΅Π½ΡΠ° ΡΡΡΠ°Π½Π°Π²Π»ΠΈΠ²Π°Π΅Ρ Π½Π°ΠΏΡΠ°Π²Π»Π΅Π½ΠΈΠ΅ ΠΈΠ½Π΄ΡΡΠΈΡΠΎΠ²Π°Π½Π½ΠΎΠ³ΠΎ ΡΠΎΠΊΠ°, Π° Π·Π°ΠΊΠΎΠ½ Π€Π°ΡΠ°Π΄Π΅Ρ ΡΠ²ΡΠ·ΡΠ²Π°Π΅Ρ Π²Π΅Π»ΠΈΡΠΈΠ½Ρ ΠΈΠ½Π΄ΡΡΠΈΡΠΎΠ²Π°Π½Π½ΠΎΠΉ ΠΏΡΠΎΡΠΈΠ²ΠΎ-ΠΠΠ‘ ΡΠΎ ΡΠΊΠΎΡΠΎΡΡΡΡ ΠΈΠ·ΠΌΠ΅Π½Π΅Π½ΠΈΡ ΠΈΠ½Π΄ΡΡΠΈΡΡΡΡΠ΅Π³ΠΎ ΠΌΠ°Π³Π½ΠΈΡΠ½ΠΎΠ³ΠΎ ΠΏΠΎΠ»Ρ.
ΠΠ°Π³Π½ΠΈΡΠ½Π°Ρ ΠΈΠ½Π΄ΡΠΊΡΠΈΡ Π² ΡΡΠ°Π½ΡΡΠΎΡΠΌΠ°ΡΠΎΡΠ΅ ΡΠ΅Π³ΡΠ»ΠΈΡΡΠ΅ΡΡΡ Π·Π°ΠΊΠΎΠ½ΠΎΠΌ ΠΠ΅Π½ΡΠ° ΠΈ Π·Π°ΠΊΠΎΠ½ΠΎΠΌ Π€Π°ΡΠ°Π΄Π΅Ρ.
ΠΠ°ΡΡΡΠΊΠΈ ΠΈΠ½Π΄ΡΠΊΡΠΈΠ²Π½ΠΎΡΡΠΈ ΠΈ ΡΡΠ°Π½ΡΡΠΎΡΠΌΠ°ΡΠΎΡΡ Π½Π΅ ΡΠ°Π±ΠΎΡΠ°Π»ΠΈ Π±Ρ, Π΅ΡΠ»ΠΈ Π±Ρ Π½Π΅ ΡΡΡΠ΅ΡΡΠ²ΠΎΠ²Π°Π»ΠΈ ΡΡΠ½Π΄Π°ΠΌΠ΅Π½ΡΠ°Π»ΡΠ½ΡΠ΅ Π·Π°ΠΊΠΎΠ½Ρ ΡΠ»Π΅ΠΊΡΡΠΎΠΌΠ°Π³Π½Π΅ΡΠΈΠ·ΠΌΠ°. Π£ Π½Π°Ρ Π±ΡΠ»ΠΎ Π±Ρ ΡΠΎΠ»ΡΠΊΠΎ ΡΠ»Π΅ΠΊΡΡΠΈΡΠ΅ΡΠΊΠΎΠ΅ ΠΏΠΎΠ»Π΅ ΠΈ Π½ΠΈΠΊΠ°ΠΊΠΈΡ ΠΌΠ°Π³Π½ΠΈΡΠ½ΡΡ ΡΡΡΠ΅ΠΊΡΠΎΠ², ΡΠΎΠ·Π΄Π°Π²Π°Π΅ΠΌΡΡ Π΄Π²ΠΈΠΆΡΡΠΈΠΌΠΈΡΡ ΡΠΎΠΊΠ°ΠΌΠΈ. ΠΠ²ΡΠΌΡ ΠΎΡΠ½ΠΎΠ²Π½ΡΠΌΠΈ ΡΠ»Π΅ΠΊΡΡΠΎΠΌΠ°Π³Π½ΠΈΡΠ½ΡΠΌΠΈ Π·Π°ΠΊΠΎΠ½Π°ΠΌΠΈ, ΠΎΠΏΠΈΡΡΠ²Π°ΡΡΠΈΠΌΠΈ Π²Π·Π°ΠΈΠΌΠΎΡΠ²ΡΠ·Ρ ΠΌΠ΅ΠΆΠ΄Ρ ΠΈΠ½Π΄ΡΡΠΈΡΠΎΠ²Π°Π½Π½ΡΠΌΠΈ Π½Π°ΠΏΡΡΠΆΠ΅Π½ΠΈΡΠΌΠΈ ΠΈ ΠΌΠ°Π³Π½ΠΈΡΠ½ΡΠΌ ΠΏΠΎΠ»Π΅ΠΌ, ΡΠ²Π»ΡΡΡΡΡ Π·Π°ΠΊΠΎΠ½ ΠΠ΅Π½ΡΠ° ΠΈ Π·Π°ΠΊΠΎΠ½ Π€Π°ΡΠ°Π΄Π΅Ρ. ΠΠ° ΡΡΠΎΠ²Π½Π΅ ΠΏΠ΅ΡΠ°ΡΠ½ΠΎΠΉ ΠΏΠ»Π°ΡΡ ΡΡΠΈ Π΄Π²Π° Π·Π°ΠΊΠΎΠ½Π° ΠΎΠ±ΡΠ΅Π΄ΠΈΠ½ΡΡΡΡΡ Π΄Π»Ρ ΡΠΎΠ·Π΄Π°Π½ΠΈΡ ΠΈΠ½Π΄ΡΠΊΡΠΈΠ²Π½ΠΎΠΉ ΡΠ²ΡΠ·ΠΈ ΠΌΠ΅ΠΆΠ΄Ρ ΡΠ°Π·Π»ΠΈΡΠ½ΡΠΌΠΈ ΡΠ΅ΠΏΡΠΌΠΈ.
ΠΠΎΡΠΎΠΊ ΠΈ ΠΌΠ°Π³Π½ΠΈΡΠ½ΡΠΉ ΠΏΠΎΡΠΎΠΊ Π½Π°ΠΏΡΡΠΌΡΡ Π²Π»ΠΈΡΡΡ Π½Π° ΡΠ»Π΅ΠΊΡΡΠΎΠΌΠ°Π³Π½Π΅ΡΠΈΠ·ΠΌ, ΠΈ ΡΡΠΎΠ±Ρ ΠΏΠΎΠ½ΡΡΡ Π²Π·Π°ΠΈΠΌΠΎΡΠ²ΡΠ·Ρ ΡΠ°ΠΊΠΈΡ Π²ΠΎΠΏΡΠΎΡΠΎΠ², ΠΊΠ°ΠΊ Π·Π°ΠΊΠΎΠ½ Π€Π°ΡΠ°Π΄Π΅Ρ ΠΈ Π·Π°ΠΊΠΎΠ½ ΠΠ΅Π½ΡΠ°, Π²Ρ ΡΠ°ΡΡΠΌΠΎΡΡΠΈΡΠ΅ ΡΠ°ΠΊΠΈΠ΅ ΠΎΡΠ½ΠΎΠ²Π½ΡΠ΅ ΡΠ΅ΠΌΡ, ΠΊΠ°ΠΊ ΡΠ»Π΅ΠΊΡΡΠΎΠΌΠ°Π³Π½ΠΈΡΠ½Π°Ρ ΠΈΠ½Π΄ΡΠΊΡΠΈΡ ΠΈΠ»ΠΈ ΠΈΠ½Π΄ΡΡΠΈΡΠΎΠ²Π°Π½Π½Π°Ρ ΠΠΠ‘, ΠΏΠΎΡΠΎΠΊΠΎΡΡΠ΅ΠΏΠ»Π΅Π½ΠΈΠ΅, ΡΠ»Π΅ΠΊΡΡΠΈΡΠ΅ΡΠΊΠΈΠΉ ΡΠΎΠΊ ΠΈ ΠΈΠ½Π΄ΡΡΠΈΡΠΎΠ²Π°Π½Π½ΠΎΠ΅ ΡΠ»Π΅ΠΊΡΡΠΈΡΠ΅ΡΠΊΠΎΠ΅ ΠΏΠΎΠ»Π΅, ΡΠΈΠ»Π° ΠΠΎΡΠ΅Π½ΡΠ° ΠΈ ΡΠΈΠ»ΠΎΠ²ΡΠ΅ Π»ΠΈΠ½ΠΈΠΈ ΠΌΠ°Π³Π½ΠΈΡΠ½ΠΎΠ³ΠΎ ΠΏΠΎΠ»Ρ. ΠΠ΅Π·Π°Π²ΠΈΡΠΈΠΌΠΎ ΠΎΡ ΡΠΎΠ³ΠΎ, ΠΈΠΌΠ΅Π΅Ρ Π»ΠΈ ΠΌΠ΅ΡΡΠΎ ΠΎΠ΄Π½ΠΎΡΠΎΠ΄Π½ΠΎΠ΅ ΠΌΠ°Π³Π½ΠΈΡΠ½ΠΎΠ΅ ΠΏΠΎΠ»Π΅ ΠΈΠ»ΠΈ ΠΏΠ΅ΡΠ΅ΠΌΠ΅Π½Π½ΡΠΉ ΠΌΠ°Π³Π½ΠΈΡΠ½ΡΠΉ ΠΏΠΎΡΠΎΠΊ Π² Π²Π°ΡΠ΅ΠΉ ΠΏΡΠΎΠ²ΠΎΠ»ΠΎΡΠ½ΠΎΠΉ ΠΏΠ΅ΡΠ»Π΅ ΠΈΠ»ΠΈ ΡΠ»Π΅ΠΊΡΡΠΈΡΠ΅ΡΠΊΠΎΠ΅ ΠΏΠΎΠ»Π΅ Π² ΡΠ΅Π»ΠΎΠΌ, Π²Ρ Π΄ΠΎΠ»ΠΆΠ½Ρ Π±ΡΡΡ ΡΠ²Π΅ΡΠ΅Π½Ρ, ΡΡΠΎ Ρ Π²Π°Ρ Π΅ΡΡΡ Ρ ΠΎΡΠΎΡΠΈΠΉ Π΄Π°ΡΡΠΈΠΊ ΠΏΠ»ΠΎΡΠ½ΠΎΡΡΠΈ ΠΏΠΎΡΠΎΠΊΠ° ΠΈ Π½Π°ΠΏΡΡΠΆΠ΅Π½Π½ΠΎΡΡΠΈ ΠΌΠ°Π³Π½ΠΈΡΠ½ΠΎΠ³ΠΎ ΠΏΠΎΠ»Ρ.
ΠΡΠ»ΠΈ ΡΠ΅ΡΠΌΠΈΠ½ Β«ΠΈΠ½Π΄ΡΠΊΡΠΈΠ²Π½Π°Ρ ΡΠ²ΡΠ·ΡΒ» Π·Π²ΡΡΠΈΡ ΡΠ°ΡΠΏΠ»ΡΠ²ΡΠ°ΡΠΎ, ΠΏΡΠΎΡΡΠΎ ΠΏΠΎΠΌΠ½ΠΈΡΠ΅, ΡΡΠΎ ΠΌΡ Π³ΠΎΠ²ΠΎΡΠΈΠΌ ΠΎ ΠΏΠ΅ΡΠ΅ΠΊΡΠ΅ΡΡΠ½ΡΡ ΠΏΠΎΠΌΠ΅Ρ Π°Ρ , ΡΠ»Π΅ΠΊΡΡΠΎΠΌΠ°Π³Π½ΠΈΡΠ½ΡΡ ΠΏΠΎΠΌΠ΅Ρ Π°Ρ , ΠΏΠ΅ΡΠ΅Π΄Π°ΡΠ΅ ΡΡΠΌΠ° ΠΈ Π»ΡΠ±ΡΡ Π΄ΡΡΠ³ΠΈΡ ΡΠΏΠΎΡΠΎΠ±Π°Ρ , ΠΊΠΎΡΠΎΡΡΠΌΠΈ ΠΌΠ°Π³Π½ΠΈΡΠ½ΠΎΠ΅ ΠΏΠΎΠ»Π΅ ΠΌΠΎΠΆΠ΅Ρ Π²ΡΠ·Π²Π°ΡΡ ΡΡΠΌ Π² ΡΠ»Π΅ΠΊΡΡΠΈΡΠ΅ΡΠΊΠΎΠΉ ΡΠ΅ΠΏΠΈ. Π§ΡΠΎΠ±Ρ Π»ΡΡΡΠ΅ ΡΠ²ΠΈΠ΄Π΅ΡΡ ΠΈ ΠΏΡΠ΅Π΄ΡΠΊΠ°Π·Π°ΡΡ, ΠΊΠΎΠ³Π΄Π° ΡΠΈΠ³Π½Π°Π» Π² ΠΎΠ΄Π½ΠΎΠΉ ΠΎΠ±Π»Π°ΡΡΠΈ ΠΏΠ΅ΡΠ°ΡΠ½ΠΎΠΉ ΠΏΠ»Π°ΡΡ ΠΌΠΎΠΆΠ΅Ρ Π²ΡΠ·Π²Π°ΡΡ ΡΡΠΌ Π² Π΄ΡΡΠ³ΠΎΠΉ ΠΎΠ±Π»Π°ΡΡΠΈ ΠΏΠ΅ΡΠ°ΡΠ½ΠΎΠΉ ΠΏΠ»Π°ΡΡ, ΠΏΠΎΠ»Π΅Π·Π½ΠΎ Π·Π½Π°ΡΡ ΠΊΠΎΠ΅-ΡΡΠΎ ΠΎ ΡΠ°Π·Π½ΠΈΡΠ΅ ΠΌΠ΅ΠΆΠ΄Ρ Π·Π°ΠΊΠΎΠ½ΠΎΠΌ ΠΠ΅Π½ΡΠ° ΠΈ Π·Π°ΠΊΠΎΠ½ΠΎΠΌ Π€Π°ΡΠ°Π΄Π΅Ρ. ΠΠΎΡ ΡΠ΅ΠΌ ΠΎΠ½ΠΈ ΠΎΡΠ»ΠΈΡΠ°ΡΡΡΡ ΠΈ Π³Π΄Π΅ ΠΎΠ½ΠΈ ΠΏΡΠΈΠ²ΠΎΠ΄ΡΡ ΠΊ ΠΏΠ΅ΡΠ΅ΠΊΡΠ΅ΡΡΠ½ΡΠΌ ΠΏΠΎΠΌΠ΅Ρ Π°ΠΌ Π² ΡΠΎΠΏΠΎΠ»ΠΎΠ³ΠΈΠΈ ΠΏΠ΅ΡΠ°ΡΠ½ΠΎΠΉ ΠΏΠ»Π°ΡΡ.
ΠΠ°ΠΊΠΎΠ½ ΠΠ΅Π½ΡΠ° ΠΏΡΠΎΡΠΈΠ² Π·Π°ΠΊΠΎΠ½Π° Π€Π°ΡΠ°Π΄Π΅Ρ
ΠΡΠΈ Π΄Π²Π° ΡΡΠ½Π΄Π°ΠΌΠ΅Π½ΡΠ°Π»ΡΠ½ΡΡ ΡΠΈΠ·ΠΈΡΠ΅ΡΠΊΠΈΡ Π·Π°ΠΊΠΎΠ½Π° ΠΎΠΏΡΠ΅Π΄Π΅Π»ΡΡΡ, ΠΊΠ°ΠΊ ΠΌΠ°Π³Π½ΠΈΡΠ½ΠΎΠ΅ ΠΏΠΎΠ»Π΅ Π²Π·Π°ΠΈΠΌΠΎΠ΄Π΅ΠΉΡΡΠ²ΡΠ΅Ρ Ρ ΠΊΠΎΠ½ΡΡΡΠΎΠΌ ΠΏΡΠΎΠ²ΠΎΠ΄Π½ΠΈΠΊΠ°. Π£ΡΡΠΈΡΠ΅, ΡΡΠΎ Ρ Π½Π°Ρ Π΅ΡΡΡ Π΄Π²Π΅ ΠΏΠ΅ΡΠ»ΠΈ ΠΏΡΠΎΠ²ΠΎΠ΄Π½ΠΈΠΊΠ°, ΠΎΠ±ΡΠ°ΡΠ΅Π½Π½ΡΠ΅ Π΄ΡΡΠ³ ΠΊ Π΄ΡΡΠ³Ρ. ΠΠΎ ΠΎΠ΄Π½ΠΎΠΉ ΠΏΠ΅ΡΠ»Π΅ ΡΠ΅ΡΠ΅Ρ ΡΠΎΠΊ, ΠΊΠΎΡΠΎΡΡΠΉ ΠΌΡ Π½Π°Π·ΠΎΠ²Π΅ΠΌ ΠΈΠ½Π΄ΡΠΊΡΠΈΠΎΠ½Π½ΡΠΌ ΡΠΎΠΊΠΎΠΌ, Π° ΠΏΠΎ Π΄ΡΡΠ³ΠΎΠΉ ΠΏΠ΅ΡΠ»Π΅ ΡΠΎΠΊ Π½Π΅ ΡΠ΅ΡΠ΅Ρ. ΠΠΎ Π·Π°ΠΊΠΎΠ½Ρ ΠΠΌΠΏΠ΅ΡΠ° ΠΌΡ Π·Π½Π°Π΅ΠΌ, ΡΡΠΎ ΡΠΎΠΊ Π² ΠΎΠ΄Π½ΠΎΠΌ ΠΊΠΎΠ½ΡΡΡΠ΅ ΡΠΎΠ·Π΄Π°Π΅Ρ ΠΌΠ°Π³Π½ΠΈΡΠ½ΠΎΠ΅ ΠΏΠΎΠ»Π΅. ΠΠ°ΠΊ ΡΡΠΎ ΠΌΠ°Π³Π½ΠΈΡΠ½ΠΎΠ΅ ΠΏΠΎΠ»Π΅ Π²Π·Π°ΠΈΠΌΠΎΠ΄Π΅ΠΉΡΡΠ²ΡΠ΅Ρ Ρ Π΄ΡΡΠ³ΠΎΠΉ ΠΏΠ΅ΡΠ»Π΅ΠΉ? Π§ΡΠΎΠ±Ρ ΠΏΠΎΠ»ΡΡΠΈΡΡ ΠΎΡΠ²Π΅Ρ, Π½Π°ΠΌ Π½ΡΠΆΠ½ΠΎ ΠΏΠΎΡΠΌΠΎΡΡΠ΅ΡΡ Π½Π° ΡΠ°Π·Π½ΠΈΡΡ ΠΌΠ΅ΠΆΠ΄Ρ Π·Π°ΠΊΠΎΠ½ΠΎΠΌ ΠΠ΅Π½ΡΠ° ΠΈ Π·Π°ΠΊΠΎΠ½ΠΎΠΌ Π€Π°ΡΠ°Π΄Π΅Ρ.
ΠΠ°ΠΊΠΎΠ½ Π€Π°ΡΠ°Π΄Π΅Ρ
ΠΡΠΎΡΠ΅ Π³ΠΎΠ²ΠΎΡΡ, Π·Π°ΠΊΠΎΠ½ Π€Π°ΡΠ°Π΄Π΅Ρ Π³Π»Π°ΡΠΈΡ ΡΠ»Π΅Π΄ΡΡΡΠ΅Π΅:
ΠΡΠΎ Π³ΠΎΠ²ΠΎΡΠΈΡ Π½Π°ΠΌ ΠΊΠΎΠ΅-ΡΡΠΎ Π²Π°ΠΆΠ½ΠΎΠ΅ ΠΎΠ± ΡΡΠ»ΠΎΠ²ΠΈΡΡ , Π½Π΅ΠΎΠ±Ρ ΠΎΠ΄ΠΈΠΌΡΡ Π΄Π»Ρ ΠΈΠ½Π΄ΡΠΊΡΠΈΠΈ: ΠΠ°Π³Π½ΠΈΡΠ½ΠΎΠ΅ ΠΏΠΎΠ»Π΅, ΡΠΎΠ·Π΄Π°Π²Π°Π΅ΠΌΠΎΠ΅ ΡΠΎΠΊΠΎΠΌ, Π΄ΠΎΠ»ΠΆΠ½ΠΎ ΠΈΠ·ΠΌΠ΅Π½ΡΡΡΡΡ Π²ΠΎ Π²ΡΠ΅ΠΌΠ΅Π½ΠΈ (Π±ΡΠ΄Ρ ΡΠΎ ΠΊΠΎΠ»Π΅Π±Π°Π½ΠΈΡ, Π²ΠΎΠ·ΡΠ°ΡΡΠ°Π½ΠΈΠ΅ ΠΈΠ»ΠΈ ΠΏΠ°Π΄Π΅Π½ΠΈΠ΅) ΡΡΠΎΠ±Ρ ΠΈΠ½Π΄ΡΡΠΈΡΠΎΠ²Π°ΡΡ ΡΠΎΠΊ Π²ΠΎ Π²ΡΠΎΡΠΎΠΌ Π²ΠΈΡΠΊΠ΅ ΠΏΡΠΎΠ²ΠΎΠ΄Π½ΠΈΠΊΠ°. ΠΠ±ΡΡΠ½ΠΎ ΡΡΠΎ Π²ΠΈΠ΄Π½ΠΎ Π² Π±Π°Π·ΠΎΠ²ΠΎΠΌ ΡΠΊΡΠΏΠ΅ΡΠΈΠΌΠ΅Π½ΡΠ΅ ΠΏΠΎ ΠΏΠ΅ΡΠ΅ΠΌΠ΅ΡΠ΅Π½ΠΈΡ ΠΌΠ°Π³Π½ΠΈΡΠ½ΠΎΠ³ΠΎ ΠΏΠΎΠ»Ρ ΠΊ ΠΊΠΎΠ½ΡΡΡΡ ΠΏΡΠΎΠ²ΠΎΠ΄Π½ΠΈΠΊΠ° ΠΈ ΠΎΡ Π½Π΅Π³ΠΎ, ΠΊΠ°ΠΊ ΠΏΠΎΠΊΠ°Π·Π°Π½ΠΎ Π½Π° ΠΈΠ·ΠΎΠ±ΡΠ°ΠΆΠ΅Π½ΠΈΠΈ Π½ΠΈΠΆΠ΅.
ΠΠ°ΠΏΡΡΠΆΠ΅Π½ΠΈΠ΅ ΠΈ ΡΠΎΠΊ ΠΈΠ½Π΄ΡΡΠΈΡΡΡΡΡΡ Π² ΡΠ΅ΠΏΠΈ ΡΠΎΠ»ΡΠΊΠΎ ΠΈΠ·ΠΌΠ΅Π½ΡΡΡΠΈΠΌΡΡ ΠΌΠ°Π³Π½ΠΈΡΠ½ΡΠΌ ΠΏΠΎΠ»Π΅ΠΌ.
ΠΠΎΡΠΊΠΎΠ»ΡΠΊΡ ΠΈΠ·ΠΌΠ΅Π½ΡΡΡΠ΅Π΅ΡΡ ΠΌΠ°Π³Π½ΠΈΡΠ½ΠΎΠ΅ ΠΏΠΎΠ»Π΅ ΠΈΠ½Π΄ΡΡΠΈΡΡΠ΅Ρ Π½Π°ΠΏΡΡΠΆΠ΅Π½ΠΈΠ΅, ΠΎΠ½ΠΎ ΡΠ°ΠΊΠΆΠ΅ ΠΈΠ½Π΄ΡΡΠΈΡΡΠ΅Ρ ΡΠΎΠΊ. Π ΠΊΠ°ΠΊΠΎΠΌ Π½Π°ΠΏΡΠ°Π²Π»Π΅Π½ΠΈΠΈ ΡΠ΅ΡΠ΅Ρ ΡΡΠΎΡ ΡΠΎΠΊ? ΠΠ»Ρ ΡΡΠΎΠ³ΠΎ Π½Π°ΠΌ ΠΏΠΎΠ½Π°Π΄ΠΎΠ±ΠΈΡΡΡ Π·Π°ΠΊΠΎΠ½ ΠΠ΅Π½ΡΠ°.
ΠΠ°ΠΊΠΎΠ½ ΠΠ΅Π½ΡΠ°
Β ΠΠΎΠ½ΠΈΠΌΠ°Π½ΠΈΠ΅ ΡΠ»Π΅ΠΊΡΡΠΈΡΠ΅ΡΠΊΠΎΠΉ ΡΠ½Π΅ΡΠ³ΠΈΠΈ ΠΎΠ·Π½Π°ΡΠ°Π΅Ρ ΠΏΠΎΠ½ΠΈΠΌΠ°Π½ΠΈΠ΅ Π½Π°ΠΏΡΠ°Π²Π»Π΅Π½ΠΈΡ ΡΠΎΠΊΠ° ΠΈ ΠΊΠΎΠ½ΡΡΡΠ° ΠΏΡΠΎΠ²ΠΎΠ΄ΠΈΠΌΠΎΡΡΠΈ, Π° ΡΠ°ΠΊΠΆΠ΅ Π»ΡΠ±ΠΎΠ³ΠΎ ΠΈΠ·ΠΌΠ΅Π½ΡΡΡΠ΅Π³ΠΎΡΡ ΠΏΠΎΡΠΎΠΊΠ°, ΠΌΠ΅Ρ Π°Π½ΠΈΡΠ΅ΡΠΊΠΎΠΉ ΡΠ½Π΅ΡΠ³ΠΈΠΈ ΠΈΠ»ΠΈ ΠΏΠ΅ΡΠ΅ΠΌΠ΅Π½Π½ΠΎΠ³ΠΎ ΠΌΠ°Π³Π½ΠΈΡΠ½ΠΎΠ³ΠΎ ΠΏΠΎΠ»Ρ Π² Π²Π°ΡΠ΅ΠΌ ΠΈΠ·Π΄Π΅Π»ΠΈΠΈ. ΠΠ°ΠΊΠΎΠ½ ΠΠ΅Π½ΡΠ° Π½Π° ΡΠ°ΠΌΠΎΠΌ Π΄Π΅Π»Π΅ ΡΠ²Π»ΡΠ΅ΡΡΡ Π°Π½Π°Π»ΠΎΠ³ΠΎΠΌ Π·Π°ΠΊΠΎΠ½Π° Π€Π°ΡΠ°Π΄Π΅Ρ Π² ΡΠΎΠΌ ΡΠΌΡΡΠ»Π΅, ΡΡΠΎ ΠΎΠ½ Π³ΠΎΠ²ΠΎΡΠΈΡ Π²Π°ΠΌ Π½Π°ΠΏΡΠ°Π²Π»Π΅Π½ΠΈΠ΅ ΠΈΠ½Π΄ΡΡΠΈΡΠΎΠ²Π°Π½Π½ΠΎΠ³ΠΎ ΡΠΎΠΊΠ°, Π½ΠΎ Π½Π΅ ΡΠ²Π½ΠΎ.
ΠΡΠΎ ΠΈΠΌΠ΅Π΅Ρ Π²Π°ΠΆΠ½ΠΎΠ΅ ΡΠ»Π΅Π΄ΡΡΠ²ΠΈΠ΅: Π½Π°ΠΏΡΠ°Π²Π»Π΅Π½ΠΈΠ΅ ΠΈΠ½Π΄ΡΠΊΡΠΈΠΎΠ½Π½ΠΎΠ³ΠΎ ΡΠΎΠΊΠ° ΠΏΡΠΎΡΠΈΠ²ΠΎΠΏΠΎΠ»ΠΎΠΆΠ½ΠΎ Π½Π°ΠΏΡΠ°Π²Π»Π΅Π½ΠΈΡ ΠΈΠ½Π΄ΡΠΊΡΠΈΠΎΠ½Π½ΠΎΠ³ΠΎ ΡΠΎΠΊΠ°. ΠΡΠ»ΠΈ ΠΈΠ½Π΄ΡΡΠΈΡΡΡΡΠ΅Π΅ ΠΈ ΠΈΠ½Π΄ΡΡΠΈΡΡΠ΅ΠΌΠΎΠ΅ ΠΏΠΎΠ»Ρ Π½Π°ΠΏΡΠ°Π²Π»Π΅Π½Ρ Π² ΠΏΡΠΎΡΠΈΠ²ΠΎΠΏΠΎΠ»ΠΎΠΆΠ½ΡΠ΅ ΡΡΠΎΡΠΎΠ½Ρ, ΡΠΎ ΠΈ ΡΠΎΠΊΠΈ ΡΠ°ΠΊΠΆΠ΅ Π΄ΠΎΠ»ΠΆΠ½Ρ Π±ΡΡΡ Π½Π°ΠΏΡΠ°Π²Π»Π΅Π½Ρ Π² ΠΏΡΠΎΡΠΈΠ²ΠΎΠΏΠΎΠ»ΠΎΠΆΠ½ΡΠ΅ ΡΡΠΎΡΠΎΠ½Ρ Π±Π»Π°Π³ΠΎΠ΄Π°ΡΡ ΠΎΠΏΡΠ΅Π΄Π΅Π»Π΅Π½ΠΈΡ ΠΏΡΠ°Π²ΠΈΠ»Π° ΠΏΡΠ°Π²ΠΎΠΉ ΡΡΠΊΠΈ. ΠΠ°ΠΏΡΠ°Π²Π»Π΅Π½ΠΈΠ΅ ΠΈΠ½Π΄ΡΡΠΈΡΠΎΠ²Π°Π½Π½ΠΎΠ³ΠΎ ΠΌΠ°Π³Π½ΠΈΡΠ½ΠΎΠ³ΠΎ ΠΏΠΎΠ»Ρ ΠΏΠΎΠΊΠ°Π·Π°Π½ΠΎ Π½Π° ΠΈΠ·ΠΎΠ±ΡΠ°ΠΆΠ΅Π½ΠΈΠΈ Π½ΠΈΠΆΠ΅, Π³Π΄Π΅ ΠΈΠ½Π΄ΡΡΠΈΡΠΎΠ²Π°Π½Π½ΠΎΠ΅ ΠΌΠ°Π³Π½ΠΈΡΠ½ΠΎΠ΅ ΠΏΠΎΠ»Π΅, ΡΠΎΠ·Π΄Π°Π²Π°Π΅ΠΌΠΎΠ΅ ΠΈΠ½Π΄ΡΡΠΈΡΠΎΠ²Π°Π½Π½ΡΠΌ ΡΠΎΠΊΠΎΠΌ, ΠΎΠΏΡΠ΅Π΄Π΅Π»ΡΠ΅ΡΡΡ ΠΏΠΎ ΠΏΡΠ°Π²ΠΈΠ»Ρ ΠΏΡΠ°Π²ΠΎΠΉ ΡΡΠΊΠΈ.
ΠΠ°ΠΊΠΎΠ½ ΠΠ΅Π½ΡΠ° ΠΎΠΏΡΠ΅Π΄Π΅Π»ΡΠ΅Ρ Π½Π°ΠΏΡΠ°Π²Π»Π΅Π½ΠΈΠ΅ ΠΈΠ½Π΄ΡΡΠΈΡΠΎΠ²Π°Π½Π½ΠΎΠ³ΠΎ ΡΠΎΠΊΠ° ΠΈ Π½Π°ΠΏΡΡΠΆΠ΅Π½ΠΈΡ Π² ΠΊΠΎΠ½ΡΡΡΠ΅ ΠΏΡΠΎΠ²ΠΎΠ΄Π½ΠΈΠΊΠ°.
ΠΠΌΠ΅ΡΡΠ΅ ΡΡΠΈ Π΄Π²Π° Π·Π°ΠΊΠΎΠ½Π° Π³ΠΎΠ²ΠΎΡΡΡ Π½Π°ΠΌ Π²ΡΠ΅, ΡΡΠΎ Π½Π°ΠΌ Π½ΡΠΆΠ½ΠΎ Π·Π½Π°ΡΡ ΠΎ ΠΏΠΎΠ²Π΅Π΄Π΅Π½ΠΈΠΈ ΡΠ»Π΅ΠΊΡΡΠΎΠΌΠ°Π³Π½ΠΈΡΠ½ΠΎΠ³ΠΎ ΠΏΠΎΠ»Ρ Π² ΠΏΠ΅ΡΠ°ΡΠ½ΠΎΠΉ ΠΏΠ»Π°ΡΠ΅. Π‘ΠΎΠ±ΡΠ°Π² ΠΈΡ Π²ΠΌΠ΅ΡΡΠ΅, ΠΌΡ ΡΠ΅ΠΏΠ΅ΡΡ ΠΌΠΎΠΆΠ΅ΠΌ Π»ΡΡΡΠ΅ ΠΏΠΎΠ½ΡΡΡ, ΠΊΠ°ΠΊ Π²ΠΎΠ·Π½ΠΈΠΊΠ°ΡΡ ΠΏΠ΅ΡΠ΅ΠΊΡΠ΅ΡΡΠ½ΡΠ΅ ΠΏΠΎΠΌΠ΅Ρ ΠΈ, ΡΠ»Π΅ΠΊΡΡΠΎΠΌΠ°Π³Π½ΠΈΡΠ½ΡΠ΅ ΠΏΠΎΠΌΠ΅Ρ ΠΈ ΠΈ ΡΡΠΌΠΎΠ²Π°Ρ ΡΠ²ΡΠ·Ρ ΠΌΠ΅ΠΆΠ΄Ρ ΡΠ°Π·Π»ΠΈΡΠ½ΡΠΌΠΈ ΠΎΠ±Π»Π°ΡΡΡΠΌΠΈ ΠΏΠ΅ΡΠ°ΡΠ½ΠΎΠΉ ΠΏΠ»Π°ΡΡ.
ΠΠ°ΠΊ ΡΡΠΈ Π·Π°ΠΊΠΎΠ½Ρ ΡΠΏΡΠ°Π²Π»ΡΡΡ ΠΈΠ½Π΄ΡΠΊΡΠΈΠ²Π½ΡΠΌΠΈ ΠΏΠ΅ΡΠ΅ΠΊΡΠ΅ΡΡΠ½ΡΠΌΠΈ ΠΏΠΎΠΌΠ΅Ρ Π°ΠΌΠΈ ΠΈ ΡΠ»Π΅ΠΊΡΡΠΎΠΌΠ°Π³Π½ΠΈΡΠ½ΡΠΌΠΈ ΠΏΠΎΠΌΠ΅Ρ Π°ΠΌΠΈ Π² ΠΏΠ΅ΡΠ°ΡΠ½ΠΎΠΉ ΠΏΠ»Π°ΡΠ΅
ΠΡΠ΅Π²ΠΈΠ΄Π½ΠΎ, ΡΡΠΎ ΠΌΡ ΠΈΠΌΠ΅Π΅ΠΌ Π΄Π΅Π»ΠΎ Ρ ΠΏΠΎΠ²Π΅Π΄Π΅Π½ΠΈΠ΅ΠΌ ΠΈΠ½Π΄ΡΠΊΡΠΈΠ²Π½ΠΎΠ³ΠΎ ΡΠΈΠ³Π½Π°Π»Π°, Π° ΡΡΠΎ ΠΎΠ·Π½Π°ΡΠ°Π΅Ρ, ΡΡΠΎ Π½Π°ΠΌ Π½Π΅ΠΎΠ±Ρ ΠΎΠ΄ΠΈΠΌΠΎ ΡΠ°ΡΡΠΌΠΎΡΡΠ΅ΡΡ Π΄Π²Π° Π²ΠΎΠ·ΠΌΠΎΠΆΠ½ΡΡ ΡΡΡΠ΅ΠΊΡΠ°, ΡΠ²ΡΠ·Π°Π½Π½ΡΡ Ρ ΠΈΠ½Π΄ΡΠΊΡΠΈΠ΅ΠΉ Π² ΠΏΠ΅ΡΠ°ΡΠ½ΠΎΠΉ ΠΏΠ»Π°ΡΠ΅:
ΠΠ΅ΡΠ΅ΠΊΡΠ΅ΡΡΠ½ΡΠ΅ ΠΏΠΎΠΌΠ΅Ρ ΠΈ : ΠΠΎΠ³Π΄Π° ΡΠΈΠ³Π½Π°Π» ΠΏΠ΅ΡΠ΅ΠΊΠ»ΡΡΠ°Π΅ΡΡΡ, ΠΌΠ°Π³Π½ΠΈΡΠ½ΠΎΠ΅ ΠΏΠΎΠ»Π΅, Π³Π΅Π½Π΅ΡΠΈΡΡΠ΅ΠΌΠΎΠ΅ ΠΏΠ΅ΡΠ΅ΠΊΠ»ΡΡΠ°ΡΡΠΈΠΌ ΡΠΎΠΊΠΎΠΌ, ΠΈΠ½Π΄ΡΡΠΈΡΡΠ΅Ρ ΡΠΈΠ³Π½Π°Π» Π² ΡΡΠ°ΡΡΠ΅-ΠΆΠ΅ΡΡΠ²Π΅, ΠΊΠΎΡΠΎΡΡΠΉ Π·Π°ΡΠ΅ΠΌ ΠΌΠΎΠΆΠ΅Ρ ΡΠ°ΡΠΏΡΠΎΡΡΡΠ°Π½ΡΡΡΡΡ ΠΏΠΎ ΠΌΠ΅ΠΆΡΠΎΠ΅Π΄ΠΈΠ½Π΅Π½ΠΈΡ ΠΈ Π΄ΠΎΡΡΠΈΠ³Π°ΡΡ ΠΏΡΠΈΠ΅ΠΌΠ½ΠΈΠΊΠ°.
Π¨ΡΠΌΠΎΠ²Π°Ρ ΡΠ²ΡΠ·Ρ : ΠΡΠΎ Π² ΠΎΡΠ½ΠΎΠ²Π½ΠΎΠΌ ΡΠΎΡΠΌΠ° ΠΏΠ΅ΡΠ΅ΠΊΡΠ΅ΡΡΠ½ΡΡ ΠΏΠΎΠΌΠ΅Ρ , ΠΏΡΠΈ ΠΊΠΎΡΠΎΡΠΎΠΉ ΠΊΠΎΠ»Π΅Π±Π°ΡΠ΅Π»ΡΠ½ΡΠΉ ΡΠΈΠ³Π½Π°Π» ΠΈΠ½Π΄ΡΡΠΈΡΡΠ΅ΡΡΡ Π² ΠΏΡΠΎΠ²ΠΎΠ΄Π½ΠΈΠΊΠ΅; ΠΎΠ΄Π½ΠΎΠΉ ΠΈΠ· ΡΠ°ΡΠΏΡΠΎΡΡΡΠ°Π½Π΅Π½Π½ΡΡ ΡΠΎΡΠΌ ΡΠ²Π»ΡΠ΅ΡΡΡ ΡΡΠΌΠΎΠ²Π°Ρ ΡΠ²ΡΠ·Ρ ΠΌΠ΅ΠΆΠ΄Ρ ΠΏΠ΅ΡΠ΅Ρ ΠΎΠ΄Π½ΡΠΌΠΈ ΠΎΡΠ²Π΅ΡΡΡΠΈΡΠΌΠΈ.
ΠΠ°ΠΊΠΎΠ½ ΠΠ΅Π½ΡΠ° Π³Π»Π°ΡΠΈΡ, ΡΡΠΎ ΠΊΠΎΠ³Π΄Π° Π²ΠΎΠ·Π½ΠΈΠΊΠ°ΡΡ ΡΡΠΈ ΡΡΡΠ΅ΠΊΡΡ, ΠΈΠ½Π΄ΡΡΠΈΡΠΎΠ²Π°Π½Π½ΡΠΉ ΡΠΈΠ³Π½Π°Π» ΡΠΎΠ·Π΄Π°Π΅Ρ ΡΠΎΠ±ΡΡΠ²Π΅Π½Π½ΠΎΠ΅ ΠΌΠ°Π³Π½ΠΈΡΠ½ΠΎΠ΅ ΠΏΠΎΠ»Π΅, ΠΈ Π½Π°ΠΏΡΠ°Π²Π»Π΅Π½ΠΈΠ΅ ΠΈΠ½Π΄ΡΡΠΈΡΠΎΠ²Π°Π½Π½ΠΎΠ³ΠΎ ΠΌΠ°Π³Π½ΠΈΡΠ½ΠΎΠ³ΠΎ ΠΏΠΎΠ»Ρ ΡΠΊΠ°Π·ΡΠ²Π°Π΅Ρ Π½Π° Π½Π°ΠΏΡΠ°Π²Π»Π΅Π½ΠΈΠ΅, ΠΏΡΠΎΡΠΈΠ²ΠΎΠΏΠΎΠ»ΠΎΠΆΠ½ΠΎΠ΅ Π½Π°ΠΏΡΠ°Π²Π»Π΅Π½ΠΈΡ ΠΈΠ½Π΄ΡΡΠΈΡΠΎΠ²Π°Π½Π½ΠΎΠ³ΠΎ ΠΌΠ°Π³Π½ΠΈΡΠ½ΠΎΠ³ΠΎ ΠΏΠΎΠ»Ρ. ΠΠ΅ΠΆΠ΄Ρ ΡΠ΅ΠΌ, Π·Π°ΠΊΠΎΠ½ Π€Π°ΡΠ°Π΄Π΅Ρ Π³Π»Π°ΡΠΈΡ, ΡΡΠΎ Π²Π΅Π»ΠΈΡΠΈΠ½Π° ΠΈΠ½Π΄ΡΡΠΈΡΠΎΠ²Π°Π½Π½ΠΎΠΉ ΠΎΠ±ΡΠ°ΡΠ½ΠΎΠΉ ΠΠΠ‘ ΡΠ΅ΠΌ Π±ΠΎΠ»ΡΡΠ΅, ΡΠ΅ΠΌ Π²ΡΡΠ΅ ΡΠ°ΡΡΠΎΡΠ° ΠΈΠ½Π΄ΡΡΠΈΡΡΡΡΠ΅Π³ΠΎ ΡΠΈΠ³Π½Π°Π»Π°. ΠΡΠΈ Π΄Π²Π° Π·Π°ΠΊΠΎΠ½Π° Π²ΠΌΠ΅ΡΡΠ΅ Ρ Π·Π°ΠΊΠΎΠ½ΠΎΠΌ ΠΠΌΠ° ΠΏΠΎΠ»Π½ΠΎΡΡΡΡ ΠΎΠΏΠΈΡΡΠ²Π°ΡΡ ΠΏΠΎΠ²Π΅Π΄Π΅Π½ΠΈΠ΅ ΠΈΠ½Π΄ΡΠΊΡΠΈΠ²Π½ΡΡ ΠΏΠ΅ΡΠ΅ΠΊΡΠ΅ΡΡΠ½ΡΡ ΠΏΠΎΠΌΠ΅Ρ .
ΠΡΡΠ΅ΡΠΏΠΎΠΌΡΠ½ΡΡΡΠ΅ ΠΏΡΠ½ΠΊΡΡ ΠΎΡΠ½ΠΎΡΡΡΡΡ ΠΊ ΠΈΠ½Π΄ΡΡΠΈΡΠΎΠ²Π°Π½Π½ΡΠΌ ΠΏΠ΅ΡΠ΅ΠΊΡΠ΅ΡΡΠ½ΡΠΌ ΠΏΠΎΠΌΠ΅Ρ Π°ΠΌ ΠΌΠ΅ΠΆΠ΄Ρ ΡΡΠ°ΡΡΠ°ΠΌΠΈ Π°Π³ΡΠ΅ΡΡΠΈΠ²Π½ΠΎΠ³ΠΎ ΠΈΡΡΠΎΡΠ½ΠΈΠΊΠ° ΠΈ ΠΆΠ΅ΡΡΠ²Ρ, Π° ΡΠ°ΠΊΠΆΠ΅ ΠΊ ΡΠ»Π΅ΠΊΡΡΠΎΠΌΠ°Π³Π½ΠΈΡΠ½ΡΠΌ ΠΏΠΎΠΌΠ΅Ρ Π°ΠΌ, ΠΈΠ½Π΄ΡΡΠΈΡΠΎΠ²Π°Π½Π½ΡΠΌ Π² ΡΠΎΠΊΠΎΠ²ΠΎΠΉ ΠΏΠ΅ΡΠ»Π΅ Π½Π° ΡΠ΅Π°Π»ΡΠ½ΠΎΠΉ ΠΏΠ΅ΡΠ°ΡΠ½ΠΎΠΉ ΠΏΠ»Π°ΡΠ΅. ΠΡΡΠΊΠΈΠΉ ΡΠ°Π·, ΠΊΠΎΠ³Π΄Π° Π΅ΡΡΡ ΠΈΠ·ΠΌΠ΅Π½ΡΡΡΠ΅Π΅ΡΡ ΠΌΠ°Π³Π½ΠΈΡΠ½ΠΎΠ΅ ΠΏΠΎΠ»Π΅, ΠΏΠ°Π΄Π°ΡΡΠ΅Π΅ ΠΏΠ°ΡΠ°Π»Π»Π΅Π»ΡΠ½ΠΎ ΠΏΠ΅ΡΠ»Π΅ ΠΏΡΠΎΠ²ΠΎΠ΄Π½ΠΈΠΊΠ°, ΡΠΎΠ³Π΄Π° Π±ΡΠ΄Π΅Ρ ΠΈΠ½Π΄ΡΡΠΈΡΠΎΠ²Π°Π½Π½ΡΠΉ ΡΠΎΠΊ. Π§ΡΠΎΠ±Ρ ΡΠΌΠ΅Π½ΡΡΠΈΡΡ Π²Π΅Π»ΠΈΡΠΈΠ½Ρ ΠΎΠ±ΠΎΠΈΡ ΡΡΡΠ΅ΠΊΡΠΎΠ², Ρ Π²Π°Ρ Π΅ΡΡΡ Π΄Π²Π° Π²Π°ΡΠΈΠ°Π½ΡΠ° Π½Π°ΡΡΡΠΎΠΉΠΊΠΈ Π³Π΅ΠΎΠΌΠ΅ΡΡΠΈΠΈ ΡΡΠ°ΡΡΡ:
ΠΡΠΏΠΎΠ»ΡΠ·ΡΠΉΡΠ΅ Π½Π΅ΠΌΠ½ΠΎΠ³ΠΎ Π±ΠΎΠ»Π΅Π΅ ΡΠΈΡΠΎΠΊΠΈΠ΅ Π΄ΠΎΡΠΎΠΆΠΊΠΈ
Π Π°ΡΠΏΠΎΠ»Π°Π³Π°ΠΉΡΠ΅ ΡΡΠ°ΡΡΡ Π±Π»ΠΈΠΆΠ΅ ΠΊ ΠΈΡ ΠΎΠΏΠΎΡΠ½ΠΎΠΉ ΠΏΠ»ΠΎΡΠΊΠΎΡΡΠΈ (Ρ. Π΅. ΠΈΡΠΏΠΎΠ»ΡΠ·ΡΠΉΡΠ΅ Π±ΠΎΠ»Π΅Π΅ ΡΠΎΠ½ΠΊΠΈΠΉ Π»Π°ΠΌΠΈΠ½Π°Ρ Π΄Π»Ρ ΠΌΠΈΠΊΡΠΎΠΏΠΎΠ»ΠΎΡΠΊΠΎΠ²ΡΡ ΠΈΠ»ΠΈ ΠΏΠΎΠ»ΠΎΡΠΊΠΎΠ²ΡΡ ΡΡΠ°ΡΡ)
ΠΡΠ°Π²ΠΈΠ»ΡΠ½ΡΠΉ Π½Π°Π±ΠΎΡ ΠΈΠ½ΡΡΡΡΠΌΠ΅Π½ΡΠΎΠ² Π΄Π»Ρ ΡΡΠ°ΡΡΠΈΡΠΎΠ²ΠΊΠΈ ΠΈ ΠΏΡΠΎΠ΅ΠΊΡΠΈΡΠΎΠ²Π°Π½ΠΈΡ ΡΡΠ°ΡΡ ΠΏΠΎΠΌΠΎΠΆΠ΅Ρ Π²Π°ΠΌ ΠΏΠΎΠ΄Π΄Π΅ΡΠΆΠΈΠ²Π°ΡΡ ΡΡΠ΅Π±ΡΠ΅ΠΌΡΠΉ ΠΈΠΌΠΏΠ΅Π΄Π°Π½Ρ, Π° ΡΠ°ΠΊΠΆΠ΅ ΠΏΠΎΠΌΠΎΠΆΠ΅Ρ Π²Π°ΠΌ ΠΎΡΡΠ΅Π³ΡΠ»ΠΈΡΠΎΠ²Π°ΡΡ ΡΠ°Π·ΠΌΠ΅ΡΡ ΡΡΠ°ΡΡΡ, ΡΡΠΎΠ±Ρ ΠΎΡΡΠ°Π²Π°ΡΡΡΡ Π² ΠΏΡΠ΅Π΄Π΅Π»Π°Ρ ΡΡΠ΅Π±ΡΠ΅ΠΌΡΡ Π΄ΠΎΠΏΡΡΠΊΠΎΠ². Π£ Π²Π°Ρ ΡΠ°ΠΊΠΆΠ΅ Π±ΡΠ΄ΡΡ ΠΈΠ½ΡΡΡΡΠΌΠ΅Π½ΡΡ, Π½Π΅ΠΎΠ±Ρ ΠΎΠ΄ΠΈΠΌΡΠ΅ Π΄Π»Ρ Π°Π½Π°Π»ΠΈΠ·Π° ΠΏΠ΅ΡΠ΅ΠΊΡΠ΅ΡΡΠ½ΡΡ ΠΏΠΎΠΌΠ΅Ρ Π² Π³ΠΎΡΠΎΠ²ΠΎΠΉ ΡΠ°Π·Π²ΠΎΠ΄ΠΊΠ΅ ΠΏΠ΅ΡΠ°ΡΠ½ΠΎΠΉ ΠΏΠ»Π°ΡΡ. ΠΡ ΡΠ°ΠΊΠΆΠ΅ Π΄ΠΎΠ»ΠΆΠ½Ρ Π±ΡΡΡ ΡΠ²Π΅ΡΠ΅Π½Ρ, ΡΡΠΎ Π½Π°ΠΉΠ΄Π΅ΡΠ΅ ΠΏΡΠ°Π²ΠΈΠ»ΡΠ½ΡΡ Π΄ΠΈΡΡΠ΅ΡΠ΅Π½ΡΠΈΠ°Π»ΡΠ½ΡΡ ΡΠΎΡΠΌΡ Π² ΡΠ²ΠΎΠΈΡ ΡΡΠ°Π²Π½Π΅Π½ΠΈΡΡ Π΄Π»Ρ ΡΠ΅ΡΠ΅Π½ΠΈΡ Π»ΡΠ±ΡΡ Π·Π°Π΄Π°Ρ, ΡΠ°ΠΊΠΈΡ ΠΊΠ°ΠΊ Π²Π·Π°ΠΈΠΌΠ½Π°Ρ ΠΈΠ½Π΄ΡΠΊΡΠΈΡ ΠΈΠ»ΠΈ Π°Π½ΠΎΠΌΠ°Π»ΡΠ½ΡΠ΅ ΡΠΎΡΡΠΎΡΠ½ΠΈΡ ΡΠ»Π΅ΠΊΡΡΠΎΠΌΠ°Π³Π½ΠΈΡΠ½ΠΎΠΉ ΠΈΠ½Π΄ΡΠΊΡΠΈΠΈ, ΡΠ»Π΅ΠΊΡΡΠΎΠ»ΠΈΠ· ΠΈ Π²Π°ΡΠ° ΡΠ°Π·Π»ΠΈΡΠ½Π°Ρ ΠΈΠ½Π΄ΡΡΠΈΡΠΎΠ²Π°Π½Π½Π°Ρ ΡΠ»Π΅ΠΊΡΡΠΎΠ΄Π²ΠΈΠΆΡΡΠ°Ρ ΡΠΈΠ»Π°. ΠΠΏΡΠ΅Π΄Π΅Π»Π΅Π½ΠΈΠ΅ ΡΠ»Π΅ΠΊΡΡΠΎΠ΄Π²ΠΈΠΆΡΡΠ΅ΠΉ ΡΠΈΠ»Ρ, ΠΈΠ½Π΄ΡΡΠΈΡΠΎΠ²Π°Π½Π½ΠΎΠ³ΠΎ Π½Π°ΠΏΡΡΠΆΠ΅Π½ΠΈΡ ΠΈ ΠΏΠΎΠ»ΠΎΠΆΠ΅Π½ΠΈΡ Π²Π°ΡΠΈΡ ΡΠ΅ΠΏΠ΅ΠΉ Π² Π²ΠΈΡ ΡΠ΅Π²ΡΡ ΡΠΎΠΊΠ°Ρ β Π²ΡΠ΅ ΡΡΠΎ ΠΆΠΈΠ·Π½Π΅Π½Π½ΠΎ Π²Π°ΠΆΠ½ΠΎ Π΄Π»Ρ ΡΠ΅Π»ΠΎΡΡΠ½ΠΎΡΡΠΈ Π²Π°ΡΠ΅Π³ΠΎ ΠΏΡΠΎΠ΄ΡΠΊΡΠ°, ΠΎΡ Π΅Π³ΠΎ ΠΌΠ΅Π΄Π½ΠΎΠ³ΠΎ ΠΏΡΠΎΠ²ΠΎΠ΄Π° Π΄ΠΎ ΠΏΠ»Π°ΡΡ ΠΈ ΡΠ°ΠΌΠΎΠΉ ΡΠΏΠ°ΠΊΠΎΠ²ΠΊΠΈ.
ΠΡΠ»ΠΈ Π²Π°Ρ Π±Π΅ΡΠΏΠΎΠΊΠΎΠΈΡ Π·Π°ΠΊΠΎΠ½ ΠΠ΅Π½ΡΠ° ΠΈΠ»ΠΈ Π·Π°ΠΊΠΎΠ½ Π€Π°ΡΠ°Π΄Π΅Ρ, Π»ΡΡΡΠ΅Π΅ ΠΏΡΠΎΠ³ΡΠ°ΠΌΠΌΠ½ΠΎΠ΅ ΠΎΠ±Π΅ΡΠΏΠ΅ΡΠ΅Π½ΠΈΠ΅ Π΄Π»Ρ ΠΊΠΎΠΌΠΏΠΎΠ½ΠΎΠ²ΠΊΠΈ ΠΈ ΠΏΡΠΎΠ΅ΠΊΡΠΈΡΠΎΠ²Π°Π½ΠΈΡ ΠΏΠ΅ΡΠ°ΡΠ½ΡΡ ΠΏΠ»Π°Ρ, Π° ΡΠ°ΠΊΠΆΠ΅ ΠΏΠΎΠ»Π½ΡΠΉ Π½Π°Π±ΠΎΡ ΠΈΠ½ΡΡΡΡΠΌΠ΅Π½ΡΠΎΠ² Π°Π½Π°Π»ΠΈΠ·Π° ΠΏΠΎΠΌΠΎΠ³ΡΡ Π²Π°ΠΌ ΠΏΠΎΠ½ΡΡΡ, ΠΊΠ°ΠΊ ΡΡΠΈ Π΄Π²Π° ΡΡΡΠ΅ΠΊΡΠ° Π²Π»ΠΈΡΡΡ Π½Π° ΡΠ»Π΅ΠΊΡΡΠΎΠΌΠ°Π³Π½ΠΈΡΠ½ΠΎΠ΅ ΠΏΠΎΠ²Π΅Π΄Π΅Π½ΠΈΠ΅ Π²Π°ΡΠ΅ΠΉ ΠΏΠ΅ΡΠ°ΡΠ½ΠΎΠΉ ΠΏΠ»Π°ΡΡ. Allegro PCB Editor Π²ΠΊΠ»ΡΡΠ°Π΅Ρ Π² ΡΠ΅Π±Ρ ΡΡΠ½ΠΊΡΠΈΠΈ, Π½Π΅ΠΎΠ±Ρ ΠΎΠ΄ΠΈΠΌΡΠ΅ Π΄Π»Ρ ΠΏΠ»Π°Π½ΠΈΡΠΎΠ²Π°Π½ΠΈΡ ΠΊΠΎΠΌΠΏΠΎΠ½ΠΎΠ²ΠΊΠΈ ΠΏΠ»Π°ΡΡ Π΄Π»Ρ Π»ΡΠ±ΠΎΠ³ΠΎ ΠΏΡΠΈΠ»ΠΎΠΆΠ΅Π½ΠΈΡ, Π° ΡΠ°ΠΊΠΆΠ΅ ΡΠ°ΡΡΠΈΡΠ΅Π½Π½ΡΠ΅ ΠΈΠ½ΡΡΡΡΠΌΠ΅Π½ΡΡ ΠΏΡΠΎΠ²Π΅ΡΠΊΠΈ ΠΏΡΠΎΠ΅ΠΊΡΠ° ΠΈ ΡΡΠΈΠ»ΠΈΡΡ ΠΏΠΎΠ»Π΅Π²ΠΎΠ³ΠΎ ΡΠ΅ΡΠ°ΡΠ΅Π»Ρ Π΄Π»Ρ Π°Π½Π°Π»ΠΈΠ·Π° ΠΏΠΎΠ²Π΅Π΄Π΅Π½ΠΈΡ Π²Π°ΡΠ΅ΠΉ Π²ΡΡΠΎΠΊΠΎΡΠΊΠΎΡΠΎΡΡΠ½ΠΎΠΉ ΠΈ Π²ΡΡΠΎΠΊΠΎΡΠ°ΡΡΠΎΡΠ½ΠΎΠΉ ΡΠ»Π΅ΠΊΡΡΠΎΠ½ΠΈΠΊΠΈ.
ΠΡΠ»ΠΈ Π²Ρ Ρ ΠΎΡΠΈΡΠ΅ ΡΠ·Π½Π°ΡΡ Π±ΠΎΠ»ΡΡΠ΅ ΠΎ ΡΠΎΠΌ, ΠΊΠ°ΠΊΠΎΠ΅ ΡΠ΅ΡΠ΅Π½ΠΈΠ΅ Ρ Cadence Π΅ΡΡΡ Π΄Π»Ρ Π²Π°Ρ, ΠΎΠ±ΡΠ°ΡΠΈΡΠ΅ΡΡ ΠΊ Π½Π°ΠΌ ΠΈ Π½Π°ΡΠ΅ΠΉ ΠΊΠΎΠΌΠ°Π½Π΄Π΅ ΡΠΊΡΠΏΠ΅ΡΡΠΎΠ². Π§ΡΠΎΠ±Ρ ΠΏΠΎΡΠΌΠΎΡΡΠ΅ΡΡ Π²ΠΈΠ΄Π΅ΠΎ ΠΏΠΎ ΡΠ²ΡΠ·Π°Π½Π½ΡΠΌ ΡΠ΅ΠΌΠ°ΠΌ ΠΈΠ»ΠΈ ΡΠ·Π½Π°ΡΡ, ΡΡΠΎ Π½ΠΎΠ²ΠΎΠ³ΠΎ Π² Π½Π°ΡΠ΅ΠΌ Π½Π°Π±ΠΎΡΠ΅ ΠΈΠ½ΡΡΡΡΠΌΠ΅Π½ΡΠΎΠ² Π΄Π»Ρ ΠΏΡΠΎΠ΅ΠΊΡΠΈΡΠΎΠ²Π°Π½ΠΈΡ ΠΈ Π°Π½Π°Π»ΠΈΠ·Π°, ΠΏΠΎΠ΄ΠΏΠΈΡΠΈΡΠ΅ΡΡ Π½Π° Π½Π°ΡΒ ΠΊΠ°Π½Π°Π»Β YouTube.
Β
Π Π΅ΡΠ΅Π½ΠΈΡ Cadence PCB β ΡΡΠΎ ΠΊΠΎΠΌΠΏΠ»Π΅ΠΊΡΠ½ΡΠΉ ΠΈΠ½ΡΡΡΡΠΌΠ΅Π½Ρ Π΄Π»Ρ ΠΏΡΠΎΠ΅ΠΊΡΠΈΡΠΎΠ²Π°Π½ΠΈΡ ΠΎΡ Π½Π°ΡΠ°Π»Π° Π΄ΠΎ ΠΊΠΎΠ½ΡΠ°, ΠΏΠΎΠ·Π²ΠΎΠ»ΡΡΡΠΈΠΉ Π±ΡΡΡΡΠΎ ΠΈ ΡΡΡΠ΅ΠΊΡΠΈΠ²Π½ΠΎ ΡΠΎΠ·Π΄Π°Π²Π°ΡΡ ΠΏΡΠΎΠ΄ΡΠΊΡΡ. Cadence ΠΏΠΎΠ·Π²ΠΎΠ»ΡΠ΅Ρ ΠΏΠΎΠ»ΡΠ·ΠΎΠ²Π°ΡΠ΅Π»ΡΠΌ ΡΠΎΡΠ½ΠΎ ΡΠΎΠΊΡΠ°ΡΠΈΡΡ ΡΠΈΠΊΠ»Ρ ΠΏΡΠΎΠ΅ΠΊΡΠΈΡΠΎΠ²Π°Π½ΠΈΡ ΠΈ ΠΏΠ΅ΡΠ΅Π΄Π°ΡΡ ΠΈΡ Π² ΠΏΡΠΎΠΈΠ·Π²ΠΎΠ΄ΡΡΠ²ΠΎ Ρ ΠΏΠΎΠΌΠΎΡΡΡ ΡΠΎΠ²ΡΠ΅ΠΌΠ΅Π½Π½ΠΎΠ³ΠΎ ΠΎΡΡΠ°ΡΠ»Π΅Π²ΠΎΠ³ΠΎ ΡΡΠ°Π½Π΄Π°ΡΡΠ° IPC-2581.
ΠΠΎΠ΄ΠΏΠΈΡΠΈΡΠ΅ΡΡ Π½Π° Linkedin ΠΠΎΡΠ΅ΡΠΈΡΡ ΡΠ°ΠΉΡ ΠΠΎΠ»ΡΡΠ΅ ΠΊΠΎΠ½ΡΠ΅Π½ΡΠ° ΠΎΡ Cadence PCB Solutions
ΠΠ°Π³ΡΡΠ·ΠΊΠ°, ΠΏΠΎΠ΄ΠΎΠΆΠ΄ΠΈΡΠ΅
ΠΡΠΈΠ±ΠΊΠ° — ΡΡΠΎ-ΡΠΎ ΠΏΠΎΡΠ»ΠΎ Π½Π΅ ΡΠ°ΠΊ!
Π₯ΠΎΡΠΈΡΠ΅ ΠΏΠΎΡΠ»Π΅Π΄Π½ΠΈΠ΅ Π½ΠΎΠ²ΠΎΡΡΠΈ ΠΎ ΠΏΠ΅ΡΠ°ΡΠ½ΡΡ
ΠΏΠ»Π°ΡΠ°Ρ
?
ΠΠΎΠ΄ΠΏΠΈΡΠΈΡΠ΅ΡΡ Π½Π° Π½Π°ΡΡ Π΅ΠΆΠ΅ΠΌΠ΅ΡΡΡΠ½ΡΡ ΡΠ°ΡΡΡΠ»ΠΊΡ Π½ΠΎΠ²ΠΎΡΡΠ΅ΠΉ
Π‘ΠΏΠ°ΡΠΈΠ±ΠΎ!
Π£Π·Π½Π°ΠΉΡΠ΅ ΠΎΠ± ΠΠΠ‘ Π΄Π²ΠΈΠΆΠ΅Π½ΠΈΡ | Chegg.
comΠΠΏΡΠ΅Π΄Π΅Π»Π΅Π½ΠΈΠ΅ ΠΠΠ‘ Π΄Π²ΠΈΠΆΠ΅Π½ΠΈΡ
ΠΠ»Π΅ΠΊΡΡΠΎΠ΄Π²ΠΈΠΆΡΡΠ°Ρ ΡΠΈΠ»Π° Π΄Π²ΠΈΠΆΠ΅Π½ΠΈΡ ΠΎΠΏΡΠ΅Π΄Π΅Π»ΡΠ΅ΡΡΡ ΠΊΠ°ΠΊ ΡΠ»Π΅ΠΊΡΡΠΎΠ΄Π²ΠΈΠΆΡΡΠ°Ρ ΡΠΈΠ»Π°, ΠΊΠΎΡΠΎΡΠ°Ρ ΠΈΠ½Π΄ΡΡΠΈΡΡΠ΅ΡΡΡ ΠΈΠ·-Π·Π° Π΄Π²ΠΈΠΆΠ΅Π½ΠΈΡ ΠΏΡΠΎΠ²ΠΎΠ΄Π½ΠΈΠΊΠ° ΠΏΠΎΠΏΠ΅ΡΠ΅ΠΊ ΠΌΠ°Π³Π½ΠΈΡΠ½ΠΎΠ³ΠΎ ΠΏΠΎΠ»Ρ. Π£ΡΠ°Π²Π½Π΅Π½ΠΈΠ΅ Π΄Π»Ρ Π΄Π²ΠΈΠΆΡΡΠ΅ΠΉ ΡΠ»Π΅ΠΊΡΡΠΎΠ΄Π²ΠΈΠΆΡΡΠ΅ΠΉ ΡΠΈΠ»Ρ ΠΈΠΌΠ΅Π΅Ρ Π²ΠΈΠ΄
E=Blv E=BlvE=Blv
ΠΠ±Π·ΠΎΡ ΠΠΠ‘ Π΄Π²ΠΈΠΆΠ΅Π½ΠΈΡ
ΠΡΠΈ ΠΈΠ·ΠΌΠ΅Π½Π΅Π½ΠΈΠΈ ΠΌΠ°Π³Π½ΠΈΡΠ½ΠΎΠ³ΠΎ ΠΏΠΎΡΠΎΠΊΠ°, ΠΏΡΠΎΡ ΠΎΠ΄ΡΡΠ΅Π³ΠΎ ΡΠ΅ΡΠ΅Π· ΠΊΠ°ΡΡΡΠΊΡ ΠΈΠ»ΠΈ ΡΠ΅ΠΏΡ, Π²ΠΎΠ·Π½ΠΈΠΊΠ°Π΅Ρ ΡΠ»Π΅ΠΊΡΡΠΎΠ΄Π²ΠΈΠΆΡΡΠ°Ρ ΡΠΈΠ»Π° (ΠΠΠ‘), ΠΈ ΡΡΠ° ΠΠΠ‘ ΠΌΠΎΠΆΠ΅Ρ ΡΠΎΠ·Π΄Π°Π²Π°ΡΡ ΡΠΎΠΊ Π΅ΡΠ»ΠΈ ΠΎΠ½ ΠΏΡΠΈΠΌΠ΅Π½ΡΠ΅ΡΡΡ Π½Π°Π΄ ΠΏΡΠΎΠ²ΠΎΠ΄Π½ΠΈΠΊΠΎΠΌ. ΠΡΠΎ ΡΠ²Π»Π΅Π½ΠΈΠ΅ Π½Π°Π·ΡΠ²Π°Π΅ΡΡΡ ΡΠ»Π΅ΠΊΡΡΠΎΠΌΠ°Π³Π½ΠΈΡΠ½ΠΎΠΉ ΠΈΠ½Π΄ΡΠΊΡΠΈΠ΅ΠΉ. ΠΠ°Π³Π½ΠΈΡΠ½ΡΠΉ ΠΏΠΎΡΠΎΠΊ ΠΌΠΎΠΆΠ½ΠΎ ΠΈΠ·ΠΌΠ΅Π½ΠΈΡΡ Π·Π° ΡΡΠ΅Ρ Π΄Π²ΠΈΠΆΠ΅Π½ΠΈΡ ΠΏΡΠΎΠ²ΠΎΠ΄Π½ΠΈΠΊΠ° Π² ΠΌΠ°Π³Π½ΠΈΡΠ½ΠΎΠΌ ΠΏΠΎΠ»Π΅ ΠΈΠ»ΠΈ Π·Π° ΡΡΠ΅Ρ Π΄Π²ΠΈΠΆΠ΅Π½ΠΈΡ ΠΊΠ°ΡΡΡΠΊΠΈ Π² ΠΌΠ°Π³Π½ΠΈΡΠ½ΠΎΠΌ ΠΏΠΎΠ»Π΅. ΠΡΠΊΡΡΡΠΈΠ΅ ΡΠ²Π»Π΅Π½ΠΈΡ ΡΠ»Π΅ΠΊΡΡΠΎΠΌΠ°Π³Π½ΠΈΡΠ½ΠΎΠΉ ΠΈΠ½Π΄ΡΠΊΡΠΈΠΈ ΠΏΡΠΎΠΈΠ·Π²Π΅Π»ΠΎ ΡΠ΅Π²ΠΎΠ»ΡΡΠΈΡ Π² ΠΆΠΈΠ·Π½ΠΈ ΡΠ΅Π»ΠΎΠ²Π΅ΡΠ΅ΡΡΠ²Π°. Π§Π΅Π»ΠΎΠ²Π΅ΠΊ ΡΠ·Π½Π°Π», ΠΊΠ°ΠΊ ΡΠ½Π΅ΡΠ³ΠΈΡ Π΄Π²ΠΈΠΆΠ΅Π½ΠΈΡ ΠΌΠΎΠΆΠ΅Ρ Π±ΡΡΡ ΠΏΡΠ΅ΠΎΠ±ΡΠ°Π·ΠΎΠ²Π°Π½Π° Π² ΡΠ»Π΅ΠΊΡΡΠΈΡΠ΅ΡΠΊΡΡ ΡΠ½Π΅ΡΠ³ΠΈΡ. Π’Π°ΠΊΠΈΠΌ ΠΎΠ±ΡΠ°Π·ΠΎΠΌ, ΠΎΠ½ ΡΠ·Π½Π°Π», ΠΊΠ°ΠΊ ΠΌΠΎΠΆΠ½ΠΎ Π³Π΅Π½Π΅ΡΠΈΡΠΎΠ²Π°ΡΡ ΡΠ»Π΅ΠΊΡΡΠΈΡΠ΅ΡΡΠ²ΠΎ. ΠΠ°ΠΉΠΊΠ» Π€Π°ΡΠ°Π΄Π΅ΠΉ Π±ΡΠ» ΠΏΠ΅ΡΠ²ΡΠΌ, ΠΊΡΠΎ ΠΏΡΠΎΠ²Π΅Π» Π½Π΅ΡΠΊΠΎΠ»ΡΠΊΠΎ ΡΠΊΡΠΏΠ΅ΡΠΈΠΌΠ΅Π½ΡΠΎΠ² Π² ΡΡΠΎΠΉ ΠΎΠ±Π»Π°ΡΡΠΈ. ΠΠ° ΠΎΡΠ½ΠΎΠ²Π΅ ΡΡΠΈΡ ΠΎΠΏΡΡΠΎΠ² ΠΎΠ½ ΡΡΠΎΡΠΌΡΠ»ΠΈΡΠΎΠ²Π°Π» Π½Π΅ΡΠΊΠΎΠ»ΡΠΊΠΎ Π·Π°ΠΊΠΎΠ½ΠΎΠ², ΠΈΠ·Π²Π΅ΡΡΠ½ΡΡ ΡΠ°ΠΊΠΆΠ΅ ΠΊΠ°ΠΊ Π·Π°ΠΊΠΎΠ½Ρ ΡΠ»Π΅ΠΊΡΡΠΎΠΌΠ°Π³Π½ΠΈΡΠ½ΠΎΠΉ ΠΈΠ½Π΄ΡΠΊΡΠΈΠΈ Π€Π°ΡΠ°Π΄Π΅Ρ.
ΠΡΡΡ Π²ΠΎΠΏΡΠΎΡ ΠΏΠΎ ΡΡΠΎΠΉ ΡΠ΅ΠΌΠ΅?
Π§ΡΠΎ Π²Ρ ΡΠ·Π½Π°Π΅ΡΠ΅:
- ΠΠΏΡΠ΅Π΄Π΅Π»Π΅Π½ΠΈΠ΅ Π΄Π²ΠΈΠΆΠ΅Π½ΠΈΡ ΠΠΠ€
- ΠΠ±Π·ΠΎΡ Π΄Π²ΠΈΠΆΠ΅Π½ΠΈΡ EMF
- ΠΠ°ΠΊΠΎΠ½ Π€Π°ΡΠ°Π΄Π΅Ρ ΠΠ½Π΄ΡΠΊΡΠΈΠΈ:
- Π Π°Π·Π½ΠΈΡΠ° ΠΌΠ΅ΠΆΠ΄Ρ Π΄Π²ΠΈΠΆΠ΅Π½ΠΈΠ΅ΠΌ EMF ΠΈ ΠΈΠ½Π΄ΡΡΠΈΡΠΎΠ²Π°Π½Π½ΠΎΠΉ ΠΠΠ€:
- ΠΠ΅Ρ Π°Π½ΠΈΠ·ΠΌ ΡΠΎΡΠΌΠΈΡΠΎΠ²Π°Π½ΠΈΡ Π΄Π²ΠΈΠΆΠ΅Π½ΠΈΡ EMF:
- .
- . ΠΠΠ‘ Π΄Π²ΠΈΠΆΠ΅Π½ΠΈΡ:
- ΠΡΠ°ΠΊΡΠΈΡΠ΅ΡΠΊΠΈΠΉ Π²ΠΎΠΏΡΠΎΡ
ΠΠ°ΠΊΠΎΠ½ ΠΈΠ½Π΄ΡΠΊΡΠΈΠΈ Π€Π°ΡΠ°Π΄Π΅Ρ:
ΠΠ²ΠΎΠ΅ ΡΡΠ΅Π½ΡΡ , ΠΠ΅Π½ΡΠΈ ΠΈ Π€Π°ΡΠ°Π΄Π΅ΠΉ, ΠΏΡΠΎΠ²Π΅Π»ΠΈ ΡΠ°Π·Π»ΠΈΡΠ½ΡΠ΅ ΡΠΊΡΠΏΠ΅ΡΠΈΠΌΠ΅Π½ΡΡ Π΄Π»Ρ ΠΎΡΠΊΡΡΡΠΈΡ ΡΠ»Π΅ΠΊΡΡΠΎΠΌΠ°Π³Π½ΠΈΡΠ½ΠΎΠΉ ΠΈΠ½Π΄ΡΠΊΡΠΈΠΈ. ΠΠ· ΡΠΊΡΠΏΠ΅ΡΠΈΠΌΠ΅Π½ΡΠΎΠ² Π€Π°ΡΠ°Π΄Π΅ΠΉ ΡΠ΄Π΅Π»Π°Π» ΡΠ»Π΅Π΄ΡΡΡΠΈΠΉ Π²ΡΠ²ΠΎΠ΄, ΠΊΠΎΡΠΎΡΡΠΉ ΡΠ°ΠΊΠΆΠ΅ ΠΈΠ·Π²Π΅ΡΡΠ΅Π½ ΠΊΠ°ΠΊ Π·Π°ΠΊΠΎΠ½Ρ ΠΏΠΎ Π΅Π³ΠΎ ΠΈΠΌΠ΅Π½ΠΈ.
Β·Β Β ΠΠ΅ΡΠ²ΡΠΉ Π·Π°ΠΊΠΎΠ½ ΡΠ»Π΅ΠΊΡΡΠΎΠΌΠ°Π³Π½ΠΈΡΠ½ΠΎΠΉ ΠΈΠ½Π΄ΡΠΊΡΠΈΠΈ Π€Π°ΡΠ°Π΄Π΅Ρ:
ΠΠ΅ΡΠ²ΡΠΉ Π·Π°ΠΊΠΎΠ½ ΡΠ»Π΅ΠΊΡΡΠΎΠΌΠ°Π³Π½ΠΈΡΠ½ΠΎΠΉ ΠΈΠ½Π΄ΡΠΊΡΠΈΠΈ Π€Π°ΡΠ°Π΄Π΅Ρ ΡΡΠ²Π΅ΡΠΆΠ΄Π°Π΅Ρ, ΡΡΠΎ ΠΈΠ·ΠΌΠ΅Π½Π΅Π½ΠΈΠ΅ ΠΌΠ°Π³Π½ΠΈΡΠ½ΠΎΠ³ΠΎ ΠΏΠΎΡΠΎΠΊΠ°, ΡΠ²ΡΠ·Π°Π½Π½ΠΎΠ³ΠΎ Ρ ΡΠΈΡΡΠ΅ΠΌΠΎΠΉ, ΡΠ²ΡΠ·Π°Π½ΠΎ Ρ Π΄Π²ΠΈΠΆΠ΅Π½ΠΈΠ΅ΠΌ Π½Π΅ΠΊΠΎΡΠΎΡΡΡ Π΅Π΅ ΡΠ°ΡΡΠ΅ΠΉ ΠΈΠ»ΠΈ ΠΈΠ·ΠΌΠ΅Π½Π΅Π½ΠΈΠ΅ΠΌ ΠΌΠ°Π³Π½ΠΈΡΠ½ΠΎΠ³ΠΎ ΠΏΠΎΠ»Ρ. ΡΠ°ΠΌΠΎ ΠΏΠΎΠ»Π΅ ΠΈΠ»ΠΈ ΠΈΠ·-Π·Π° ΠΈΠ·ΠΌΠ΅Π½Π΅Π½ΠΈΡ Π½Π°ΠΏΡΠ°Π²Π»Π΅Π½ΠΈΡ ΠΌΠ°Π³Π½ΠΈΡΠ½ΠΎΠ³ΠΎ ΠΏΠΎΠ»Ρ Π² ΡΠΈΡΡΠ΅ΠΌΠ΅ Π²ΠΎΠ·Π½ΠΈΠΊΠ°Π΅Ρ ΠΠΠ‘. ΠΡΠ° ΠΠΠ‘ ΡΠΎΡ ΡΠ°Π½ΡΠ΅ΡΡΡ Π΄ΠΎ ΡΠ΅Ρ ΠΏΠΎΡ, ΠΏΠΎΠΊΠ° Π΄Π»ΠΈΡΡΡ ΡΠΊΠ°Π·Π°Π½Π½ΠΎΠ΅ Π²ΡΡΠ΅ ΠΈΠ·ΠΌΠ΅Π½Π΅Π½ΠΈΠ΅.
ΠΠ° ΡΠ»Π΅Π΄ΡΡΡΠ΅ΠΌ ΡΠΈΡΡΠ½ΠΊΠ΅ Π·Π°ΠΌΠΊΠ½ΡΡΠ°Ρ ΠΊΠ°ΡΡΡΠΊΠ° ΠΏΡΠΈΠ±Π»ΠΈΠΆΠ°Π΅ΡΡΡ ΠΊ ΠΏΠΎΠΊΠΎΡΡΠ΅ΠΌΡΡΡ ΠΌΠ°Π³Π½ΠΈΡΡ. ΠΠ·-Π·Π° Π΄Π²ΠΈΠΆΠ΅Π½ΠΈΡ ΠΊΠ°ΡΡΡΠΊΠΈ ΠΈΠ·ΠΌΠ΅Π½ΡΠ΅ΡΡΡ ΠΌΠ°Π³Π½ΠΈΡΠ½ΡΠΉ ΠΏΠΎΡΠΎΠΊ, ΠΏΡΠΎΡ ΠΎΠ΄ΡΡΠΈΠΉ ΡΠ΅ΡΠ΅Π· ΠΊΠ°ΡΡΡΠΊΡ. ΠΡΠΎ ΠΏΡΠΈΠ²ΠΎΠ΄ΠΈΡ ΠΊ Π²ΠΎΠ·Π½ΠΈΠΊΠ½ΠΎΠ²Π΅Π½ΠΈΡ ΡΠΎΠΊΠ° Π² ΠΊΠ°ΡΡΡΠΊΠ΅, ΠΊΠΎΡΠΎΡΡΠΉ ΠΌΠΎΠΆΠ½ΠΎ Π½Π°Π±Π»ΡΠ΄Π°ΡΡ ΠΊΠ°ΠΊ ΡΠ»ΡΠΊΡΡΠ°ΡΠΈΡ Π½Π° Π³Π°Π»ΡΠ²Π°Π½ΠΎΠΌΠ΅ΡΡΠ΅. ΠΠΎΠ³Π΄Π° ΠΊΠ°ΡΡΡΠΊΠ° ΠΎΡΠΎΠ΄Π²ΠΈΠ½ΡΠ² ΠΎΡ ΠΌΠ°Π³Π½ΠΈΡΠ° (ΡΡΠ°ΠΏ 2), ΡΠ½ΠΎΠ²Π° Π³Π΅Π½Π΅ΡΠΈΡΡΠ΅ΡΡΡ ΡΠΎΠΊ, ΠΏΠΎΠΊΠ°Π·ΡΠ²Π°ΡΡΠΈΠΉ ΡΠ»ΡΠΊΡΡΠ°ΡΠΈΠΈ Π² Π³Π°Π»ΡΠ²Π°Π½ΠΎΠΌΠ΅ΡΡΠ΅.
Β·Β Β ΠΡΠΎΡΠΎΠΉ Π·Π°ΠΊΠΎΠ½ ΡΠ»Π΅ΠΊΡΡΠΎΠΌΠ°Π³Π½ΠΈΡΠ½ΠΎΠΉ ΠΈΠ½Π΄ΡΠΊΡΠΈΠΈ Π€Π°ΡΠ°Π΄Π΅Ρ:
ΠΡΠΎΡΠΎΠΉ Π·Π°ΠΊΠΎΠ½ ΡΠ»Π΅ΠΊΡΡΠΎΠΌΠ°Π³Π½ΠΈΡΠ½ΠΎΠΉ ΠΈΠ½Π΄ΡΠΊΡΠΈΠΈ Π€Π°ΡΠ°Π΄Π΅Ρ Π³Π»Π°ΡΠΈΡ, ΡΡΠΎ ΠΈΠ½Π΄ΡΡΠΈΡΠΎΠ²Π°Π½Π½Π°Ρ ΡΠ»Π΅ΠΊΡΡΠΎΠ΄Π²ΠΈΠΆΡΡΠ°Ρ ΡΠΈΠ»Π° Π² ΠΊΠ°ΡΡΡΠΊΠ΅ ΠΏΡΠΎΠΏΠΎΡΡΠΈΠΎΠ½Π°Π»ΡΠ½Π° ΡΠΊΠΎΡΠΎΡΡΠΈ ΠΈΠ·ΠΌΠ΅Π½Π΅Π½ΠΈΡ ΡΠ²ΡΠ·Π°Π½Π½ΠΎΠ³ΠΎ ΠΏΠΎΡΠΎΠΊΠ°. ΠΠΎΡΠΎΠΊ ΠΎΠΏΡΠ΅Π΄Π΅Π»ΡΠ΅ΡΡΡ ΠΊΠ°ΠΊ ΠΏΡΠΎΠΈΠ·Π²Π΅Π΄Π΅Π½ΠΈΠ΅ ΠΌΠ°Π³Π½ΠΈΡΠ½ΠΎΠ³ΠΎ ΠΏΠΎΠ»Ρ, ΠΏΠ»ΠΎΡΠ°Π΄ΠΈ ΠΏΠΎΠ²Π΅ΡΡ Π½ΠΎΡΡΠΈ ΠΊΠ°ΡΡΡΠΊΠΈ ΠΈ ΠΎΠ±ΡΠ΅Π³ΠΎ ΡΠΈΡΠ»Π° Π²ΠΈΡΠΊΠΎΠ² ΠΊΠ°ΡΡΡΠΊΠΈ, ΡΠ²ΡΠ·Π°Π½Π½ΠΎΠΉ Ρ ΠΊΠ°ΡΡΡΠΊΠΎΠΉ.
Π Π°Π·Π½ΠΈΡΠ° ΠΌΠ΅ΠΆΠ΄Ρ ΠΠΠ‘ Π΄Π²ΠΈΠΆΠ΅Π½ΠΈΡ ΠΈ ΠΠΠ‘ ΠΈΠ½Π΄ΡΠΊΡΠΈΠΈ:
ΠΠΠ‘ ΠΈΠ½Π΄ΡΠΊΡΠΈΠΈ ΡΠΎΠ·Π΄Π°Π΅ΡΡΡ ΠΈΠ·ΠΌΠ΅Π½Π΅Π½ΠΈΠ΅ΠΌ ΠΌΠ°Π³Π½ΠΈΡΠ½ΠΎΠ³ΠΎ ΠΏΠΎΡΠΎΠΊΠ° ΡΠ΅ΡΠ΅Π· Π»ΡΠ±ΠΎΠΉ ΠΊΠΎΠ½ΡΡΡ. ΠΠ΅ΡΠ»ΠΈ, ΠΊΠ°ΠΊ ΠΏΡΠ°Π²ΠΈΠ»ΠΎ, Π·Π°ΠΊΡΡΡΡ. Π ΠΊΠ°ΡΡΡΠΊΠ΅ Π±ΡΠ΄Π΅Ρ ΡΠ΅ΡΡ ΡΠΎΠΊ ΠΈΠ·-Π·Π° ΠΠΠ‘ ΠΈΠ½Π΄ΡΠΊΡΠΈΠΈ. ΠΠ·ΠΌΠ΅Π½Π΅Π½ΠΈΠ΅ ΠΌΠ°Π³Π½ΠΈΡΠ½ΠΎΠ³ΠΎ ΠΏΠΎΡΠΎΠΊ ΡΠ°ΠΊΠΆΠ΅ Π²ΠΎΠ·Π½ΠΈΠΊΠ°Π΅Ρ ΠΈΠ·-Π·Π° Π΄Π²ΠΈΠΆΠ΅Π½ΠΈΡ ΠΏΡΠΎΠ²ΠΎΠ΄Π½ΠΈΠΊΠ° Π² ΠΌΠ°Π³Π½ΠΈΡΠ½ΠΎΠΌ ΠΏΠΎΠ»Π΅. ΠΡΠΎΠ²ΠΎΠ΄Π½ΠΈΠΊ ΠΌΠΎΠΆΠ΅Ρ ΠΈΠΌΠ΅ΡΡ ΡΠΎΡΠΌΡ ΠΏΡΡΠΌΠΎΠ³ΠΎ ΡΡΠ΅ΡΠΆΠ½Ρ ΠΈΠ»ΠΈ ΠΏΡΠΎΠ²ΠΎΠ»ΠΎΠΊΠΈ. ΠΠΠ‘, ΠΈΠ½Π΄ΡΡΠΈΡΠΎΠ²Π°Π½Π½Π°Ρ Π½Π° Π΄Π²ΡΡ ΠΊΠΎΠ½ΡΠ°Ρ ΠΏΡΠΎΠ²ΠΎΠ΄Π° ΠΈΠ»ΠΈ ΠΏΡΠΎΠ²ΠΎΠ΄Π½ΠΈΠΊΠ°, ΠΈΠ·Π²Π΅ΡΡΠ½Π° ΠΊΠ°ΠΊ ΠΠΠ‘ Π΄Π²ΠΈΠΆΠ΅Π½ΠΈΡ 9.0034
ΠΠ΅Ρ Π°Π½ΠΈΠ·ΠΌ ΡΠΎΡΠΌΠΈΡΠΎΠ²Π°Π½ΠΈΡ ΠΠΠ‘ Π΄Π²ΠΈΠΆΠ΅Π½ΠΈΡ:
ΠΡΠ΅Π΄ΠΏΠΎΠ»ΠΎΠΆΠΈΠΌ, ΡΡΠΎ ΠΏΡΡΠΌΠΎΠΉ ΠΏΡΠΎΠ²ΠΎΠ΄Π½ΠΈΠΊ Π΄Π²ΠΈΠΆΠ΅ΡΡΡ Π² ΠΏΡΠ°Π²ΠΈΠ»ΡΠ½ΠΎΠΌ Π½Π°ΠΏΡΠ°Π²Π»Π΅Π½ΠΈΠΈ. Π ΠΎΠ±Π»Π°ΡΡΠΈ Π»Π΅ΠΆΠΈΡ Π²Π΅ΡΡΠΈΠΊΠ°Π»ΡΠ½ΠΎΠ΅ ΠΌΠ°Π³Π½ΠΈΡΠ½ΠΎΠ΅ ΠΏΠΎΠ»Π΅, ΠΈ ΠΏΡΠΎΠ²ΠΎΠ΄Π½ΠΈΠΊ ΠΏΠ΅ΡΠ΅ΠΌΠ΅ΡΠ°Π΅ΡΡΡ ΠΏΠ΅ΡΠΏΠ΅Π½Π΄ΠΈΠΊΡΠ»ΡΡΠ½ΠΎ ΠΌΠ°Π³Π½ΠΈΡΠ½ΠΎΠΌΡ ΠΏΠΎΠ»Ρ. ΠΠΎΠ±ΠΈΠ»ΡΠ½ΡΠΉ ΡΠ»Π΅ΠΊΡΡΠΎΠ½Ρ Π² ΠΏΡΠΎΠ²ΠΎΠ΄Π½ΠΈΠΊΠ΅ ΠΏΠ΅ΡΠ΅ΠΌΠ΅ΡΠ°ΡΡΡΡ Π² ΠΌΠ°Π³Π½ΠΈΡΠ½ΠΎΠΌ ΠΏΠΎΠ»Π΅ Π² Π½Π°ΠΏΡΠ°Π²Π»Π΅Π½ΠΈΠΈ Π΄Π²ΠΈΠΆΠ΅Π½ΠΈΡ ΠΏΡΠΎΠ²ΠΎΠ΄Π½ΠΈΠΊΠ°. ΠΡΠΎ ΡΠΊΠ²ΠΈΠ²Π°Π»Π΅Π½ΡΠ½ΠΎ Π΄Π²ΠΈΠΆΠ΅Π½ΠΈΡ Π·Π°ΡΡΠ΄Π° Π² ΠΌΠ°Π³Π½ΠΈΡΠ½ΠΎΠΌ ΠΏΠΎΠ»Π΅. ΠΠ½ ΡΠΎΠ·Π΄Π°Π΅Ρ ΡΠΈΠ»Ρ Π½Π° ΡΠ»Π΅ΠΊΡΡΠΎΠ½Π°Ρ . ΠΡΠΈ ΡΠΈΠ»Ρ ΡΠΎΠ»ΠΊΠ°ΡΡ ΡΠ»Π΅ΠΊΡΡΠΎΠ½Ρ ΠΊ ΠΎΠ΄Π½ΠΎΠΌΡ ΠΊΠΎΠ½ΡΡ ΠΏΡΠΎΠ²ΠΎΠ΄Π½ΠΈΠΊΠ°. ΠΡΠΎ ΠΏΡΠΈΠ²ΠΎΠ΄ΠΈΡ ΠΊ ΡΠΎΠΌΡ, ΡΡΠΎ ΠΎΠ΄ΠΈΠ½ ΠΈΠ· ΠΊΠΎΠ½ΡΠΎΠ² ΠΏΡΠΎΠ²ΠΎΠ΄Π½ΠΈΠΊΠ° ΡΡΠ°Π½ΠΎΠ²ΠΈΡΡΡ Π±ΠΎΠ»Π΅Π΅ ΠΎΡΡΠΈΡΠ°ΡΠ΅Π»ΡΠ½ΡΠΌ. ΠΡΡΠ΅ΡΡΠ²Π΅Π½Π½ΠΎ Π΄ΡΡΠ³ΠΎΠΉ ΠΊΠΎΠ½Π΅Ρ ΡΡΠ°Π½ΠΎΠ²ΠΈΡΡΡ ΠΏΠΎΠ»ΠΎΠΆΠΈΡΠ΅Π»ΡΠ½ΡΠΌ. Π’Π°ΠΊΠΈΠΌ ΠΎΠ±ΡΠ°Π·ΠΎΠΌ ΡΠΎΠ·Π΄Π°Π΅ΡΡΡ ΡΠ°Π·Π½ΠΎΡΡΡ ΠΏΠΎΡΠ΅Π½ΡΠΈΠ°Π»ΠΎΠ² Π½Π° Π΄Π²ΡΡ ΠΊΠΎΠ½ΡΠ°Ρ ΠΏΡΠΎΠ²ΠΎΠ΄Π½ΠΈΠΊΠ°. ΠΠ½Π° ΠΈΠ·Π²Π΅ΡΡΠ½Π° ΠΊΠ°ΠΊ Π΄Π²ΠΈΠΆΡΡΠ°Ρ ΠΠΠ‘. ΠΡΠ»ΠΈ ΠΌΡ ΡΠΎΠ΅Π΄ΠΈΠ½ΠΈΠΌ Π΄Π²Π° ΠΊΠΎΠ½ΡΠ° ΠΏΡΠΎΠ²ΠΎΠ΄ΠΎΠΌ, ΠΏΠΎ ΠΏΡΠΎΠ²ΠΎΠ΄Ρ ΠΏΠΎΡΠ΅ΡΠ΅Ρ ΡΠΎΠΊ. ΠΡΠΎ Π½Π°Π·ΡΠ²Π°Π΅ΡΡΡ ΠΈΠ½Π΄ΡΡΠΈΡΠΎΠ²Π°Π½Π½ΡΠΌ ΡΠΎΠΊΠΎΠΌ.
Π Π°ΡΡΠ΅Ρ ΠΠΠ‘ Π΄Π²ΠΈΠΆΠ΅Π½ΠΈΡ:
ΠΠΎΠΆΠ½ΠΎ ΡΠ°ΡΡΡΠΈΡΠ°ΡΡ ΠΠΠ‘, Π²ΠΎΠ·Π½ΠΈΠΊΠ°ΡΡΡΡ Π² ΠΏΡΠΎΠ²ΠΎΠ΄Π½ΠΈΠΊΠ΅ ΠΈΠ·-Π·Π° Π΅Π³ΠΎ Π΄Π²ΠΈΠΆΠ΅Π½ΠΈΡ Π² ΠΌΠ°Π³Π½ΠΈΡΠ½ΠΎΠΌ ΠΏΠΎΠ»Π΅, ΡΠ»Π΅Π΄ΡΡΡΠΈΠΌ ΠΎΠ±ΡΠ°Π·ΠΎΠΌ:
ΠΡΡΡΡ ΠΌΠ°Π³Π½ΠΈΡΠ½ΠΎΠ΅ ΠΏΠΎΠ»Π΅ ΡΠ°Π²Π½ΠΎ B, Π΄Π»ΠΈΠ½Π° ΠΏΡΠΎΠ²ΠΎΠ΄Π½ΠΈΠΊΠ° L ΠΈ Π΅Π³ΠΎ ΡΠΊΠΎΡΠΎΡΡΡ Π±ΡΡΡ Π.
ΠΠ·ΠΌΠ΅Π½Π΅Π½ΠΈΠ΅ ΠΌΠ°Π³Π½ΠΈΡΠ½ΠΎΠ³ΠΎ ΠΏΠΎΡΠΎΠΊΠ° Π² ΡΡΠΎΠΌ ΡΠ»ΡΡΠ°Π΅ ΡΠ°ΠΊΡΠΈΡΠ΅ΡΠΊΠΈ ΡΠ°ΡΡΡΠΈΡΡΠ²Π°Π΅ΡΡΡ ΠΏΡΡΠ΅ΠΌ ΡΠ°ΡΡΠ΅ΡΠ° ΠΏΠΎΡΠΎΠΊΠ°, ΠΊΠΎΡΠΎΡΡΠΉ ΠΎΡΡΠ΅ΠΊΠ°Π΅Ρ ΡΡΠ΅ΡΠΆΠ΅Π½Ρ. ΠΠΎΠ³Π΄Π° ΠΏΡΠΎΠ²ΠΎΠ΄Π½ΠΈΠΊ Π΄Π²ΠΈΠΆΠ΅ΡΡΡ Π² ΠΌΠ°Π³Π½ΠΈΡΠ½ΠΎΠΌ ΠΏΠΎΠ»Π΅, ΠΎΠ½ ΠΏΠ΅ΡΠ΅ΡΠ΅ΠΊΠ°Π΅Ρ ΡΠΈΠ»ΠΎΠ²ΡΠ΅ Π»ΠΈΠ½ΠΈΠΈ, Π±Π΅Π³ΡΡ ΠΏΠ΅ΡΠΏΠ΅Π½Π΄ΠΈΠΊΡΠ»ΡΡΠ½ΠΎ Π²Π½ΠΈΠ·.
ΠΠ»ΠΎΡΠ°Π΄Ρ, ΠΏΠΎΠΊΡΡΠ²Π°Π΅ΠΌΠ°Ρ ΡΡΠ΅ΡΠΆΠ½Π΅ΠΌ Π·Π° ΠΎΠ΄Π½Ρ ΡΠ΅ΠΊΡΠ½Π΄Ρ = L x V
ΠΠΎΡΠΎΠΊ, ΠΏΠ΅ΡΠ΅ΡΠ΅ΠΊΠ°ΡΡΠΈΠΉ ΡΡΡ ΠΏΠ»ΠΎΡΠ°Π΄Ρ = B x L x V
ΠΡΠ°ΠΊ, ΠΏΠΎΡΠΎΠΊ, ΠΏΠ΅ΡΠ΅ΡΠ΅ΠΊΠ°Π΅ΠΌΡΠΉ ΠΏΡΠΎΠ²ΠΎΠ΄Π½ΠΈΠΊΠΎΠΌ Π² ΡΠ΅ΠΊΡΠ½Π΄Ρ = BLV
Π‘ΠΎΠ³Π»Π°ΡΠ½ΠΎ Π·Π°ΠΊΠΎΠ½Ρ Π€Π°ΡΠ°Π΄Π΅Ρ ΡΡΠΎ ΡΠ°Π²Π½ΠΎ ΠΠΠ‘ ΠΈΠ½Π΄ΡΡΠΈΡΠΎΠ²Π°Π½Π½Π°Ρ
ΠΠΠ‘ Π΄Π²ΠΈΠΆΠ΅Π½ΠΈΡ, ΠΈΠ½Π΄ΡΡΠΈΡΠΎΠ²Π°Π½Π½Π°Ρ Π² ΠΏΡΠΎΠ²ΠΎΠ΄Π½ΠΈΠΊΠ΅ = BLV.
ΠΡΠ°ΠΊΡΠΈΡΠ΅ΡΠΊΠΈΠΉ Π²ΠΎΠΏΡΠΎΡ
Π’Π΅ΠΏΠ΅ΡΡ ΠΏΠΎΠΏΡΠΎΠ±ΡΠΉΡΠ΅ ΡΠ°ΠΌΠΈ ΠΈ ΠΏΡΠΈΠΌΠ΅Π½ΠΈΡΠ΅ ΠΏΠΎΠ»ΡΡΠ΅Π½Π½ΡΠ΅ Π·Π½Π°Π½ΠΈΡ ΠΊ ΠΏΡΠ°ΠΊΡΠΈΡΠ΅ΡΠΊΠΎΠΌΡ Π²ΠΎΠΏΡΠΎΡΡ Π½ΠΈΠΆΠ΅.
ΠΡΠΎΠ²ΠΎΠ΄ΡΡΠΈΠΉ ΡΡΠ΅ΡΠΆΠ΅Π½Ρ Π΄Π»ΠΈΠ½ΠΎΠΉ Π΄Π²ΠΈΠΆΠ΅ΡΡΡ ΡΠΎ ΡΠΊΠΎΡΠΎΡΡΡΡ ΠΏΠΎ U-ΠΎΠ±ΡΠ°Π·Π½ΠΎΠΉ Π½Π°ΠΏΡΠ°Π²Π»ΡΡΡΠ΅ΠΉ, ΠΏΠΎΠΌΠ΅ΡΠ΅Π½Π½ΠΎΠΉ Π² ΠΌΠ°Π³Π½ΠΈΡΠ½ΠΎΠ΅ ΠΏΠΎΠ»Π΅ Π½Π°ΠΏΡΡΠΆΠ΅Π½Π½ΠΎΡΡΡΡ, Π½Π°ΠΏΡΠ°Π²Π»Π΅Π½Π½ΠΎΠΉ Π² ΡΡΠΎΡΠΎΠ½Ρ ΡΡΡΠ°Π½ΠΈΡΡ. ΠΠ°ΠΉΠ΄ΠΈΡΠ΅ ΠΌΠ°ΠΊΡΠΈΠΌΠ°Π»ΡΠ½ΡΡ ΠΠΠ‘ ΠΈΠ½Π΄ΡΠΊΡΠΈΠΈ Π² ΡΡΠ΅ΡΠΆΠ½Π΅.
ΠΡΠ±Π΅ΡΠΈΡΠ΅ ΠΎΡΠ²Π΅ΡΠ²ΡΠ±ΠΎΡ AA
Π²ΡΠ±ΠΎΡ BB
Π²ΡΠ±ΠΎΡ CC
Π²ΡΠ±ΠΎΡ DD
ΠΠ°ΠΈΠ½ΡΠ΅ΡΠ΅ΡΠΎΠ²Π°Π½Ρ Π² ΡΠΎΠΌ, ΡΡΠΎΠ±Ρ ΠΏΡΠ°ΠΊΡΠΈΠΊΠΎΠ²Π°ΡΡ Π±ΠΎΠ»ΡΡΠ΅ ΠΏΠΎΠ΄ΠΎΠ±Π½ΡΡ Π²ΠΎΠΏΡΠΎΡΠΎΠ² ΠΎΠ± ΠΠΠ‘ Π΄Π²ΠΈΠΆΠ΅Π½ΠΈΡ?
ΠΡΠ°ΠΊΡΠΈΠΊΡΠΉΡΠ΅ΡΡ Π±ΠΎΠ»ΡΡΠ΅
ΠΡΠΎΠ΄ΠΎΠ»ΠΆΠ°ΠΉΡΠ΅ ΡΡΠΈΡΡΡΡ
Π§ΡΠΎ ΠΈΠ·ΡΡΠ°ΡΡ Π΄Π°Π»ΡΡΠ΅ Π½Π° ΠΎΡΠ½ΠΎΠ²Π΅ ΡΡΠ΅Π±Π½ΠΎΠΉ ΠΏΡΠΎΠ³ΡΠ°ΠΌΠΌΡ ΠΊΠΎΠ»Π»Π΅Π΄ΠΆΠ°0001
Π‘ΠΏΡΠΎΡΠΈΠ»
ΠΠ·ΠΌΠ΅Π½Π΅Π½ΠΎ 2 Π³ΠΎΠ΄Π°, 3 ΠΌΠ΅ΡΡΡΠ° Π½Π°Π·Π°Π΄
ΠΡΠΎΡΠΌΠΎΡΡΠ΅Π½ΠΎ 2ΠΊ ΡΠ°Π·
$\begingroup$
ΠΠΎΠ³Π΄Π° ΠΏΡΠΎΠ²ΠΎΠ΄ΡΡΠΈΠΉ ΡΡΠ΅ΡΠΆΠ΅Π½Ρ Π΄Π²ΠΈΠΆΠ΅ΡΡΡ Π² ΠΎΠ΄Π½ΠΎΡΠΎΠ΄Π½ΠΎΠΌ ΠΌΠ°Π³Π½ΠΈΡΠ½ΠΎΠΌ ΠΏΠΎΠ»Π΅, ΠΊΠ°ΠΊ ΠΏΠΎΠΊΠ°Π·Π°Π½ΠΎ Π½Π° ΡΠΈΡΡΠ½ΠΊΠ΅.
Π‘ ΠΏΠΎΠΌΠΎΡΡΡ ΡΠΈΠ»Ρ ΠΠΎΡΠ΅Π½ΡΠ° Π»Π΅Π³ΠΊΠΎ ΠΎΠ±ΡΡΡΠ½ΠΈΡΡ, ΡΡΠΎ ΠΈΠ½Π΄ΡΡΠΈΡΠΎΠ²Π°Π½Π½Π°Ρ ΠΠΠ‘ ΡΠ°Π²Π½Π° BvL ΠΈ Π²Π΅ΡΡ Π½ΠΈΠΉ ΠΊΠΎΠ½Π΅Ρ ΠΏΠΎΠ»ΠΎΠΆΠΈΡΠ΅Π»ΡΠ½ΡΠΉ, Π° Π½ΠΈΠΆΠ½ΠΈΠΉ ΠΎΡΡΠΈΡΠ°ΡΠ΅Π»ΡΠ½ΡΠΉ.
ΠΠΎ Π² ΠΊΠ½ΠΈΠ³Π°Ρ ΡΡΠ° ΠΊΠΎΠ½ΡΠ΅ΠΏΡΠΈΡ ΠΎΠ±ΡΡΡΠ½ΡΠ΅ΡΡΡ Π·Π°ΠΊΠΎΠ½ΠΎΠΌ Π€Π°ΡΠ°Π΄Π΅Ρ ΠΎΠ± ΡΠ»Π΅ΠΊΡΡΠΎΠΌΠ°Π³Π½ΠΈΡΠ½ΠΎΠΉ ΠΈΠ½Π΄ΡΠΊΡΠΈΠΈ, ΡΠ°ΠΊ ΠΊΠ°ΠΊ ΠΏΠ»ΠΎΡΠ°Π΄Ρ, Π·Π°ΠΌΠ΅Π½ΡΠ΅ΠΌΠ°Ρ ΠΏΡΠΎΠ²ΠΎΠ΄Π½ΠΈΠΊΠΎΠΌ, ΠΈΠ·ΠΌΠ΅Π½ΡΠ΅ΡΡΡ ΠΈ Π²ΠΎΠ·Π½ΠΈΠΊΠ°Π΅Ρ ΠΠΠ‘. ΠΠΎ ΠΏΠΎΡΠ΅ΠΌΡ ΠΌΡ ΠΏΡΠΈΠ½ΠΈΠΌΠ°Π΅ΠΌ Π²ΠΎ Π²Π½ΠΈΠΌΠ°Π½ΠΈΠ΅ ΠΏΠ»ΠΎΡΠ°Π΄Ρ, Π·Π°ΠΌΠ΅Π½Π΅Π½Π½ΡΡ ΠΌΠ΅ΡΡΠ°ΠΌΠΈ?
Π― Π΄ΡΠΌΠ°Ρ, ΡΡΠΎ ΠΌΠ°Π³Π½ΠΈΡΠ½ΡΠΉ ΠΏΠΎΡΠΎΠΊ ΡΠ΅ΡΠ΅Π· ΠΏΡΠΎΠ²ΠΎΠ΄Π½ΠΈΠΊ ΠΎΡΡΠ°Π΅ΡΡΡ ΠΏΠΎΡΡΠΎΡΠ½Π½ΡΠΌ, ΡΠ°ΠΊ ΠΊΠ°ΠΊ B ΠΏΠΎΡΡΠΎΡΠ½Π½Π°. Π― Π½Π΅ ΠΌΠΎΠ³Ρ ΠΎΠ±ΠΎΡΠ½ΠΎΠ²Π°ΡΡ ΡΡΡ ΠΊΠΎΠ½ΡΠ΅ΠΏΡΠΈΡ Ρ ΠΏΠΎΠΌΠΎΡΡΡ Π·Π°ΠΊΠΎΠ½Π° Π€Π°ΡΠ°Π΄Π΅Ρ (ΠΏΠΎ ΠΏΠ»ΠΎΡΠ°Π΄ΠΈ ΠΌΠ΅ΡΡΠ°ΠΌΠΈ). ΠΠΎΡΠ΅ΠΌΡ ΠΈΡΠΏΠΎΠ»ΡΠ·ΡΠ΅ΡΡΡ ΠΌΠ΅ΡΠΎΠ΄ Π·Π°ΠΌΠ΅Π½Ρ ΠΎΠ±Π»Π°ΡΡΠΈ? ΠΠΎΠΆΠ°Π»ΡΠΉΡΡΠ° ΠΏΠΎΠΌΠΎΠ³ΠΈ.
- ΡΠ»Π΅ΠΊΡΡΠΎΠΌΠ°Π³Π½Π΅ΡΠΈΠ·ΠΌ
- ΠΌΠ°Π³Π½ΠΈΡΠ½ΡΠ΅ ΠΏΠΎΠ»Ρ
- ΡΠ»Π΅ΠΊΡΡΠΎΠΌΠ°Π³Π½ΠΈΡΠ½Π°Ρ ΠΈΠ½Π΄ΡΠΊΡΠΈΡ
$\endgroup$
4
$\begingroup$
«Π― Π΄ΡΠΌΠ°Ρ, ΡΡΠΎ ΠΌΠ°Π³Π½ΠΈΡΠ½ΡΠΉ ΠΏΠΎΡΠΎΠΊ ΡΠ΅ΡΠ΅Π· ΠΏΡΠΎΠ²ΠΎΠ΄Π½ΠΈΠΊ ΠΎΡΡΠ°Π΅ΡΡΡ ΠΏΠΎΡΡΠΎΡΠ½Π½ΡΠΌ, ΡΠ°ΠΊ ΠΊΠ°ΠΊ B ΠΏΠΎΡΡΠΎΡΠ½Π½Π°.»
ΠΠ½Π°ΡΠ΅Π½ΠΈΠ΅ ΠΈΠΌΠ΅Π΅Ρ Π½Π΅ ΠΏΠΎΡΠΎΠΊ «ΡΠ΅ΡΠ΅Π· ΠΏΡΠΎΠ²ΠΎΠ΄Π½ΠΈΠΊ». ΠΡΠΎ ΠΏΠΎΡΠΎΠΊ ΡΠ΅ΡΠ΅Π· ΠΎΠ±Π»Π°ΡΡΡ , Π²ΡΠΌΠ΅ΡΠ°Π΅ΠΌΡΠΉ ΠΏΡΠΎΠ²ΠΎΠ΄Π½ΠΈΠΊΠΎΠΌ ΠΈΠ· ΠΎΠ±Π»Π°ΡΡΠΈ . ΠΡΠ΅Π΄ΡΡΠ°Π²ΡΡΠ΅, ΡΡΠΎ ΠΏΡΡΠΌΠΎΠΉ ΠΏΡΠΎΠ²ΠΎΠ΄Π½ΠΈΠΊ (Π΄Π»ΠΈΠ½Π° $\ell$) Π»Π΅ΠΆΠΈΡ Π½Π° ΡΡΠΎΠ»Π΅, ΠΈ Π½Π° Π½Π΅Π³ΠΎ Π΄Π΅ΠΉΡΡΠ²ΡΠ΅Ρ ΠΎΠ΄Π½ΠΎΡΠΎΠ΄Π½ΠΎΠ΅ ΠΌΠ°Π³Π½ΠΈΡΠ½ΠΎΠ΅ ΠΏΠΎΠ»Π΅. (ΠΠ° ΡΠ°ΠΌΠΎΠΌ Π΄Π΅Π»Π΅ Π΅ΡΡΡ: Π²Π΅ΡΡΠΈΠΊΠ°Π»ΡΠ½Π°Ρ ΡΠΎΡΡΠ°Π²Π»ΡΡΡΠ°Ρ ΠΏΠΎΠ»Ρ ΠΠ΅ΠΌΠ»ΠΈ.) ΠΠ°ΡΠ΅ΠΌ Π²Ρ ΠΏΠ΅ΡΠ΅ΠΌΠ΅ΡΠ°Π΅ΡΠ΅ ΠΏΡΠΎΠ²ΠΎΠ΄Π½ΠΈΠΊ ΠΏΠΎ ΡΡΠΎΠ»Ρ ΡΠΎ ΡΠΊΠΎΡΠΎΡΡΡΡ v Π² Π½Π°ΠΏΡΠ°Π²Π»Π΅Π½ΠΈΠΈ, ΠΏΠ΅ΡΠΏΠ΅Π½Π΄ΠΈΠΊΡΠ»ΡΡΠ½ΠΎΠΌ ΡΠ°ΠΌΠΎΠΌΡ ΡΠ΅Π±Π΅. ΠΠ° Π²ΡΠ΅ΠΌΡ $\Delta t$ Π·Π°ΠΌΠ΅ΡΠ°Π΅Ρ ΠΏΠ»ΠΎΡΠ°Π΄Ρ $\ell v \Delta t$
ΠΠΎΡΠΎΠΊ ΡΠ΅ΡΠ΅Π· Π·Π°ΠΌΠ΅ΡΠ°Π΅ΠΌΡΡ ΠΏΠ»ΠΎΡΠ°Π΄Ρ ΡΠ°Π²Π΅Π½ $$\Delta \Phi = (\ell v \Delta t)B$$
Π’Π°ΠΊΠΈΠΌ ΠΎΠ±ΡΠ°Π·ΠΎΠΌ, ΡΠΎΠ³Π»Π°ΡΠ½ΠΎ Π·Π°ΠΊΠΎΠ½Ρ Π€Π°ΡΠ°Π΄Π΅Ρ, ΠΠΠ‘ ΠΈΠ½Π΄ΡΠΊΡΠΈΠΈ ΡΠ°Π²Π½Π° $$\mathscr E=\frac {\Delta \Phi}{\Delta t}=\frac {(\ell v \Delta t)B}{\Delta t}=B\ell v$$ ΠΡΠ°ΠΊ, ΠΌΡ Π²ΠΎΡΡΡΠ°Π½ΠΎΠ²ΠΈΠ»ΠΈ ΡΠ΅Π·ΡΠ»ΡΡΠ°Ρ, ΠΊΠΎΡΠΎΡΡΠΉ Π²Ρ ΠΏΠΎΠ»ΡΡΠΈΠ»ΠΈ ΠΈΠ· ΠΌΠ°Π³Π½ΠΈΡΠ½ΠΎΠΉ ΡΠΈΠ»Ρ ΠΠΎΡΠ΅Π½ΡΠ°. ΠΠ° ΠΌΠΎΠΉ Π²Π·Π³Π»ΡΠ΄, ΠΌΠ°Π³Π½ΠΈΡΠ½Π°Ρ ΡΠΈΠ»Π° ΠΠΎΡΠ΅Π½ΡΠ° Π±ΠΎΠ»Π΅Π΅ ΡΡΠ½Π΄Π°ΠΌΠ΅Π½ΡΠ°Π»ΡΠ½Π°, ΡΠ΅ΠΌ Π·Π°ΠΊΠΎΠ½ Π€Π°ΡΠ°Π΄Π΅Ρ, ΠΊΠΎΠ³Π΄Π° ΠΠΠ‘ Π²ΠΎΠ·Π½ΠΈΠΊΠ°Π΅Ρ ΠΈΠ·-Π·Π° Π΄Π²ΠΈΠΆΠ΅Π½ΠΈΡ ΠΏΡΠΎΠ²ΠΎΠ΄Π½ΠΈΠΊΠΎΠ². ΠΠ΄Π½Π°ΠΊΠΎ Π·Π°ΠΊΠΎΠ½ Π€Π°ΡΠ°Π΄Π΅Ρ ΠΈΠΌΠ΅Π΅Ρ ΡΠΎ Π΄ΠΎΡΡΠΎΠΈΠ½ΡΡΠ²ΠΎ, ΡΡΠΎ ΠΎΠ½ ΠΎΡ Π²Π°ΡΡΠ²Π°Π΅Ρ Π΄Π²Π° ΡΠΈΠΏΠ° ΡΠ»Π΅ΠΊΡΡΠΎΠΌΠ°Π³Π½ΠΈΡΠ½ΠΎΠΉ ΠΈΠ½Π΄ΡΠΊΡΠΈΠΈ: ΡΡΠΎΡ ΠΈ ΡΠΈΠΏ, ΠΎΠ±ΡΡΠ»ΠΎΠ²Π»Π΅Π½Π½ΡΠΉ ΠΈΠ·ΠΌΠ΅Π½Π΅Π½ΠΈΠ΅ΠΌ ΠΏΠΎΡΠΎΠΊΠ° ΡΠ΅ΡΠ΅Π· ΡΡΠ°ΡΠΈΠΎΠ½Π°ΡΠ½ΡΠΉ ΠΊΠΎΠ½ΡΡΡ, ΠΊΠΎΡΠΎΡΡΠΉ Π·Π°Π²ΠΈΡΠΈΡ ΠΎΡ ΡΠ»Π΅ΠΊΡΡΠΈΡΠ΅ΡΠΊΠ°Ρ ΠΏΠΎΠ»Π΅ ΡΠ°ΡΡΡ ΡΠΈΠ»Ρ ΠΠΎΡΠ΅Π½ΡΠ°.
$\endgroup$
$\begingroup$
Π― Π΄ΠΎΠΊΠ°Π·ΡΠ²Π°Ρ, ΡΡΠΎ ΠΌΠ΅ΡΠΎΠ΄ ΠΏΠΎΠ΄ΠΌΠ΅ΡΠ°Π½ΠΈΡ ΠΏΠ»ΠΎΡΠ°Π΄ΠΈ Π΄Π°Π΅Ρ ΡΠΎΡ ΠΆΠ΅ ΡΠ΅Π·ΡΠ»ΡΡΠ°Ρ, ΡΡΠΎ ΠΈ ΠΌΠ΅ΡΠΎΠ΄ ΡΠΈΠ»Ρ ΠΠΎΡΠ΅Π½ΡΠ°. ΠΠ°ΠΊ ΠΌΡ ΡΠ²ΠΈΠ΄ΠΈΠΌ, ΠΏΠ°ΡΠ°Π»Π»Π΅Π»ΡΠ½ΠΎΠ΅ ΠΈΡΠΏΠΎΠ»ΡΠ·ΠΎΠ²Π°Π½ΠΈΠ΅ Π±Π°ΡΠ°ΡΠ΅ΠΈ Π½Π° Π΄Π²ΡΡ ΡΡΠ΅ΡΠΆΠ½ΡΡ Π½Π΅ ΠΌΠ΅Π½ΡΠ΅Ρ ΠΈΠ΄Π΅ΠΈ.
ΠΠ°Π³Π½ΠΈΡΠ½ΡΠΉ ΠΏΠΎΡΠΎΠΊ $\phi=\int_A \mathbf{B}.d\mathbf{A}$
ΠΠ°ΠΊΠΎΠ½ ΡΠ»Π΅ΠΊΡΡΠΎΠΌΠ°Π³Π½ΠΈΡΠ½ΠΎΠΉ ΠΈΠ½Π΄ΡΠΊΡΠΈΠΈ Π€Π°ΡΠ°Π΄Π΅Ρ ΠΏΡΠ΅ΠΎΠ±ΡΠ°Π·ΡΠ΅ΡΡΡ ΡΠ»Π΅Π΄ΡΡΡΠΈΠΌ ΠΎΠ±ΡΠ°Π·ΠΎΠΌ: \Π½Π°ΡΠ°ΡΡ{Π²ΡΡΠ°Π²Π½ΠΈΠ²Π°ΡΡ*} \text{ΠΠΠ‘}\varepsilon&=-\frac{d\phi}{dt}\\ \varepsilon&=-\frac{d}{dt}\left(\int_A \mathbf{B}.d\mathbf{A}\right)\\ \varepsilon&=-\mathbf{B}.\frac{d}{dt}\left(\int_A d\mathbf{A}\right)&(\ΠΏΠΎΡΠΎΠΌΡ ΡΡΠΎ \mathbf{B}\text{ΡΠ°Π²Π½ΠΎΠΌΠ΅ΡΠ½ΠΎ})\\ \varepsilon&=-\mathbf{B}.\frac{d\mathbf{A}}{dt}&(\ΠΏΠΎΡΠΎΠΌΡ ΡΡΠΎ \mathbf{A}\text{ ΡΠ²Π»ΡΠ΅ΡΡΡ ΠΎΠ΄Π½ΠΎΠ½Π°ΠΏΡΠ°Π²Π»Π΅Π½Π½ΡΠΌ})\tag{1}\\ \varepsilon&=-\mathbf{B}.\frac{d(\mathbf{l}\times\mathbf{L})}{dt}\\ \varepsilon&=-\mathbf{B}.\left(\frac{d\mathbf{l}}{dt}\times\mathbf{L}\right)&(\ΠΏΠΎΡΠΎΠΌΡ ΡΡΠΎ \mathbf{L}\text{ ΠΊΠΎΠ½ΡΡΠ°Π½ΡΠ° })\\ \varepsilon&=-\mathbf{B}. \left(\mathbf{v}\times\mathbf{L}\right)&(\ΠΏΠΎΡΠΎΠΌΡ ΡΡΠΎ \mathbf{v}dt=d\mathbf{l})\\ \varepsilon&=-\mathbf{L}.\left(\mathbf{B}\times\mathbf{v}\right)&(\ΠΏΠΎΡΠΎΠΌΡ ΡΡΠΎ \mathbf{B}.(\mathbf{C}\times\mathbf{A })=\mathbf{A}.(\mathbf{B}\times\mathbf{C}))\\ \varepsilon&=\left(\mathbf{v}\times\mathbf{B}\right).\mathbf{L}&(\ΠΏΠΎΡΠΎΠΌΡ ΡΡΠΎ \mathbf{A}\times\mathbf{B}=-\mathbf{B} \times\mathbf{A})\ΡΠ΅Π³{2} \ΠΊΠΎΠ½Π΅Ρ{Π²ΡΡΠ°Π²Π½ΠΈΠ²Π°Π½ΠΈΠ΅*} ΠΠ° ΡΠΈΡΡΠ½ΠΊΠ΅ $\mathbf{F}_{\text{ΠΠΎΡΠ΅Π½Ρ}}=q(\mathbf{E}+\mathbf{v}\times\mathbf{B})=q(\mathbf{v}\times \mathbf{B})$
Π Π°Π±ΠΎΡΠ° ΠΏΠΎ Π΄Π²ΠΈΠΆΡΡΠ΅ΠΌΡΡΡ ΡΡΠ΅ΡΠΆΠ½Ρ $=q(\mathbf{v}\times\mathbf{B}).\mathbf{L}\Π‘ΡΡΠ΅Π»ΠΊΠ° Π²ΠΏΡΠ°Π²ΠΎ \varepsilon = (\mathbf{v}\times\mathbf{B}). \mathbf{L}\tag{3}$
Π’Π°ΠΊΠΈΠΌ ΠΎΠ±ΡΠ°Π·ΠΎΠΌ, ΠΌΠ΅ΡΠΎΠ΄ ΠΏΠΎΠ΄ΠΌΠ΅ΡΠ°Π½ΠΈΡ ΠΎΠ±Π»Π°ΡΡΠΈ $(1)$ Π΄Π°Π΅Ρ $(2)$ Π΄Π»Ρ ΠΏΠΎΠΊΠ°Π·Π°Π½Π½ΠΎΠΉ ΠΊΠΎΠ½ΡΠΈΠ³ΡΡΠ°ΡΠΈΠΈ. Π’ΡΡΠΊ ΡΠ°ΠΊΠΆΠ΅ ΡΠ°Π±ΠΎΡΠ°Π΅Ρ Π΄Π»Ρ ΠΎΠ΄Π½ΠΎΠ³ΠΎ ΡΡΠ΅ΡΠΆΠ½Ρ Π±Π΅Π· ΡΠ΅ΠΏΠΈ, Π΄Π°ΠΆΠ΅ Π΅ΡΠ»ΠΈ Π½Π΅Ρ ΡΠ΅Π°Π»ΡΠ½ΠΎΠ³ΠΎ ΠΈΠ·ΠΌΠ΅Π½Π΅Π½ΠΈΡ ΠΏΠ»ΠΎΡΠ°Π΄ΠΈ, ΠΊΠΎΡΠΎΡΠΎΠ΅ ΠΈΠ·ΠΌΠ΅Π½ΡΠ΅Ρ ΠΏΠΎΡΠΎΠΊ, Π² ΡΠ²ΠΎΡ ΠΎΡΠ΅ΡΠ΅Π΄Ρ, Π²ΡΠ·ΡΠ²Π°Ρ ΠΠΠ‘. Π ΠΏΠΎΡΠ»Π΅Π΄Π½Π΅ΠΌ ΡΠ»ΡΡΠ°Π΅ $\mathbf{F}_{\text{ΠΠΎΡΠ΅Π½Ρ}}$ Π½Π°Ρ ΠΎΠ΄ΠΈΡΡΡ Π²Π΄ΠΎΠ»Ρ ΡΡΠ΅ΡΠΆΠ½Ρ. ΠΠ΅ΡΠΌΠΎΡΡΡ Π½Π° ΡΡΠΎ, ΠΏΠΎΠ»ΡΡΠ΅Π½Π½Π°Ρ ΠΠΠ‘ ΠΎΠ΄ΠΈΠ½Π°ΠΊΠΎΠ²Π° ΠΈΠ·-Π·Π° ΡΠΊΠ°Π»ΡΡΠ½ΠΎΠ³ΠΎ ΠΏΡΠΎΠΈΠ·Π²Π΅Π΄Π΅Π½ΠΈΡ $\mathbf{F}_{\text{Lorentz}}\equiv q(\mathbf{v}\times\mathbf{B})$ Ρ $\ mathbf{L}$ Π² $(3)$.